Ro-Aro Solved Papers (English)

Download as pdf or txt
Download as pdf or txt
You are on page 1of 384

CLICK HERE FOR FREE MATERIAL

Youth Compitition Times

Review Officer
Assistant Review Officer

Solved Paper
General Studies
&

General Hindi
Chief Editor
Mr. Anand Kumar Mahajan
Computer Graphics by
Balkrishna, Charan Singh, Ashish Giri
Editorial Office
Youth Competition Times
12, Church Lane Prayagraj-211002
Mob. : 9415650134
Email : [email protected]
website : www.yctbooks.com

Publisher Declaration
Edited and Published by A.K. Mahajan for YCT Publications Pvt. Ltd.
and printed by Om Sai Offset, Prayagraj. In order to publish the book,
full care has been taken by the editor and the publisher,
still your suggestions and queries are welcomed. Rs. : 595/-
In the event of any dispute, the Judicial area will be Prayagraj.
CLICK HERE FOR FREE MATERIAL

Index
Uttar Pradesh Public Service Commission (UDA/LDA)/Review
Officer/Assistant Review Officer Exam
Uttar Pradesh Review Officer/Assistant Review Officer (Mains) Exam 2021 (Exam Date : 24/04/2022)
General Studies .................................................................................................................................................................................. 3-17
General Hindi ................................................................................................................................................................................... 18-20
Uttar Pradesh Review Officer/Assistant Review Officer (Pre) Exam 2021 (Exam Date : 05/12/2021)
General Studies ................................................................................................................................................................................ 21-36
General Hindi ................................................................................................................................................................................... 37-42
Uttar Pradesh Review Officer/Assistant Review Officer (Mains) Exam 2016 (Exam Date : 22/12/2020)
General Studies ................................................................................................................................................................................ 43-58
General Hindi ................................................................................................................................................................................... 59-61
Uttar Pradesh Review Officer/Assistant Review Officer (Pre) (Re-Exam) 2016 (Exam Date : 20/09/2020)
General Studies ................................................................................................................................................................................ 62-77
General Hindi ................................................................................................................................................................................... 78-82
Uttar Pradesh Review Officer/Assistant Review Officer (Mains) Exam 2017
General Studies ................................................................................................................................................................................ 83-95
General Hindi ................................................................................................................................................................................... 96-98
Uttar Pradesh Review Officer/Assistant Review Officer (Pre) Exam 2017
General Studies .............................................................................................................................................................................. 99-113
General Hindi ............................................................................................................................................................................... 114-118
Uttar Pradesh Review Officer/Assistant Review Officer (Pre) (Cancelled) Exam 2016
General Studies ............................................................................................................................................................................ 119-135
General Hindi ............................................................................................................................................................................... 136-140
Uttar Pradesh Review Officer/Assistant Review Officer (Mains) Exam 2014
General Studies ............................................................................................................................................................................ 141-151
General Hindi ............................................................................................................................................................................... 152-154
Uttar Pradesh Review Officer/Assistant Review Officer (Pre) Exam 2014
General Studies ............................................................................................................................................................................ 155-166
General Hindi ............................................................................................................................................................................... 167-170
Uttar Pradesh Review Officer/Assistant Review Officer (Mains) Exam 2013
General Studies ............................................................................................................................................................................ 171-182
General Hindi ............................................................................................................................................................................... 183-184
Uttar Pradesh Review Officer/Assistant Review Officer (Pre) Exam 2013
General Studies ............................................................................................................................................................................ 185-196
General Hindi ............................................................................................................................................................................... 197-200
Uttar Pradesh Review Officer/Assistant Review Officer (Mains) Exam 2010
General Studies ............................................................................................................................................................................ 201-213
General Hindi ............................................................................................................................................................................... 214-216
Uttar Pradesh Review Officer/Assistant Review Officer (Pre) Exam 2010
General Studies ............................................................................................................................................................................ 217-233
General Studies ............................................................................................................................................................................ 234-237
Uttar Pradesh Review Officer/Assistant Review Officer Spl. (Mains) Exam 2010
General Studies ............................................................................................................................................................................ 238-251
General Hindi ............................................................................................................................................................................... 252-253
Uttar Pradesh Review Officer/Assistant Review Officer Spl. (Pre) Exam 2010
General Studies ............................................................................................................................................................................ 254-267
General Hindi ............................................................................................................................................................................... 268-281
UPPSC UDA/LDA (Pre) Exam. 2006 ............................................................................................................................................ 272-280
UPPSC UDA/LDA (Pre) Exam. 2001 ............................................................................................................................................ 281-299

Uttarakhand Public Service Commission (UDA/LDA)/ Review


Officer/Assistant Review Officer Exam.
Uttarakhand UDA/LDA (Pre) General Studies Exam 2003.................................................................................................... 300-313
Uttarakhand UDA/LDA (Pre) General Studies Exam 2006 .................................................................................................... 314-324
Uttarakhand UDA/LDA (Mains) General Studies Exam 2006 .............................................................................................. 325-332
Uttarakhand Review Officer/Assistant Review Officer (Pre) Exam 2016 ........................................................................... 333-346
Uttarakhand Review Officer/Assistant Review Officer (Mains) Exam 2016 ...................................................................... 347-354
Uttarakhand Review Officer/Assistant Review Officer (Pre) Exam 2021 ........................................................................... 355-366
Uttarakhand Review Officer/Assistant Review Officer (Pre) Advocate General Exam 2022 ........................................ 367-384
2
CLICK HERE FOR FREE MATERIAL

UPPSC RO-ARO (Mains) Exam-2021


GENERAL STUDIES
Solved Paper [ Exam Date : 24-04-2022

1. During his annexation of Afghanistan, who Code-


invited Babur to invade India? A B C D
(a) Ustad Ali (b) Mustafa (a) 1 2 3 4
(c) Daulat Khan (d) Hasan Khan (b) 1 2 4 3
Ans. (c) : During the annexation of Afghanistan, Daulat (c) 2 1 4 3
Khan Lodhi invited Babur to invade India, Daulat khan (d) 2 1 3 4
was Afghan governor of Punjab during the rule of the
Lodi dynasty in the Delhi Sultanate. When Ibrahim Ans. (c) : Correct match is as follows-
Lodhi tired to recapture Punjab, Daulat Khan lodhi, New South Wales – Sydney.
alarmed at his safety, sent his son Dilawar khan to Queensland – Brisbane.
Kabul to invite Babur to help overthrow Ibrahim Lodhi. Victoria – Melbourne.
2. Who is the author of the book "Death in Western Australia – Perth.
Banaras"? Hence, option C is correct.
(a) Meena Kaushik (b) Jonthan Parry 5. Match List-I with List-II and select the correct
(c) B.D. Tripathi (d) Ron Barrett answer using the code given below the lists-
Ans. (b) : Jonathan Parry is the author of the book List-I List-II
"Death in Banaras". His other books are "The Politics of (Organisation) (Establishment
Patriotism" and "Caste and Kinship in Kangra". year)
3. Which one of the following pairs is not A Lalit Kala Akademy 1 1954
correctly matched? B Sangeet Natak 2 1952
Bleaching – Calcium Academy
(a)
powder oxychloride C National School of 3 1959
Plaster of Paris – Calcium Drama
(b) bicarbonate D Centre for Cultural 4 1979
(c) Washing Powder – Sodium carbonate Resources and
Baking Soda – Sodium Training
(d) Code -
bicarbonate
Ans. (b) : Correctly Matched pairs:- A B C D
Bleaching powder – Calcium oxychloride (a) 1 2 3 4
[Ca (ClO)2] (b) 2 4 1 3
Plaster of Paris – Calcium sulphate hemihydrates (c) 4 3 2 1
[CaSO4. 1/2 H2O] (d) 3 2 4 1
Washing Powder – Sodium carbonate Ans. (a) :
[Na2 CO3] Organisation Establishment year
Baking Soda – Sodium bicarbonate a. Lalit Kala Akademy – 1954
[Na HCO3] b. Sangeet Natak Academy – 1952
4. Match List-I with List-II and select the correct c. National School of Drama – 1959
answer using the code given below the lists- d. Centre for Cultural Resources – 1979
List-I List-II and Training
(Provinces of (Capital city) • Note :– Sangeet Natak Akadami was at up in 1953 for
Australia) the preservation and promotion of performing arts.
A New South Wales 1 Brisbane 6. Which of the following Tiger Reserves is not
B Queensland 2 Sydney situated in Madhya Pradesh?
C Victoria 3 Perth (a) Amangarh (b) Bandhavgarh
D Western Australia 4 Melbourne (c) Kanha (d) Sanjay Dubri
UP RO-ARO (Mains) Exam-2021 (Ex. Dt. 24.04.2022) 3 YCT
CLICK HERE FOR FREE MATERIAL

Ans. (a) : Amangarh Tiger Reserve is in Bijnor district 10. Which one of the following groups of diet
in state of Uttar Pradesh. Amangarh Tiger reserve was provides maximum energy to human body?
originally part of Jim Corbett National Park, and after (a) Fat, Protein, Vitamin
the state of Uttarakhand was carved out of Uttar (b) Vitamin, Mineral, Water
Pradesh, Amangarh remained in Uttar Pradesh, (c) Carbohydrate, Fat, Protein
Bandhavgarh, Kanha and Sanjay Dubri Tiger Reserve (d) Carbohydrate, Protein, Vitamin
are situated in Madhya Pradesh.
Ans. (c) : Groups of diet which provides maximum
7. Which one of the following coal based power energy to human body is carbohydrates, fat, protein.
plant is situated in Uttar Pradesh? These are main types of macronutrients in food.
1. Darlipali 2. Dadri Macronutrients are nutrients that are required daily in
3. Singrauli 4. Vindhyachal large quantities. They differ in amount of energy and
Select the correct answer using the code given how quickly they supply energy. Carbohydrates are the
below quickest and Fats are the slowest in terms of energy
Code - supply
(a) 1 and 3 (b) 2 and 3 11. Who among the following Indian leaders
(c) 2 and 4 (d) 3 and 4 participated in all the three round table
Ans. (b) : Dadri and Singrauli are coal based power conferences?
plants situated in Uttar Pradesh. There are 5 coal based (a) Mahatama Gandhi (b) B.S. Munje
power plants Situated in Uttar Pradesh namely (c) Dr. B.R. Ambedakar (d) C.Y. Chintamani
Singrauli, Dadri, Rihand, Firozgandhi Unchahar, Tanda. Ans. (c) : Dr. B.R. Ambedakar was the Indian leader
8. Which of the following is not correctly who participated in all the three round table
matched? conferences. The first conference took place in 1930,
(Animal) (Breed) second conference in 1931 and Third conference in
(a) Cow – Deoni 1932. Gandhi ji joined only in second round table
conference. Tej Bahadur sapru also attended all 3 round
(b) Goat – Barbari
table conferences.
(c) Buffalo – Bhadawari
12. Which one of the following countries is not
(d) Sheep – Gir
situated along the coast of Baltic Sea?
Ans. (d) : (a) Latvia and Poland
Animal Breed (b) Latvia and Russia
a. Cow – Deoni (c) Sweden and Russia
b. Goat – Barbari (d) Sweden and Norway
c. Buffalo – Bhadawari Ans. (d) : There are nine countries with a shoreline at
d. Cow – Gir/Gyr. the Baltic sea i.e. Denmark, Germany, Poland, Russia,
9. Match List-I with List-II and select the correct Lithuania, Latvia, Estonia, Finland and Sweden.
answer using the code given below the lists- Norway is NOT situated along the Coast of Baltic sea.
List-I List-II 13. Which of the following state has won maximum
(Books) (Breed) number of medals in India skills 2021
A Ras Panchadhyayi 1 Vrindavan competition, the result of which was declared
Das by Government of India in January 2022?
B Chaitanya Bhagvata 2 Jayananda (a) Gujarat (b) Maharashtra
C Chaitanya Mangal 3 Nand Das (c) Kerala (d) Odisha
D Bhakti Ratnakara 4 Narahari Ans. (d) : Odisha won maximum number of medals
Cakravarti (51) in India skills 2021 competition, the result of which
Code- was declared by Government of India in January 2022.
Competition was held at New Delhi.
A B C D
(a) 3 4 2 1 14. With reference to the 'Trishul Peak in
(b) 2 3 4 1 Himalayas', which of the following statements
is/are correct?
(c) 3 1 2 4
(1) It is situated in Bageshwar district of
(d) 4 3 1 2
Uttarakhand.
Ans. (c) : (2) The peak is more than 7500 m high.
Books Author Select the correct answer from the code given
a. Ras Panchadhyayi – Nand Das below
b. Chaitanya Bhagvata – Vrindavan Das Code -
c. Chaitanya Mangal – Jayananda (a) Only 1 (b) Only 2
d. Bhakti Ratnakara – Narahari (c) Both 1 and 2 (d) Neither 1 nor 2
UP RO-ARO (Mains) Exam-2021 (Ex. Dt. 24.04.2022) 4 YCT
CLICK HERE FOR FREE MATERIAL

Ans. (a) : Trishul peak in Himalayas is situated in 18. Which one of the following is a bio-diesel
Bageshwar district of Uttarakhand. Trishul is a group of plant?
three Himalayan mountain peaks of western kumaon (a) Jatropha (b) Java Grass
with highest reaching 7120m. So, statement (2) is (c) Rosha Grass (d) Guggul
wrong. The Trishul group forms the southwest corner of Ans. (a) : Biodiesel is an alternative clean- burning
the ring of peaks enclosing the Nandadevi Sanctuary. renewable fuel similar to conventional diesel. It is
15. Which one of the following is not a sub- produced by using animal fats, vegetable oils and waste
category of terrestrial ecosystem? cooking oil. First generation biofuels are made from
(a) Mountain ecosystem food sources, second generation biofuels are produced
(b) Marine ecosystem from non-food crops or not edible food crops and third
(c) Lowland ecosystem generation biofuels are produced from micro-organisms
(d) Semi-arid ecosystem like algae, Fourth generation biofuels are made from
Ans. (b) : Marine ecosystem is not a sub-category of crops that are genetically engineered. Jatropha,
Terrestrial ecosystem whereas mountain ecosystem, Pongamia etc, are example of bio-fuel plants.
Lowland ecosystem and semi-arid ecosystem are sub- 19. Mandsaur - Prashasti presents the details of
category of Terrestrial ecosystem. Terrestrial ecosystem the achievements of which ruler?
is a land based community of organisms in a given area. (a) Skandagupta (b) Chandragupta-II
Examples are Taiga, Tundra, Forests, temperate (c) Ramagupta (d) Kamaragupta-I
deciduous, Tropical Rainforests, Grasslands and
Deserts. Ans. (d) : Mandsaur - Prashasti presents the details of
the achievements of ruler Kumaragupta-I. It has also
16. With reference to India's Third National Water given historical details about Yashodharman. Mandsaur
Awards declared in January 2022, which of the
inscription of Kumaragupta was written by Vatsabhatta,
following statements is/are correct?
The mandsaur inscription refers to Kumara Gupta
(1) Uttar Pradesh was declared winner in the 'Best
'reigning over the whole earth'.
State Category'.
(2) Muzaffarnagar in Uttar Pradesh was declared 20. With reference to 'Parvati Arga' Ramsar site,
as the best district in 'North Zone Category'. which of the following statements is/are
Select the correct answer using the code given correct?
below (1) It is situated in Gonda district of Uttar
Code- Pradesh
(2) It is a bird sanctuary.
(a) Only 1 (b) Only 2
Select the correct answer using the code given
(c) Both 1 and 2 (d) Neither 1 nor 2
below
Ans. (c) : With reference to India's Third National
Code -
Water Awards declared in January 2022, Uttar Pradesh
was declared winner in the "best state category" (a) Only 1 (b) Only 2
Muzaffarnagar in the Uttar Pradesh was declared as the (c) Both 1 and 2 (d) Neither 1 nor 2
best district in 'North zone category'. National Water Ans. (c) : 'Parvati Arga' Ramsar site is situated in
Awards is given for exemplary work in the field of Gonda district of Uttar Pradesh, it is a bird sanctuary.
water resource management, First award was launched The sanctuary is a refuge for some of India's threatened
in 2018, by Jal shakti ministry. vulture species. India has presently 75 Ramsar sites,
17. Part IX - A of the Constitution of India gave a Tamil Nadu has maximum number of Ramsar sites
constitutional foundation to the Local Self followed by Uttar Pradesh.
Government units in the following areas- 21. The first Krishi Vigyan Kendra was
1. Rural areas established in 1974 at which of the following
2. Urban areas place?
Select the correct answer from the code given (a) Chandigarh (b) Nagpur
below the lists (c) Hyderabad (d) Pondicherry
Code- Ans. (d) : The first Krishi Vigyan Kendra was
(a) Only 1 (b) Only 2 established in 1974 at Pondicherry. It was established
(c) Both 1 and 2 (d) Neither 1 nor 2 under the administrative control of the Tamil Nadu
Ans. (b) : Part IX-A of the constitution of India gave a Agricultural University, Coimbatore,
Constitutional foundation to the local self Government 22. Uttar Pradesh has secured which place in
units in urban areas. This part was added by the 74th Swachh Survekshan, 2021?
Amendment Act 1992 comprising Articles 243P to (a) Fourth (b) Third
243ZG. (c) Sixth (d) None of the above
UP RO-ARO (Mains) Exam-2021 (Ex. Dt. 24.04.2022) 5 YCT
CLICK HERE FOR FREE MATERIAL

Ans. (c) : Uttar Pradesh has secured sixth place in Ans. (b) : 'Krishak Samridhi Aayog' was established in
Swachh survekshan, 2021. Uttar Pradesh, among the 13 Uttar Pradesh in the year 2017. It was established for
states in the country with more than 100 urban local the benefit of the state's farmers. The goal of this
bodies, cities were ranked based on three broad commission was to quadruple farmer's income by the
parameters - service level progress, citizens voice and year 2022. The chief minister of the state acts as the
certification. It is the 6th Edition of Swachh Survekshan. Chairman of this Aayog.
For the year 2022, U.P. has been ranked 10th. 27. With reference to Sher Shah Suri, which of the
following statements is/are correct?
23. Which one of the following river basin is the
1. He was the architect of a brilliant
largest in area?
administrative system.
(a) Kaveri (b) Mahanadi 2. However his qualities as a ruler were not more
(c) Narmada (d) Tapti remarkable than his victories on the field of
Ans. (b) : Mahanadi river basin is the largest in area. Its battle.
area is 1,41,589 sq. km. Narmada river basin area is 98, Select the correct answer from the code given
796 sq.km and Tapti river basin area is 65,145 sq. km. below
24. Match List-I with List-II and select the correct Code -
answer from the code given below the lists- (a) Only 1 (b) Only 2
List-I List-II (c) Both 1 and 2 (d) Neither 1 nor 2
(Name of Scientist) (Area of work) Ans. (a) : Sher Shah Suri was a brilliant administrator.
A Birbal Sahni 1 Green He introduced many reforms that became a solid base of
Revolution Mughal Administration. He was one of the greatest
administrator of medieval India. His qualities as a ruler is
B K.C. Mehta 2 Embryology more remarkable than his victories on the field of battle.
C P. Maheshwari 3 Paleobotany 28. At which one of the following places was
D M.S. Swaminathan 4 Plant India's first 'Open Rock Museum' inugurated
pathology in January 2022?
Code- (a) Dehradun (b) Nanital
A B C D (c) Hyderabad (d) Mount Abu
(a) 1 2 3 4 Ans. (c) : India's first 'Open Rock museum was'
(b) 3 4 2 1 inaugurated in January 2022 in Hyderabad. It displays
(c) 2 1 3 4 around 35 different types of rocks from different parts
of India with ages raging from 3.3 billion years to
(d) 4 3 2 1 around 55 million years to the Earth's history.
Ans. (b) : 29. Who built the mosque "Qila-E Kuhna" inside
Name of scientist Area of work Purana Qila of Delhi?
a. Birbal Sahni Paleobotany (a) Humayun (b) Sher Shah
b. K.C. Mehta Plant pathology (c) Akbar (d) Shah Jahan
c. P. Maheshwari Embryology Ans. (b) : Sher Shah built the mosque "Qila-E Kuhna"
d. M.S. Swaminathan Green Revolution inside Purana Qila of Delhi. It was built in 1541. This
25. 'Uttar Pradesh Diversified Agriculture Support along with Sher mandal, are the only two surviving
Project' (UP DASP) was sponsored in Uttar structures of Delhi's Purana Qila, Which was
Pradesh by- conceptualized and started by Shershah.
(a) Government of Uttar Pradesh 30. As per the 'India State of Forest Report 2021',
(b) Central Government how many states/union territories have over
33% of their geographical area under forest?
(c) World Bank
(a) 07 (b) 13
(d) None of the above (c) 17 (d) 23
Ans. (c) : Uttar Pradesh Diversified Agriculture support Ans. (c) : As per the 'India State of Forest Report 2021',
Project (UP DASP) was sponsored in Uttar Pradesh by 17 states union territories have over 33% of their
World Bank. The principle objectives of UP DASP are geographical area under forest. It is the 17th biennial
to increase Agricultural productivity, to promote private assessment of India's forest. The Total Forest and tree
sector indevelopment, and to improve rural cover is 24.62% of the geographical area of the country.
infrastructure. 31. Who among the following athletes was declared
26. 'Krishak Samridhi Aayog' was established in 'Athlete of the year 2021' by the Time
Uttar Pradesh in which of the following year? Magazine in December 2021?
(a) 2016 (b) 2017 (a) Caeleb Dressel (b) Lewis Hamilton
(c) 2018 (d) None of the above (c) Neeraj Chopra (d) Simone Biles
UP RO-ARO (Mains) Exam-2021 (Ex. Dt. 24.04.2022) 6 YCT
CLICK HERE FOR FREE MATERIAL

Ans. (d) : Simone Biles was declared 'Athlete of the Ans. (c) : Indore is the city which was not among the
year 2021' by the Time Magazine in December 2021. top three cities in the sustainable Development Goal
Simone Biles is world's number one gymnast. She was Urban Index and Dashboard 2021-22 declared by the
hailed for putting her mental health first when she NITI Aayog in November 2021. Top performers are
withdrew from four event finals at the Tokyo Olympics. Shimla, Coimbatore, Chandigarh, Thiruvanthanapuram
and Kochi, while the worst performer is Dhanbad.
32. Consider the following events and arrange
them into chronological order- 36. The National Development Council was formed
in the year-
(1) Establishment of NABARD
(a) 1950 (b) 1952
(2) Bank linkage programme of self help group
(c) 1954 (d) None of the above
(3) Kisan Credit Card yojana
Ans. (b) : The National Development Council was
(4) Establishment of Regional Rural Bank formed in the year 1952. It is an apex body for decision
Code - making and deliberations on development matters in
(a) (4), (1), (2) and (3) (b) (4), (2), (3) and (1) India presided over by the Prime Minister. It was setup
(c) (1), (2), (3) and (4) (d) (4), (3), (2) and (1) to strengthen and mobilize the effort and resources of
the nation in support of the plan and to ensure the
Ans. (a) : Correct chronological order is-
balanced and rapid development of all parts of the
Establishment of Regional Rural Bank (RRB) - 1975 country.
Establishment of NABARD- 1982. 37. The headquarter of Small Industries
Bank linkage programme of self help group (SHP) - Development Bank of India (SIDBI) is located
1992-93 at which one of the following place?
Ksan Credit card yojana- 1998 (a) Kanpur (b) Lucknow
33. With reference to the India's National Youth (c) Ghaziabad (d) None of the above
Festival organised in January 2022, which of Ans. (b) : The headquarter of small industries
the following statements is/are correct? development Bank of India (SIDBI) is located in
(1) This was 20th edition of the National Youth Lucknow. Set up on 2nd April, 199, it acts as the
Festival. principle financial institution for promotion financing
(2) It was organised at Pondicherry. and development of MSMEs.
Select the correct answer using the code given 38. 'Ringelmann Scale' is used to measure the
below density of the following-
Code - (a) fog (b) noise
(c) polluted water (d) smoke
(a) Only 1 (b) Only 2
(c) Both 1 and 2 (d) Neither 1 nor 2 Ans. (d) : 'Ringelmann scale' is used to measure the
density of smoke, It was developed by Maximilien
Ans. (b) : With reference to the India's National youth Ringelmann of Paris in 1888. The scale has 5 levels of
festival organised in January 2022, It was organised at density inferred from a grid of black lines on a white
Pondicherry. It was organised by Ministry of Youth surface.
Affairs and Sports. This was 25th edition of National 39. With reference to a book, the following
youth festival. It was started on birth anniversary of information are given-
Swami Vivekananda on 12 January. (1) Introduction
34. Government of India has passed 'Protection of (2) Contents
Plant Varieties and Farmers Right Act' in (3) Chapters
which of the following year? (4) Index
(a) 2001 (b) 2005 (5) Title
(c) 2015 (d) None of the above Select the correct logical arrangement from the
Ans. (a) : Government of India has passed 'Protection code given below
of plant varieties and Farmers Right Act' in 2001. It Code-
provide for the establishment of an effective system for (a) 1, 3, 5, 2 and 4 (b) 2, 4, 1, 5 and 3
the protection of plant varieties, the rights of framers (c) 5, 2, 1, 3 and 4 (d) 3, 5, 1, 4 and 2
and plant breeders and to encourage the development of Ans. (c) : The words are given found typically found
new varieties of plants. inside a book.
35. Which one of the following city was not among They must be arranged in the order in which they
the top three positions in the Sustainable appear. So the order of these word. 5, 2, 1, 3 and 4
Development Goal Urban Index and 40. Which one of the following Scheduled Tribes is
Dashboard 2021-22 declared by the NITI not found in Varanasi district of Uttar
Aayog in November 2021? Pradesh?
(a) Coimbatore (b) Chandigarh (a) Chero (b) Gond
(c) Indore (d) Shimla (c) Kharwar (d) Sahariya
UP RO-ARO (Mains) Exam-2021 (Ex. Dt. 24.04.2022) 7 YCT
CLICK HERE FOR FREE MATERIAL

Ans. (d) : Sahariya Scheduled Tribe is not found in Ans. (a) : Ambika Chakravarti was not the initial
Varanasi district of Uttar Pradesh. Chero, Gond and member of the 'Naujawan sabha' formed in 1926. It was
Kharwar is found in Varanasi. Sahariya tribes in Uttar founded by Bhagat Singh in March 1926. The
Pradesh are found in Lalitpur district. association sought to encourage revolution against the
41. River Ganga enters in Uttar Pradesh from British Raj by gathering together worker and peasant
which district? youth. Its leaders were- Bhagat Singh, Yashpal',
(a) Bijnor (b) Meerut Chhabil Das, Chandrashekhar Azad etc.
(c) Saharanpur (d) J.P. Nagar 46. With reference to 'EASE-Index', which of the
Ans. (a) : River Ganga enters in Uttar Pradesh from following statements is/are correct?
Bijnor district. It flows through 27 districts of Uttar (1) It is a way to measure the performance of the
Pradesh, covering 1140 km. River Ganga Passes Public Sector Bank.
through major districts like Prayagraj, Varanasi, (2) It measures the ease of doing business for
Kanpur, Aligarh, Mirzapur etc. MSMEs in India.
42. The Provision of a joint sitting of both the Select the correct answer using the code given
Houses of Parliament is mentioned in the below
Constitution under the following Article-
Code -
(a) 100 (b) 108
(c) 110 (d) None of the above (a) Only 1 (b) Only 2
Ans. (b) : The Provision of a joint sitting of both the (c) Both 1 and 2 (d) Neither 1 nor 2
Houses of Parliament is mentioned in the constitution Ans. (a) : 'EASE-Index' is a way to measure the
under Article 108. The joint sitting of the parliament is performance of the Public Sector Bank. Enhanced
called by the President and is presided over by the Access and service Excellence 4.0 (EASE 4.0) was
speaker. Till now joint sitting of Indian parliament has launched recently by union Finance minister to
been called for 3 bills- undertook the annual performance review of the public
1. Dowry Prohibition Bill, 1961 sector Banks (PSBs).
2. Banking service Commission (Repeal) Bill, 1987. 47. Which vitamin is soluble in water?
3. Prevention of Terrorism Bill, 2002. (a) Vitamin 'A' (b) Vitamin 'B'
43. Who among the following was not a great (c) Vitamin 'C' (d) Vitamin 'D'
entity in the field of Hindustani music?
Ans. (b & c) : Vitamin 'B' and Vitamin 'C' are soluble
(a) Raja Mansingh Tomar
in water. Vitamin which dissolve in water are called
(b) Tansen water- soluble vitamins and which dissolve in liquid fat
(c) Sadarang - Adarang are called fat-soluble vitamins. Water-soluble vitamins
(d) Mal Gurjari are generally not stored in the body.
Ans. (d) : Mal Gurjari was not a great entity in the field 48. Which one among the following is measured by
for Hindustani music. Mal Gurjari is a dhrupad piece
Dobson unit?
developed by Raja Man Singh Tomar. Rest of the given
names are of famous Hindustani musicians. (a) Ozone concentration
(b) Thermal conductivity
44. The Provision of three legislative lists is found
in the following Schedule of the Constitution- (c) Soil moisture
(a) VI (b) VII (d) Radiation
(c) VIII (d) None of the above Ans. (a) : Ozone concentration is measured by Dobson
Ans. (b) : The Provision of three legislative lists is unit. The "Dobson unit" indicates now much of a given
found in VII- Schedule of the Constitution. There are 12 trace gas is in the air above a certain point on earth.
schedules of Indian Constitution. 7th schedule deals with 49. Who among the following was the President of
division of power between the Union government and the 'All India Trade Union Congress' in 1929?
state governments. Three lists given in seventh schedule (a) M.N. Roy
are – (b) Jayaprakash Narayan
1. Union list – List I – 97 subjects (c) Jawaharlal Nehru
2. State list – List II – 66 subjects (d) S.A. Dange
3. Concurrent list – List III – 57 subjects Ans. (c) : Jawaharlal Nehru was the President of the
45. Who was not the initial member of the "All India Trade Union congress" in 1929. AITUC is
'Naujawan Sabha' formed in 1926? the oldest trade union in India. It was formed in 1920 in
(a) Ambika Chakravarti Bombay with its first President Lala lajpat Rai, Since its
(b) Yashpal inception, AITUC was the primary trade union
(c) Bhagat Singh organisation in India working under the Indian National
(d) Chhabil Das congress.
UP RO-ARO (Mains) Exam-2021 (Ex. Dt. 24.04.2022) 8 YCT
CLICK HERE FOR FREE MATERIAL

50. Lake Balkhash is situated in which one of the Code -


following countries? A B C D
(a) Kazakhstan (b) Kyrgyzstan (a) 3 1 2 4
(c) Russia (d) Turkmenistan (b) 2 4 1 3
Ans. (a) : Lake Balkhash is situated in Kazakhstan. It is (c) 4 3 1 2
one of the largest lakes in Asia. It lies is the Balqash - (d) 3 2 4 1
Alakol basin, the basin drains seven rivers. It is
Ans. (c) : Place Related Tirthankar
shrinking due to diversion and detraction of water from
its feeders. The lake's eastern part is saline and western a. Shravasti – Sambhavnath
part is fresh water. b. Kakandi – Suvidhanath
c. Ayodhya – Rishabhanath
51. Arrange the following Mesolithic sites
geographically in order from west to east- d. Pabhosa – Padmaprabhu
1. Paisra 54. Which one of the following city was awarded a
2. Lekhahia star rating in the 'Garbage Free Cities'
3. Birbhanpur category in Uttar Pradesh in Swachh
Survekshan 2021?
4. Mahadaha
(a) Aligarh (b) Gorakhpur
Select the correct answer from the code given
(c) Prayagraj (d) Varansi
below
Code - Ans. (a) : Aligarh was awarded a star rating in the
'Garbage Free Cities' category in Uttar Pradesh in
(a) 4, 2, 3 and 1 (b) 1, 4, 3 and 2 swachh Survekshan 2021. Nodal Ministry is Ministry of
(c) 4, 2, 1 and 3 (d) 2, 4, 1 and 3 Housing and urban Affairs (MoHUA). Other cities of
Ans. (c) : Mesolithic sites geographically in order from Uttar Pradesh which got awarded are Noida, Ghaziabad,
west to east is Mahadaha, Lekhahia, Paisra and Jhansi and Lucknow.
Birbhanpur. Mahadaha is in Pratapgarh (Uttar Pradesh), 55. Find the missing number-
Lekhahia in Mirzapur (Uttar pradesh), Paisra in Bihar
and Birbhanpur in West Bengal.
52. Match List-I with List-II and select the correct
answer from the code given below the lists-
List-I List-II (a) 16 (b) 20
A Vitamin 1 Pepsin (c) 28 (d) 32
B Enzyme 2 Carotene 5×5 + 5
Ans. (c) : =6
C Hormone 3 Keratin 5
D Protein 4 Progesterone and
Code- 3 × 3 + 21
= 10
A B C D 3
(a) 1 2 3 4 Similarly
(b) 1 2 4 3 4× 4 + ?
(c) 2 1 3 4 = 11
4
(d) 2 1 4 3
16+? = 44
Ans. (d) : List-I List-II ? = 28
a. Vitamin – Carotene Hence, 'c' is correct option.
b. Enzyme – Pepsin
56. In 1809, Raja Ram Mohan Roy wrote his
c. Hormone – Progesterone famous book "Gift to Monotheist" in which of
d. Protein – Keratin. the following language?
53. Match List-I with List-II and select the correct (a) English (b) Urdu
answer using the code given below the lists- (c) Persian (d) Bengali
List-I (Place) List-II Ans. (c) : In 1809, Raja Ram Mohan Roy wrote his
(Related famous book "Gift to Monotheists" in Persian language.
Tirthankar) The original title of the book is Tuhafat-al-Muwahidin
A Shravasti 1 Rishbhanath in which he strongly supported believe in one God.
B Kakandi 2 Padmaprabhu 57. Which one of the following is a plant hormone?
C Ayodhya 3 Suvidhanath (a) Insulin (b) Cytokinin
D Pabhosa 4 Sambhavnath (c) Thyroxin (d) Oestrogen
UP RO-ARO (Mains) Exam-2021 (Ex. Dt. 24.04.2022) 9 YCT
CLICK HERE FOR FREE MATERIAL

Ans. (b) : Cytokinin is a plant hormone. Plant 61. Ahmed Naser Al-Raisi, who was elected
hormones are chemicals produced by plants that President of the Interpol in November 2021, is
regulate their growth, development, reproductive from which of the following country?
processes, longevity and death. Six major plant (a) Kuwait (b) Saudi Arabia
hormones are auxins, abscisic acid, cytokinins, (c) Turkey (d) United Arab Emirates
ethylene, gibberellins and brassinosteroids. Insulin, Ans. (d) : Ahmad Naser Al-Raisi, who currently serves
Thyroxin and Oestrogen are human hormones. as the President of Interpol belongs to UAE (United
58. Plants accepts nitrogen in which form? Arab Emirate). His tenure will be of 4 years. Interpol
(International Germinal Police Organization) is an inter-
(a) Nitrogen gas (b) Nitrite
governmental organization of 195 member countries.
(c) Nitrate (d) Ammonia Head quarter of Interpol is at Lyon (France).
Ans. (c) : Plants accepts nitrogen in nitrate form. 62. Which of the following crop is covered for price
Nitrification is the biological oxidation of ammonia to control under the price stabilization fund of the
nitrite which is followed by oxidation of nitrite to Government of India?
nitrate. Nitrates are used by the plants for making amino (a) Wheat (b) Potato
acids, which are needed to make proteins. Plant absorb (c) Rice (d) None of the above
nitrates from the soil through their root hairs. Ans. (b) : The Price stabilization Fund (PSF) was set up
59. Match List-I with List-II and select the correct in 2014-15 under the Department of Agriculture,
answer from the code given below the lists- cooperation & Farmers welfare (DAC & FW) to help
List-I List-II (Diwan) regulate the price volatility of important Agri-
(Emperor) Horticultural commodities like onion, potatoes. The
PSF scheme was transferred from DAC&FW to the
A Akbar 1 Ghiyas Beg Department of Consumer Affairs (DOCA) w.e.f 1st
Itmad-ud-Daulah April 2016. PSF scheme provides for the advancement
B Jahangir 2 Asad Khan of interest- free loans to state Governments/Union
C Shah Jahan 3 Muzaffar Khan Territories (UTs) and central Agencies to finance their
D Aurangzeb 4 Sadullah Khan working capital and other expenses.
Code- 63. With reference to Commerce, which of the
following statements is/are correct?
A B C D (1) It is the field of business that assists in the
(a) 1 4 3 2 removal of all impediments to the
(b) 2 3 1 4 facilitation of exchange.
(c) 3 2 1 4 (2) It deals with the factors of delivering the
(d) 3 1 4 2 products to the consumers.
Ans. (d) : Select the correct answer using the code given
below
Emperor Diwan
Code -
a. Akbar Muzaffar khan
b. Jahangir Ghiyas Beg Itmad-ud-Daulah. (a) Only 1 (b) Only 2
(c) Both 1 and 2 (d) Neither 1 nor 2
c. Shah Jahan Sadullah khan.
d. Aurangzeb Asad khan. Ans. (c) : Commerce is the exchange of goods and
services between businesses. Commerce is the trade of
60. Which one of the following is not correctly goods, services or other things of value between
matched? companies or organizations. It is the field of business
(Institute) (Location) that assists in the removal of all impediments to the
(a) National Centre of – Ghaziabad facilitation of exchange. It deals with the factors of
Organic Farming delivering the products to the consumers.
(b) Indian Institute of – Kanpur 64. United Nations General Assembly has declared
Pulses Research the year 2023 as the International year of-
(a) Oilseed crops (b) Pulse crops
(c) Indian Institute of – Gorakhpur
Vegetable Research (c) Millet crops (d) Animal fodder crops
Ans. (c) : United Nations General Assembly has
(d) Indian Institute of – Lucknow
declared the year 2023 as the International Year of
Sugarcane Research Millet. The International Year of Millets stands to
Ans. (c) : provide a unique opportunity to increase global
• National center of organic Farming- Ghaziabad production, ensure efficient processing and
• Indian Institute of pulses Research-Kanpur consumption, promote a better utilization of crop
rotations, and encourage better connectivity throughout
• Indian Institute of Vegetable Research-Varanasi food systems to promote millets as a key component of
• Indian Institute of sugarcane Research-Lucknow the food basket.
UP RO-ARO (Mains) Exam-2021 (Ex. Dt. 24.04.2022) 10 YCT
CLICK HERE FOR FREE MATERIAL

65. Which one of the following tree is not found in 70. Consider the following events and arrange
the Mediterranean forest? them into chronological order-
(a) Teak (b) Oak 1. Project Elephant
(c) Chestnut (d) Olive 2. Biological Diversity Act
Ans. (a) : Teak is found in Tropical forests. 3. Wildlife (Protection) Act
Mediterranean forests are generally composed of 4. Project Tiger
broadleaf trees such as oak, Chestnut, olive, pine etc Select the correct answer using the code given
Mediterranean forests are found in central Chile, below
Australia, Africa, southwest USA and regions around Code -
Mediterranean sea. (a) 3, 4, 1 and 2 (b) 3, 2, 4 and 1
66. Which one of the following mangrove areas is (c) 4, 3, 2 and 1 (d) 4, 3, 1 and 2
situated in the delta region of Godavari river? Ans. (a) : Wild life protection Act- 1972
(a) Coondapoor (b) Honnavar Project Tiger-1973
(c) Malpe (d) Coringa Project elephant -1992
Ans. (d) : Coringa mangrove forest is located in the Biological Diversity Act - 2002
Godavari delta, Andhra Pradesh, India. The following 71. Which of the following is not a part of the
three dominant mangrove plants, Avicennia marina, Middle East?
Excoecaria agallocha and Sonneratia apetala were found (a) Jordan and Israel
to be present on the banks of a major channel of the (b) Lebanon and Israel
Godavari river running through the forest. (c) Syria and Lebanon
67. Which one of the following Ramsar Wetland (d) Jordan and Turkey
sites is situated in any state of India? Ans. (*) : The Middle east is a geographical and
(a) Hokera Wetland cultural region located primarily in Western Asia, but
(b) Point Calimere Wildlife and Bird Sanctuary also in parts of Northern Africa and southeastern
Europe
(c) Tso Moriri Lake
Countries:- Bahrain, Cyprus, Egypt, Iran, Iraq, Israel,
(d) Wular Lake Jordan, Kuwait, Lebanon, Oman, Qatar, Saudi Arabia,
Ans. (b) : Established in the year of 1967, Point Syria, Turkey, UAE.
Calimere wildlife and sanctuary (PCWBS) is situated 72. Gwadar Port in Pakistan is situated in which of
in protected area in Tamil Nadu. It was created for the following provinces?
conservation of the least concern black buck antelope, (a) Balochistan
an endemic mammal species of India. (b) Khyber Pakhtunkhwa
PCWBC forms the easternmost and most biologically (c) Punjab
diverse part of Ramsar site (2002) in India. It extends (d) Sindh
upto 385 sq. km and comprises Panchanadikulam
Ans. (a) : Gwadar is a port city on the south western
wetland, Unsurveyed salt swamp and Muthupet
Arabian sea coastline of Pakistan in Balochistan
Mangroves.
province. Gwadar's Economy has, in the past been
• Note :- Other Ramsar sites given in the question are dependant mostly on fishing but now its economy is
situated in Union Territories of Jammu & Kashmir and changing as Gwadar is slowly transforming into a major
Ladakh of India. port city.
68. In the event of vacancy in the office of both 73. In 1817, the famous Hindu College was
President and Vice President, who shall established by David Hare in-
discharge the functions of President? (a) Agra (b) Calcutta
(a) Prime Minister (c) Patna (d) Varanasi
(b) Lok Sabha Speaker
Ans. (b) : Hindu college was established on 2th January
(c) Chief Justice of India 1817 in Calcutta. Founder were Raja Ram Mohan Roy,
(d) None of the above Radhakanta Deb, Rasamay Dutt, David Hare and
Ans. (c) : In the event of vacancy in the office of both, Baidyanath Mukhopadhya.
President and Vice President, the function of the 74. How many times the 'date 29' will fall between
president is discharged by the Chief Justice of India. January 1, 2022 and January 1, 2027?
69. The first day of the year 2020 was Wednesday. (a) 55 (b) 65
What was the last day of the year 2021? (c) 56 (d) None of the above
(a) Thursday (b) Friday Ans. (c) : February will be of 29 days in a leap year.
(c) Saturday (d) Sunday So the date 29 between January 1, 2022 to January 1,
Ans. (a) : 2027 will be = 11+11+12+11+11 = 56 days
UP RO-ARO (Mains) Exam-2021 (Ex. Dt. 24.04.2022) 11 YCT
CLICK HERE FOR FREE MATERIAL

75. Rocket works on which of the following Select the correct answer using the code given
principles? below
(a) Avogadro's concept Code-
(b) Energy conservation (a) 1, 2, 3 and 4 (b) 2, 1, 4 and 3
(c) Bernoulli's theorem (c) 2, 4, 3 and 1 (d) 3, 1, 4 and 2
(d) Momentum conservation Ans. (d) : Mughal Invasion of Malwa- 1560
Ans. (d) : Rocket works on principle of momentum
Akbar Marries Princes of Amber-1562
conservation i.e. principle of conservation of linear
Karrani's conquest of Orrissa - 1567-68
momentum. When the rocket is fired, the exhaust gases
Battle of Tukaroi- 1575
rush downward with tremendous speed, which gives the
79. The Aspirational District Programme (ADP) of
opposite momentum to the rocket which pushed the
rocket upward. NITI Aayog was launched in-
(a) January 2016 (b) January 2017
76. Match List-I with List-II and select the correct
answer from the code given below the lists- (c) January 2018 (d) None of the above
List-I Ans. (c) : The Aspirpirational District Programme
List-II
(ADP) of NITI Aayog was launched in January 2018 by
(Monument) (Builder)
Prime minister of India, Narendra Modi. The
A Atala Devi 1 Nusrat Shah
Programme is anchored by NITI Aayog in partnership
Masjid with state governments and district level administration.
B Chhota Sona 2 Ibrahim Shah
The programme aims to quickly and effectively
Masjid Sharkitransform 112 most under- developed districts across
C Qadam Rasul 3 the country.
Sikandar Shah
D Adina Masjid, 4 Key focus Areas- health and Nutrition, education,
Wali Muhammad
Pandua Agriculture & water Resources, Financial Inclusion &
Code - Skill Development and Basic infrastructure.
A B C D 80. Bhutan conferred the 'Order of the Druk
(a) 1 2 3 4 Ghyalpo', its highest civilian honour to the
Prime Minister of which one of the following
(b) 2 4 1 3
country in December 2021?
(c) 2 1 3 4 (a) India (b) Mongolia
(d) 1 3 4 2 (c) Nepal (d) Thailand
Ans. (b) : Monument Builder Ans. (a) : Prime Minsiter Narendra Modi has been
Atala Devi Masjid - Ibrahim Shah Sharqi
conferred with the highest civilian award of Bhutan,
Chhota Sona Masjid - Wali Muhammad
"Order of the Druk gyalpo also known as Ngadag pel gi
Qadam Rasul - Nusrat ShahKhorla, on the occasion of the country's National Day-
Adina Masjid, Pandua - This was Modi's 10th international award given by
Sikandar Shah
Various governments.
77. Find out the alternative which will replace the
question mark? 81. As per Article 300 of the Constitution of India,
WATCH : 110 :: Time : ? the legal name of the Government of India is-
(a) 54 (b) 72 (a) The Union Government
(c) 94 (d) 98 (b) The Union of India
Ans. (c) : (c) India
(d) Bharat
Ans. (b) : As per Article 300 of the constitution of
India, the legal name of the Government of India is-
The Union of India. The Government of India may sue
Similarly or be sued by the name of the Union of India.
82. Which of the following Article is related to the
appointment of a Finance Commission by the
President of India?
(a) 200 (b) 250
Hence 'c' is correct. (c) 280 (d) None of the above
78. Consider the following events and arrange Ans. (c) : Article 280 provides for the Finance
them into chronological order- commission of India. The President of India constitute
1. Akbar marries Princess of Amber Finance commission at the expiration of every fifth year
2. Battle of Tukaroi or at such other time as he considers necessary.
3. Mughal Invasion of Orissa N.K. Singh is currently serving as the 15th finance
4. Karrani's conquest of Orissa commission of India.
UP RO-ARO (Mains) Exam-2021 (Ex. Dt. 24.04.2022) 12 YCT
CLICK HERE FOR FREE MATERIAL

83. The Parliament of India has the 'Power to Ans. (a) : By option (a) -
Legislate' for the following-
1. Union of India
2. Territory of India
3. Indian subject resident and their property
situated anywhere in the world
Select the correct answer from the code given Hence, minimum number of boys = 10
below 87. With reference to trade, which of the following
Code- statements is/are correct?
(1) It represents demand and supply both.
(a) Only 1 (b) 1 and 2 only (2) It deals with only the demand side.
(c) 2 and 3 only (d) 1, 2 and 3 Select the correct answer using the code given
Ans. (b) : The parliament of India has the" power to below
legislate and make laws for union of India, Territory of Code-
India and Indian subject and their property situated (a) Only 1 (b) Only 2
within the national territory of India. (c) Both 1 and 2 (d) Neither 1 nor 2
84. Which one of following city was not among the Ans. (a) : Trade in simple terms is basically an
top three cleanest cities in more than one lakh exchange, voluntary in nature between two parties in
population category in Swachh Survekshan requirement of each others resources i.e. goods and
2021? services. This system is based purely on the concept of
(a) Indore (b) Surat demand and supply both.
(c) Tirupati (d) Vijayawada 88. Antibiotics are mostly obtained from-
Ans. (c) : Swachh Survekshan, conducted by MoHUA (a) Fungi (b) Vinises
since 2016, is the world's largest urban sanitation and (c) Bacteria (d) Augiosperms
cleanliness survey. It has been instrumental in fostering Ans. (c) : Antibiotics are mostly obtained from bacteria
a spirit of healthy competition among towns and cities while they are also obtained from fungi. Antibiotic is a
to improve their service delivery to citizens. Swachh medicine which is used for treatment of bacterial
survekshan- 2021 was, the 6th edition. infections
Top three clearnest cities in more than one lakh 89. Which of the following is not correctly
population category in Swachh Survkshan 2021 are – matched?
(a) Maize - Tasseling
• Indore – First
(b) Groundnut - Pegging
• Surat – Second (c) Chick pea - Tillering
• Vijayawada – Third (d) Tobacco - Topping
85. With reference to 'Lake Victoria in Africa'. Ans. (c) : Maize – Tasseling
which of the following statements is/are Groundnut – Pegging
correct? Tobacco – Topping
(1) The lake is surrounded by four countries. chick pea is not associated with Tillering.
(2) It is situated in the African rift valley. Crops associated with Tillering are- Rice, Wheat,
Select the correct answer using the code given Sugarcane, Maize, etc.
below 90. Which one of the following country was not
Code- part of exercise 'Sea Dragon 2022' that started
in the Pacific Ocean in January 2022?
(a) Only 1 (b) Only 2
(a) India (b) Japan
(c) Both 1 and 2 (d) Neither 1 nor 2 (c) Philippines (d) South Korea
Ans. (b) : Kenya, Tanzania and Uganda share lake Ans. (c) : Sea Dragon-2022 exercise that started in the
Victoria. It is world's largest tropical lake and the Pacific Ocean in January 2022 sees the participation of
second largest freshwater lake. It is situated in the QUAD Countries (India, Australia, Japan, USA), along
African rift Valley. White Nile (also known as Victoria with Canada and South Korea.
Nile) flows out of this lake.
Sea Dragon is an annual, multi-national high end
86. In a queue of boys, Sohan is 9th from the back exercise hosted by the U.S Navy.
Ramesh's place is 8th from the front. Rohit is 91. Who among the following was not nationalist
standing in between the two. What could be the poet in Pre-independent India?
minimum number of boys standing in the (a) Ranglal
queue? (b) Nabin Chandra Sen
(a) 10 (b) 12 (c) Hem Chandra Banerjee
(c) 14 (d) 15 (d) Shiv Kumar Batalvi
UP RO-ARO (Mains) Exam-2021 (Ex. Dt. 24.04.2022) 13 YCT
CLICK HERE FOR FREE MATERIAL

Ans. (d) : Shiv Kumar Batalvi was an Indian poet the greatest threats faced by democracies today through
known for romantic poetry. He was a writer and a collective action- India, Nepal, Maldives- Countries
playwrighter of the Punjabi language. He was also from south Asia attended this summit.
known as "Birah Ka Sultan". He was not a nationalist 96. If MET, SET and TET are coded as 52, 76 and
poet in Pre-independence India. 80 respectively, then what will be the code for
92. With reference to 'Alaska' which of the PET?
following statements is/are correct? (a) 48 (b) 56
(1) It is oil producing state of United States of (c) 64 (d) 72
America. Ans. (c) :
(2) It was purchased from Russians in 1867.
Select the correct answer using the code given
below
Code -
(a) Only 1 (b) Only 2
(c) Both 1 and 2 (d) Neither 1 nor 2
Ans. (c) : Alaska was admitted in United States of
America as 49th constituent State. It's capital is Juneau.
It is largest U.S state by area. Oil and gas industry
dominates the Alaskan economy and it is oil producing Similarly
state of United States of America.
It was purchased from Russia in 1867 (30th March
1867).
Hence (c) is correct.
93. Who among the following was appointed as the
tenth Chairman of the Indian Space Research 97. Which among the following was the first
Organisation (ISRO) in January 2022? successfully cloned animal?
(a) P.L.N. Raju (b) V. Hari Prasad (a) Dog (b) Rabbit
(c) C. Jagannathan (d) S. Somanath (c) Gibbon (d) Sheep
Ans. (d) : S. Somanath was appointed as the 10th Ans. (d) : On July 5, 1996, Dolly the sheep- the first
chairman of the ISRO (Indian Space Research mammal to have been successfully cloned from an adult
Organisation) in January 2022. He has replaced K. cell- is born of the Roslin Institute in Scotland. Dolly
Sivan. was part of a larger project to create transgenic
(genetically modified) sheep that produced therapeutic
94. 'Stand-up-India' scheme for promoting
proteins in their milk.
entrepreneurship is related to-
(a) Minorities (b) OBC 98. What was the disinvestment target for Public
(c) Handicapped (d) Women, SC and ST Sector Banks and Financial Institutions in the
Union Budget 2020-21?
Ans. (d) : Stand-up India scheme (2016) is for
(a) ` 80,000 Crores (b) ` 90,000 Crores
financing SC/ST and/or women entrepreneurs. The
objective of the stand up India scheme is to facilitate (c) ` 100,000 Crores (d) None of the above
bank loans between 10 lakh and 1 Crore to at least one Ans. (c) : Disinvestment target for public sector Banks
scheduled caste (SC) or scheduled Tribe (ST) borrower and Financial Institutions in Union Budget 2020-21 was
and at least on women borrower per bank branch for ` 100,000 crore.
setting up a Greenfield enterprise. Disinvestment refers to an act of a state to raise funds
Eligibility- SC/ST or women entrepreneur above 18 by selling ownership stake.
years of age. 99. Central Institute of Medicinal and Aromatic
95. Which of the following countries from South Plants is situated at which of the following
Asia attended the Summit for Democracy place?
hosted by the President of the United States, of (a) Lucknow (b) Varanasi
America in December 2021? (c) Gorakhpur (d) None of the above
(a) India, Nepal and Maldives Ans. (a) : Central Institute of Medicinal and Aromatic
(b) India, Pakistan and Nepal Plants is situated at Lucknow, Uttar Pradesh. It is a
(c) India, Sri Lanka and Maldives frontier plant research laboratory of Council of
(d) India, Bhutan and Nepal Scientific and Industrial Research (CSIR). It was
Ans. (a) : On December 9-10 2021, President Biden established in 1959.
held the first of two summits for Democracy, which 100. With reference to 'Indonesia', which of the
brought together leaders from government, civil society, following statements is/are correct?
and the private sector in the shared effort to set forth an (1) It comprises of over 15,000 islands.
affirmative agenda for democratic renewal and to tackle (2) It has over 150 active volcanoes.
UP RO-ARO (Mains) Exam-2021 (Ex. Dt. 24.04.2022) 14 YCT
CLICK HERE FOR FREE MATERIAL

Select the correct answer using the code given Select the correct answer using the code given
below below
Code - Code-
(a) Only 1 (b) Only 2 (a) Only 1 (b) Only 2
(c) Both 1 and 2 (d) Neither 1 nor 2 (c) Both 1 and 2 (d) Neither 1 nor 2
Ans. (a) : Indonesia, the island country is located in Ans. (c) : Highest number of national parks are in
Maritime Southeast Asia, between the Indian ocean, Madhya Pradesh (9).
Timor sea, Arafura sea and South China sea. Country Highest number of wild life sanctuaries are in Andaman
consists of 5 major islands and about 30 smaller Island and Nicobar Island (96) followed by Maharashtra.
groups. There are a total of 17508 islands.
105. Which of the following two states are the
As of 2021, there were 76 Active volcanoes in
leading producer of bauxite in India, as per the
Indonesia. Hence 2nd statement is incorrect.
Annual Report of the Ministry of Mines, 2020-
101. As per the fourth edition of State Health Index 21?
released by the NITI Aayog in December 2021, 1. Odisha
which one of the following states performed
2. Andhra Pradesh
best among the 'Smaller States' category?
3. Chhattigarh
(a) Goa (b) Manipur
(c) Mizoram (d) Sikkim 4. Gujarat
Select the correct answer using the code given
Ans. (c) : NITI Aayog with assistance from the World
below
Bank and with consultation with ministry of Health and
Family welfare (MoHFW) had released 4th edition of Code-
state health Index in December 2021. (a) 1 and 2 (b) 1 and 3
Ranking of states (c) 1 and 4 (d) 2 and 4
Larger states Ans. (c) : As per the Annual Report of the Ministry of
Top three states were UP, Assam, Telangana. Mines, 2020-21 the two states which are leading
• Among smaller states Mizoram and Meghalaya have producer of Bauxite are- Odisha and Gujarat.
performed best. 106. To reduce the interest payment on the
• Union Territories Electricity sector, which scheme has been
Delhi followed by J&K. launched by the Government of India?
102. Turmeric is an example of underground stem, (a) UJALA Scheme (b) Urja Ganga Scheme
known as- (c) UDAY Scheme (d) Saubhagya Scheme
(a) Bulb (b) Tuber Ans. (c) : UDAY (Ujwal DISCOM Assurance Yojana)
(c) Corm (d) Rhizome was launched in November 2015 as centrally sponsored
Ans. (d) : Turmeric is an example of underground stem, scheme under the Ministry of Power with objective of
known as Rhizome, which is divided into two parts:- "financial turnaround of power distribution companies.
The scheme envisages:-
• Financial turnaround
. • Operational improvement
103. What is cholesterol among the following? • Reduction of cost of generation of power
(a) Steroid (b) Vitamin • Development of Renewable energy
(c) Enzyme (d) Insecticide • Energy efficiency and conservation.
107. In the following table, the missing number 'x'
Ans. (a) : Cholesterol is a waxy, fat like substance that's
found in all the cells of body. It is also found in foods is-
from animal sources. It is of two types:- 3 4 6
(1) Low density lipoprotein (LDL) 8 x 14
(2) High density lipoprotein (HDL)
18 20 24
Cholesterol belongs to the steroid family of lipid (fat)
compounds. (a) 9 (b) 10
(c) 11 (d) 12
104. With reference to 'National Parks and Wildlife
Sanctuaries in India', which of the following Ans. (b) :
statements is/are correct?
(1) Highest number of national parks are in
Madhya Pradesh.
(2) Highest number of wildlife sanctuaries are
s
in the Andaman and Nicobar island. Hence, ? = 10
UP RO-ARO (Mains) Exam-2021 (Ex. Dt. 24.04.2022) 15 YCT
CLICK HERE FOR FREE MATERIAL

108. Which one of the following gases in used in Select the correct answer using the code given
manufacturing of polythene? below-
(a) Ethylene (b) Carbon-di-oxide Code-
(c) Nitrogen (d) Carbon monoxide
(a) Only 1 (b) Only 2
Ans. (a) : Ethylene (C2H4) is used in manufacturing of
(c) Both 1 and 2 (d) Neither 1 nor 2
polythene. Polythene is made by several methods by
addition polymerization of Ethylene, which is Ans. (a) : Galapagos Islands is a volcanic archipelago
principally produced by the cracking of ethane and situated in Pacific ocean. They are part of the Republic
propane, naphtha and gas oil. of Ecuador. Most of the island is covered in semi desert
109. According to the Constitution of India, the vegetation, including scrublands, grasslands and dry
'Executive Power of the Union' shall be vested forest. Galapagos Island is known for their distinctive
in the endemic species such as giant tortoises, finches,
1. President galapagos lava lizards etc.
2. Prime Minister 112. India is the largest producer of which of the
Select the correct answer from the code given following fruits in the world as per the Annual
below the lists- Report of the Department of Agriculture Co-
Code- operation and Farmers Welfare, Government
of India, 2020-21?
(a) Only 1 (b) Only 2
1. Grapes
(c) Both 1 and 2 (d) Neither 1 not 2
2. mango
Ans. (a) : Article 53 (1) of the Indian Constitution
3. Orange
states that the executive power of the union shall be
vested in the President and shall be exercised by him 4. Papaya
either directly or through officers subordinate to him in Select the correct answer using the code given
accordance with this constitution. below-
Part V of the Indian Constitution deals with "The Code-
executive of the Union" from Article 52 to 151. (a) 1 and 2 (b) 1 and 3
110. Match List-I with List-II and select the correct (c) 2 and 3 (d) 2 and 4
answer from the code given below the lists-
Ans. (d) : According to Annual report of the
List-I List-II Department of Agriculture Co-operation and Farmers
(Minimum (Office/Post) Welfare, India is largest producer of Banana, mango
Qualification and papaya in the world. India produces over 40% of
of Age)
mangoes in the world. India and Brazil accounts for
A 21 years 1 President of India 50% production of papaya in the world.
B 25 years 2 Member of Panchayat 113. With reference to the statue of 'Nataraj the
C 30 years 3 Member of State Lord of Dance', which of the following
Legislatvie Assembly statements is/are correct?
D 35 years 4 Member of State 1. The image represents dancing Shiva
Legislative Council having four hands.
Code- 2. In his right ear, he wears a man's earring,
A B C D a women's in the left.
(a) 2 3 4 1 Select the correct answer using the code given
(b) 1 2 4 3 below:
(c) 2 1 3 4 Code-
(d) 4 1 2 3 (a) Only 1 (b) Only 2
Ans. (a) : Minimum Qualification Office/Post (c) Both 1 and 2 (d) Neither 1 nor 2
21 years Member of Panchayat Ans. (c) : Statue of "Nataraj the Lord of Dance" is of
25 years Member of state legislative Assembly Chola period. His dance is a manifestation of his
30 years Member of state legislative council panchkritya as or the 5 activities of Shiva that are
35 years President of India. creation, preservation, destruction, veiling and grace. The
111. With reference to the 'Galapagos Islands' dancing shiva image represented by 4 hands. In his right
situated in the Pacific Ocean, which of the ear, he wears a man's earring, a women's in the left.
following statements is/are correct? 114. If 44 × 75 = 7454, 83 × 74 = 7348 and 55 × 48 =
(1) They are formed due to volcanic activity. 4585, then 14 × 45 will be equal to-
(2) They are part of South American nation (a) 4145 (b) 4415
Peru. (c) 4451 (d) 4154
UP RO-ARO (Mains) Exam-2021 (Ex. Dt. 24.04.2022) 16 YCT
CLICK HERE FOR FREE MATERIAL

Ans. (c) : Just as, 118. With reference to the India International
44 × 75 = 7 4 5 4 Trade Fair-2021, which of the following
statements is/are correct?
ab cd c b d a
(1) It was organised in November 2021 at
83 × 74 = 7 3 4 8
Jaipur, Rajasthan,
ab cd c b d a
(2) Main theme of the event was 'Atmanirbhar
55 × 48 = 4 5 8 5
Bharat'.
ab cd c b d a
Select the correct answer from the code given
Similarly, below-
14 × 45 = 4 4 5 1
Code-
ab cd c b d a
(a) Only 1
115. As 'Part' is related to 'Whole', in the same way
(b) Only 2
'Arc' is related to, which of the following?
(c) Both 1 and 2
(a) Area
(d) Neither 1 nor 2
(b) Chord
(c) Circumference Ans. (b) : 2021 India International Trade Fair was held
(d) Segment between 14 to 27 November 2021 at Pragati Maidan,
New Delhi. Main theme of the fair was 'Atmanirbhar
Ans. (c) : Just as a 'part' is a small unit of the 'whole' Bharat'.
similarly an arc is a small unit of. "Circumference".
Focus states were UP and Uttarakhand and partner state
Hence option 'c' is correct. was Bihar.
116. Uttar Pradesh Council of Agricultural 119. When was India's first National Action Plan on
Research is located at which of the following climate change released?
place?
(a) 2012 (b) 2015
(a) Lucknow
(c) 2008 (d) 2000
(b) Meerut
Ans. (c) : The Government of India launched first
(c) Gorakhpur
National Action Plan on Climate Change (NAPCC) on
(d) None of the above
30th June 2008 outlining 8 National Missions. These
Ans. (a) : Uttar Pradesh Council of Agricultural include:-
Research is located at Lucknow, Uttar Pradesh. UPCAR (1) National Solar Mission
was established on June 14, 1989. It is an autonomous (2) National Mission on Enhanced Energy Efficiency
apex state organisation registered as a society, which
(3) National Mission on Sustainable Habitat
plans, co-ordinates and promotes research, education,
(4) National Water Mission
training and transfer of technology for advancement of
agriculture and allied sciences research, teaching, (5) National Mission for Sustaining Himalayan Eco-
training and technology transfer. system.
(6) National Mission for Green India
117. With reference to Rhine river in Europe, which
of the following statements is/are correct? (7) National Mission for Sustainable Agriculture.
(1) Rhine river is an important pathway for (8) National Mission on Strategic Knowledge for
industrial activity in Northern Germany. Climate Change.
(2) Port of Rotterdam is located at the mouth 120. Which one of the following is not correctly
of Rhine river. matched?
Select the correct answer using the code given (Place) (Districts of Uttar
below- Pradesh)
(a) Bateshwar - Agra
Code-
(a) Only 1 (b) Only 2 (b) Deva Sharif - Jaunpur
(c) Both 1 and 2 (d) Neither 1 nor 2 (c) Sankisa - Farrukhabad
(d) Bithoor - Kanpur
Ans. (c) : Rhine is one of the major European river. Its
basin lies in Switzerland, Liechtenstein, Austria, Ans. (b) :
Germany, France and Netherlands. It is second longest (Place) (Districts of Uttar Pradesh)
river in central and western Europe. Rhine river is an Bateshwar Agra
important pathway for industrial activity in western Deva Sharif Barabanki
Germany. Port of Rotterdam is located at the mouth of Sankisa Farrukhabad
Rhine river. Bithoor Kanpur
UP RO-ARO (Mains) Exam-2021 (Ex. Dt. 24.04.2022) 17 YCT
CLICK HERE FOR FREE MATERIAL

Gòej ØeosMe meceer#ee DeefOekeâejer/meneÙekeâ meceer#ee DeefOekeâejer (cegKÙe) hejer#ee, 2021


meeceevÙe efnvoer
JÙeeKÙee meefnle nue ØeMve-he$e (hejer#ee efleefLe : 24 DeØewue, 2022)

1. efvecveefueefKele ceW keâewve mee JeekeäÙe DeMegæ nw? 5. efvecveefueefKele ceW Skeâ leLÙe ieuele nw–
(a) Fmekeâe leelkeâeefuekeâ ØeYeeJe heÌ[e~ (a) efJeMes<eCe mes meb%ee keâer JÙeeeqhle heefjefcele nes peeleer nw~
(b) he$e efkeâmekesâ veece hej efueKee ieÙee nw~ (b) efpeme efJekeâejer Meyo mes meb%ee keâer JÙeeeqhle ceÙee&efole nesleer nw,
(c) Skeâ ®heÙes ceW efkeâleves hewmes nesles nQ? Gmes efJeMes<eCe keânles nQ~
(d) Yeejle Skeâ mece=efæMeeueer osMe nw~ (c) ØeÙeesie keâer eqmLeefle kesâ DeeOeej hej ‘Debefkeâle’ Meyo efJeMes<eCe
Deewj efJeMes<Ùe oesveeW nw~
Ans. (b) : ‘he$e efkeâmekesâ veece hej efueKee ieÙee nw’ JeekeäÙe DeMegæ nw~
(d) efJeMes<eCe Meyo meowJe efJeMes<Ùe kesâ henues ØeÙegòeâ neslee nw,
Fmekeâe Megæ ™he - ‘he$e efkeâmekesâ veece efueKee ieÙee nw’ nesiee~ Mes<e
efJeMes<Ùe kesâ yeeo keâYeer ØeÙegòeâ veneR neslee~
JeekeäÙe Megæ nQ~
Ans. (d) : JeekeäÙe ceW efJeMes<eCe keâe ØeÙeesie oes Øekeâej mes neslee nw–keâYeer
2. FveceW mes efJeueesce MeyoeW keâe ieuele Ùegice nw–
efJeMes<eCe efJeMes<Ùe kesâ henues ØeÙegòeâ neslee nw Deewj keâYeer efJeMes<Ùe kesâ yeeo~
(a) DeeefJeYee&Je - efJeYeeJe (b) DehejeOeer - efvejhejeOe peye efJeMes<eCe efJeMes<Ùe mes henues ØeÙegkeäle neslee nw lees Gmes GodosMÙe-
(c) Gvcetueve - jesheCe (d) efJeefnle - efveef<eæ efJeMes<eCe Ùee efJeMes<eCe-efJeMes<Ùe keânles nQ Deewj peye efJeMes<eCe efJeMes<Ùe kesâ
Ans. (a) : efoS ieS efJeueesce Meyo ÙegiceeW ceW ‘DeeefJeYee&Je - efJeYeeJe’ yeeo ØeÙegkeäle neslee nw lees Gmes efJeOesÙe-efJeMes<eCe keânles nQ~ Dele: efJekeâuhe
Ùegice ieuele nw~ DeeefJeYee&Je keâe efJeueesce eflejesYeeJe nesiee ve efkeâ efJeYeeJe~ (d) ieuele nw~
3. lelmece Deewj Gmekesâ leodYeJe ™he keâer Âef„ mes veerÛes efueKes 6. mebKÙeeJeeÛekeâ efJeMes<eCe ‘Ûeewiegvee’ FveceW mes nw–
efJekeâuheeW ceW keâewve-mee efJekeâuhe ieuele nw? (a) ›eâceJeeÛekeâ efJeMes<eCe (b) DeeJe=efòeJeeÛekeâ efJeMes<eCe
(a) Deeõ&keâ - ieeruee (b) DeeefolÙeJeej - FleJeej (c) ieCeveeJeÛekeâ efJeMes<eCe (d) mecegoeÙeJeeÛekeâ efJeMes<eCe
(c) Godieueve - Gieuevee (d) GodIeešve - GIeeÌ[vee Ans. (b) : ‘Ûeewiegvee’ DeeJe=efòeJeeÛekeâ efJeMes<eCe nw, pees efkeâ
Ans. (a) : efoS ieS lelmece Deewj leodYeJe ÙegiceeW ceW ‘Deeõ&keâ - ieeruee’ mebKÙeeJeeÛekeâ efJeMes<eCe keâe Skeâ Øekeâej nw~
Ùegice ieuele nw~ ‘Deeõ&keâ’ keâe leodYeJe ‘Deojkeâ’ nesiee peyeefkeâ ‘ieeruee’, ›eâceJeeÛekeâ efJeMes<eCe - henuee, otmeje, leermeje~
‘Deeõ&’ keâe leodYeJe nw~ ieCeveeJeÛekeâ efJeMes<eCe - Skeâ, oes, leerve~
4. efvecveefueefKele ceW keâewve mee JeekeäÙe Megæ nw? mecegoeÙeJeeÛekeâ efJeMes<eCe - oesveeW, leerveeW, ÛeejeW~
(a) cesjs efueS "b[er yeHe&â Deewj iece& Deeie ueeDees~ 7. efvecveefueefKele MeyoeW ceW mes keâewve-mee Meyo cetue ™he mes
(b) JÙeeÙeece keâjvee ÛeeefnS efpememes efkeâ mJemLe jnW~
efJeMes<eCe nw?
(a) heleuee (b) DeLe&
(c) Skeâ-Skeâ keâjkesâ meYeer cej ieS~
(c) heefjJeej (d) hejueeskeâ
(d) legcneje Ùen keânvee cesjs efueS yeÌ[er yeele nesieer~
Ans. (a) : ‘heleuee’ Meyo iegCeJeeÛekeâ efJeMes<eCe nw~ peyeefkeâ DevÙe meYeer
Ans. (*) : efoS ieS meYeer JeekeäÙe DeMegæ nw, Fvekeâe Megæ ¤he
efJeMes<Ùe nw, efpevekeâe efJeMes<eCe efvecve nw–
efvecve nQ–
efJeMes<Ùe efJeMes<eCe
DeMegæ JeekeäÙe Megæ JeekeäÙe
DeLe& DeeefLe&keâ
cesjs efueS "b[er yeHe&â Deewj iece& cesjs efueS yeHe&â Deewj Deeie heefjJeej heeefjJeeefjkeâ
Deeie ueeDees~ ueeDees~ hejueeskeâ heejueewefkeâkeâ
JÙeeÙeece keâjvee ÛeeefnS efpememes JÙeeÙeece keâjvee ÛeeefnS leeefkeâ 8. FveceW mes efJeMes<eCe Meyo veneR nw–
efkeâ mJemLe jnW~ mJemLe jnW~ (a) Deketâle (b) DekeâCe&
Skeâ-Skeâ keâjkesâ meYeer cej ieS~ Skeâ-Skeâ keâj meYeer cej ieS~ (c) DeÛeej (d) DeÛeb[
legcneje Ùen keânvee cesjs efueS yeÌ[er legcneje Ùen keânvee cesjs efueS Ans. (c) : efoS ieS MeyoeW ceW ‘DeÛeej’ Meyo efJeMes<eCe ve neskeâj
yeele nesieer~ yeÌ[er yeele nesiee~ efJeMes<Ùe nw Fmekeâe efJeMes<eCe ‘DeÛeejer’ nesiee~
UP RO/ARO (Mains) Hindi 2021 (Ex.dt. 24.04.2022) 18 YCT
CLICK HERE FOR FREE MATERIAL

9. ‘Deveskeâ MeyoeW kesâ efueS Skeâ Meyo’ kesâ veerÛes efueKes 15. efvecveefueefKele ceW efJeueesce MeyoeW keâe mener Ùegice nw–
efJekeâuheeW ceW keâewve-mee efJekeâuhe ieuele nw? (a) Dee«en - efJe«en (b) ieCelev$e - ueeskeâlev$e
(a) pebieue keâer Deeie - oeJeeveue (c) DeekeâerCe& - efJekeâerCe& (d) Leeskeâ - hejÛetve
(b) mecegõ keâer Deeie - yeÌ[Jeeveue Ans. (c) : ‘DeekeâerCe& - efJekeâerCe&’ efJeueesce Meyo keâe mener Ùegice nw~
(c) hesš keâer Deeie - pe"jeeqive peyeefkeâ DevÙe MeyoeW keâe mener efJeueesce Ùegice Fme Øekeâej nw–
(d) Deeefo mes Devle lekeâ - Deveble Meyo efJeueesce
Ans. (d) : JeekeäÙeebMe ‘Deeefo mes Devle lekeâ’ kesâ efueS Skeâ Meyo Dee«en Devee«en
‘DeeÅeesheevle’ nesiee~ ‘Deveble’ Meyo ‘efpemekeâe Devle ve nes’ kesâ mLeeve hej ieCelev$e jepeleb$e
ØeÙegòeâ neslee nw~ Leeskeâ Hegâškeâj
10. ‘Deheveer efJeJeeefnlee Œeer mes GlheVe heg$e’ - Fve Deveskeâ Deveg«en efJe«en
MeyoeW kesâ efueS Skeâ Meyo nw– 16. efvecveefueefKele ceW mes Skeâ DeMegæ JeekeäÙe nw–
(a) pejeÙegpe (b) peejpe (a) hetpÙeemheo JÙeefòeâ keâe mecceeve keâjvee ÛeeefnS~
(c) oòekeâ (d) Deewjme (b) oef#eCe Yeejle kesâ ueesie efnvoerlej Yee<eer nQ~
Ans. (d) : JeekeäÙeebMe ‘Deheveer efJeJeeefnlee Œeer mes GlheVe heg$e’ kesâ efueS (c) megjsKee ves ieerle keâer oes-Ûeej keâef[
Ì ÙeeB ieeÙeeR~
Skeâ Meyo ‘Deewjme’ nesiee~ (d) Deye hejer#ee-ØeCeeueer yeoueveer ÛeeefnS~
pejeÙegpe - efMeMeg/peerJe pees Kes[Ì er ceW efueheše ngDee hewoe nes~ Ans. (a) : hetpÙeemheo JÙeefòeâ keâe mecceeve keâjvee ÛeeefnS ceW
peejpe - Gheheefle/iewj heg®<e mes GlheVe heg$e~ ‘hetpÙeemheo’ Meyo keâer Jele&veer DeMegæ nw Fmekeâe Megæ ™he ‘hetpeemheo’
oòekeâ - GllejeefOekeâej kesâ efueS ieeso efueÙee ieÙee heg$e~ nesiee~ peyeefkeâ Mes<e JeekeäÙe Megæ nQ~
17. ‘GhelÙekeâe’ keâe efJeueesce Meyo nw–
11. ‘DeeJeeB’ keâe lelmece ™he nesiee–
(a) DeveglÙekeâe (b) DeefOelÙekeâe
(a) Deeie (b) Deeqive
(c) GÛelÙekeâe (d) DehelÙekeâe
(c) DeeJe (d) Deeheekeâ
Ans. (b) : ‘GhelÙekeâe’ keâe efJeueesce ‘DeefOelÙekeâe’ nesiee~
Ans. (d) : ‘DeeJeeB’ keâe lelmece ™he ‘Deeheekeâ’ nesiee~ efoS ieÙes MeyoeW
ceW ‘Deefive’ Deeie keâe lelmece ¤he nw~ 18. efvecveefueefKele ceW Megæ Jele&veer Jeeuee Meyo keâewve-mee nw?
(a) me=„er (b) DeveefOekeâej
12. FveceW mes ‘mejue’ Meyo keâe efJeueesce veneR nw–
(c) DeYÙebleefjkeâ (d) mecegefõkeâ
(a) efJejue (b) Je›eâ
Ans. (b) : ‘DeveefOekeâej’ Megæ Jele&veer Jeeuee Meyo nw, peyeefkeâ DevÙe
(c) kegâefšue (d) keâef"ve
MeyoeW keâer Megæ Jele&veer efvecveefueefKele nw–
Ans. (a) : ‘efJejue’ Meyo ‘meIeve’ keâe efJeueesce neslee nw ve efkeâ ‘mejue’
DeMegæ Megæ
keâe, peyeefkeâ DevÙe Meyo ‘mejue’ kesâ efJeueesce nQ~
me=„er me=ef<š
13. ‘pees %eele Fefleneme mes henues keâe nes’ kesâ efueS Skeâ Meyo
DeYÙebleefjkeâ DeeYÙeebleefjkeâ
nw–
mecegefõkeâ meecegefõkeâ
(a) Devewefleneefmekeâ (b) Ssefleneefmekeâ
19. ‘pees yeeÙeW neLe mes meOee ngDee nw’ kesâ efueS Skeâ Meyo nw–
(c) Øeeiewefleneefmekeâ (d) FveceW mes keâesF& veneR
(a) meJÙemeeÛeer (b) JeecehebLeer
Ans. (c) : JeekeäÙeebMe ‘pees %eele Fefleneme mes henues keâe nes’ kesâ efueS
(c) Je»eheeefCe (d) JeerCeeheeefCe
Skeâ Meyo ‘Øeeiewefleneefmekeâ’ nesiee~
Ans. (a) : JeekeäÙeebMe ‘pees yeeÙeW neLe mes meOee ngDee nw’ kesâ efueS Skeâ
Devewefleneefmekeâ - pees Fefleneme mes mebyebefOele ve nes~
Meyo ‘meJÙemeeÛeer’ nesiee~
Ssefleneefmekeâ - pees Fefleneme mes mebyebefOele nes~ JeecehebLeer - meecÙeJeeoer efJeÛeejOeeje keâe JÙeefòeâ~
14. ‘pees DeÛeevekeâ DeeBKeeW mes DeesPeue nes ieÙee nes’ - Fve Je»eheeefCe - efpemekesâ neLe ceW Je»e jnlee nw~
Deveskeâ MeyoeW kesâ efueS Skeâ Megæ Meyo nw– JeerCeeheeefCe - efpemekesâ neLe ceW JeerCee jnleer nes~
(a) DevleOÙee&ve (b) DevlejOÙeeve
20. FveceW mes lelmece - leodYeJe keâe Skeâ Meyo Ùegice DeMegæ nw,
(c) DevleOee&ve (d) DevlejOeeve
Jen nw–
Ans. (c) : JeekeäÙeebMe ‘pees DeÛeevekeâ DeeBKeeW mes DeesPeue nes ieÙee nes’ (a) FeqÛÚkeâe - ÚeÚ (b) F&Ke - FkeäKeg
kesâ efueS Megæ Meyo ‘DevleOee&ve’ nesiee~ (c) GÉle&ve - Gyešve (d) ieesceÙe - ieesyej
UP RO/ARO (Mains) Hindi 2021 (Ex.dt. 24.04.2022) 19 YCT
CLICK HERE FOR FREE MATERIAL

Ans. (a&b) : ‘ÚeÚ’ keâe lelmece ‘ÚeqÛÚkeâe’ neslee nw ve efkeâ 27. ‘Jen yengle Keelee nw’ Fme JeekeäÙe ceW ØeÙegòeâ ‘yengle’ Meyo
‘FeqÛÚkeâe’ leLee ‘F&Ke’ Ùee lelmece ‘F#eg’ neslee nw ve efkeâ FkeäKeg~ Dele: FveceW mes nw–
oesveeW ner efJekeâuhe DeMegæ nQ~ (a) DehetCeeËkeâ yeesOekeâ efJeMes<eCe
21. ‘ceme=Ce’ keâe efJehejerleeLe&keâ Meyo nesiee– (b) heefjceeCe yeesOekeâ efJeMes<eCe
(a) megkegâceej (b) efÛekeäkeâCe
(c) efveefMÛele mebKÙeeJeeÛekeâ efJeMes<eCe
(c) ®#e (d) vejce
(d) legueveelcekeâ efJeMes<eCe
Ans. (c) : ‘ceme=Ce’ keâe efJehejerleeLe&keâ Meyo ‘®#e’ nesiee~ DevÙe MeyoeW
kesâ efJeueesce efvecve nQ– Ans. (b) : ‘Jen yengle Keelee nw’ Fme JeekeäÙe ceW ØeÙegòeâ ‘yengle’ Meyo
Meyo efJeueesce heefjceeCe yeesOekeâ efJeMes<eCe nw~
megkegâceej keâceemegle DehetCeeËkeâ yeesOekeâ efJeMes<eCe– heewves, meJee, [sÌ{, {eF&~
efÛekeäkeâCe Kegjogje efveefMÛele mebKÙeeJeeÛekeâ efJeMes<eCe– Skeâ, heeBÛe, meele, otmeje,
vejce keâÌ[e/keâ"esj efleiegvee~
22. efvecveefueefKele ceW lelmece - leodYeJe keâe ieuele Ùegice nw– legueveelcekeâ efJeMes<eCe– efJeMes<eCe keâer cetueeJemLee ceW ‘lej’ DeLeJee
(a) megIešdš - megIeÌ[ (b) %eeefleie=n - vewnj ‘lece’ ueieekeâj Gmekeâer GllejeJemLee Deewj GlleceeJemLee keâes legueveelcekeâ
(c) heCÙeMeeefuekeâ - hevemeejer (d) ØeefleJesMeer - ceJesMeer Âef<š mes efoKeeÙee peelee nw~ pewmes– DeefOekeâ keâe DeefOekeâlej Deewj
Ans. (d) : ‘ØeefleJesMeer - ceJesMeer’ lelmece - leodYeJe keâer Âef„ mes ieuele DeefOekeâlece~
Ùegice nw~ ‘ØeefleJesMeer’ keâe leodYeJe ‘heÌ[esmeer’ nesiee~ peyeefkeâ DevÙe Ùegice
28. FveceW mes lelmece - leodYeJe keâe Megæ Ùegice nw–
mener nQ~
23. ‘cees#e heeves keâer FÛÚe keâjves Jeeuee’ Fve Deveskeâ MeyoeW kesâ (a) ßes… - mes" (b) Dejnšdš - jnš
efueS Megæ Meyo FveceW mes nw– (c) kebâkeâleer - kebâIeer (d) efce… - ceer"e
(a) efleefle#ee (b) efleleer<eg& Ans. (c) : ‘kebâkeâleer - kebâIeer’, lelmece - leodYeJe keâe Megæ Ùegice nw~
(c) efleefle#eg (d) cejpeerJee peyeefkeâ DevÙe lelmece - leodYeJe kesâ Megæ Ùegice efvecve nQ–
Ans. (b) : JeekeäÙeebMe ‘cees#e heeves keâer FÛÚe keâjves Jeeuee’ kesâ efueS lelmece leodYeJe
Megæ Meyo ‘efleleer<eg&’ nesiee~ ßes…er mes"
efleefle#ee - meoea-ieceea, og:Ke Deeefo menve keâjves keâer Meefòeâ~ DejIešdš jnš
efleefle#eg - efpemeceW menveMeeruelee nes~
efce„ ceer"e
cejpeerJee - cej keâj peerves Jeeuee~
29. efvecveefueefKele ceW ‘DeveIe’ keâe efJeueesce nw–
24. DeMegæ Jele&veer Jeeuee Meyo nw–
(a) efvejIe (b) efve<heehe
(a) GBieueer (b) yeeBme
(c) heebÛeJeeb (d) oeBle (c) ke=âleer (d) DeIeer
Ans. (c) : DeMegæ Jele&veer Jeeuee Meyo heebÛeJeeb nw, Fmekeâe Megæ ™he Ans. (d) : ‘DeveIe’ keâe efJeueesce ‘DeIeer’ nesiee~ DevÙe MeyoeW kesâ
‘heeBÛeJeeB’ nesiee peyeefkeâ DevÙe MeyoeW keâer Jele&veer Megæ nw~ efJeueesce efvecve nQ–
25. Jele&veer keâer Âef„ mes efvecveefueefKele ceW mes Megæ Meyo nw– Meyo efJeueesce
(a) Deveg«enerle (b) Devet«enerle efvejIe DeIeer
(c) Devegie=nerle (d) Devegie=efnle efve<heehe heeheer
Ans. (c) : Jele&veer keâer Âef„ mes ‘Devegie=nerle’ Megæ Meyo nw~ ke=âleer Deke=âleer
26. efvecveefueefKele ceW leodYeJe Meyo nQ–
30. ‘ØeefJeMes<eCe’ keâes keâecelee Øemeeo ieg® ves FveceW mes keâne nw–
(a) Yeeb[ej (b) kegâkeäkegâj
(a) DevleefJe&Mes<eCe (b) meeceevÙe efJeMes<eCe
(c) keâšgkeâ (d) ceÙebo
(c) yee¢e efJeMes<eCe (d) DemeeceevÙe efJeMes<eCe
Ans. (a&d) : ‘Yeeb[ej’ leodYeJe’ Meyo nw efpemekeâe lelmece Yeeb[eieej
nesiee~ peyeefkeâ ‘ceÙebo’ Meyo Yeer leodYeJe nw, efpemekeâe lelmece ‘ce=iesvõ’ Ans. (a) : ‘ØeefJeMes<eCe’ keâes keâecelee Øemeeo ieg® ves DevleefJe&Mes<eCe keâne
nesiee~ DevÙe Meyo lelmece nw, efpevekeâe leodYeJe ™he efvecve nw– nw~ efJeMes<eCe MeyoeW keâer efJeMes<elee yeleeves Jeeues MeyoeW keâes ØeefJeMes<eCe
lelmece leodYeJe keâne peelee nw~ pewmes– ceesnve yengle megvoj nw~ ÙeneB ‘megvoj’ efJeMes<eCe nw
kegâkeäkegâj kegâòee Deewj ‘yengle’ ØeefJeMes<eCe nw keäÙeeWefkeâ ‘yengle’ ‘megvoj’ keâer efJeMes<elee yelee
keâšgkeâ keâ[gDee jne nw~
UP RO/ARO (Mains) Hindi 2021 (Ex.dt. 24.04.2022) 20 YCT
CLICK HERE FOR FREE MATERIAL

UPPSC RO-ARO (Pre) Exam-2021


GENERAL STUDIES
Solved Paper [ Exam Date : 14-12-2021

General Studies (1 to 140) 4. Which of the following is not correctly


1. Match list-I with list-II and select the correct matched?
answer using the code given below the lists- (a) Companies Act – 2013
List-I List-II (b) Indian Contract Act – 1870
(Folk Song) (Occasion of Singing) (c) Partnership Act – 1932
(d) Sale of Goods Act – 1930
A. Chaiti 1. Sanskar geet
Ans. (b) : Indian Contract Act, 1872 enforced on 1st
B. Kajari 2. Braj Lokgeet
September 1872. It is the law which governs contracts
C. Rasia 3. Vivah and Seasonal geet in India. It determines the situation in which the
D. Sohar 4. Seasonal geet promises made by parties to a contract shall be legally
Codes binding on them. Rests are correctly matched.
A B C D 5. What was the total percentage of Central
(a) 4 3 2 1 revenue spent on Military force in British
(b) 3 4 1 2 India?
(c) 1 2 3 4 (a) 40% (b) 45%
(d) 2 1 4 3 (c) 50% (d) 55%
Ans. (d) : The total percentage of Central revenue spent
Ans. (a) : The correct match is as follows:-
on the Military force in British India was about 55%.
List-I List-II The Military was the backbone of the Company's rule in
(Folk Song) (Occasion of Singing) India. Britisher's spent a large proportion of the
Chaiti - Seasonal geet government budget on maintaining army. While the
Kajari - Vivah and Seasonal geet commission had given option (a) as a correct answer to
Rasia - Braj Lokgeet this question in it's initial answer key.
Sohar - Sanskar geet 6. In the Kakori Conspiracy Case, the main
2. Which of the following relation is not correct? accused Pt. Ram Prasad Bismil was hanged in
(a) 1 Byte = 8 Bits which city of Uttar Pradesh?
(b) 1 Kilobyte = 1024 Bytes (a) Varanasi (b) Gorakhpur
(c) 1 Gigabyte = 1024 Kilobytes (c) Kanpur (d) Lucknow
(d) 1 Terabyte = 1024 Gigabytes Ans. (b) : Ram Prasad Bismil was the revolutionary
Ans. (c) : who participated in Mainpuri Conspiracy of 1918 and
the Kakori Conspiracy of 1925 both against the British
Unit Value
empire. He was hanged at Gorakhpur Jail for his role in
1 Byte = 8 Bits
the Kakori Conspiracy on 19th December 1927.
1 Kilobyte = 1024 bytes
7. East Uttar Pradesh first Cable Car Service has
1 Megabyte = 1024 Kilobyte
started in which of the following district?
1 Gigabyte = 1024 Megabyte
(a) Etawah (b) Maharajganj
1 Terabyte = 1024 Gigabyte
(c) Jaunpur (d) Mirzapur
1 Petabyte = 1024 Terabyte
Ans. (d) : In August 2021, Home Minister of India
1 Exabyte = 1024 Petabyte
Amit Shah launched the first Cable Car Service of
3. Which of the following is not correctly Eastern Uttar Pradesh at Vindhyachal in Mirzapur. It
matched? was started in the Kali Khoh of the holy 'triangle' of
(State) (Hot Springs) Vindhyachal dham.
(a) Himachal Pradesh – Manikaran 8. Tulsi plant has medicinal values due to the
(b) Odisha – Taptapani presence of the following–
(c) Maharashtra – Vajreshwari (a) Inorganic Acid
(d) Gujarat – Narayani (b) Organic Acid
Ans. (d) : Naraini Hot spring is situated in Rajasthan (c) Phenols and Flavonoids
While rest all are correctly matched. (d) Lithium Bicarbonate
UP RO/ARO (Pre) Exam-2021 (Ex. Dt. 14.12.2021) 21 YCT
CLICK HERE FOR FREE MATERIAL

Ans. (c) : Tulsi Plant has medicinal values due to the 13. The chief source of Hydrogen Sulphide
presence of Phenols and flavonoids. Several bioactive pollutant is-
constituents such as phenols, flavonoids and (a) Decaying vegetation and animal matter
polyphenols acts as antioxidants are found in Tulsi. (b) Automobiles
9. Which one of the following words cannot (c) Oil Refineries
written from 'TEACHERS' word? (d) Thermal Power plants
(a) REACH (b) CHEER Ans. (a) : Hydrogen Sulfide (H2S) is a colourless gas
(c) SEARCH (d) CHAIR with the odor of rotten eggs. It is an air pollutant present
Ans. (d) : 'CHAIR' Word can not be written from the at high levels in various regions. Decaying vegetation
word 'TEACHERS' because the word 'TEACHERS' and animal matters are the chief sources of H2S. Other
does not contain the Letter 'I'. source of H2S emissions are oil and natural gas
10. Which of the following is considered a extraction, processing, and natural emission from
"Hotspot" of biodiversity in India? geothermal fields.
(a) Aravalli Hills 14. Consider the following events and arrange
(b) Western Ghats these in chronological order–
(c) Eastern Ghats I Launching of Community Development
(d) Indo-Gangetic Plains Programme (CDP)
Ans. (b): Hotspot of biodiversity are the areas which II Recommendation of Democratic
have a high density of biodiversity or mega diversity Decentralization
which are the most threatened points. Out of the 36 III Initiation of Key Village Scheme (KVS)
biodiversity hotspots in the world, 4 are in India. These IV Setting up of District Rural Development
hotspots are:- Eastern Himalayas, The Western Ghats, Agency (DRDA) at district level
Indo-Burma region and Sundaland (includes Nicobar Select correct answer from the code given
Group of Islands). below–
11. Consider the following events and arrange (a) I, II, III, IV (b) II, I, III, IV
them is chronological order: (c) III, I, II, IV (d) IV, III, II, I
I. Establishment of 'Ghadar Party' by Lala Ans. (c): The first systematic attempt in India to
Hardayal. improve the quality of Cattle was the 'Key Village
II. Establishment of "Servants of India Society' by Scheme (KVS) in 1950 during First Five Year Plan.
Gopal Krishna Gokhale. Community Development Programme (CDP) started in
III. Publication of Hicky's 'Bengal Gazette'. October 1952. It was a multi-project programme with
IV. Establishment of 'Bengal Asiatic Society' by Sir the aim of an overall development of rural people.
William Jones. District Rural Development Agency (DRDA) is the
Select the correct answer using the code given principal organization at the district level to manage and
below: oversee the implementation of different anti-poverty
Code programmes of the Ministry of Rural Development.
(a) II, I, III and IV (b) III, IV, II and I DRDA administration has been introduced in 1999.
(c) IV, III, I and II (d) III, IV, I and II Democratic decentralization was recommended by
Balwant Rai Mehta in 1957
Ans. (b) : From 1780, James Augustus Hicky began to
publish the Bengal Gazette a weekly magazine. ‘Asiatic 15. Which of the following state in India has
Society of Bengal’ was founded in 1784 by Sir William maximum number of tribal districts as per
Jones to enhance and further the cause of Oriental "India State of Forest Report 2019"?
research. ‘Servants of India Society’ was established by (a) Nagaland (b) Meghalaya
Gopal Krishna Gokhale in 1905 to unite and train (c) Mizoram (d) Manipur
Indians of different ethnicities and religions in welfare Ans. (*) : As per Forest Report 2019, Nagaland and
work. Meghalaya both has 11 districts, all of which are tribal
The Ghadar Party was a revolutionary group established and hilly districts. Whereas Manipur has 9 and Mizoram
in 1913 headquartered in San Francisco under the have 8 tribal and hilly districts.
leadership of Lala Hardayal, Sohan Singh Bhakna, Sant However, UPPSC had considered option (a) as correct.
Baba Wasakha etc. 16. Which of the following not mentioned in the
12. Where in India, the first rubber based tyre 'Farmers Produce, Trade and Commerce Bill
metro is being built? 2020' of India?
(a) Ahmednagar (b) Surat (a) Essential Commodities
(c) Vadodara (d) Nasik (b) Contract Farming
Ans. (d) : India's first rubber based tyre metro is being (c) Minimum Support Price
built in Nasik, Maharashtra. (d) Agriculture Produce Market Committees
UP RO/ARO (Pre) Exam 2021 (Ex. Dt. 14.12.2021) 22 YCT
CLICK HERE FOR FREE MATERIAL

Ans. (c): The Farmer's Produce Trade and Commerce 21. Niger crops is a oil seed crop. What is the
Bill, 2020 aims to provide an eco-system that facilitates content of oil in its seeds?
freedom of choice for farmers and traders with respect to (a) 7-16% (b) 17-26%
the sale and produce outside of conventionally defined (c) 27-36% (d) 37-47%
market places that otherwise exist. Essential commodities, Ans. (d) : Niger Crops is a oil seed crop cultivated in
contract farming and Agriculture produce Market India and Ethiopia for its edible oil. The seed contains
committee all three are mentioned in the Farmers Produce, 37-47% oil which is pale yellow with nutty taste and
Trade and Commerce Bill, 2020. while Minimum Support pleasant odor.
Price (MSP) is not mentioned in the bill.
22. Which of the following Union Territory of
17. As per census 2011 of India, which one of the India has the lowest sex ratio as per census
following city of Uttar Pradesh is not a 'Million
2011?
City'?
(a) Chandigarh
(a) Agra (b) Prayagaraj
(b) Daman and Diu
(c) Meerut (d) Moradabad
(c) Andaman and Nicobar Islands
Ans. (d) : As per Census 2011, Uttar Pradesh has 7
(d) Lakshadweep
cities with a population of more than 10 lakh (one
million). These cities are:- Kanpur, Lucknow, Ans. (b): As per Census 2011, among all the Union
Ghaziabad, Agra, Meerut, Prayagraj and Varanasi. Territories of India Daman and Diu has lowest sex ratio.
Moradabad city of UP is not a million city. It's total Sex Ratio in Daman and Diu is 618 for each 1000
population is approximately 9 lakhs. males, which is below national average of 940.
18. In the Preamble of the Indian Constitution, 23. With reference to the cultural heritage of Uttar
which Justice is provided to all its citizens? Pradesh which of the following statement/s
(a) Social, Religions and Political is/are correct?
(b) Social, Economic and Cultural (1) The oldest evidence of agriculture in the global
(c) Social, Political and Cultural context has been obtained from many excavated
(d) Social, Economic and Political sites in this region.
(2) The oldest cultivated food grains are barley and
Ans. (d) : Preamble is an introductory statement that
paddy.
explains the document's philosophy and objectives. The
term 'Justice' in the Preamble embraces three distinct Select the correct answer using the code given
forms- Social, Economic and Political. below–
Code-
19. With reference to crop rotation, which of the
following statement is/are correct? (a) Only 1 (b) Only 2
(1) Deep rooted crops should be followed by the (c) Both 1 and 2 (d) Neither 1 nor 2
same type of crop. Ans. (a) : The earliest evidence of rice has been found
(2) Legume must be followed by a non-legume. from Lahuradewa in Sant Kabir Nagar district of U.P.
Select the correct answer using the code given On the basis of the earliest evidence of agriculture,
below- Wheat and Barley are considered to be the oldest
code cultivated grain.
(a) Only 1 (b) Only 2 24. The Six Points Formula proposal of India for
(c) Both 1 and 2 (d) Neither 1 nor 2 South Asian Region was presented by which
Ans. (b) : Crop rotation is a system of growing different Prime Minister of India?
kind of crops in recurrent succession on the same land. (a) Manmohan Singh
In a crop rotation, deep rooted crop should be followed (b) Atal Bihari Vajpayee
by shallow-rooted crop. The leguminous crops should (c) Narendra Modi
be grown after non-leguminous crops as leguminous (d) Indra Kumar Gujral
crops does nitrogen fixation which will facilitate the Ans. (b) : The Six Points Formula proposal of India for
successive non -leguminous crop. South Asian Region was presented by former Prime
20. Which of the following Indian President also Minister of India Atal Bihari Vajpayee.
served as Speaker of Lok Sabha?
25. How many Squares and Triangles are there in
(a) Fakhruddin Ali Ahmad
the following figure?
(b) Dr. Shankar Dayal Sharma
(c) Dr. Zakir Husain
(d) Neelam Sanjiva Reddy
Ans. (d) : Neelam Sanjiva Reddy became the 6th
(a) 2 Squares and 16 Triangles
President of India, serving from 1977 to 1982. He had
the distinction of being the only Speaker of Lok Sabha (b) 2 squares and 14 Triangles
who was later unanimously elected as the President of (c) 3 Squares and 16 Triangles
India. (d) 3 Squares and 18 Triangles
UP RO/ARO (Pre) Exam 2021 (Ex. Dt. 14.12.2021) 23 YCT
CLICK HERE FOR FREE MATERIAL

Ans. (d) : In above given figure, there are 3 squares and 29. In which of the following countries 99 percent
18 Triangles. Each Square inside rectangular shape of its total energy requirement is obtained by
contains 4 small triangles and 4 medium triangle, a Hydro-power?
total of 8 triangle. And rectangular shape has 2 large (a) New Zealand (b) Switzerland
triangles. So, a total of (2 × 8 + 2 = 18) triangles are (c) Norway (d) Brazil
there in the given figure.
Ans.(c) : Hydropower is the mainstay of the Norwegian
26. With reference to Indian Civil Services, which electricity system. Norway obtains more than 99% of its
of the following statements/s is/are correct? total energy requirement with hydropower.
1. Government of India Act, 1919 provided for a
separate examination for the Indian Civil Service, 30. Which of the following countries is the largest
which was to be held in India. producer of fish in the world?
2. In the Indian Civil Service in 1941, the percentage of (a) Japan (b) China
Indians was more as compared to the Europeans. (c) Russia (d) USA
Select the correct answer using the code given Ans. (b) : China is the largest producer of fish in the
below: world. India is the 3rd largest fish producting and 2nd
Code- largest aquaculture nation in the world after China.
(a) Only 1 (b) Only 2 31. The largest source of National Income in India
(c) Both 1 and 2 (d) Neither 1 nor 2 is-
Ans. (c) : The Government of India Act 1919 in also (a) Service Sector (b) Agriculture Sector
known as Montagu-Chelmsford Reforms. This Act (c) Industrial Sector (d) Trade Sector
provided for a separate examination for the Indian Civil Ans. (a) : Service Sector is the largest source of
Services, which was to be held in India. It provided for
National Income in India. The service sector accounts
the establishment of a Public Service Commision.
for 53.89% of total India's GVA of `179.15 Lakh crore
The Government of India Act of 1935 provided for the
with GVA of `46.44 lakh crore, the industry contributes
establishment of not only a Federal Public Service
25.92% while agriculture and allied sector share
Commission but also a Provincial Public Service
Commission and Joint Public Service Commission for 20.19%.
two or more provinces. Also, in the Indian Civil 32. 'Geographical Indication Tag' for black
Services in 1941, the percentage of Indians was more as pottery is associated with which of the
compared to the Europeans. following place in Uttar Pradesh?
27. The 'Tamaria' Tribes are found in which of the (a) Najibabad (b) Khurja
following state of India? (c) Nizamabad (d) Kasganj
(a) West Bengal (b) Rajasthan Ans. (c) : The ‘Geographical Indication Tag’ was
(c) Jharkhand (d) Madhya Pradesh provided to the black pottery of Nizamabad in
Ans. (c) : The 'Tamaria' tribes are found in Jharkhand. Azamgarh districts of Uttar Pradesh in December 2015.
Baiga, Birhor, Banjara, Chero Santhal, Chick Baraik, 33. Dhoopgarh Peak is situated in which of the
Gond etc tribes are also found in Jharkhand. following National Parks?
28. As per the SDG India Index and Dashboard (a) Satpura National Park
2020-21 published by the NITI Aayog of India, (b) Kanha National Park
which one of the following state was not among (c) Bandhavgarh National Park
the top ten states in their performance
(d) Madhav National Park
regarding Sustainable Development Goals?
(a) Punjab (b) Sikkim Ans. (a) : Dhupgarh is the highest peak in the Mahadeo
(c) Uttrakhand (d) Goa Hills (Satpura Range), Madhya Pradesh India. It is
situated in Satpura National Park at an altitude of 1350
Ans.(a): On 3rd June 2021, the 3rd edition of the SDG
India Index and Dashboard 2020-21 was released by meters.
NITI Aayog. In the index Punjab was at 13th position 34. In secret code language, 'HOTEL' is coded as
The top five states and their performance regarding 300, What will be the code for 'BORE' in the
SDG were:- Same code?
Top-5 Number States (a) 60 (b) 160
Rank-1 75 Kerala (c) 140 (d) 40
Rank -2 74 Himanchal Pradesh Tamil Ans. (b) :
Nadu Clearly A = 1, B = 2, C = 3, D = 4, E = 5, F = 6, ...........
Rank-3 72 Andhra Pradesh, Goa, HOTEL = 8 + 15 + 20 + 5 + 12 = 60 × No. of letters
Karnatka, Uttarakhand = 60 × 5 = 300
Rank-4 71 Sikkim So, BORE = 2 + 15 + 18 + 5 = 40 × No. of letters
Rank-5 70 Maharashra = 40 × 4 = 160
UP RO/ARO (Pre) Exam 2021 (Ex. Dt. 14.12.2021) 24 YCT
CLICK HERE FOR FREE MATERIAL

35. Which of the following state of India reported lowest 39. Which of the following statement is not
percent of urban population, as per 2011 census? correct?
(a) Himachal Pradesh (b) Bihar (a) Email-refers to Electronic mail
(c) Odisha (d) Rajasthan (b) WWW-refers to World Wide Web
Ans. (a) : In the given options, As per Census 2011, (c) CPU-refers to Control Processing Unit
Himachal Pradesh has lowest urban population 688,552. (d) ALU-refers to Arithmetic Logic Unit
36. Consider the following Governors of Reserve Ans. (c) : CPU stands for Central Processing Unit. It is
Bank of India and arrange them in known as ‘brain of a computer’. Rest are correctly
chronological order- defined.
I. Dr. C. Rangarajan 40. Cyclone "Tauktae" was formed in which
II. Dr. I. G Patel ocean?
III. Dr. D. Subbarao (a) Bay of Bengal (b) Indian Ocean
IV. Dr. Manmohan Singh (c) Caspian Sea (d) Arabian Sea
Select correct answer using the code given
below- Ans. (d) : Cyclone 'Tauktae' originated over the East
Central Arabian Sea. It was a tropical Cyclone, named
Code-
by Myanmar.
(a) IV, II, III and I (b) II, IV, I, and III
(c) IV, I, II and III (d) II, I, IV and III 41. Which of the following is not correctly
matched?
Ans. (b) : The correct chronological order of above
given Governor of RBI are:- (a) Primary Energy-Tidal Power
(b) Commercial Energy-Oil and Gas
Governor of RBI Period
(c) Non-Commercial Energy-Animal Dung
Dr. I. G Patel 01-12-1977 to 15-09-1982 (d) Non-Conventional Energy-Solar Energy
Dr. Manmohan Singh 16-09-1982 to 14-01-1985
Ans. (a) : Option 'A' is not correctly matched because
Dr. C. Rangarajan 22-12-1992 to 22-11-1997 Tidal energy is a renewable source of energy, which is
Dr. D. Subbarao 05-09-2008 to 04-09-2013 produced by the surge of ocean water during the rise
37. Which of the countries is signatory to the and fall of tides.
Abraham Accord? 42. Traditional Folk-drama "Bhand-Pather" is
(a) Israel, Saudi Arabia, Egypt, Jordan related to which State/Union Territories?
(b) Israel, United Arab Emirates, Egypt Bahrain
(a) Jammu and Kashmir (b) Gujarat
Libya
(c) Israel, United Arab Emirates, Bahrain (c) Uttar Pradesh (d) Assam
(d) Israel, Sudan Afghanistan, Pakistan Ans. (a) : Bhand Pather is a popular form of folk theatre
Ans. (c) : Abraham Accords are a joint statement made of Jammu and Kashmir. It acted as a mirror to a society
between Israel, United States and United Arab Emirates at a time when there was no media.
on 13th August, 2020 to normalize their relations. 43. Choose the odd one out from the following.
38. Match List-I with List-II and select the correct (a) JR6 (b) RV3
answer using the code given below- (c) EK5 (d) DG2
List-I List-II Ans. (a) :
(Soil) (State)
A. Alluvial 1. Rajasthan
B. Black 2. Uttar Pradesh
C. Red 3. Maharashtra
D. Desert 4. Andra Pradesh
Code
A B C D
(a) 1 4 3 2
(b) 2 3 4 1
(c) 4 2 1 3
(d) 3 4 2 1
Ans. (b) : The correct match are as follows – Hence, Option (a) is different from other options.
List-I List-II 44. Which of the following is correct sequence of
(Soil) (State) ecosystem in order of decreasing productivity?
Alluvial - Uttar Pradesh (a) Ocean, Lakes, Grasslands and Mangroves
Black - Maharashtra (b) Managroves, Oceans, Grasslands and Lakes
Red - Andhra Pradesh (c) Mangroves, Grasslands, Lakes and Oceans
Desert - Rajasthan (d) Oceans, Mangroves, Lakes and Grasslands
UP RO/ARO (Pre) Exam 2021 (Ex. Dt. 14.12.2021) 25 YCT
CLICK HERE FOR FREE MATERIAL

Ans. (c) : In ecology, productivity refers to the rate of 49. Which of the following pairs in not correctly
production of biomass in the ecosystem. It can also be matched?
defined as the energy accumulated in plants by (Crops) (Country)
photosynthesis. The correct sequence of ecosystem in (a) Rubber – Thailand
decreasing order of productivity is:- Mangroves > (b) Coffee – Mangolia
Grasslands > Lakes > Oceans. (c) Olive – Spain
45. Under which article of the Indian Constitution, (d) Sugarcane – Brazil
the Parliament can make laws on State List Ans. (b) : Brazil is the largest producer of coffee and
subject for giving effect to international sugar in the world. Rest all pairs are correctly matched.
agreements?
50. Water Pollution of river is measured by-
(a) Article–252 (b) Article–249 (a) Amount of Chlorine dissolved in water
(c) Article–250 (d) Article–253 (b) Amount of Ozone dissolved in water
Ans. (d) : According to Article - 253, the Parliament (c) Amount of Nitrogen dissolved in water
has the sole right to make laws for whole or any part of (d) Amount of Oxygen dissolved in water
India with the motive of executing an international Ans. (d): Biological Oxygen Demand (BOD) is a
treaty, agreement or convention with other countries or measure of the amount of oxygen required to
any decision mode at any association or conference. decompose waste organic matter in water by aerobic
46. With reference to the temple of Bhitargaon, bacteria. Water pollution of river is measured by the
Kanpur, Uttar Pradesh, which of the following amount of oxygen dissolved in water.
statement/s is/are correct? 51. Mitali Express runs between which two cities?
(1) This temple was constructed during 5th century (a) Dhaka – Guwahati
A.D. to 6th century A.D. (b) Dhaka-Chattgaon
(2) It is one of the oldest brick temples of India. (c) Dhaka-Kolkata
Select the correct answer using the code given (d) Dhaka-New Jalpaiguri
below- Ans. (d) : Mitali Express is an international express
(a) Only 1 (b) Only 2 train service which runs between New Jalpaiguri and
Bangladeshi Capital Dhaka every week.
(c) Both 1 and 2 (d) Neither 1 nor 2
52. A Pan-India Campaign 'Ek Pahal' launched on
Ans. (c) : The temple at Bhitargaon in Kanpur, Uttar September 17, 2021 was related to-
Pradesh is a terraced brick building fronted with a (a) Justice delivery at doorstep
terracotta panel. It was built in the 6th century during the (b) Vaccination
Gupta Empire. It is the oldest remaining terracotta (c) Swachh Bharat, Swasth Bharat
Hindu Shrine with a roof and high Shikhara. (d) Use of Green Energy
47. Who among the following visitors to India did Ans. (a) : On 17th September 2021, Ministry of Law
not belong to Portuguese country? and Justice launched Pan-India special campaign 'Ek
(a) Duarte Barbosa (b) Domingo Paes Pahal' for justice delivery at doorstep. The campaign
(c) Peter Mundy (d) Fernao Nuniz was launched to encourage mass registration under Tele
law.
Ans. (c) : Peter Mundy was the British traveller who
came to India in the early 17th Century and wrote about 53. How two identical bulbs should be connected to
the Banjaras whereas Duarte Barbosa, Domingo Paes get maximum light?
and Fernao Nuniz were the Portuguese travelers, visited (1) Both are connected in Series
India during the medieval times. (2) Both are connected in parallel
Select the correct answer using the code given
48. Who led the 'Banaras Rebellion' against the below-
Britishers?
Code
(a) Shujauddaullah (a) In case (1) more
(b) Asaf-ud-Daulah (b) In case (2) more
(c) Raja Chait Singh (c) Same in both (1) and (2)
(d) Raja Maheep Narayan Singh (d) Nothing can be said
Ans. (c) : Second treaty of Banaras is also known as Ans. (b) : Two identical bulbs should be connected in
Treaty of Faizabad. Under this treaty, the Vazir of Oudh parallel circuit to get maximum light. It is because in a
had to pay a larger subsidy for the use of British troops parallel combination, each bulb receives full voltage of
and hand over Banaras to the East India Company. This the battery. As the potential difference of each bulb is
treaty led to a revolt by the Raja Chait Singh of Banaras same as the terminal voltage of the battery, it can be
in 1781. conduced that both of bulbs grows brightly.
UP RO/ARO (Pre) Exam 2021 (Ex. Dt. 14.12.2021) 26 YCT
CLICK HERE FOR FREE MATERIAL

54. Which of the following is not correctly matched? Code


Early Medieval – Present Status A B C D
Cities of Uttar (a) 1 3 2 4
Pradesh (b) 2 4 1 3
(a) Koil – Aligarh (c) 3 1 2 4
(b) Mahotsav Nagar – Mahoba (d) 2 1 4 3
(c) Mahoday Shree – Kannauj Ans. (b) : The correct match is as follows:-
(d) Jejakabhukti – Kaushambi List-I List-II
Ans. (d) : Jejakabhukti was the ancient name of Five year Plan Applied Growth Model
Bundelkhand. The Chandela dynasty ruled the First - Harrod-Domar Model
Jejakabhukti region. Second - Mahalanobis Model
Rest all the pairs are correctly matched. Third - S. Chakravarty Model
55. In which of the following states, the area under Fourth - Ashok Rudra Model
Tube-well irrigation is the highest? 60. Which of the following Sultan appointed a
(a) Uttar Pradesh (b) Bihar Special army to protect the boundaries of the
(c) Punjab (d) Haryana empire?
Ans. (a) : Uttar Pradesh has the largest area under tube (a) Iltutmish (b) Nasiruddin Mahmud
well irrigation followed by Rajasthan, Madhya Pradesh, (c) Balban (d) Alauddin Khilji
Punjab. Ans. (d) : Alauddin Khalji established a huge
56. 'DAVINCI+' and 'VERITAS'-two new mission permanent standing army to satisfy his ambition of
announced by NASA, are related to- conquest and to protect the country from Mongol
(a) Moon (b) Venus invasion. He appointed a special army to protect the
(c) Mars (d) Jupiter boundaries of the empire.
Ans. (b) : NASA has announced DAVINCI + and 61. Which of the following in not a part of
VERITAS two new mission to Venus to examine Polynesia group of Islands?
atmosphere and geological feature of the planet. (a) Tonga (b) Tuvalu
DAVINCI + will measure the composition of 'Venus' (c) Tahiti (d) Tinian
atmosphere to understand how it formed and evolved,
Ans. (d) : Polynesia is a group of over one thousand
whereas VERITAS will map Venus surface to
islands located with in a triangle over the Central and
determine the planet's geological history.
Southern Pacific Ocean. Tonga, Tuvalu, Tahiti, Niue,
57. The main objective of "Sangam Yojana" is- Wallis, Samoa etc are the groups of island and located with
(a) To increase welfare of handicapped person in the Polynesian triangle, whereas Tinian is one of the
(b) To link rivers mutually three principal island of the Northern Mariana Islands.
(c) To make rivers pollution free 62. Which of the following Eatables is the best
(d) To enhance national integrity source of all the essential amino acids?
Ans. (a): Sangam Yojana was launched on 15th August (a) Banana (b) Pulses
1996 to increase the welfare of the handicapped person (c) Egg (d) Fish
under this scheme, disabled person living in rural areas Ans. (c) : Eggs are an excellent source of Protein,
are organized into groups. containing all of the essential amino acids. The total
58. Which among the following states was awarded amino acids in eggs are 10.0 and 10.1 mg/g in the dry
as the best training state for "Van Dhan Yojana" yolk of eggs in corn and wheat based diets, respectively.
by the Government of India in August 2021? Amino acids are organic compounds that come together
(a) Nagaland (b) Mizoram to form proteins in our body.
(c) Meghalaya (d) Madhya Pradesh 63. Given below are two statements, one labelled as
Ans. (a) : Nagaland was awarded as the best training Assertion (A) and the other as Reason (R) :
state for ‘Van Dhan Yojana’ by the Government of Assertion (A) : Dadabhai Naoroji organised the
India in August 2021. The State topped in 3 Categories East India Association in 1866 in London to
- Best Survey State, Best Training State and the most discuss on Indian questions.
number of Van Dhan Vikas Clusters (VDVKCS) Reason (R): He wanted to influence the British
Public opinion.
59. Match list-I with list-II and select the correct
answer using the code given below: Choose the correct answer from the code given
below:
List-I List-II
(a) (A) is true but (R) is false
Five Year Plan Applied Growth Model
(b) (A) is false but (R) is true
A. First 1. S. Chakravarty Model
(c) Both (A) and (R) are true and (R) is the
B. Second 2. Harrod-Domar Model correct explanation of (A)
C. Third 3. Ashok Rudra Model (d) Both (A) and (R) are true but (R) is not the
D. Fourth 4. Mahalanobis Model correct explanation of (A)
UP RO/ARO (Pre) Exam 2021 (Ex. Dt. 14.12.2021) 27 YCT
CLICK HERE FOR FREE MATERIAL

Ans. (c) : The East India Association was established by 69. Which one of the following iron-steel plants in
Dadabhai Naoroji in 1866 in London to influence the public not located on the river side?
in England for India's welfare. This was the first orgnisaiton (a) Bhilai (b) Bokaro
to use persuasion in foreign land for India's interest. (c) Jamshedpur (d) Bhadravati
64. How many letters of the English alphabet Ans. (a) : Bhilai Iron and Steel Plant is located in Bhilai
(Capitals) appear same when looked at in a district of Chhattisgarh. It was set up in 1955 in
mirror? collaboration with the erstwhile USSR. It is not located
(a) 9 (b) 10 on a river side. Whereas, Bokaro Iron and Steel Pant is
(c) 11 (d) 12 located on southern bank of River Damodar.
Ans. (c) : Capital English Alphabet has 11 symmetric Tata Iron and Steel Corporation (TISCO) is situated in
letters that appears same when looked at in a mirror. Jamshedpur, Jharkhand on the banks of River
These letters are :- A, H, I, M, O, T, U, V, W, X, and Y. Subarnarekha and Kharkai.
65. Which one of the following is different from the Bhadravati Iron and Steel plant is located on the Bhadra
rest of the three? River.
(a) Direction (b) Compass 70. Which of the following is related to Brent
(c) Needle (d) Magnet Index?
Ans. (a) : Compass, Needle and Magnet are used to find (a) Crude oil prices
direction. So, direction is different from the rest three. (b) Copper future Prices
66. Which of the following is not correctly (c) Gold future prices
matched? (d) Shipping rate Index
(Language) (Country) Ans. (a) Brent Index is associated with crude oil prices.
(a) Danish Denmark Brent crude is a trading classification of the crude oil
(b) Dutch Netherland that serves as a benchmark price for purchaser of oil
(c) Mandarin China worldwide.
(d) Bahasa Thailand 71. The Silk produced by spiders is called-
Ans. (d) : Bahasa is the language of Indonesia while (a) Tussar silk (b) Gossamer silk
Thai language is widely spoken throughout Thailand. (c) Munga silk (d) Ahimsa silk
Rest all the pairs are correctly matched. Ans. (b) : The silk produced by spiders is called
67. Which of the following statement is not Gossamer silk. Spider silk is comprised of proteins called
correct? Spidroins,that are produced in special glands of Spider.
(a) Human ear becomes most sensitive at 1000 Hz. 72. In Which of the following year almost the
(b) Human eye becomes most sensitive for red entire area of present Uttar Pradesh was
colour. separated from Bengal Presidency and placed
(c) Persistence of hearing is 1/10 second. under Agra Presidency
(d) Persistence of vision is 1/16 second (a) 1832 A.D (b) 1833 A.D
Ans. (b) : Human eye are most sensitive to wavelength (c) 1834 A.D (d) 1835 A.D
of 555 nanometers, which corresponds to green light to Ans. (b) : At, the begining of British period, Uttar
visible spectrum. Hence, Human eye becomes most Pradesh was a part of Bengal Presidency, but in 1833
sensitive for green colour. Rest statements are correct. AD, it was seperated from Bengal Presidency and
68. Find out the missing number in the following placed under Agra Presidency (one of the six North
figure- Western Provinces of British India).
73. What was the name of the poet, who wrote the
5 6 12 popular poem "Firangiya" in Bhojpuri against
4 3 4 'British Colonialism'?
2 3 ? (a) Manoranjan (b) Ranjan Prasad
(c) Triloki Singh (d) Rajendra Pandey
18 27 96
Ans. (a) : Manoranjan Prasad Sinha wrote the Popular
(a) 4 (b) 5 Bhojpuri Poem 'Firangiya' against the British
(c) 3 (d) 6 colonialism in 1921 during non-cooperation movement.
Ans. (d) : In column, 74. Match List-I with List-II and select the correct
Just as, answer using the code given below-
(5 + 4) × 2 = 18 List-I (Fruit) List-II (Largest Producer)
(6 + 3) × 3 = 27 A. Mango 1. Jammu and Kashmir
Similarly, B. Litchi 2. Kerala
(12 + 4) × 6 = 96 C. Coconut 3. Bihar
Hence, the required answer will be option (d). D. Apple 4. Uttar Pradesh
UP RO/ARO (Pre) Exam 2021 (Ex. Dt. 14.12.2021) 28 YCT
CLICK HERE FOR FREE MATERIAL

Code- 78. The practice of gootee in plants is used for the


A B C D purpose of-
(a) 4 3 1 2 (a) Control of insects
(b) 1 2 3 4 (b) For vegetative propagation
(c) 4 3 2 1 (c) Germination of seeds
(d) 1 2 4 3 (d) Weed control
Ans. (c) : The correct match is as follows :- Ans. (b) : Vegetative propagation is asexual method of
List-I List-II plant reproduction that occurs in its leaves, root and
(Fruit) (Largest Producer) Stem. The practice of Gootee in plants is used for the
Mango - Uttar Pradesh vegetative propagation. In Gootee, the roots of aerial
Litchi - Bihar stems are allowed to develop while they are linked to
Coconut - Kerala the parent plant.
Apple - Jammu and Kashmir 79. Which of the following is not one of the twelve
75. Consider the following events and arrange 'HRIDAY' cities chosen for holistic
these in Chronological order- development of heritage cities?
I. Assassination of Curzon Wyllie (a) Agra (b) Gaya
II. Execution of Khudiram Bose (c) Puri (d) Warangal
III. Starting of 'Kesari' newspaper by Bal Ans. (a) : The National Heritage City Development and
Gangadhar Tilak Augmentation Yojana (HRIDAY) is a central sector
IV. Starting of 'Al Hilal' newspaper by Abul scheme launched in January 2015 and was implemented
Kalam Azad in 12 cities of Ajmer (Rajasthan), Amaravati (Andhra
Select the correct answer using the code given Pradesh), Amritsar (Punjab), Badami (Karnataka),
below: Dwarka (Gujarat), Gaya (Bihar), Kanchipuram and
(a) III, II, I and IV (b) II, I, III and IV Velankanni (Tamil Nadu), Mathura and Varanasi (U.P.)
(c) I, III, IV and II (d) III, IV, II and I Puri (Odisha) and Warangal (Telangana). The mission
Ans. (a) : The correct chronological order of the has ended on 31st March 2019.
mentioned events are as follows :- 80. The second most important source after fossil
(1) Kesari is a Marathi newspaper which was founded by fuels contributing to Indian energy need is-
Lokmanya Bal Gangadhar Tilak in January 1881. (a) Hydropower energy (b) Wind energy
(2) Khudiram Bose was one of India's youngest (c) Nuclear energy (d) Solar energy
freedom fighters; he was executed in August, Ans.(d): Solar energy is second most important
1908 in Muzaffarpur Jail in Bihar, at the age of 18. source after fossil fuels contributing to Indian energy,
He was arrested for trying to assassinate British coal is the most important and abudant fossil fuel in
Judge Douglas Kingsford. India. It accounts for 55% of country's energy need.
(3) Madan Lal Dhingra assassinated Curzon Wyllie on 81. "Kashi Lalima" is the variety of which of the
1st July 1909 in Kensington, London.
following crops?
(4) Al-Hilal was a Urdu language newspaper, founded
(a) Carrot (b) Okra Lady finger
by Maulana Abul Kalam Azad in 1912.
(c) Onion (d) Tomato
76. Indonesia does not share its land boundary
with which one of the following countries? Ans. (b) 'Kashi Lalima' is the variety of Lady finger. It
(a) Brunei (b) Timor-Leste is also known as 'Red Bhindi' which was developed by
agriculture experts at the Indian Institute of Vegetable
(c) Malaysia (d) Papua New Guinea
Research (IIVR) in Varanasi.
Ans. (a) : Indonesia shares its land border with
Malaysia on Borneo Island, with Papua New Guinea on 82. In India, businesses with an investment upto 1
Papua island and with East Tomar on Timor island. Crore and turnover upto 5 Crore are known
Brunei is bordered by the South China Sea to the north as-
and Malaysia to the south, east & west. (a) Tiny Enterprise (b) Small Enterprise
77. India Urban Observatory is situated at which (c) Micro Enterprise (d) Medium Enterprise
one of the following place? Ans. (c) : In India, business with an investment upto 1
(a) Dehradun (b) New Delhi crore and turnover upto 5crore are known as Micro
(c) Chandigarh (d) Varanasi Enterprises.
Ans. (b) : The Ministry of Housing and Urban Affairs Business with an investment upto ` 10 crore and
(MOHUA) had inaugurated India Urban Observatory in turnover upto not more than 50 crores is known as
New Delhi in 2019. The observatory is at the heart of all Small Enterprises.
the technology initiatives and plug into the different Business with an investment upto 50 crore and annual
sources of data from the cities, both from real time and turnover upto 250 crore is known as Medium
archival sources. Enterprises.
UP RO/ARO (Pre) Exam 2021 (Ex. Dt. 14.12.2021) 29 YCT
CLICK HERE FOR FREE MATERIAL

83. Which of the following king/kings was/were 88.


Which of the following was declared as his
elected/chosen directly by the public? successor by Guru Nanak?
1. Harshavardhan (a) Bala (b) Lahna
2. Gopal (c) Mardana (d) Sri Chand
3. Yashovarman Ans. (b) : Guru Nanak was the founder of Sikh religion.
4. Nandivarman Pallavamalla He became the first Sikh Guru and his spiritual teaching
Select the correct answer using the code given laid the foundation on which Sikhism was formed. He
below- appointed Lahna as successor. Guru Nanak renamed
Code- Lahna as Guru Angad. Guru Nanak was died in 1539 at
Kartarpur.
(a) Only (1), (2) and (4)
(b) Only (2), (3) and (4) 89. Match list-I with list-II and select the correct
answer using the code given below-
(c) Only (2) and (4)
List-I List-II
(d) Only (2) and (3)
(Minerals) (Industries in which
Ans. (c) : Gopal was the founder of Pala Dynasty. He largely used
was directly elected by the Public. He was succeeded by
A. Limestone 1. Cement
his son Dharmapala. According to the Khalimpur
Copper plate inscription, Gopala, in order to rescue the B. Copper 2. Electrical goods
people from matsya-nyaya, founded the Pala dynasty C. Bauxite 3. Aeroplane
when he was elected the king by noblemen of the realm. Manufacturing
Nandivarman Palavamalla was a Pallava ruler. In 731 D. Manganese 4. Steel
CE, the Pallava king Parameshvaraman II died without Codes
any heir and hence, Pallava empire was kingless. On the A B C D
request of people, Nadivarman II was elected as the (a) 1 2 3 4
Pallava king at the age of 13. (b) 2 3 1 4
84. Which of the following Island is known as "The (c) 4 3 2 1
Land of Promise"? (d) 3 4 1 2
(a) Cuba (b) Java Ans. (a) : The correct match is as follows :-
(c) Sulawesi (d) Mindanao List-I List-II
Ans.(d) : Mindanao is known as the ‘Land of Promise’. (Mineral) (Industries in which
It is the most culturally diverse Island in Philippines largely used)
where people of different languages, tribes and races Limestone - Cement
meet. It is known as the ‘Land of Promise’ due to its Copper - Electrical goods
rich biodiversity and natural resources. Bauxite - Aeroplane Manufacturing
85. The percentage of work force in total Manganese - Steel
population of Uttar Pradesh 90. In the following number series, a wrong
(a) 18.5% (b) 23.7% number is given-
(c) 30.2% (d) 31.2% 2, 6, 16, 38, 84, 168, 368, 750
Ans.(b): The percentage of work force in total The wrong number is-
population of Uttar Pradesh is 23.7%. (a) 38 (b) 84
(c) 168 (d) 750
86. Which one is odd among the following group?
(a) Tokyo (b) London Ans. (c) : (2 × 2) + 2 = 6
(2 × 6) + 4 = 16
(c) New York (d) Beijing
(2 × 16) + 6 = 38
Ans. (c) : Except New York, all are capital cities in (2 × 38) + 8 = 84
among given option. New York is a state in U.S. Hence,
New York is odd among them. (2 × 84) + 10 = 178
87. How many 3's are there in the following (2 × 178) + 12 = 368
sequence? Which are neither preceded by 6 nor (2 × 368) + 14 = 750
immediately followed by 9? Hence, the wrong number is 168.
91. Which of the following state started E-ration
9, 3, 6, 6, 3, 9, 5, 9, 3, 7, 8, 9, 1, 6, 3, 9, 6, 3, 9
(a) Four (b) Three Card Service in first time?
(c) Two (d) One (a) Gujarat (b) Maharashtra
(c) Uttar Pradesh (d) Delhi
Ans. (c) : 9,3, 6 ,6,3,9,5, 9,3, 7 ,8,9,1,6,3,9,6,3,9
Ans. (d) : In March 2015, Chief Minister Arvind
Hence, there are two 3's in the sequence that are neither Kejariwal launched the country's first e-ration Card
preceded by 6 nor immediately followed by 9. Service in Delhi.
UP RO/ARO (Pre) Exam 2021 (Ex. Dt. 14.12.2021) 30 YCT
CLICK HERE FOR FREE MATERIAL

92. In which of the following historical melas/fairs of Ans. (b) : Santo Domingo is the capital and the largest
Uttar Pradesh both Hindu and Muslims come? city of the Dominican Republic and Roseau is the
(a) Nauchandi Mela (b) Bavaro Baba Mela capital of Dominica. Rest all the pairs are correctly
(c) Fuldol Mela (d) Bateshwar Mela matched.
Ans. (a) : Nauchandi Mela is an annual fair held at 97. How is liver affected on fasting for more than
Nauchandi Ground in Meerut. This fair is a rare symbol 10 day?
of communal harmony of Hindus and Muslims. (a) Glucose level in liver diminished
93. Match list-I with list-II and select the corret (b) Glucose Level in liver increases
answer using the code given below- (c) In liver triglycerides decrease
List-I List-II (d) In liver triglycerides increase
(War) (Years) Ans. (a) : The liver is a central organ required for
A. Second Anglo- 1. 1803 - 05 A. D. metabolic homeostasis. On fasting for more than 10
Mysore War days, the glucose level in liver gets decreased. Our body
stores excess glucose as glycogen (a polymer of
B. Second Anglo- 2. 1780 - 84 A. D.
French War glucose), which becomes liberated in times of fasting.
C. Second Anglo- 3. 1878 - 80 A.D. 98. Which of the following statement/s is/are
Maratha War correct with reference to Indian Railways?
1. Achieving 100 percent electrification by 2023.
D. Second Anglo- 4. 1750 - 54 A.D.
Afghan War 2. A net zero carbon emission network by 2030.
Select the correct answer using the code given
Codes : below-
A B C D Code
(a) 2 4 1 3 (a) Only (1) (b) Only (2)
(b) 3 2 4 1 (c) Both (1) and (2) (d) Neither (1) nor (2)
(c) 1 3 2 4
Ans. (c) : As per Economic Survey 2021-22, Indian
(d) 4 2 3 1 Railways (IR) has set a target of 100% electrification of
Ans. (a) : The correct match is as follows- its network by December 2023. Indian Railways (IR) is
List-I List-II aiming for 100% electrification to reduce carbon
Second Anglo Mysore War - 1780-1784 A.D. emissions and improve train operations efficiency in
Second Anglo French War - 1750-1754 A.D. order to become the world's largest green railway by
Second Anglo Maratha War - 1803-1805 A.D. 2030. Indian Railway plans to achieve net zero carbon
Second Anglo Afghan War - 1878-1880 A.D. emission by 2030.
94. The last Subedar of 'Bengal, who was 99. Who is the author of the book "Sanskriti Ke
appointed by Mughal Emperor- Char Adhyay"?
(a) Murshid Quli Khan (a) Mahavir Prasad Dwivedi
(b) Sarfaraz Khan (b) Ramdhari Singh Dinkar
(c) Shuja-ud-Din Muhammad Khan (c) Maithili Sharan Gupt
(d) Alivardi Khan (d) Jaishankar Prasad
Ans. (a) : Murshid Quli Khan was the last subedar of Ans. (b) :
'Bengal', who was appointed by Mughal Emperor, Aurthor Important works
Farrukh Siyar. He was the founder of the Nawab regime Ramdhari Singh Dinkar Sanskriti Ke Char Adhyay,
in Bengal. He served from 1717 to 1727. Urvashi, Rashmirathi,
95. Which one of the following districts of Uttar Neel Kusum
Pradesh does not share border with Napal? Mahavir Prasad Diwedi Hindi Bhasha Ki Utpatti,
(a) Balarampur (b) Shravasti Sampatti Shastra
(c) Kushinagar (d) Lakhimpur Kheri Maithali Sharan Gupt Bharat Bharati, Urmila,
Ans. (c) : A total seven districts of Uttar Pradesh share PanchVati, Jayadrath Vadh
border with Nepal. They are- Pilibhit, Lakhimpur Kheri, Jaishankar Prasad Kamayani, Dhruv
Baharaich, Shravasti, Balrampur, Siddharthanagar and Swamini, Ansoo.
Maharajganj. 100. Which of the following is not correctly
96. Which of the following pair is not correctly matched?
matched? (Historical Melas/fairs (Places/Region)
(Country) (Capital) of Uttar Pradesh )
(a) Antigua and Barbuda – Saint John's (a) Jal Vihar Mela – Bundelkhand
(b) Dominica – Santo Domingo (b) Bateshwar Mela – Western Region
(c) El Salvador – San Salvador (c) Baldev Chhath Mela – Purvanchal
(d) Bahamas – Nassau (d) Shukratal Mela – Muzaffar Nagar
UP RO/ARO (Pre) Exam 2021 (Ex. Dt. 14.12.2021) 31 YCT
CLICK HERE FOR FREE MATERIAL

Ans. (c) : Baldev Chhath Mela is organised at Palwal, Choose the correct answer from the code given
in Haryana. There is also a temple dedicated to Lord below-
Balram near Palwal Municipal Office Chowk. Rest all Code-
pairs are correctly matched. (a) Both (A) and (R) are true and (R) is the
101. Which of the following pair/s is/are correctly correct explanation
matched? (b) Both (A) and (R) are true and (R) is not the
(1) Beet – Sugar correct explanation of (A)
(2) Honey – Glucose and Fructose (c) (A) is true but (R) is false
(3) Cotton – Cellulose (d) (A) is false but (R) is true
(4) Milk – Lactose
Ans. (c) : Rabindra Nath Tagore renounced his title of
Select the correct answer using the code given
'Knighthood' in protest against the Jallianwala Bagh
below-
massacre of 1919, where General Dyer had committed
Code-
brutal murder of innocent & unarmed Indians.
(a) Only (1), (2) and (3)
Hence (A) is true but (R) is false.
(b) Only (2), (3) and (4)
(c) Only (1), (2) and (4) 105. 'Electricity' is related to 'wire' in the same way
(d) (1), (2), (3) and (4) as 'water' is related to-
Ans.(d) : Cotton fibers are the purest forms of cellulose. (a) Jug (b) River
Approximately, 90% of the cotton fibers are cellulose. (c) Bottle (d) Pipe
Lactose is found primarily in milk products, including Ans. (d) : Electricity flows in the 'wire' in the same way
yogurt, Cheese, and butter. All milk (raw or water flows in the pipe.
pasteurized) contains lactose. 106. How many Mahajanapadas were located in
Honey composed primarily of water and two sugars present day of Uttar Pradesh among the oldest
called fructose and glucose. sixteen Mahajanapadas of India?
Beet contains Sugar. Sugar beets are good source of
(a) 06 (b) 07
fiber, which help regulate the digestive system.
Hence, all the pairs are correctly matched. (c) 08 (d) 09
102. In the second half of 18th century, who was Ans. (c) : The Mahajanpadas were a set of Sixteen
often called the local representative of "Aura kingdoms that existed in ancient India. According to
and Authority of Company Bahadur"? Buddhist text, Anguttara Nikaya (the land between
(a) Kotwal (b) Daroga Himalayas and Narmada) was divided into 16
(c) Amala (d) Landlord (Jamindar) independent States Known as Mahajanpadas. They are-
Anga, Kosala, Vajji, Kashi, Malla, Chedi, Vatsa, Kuru,
Ans. (b) : Lord Cornwallis created a ‘Permanent Police
Force’ in India. Each district was placed under many Panchala, Matsya, Surasena, Avanti, Gandhara,
'Thanas', each headed by 'Daroga'. In the second half of 18th Kamboja, Asmaka and Magadha. Out of these 16
century, Daroga was often called the local representative of Mahajanpadas, 8 Mahajanpadas were located in present
"Aura and Authority of Company Bahadur". day Uttar Pradesh, they were:- Kashi, Kosala, Mallas,
103. A farmer,which has land holding between 1 to Chedi, Vatsa, Kuru, Panchala and Shurasena/Surasena.
2 hectare is known as- 107. Which of the following cities in India has the
(a) Marginal Farmer (b) Big Farmer lowest percentage of slum population as per
(c) Medium Farmer (d) Small Farmer 2011 census?
Ans. (d) : As per Agriculture Census, 2015-16 the (a) Ahmedabad (b) Jaipur
operational holding are categorized in five size (c) Lucknow (d) Bengaluru
classes they are as follows:- Ans. (a) : As per Census 2011, Ahmedabad has the
SN Category Size-Class lowest percentage of Slum population among the above
1 Marginal Below 1.00 hectare mentioned cities. Here about 4.49% of the population
2 Small 1.00-2.00 hectare lives in slums. Whereas 10.62% in Jaipur, 12.9% in
Lucknow and 8.39% in Bangalore of population lives in
3 Semi-Medium 2.00-4.00 hectare
slums approximately.
4 Medium 4.00-10.00 hectare
108. 'Jhini Jhini Bini Chadariya,' a famous Novel of
5 Large 10.00 hectare and above
his times is written by-
104. Given above are two statements, one labelled as (a) Kashi Nath Singh (b) Neerja Jadhav
Assertion (A) and the other as Reason (R)-
(c) Abdul Bismillah (d) Ajay Mishra
Assertion (A): Rabindranath Tagore renounced
Knighthood. Ans. (c) : 'Jhini Jhini Bini Chadariya' is a famous novel
Reason (R): He wanted to participate in the Non- written by Abdul Bismillah. It is Hindi novel based on
Cooperation Movement. the life of weavers of Banaras.
UP RO/ARO (Pre) Exam 2021 (Ex. Dt. 14.12.2021) 32 YCT
CLICK HERE FOR FREE MATERIAL

109. In 1993, which of the following language was 114. Consider the following-
not included in the Eighth Schedule? 1. Kalaripayattu
(a) Maithili (b) Santhali 2. Thang-Ta
(c) Bodo (d) Dogri 3. Mallakhamba
Ans. (*) : The Eight Schedule of Indian Constitution 4. Gatka
consists of 22 languages. Out of these 22 languages, 14 Which of the above game/s has been included
were initially included in the Constitution. Sindhi in "Khelo India Youth Games"?
language was added in 1967. Thereafter, three more Select the correct answer using the code given
language Konkani, Manipuri and Nepali were included below-
in 1992. Subsequently, Bodo, Dogri, Maithali and Code
Santhali were added in 2004. Hence, all the given (a) Only 1 and 2 (b) Only 2, 3 and 4
options are correct. (c) Only 1, 3 and 4 (d) 1, 2, 3, and 4
110. In a Sitar, which type of sound vibrations are Ans. (d) : 'Khelo India Youth Games' is a part of the
produced? revamped national programme for development of
(a) Progressive and Longitudinal sports, 'Khelo India'. It is the Indian Government
(b) Progressive and Transverse flagship grassroot talent hunt programme. The Sports
(c) Stationary and Longitudinal Ministry had approved the inclusion of four indigenous
games to be a part of the Khelo India Youth Games,
(d) Stationary and Transverse
2021. These games were- Gatka, Kalaripayattu, Thang-
Ans. (d) : A sound wave is produced by vibrating Ta and Mallakhamba. These four games represent
objects. In Sitar, both the ends are fixed, so stationary and different parts of the country.
transverse waves are produced. Stationary transverse 115. 'Djibouti' Code of Conduct is related to-
waves are the result of superimposing of two identical
(a) Conservation of Congo basin
transverse waves travelling in opposite direction.
(b) International Maritime grouping against
111. Which of the following nutrient is most useful piracy
for enhancing oil content in mustard crop? (c) Charter of ethics
(a) Calcium (b) Sulphur (d) Principles for conducting nuclear test
(c) Zinc (d) Iron Ans. (b) : The Djibouti Code of Conduct concerning the
Ans. (b) : Sulphur is most useful for enhancing oil Repression of Piracy and Armed Robbery against ships
content in mustard crop. Sulphur is involved in the in Western Indian Ocean and the Gulf of Aden. It was
formation of Chlorophyll, glucosides and glucosinolates adopted in January 2009.
(musturd oil), activation of enzymes and Sulphydryl 116. With reference to the Indian National Congress
Linkage that are the source of pungency in oilseeds. Session held at different places of Uttar
Hence, adequate Sulphur is very much crucial for Pradesh, match list-I with list-II and select the
mustard crop. correct answer using the code given below-
112. Which of the following is not correctly List-I (Place) List-II (Year)
matched? A. Meerut 1. 1916
Foreign Travellers Year/s of Tour to India
B. Kanpur 2. 1905
(a) Fa-Hien – 399-414 A.D.
C. Lucknow 3. 1946
(b) Hiuen-Tsang – 629-645 A.D.
(c) I-tsing – 679-695 A.D D. Banaras 4. 1925
(d) Al-Masudi – 957 A.D. Code
Ans. (d) : In the given option Al Masudi came in India A B C D
during 10th century and he died in 956 AD. Hence, (a) 1 4 3 2
option (d) is correct option while commission had given (b) 3 1 2 4
option (c) as correct answer to the question. (c) 3 4 1 2
113. "Shabri Snakalp Abhiyan" is associated with (d) 2 3 4 1
(a) To help those poor people who are below we Ans. (c) : The Correct matches are as follows:-
the poverty line List-I List-II
(b) Empowerment of Women (Indian National (Year) President
(c) A scheme for help groups Congress Session
(d) National Nutrition Mission Place)
Ans. (d) : 'Shabri Sankalp Abhiyan' is associated with Meerut - 1946 - J.B. Kripalani
National Nutrition Mission. It was launched by UP Kanpur - 1925 - Mrs. Sarojini Nadu
Government to eradicate malnutrition. It would also Lucknow - 1916 – Ambica Charan Mazumdar
help to bring down mother and child mortality rates. Banaras - 1905 - Gopal Krishna Gokhale
UP RO/ARO (Pre) Exam 2021 (Ex. Dt. 14.12.2021) 33 YCT
CLICK HERE FOR FREE MATERIAL

117. Italy launched its first "Food Mega Park" in (b) Meghalaya, Arunachal Pradesh Nagaland,
India in June 2021 at which of the following Mizoram
place? (c) Arunachal Pradesh, Nagaland, Meghalaya,
(a) Jaipur (b) Fanidhar Mizoram
(c) Ludhiana (d) Kochi (d) Arunachal Pradesh, Mizoram, Nagaland,
Ans. (b) : Italy launched its first 'Mega Food Park' Meghalaya
project in India at Fanidhar in Mehsana district of Ans. (d) : As per Census 2011, total Scheduled Tribes
Gujarat in June 2021. The project aims to create population of above mentioned states are as:-
synergy between agriculture and industry of the two States Schedule Tribe Population
countries and focuses on the research and development Arunachal Pradesh - 9,51,821
of new and more efficient technologies in the sector. Mizoram - 1,036,115
118. Consider the following events and arrange Nagaland - 1,710,973
these in chronological order: Meghalaya - 2,555,861
I. Development of sugar industry by Indians 121. Which of the following is not correctly
II. Starting of first jute mill in Rishra matched?
III. Production of steel in India for the first time (Animal) (Breed)
IV. Starting of first textile mill in Bombay (a) Buffalo – Bhadawari
Select the correct answer from the code given (b) Cow – Sindhi
below: (c) Goat – Jamunapari
Code (d) Sheep – Tharparkar
(a) I, II, IV and III (b) IV, II, III and I
Ans. (d): In the given option Tharparkar is a breed of
(c) II, I, III and IV (d) III, II I and IV cow not sheep. Rest all pairs are correctly matched.
Ans. (b) : Bombay Spinning and Weaving Company However, UPPSC had considered option (a) as correct
was known to be first textile mill was established in answer.
Bombay in 1854 by Cowasjee Nanabhai Davar along 122. Which of the following statement/s is/are true
with his associates. The first Jute Mill was established for Uttar Pradesh?
by George Acland and Babu Bysumber Sen in Rishra,
1. Is the largest hub of sugarcane production in
Bengal in 1855.
India
Modern Steelmaking in India began with the setting of
2. Is the largest milk producing state in India.
the first blast furnace of India at Kulti in 1870 and
production began in 1874, which was setup by Bengal 3. Is the highest Producer of rice in India.
Iron works. Development of Sugar industry by Indians (a) Only (1) is correct
was started in 1903. In the year 1903, the first Sugar (b) (2) and (3) are correct
Mill in India was set up at Pratappur in Deoria district (c) (1), (2) and (3) are correct
of U.P. (d) Only (1) and (2) are correct
119. Arrange the following seasons in chronological Ans. (d) : Uttar Pradesh is largest producer of
order accounting to Hindu calendar: sugarcane and milk in India. West Bengal is the largest
(i) Autumn producer of rice in India followed by UP and Punjab.
(ii) Summer Hence, only statement (1) and (2) are correct.
(iii) Spring 123. During the reign of which of the following
(iv) Rainy Sultan of Delhi, Hindu gods and Goddesses
Select the correct answer using the code given were worshiped in royal palaces?
below- (a) Qutb-ud-din Mubarak Khilji
(a) (ii), (iv), (iii) and (i) (b) Nashiruddin Mahmud
(b) (iii), (ii), (iv) and (i) (c) Nasiruddin Khusro Shah
(c) (iv), (ii), (i) and (iii) (d) Alauddin Masud Shah
(d) (i), (iv), (ii) and (iii) Ans. (c) : After the murder of Mubarak Shah, Khusrau
Ans. (b) : According to Hindu calendar, there are six Khan begged pardon to the nobles for his bloody act
seasons in India, they are:- Spring(Vasant Ritu), and ascended the throne with support of nobles in 1320
Summer(Grishma Ritu), Monsoon(Varsha Ritu), CE. He assumed the title of Nasiruddin Khusrau Shah.
Autumn(Sharad Ritu), Pre-Winter(Hemant Ritu), and He belonged to a Hindu military caste called Baradu
Winter(Shishir Ritu). later he converted into Islam, during his reign, Hindu
120. As per the 2011 census, the correct ascending Gods and Goddesses were worshiped in royal palaces.
order of the following states in terms of 124. Which of the following river does not flow
Scheduled Tribes population is- through the state of Uttar Pradesh?
(a) Arunachal Pradesh, Meghalaya, Nagaland, (a) Son River (b) Hindon River
Mizoram (c) Gandak River (d) Sharda River
UP RO/ARO (Pre) Exam 2021 (Ex. Dt. 14.12.2021) 34 YCT
CLICK HERE FOR FREE MATERIAL

Ans. (c) : Gandak river rises in the glacier of Southern 129. Match list-I with list-II and select the correct
Tibet. It is one of the major rivers in Nepal & India. The answer using the code given below-
river flows across the Gangatic plain of Bihar and List-I List-II
eventually merges with the river Ganga near at Patna. It (Name of the Act in India) (Year)
does not flow through the state of Uttar Pradesh whereas, A. Forest Conservation Act 1. 1980
Son, Hindon and Sharda rives flow through U.P. B. The Environmental 2. 1986
125. The Disaster Management Act was passed by Protection Act
the Parliament of India in- C. The Air (Prevention and 3. 1981
(a) 2001 (b) 2005 Control of Pollution) act
(c) 2009 (d) 2013 D. The Water Pollution 4. 1974
Ans. (b) : The Disaster Management Act, 2005 was Control Act
enacted in the country on 26th December, 2005. It is an Codes
act passed by the Government of India for the efficient A B C D
management of disasters and other matters connected to it. (a) 3 1 4 2
126. Which one of the following book by former (b) 1 4 2 3
RBI Governor, Bimal Jalan was published in (c) 4 3 2 1
July, 2021? (d) 1 2 3 4
(a) The future of India Ans. (d) : The correct match is as follows.
(b) India's Politics List-I List-II
(c) Emerging India (Name of the Act in India) (Year)
(d) The India Story Forest Conservation Act - 1980
Ans. (d) : In July 2021, a book titled 'The India Story' The Environment Protection Act - 1986
was published, which is written by former RBI The Air (Prevention and - 1981
Governer Bimal Jalan. The book focuses on India's Control of Pollution) Act
economic history and aims to provide lesson for the The Water Pollution Control Act - 1974
future of India's political economy. 130. Which Government enterprises signed MOU in
127. "When ......... was born, the earth sank two and August 2021 to invest in Uttar Pradesh Defense
a half hands? Corridor?
Above statement/Folk saying is related to (a) Bharat Earth Movers Limited
which historical personality of Uttar Pradesh? (b) Bharat Dynamics Limited
(a) Vidhyadhar (c) Hindustan Copper Limited
(b) Mahil (d) Mishra Dhatu Nigam Limited
(c) Alha Ans. (b) : In 2021, Bharat Dynamics Limited (BDL)
(d) Paramardi signed MoU to invest in UP Defence Corridor. As per
the MoU, BDL will be setting up a manufacturing unit
Ans. (c) : 'When Alha was born, the earth sank two and in Jhansi and will acquire 215 acres of land in Jhansi
a half hands'. It is a popular folk singing style of under a lease agreement for an initial period of 30 years.
Bundelkhand region. This style of folk music is
131. Which of the following is the largest Fresh
popularly known as 'Alha'.
water lake of India?
128. Who publishes Global Climate Risk Index (a) Kolleru (b) Wular
Report? (c) Nalsarovar (d) Sambhar
(a) United Nations Environment Programme
Ans. (b) : Wular lake is largest fresh water lake of
(b) Germanwatch India. It is located at Bandipora district of Jammu and
(c) World Meteorological Organisation Kashmir. The lake supports fishing industry and is
(d) World Bank valuable source of water for irrigation and domestic
Ans. (b) : ''Global Climate Risk Index'' show to what uses. Sambhar lake is India's largest inland saline lake.
extent a country and region has been affected by 132. Which of the following schemes are related to
impacts of weather related loss events, such as stroms, the skill development?
floods, heat, waves etc. This report is published by (1) STRIDE
Germanwatch. Germanwatch is an independent (2) STRIVE
development and environment organisation which (3) SANKALP
works for sustainable global development. (4) SHREYAS
UP RO/ARO (Pre) Exam 2021 (Ex. Dt. 14.12.2021) 35 YCT
CLICK HERE FOR FREE MATERIAL

Select the correct answer using code given 136. Which of the following cities was declared
below- winner in "Social Aspect Category" of the
(a) (1) and (2) only India Smart Cities Awards 2020, which were
(b) (3) and (4) only declared in June 2021?
(c) (2), (3) and (4) only (a) Tirupati (b) Bhopal
(d) (1), (2), (3) and (4) only (c) Jaipur (d) Varanasi
Ans. (c) : STRIVE, SANKALP and SHREYAS Ans. (a) : The Ministry of Housing and Urban Affairs
Scheme are related to the skill development. Where as (MOHUA) declared the Smart City Award result for
scheme for Trans-disciplinary Research for India's 2020 on 25th June 2021, Tirupati was declared winner in
Developing Economy (STRIDE) aims to promote an ‘Social Aspect Category’ of the India Smart Cities
innovative culture for trans-disciplinary research Award 2020.
especially in universities and colleges. 137. Headquarters of International Potato Centre
The Cabinet Committee on Economic Affairs approved (CIP) is situated in-
Skills Acquisition and Knowledge Awareness for (a) Lima, Peru (b) Shimla, India
Livelihood Promotion (SANKALP) and Skill (c) Dakar, Senegal (d) Dublin, Ireland
Strengthening for Industrial Value Enhancement Ans. (a): International Potato Center (C.I.P.) was
(STRIVE) Schemes to boost Skill India Mission in 2019. founded in 1971 as a research for development
Scheme for Higher Education Youth in Apprenticeship organization with a focus on potato, sweet potato and
and Skill (SHREYAS) aims to enhance the andean roots and tubers. C.I.P. is headquartered in
employability of Indian youth,by providing on the job Lima, Peru.
work exposure and earning of stipend. 138. Which of the following is not correctly
133. The Monoclimax Theory was propounded by- matched?
(a) F.E. Clements (b) C.C. Park (a) Mahila Samridhi Yojana-1993
(c) D.V. Ager (d) S.J. Gould (b) Women Self-Empowerment Plan-1998
Ans. (a) : Monoclimax Theory was propounded by (c) Women Empire Plan - 1989
Federic Clements in 1916. According to this theory (d) National Posahar Mission Plan-2003
'every region has one climax plant community towards Ans.(d): In the given options National Posahar Mission
which all communities are developing. Plan was launched in 2001 not in 2003. It was also
134. Arrange the following words according to known as Nation Nutrition Mission. Rest of all given
dictionary, and find the word that comes on option are correctly matched.
just second place in dictionary? 139. What is the 'quorum' required in the State
(a) Conclusion (b) Concentrate Legislature to hold a meeting?
(c) Confidence (d) Count (a) Thirty members or one tenth of total
Ans. (a) : In the given words, the word 'Conclusion' membership, whichever is less
will comes on just second position in dictionary. The (b) One-half of the total membership of the
correct sequence of words is as follows:- Concentrate, House
conclusion, confident, count. (c) One-fourth of the total membership of the
135. With reference to the Fort William College House
Calcutta, which of the statement/s is/are (d) Ten members or one tenth of the total
correct? members of the House, whichever is greater
1. It was founded on 10th June, 1800 A.D. within Ans. (d) The quorum to constitute a meeting of a House
the Fort William Complex in Calcutta. of the Legislature of States shall be at least ten
2. The main purpose of establishing this college, members or one tenth of the total number of members
was to teach Indian languages to British of the House whichever is greater.
officers. 140. In which of the following year, was the
Select the correct answer using the code given authority of the British established over the
below: fort of Prayagraj (U.P.)?
Code- (a) 1790 A.D. (b) 1792 A.D.
(a) Only 1 (b) Only 2 (c) 1796 A.D. (d) 1797 A.D.
(c) Both 1 and 2 (d) Neither 1 nor 2 Ans. (d): Allahabad Fort was built by Emperor Akbar
Ans. (b) : Fort William College was established by in 1583 AD. The fort stands on the banks of confluence
Lord Wellesely on 10 July 1800 AD, located within the of rivers Ganga and Yamuna. In 1797 AD, the British
Fort William complex in Calcutta. The main purpose of established authority over the Allahabad Fort. This
establishing this college was to provide instruction in place became the cantonment of British troops. In 1801,
vernacular languages of India to the civil and military Allahabad (Now Prayagraj) was formally handed over
officials of the East India Company. to East India Company by Sadat Ali Khan.
UP RO/ARO (Pre) Exam 2021 (Ex. Dt. 14.12.2021) 36 YCT
CLICK HERE FOR FREE MATERIAL

Gòej ØeosMe meceer#ee DeefOekeâejer/meneÙekeâ meceer#ee DeefOekeâejer (Øeer) hejer#ee 2021


meeceevÙe efnvoer
nue ØeMve-he$e (hejer#ee efleefLe : 5 efomecyej, 2021)

(DeOÙeeÙeJeej efJeMues<eCe meefnle JÙeeKÙee)


1. ‘vewmeefie&keâ’ keâe efJehejerleeLe&keâ Meyo nw- Ans. (d) : ‘keâhe&š’ keâe leÆJe ™he ‘keâheÌ[e’ nw~
(a) DevegJe&j (b) DeeefJeYee&Je Meyo – OJeefveÙeeW kesâ cesue mes yeves meeLe&keâ JeCe& mecegoeÙe Meyo keânueeles
(c) Øeeke=âle (d) ke=âef$ece nQ~ Glheefòe keâer Âef° mes Meyo Ûeej Øekeâej kesâ nesles nQ- (1) lelmece, (2)
Ans. (d) : vewmeefie&keâ keâe efJehejerleeLe&keâ Meyo ke=âef$ece nesiee~ leÆJe, (3) osMepe, (4) efJeosMeer
efJehejerleeLe&keâ Meyo – Jes Meyo pees efkeâmeer DevÙe Meyo kesâ meJe&oe lelmece Meyo – efkeâmeer Yee<ee kesâ Jes cetue Meyo pees mebmke=âle mes nesles
efJehejerle DeLe& keâes Øekeâš keâjles nQ, efJehejerleeLe&keâ Meyo keânueeles nQ~ ngS pÙeeW kesâ lÙeeW efnvoer ceW DeeS nQ lelmece Meyo keânueeles nQ~
DevÙe Meyo kesâ efJehejerleeLe&keâ Meyo Fme Øekeâej nQ- pewmes- keâhe&š, Deeceü, heg<he, ceÙetj, ÛelJeeefj, Kehe&j Deeefo~
Meyo efJehejerleeLe&keâ 6. FveceW mes ‘Kesuevee’ Meyo mes yevee efJeMes<eCe nw -
DevegJe&j GJe&j (a) oÙeeueg (b) ke=âheeueg
DeeefJeYee&Je eflejesYeeJe (c) efKeueeÌ[er (d) Oeeefce&skeâ

Je=ef° DeveeJe=ef° Ans. (c) : ‘Kesuevee’ Meyo mes yevee efJeMes<eCe ‘efKeueeÌ[er’ nesiee~

mekeâece efve<keâece efJeMes<eCe- pees meb%ee Ùee meJe&veece keâer efJeMes<elee yeleeS Gmes efJeMes<eCe
keânles nQ~
2. Skeâ keâer Jele&veer Megæ nw-
meeJe&veeefcekeâ efJeMes<eCe– Ùen, Jen, keâesF&, Ssmee, pewmee FlÙeeefo~
(a) DeefOekeâeefjkeâ (b) DeeefOekeâeefjkeâ
iegCeJeeÛekeâ efJeMes<eCe–veÙee, hegjevee, Pet"e, mehesâo, heeruee, Ûeew[Ì e FlÙeeefo~
(c) DeefOekeâejkeâ (d) DeOeerkeâeefjkeâ
mebKÙeeJeeÛekeâ efJeMes<eCe– Ûeej, kegâÚ, leerme, meye, leerveeW, ÛeejeW FlÙeeefo~
Ans. (b) : ‘DeeefOekeâeefjkeâ’ Meyo Jele&veer keâer Âef° mes Megæ nw~ peyeefkeâ
heefjceeCe yeesOekeâ efJeMes<eCe – oes mesj, meye Oeve, ome neLe, Ûeej iepe
DevÙe efJekeâuhe Jele&veer keâer Âef° mes DeMegæ nQ~ FlÙeeefo~
3. ‘Jen keâefJe pees lelkeâeue keâefJelee keâjs, kesâ efueS Skeâ 7. ‘Ûevõneme’ keâe heÙee&ÙeJeeÛeer Meyo nw-
Meyo nw-
(a) leerj (b) leueJeej
(a) megkeâefJe (b) jmeefmeæ keâefJe
(c) Yeeuee (d) Oeveg<e-yeeCe
(c) cenekeâefJe (d) DeeMegkeâefJe
Ans. (b) : Ûevõneme keâe heÙee&ÙeJeeÛeer Meyo leueJeej nw~ Ûevõneme kesâ
Ans. (d) : ‘Jen keâefJe pees lelkeâeue keâefJelee keâjs, kesâ efueS Skeâ Meyo DevÙe heÙee&ÙeJeeÛeer Meyo–keâjJeeue, KeÌ[ie, Deefme, keâšej, ke=âheeCe Deeefo~
‘DeeMegkeâefJe’ nesiee~ DevÙe JeekeäÙeebMe Fme Øekeâej nQ– leerj keâe heÙee&ÙeJeeÛeer Meyo– DeeMegie, F<eg, veejeÛe, Mej, MeeÙekeâ,
pees hegmlekeâeW keâer meceer#ee keâjlee nw - meceer#ekeâ yeeCe~
efpemekeâer yegefæ kegâMe kesâ De«e keâer lejn hewveer nes - kegâMee«eyegefæ Yeeuee keâe heÙee&ÙeJeeÛeer Meyo– yeÚe&, kebgâle, Meueekeâe~
pees efkeâmeer keâer Deesj mes nw - ØeefleefveefOe 8. efvecveefueefKele ceW mes Skeâ Meyo efJeMes<eCe veneR nw-
4. FveceW mes ‘JemegOee’ keâe heÙee&ÙeJeeÛeer Meyo nw - (a) Deketâle (b) DeÛeej
(a) efJeheguee (b) MeJe&jer (c) DekeâCe& (d) DeÛeb[
(c) Oesvegkeâe (d) heÙeefmJeveer
Ans. (b) : ‘DeÛeej’ efJeMes<eCe Meyo veneR nw~ peyeefkeâ Deketâle, DekeâCe&,
Ans. (a) : ‘JemegOee’ keâe heÙee&ÙeJeeÛeer Meyo efJeheguee nw~ DeÛeb[ efJeMes<eCe Meyo nw~
JemegOee kesâ DevÙe heÙee&ÙeJeeÛeer Meyo – Yet, Fuee, Yetefce, Oejleer, Oeefj$eer, efJeMes<eCe- pees meb%ee Ùee meJe&veece keâer efJeMes<elee yeleeS efJeMes<eCe Meyo
OejCeer, JemegbOeje nw~ keânueelee nw~
5. ‘keâhe&š’ keâe leodYeJe ™he nw - efJeMes<eCe Ûeej Øekeâej kessâ nesles nQ-
(a) keâheš (b) keâejhesš (1) meeJe&veeefcekeâ efJeMes<eCe (2) iegCeJeeÛekeâ efJeMes<eCe (3) heefjceeCe
(c) keâhetj (d) keâheÌ[e JeeÛekeâ efJeMes<eCe (4) mebKÙeeJeeÛekeâ efJeMes<eCe~
UP RO/ARO (Pre) Hindi 2021 37 YCT
CLICK HERE FOR FREE MATERIAL

9. ‘meew iegvee uecyee’ ceW efJeMes<eCe keâe keâewve mee Yeso nw? Ans. (c) : GòejeefOekeâej ceW Øeehle mecheefòe kesâ efueS Skeâ Meyo efjkeäLe nesiee~
(a) ieCeveeJeeÛekeâ (b) ›eâceJeeÛekeâ Jen cetuÙeJeeve Jemleg efpeme hej efkeâmeer keâe DeefOekeâej nes - heefjmecheefòe
(c) DeeJe=efòeJeeÛekeâ (d) mechetCe&leeJeeÛekeâ efkeâmeer kesâ heeme jKeer ngF& otmejs keâer Jemleg Ùee õJÙe - Oejesnj
Ans. (c) : ‘meew iegvee uecyee’ ceW DeeJe=efòeJeeÛekeâ efJeMes<eCe nw~ 14. ‘Ûeesj’ keâe heÙee&ÙeJeeÛeer Meyo nw-
efJeMes<eCe- pees meb%ee Ùee meJe&veece keâer efJeMes<elee keâe yeesOe keâjeles nQ (a) Kevekeâ (b) Gokeâ
efJeMes<eCe Meyo keânueeles nQ~ (c) Oetmej (d) LeueÛej
efJeMes<eCe kesâ Ûeej Yeso nQ- (1) meeJe&veeefcekeâ (2) iegCeJeeÛekeâ (3)
heefjceeCe JeeÛekeâ (4) mebKÙeeJeeÛekeâ~ Ans. (a) : ‘Ûeesj’ keâe heÙee&ÙeJeeÛeer Meyo ‘Kevekeâ’ nw~
mebKÙeeJeeÛekeâ efJeMes<eCe- efpeme efJeMes<eCe mes meb%ee keâer mebKÙee keâe Kevekeâ– Ûeesj, omÙeg, jpeveerÛej, Kevekeâ, cees<ekeâ, lemkeâj, kegâefcYeue, kebâefYepe
yeesOe nes mebKÙeeJeeÛekeâ efJeMes<eCe keânueeles nQ~ (keâ) ieCeveeJeeÛekeâ (Ke) Gokeâ – heeveer, veerj, meefueue, Decyeg, leesÙe, heÙe, Dece=le, meejbie~
›eâceJeeÛekeâ (ie) DeeJe=efòeJeeÛekeâ (Ie) mecegoeÙe JeeÛekeâ (Ì[) ØelÙeskeâ Oetmej – ieone, Kej, ieo&Ye, JewMeeKevevove, jemeYe, Ûe›eâerJeeve, ieOee~
yeesOekeâ 15. ‘heeC[g’ Meyo efJeMes<eCe keâer Âef° mes nw -
DeeJe=efòe JeeÛekeâ efJeMes<eCe :- pees mebKÙee JeeÛekeâ efJeMes<eCe efkeâmeer (a) kesâJeue efJeMes<eCe
mebKÙee keâer DeeJe=efòe keâes metefÛele keâjlee nw~ DeeJe=efòe JeeÛekeâ efJeMes<eCe (b) kesâJeue efJeMes<Ùe
keânueelee nw~ pewmes- Ûeej iegvee, oesiegvee, leerve iegvee, meew iegvee Deeefo~ (c) efJeMes<eCe Deewj efJeMes<Ùe oesveeW
10. ‘meye kegâÚ peeveves Jeeues’ kesâ efueS Skeâ Meyo nw- (d) GheÙeg&òeâ ceW mes Skeâ Yeer veneR
(a) %eeveer (b) cene%eeveer
Ans. (c) : ‘heeC[g’ Meyo efJeMes<eCe Deewj efJeMes<Ùe oesveeW nQ~
(c) meJe&%e (d) peevekeâej
efJeMes<eCe– meb%ee DeLeJee meJe&veece keâer efJeMes<elee yeleeves Jeeues Meyo
Ans. (c) : meye kegâÚ peeveves Jeeues kesâ efueS Skeâ Meyo ‘meJe&%e nesiee~
efJeMes<eCe keânueeles nQ~ pewmes – keâeuee, ceesše, megvoj~
DevÙe JeekeäÙeebMe SJeb Meyo Fme Øekeâej nQ–
efJeMes<Ùe– efpemekeâer efJeMes<elee yeleueeÙeer peeÙes, Jen ‘efJeMes<Ùe’ keânueelee
pees meyekeâes meceeve YeeJe mes osKelee nes – meceoMeea
nw~ pewmes, jece, ieeÙe, IeesÌ[e~
pees De#ejeW keâes heÌ{vee-efueKevee peevelee nes – mee#ej
pees Deheves Deehe mes GlheVe ngDee nes – mJeÙebYet 16. efvecveefueefKele ceW Skeâ Jele&veer Megæ nw-
pees meye keâece Deheves Yejesmes keâjlee nw - mJeeJeuebyeer (a) veghetj (b) mceCe&
11. FveceW mes ‘ØeLece’ keâe GheÙegòeâ efJehejerleeLe&keâ Meyo nesiee- (c) keâJeefÙe$eer (d) heefjefmLeleer
(a) ØeOeeve (b) ieewCe Ans. (c) : keâJeefÙe$eer Meyo Jele&veer keâer Âef° mes Megæ nw~
(c) Debeflece (d) ØelÙe#e DeMegæ Meyo Megæ Meyo
Ans. (c) : ‘ØeLece’ keâe GheÙegòeâ efJehejerleeLe& Meyo ‘Debeflece’ nesiee~ veghetj vethegj
Meyo efJeueesce mceCe& mcejCe
ØeOeeve ieewCe heefjefmLeleer heefjefmLeefle
ØeLece Debeflece 17. Jele&veer keâer Âef° mes Megæ Meyo nw-
ØelÙe#e DeØelÙe#e (a) heefjÛÚe (b) hejerÛÚe
ØeJej DeJej (c) hejer#ee (d) heefj#ee
12. efvecveefueefKele ceW Skeâ JeekeäÙe pees Megæ nw, Jen nw- Ans. (c) : ‘hejer#ee’ Meyo Jele&veer keâer Âef° mes Megæ nw-
(a) Gmeves cegòeâkeâC" mes yeÌ[eF& keâer~ DeMegæ Meyo Megæ Meyo
(b) Gmeves cegòeânmle Oeve uegšeÙee~ ØelÙegled - ØelÙegle
(c) mecemle ØeeefCecee$e keâe keâuÙeeCe keâjes~ YeeiÙeJeeve - YeeiÙeJeeved
(d) Deehekeâer DeeÙeg Ûeeueerme Je<e& nw~ efJeefOeJele - efJeefOeJeled
Ans. (b) : efvecveefueefKele ceW Megæ JeekeäÙe ‘Gmeves cegòeânmle Oeve ßeerceeve - ßeerceeved
uegšeÙee’ nesiee~ yeekeâer efJekeâuhe JeekeäÙe keâer Âef° mes DeMegæ nw~
18. ‘efyevee heuekeâ PehekeâeS’ kesâ efueS Skeâ Meyo nesiee-
DevÙe JeekeäÙeeW kesâ Megæ ™he efvecveefueefKele nw-
(a) Ûeefkeâle (b) osKeles jnvee
Gmeves cegòeâkeâC" ØeMebmee keâer~
ØeeCeer cee$e keâe keâuÙeeCe keâjes~ (c) efJeheuekeâ (d) efveefve&ces<e
Deehekeâer DeJemLee Ûeeueerme Je<e& nw~ Ans. (d) : ‘efyevee heuekeâ PehekeâeS’ kesâ efueS Skeâ Meyo ‘efveefve&ces<e’
13. ‘GòejeefOekeâej ceW Øeehle mecheefòe’ – Fve Deveskeâ MeyoeW kesâ nesiee~
efueS Skeâ Meyo nw - leke&â kesâ Éeje pees meccele nw – leke&âmeccele
(a) Oejesnj (b) GòejeefOeke=âle efpemes veneR peerlee pee mekesâ – DepesÙe
(c) efjkeäLe (d) heefjmecheefòe pees DevegkeâjCe keâjves ÙeesiÙe nes - DevegkeâjCeerÙe
UP RO/ARO (Pre) Hindi 2021 38 YCT
CLICK HERE FOR FREE MATERIAL

19. ‘Devegjefòeâ’ keâe efJehejerleeLe&keâ Meyo nw- 24. FveceW mes efJeueesce MeyoeW keâe Skeâ mener Ùegice nw-
(a) Deemeefòeâ (b) ØeMeefmle (a) Deeheefòe - efJeheefòe (b) DeJe<e&Ce – DeveeJe<e&Ce
(c) efJejefòeâ (d) Øeke=âefle (c) ieCelev$e – pevelev$e (d) DeeÂle – eflejmke=âle
Ans. (c) : ‘Devegjefòeâ’ keâe efJehejerleeLe&keâ Meyo ‘efJejefòeâ’ nesiee~ Ans. (d) : DeeÂle – eflejmke=âle, efJeueesce MeyoeW keâe Skeâ mener
Meyo efJeueesce Ùegice nw~
Meyo efJeueesce
Deemeefòeâ Deveemeefòeâ
Deeheefòe - mebheefòe
Øeke=âefle efJeke=âefle
DeJe<e&Ce - Je<e&Ce
Deveskeâlee Skeâlee ieCeleb$e - jepeleb$e
Dehekeâej Ghekeâej Œeer - heg®<e
20. efvecveebefkeâle ceW Skeâ JeekeäÙe pees Megæ nw, Jen nw- iegCe - DeJeiegCe
(a) ceQ DeveskeâeW yeej efJeosMe ieÙee~ 25. efvecveefueefKele ceW ‘nmleer’ efkeâme Meyo keâe lelmece ™he nw?
(b) Fme nerjs keâe cetuÙe veehee veneR pee mekeâlee~ (a) neLe (b) nej
(c) efyevee efškeâš Ùee$ee oC[veerÙe nw~ (c) neLeer (d) nBmeer
(d) Deehe kesâJeue Flevee ner keâece keâj oerefpeS~ Ans. (c) : ‘nmleer’ neLeer Meyo keâe lelmece ™he nw~
Ans. (c) : efyevee efškeâš Ùee$ee oC[veerÙe nw~ Ùen JeekeäÙe Megæ nw~ lelmece leÆJe
DeMegæ- ceQ DeveskeâeW yeej efJeosMe ieÙee~ nmle - neLe
Megæ – ceQ Deveskeâ yeej efJeosMe ieÙee~ neefj - nej
DeMegæ – Fme nerjs keâe cetuÙe veehee veneR pee mekeâlee~ nemÙe - nBmeer
Megæ – Fme nerjs keâe cetuÙe DeeBkeâe veneR pee mekeâlee~ 26. ‘DeeOegefvekeâ’ keâe efJeueesce nw -
DeMegæ – Deehe kesâJeue Flevee ner keâece keâj oerefpeS~ (a) meceerÛeerve (b) DeJee&Ûeerve
Megæ – Deehe Flevee ner keâece keâj oerefpeS~ (c) ØeeÛeerve (d) mecemeeceefÙekeâ
21. ‘pees F&Õej’ ceW efJeÕeeme jKelee nes’ kesâ efueS Skeâ Meyo nw- Ans. (c) : DeeOegefvekeâ keâe efJeueesce ‘ØeeÛeerve’ nw peyeefkeâ ØeeÛeerve keâe
(a) DeemLeeJeeve (b) Deeefmlekeâ efJeueesce veJeerve, DeJee&Ûeerve leLee DeeOegefvekeâ neslee nw, leLee ‘meceerÛeerve’ keâe
(c) efJeÕeemeer (d) ceeveme efJeueesce Meyo ‘DemeceerÛeerve’ nesiee~
Ans. (b) : ‘pees F&Õej ceW efJeÕeeme jKelee nes’ Fmekesâ efueS Skeâ Meyo 27. ‘peess Ùegæ ceW efmLej jnlee nw’ Fme Deveskeâ MeyoeW kesâ efueS
Deeefmlekeâ nesiee~ Skeâ Meyo nw-
DeemLeeJeeve – DeemLee jKeves Jeeuee (a) Ùegæ efmLele (b) Ùegæ efmLejer
efJeÕeemeer – efJeÕeeme efkeâÙes peeves ÙeesiÙe (c) ÙegefOeef‰j (d) Ùegæ Øesceer
melÙee«en – pees Dee«en melÙe nes Ans. (c) : ‘pees Ùegæ ceW efmLej jnlee nw’ Gmekesâ efueS Skeâ Meyo
22. efvecveefueefKele ceW ‘efJeMes<Ùe’ Meyo nw- ‘ÙegefOeef‰j’ nesiee~
(a) efJeheVe (b) Jeeoer Deveskeâ MeyoeW kesâ efueS Skeâ Meyo efvecve nw-
(c) efJeefMe° (d) ceeveme pees #ecee ve efkeâÙee pee mekesâ - De#ecÙe
efpemekeâe pevce hemeerves mes ngDee nes - mJesope
Ans. (d) : ‘ceeveme’ efJeMes<Ùe Meyo nw~
efpemekeâer ieo&ve megvoj nes - meg«eerJe
efJeMes<Ùe– efpemekeâer efJeMes<elee yeleeÙeer peeÙes Jen efJeMes<Ùe keânueelee nw~
pewmes- jece, ieeÙe, IeesÌ[e~ 28. ‘peerYe’ keâe heÙee&ÙeJeeÛeer nw-
efJeMes<eCe– meb%ee DeLeJee meJe&veece keâer efJeMes<elee yeleeves Jeeues Meyo (a) JeÛeve (b) jmevee
efJeMes<eCe keânueeles nQ~ pewmes- keâeuee, ceesše, megvoj~ (c) OJeefve (d) peerJe
23. ‘Deveg«en’ keâe GheÙegòeâ’ efJehejerleeLe&keâ Meyo nesiee- Ans. (b) : ‘peerYe’ keâe heÙee&ÙeJeeÛeer Meyo ‘jmevee’ nw~ peerYe kesâ DevÙe
(a) efJe«en (b) Dee«en
heÙee&ÙeJeeÛeer Meyo nQ- efpe£e, jme%ee, jefmekeâe, peyeeve Deeefo~
‘peerJe’ keâe heÙee&ÙeJeeÛeer Meyo – ™n, ØeeCe, Deelcee, peerJeelcee~
(c) heefj«en (d) meb«en
‘OJeefve’ keâe heÙee&ÙeJeeÛeer Meyo – veeo, jJe, mJej, leeue, DeeJeepe~
Ans. (a) : ‘Deveg«en’ keâe efJehejerleeLe&keâ Meyo ‘efJe«en’ nesiee~
‘JeÛeve’ keâe heÙee&ÙeJeeÛeer Meyo – DeeMJeemeve, Jeeoe, ØeCe, Øeefle%ee~
Meyo - efJeueesce
29. ‘Øeleehe efmebn keâe IeesÌ[e keâeuee nw~’ – FveceW ‘keâeuee’ Meyo
Dee«en - Devee«en efJeMes<eCe keâer Âef° mes nw -
heefj«en - Deheefj«en (a) meeJe&veeefcekeâ (b) ›eâceyeesOekeâ
meb«en - efJe«en (c) efJeOesÙe efJeMes<eCe (d) efJeMes<Ùe efJeMes<eCe
UP RO/ARO (Pre) Hindi 2021 39 YCT
CLICK HERE FOR FREE MATERIAL

Ans. (c) : ‘Øeleehe efmebn keâe IeesÌ[e keâeuee nw’ JeekeäÙe ceW ‘keâeuee’ Meyo Ans. (d) : ‘ÛeewLee’ keâe lelmece Meyo ‘ÛelegLe&’ nw~
efJeMes<eCe keâer Âef° mes ‘efJeOesÙe efJeMes<eCe’ nw~ lelmece – mebmke=âle Yee<ee kesâ Jes Meyo pees efnvoer Yee<ee ceW pÙeeW keâe lÙeeW
efJeOesÙe efJeMes<eCe – pees efJeMes<eCe efJeMes<Ùe Deewj ef›eâÙee kesâ yeerÛe DeeÙes, ØeÙegòeâ nesles nQ, GvnW lelmece Meyo keânles nQ~
JeneB efJeOesÙe efJeMes<eCe neslee nw~ pewmes- cesje ueÌ[keâe Deeuemeer nw~ 35. ‘IeesÌ[e’ keâe GheÙegòeâ lelmece Meyo nw -
efJeMes<Ùe efJeMes<eCe – pees efJeMes<eCe, efJeMes<Ùe mes henues DeeÙes, JeneB (a) DeÕe (b) Ieesškeâ
efJeMes<Ùe efJeMes<eCe neslee nw, pewmes:- Deveeefcekeâe megvoj ueÌ[keâer nw~ (c) legjbie (d) yeepeer
30. ‘ØeÙeeiejepe’ ceW omeJeeB JÙeefòeâ keâesjesvee heerefÌ[le nw~’ ceW Ans. (b) : ‘IeesÌ[e’ keâe GheÙegòeâ lelmece Meyo ‘Ieesškeâ’ nw~ peyeefkeâ
omeJeeB Meyo nw - DeÕe, yeeefpe leLee legjbie Iees[Ì e kesâ heÙee&ÙeJeeÛeer Meyo nQ~
(a) ieCeveeJeeÛekeâ efJeMes<eCe (b) ›eâceJeeÛekeâ efJeMes<eCe 36. ‘DeeJele&keâ’ keâe efJehejerleeLe&keâ Meyo nw -
(c) DeeJe=efòeJeeÛekeâ efJeMes<eCe (d) mecegoeÙeJeeÛekeâ efJeMes<eCe (a) ØeJele&keâ (b) DeveeJele&keâ
Ans. (b) : ØeÙeeiejepe ceW omeJeeB JÙeefòeâ keâesjesvee heerefÌ[le nw~ ceW omeJeeB (c) DeeJe=le (d) Øekeâ<e&keâ
Meyo ›eâceJeeÛekeâ efJeMes<eCe nw~ Ans. (b) : ‘DeeJele&keâ’ keâe efJehejerleeLe&keâ Meyo ‘DeveeJele&keâ’ nw~ efvecve
ØeÙeesie kesâ Devegmeej efveefMÛele mebKÙeeJeeÛekeâ efJeMes<eCe kesâ efvecve Øekeâej nQ- MeyoeW kesâ efJehejerleeLe&keâ Meyo –
ieCeveeJeeÛekeâ efJeMes<eCe – Skeâ, oes, leerve Meyo - efJehejerleeLe&keâ Meyo
ØeJele&keâ - DeØeJele&keâ
›eâceJeeÛekeâ efJeMes<eCe – henuee, otmeje, leermeje
DeeJe=le - DeveeJe=le
DeeJe=efòeJeeÛekeâ efJeMes<eCe – otvee, efleiegvee, Ûeewiegvee
Øekeâ<e& - Dehekeâ<e&
mecegoeÙeJeeÛekeâ efJeMes<eCe – oesveeW, leerveeW, ÛeejeW
De%e - efJe%e
ØelÙeskeâyeesOekeâ efJeMes<eCe – ØelÙeskeâ, nj-Skeâ, oes-oes
37. ‘pees efkeâmeer efJe<eÙe keâe %eelee nes’ kesâ efueS Skeâ Meyo nw -
31. ‘efleefcej’ keâe heÙee&ÙeJeeÛeer nw -
(a) DeefYe%e (b) ÛewlevÙe
(a) ØekeâeMe (b) jeef$e (c) efJeefMe° (d) efJeMes<e%e
(c) metÙe& (d) DevOekeâej Ans. (d) : ‘pees efkeâmeer efJe<eÙe keâe %eelee nes’ kesâ efueS Skeâ Meyo
Ans. (d) : ‘efleefcej’ keâe heÙee&ÙeJeeÛeer Meyo ‘DevOekeâej’ nw~ ‘efJeMes<e%e’ nw~
efleefcej kesâ DevÙe heÙee&ÙeJeeÛeer Meyo nQ – lece, DebOesje, leefcem$e~ 38. ‘efyepeueer’ keâe heÙee&Ùe nw -
jeef$e kesâ heÙee&ÙeJeeÛeer Meyo – jele, efveMee, efJeYeeJejer, efveMeerLe, (a) MeJe&jer (b) leveÙee
Ùeeefceveer, MeJe&jer~ (c) jceCeer (d) oeefceveer
metÙe& keâe heÙee&ÙeJeeÛeer Meyo – metjpe, jefJe, efovekeâj, efoJeekeâj, Ans. (d) : ‘efyepeueer’ keâe heÙee&Ùe oeefceveer nw~
ØeYeekeâj, DebMegceeueer, Yeemkeâj~ efyepeueer kesâ DevÙe heÙee&Ùe - Ûeheuee, ÛebÛeuee, leefÌ[led, meewoeceveer,
ØekeâeMe keâe heÙee&ÙeJeeÛeer Meyo – ØeYee, pÙeesefle, Gpeeuee, jesMeveer~ leveÙee keâe heÙee&ÙeJeeÛeer – Deelcepee, meglee, ogefnlee, heg$eer~
32. ‘ngleeMeve’ heÙee&ÙeJeeÛeer nw- MeJe&jer keâe heÙee&ÙeJeeÛeer – jeef$e, jele, jpeveer, efveMeerLe, #eCeoe~
(a) efveOe&ve (b) Deefive jceCeer keâe heÙee&ÙeJeeÛeer – megvojer, veejer, Œeer, keâevlee, Jeefvelee, Deyeuee,
(c) DeieefCele (d) DeKeb[ keâeefceveer~
Ans. (b) : ‘ngleeMeve’ keâe heÙee&ÙeJeeÛeer Meyo ‘Deefive’ nw~ 39. ‘DeheCee&’ Meyo keâe heÙee&ÙeJeeÛeer nw -
Deefive kesâ DevÙe heÙee&ÙeJeeÛeer Meyo – Deeie, heeJekeâ, onve, Deveue~ (a) Deewjle (b) osJeebievee
(c) DeejeOevee (d) heeJe&leer
‘efveOe&ve’ keâe heÙee&ÙeJeeÛeer Meyo nw – iejerye, efJeheVe~
‘DeKeC[’ keâe heÙee&ÙeJeeÛeer Meyo nw – De#eÙe, De#egCCe, DeYebie~ Ans. (d) : DeheCee& Meyo keâe heÙee&ÙeJeeÛeer ‘heeJe&leer’ nesiee~ DeheCee& kesâ
DevÙe heÙee&ÙeJeeÛeer Meyo nQ- ieewjer, efieefjpee, heeJe&leer, YeJeeveer, ®õeCeer~
33. efvecveefueefKele ceW mes efkeâme Meyo keâer Jele&veer DeMegæ nw -
efoÙes ieÙes DevÙe efJekeâuheeW kesâ heÙee&ÙeJeeÛeer Meyo efvecve nw –
(a) Ùee%eJeukeâ (b) meÂMe
Meyo heÙee&ÙeJeeÛeer Meyo
(c) nmle#eshe (d) ie=efnCeer Œeer - Œeer, Jeefvelee, keâevlee, jceCeer, veejer, Deyeuee
Ans. (a) : ‘Ùee%eJeukeâ’ Meyo keâer Jele&veer DeMegæ nw~ Fmekeâer Megæ DeejeOevee - ØeeLe&vee, hetpee, DeÛe&vee
Jele&veer ‘Ùee%eJeukeäÙe’ nw~ Mes<e MeyoeW meÂMe, nmle#eshe, ie=efnCeer keâer osJeebievee - cesvekeâe, osJeyeeuee, Dehmeje
Jele&veer Megæ nw~ 40. efvecveefueefKele efJekeâuheeW ceW mes ‘ÛebÛejerkeâ’ Meyo keâe
34. ‘ÛeewLee’ keâe lelmece Meyo nw - heÙee&ÙeJeeÛeer ÛegefveS -
(a) Ûeleg<heo (b) Ûeleg<keâ (a) nJee (b) Yeücej
(c) Ûeleg<keâe" (d) ÛelegLe& (c) efce$e (d) heg$e

UP RO/ARO (Pre) Hindi 2021 40 YCT


CLICK HERE FOR FREE MATERIAL

Ans. (b) : ÛebÛejerkeâ Meyo keâe heÙee&ÙeJeeÛeer Yeücej nesiee~ Ans. (b) : meoeÛeej keâe efJeueesce Meyo ‘keâoeÛeej’ nesiee~
Yeücej Meyo kesâ DevÙe heÙee&ÙeJeeÛeer Meyo nQ – YeeQje, uebheš, ceOeghe, efoÙes ieÙes efJekeâuheeW kesâ efJeueesce Meyo efvecve nQ–
keâecegkeâ, Deefue, <ešdheo, ceOegkeâj, efÉjsHeâ Meyo efJeueesce Meyo
efoÙes ieÙes DevÙe efJekeâuheeW kesâ DevÙe heÙee&ÙeJeeÛeer Meyo efvecve nQ – DeeÛeej - DeveeÛeej
nJee – heJeve, Deefveue, meceerj, JeeÙeg, yeÙeej DehejeOe - efvejhejeOe
efce$e – meKee, oesmle, menÛej, megùo, mebieer DevÙeeÙe - vÙeeÙe
heg$e – hetle, leveÙe, ueeue, Úesje, Deelcepe 46. efvecveefueefKele ceW mes leÆJe Meyo nw -
41. ‘ieesOetce’ keâe leodYeJe Meyo nw- (a) Debievee (b) ieÙebo
(a) ieesle (b) ieeso (c) Debieer (d) Jeevej
(c) iesntB (d) ieesn Ans. (b) : efoÙes ieÙes efJekeâuheeW ceW leodYeJe Meyo ‘ieÙebo’ nesiee~

Ans. (c) : ‘ieesOetce’ keâe leodYeJe Meyo iesntB nesiee~


ieÙebo Meyo keâe DeLe& iepesvõ Ùee neLeer neslee nw~ DevÙe Meyo lelmece nQ~
efpemekesâ leodYeJe Fme Øekeâej nw-
leÆJe Meyo lelmece Meyo
lelmece leodYeJe
ieesle - iees$e
Debievee - DeeBieve
ieeso - ›eâes[ Jeevej - yeboj
ieesn - ieesOee 47. ‘pees leesuee pee mekesâ’ kesâ efueS Skeâ Meyo nesiee-
42. ‘keâesF& Deeoceer DeeÙee nw’ JeekeäÙe ceW ØeÙegòeâ efJeMes<eCe nw - (a) DeheefjcesÙe (b) DevegcesÙe
(a) Deefve§eÙeJeeÛekeâ (b) efve§eÙeJeeÛekeâ (c) heefjcesÙe (d) efJeefveceÙe
(c) ØeMveJeeÛekeâ (d) mebyebOeJeeÛekeâ Ans. (c) : ‘pees leesuee pee mekesâ’ kesâ efueS Skeâ Meyo ‘heefjcesÙe’ nesiee~
Ans. (a) : ‘keâesF& Deeoceer DeeÙee nw’ JeekeäÙe ceW Deefve§eÙeJeeÛekeâ DeheefjcesÙe - pees leesuee ve pee mekesâ
efJeMes<eCe nesiee~ DevegcesÙe - Devegceeve keâjves ÙeesiÙe
Deefve§eÙeJeeÛekeâ efJeMes<eCe– efpeve MeyoeW mes Deefveef§elelee keâe yeesOe efJeefveceÙe - efkeâmeer Jemleg kesâ yeoues efkeâmeer otmejer
neslee nw, GvnW Deefve§eÙeJeeÛekeâ efJeMes<eCe keânles nQ~ Jemleg keâe Deeoeve-Øeoeve keâjvee~
pewmes :- keâue mketâue ceW ueieYeie Ûeeueerme efJeÅeeLeea GheefmLele Les~ 48. ‘keâce yeesueves Jeeuee’ JeekeäÙeebMe kesâ efueS Skeâ Meyo nw -
43. ‘ieewjJe keâe efJehejerleeLe&keâ Meyo nw - (a) efceleYee<eer (b) ce=ogYee<eer
(a) JewYeJe (b) ueeIeJe (c) yengYee<eer (d) DeYee<eer
(c) hejeYeJe (d) efveceexkeâ Ans. (a) : ‘keâce yeesueves Jeeuee’ JeekeäÙeebMe kesâ efueS Skeâ Meyo
Ans. (b) : ‘ieewjJe’ keâe efJehejerleeLe&keâ Meyo ‘ueeIeJe’ nesiee~ ‘efceleYee<eer’ nesiee~ DevÙe JeekeäÙeebMe Fme Øekeâej nQ–
efoÙes ieÙes efJekeâuheeW kesâ efJeueesce Meyo efvecve nQ- yengYee<eer - DeefOekeâ yeesueves Jeeuee~
Meyo efJeueesce Meyo ce=ogYee<eer - ceOegj yeesueves Jeeuee~
49. efvecveebefkeâle ceW mes leodYeJe Meyo ÛegefveS -
JewYeJe owvÙe (oeefjõÙe)
(a) yeueJeeve (b) yeeBPe
hejeYeJe efJeYeJe
(c) ÛeceÛee (d) Deemeceeve
44. ‘Dece=le’ keâe efJeueesce nw
Ans. (b) : efvecveebefkeâle efJekeâuheeW ceW ‘yeeBPe’ Meyo leodYeJe nesiee~
(a) Deke&â (b) heerÙet<e
leodYeJe Meyo– mebmke=âle kesâ Jes Meyo pees cetue Meyo mes heefjJeefle&le
(c) ceOegj (d) efJe<e
neskeâj efnvoer ceW ØeÙeesie nesles nQ leodYeJe Meyo keânueeles nQ~ pewmes- Deeie,
Ans. (d) : Dece=le keâe efJeueesce efJe<e nesiee~ ceesj, veew, hetâue FlÙeeefo~
efoÙes ieÙes efJekeâuheeW kesâ efJeueesce Meyo efvecve nQ- lelmece Meyo – mebmke=âle kesâ Jes Meyo pees efnvoer ceW pÙeeW kesâ lÙeeW ØeÙeesie
Meyo efJeueesce Meyo nesles nQ lelmece keânueeles nQ~ pewmes- Deeceü, Deefive, heg<he~
ceOegj - keâšg, keâke&âMe 50. ‘efpemekeâer DeeMee ve keâer ieÙeer nes’ kesâ efueS GheÙegòeâ Skeâ
heerÙet<e - efJe<e Meyo nw -
45. ‘meoeÛeej’ keâe efJeueesce nw- (a) DeeMeeleerle (b) DeØelÙeeefMele
(a) DeeÛeej (c) DeeMeeefvJele (d) ØelÙeeMee
(b) keâoeÛeej Ans. (b) : ‘efpemekeâer DeeMee ve keâer ieÙeer nes’ JeekeäÙeebMe kesâ efueS Skeâ
(c) DehejeOe Meyo ‘DeØelÙeeefMele’ nesiee~
(d) DevÙeeÙe DeeMeeleerle – pees DeeMee mes DeefOekeâ nes~
UP RO/ARO (Pre) Hindi 2021 41 YCT
CLICK HERE FOR FREE MATERIAL

51. Megæ Jele&veer Jeeuee Meyo nw - 57. FmeceW mes Megæ Jele&veer keâe Meyo nw -
(a) DeefOeve (b) DeOeerve (a) jÛeFlee (b) DeveeefOekeâej
(c) DeeOeerve (d) DeeefOeve (c) efÛevn (d) Devegketâue
Ans. (b) : efoÙes ieÙes efJekeâuheeW ceW Megæ Jele&veer Jeeuee Meyo ‘DeOeerve’
Ans. (d) : efoÙes ieÙes MeyoeW ceW Megæ Jele&veer keâe Meyo ‘Devegketâue’ nw~
nesiee~ DevÙe efJekeâuheeW keâer Jele&veer Megæ veneR nw~
DevÙe Jele&veer kesâ Megæ Meyo Fme Øekeâej nQ–
52. efvecveefueefKele ceW mes leodYeJe Meyo nw -
DeMegæ Meyo Megæ Meyo
(a) veive (b) oef#eCe
(c) õe#ee (d) oeÌ{er jÛeFlee – jÛeefÙelee
Ans. (d) : efoÙes ieÙes MeyoeW ceW ‘oeÌ{er’ leodYeJe Meyo nw efpemekeâe lelmece DeveeefOekeâej – DeveefOekeâej
obef<š^keâe nesiee~ peyeefkeâ veive, oef#eCe leLee õe#ee lelmece Meyo nw~ cetue efÛevn – efÛeÖ
Yee<ee mebmke=âle kesâ Jes Meyo pees efnvoer Yee<ee ceW pÙeeW kesâ lÙeeW ØeÙegòeâ nesles 58. efvecveefueefKele ceW Skeâ JeekeäÙe pees Megæ nw, Jen nw -
nQ GvnW ‘lelmece’ Meyo keânles nQ~ pewmes- Deeceü, keâCe&, Dehe&Ce Deeefo~
(a) Gme pebieue ceW Øeele:keâeue keâe ÂMÙe yengle ner megneJevee
cetue Yee<ee mebmke=âle kesâ Jes Meyo efpevekeâe efnvoer ceW ™he heefjJele&ve nes ieÙee
nw GvnW ‘leÆJe’ Meyo keânles nQ~ pewmes – Deeie, Deeueme, ieenkeâ Deeefo~ neslee Lee~
53. efvecveefueefKele MeyoeW ceW mes lelmece Meyo nw - (b) yeeIe Deewj yekeâjer Skeâ Ieeš hej heeveer heerleer nQ~
(a) peesieer (b) peesJeve (c) GvneWves Fme yeele hej Deeheefòe Øekeâš keâer~
(c) peceeF& (d) ÙeewJeve (d) leceece osMe Yej ceW Ùen yeele hewâue ieÙeer~
Ans. (d) : efoÙes ieÙes MeyoeW ceW ‘ÙeewJeve’ lelmece Meyo nw~ peyeefkeâ Ans. (a) : efoÙes ieÙes JeekeäÙeeW ceW mes Megæ JeekeäÙe nw ‘Gme pebieue ceW
peesieer, peesyeve leLee peceeF& leÆJe Meyo nw~ mebmke=âle kesâ kegâÚ Meyo Ssmes Øeele:keâeue keâe ÂMÙe yengle ner megneJevee neslee Lee~’
nesles nQ pees efnvoer ceW Yeer efyevee heefjJele&ve kesâ ØeÙegòeâ nesles nQ, Gve MeyoeW DeMegæ - yeeIe Deewj yekeâjer Skeâ Ieeš hej heeveer heerleer nQ~
keâes ‘lelmece’ keânles nQ~ pewmes – Ie=le, Ûebõ, Ûew$e Deeefo~
Megæ - yeeIe Deewj yekeâjer Skeâ Ieeš hej heeveer heerles nQ~
mebmke=âle kesâ kegâÚ Meyo Ssmes nesles nQ pees efnvoer ceW kegâÚ heefjJeefle&le neskeâj
veÙes ™he ceW ØeÙeesie efkeâÙes peeles nQ Gve MeyoeW keâes ‘leÆJe keânles nQ~ DeMegæ - GvneWves Fme yeele hej Deeheefòe Øekeâš keâer~
pewmes- Úelee, ieo&ve, ÛeeBo Deeefo~ Megæ - GvneWves Fme yeele hej Deeheefòe keâer~
54. efvecveefueefKele ceW ‘he=LJeer’ keâe heÙee&ÙeJeeÛeer Meyo veneR nw - DeMegæ - leceece osMe Yej ceW Ùen yeele hewâue ieÙeer~
(a) Fuee (b) DeÛeuee Megæ - mechetCe& osMe ceW Ùen yeele hewâue ieÙeer~
(c) DeÛeue (d) DeJeefve 59. Jele&veer keâer Âef° mes Megæ Meyo nw -
Ans. (c) : efoÙes ieÙes MeyoeW ceW ‘DeÛeue’ Meyo he=LJeer keâe heÙee&ÙeJeeÛeer (a) hetpÙeveerÙe (b) lelkeâeefuekeâ
Meyo veneR nw~ Fuee, DeÛeuee leLee DeJeefve ‘he=LJeer’ keâe heÙee&ÙeJeeÛeer nw~
(c) ØeeceeefCekeâ (d) hewef$ekeâ
‘DeÛeue’ Meyo kesâ DevÙe heÙee&ÙeJeeÛeer nQ- DeefJeÛeue, Dešue, Deef[ie,
ÂÌ{ Deeefo~ Ans. (c) : efoÙes ieÙes MeyoeW ceW mes Jele&veer keâer Âef° mes Megæ Meyo nw
55. Skeâ JeekeäÙe ceW efJeMes<eCe DeMegæ nw, Jen nw - – ØeeceeefCekeâ~
(a) megvojer ueÌ[keâer (b) megvoj ueÌ[keâer ØeeceeefCekeâ Meyo ceW cetue Meyo ‘ØeceeCe’ nw~ FmeceW Fkeâ ØelÙeÙe nw~
(c) megvoj ueÌ[keâe (d) megvoj ueÌ[kesâ DeMegæ Megæ
Ans. (a) : efoÙes ieÙes JeekeäÙe ‘megvojer ueÌ[keâer’ ceW efJeMes<eCe ‘megvojer’ hetpÙeveerÙe - hetpeveerÙe
DeMegæ nw~ Fmekeâe Megæ JeekeäÙe megvoj ueÌ[keâer nesiee~ lelkeâeefuekeâ - leelkeâeefuekeâ
efJeMes<eCe – Jes Meyo pees meb%ee Ùee meJe&veece keâer efJeMes<elee yeleeles nQ
hewef$ekeâ - hewle=keâ
efJeMes<eCe keânueeles nQ~
efJeMes<Ùe – efJeMes<eCe Meyo mes efpemekeâer efJeMes<elee yeleeÙeer peeÙes, Jen 60. efpemekeâer efJeMes<elee yeleeÙeer peeÙes, Gmes keânles nQ -
‘efJeMes<Ùe’ keânueelee nw~ (a) efJeMes<eCe (b) efJeMes<Ùe
56. efvecveefueefKele ceW ‘lelmece’ Meyo nw - (c) meJe&veece (d) efJeMes<ekeâ
(a) hebefòeâ (b) hebKe Ans. (b) : efpemekeâer efJeMes<elee yeleeÙeer peeÙes, Gmes efJeMes<Ùe keânles nQ~
(c) hebojn (d) hebiele
Jes Meyo efpeveceW meb%ee Ùee meJe&veece keâer efJeMes<elee yeleeÙeer peeÙes, Gmes
Ans. (a) : efoÙes ieÙes MeyoeW ceW ‘hebefòeâ’ lelmece Meyo nw efpemekeâe ‘efJeMes<eCe’ keânles nQ~
leodYeJe hebiele nesiee~ hebKe, hevõn leLee hebiele leÆJe Meyo nw~ cetue
Yee<ee mebmke=âle mes efueÙes ieÙes Jes Meyo efpevekeâe ØeÙeesie pÙeeW kesâ lÙeeW efnvoer efJeMes<eCe keâer Yeer efJeMes<elee yeleeves Jeeues MeyoeW keâes ‘ØeefJeMes<eCe’
ceW neslee nw Jes Meyo ‘lelmece’ keâns peeles nQ~ mebmke=âle keâer hejcheje mes keânles nQ~
heeefue, Øeeke=âle Deewj DeheYebÇMe mes neskeâj pees Meyo efnvoer ceW DeeÙes nQ, Jes Meyo pees ef›eâÙee keâer efJeMes<elee yeleeles nQ GvnW ‘ef›eâÙee efJeMes<eCe’
efpevekeâe ™he heefjJeefle&le nes ieÙee nw, ‘leodYeJe’ keânueeles nQ~ keânles nQ~
UP RO/ARO (Pre) Hindi 2021 42 YCT
CLICK HERE FOR FREE MATERIAL

UPPSC RO-ARO (Mains) Exam-2016


GENERAL STUDIES
Solved Paper [ Exam Date : 22-12-2020

1. Given below are two statements, one is labelled 11. Shreyansanatha Rhinoceros
as Assertion (A) and other as Reason (R). 12. Vasupujya Buffalo
Assertion (A) : The stone age people of the
13. Vimalanatha Boar
Vindhyas migrated to the Ganga Valley in
terminal Pleistocene period. 14. Ananatnatha Porcupine
Reason (R) : Due to climatic change, it was dry 15. Dharmanatha Vajra
phase in this period. 16. Shantinatha Deer
Choose the correct answer from the code given 17. Kunthunatha Goat
below. 18. Aranatha Fish
(a) Both (A) and (R) are true and (R) is the 19. Mallinatha Kalasha
correct explanation of (A)
20. Munisuvratanatha Tortoise
(b) Both (A) and (R) are true and (R) is not the
correct explanation of (A) 21. Naminatha Blue Water Lily
(c) (A) is true but (R) is false 22. Neminatha Shankha
(d) (A) is false but (R) is true 23. Parshvanatha Snake
Ans. (a) : The stone age people of Vindhyas migrated 24. Mahavira Lion
to the Ganga valley in terminal Pleistocene period 3. Which of the following is not correctly
because it was dry phase in this period due to climate matched?
change. Therefore, both assertion (A) and reason (R) are (a) Kamboj - Rajpur/Hatak
true and (R) is the correct explanation of (A).
(b) Ashmak - Potan/Potil
2. Consider the following 'Tirthankara' and
arrange them in chronological order: (c) Sursen - Kaushambi
1. Abhinandan (d) Koshal - Shravasti
2. Vimal Nath Ans. (c) : The correct match of Mahajanpadas with
3. Munishubratha Nath their capitals is as follows-
4. Padmaprabhu Mahajanpadas Capital
Select the correct answer from the code given 1. Anga Champa
below:
2. Magadha Rajagriha/Girivraja
(a) 1, 4, 2 and 3 (b) 3, 1, 2 and 4
(c) 4, 3, 1 and 2 (d) 4, 1, 3 and 2 3. Kashi Varanasi
Ans. (a) : In Jainism, a Tirthankara is a saviour and 4. Vajji Vaishali
spiritual teacher of the dharma. The chronological 5. Malla Kusinagar/Kushinara
order of 24 Tirthankaras with their symbols is as 6. Chedi Sothivathi
follows- 7. Vatsa Kaushambi
Tirthankara Symbol 8. Kuru Indraprastha
1. Rishabhanatha Bull 9. Panchala Ahichchhatra and Kampilya
(Adinatha) 10. Matsya Viratnagar
2. Ajitanatha Elephant 11. Shurasena Mathura
3. Sambhavanatha Horse 12. Ashmaka Podana or Potali
4. Abhinandananatha Monkey 13. Gandhara Taxila
5. Sumatinatha Goose
14. Kamboja Rajpura
6. Padmaprabha Lotus
15. Avanti Ujjayini/Mahismati
7. Suparshvanatha Swastika
16. Koshala Shravasti
8. Chandraprabha Crescent Moon
4. Which of the following King is known to have
9. Pushpadanta Crocodile sent his Ministers to suppress the cruelty
10. Shitalanatha Kalpvriksha against animal in Kashi region?
UP RO/ARO (Mains) Exam-2016 43 YCT
CLICK HERE FOR FREE MATERIAL

(a) Chalukya King Siddharaja Jayasimha (c) (A) is true but (R) is false
(b) Chalukya King Kumarapala (d) (A) is false but (R) is true
(c) Chola King Kulottunga I Ans. (a) : Marathas emerged as the strongest native
(d) Kashmirian King Jayasimha power in India after the decline of Mughal Empire So,
Ans. (b) : Kumarpala (1143 - 1172) was an Indian king assertion (A) is true. Marathas was the first ruler who
from the Chaulukya dynasty of Gujarat. He was well have a clear concept of United Indian Nation. So,
known for his patronage of Jainism and an apostle of reason (R) is the correct explanation of (A).
non violence. He closed down slaughter houses. He sent 9. Arrange the following events related to
his ministers to suppress the cruelty against animal in Chhatrapati Shivaji in chronological order and
Kashi region. select the correct answer from the code given
5. Which of the following King did send two naval below:
expenditions to Ceylon in 642 AD? 1. The victory on the fort of Chakan
(a) Rajaraj (b) Narasimha Verman I 2. The episode of Afzal Khan
(c) Kirti Verman I (d) Jayasinha I 3. Start of clash with the Mughals
Ans. (b) : Narasimha Varman I was an emperor of the 4. Attack and sack of Surat
Pallava dynasty who ruled South India from 630-668 (a) 1, 3, 2, 4 (b) 1, 2, 3, 4
AD. He rent two naval expeditions to Ceylon in 642
(c) 2, 4, 3, 1 (d) 2, 3, 1, 4
AD.
Ans. (a) : In 1648 Chhatrapati Shivaji got control over
6. Last expedition of Sher Shah Suri was against
several forts under the Bijapur Sultanate around Pune
which of the following states?
such as Torna, Chakan and Kondana. In 1656 Shivaji
(a) Kalinjar (b) Malwa
raided Mughal territory near Ahmednagar and in
(c) Kannauj (d) Gaur Junnar. In 1659 he fought the battle of Pratapgarh and
Ans. (a) : Sher Shah Suri's last expedition was against killed Adilshahi general Afzal Khan. In 1664, the
the Rajput fort of Kalinjar in Bundelkhand in 1545 AD. wealthy Mughal trading port of Surat was attacked and
Due to injuries, he died during the siege of Kalinjar fort sacked by Shivaji. Thus option (a) is correct.
and was succeeded by his son Islam Shah (Jalal Khan).
10. Which of the following coins was not a silver
7. Consider the following events and arrange coin currency in the kingdom of Chhatrapati
them in chronological order. Shivaji
1. Battle of Kannauj (a) Rupaya (b) Lari
2. Battle of Chausa (c) Taka (d) Ruka
3. Battle of Talikota
4. Khanwa Ans. (a) : Chhatrapati Shivaji Maharaj issued coins in
two metals gold and copper. The gold coins are known
Select the correct answer from the code given
as Huns and the copper known as Shivrais. Silver coin
below.
issued by the Maratha Empire was an imitation of the
(a) 4, 2, 1, 3 (b) 4, 1, 2, 3
Mughal Silver coin. Lari, Taka, Ruka were the silver
(c) 4, 2, 3, 1 (d) 2, 4, 1, 3 coin currency in the kingdom of Shivaji.
Ans. (a) :
11. What was the real name of Zain-ul-Abidin, the
Battle - Year ruler of Kashmir?
The battle of Kannauj - 1540 AD (Sher Shah (a) Alishah (b) Hasanshah
defeated Humanyun) (c) Haidarshah (d) Shahi Khan
The battle of Chausa - 1539 AD (Sher Shah
Ans. (d) : Zain-ul-Abidin was the eighth Sultan of
defeated Humanyun)
Kashmir, who ruled for a period of fifty years (1420-
The battle of Talikota - 1565 AD 1470). His real name was Shahi Khan. By his order,
The battle of Khanwa - 1527 AD (Babur Mahabharata and Rajtarangini were translated into
defeated Rana Sanga) Persian. He had a liberal policy, promoted literature, art
8. Given below are two statements, one is labelled and architecture due to which he was called the 'Akbar
as Assertion (A) and other as Reason (R). of Kashmir'.
Assertion (A) : Marathas emerged as the 12. Who was the Commander in Chief of
strongest native power in India after the decline of Maharana Pratap's Rajput Army in the 'Battle
Mughal Empire.
of Haldighati'?
Reason (R) : Maratha were the first ruler who
(a) Ibrahim Khan Gardi (b) Hakim Khan Sur
have a clear concept of United Indian Nation.
(c) Tardi Beg (d) Mohammad Lodi
Choose the correct answer from the code given
below. Ans. (b) : The battle of Haldighati (1576) was between
(a) Both (A) and (R) are true and (R) is the Mughal forces and Maharana Pratap of Mewar. The
correct explanation of (A) Mughal forces were led by Man Singh and the
(b) Both (A) and (R) are true and (R) is not the commander in chief of Maharana Pratap's Rajput army
correct explanation of (A) was Hakim Khan Sur.
UP RO/ARO (Mains) Re-exam 2016 44 YCT
CLICK HERE FOR FREE MATERIAL

13. Consider the following events and arrange Ans. (*) :The correct match of event with year is-
them in chronological order: Event Year
1. Merger of Sikkim All Indian Women's Conference - 1927
2. Merger of Jhansi All Indian Depressed Class Association - 1930
3. Merger of Punjab Bombay Social Reform Association - 1903
4. Merger of Burma Indian National Social Conference - 1887
Select the correct answer from the code given 17. Who among the following was a founder of the
below 'United Indian Patriotic Association' with Syed
(a) 1, 2, 3, 4 (b) 3, 1, 4, 2 Ahmad Khan?
(c) 2, 4, 1, 3 (d) 1, 3, 2, 4 (a) Abdul Aziz
Ans. (b) : The chronological order of the given events (b) Mohammad-ul-Hasan
is- (c) Raja Shivaprasad
Merger of Punjab-(1849), Merger of Sikkim-1850, (d) Govind Das
Merger of Burma-1852 and Merger of Jhansi-1853. The Ans. (c) :In 1888 Sir Syed Ahmad Khan established the
correct order is 3, 1, 4, 2 Hence, option (d) is correct. United Indian Patriotic Association, which included
14. Match List-I with List-II and select correct Muslim as well as hindu members, all of whom were
answer from the code given below: opposed to the congress. Raja Shivaprasad was a
founder of this association with Syed Ahmad Khan.
List-I List-II
(Revolt/Movement) (Year) 18. Who of the following was elected as General
Secretary of 'All India Kishan Congress'
A. Tebhaga Movement 1. 1859-60 established in Lucknow?
B. Mopla Rebellion 2. 1879-80 (a) Swami Sahjanand
C. Pabna Peasant Revolt 3. 1921 (b) N.G. Ranga
D. Bengal Indigo Revolt 4. 1946-47 (c) Indu Lal Yagnik
Code: (d) Ram Manohar Lohia
A B C D Ans. (b) :The All India Kisan Congress (All India
Kisan Sabha) was an important peasant movement
(a) 1 2 3 4
formed by Sahajanand Saraswati in Lucknow in 1936.
(b) 4 2 3 1 N.G. Ranga was elected as first General Secretary of
(c) 2 3 4 1 All India Kisan Congress.
(d) 4 3 2 1
19. Which of the following pair is not correctly
Ans. (d) :The correct match is as follows- matched?
List-I - List-II Editor/Writer Newspaper/
(Revolt/Movement) (Year) Magazine/Book
Tebhaga Movement - 1946-47 (a) Aurobindo Ghosh Vande Mataram
Mopla Rebellion - 1921 (b) Gopal Krishna Gokhale Satyagrah
Pabna Peasant Revolt - 1879-80 (c) Annie Besant New India
(d) Mahatma Gandhi Young India
Bengal Indigo Revolt - 1859-60
Ans. (b) : The correct match is as follows-
15. Mohammdan Anglo-Oriental College, Aligarh
was founded in which of the following years? Editor/Writer Newspaper/Magazine/Book
(a) 1876 (b) 1891 (a) Aurobindo Ghosh - Vande Mataram
(c) 1875 (d) 1874 (b) Mahatma Gandhi - Satyagrah
(c) Annie Besant - New India
Ans. (c) : Mohammdan Anglo-Oriental College in
Aligarh is currently known as Aligarh Muslim (d) Mahatma Gandhi - Young India
University. It was established by Sir Syed Ahmed Khan 20. Who among the revolutionary leaders
in 1875. attempted to assassinate Charles Tegart the
16. Which of the following is not correctly hated Police Commissioner of Calcutta?
matched? (a) Sachindranath Sanyal
Event Year (b) Rajendra Lahiri
(a) All Indian Women's Conference 1926 (c) Chandrashekhar Azad
(b) All Indian Depressed Class 1918 (d) Gopinath Saha
Association Ans. (d) : Gopinath Saha was a Bengali activist for
(c) Bombay Social Reform 1908 Indian independence. On 12th January 1924, he
Association attempted to assassinate Charles Tegart, the then head
(d) The Indian National Social 1887 of the Detective Department of Calcutta Police. Saha
Conference was arrested and hanged on 1st March 1924.
UP RO/ARO (Mains) Re-exam 2016 45 YCT
CLICK HERE FOR FREE MATERIAL

21. With reference to the Civil Disobedience 25. Which of the following pairs is not correctly
Movement which of the following statement(s) matched?
is/are correct? City Location on the
1. Mahatma Gandhi was not punished for bank of river
violating the salt law (a) Jabalpur - Narmada
2. Madan Mohan Malviya, Devadas Gandhi
and K.M. Munsi were punished for (b) Hyderabad - Krishna
violating the salt law (c) Kota - Chambal
Select the correct answer using the code given (d) Nasik - Godavari
below:
Ans. (b) :The correct match is as follow-
Code
City - River
(a) Only 1 (b) Only 2
(c) Both 1 and 2 (d) Neither 1 nor 2
Jabalpur - Narmada
Hyderabad - Musi
Ans. (b) : The Salt March also known as Dandi
Satyagraha, was an act of Civil Disobedience Kota - Chambal
Movement led by Mahatma Gandhi. The twenty-four Nasik - Godavari
day march lasted from 12th March 1930 to 6th April 26. Black cotton soil is also known by which of the
1930. For breaking the salt law, Gandhiji was arrested following?
and sent to jail. So, statement (1) is incorrect. Madan (a) Banger (b) Khadar
Mohan Malviya, Devadas Gandhi and K.M. Munshi (c) Bhur (d) Regur
were punished for violating the salt law.
Ans. (d) : Black soil is very favorable for the
Hence, statement (2) is correct.
cultivation of cotton. It is also called black cotton soil. It
22. Consider the following events and arrange is mostly found in areas such as Gujrat, Maharashtra
them in chronological order: and Madhya Pradesh. Black soil is also called 'Regur'.
1. Dandi Yatra
27. Match List-I with List-II and select correct
2. Poona Pact answer from the code given below:
3. Communal Award
List-I List-II
4. Gandhi - Irwin Pact
(River Valley Project) (River)
Select the correct answer from the code given
below. A. Rihand Project 1. Godavari
Code B. Nagarjun Sagar Project 2. Betwa
(a) 1, 2, 3, 4 (b) 3, 2, 1, 4 C. Pochampad Project 3. Rihand
(c) 1, 3, 4, 2 (d) 1, 4, 3, 2 D. Matatila Project 4. Krishna
Ans. (d) : The correct chronological order of events is Code:
Dandi March - 12th March 1930
A B C D
Gandhi-Irwin Pact - 5th March 1931
(a) 1 4 3 2
Communal Award - 16th August 1932
(b) 3 2 4 1
Poona Pact - 24th September 1932
(c) 3 1 2 4
23. Which of the following statements about India
(d) 3 4 1 2
is incorrect?
(a) India is the 7th largest country in the world Ans. (d) : The correct match is as follows-
(b) India is the second populous country of the world List-1 List-II
(c) It's land boundary extends for around 15,200 km (River Valley Project) (River)
(d) The earliest sunrise is seen in its Mizoram State Rihand Project Rihand
Ans. (d) : Dong village in Arunachal Pradesh witness Nagarjun Sagar Project Krishna
the earliest sunrise in India. India is the 7th largest Pochampad Project Godavari
country in the world and India is the second populous Matatila Project Betwa
country of the world after china. Hence, option (d) is 28. Match List-I with List-II and select correct
correct answer to the question. answer from the code given below:
24. Which of the following is not a pass in the List-I List-II
Western Ghats Mountain ranges? Coal fields State of India
(a) Thal Ghat (b) Bhor Ghat
A. Jhilmili 1. Orissa
(c) Khyber Pass (d) Palghat
B. Karanpura 2. Madhya Pradesh
Ans. (c) : The Khyber Pass is a mountain pass in the
Pakhtunkhwa province of Pakistan which is not a pass C. Rampur Himgir 3. Chhattisgarh
of Western Ghats mountain ranges. D. Sohagpur 4. Jharkhand
UP RO/ARO (Mains) Re-exam 2016 46 YCT
CLICK HERE FOR FREE MATERIAL

Code: Ans. (c) : The correct match of List-I with List-II is as


A B C D follows-
(a) 3 4 2 1 List-I List-II
(b) 1 2 4 3 (Place) (Function)
(c) 2 1 3 4 Kakrapar - Nuclear Power Plant
(d) 3 4 1 2 Pokhran - Atom bomb blast site
Ans. (d) The correct match is - Thumba - Rocket launching center
Coal Fields States of India Sri Harikota - Satellite launching station
Jhilmili - Chhattisgarh 32. In which of the following State of India 'Koyali'
Karanpura - Jharkhand petroleum refinery is located?
Rampur Himgir - Odisha (a) Assam (b) Kerala
Sohagpur - Madhya Pradesh (c) Gujarat (d) Karnataka
29. Match List-I with List-II and select correct Ans. (c) : 'Koyali' petroleum refinery is located in
answer from the code given below: Gujrat. It is under Indian Oil Corporation (IOC). There
are 23 refineries in the country, 18 in the public sector,
List-I List-II 2 in the joint venture and 3 in the private sector.
(Coal fields) (Indian States)
33. Which of the following parts of Ganga river is
A. Rangit valley 1. Jharkhand declared as 'National Water Ways'?
B. Tandur 2. Sikkim (a) From Hardiwar to Kanpur
C. Warora 3. Telangana (b) From Kanpur to Prayagraj
D. Lalmatia 4. Maharashtra (c) From Narora to Patna
Code: (d) From Prayagraj to Haldiya
A B C D Ans. (d) : In Ganga river from Prayagraj (Uttar
Pradesh) to Haldia (West Bengal) is announced as
(a) 1 2 3 4
National water ways. It is India's first National Water
(b) 4 3 1 2 way. It is 1620 km long.
(c) 2 3 4 1
34. The time period of rotation of a geostationary
(d) 2 4 3 1 satellite is
Ans. (c) : The correct match is - (a) 12 hours (b) 24 hours
Coal Fields States of India (c) 48 hours (d) one year
Rangit Valley - Sikkim Ans. (b) : A geostationary satellite is an earth-orbiting
Tandur - Telangana satellite, placed at an altitude of approximately 36,000
Warora - Maharashtra kilometers. Its time period of rotation is 24 hours.
Lalmatia - Jharkhand 35. Which of the following gases was absent in the
30. Which of the following states of India was the atmosphere of the primitive earth?
largest producer of cement in 2018-19? (a) Mehane (b) Ammonia
(a) Andhra Pradesh (b) Rajasthan (c) Oxygen (d) Carbon dioxide
(c) Madhya Pradesh (d) Gujarat Ans. (c) : Oxygen was not present in the atmosphere at
Ans. (b) : Rajasthan was the largest producer of cement the time of origin of life. The atmosphere of the earth
in India in 2018-19. Currently Madhya Pradesh is the was not initially favorable for the development of life.
largest producer of cement in India. Hence, option (c) is correct.
31. Match List-I with List-II and select correct 36. The famous Volcanic mountain 'Karakatoa' is
answer from the code given below: situated in which of the following countries?
List-I List-II (a) Italy
(Place) (Function) (b) Indonesia
(c) United States of America
A. Kakrapar 1. Atom bomb blast site
(d) Japan
B. Pokhran 2. Rocket launching center
Ans. (b) : Krakatoa is a Caldera situated in the Sunda
C. Thumba 3. Nuclear power plant Strait between the islands of Java and Sumatra in
D. Sri Harikota 4. Satellite launching station Indonesia.
Code: 38. Which of the following industries is most
developed in the Great Lake region of North
A B C D America?
(a) 3 2 1 4 (a) Cement and Paper
(b) 3 2 4 1 (b) Film Industries
(c) 3 1 2 4 (c) Food and Chemical
(d) 4 3 2 1 (d) Steel and Engineering
UP RO/ARO (Mains) Re-exam 2016 47 YCT
CLICK HERE FOR FREE MATERIAL

Ans. (d) : The Great Lakes region of North America is 42. Match List-I with List-II and select correct
the largest freshwater system in the world. The five answer from the code given below:
Great Lakes are- Superior, Huron, Michigan, Erie and List-I List-II
Ontario. Steel and Engineering industries is most (Tribe) (Location)
developed in this region. A. Bhils 1. Nagaland
39. Given below are two statements, one is labelled B. Gond 2. Rajasthan
as Assertion (A) and other as Reason (R). C. Limboos 3. Chhattisgarh
Assertion (A) : The Pampas plain is the main
D. Konyak 4. Sikkim
region for cultivation in Argentina.
Reason (R) : The fertile soil of Pampas and Code:
temperate climate are very useful for growing A B C D
crops and grasses. (a) 2 3 4 1
Choose the correct answer from the code given (b) 3 1 4 2
below. (c) 1 2 3 4
(a) Both (A) and (R) are true and (R) is the (d) 2 3 1 4
correct explanation of (A) Ans. (a) :Population and Urbanization
(b) Both (A) and (R) are true and (R) is not the List-I - List-II
correct explanation of (A)
(Tribe) - (Location)
(c) (A) is true but (R) is false
Bhils - Rajasthan
(d) (A) is false but (R) is true
Gond - Chhattisgarh
Ans. (a) : The Pampas plain is the main region for
cultivation in Argentina. The fertile soil of pampas and Limboos - Sikkim
temperate climate are very useful for growing crops and Konyak - Nagaland
grasses. Hence, both (A) and (R) are true and (R) is the 43. Given below are two statements, one is labelled
correct explanation of (A). as Assertion (A) and other as Reason (R).
40. Which of the following countries are land Assertion (A) : Urbanization follows
locked? Industralization.
1. Armenia Reason (R) : In developing countries,
2. Montenegro urbanization is a movement in itself.
3. Botswana Choose the correct answer from the code given
below.
4. Ethiopia
(a) Both (A) and (R) are true and (R) is the
Choose the correct answer from the code given correct explanation of (A)
below
(b) Both (A) and (R) are true and (R) is not the
Code: correct explanation of (A)
(a) 1 and 3 only (b) 2 and 3 only (c) (A) is true but (R) is false
(c) 2 and 4 only (d) 1, 2 and 3 only (d) (A) is false but (R) is true
Ans. (*) : A landlocked country is a sovereign state Ans. (b) : Urbanization follows industrialization.
whose costlines lie on endorheic basins. Armenia, Industrialization creates job opportunities. People
Botswana and Ethopia are landlocked countries- Hence, migrate to industrial hubs to get those jobs. When more
the correct code is (1), (3). and (4) which is not given in people start living in an area, it requires better facilities
the options. there. This process leads to urbanisation. In developing
41. Which of the following is not correctly countries, urbanization is a movement in itself. So, both
matched? (A) and (R) are true, but (R) is not the correct
explanation of (A).
(City) (Location on the bank of
river) 37. Under which Article of the Indian Constitution
there is provision for the High Courts for
(a) Bonn Rhine river
Union Territories?
(b) Cairo Nile river
(a) Article 240 (b) Article 241
(c) New York Hudson river (c) Article 242 (d) Article 239
(d) Vienna Volga river
Ans. (b) : Articles and Provisions-
Ans. (d) : The correct match is as follows- Article 240- Power of President to make regulations for
City Location on the bank of river certain Union Territories.
Bonn - Rhine river Article 241- High Courts for Union territories.
Cario - Nile river Article 242-Rep. by 17th Amendment to the
New York - Hudson river constitution.
Vienna - Danube river Article 239- Administration of Union territories.
UP RO/ARO (Mains) Re-exam 2016 48 YCT
CLICK HERE FOR FREE MATERIAL

44. Given below are two statements, one is labelled 46. The Governor of a State can promulgate
as Assertion (A) and other as Reason (R). ordinance during recess of Legislature under
Assertion (A) : The Council of States is not which of the following Articles of the
subject to dissolution. Constitution?
(a) Article 123 (b) Article 213
Reason (R) : (As nearly as possible) 1/3 of its
(c) Article 220 (d) Article 220
members retired on the expiration of every second
year. Ans. (b) : Article 213 of Indian Constitution mentions
the power of Governor to promulgate Ordinances
Choose the correct answer from the code given during recess of Legislature.
below. Article 123- Power of President promulgate ordinances
(a) Both (A) and (R) are true and (R) is the during recess of parliament.
correct explanation of (A) Article 219- Oath or affirmation by judges of High courts.
(b) Both (A) and (R) are true and (R) is not the Article 220- Restriction on Practice after being a
correct explanation of (A) permanent judge.
(c) (A) is true but (R) is false 47. NITI Aayog was established in India by
(d) (A) is false but (R) is true (a) Ordiance issued by President
Ans. (b) : Under the article 83 of Indian Constitution (b) The Act of Parliament
the Council of States shall not be subject to dissolution,. (c) Union Cabinet passing a special resolution
But as nearly as possible one-third of the members (d) None of the above
retire on the expiration of every second year in the Ans. (c) :NITI Aayog was established in the place of
accordance with the provisions made in that behalf by planning commission in January 2015. It is a non-
parliament. constitutional body. It was established by passing a
So both (A) and (R) are true. But (R) is not the correct special resolution by Union Cabinet.
explanation of (A). 48. Match List-I with List-II and select correct
45. Consider the following statements. Which of answer from the code given below:
these statements is/are correct? List-I List-II
1. Like the election of the President, the (Act) (Amendment)
election of Vice-President is indirect A. The Constitution 1. Amendment
2. The member of the State Legislature play (Ninety Fourth in Article
an important role in the election of the Amendment) Act, 2006 164
both. B. The Constitution 2. Amendment
Select the correct answer from the code given (Ninety Fifth in Article
below. Amendment) Act, 2009 334
Code: C. The Constitution 3. Amendment
(a) Only 1 (b) Only 2 (Ninety Sixth in Eight
Amendment) Act, 2011 Schedule
(c) Both 1 and 2 (d) Neither 1 nor 2
D. The Constitution (One 4. Amendment
Ans. (a) :Article 52 of Indian Constitution mentions hundred third in Article 15
that there shall be a President of India and article 63 Amendment) Act, 2019
mentions that there shall be a vice president of India.
Code:
Both the President and vice president are elected
indirectly by the single-transferable voting system. A B C D
(a) 1 2 3 4
Electoral College - Electoral College of (b) 1 2 4 3
of President Vice President (c) 1 3 4 2
Elected Members of - Total members (Elected (d) 1 4 3 2
Parliament Nominated) of Ans. (a) : The correct match is as follows-
(Lok Sabha + Rajya Parliament (Lok Sabha
Sabha) + Rajya Sabha)
List-I (Act) - List-II(Amendment )
The Constitution - Amendment in Article -
Elected members of (Ninety Fourth 164
Legislative Amendment) Act, 2006
Assemblies of the
The Constitution - Amendment in Article -
states (Ninety Fifth 334
Elected member of Amendment) Act, 2009
Legislative The Constitution - Amendment in Eight
assemblies of union (Ninety Sixth Schedule
territories of Delhi, Amendment) Act, 2011
Jammu and The Constitution (One - Amendment in Article -
Kashmir and hundred third 15
Puducherry Amendment) Act, 2019
UP RO/ARO (Mains) Re-exam 2016 49 YCT
CLICK HERE FOR FREE MATERIAL

49. Indian Economy is 54. Inflation is beneficial to which of the following


(a) Mixed economy section of economy?
(b) Socialist economy (a) Creditors
(c) Capitalist economy (b) Investors in bonds and securities
(d) Gandhian socialist economy (c) Debtors
Ans. (a) : Indian Economy is a mixed economy. A Mixed (d) Consumers
economy combines the characteristics of capitalism and
socialism. In this both the government and the private Ans. (c) : Inflation is a decrease in the purchasing
sector exercise control over the economy. power of money, reflected in the general increase in the
50. Which of the following comes/come under the prices of goods and services in an economy. It brings
definition of Adjusted Gross Revenue (AGR)? most benefits to debtors because debtors repay their
1. Interest Income loans with the money that is less valuable then the
2. Dividend money they borrowed.
3. Forex Gain 55. International Monetary Fund loans can be used
Choose the correct answer from the code given for which of the following purposes?
below (a) Project for economic development
(a) 1 only (b) 2 and 3 only (b) To meet temporary deficits in balance of
(c) 1, 2 and 3 all (d) 1 and 3 only payments
Ans. (c) : Adjusted Gross Revenue (AGR) is defined as (c) Increasing the gold reserve of the Central
gross income minus adjustments to income. Gross banks
revenue includes wages, dividends, capital gains,
business income, interest income, forex gain as well as (d) Capital formation for industrial growth
other income. Hence, all statements (1), (2) and (3) are Ans. (b) : The International Monetary Fund (IMF) is an
correct. agency of United Nations, headquartered in Washington
51. Which of the following statements in correct D.C, establish in 1945. IMF loans can be used to meet
about 'India Trade Gross Domestic Product' temporary deficits in balance of payments.
ratio? 56. White clothes are cooler than black clothes
(a) India Trade GDP ratio has been more than because
50% since 2000
(a) They absorb whole of light
(b) Indian Trade GDP ratio was minimum in
2007 (b) They inhibit penetration of light
(c) Indian Trade GDP ratio was 46.02% in 2019 (c) They make sunlight completely cool
(d) Indian Trade GDP ratio in 2019 was less than (d) They reflect the whole light
2018 by nearly 3.4% Ans. (d) : White clothes are cooler than black clothes
Ans. (d) : Indian trade GDP ratio in 2019 was less than because white clothes reflect all the light reaching on
2018 by 3.39% or nearly 3.4%. Hence option (d) is them while black colour is the best absorber of heat.
correct. India trade to GDP ratio for 2021 was 43.68%, Black clothes absorb most of the heat.
a 5.87% increase from 2020.
57. Arrange the following in chronological order as
52. The Head Office of Small Industries Development
per their discovery and select correct answer
Bank of India (SIDBI) is located at
(a) Ghaziabad (b) Lucknow from the code given below:
(c) Kanpur (d) New Delhi 1. Transistor
Ans. (b) : The head office of Small Industries Development 2. Diode
Bank of India (SIDBI) is located at Lucknow. Established in 3. SMD (Surface Mounted Device)
1990, SIDBI is the apex regulatory body for overall 4. Integrated Circuit (IC)
licensing and regulation of micro, small and medium (a) 1, 2, 3, 4 (b) 2, 1, 3, 4
enterprise finance companies in India. (c) 2, 1, 4, 3 (d) 1, 3, 2, 4
53. Monetary Policy is
Ans. (c) : The chronological order as per their discovery
(a) Opposed to fiscal policy
is as follows-
(b) Complementary to fiscal policy
Diode-1904, John Ambrose Fleming
(c) More effective during depression
(d) Direct measure to control effective demand Transistor- 1947, William Shockley, John Bardeen and
Walter Brattain
Ans. (d) : The Reserve Bank of India is entrusted with the
responsibility of conducting monetry policy in India with Intigrated Circuit (IC) - 1959, Robert Noyce, Jack Kilby
the primary objective of maintaining price stability while Surface Mounted Device (SMD) - 1960
keeping in mind the objective of growth. It is published 58. Match List-I with List-II and select correct
once in two months which means 6 times in a year. answer from the code given below:
UP RO/ARO (Mains) Re-exam 2016 50 YCT
CLICK HERE FOR FREE MATERIAL

List-I List-II 61. Match List-I with List-II and select correct
answer from the code given below:
A. Acid present in 1. Calcium Phosphate
Vinegar List-I List-II
B. Souring of milk 2. Hyrochloric acid A. Bleaching powder 1. Sodium
bicarbonate
C. Compound 3. Acetic acid
B. Baking Soda 2. Sodium carbonate
present in bones
C. Washing powder 3. Calcium
D. Acid present in 4. Lactic acid oxychloride
Gastric juices D. Plaster of Paris 4. Calcium sulphate
Code: hemihydrate
A B C D Code:
(a) 2 1 4 3 A B C D
(b) 3 4 1 2 (a) 3 4 2 1
(c) 4 3 1 2 (b) 2 3 4 1
(d) 2 4 1 3 (c) 3 1 2 4
(d) 4 3 1 2
Ans. (b) : This correct match of list-I with list-II is as
follows- Ans. (c): The correct match of list-I with list-II is as
follows-
List-I List-II
Acid Present in Vinegar - Acetic Acid
List-I - List-II
Souring of Milk - Lactic Acid Bleaching Powder -
Calcium oxychloride
Compound Present in bones - Calcium Phosphate (CaOCl2)
Acid Present in Gastric Juices - Hydrochloric Acid Baking Soda - Sodium bi-carbonate
(NaHCO3)
59. With reference to the radioactivity, which of
the following statement(s) is/are correct?
Washing Powder - Sodium Carbonate
(Na2CO3)
1. Radioactivity is a nuclear property
2. Hydrogen bomb is prepared on the
Plaster of Paris - Calcium sulphate
Hemihydrate
principle of nuclear fission
 1 
Which of the above statement(s) is/are correct?  CaSO 4 ⋅ H 2 O 
(a) Only 1 (b) Only 2  2 
(c) Both 1 and 2 (d) Neither 1 nor 2 62. Match List-I with List-II and select correct
answer from the code given below:
Ans. (a) : Radioactivity is a phenomenon exhibited by a
few matters of emitting energy and subatomic particles List-I List-II
spontenuously. It is a nuclear property. It was Plant hormone Function
discovered by Henri Becquerel in 1896. Hydrogen A. Auxin 1. Cell elongation
Bomb is based on the principle of nuclear fusion while B. Gibberellin 2. Cell division
atom bomb is based on nuclear fission. Fusion occurs C. Cytokinin 3. Growth inhibition
when two atoms fuses together to form a heavier atom. D. Abscisic acid 4. Apical dominance
Nuclear fusion is the process that powers the sun and Code :
creates huge amount of energy. Therefore, Statement
A B C D
(1) is correct and statement (2) is incorrect.
(a) 2 3 4 1
60. Which of the following pair of gases is used for (b) 4 1 2 3
the respiratory activities of diver? (c) 3 4 2 1
(a) Oxygen and Helium (d) 4 1 3 2
(b) Oxygen and Neon Ans. (b) : The correct match of list- I with list- II is
(c) Oxygen and Nitrogen as follows-
(d) Oxygen and Argon List-I - List-II
Ans. (a) : Deep-sea divers are required to carry gas (A) Auxin - Apical dominance
cylinders for breathing purposes while diving. Excess
oxygen causes oxygen toxicity at a higher pressure
(B) Gibberelin - Cell elongation
more than 6 meters underwater, so divers use mixture of (C) Cytokinin - Cell Division
oxygen and helium (heliox) gas. (D) Abscisic Acid - Growth inhibition

UP RO/ARO (Mains) Re-exam 2016 51 YCT


CLICK HERE FOR FREE MATERIAL

63. Given below are two statements, one is labelled 67. Which of the following micro-organisms are
as Assertion (A) and other as Reason (R). used as biofertilizers?
Assertion (A) : Human body synthesizes all the (a) Virus (b) Fungi
vitamins required by it. (c) Protozoa (d) Cyano-bacteria
Reason (R) : Vitamins are essential for proper Ans. (d) : Cyanobacteria, common name blue-green algae,
growth of the body. appear to have originated in a freshwater or terrestrial
Choose the correct answer from the code given environment. It is used as biofertilizer. Biofertilizer are
below. group of micro-organisms that help in increases fertility and
(a) Both (A) and (R) are true and (R) is the growth of plants when applied. Rhizobium, Azotobacter
correct explanation of (A) and Azospirilium are biofertilizers.
(b) Both (A) and (R) are true and (R) is not the 68. 'Safflower' is a oil seed crop. Its scientific name
correct explanation of (A) is
(c) (A) is true but (R) is false (a) Glycine max (b) Ricinus Communis
(d) (A) is false but (R) is true (c) Carthamus Tinctorius (d) Bunchy top
Ans. (d) : The vitamin is an organic molecule that is an Ans. (c) : Safflower is a oil seed crop. Its scientific
essential micronutrient that an organism needs in the name is 'Carthamus Tinctorius'.
proper functioning of its metabolism. Generally
Vitamins cannot be synthesized in human body, so 69. Consider the following Technologies Projects of
vitamins must be obtained through diet. Therefore, Indian Council of Agricultural Research
assertion (A) is incorrect. Vitamins are essential for (ICAR) and arrange them in chronological
proper growth of the body. So, reason (R) is correct. order according to their year of starting.
1. National Agricultural Innovation Project
64. Hydroxy chloroquine drug used for the
treatment of CORONA patients is obtained 2. Krishi Vignan Kendra
from the bark of which of the following plants? 3. National Agricultural Technology Project
(a) Aswagandha (b) Sarpagandha 4. Lab to Land Program
(c) Cinchona (d) Aloevera Select the correct answer from the code given
below
Ans. (c) : Hydroxy Chloroquine drug used for the
treatment of CORONA patients is obtained from the bark (a) 2, 4, 3, 1 (b) 4, 3, 1, 2
of Chinchona plant. Cinchona is used for the production of (c) 1, 2, 4, 3 (d) 3, 1, 2, 4
quinine, which is an anti-fever agent. It is especially useful Ans. (a) : The chronological order of technologies
in the prevention and treatment of malaria. projects of Indian Council of Agricultural Research
65. Which of the following is not correctly (ICAR) is as follows-
matched? Krishi Vigyan Kendra- 1974
Crop Variety Land to Lab Program - 1979
(a) Mustard - Varuna National Agricultural Technology Project- 1998
(b) Pea - Rachna National Agricultural Innovation Project- 2006
(c) Linseed - Neelam 70. Alcohol that is derived from fermentation of
germinated barely grains is known as
(d) Paddy - Radhey (a) Beer (b) Wine
Ans. (d) :The correct match is as follows- (c) Vodka (d) Rum
Crop Variety Ans. (a) : Beer is produced by the brewing and
(A) Mustard - Varuna fermentation of starches mainly derived from barely
(B) Pea - Rachna grains.
(C) Linseed - Neelam 71. Which space craft of NASA has confirmed the
(D) Gram - Radhey presence of water on the moon?
So, it is clear that Radhey is the variety of Gram. (a) ORION (b) GEMINI
66. What is the importance of crop rotation in the (c) SOFIA (d) MERCURY
farming? Ans. (c) : The Stratospheric Observatory for Infrared
(a) It increase the production Astronomy (SOFIA), a project of NASA, has confirmed
(b) Soil fertility is conserved the presence of water on the moon GEMINI was NASA's
(c) Soil erosion is reduced second human spaceflight project program which was
(d) All the above conducted between project Mercury and Apollo.
Ans. (d) : Crop rotation is the practice of growing a 72. On 18th October, 2020, India successfully test
series of different types of crops in the same area across fired which of the following missiles?
a sequence of growing season. It helps to manage soil (a) Brahmos Supersonic Cruise missile, Airforce
fertility, reduce erosion, improve soil's health and increase missile
nutrients available for crops. It also helps to increase the (b) Brahmos Supersonic Cruise missile, Naval
production of corps. Hence, option (d) is correct. version
UP RO/ARO (Mains) Re-exam 2016 52 YCT
CLICK HERE FOR FREE MATERIAL

(c) Brahmos Supersonic missile suitable for all Ans. (d) : The Tomb of Bahu Begum was commissioned
three forces by Nawab Suja-ud-Daula in the memory of his wife
(d) Brahmos Supersonic Cruise missile, Army Begum Unmatuzzoha Bano in Faizabad in 1816.
version 77. For fast pace of industrialization to make
Ans. (b) : BRAHMOS supersonic cruise missile has investment and for generation of employment
been jointly designed, developed and produced by India opportunities U.P. Government would set-up
and Russia. On 18th October 2020, it was successfully State's first data center in
test-fired from Indian Navy's indegnously-built stealth (a) Gautam Budh Nagar (b) Greater Noida
destroyer INS Chennai, hitting a target in the Arabia Sea. (c) Ghaziabad (d) Ghazipur
73. Given below are two statements, one is labelled Ans. (b) : For fast pace of industrialization to make
as Assertion (A) and other as Reason (R). investment and for generation of employment
Assertion (A) : Various components of an opportunities, U.P. Government would set-up State's
ecosystem are non interdependent. first data center in Greater Noida.
Reason (R) : Human activities have the impacts 78. In a class of 60 students where girls are twice
on the environment. that of boys. Laxmi ranked 27th from the top.
Choose the correct answer from the code given If there are 9 boys ahead of Laxmi, how many
below. girls are after her rank?
(a) Both (A) and (R) are true and (R) is the (a) 23 (b) 21
correct explanation of (A) (c) 22 (d) 24
(b) Both (A) and (R) are true and (R) is not the
correct explanation of (A) Ans. (c) : Let number of boys = x
(c) (A) is true but (R) is false And number of girls = 2x
(d) (A) is false but (R) is true According to the question,
2x + x = 60
Ans. (d) : Various components of an ecosystem are
interdependent. So, assertion (A) is not true. Human 3x = 60
activities have the impacts on the environment. Hence, x = 20
assertion (A) is false but reasons (R) is true. No of boys = 20
74. In Uttar Pradesh which of the following assist No of girls = 40
entrepreneurs in setting-up of industries and
enterprises?
(a) U.P. State Financial Corporation
(b) U.P. State Industrial Development Authority
40 – (27 – 9)
(c) Department of Infrastructure and Industrial
= 40 – 18 = 22
Development
So, there will be 22 girls after Laxmi.
(d) Udyog Bandhu
Ans. (d) : Udyog Bandhu is an organisation developed by 79. In a certain code language, DISTANCE is
the state government of Uttar Pradesh to facilitate written as IDTUBECN and DOCUMENT is
investment in the industrial and service sectors. It assist written as ODDVNTNE. How will THURSDAY
entrepreneurs in setting up industries and enterprise in UP, be written in that code language?
getting approvals and clearances, solving problems for them. (a) HTTQRYAD (b) HTVSTYDA
(c) HTVSTYAD (d) HTVSYADS
75. With reference to Thumri which of the
following statement(s) is/are correct? Ans. (c) : The code of given language-
1. Thumri originated in UP
2. Thumri is light and attractive
Select the correct answer using the code given
below: and
Code
(a) Only 1 (b) Only 2
(c) Both 1 and 2 (d) Neither 1 nor 2
Ans. (c) : Thumri is North India's most popular light- According to the code
classical song form, developed during the 19th century at the
court of Lucknow's ruler Wajid Ali Shah. Thumri is light
and attractive from. Hence, both statement are correct.
76. The famous tomb of wife of Nawab So, the code = H T V S T Y A D
Shuzaudaullah, 'Bahu Begam Tomb' is situated 80. Complete the following series
at which of the following places? 2, 6, 14, 26, ?, 62
(a) Azamgarh (b) Lucknow (a) 54 (b) 42
(c) Prayagraj (d) Faizabad (c) 40 (d) 50
UP RO/ARO (Mains) Re-exam 2016 53 YCT
CLICK HERE FOR FREE MATERIAL

Ans. (b) : The missing number of the given series – Ans. (d) : A circle has an arc. A television has a screen
and a book has a cover. But charger is another device to
charge laptop.
86. X and Y are brothers. R is the father of Y. S is
brother of T and maternal uncle of X. What is
So, missing number = 42 relation of R with T?
(a) Mother (b) Husband
81. My brother is 562 days elder than me and my
sister is 75 weeks elder than my brother. If my (c) Brother (d) Wife
sister was born on Tuesday then on which day I Ans. (b) :
was born?
(a) Sunday (b) Thursday
(c) Tuesday (d) Wednesday
Ans. (b) :

So, R is the husband of T.


87. Find the missing number in the following table.
Total number of odd days between my sister and 4 5 6
525 + 562 1087
me = = = 2 odd days 2 3 7
2 7 1 8 3
My birthday = Tuesday +2 = Thursday
21 98 ?
82. Find the missing number in the following table.
(a) 94 (b) 76
9A 8C 4B
(c) 73 (d) 16
3A 19B 2C
Ans. (a) :
23B 10C ?
42 + 22 + 12 = 16 + 4 + 1 = 21
(a) 28A (b) 16C 52 + 32 + 82 = 25 + 9 + 64 = 98
(c) 25A (d) 18B
62 + 72 + 32 = 36 + 49 + 9 = 94
Ans. (a) : ⇒ 9A + 19 B ⇒ 28 A
In every row there are A, B an C, so there will be an A 88. What is the number of triangles in the
in the last row. following figure?
83. Find the missing number in the series.
4, 18, ?, 100, 180, 294, 448
(a) 48 (b) 50
(c) 58 (d) 60
Ans. (a) :

(a) 18 (b) 28
(c) 20 (d) 24
Ans. (b) :
84. If you write down all the numbers from 1 to
100, then how many times will you write 3?
(a) 11 (b) 18
(c) 20 (d) 21
Ans. (c) : We have to write twenty times three, if we
write down all the numbers from 1 to 100-
3, 13, 23, 30, 31, 32, 33, 34, 35, 36, 37, 38, 39, 43,
53, 63, 73, 83, 93. = 20 times
85. Out of four options given below, three are of a Number of triangles made with one number = 14
kind while one does not belong to the group. Number of triangles made with two numbers = (1,2),
Choose the one which is unlike the others. (3,4), (5,6), (7,8), (9,10) ,(12,13), (8,9), (7,10), (13,3),
(a) Circle : Arc (4,12) = 10
(b) TV : Screen Number of triangles made with three number =
(c) Book : Cover (9,10,11), (12,13,14), (9,10,14), (11,12,13) = 4
(d) Laptop : Charger So, the total number of triangles = 14+10+4 = 28
UP RO/ARO (Mains) Re-exam 2016 54 YCT
CLICK HERE FOR FREE MATERIAL

89. Pointing towards a man's photograph, Sanjay 93. When was the Citizenship (Amendment) Bill
said, "This man's mother is wife of my father's 2019 passed by parliament?
son and I do not have any Bhabhi (elder (a) 10 December, 2019 (b) 11 December, 2019
brother's wife) and sister. (c) 12 December, 2019 (d) 13 December, 2019
The photograph is of Sanjay's which relative? Ans. (c) : The Citizenship (Amendment) Act, 2019 was
(a) Uncle (b) Cousin brother passed in Lok Sabha on 9th December and in Rajya
(c) Son (d) Father Sabha on 11th December 2019 and was assented by the
Ans. (c) : President on 12th December 2019. It amends the
Citizenship Act, 1955 by providing a pathway to Indian
citizenship for persecuted religious minorities from
Afghanistan, Bangladesh and Pakistan who are Hindus,
Sikhs, Buddhists, Jains, Parsis or Christians, and are
arrived in India before the end of December 2014.
94. During Trump Presidency, US has withdrawn
The person in the photograph is the son of Sanjay. from which of the following treaties/
agreements?
90. Find the missing number in the following in
1. Iran Nuclear Deal
place of question (?) mark.
2. Paris Agreement
3. Open Skies Treaty
Select the correct answer using the code given
below
Code
(a) 639 (b) 542 (a) All 1, 2 and 3 (b) Only 3
(c) 529 (d) 641 (c) Only 1 and 3 (d) Only 2 and 3
Ans. (c) : 1. 17 + 18 = 35 Ans. (a) : During Donald Trump Presidency (2016-
35 × 35 = 1225 2020). US has withdrawan from Iran Nuclear Deal,
2. 13 + 14 = 27 Paris Agreement and Open Skies Treaty.
27 × 27 = 729 95. Who is named as the Chairman of Governing
3. 19 + 4 = 23 Council of Film and Television Institute of
Indian in September 2020?
23 × 23 = 529
(a) Smriti Irani
91. In a certain code language HAND is written (b) Amresh Chakraborty
SZMW, then what will be the code for MILK? (c) Shekhar Kapoor
(a) ORNP (b) PNRO (d) Prakash Javadekar
(c) NROP (d) RNOP
Ans. (c) : Shekhar Kapoor was named as the Chairman
Ans. (c) : of Governing Council of Film and Television Institute
of India in September 2020.
96. Which of the following women has become New
Zealand's first ever India-origin minister
(a) Kamla Haris
(b) Priyanka Radhakrishnan
(c) Sudha Vergese
(d) Tulsi Gabbard
Ans. (b) : In November 2020, Priyanka Radhakrishnan
92. The collection of 'Goods and Service Tax' in become New Zealand's first-ever Indian-orgin minister.
Indian economy in October, 2020 has been Born in Chennai, Radhakrishnan was elected first as a
(a) More than Rs. One lakh crore member of parliament belonging to the Labour Party in
September 2017.
(b) Less than Rs. One lakh crore
(c) Equal to Rs. One lakh crore 97. Which state of India has recently decided to fix
(d) Equal to Rs. Two lakh crore a Minimum Support Price (MSP) for fruits and
vegetables?
Ans. (a) : The gross GST revenue collected in the
(a) Kerala (b) Manipur
month of October 2020 is ` 1,05,155 crore (more than
Rs. 1 lakh crore). Hence, option (a) is correct. The gross (c) Himachal Pradesh (d) Telangana
GST revenue collected in the month of September, 2022 Ans. (a) : Kerala became the first state to fix a
is ` 1,47,686 crore. minimum support price (MSP) for fruits and vegetables.
UP RO/ARO (Mains) Re-exam 2016 55 YCT
CLICK HERE FOR FREE MATERIAL

98. With reference to 'Khelo India Youth Games' (a) 1 and 2 only (b) 1 and 3 only
which of the following statement(s) is/are (c) 2 and 3 only (d) 1, 2 and 3
correct? Ans. (b) : Regarding the Micro, Small and Medium
1. Haryana received the Winner's Trophy Enterprises (MSME) Act. 2020, statement (1) and
'Khelo India Youth Games, 2020'
statement (3) are correct:
2. Haryana will host 'Khelo India Youth
Games' in 2021 The Government to India announced a revision in
MSME definition in Atmanirbhar Bharat Package on
Choose the correct answer from the code given
below 13th May 2020. The change in definition is effective
Code: from 1st July 2020.
The new definition has the same formula of
(a) Only 1 (b) Only 2
classification of manufacturing and service units.
(c) Both 1 and 2 (d) Neither 1 nor 2
102. The term "Revenue Neutral Rate" was in news
Ans. (b) : Maharashtra recived the Winner's Trophy
with 78 gold medals in 'Khelo India Youth Games recently is related to
2020'. Harayana will host 'Khelo India Youth Games (a) Goods and Service Tax (GST)
2021'. Hence, statement (1) is incorrect and statement (b) Foreign Portfolio Investment (FPI)
(2) is correct. Due to COVID-19 the fourth edition of (c) Disinvestment of Public Sector Units
Khelo India Youth Games was held in June 2022 in (d) Foreign Direct Investment (FDI)
Panchkula (Harayana). In this game Harayana topped
Ans. (a) : Revenue Neutral Rate is the rate of tax that
the medal's talley with 52 gold medals followed by
allows the government to receive the same amount of
Maharastrata and Karnataka.
money despite changes in the tax laws. It is related to
99. Why was the Science and Technology Minister of Goods and Service Tax (GST).
Pakistan, Fawad Choudhry in news recently?
(a) For criticising Imran Khan, P.M. of Pakistan 103. Which of the following places of U.P. was
(b) For taking credit of Pulwana for Pakistan known as 'Rang Theatre' during the British
while speaking in the National Assembly Rule in India?
(c) For giving credit for Pulwama to Rahul Gandhi (a) Banaras Hindu University, Varanasi
(d) For giving credit of Pulwama to Narendra Modi (b) High Court at Prayagraj
Ans. (b) : The Science and Technology Minister of (c) Chhatar Manjil at Lucknow
Pakistan, Fawad Choudhry was in news recently for (d) G.P.O. of Lucknow
taking credit of pulwama attack while speaking in the Ans. (d) : General Post Office (GPO) building of
National Assembly. Agha Hassan Baloch is the current Lucknow was known as Rang Theatre during British
Science and Technology Minister of Pakistan. Rule in India. The building was generally recognised as
100. What was the theme of the 'World Tourism the Entertainment Center of Britishers.
Day, 2020' celebrated in September, 2020? 104. Consider the following statements regarding
(a) Tourism and Rural Development 'Atal Tunnel' which is/are rights?
(b) Tourism and Jobs
1. It is built by Border Roads Organisation
(c) Tourism and Digital Transformation
2. It connects Manali to Lahaul Spiti Valley
(d) Accessible Tourism
3. It is located in Pir-Panjal Range of
Ans. (a) : The theme of the 'World Tourism Day, 2020'
Himalayas
celebrated on September 27, was 'Tourism and Rural
Development'. The theme of 'World Tourism Day 2022' 4. It is strategically important for India
is - 'Rethinking Tourism.' Choose the correct answer from the code given
101. Consider the following statements about Micro, below
Small and Medium Enterprises (MSME) Act, Code:
2020 : (a) Only 1 and 2 (b) Only 2 and 3
1. The Government of India announced a (c) Only 1, 2 and 4 (d) All 1, 2, 3 and 4
revision in MSME definition in the Ans. (d) : Atal Tunnel was inaugurated by the Prime
Atmanirbhar Bharat package on 13th Minister Narendra Modi on 3rd October 2020.
May, 2020
It is built by Border Roads Organistion.
2. The change in definition is effective from
It is connects Manali to Lahaul Spiti Valley.
1st June, 2020
3. The new definition has the same formula of It is located in Pir-Panjal Range of Himalayas.
classification of manufacturing and service It is strategically important for India.
units It is officially recognised as 'Longest Highway Tunnel
Which are correct among the above above 10,000 feet'.
statements? Therefore, all statement are correct.
UP RO/ARO (Mains) Re-exam 2016 56 YCT
CLICK HERE FOR FREE MATERIAL

105. The countries which participated in the Ans. (d) : The correct match from List-I to List-II is
Malabar Naval Exercise in November, 2020 are as follows-
(a) India and United States of America List-I (Event) List-II (Place)
(b) India, USA, Japan and Australia A. 105th Indian Science Congress Imphal
(c) India, USA and Japan B. 106th Indian Science Congress Jalandhar
(d) India, USA, Japan and Sri Lanka C. 107th Indian Science Congress Bangalore
Ans. (b) : The second phase of Exercise Malabar was D. 108th Indian Science Congress Pune
conducted in the Northern Arabian Sea in November
108th Indian Science Congress will be held from 3-7
2020. This phase involved coordinated operations
January 2023 at R.T.M. Nagpur University Nagpur
among the navies of Australia, India, Japan and the (rescheduled).
USA. It is a multilateral war gaming naval exercise that
was started in 1992. 110. Which of the following Arab countries has not
normalised relations with Israel?
106. The women's (single) title of the French Open (a) Sudan (b) Bahrain
2020 was won by which of the following (c) Jordan (d) Kuwait
players?
Ans. (d) : In the given options, except Kuwait all other
(a) Iga Swiatek (b) Petra Kuitova three countries have normalised relations with Israel.
(c) Serena Williams (d) Sophia Kenin Currently, UAE, Bahrain, Egypt, Sudan and Jordan
Ans. (a) : Igo Swiatek (Poland) won the women's single have normalised relation with Israel.
title of French Open 2020. Rafael Nodal (Spain) won 111. Recently, the first "Gender Social Norms
the men's single title and Igo Swiatek won the women's Index" was released by which of the following?
singles title of French Open 2022. (a) World Bank
107. India-born 'Wavel Ramkalawan' has been (b) World Bank and United Nations Development
recently elected President of which of the Programme
following countries? (c) World Bank and NITI Aayog
(a) Guyana (b) Fiji (d) United Nations Development Programme
(c) Surinam (d) Seychelles Ans. (d) : The first 'Gender Social Norms Index' was
Ans. (d) : Recently India-born 'Wavel Ramkalawan' has relased by the United Nations Development
been elected President of Seychelles in October 2020. Programme. According to this index, about 90% of the
world's population has some bias against women. In this
108. On October 31, 2020 the first sea-plane has index, Pakistan ranked first followed by Qatar and Nigeria.
been started in 112. 'Mumbai : Kolkata :: Mangalore' are related in
(a) Mumbai (b) Kolkata some way. Choose from following which has
(c) Ahmedabad (d) Panjim the same relation with them.
Ans. (c) : Prime Minister Narendra Modi inaugurated (a) Hyderabad (b) Cochin
India's first seaplane service on 31st October 2020, the (c) Delhi (d) Jaipur
birth anniversary of Sardar Vallabhbhai Patel. It is Ans. (b) : Mumbai, Kolkata and Mangalore all are port
likely to take off from Sabarmati river front to the cities. In the same way option (b) Cochin (Kerala) is
Statue of Unity. also a port city.
109. Match List-I with List-II and select correct 113. Arrange the following words in meaningful
answer from the code given below: order and select correct answer from code
given below:
List-I (Event) List-II (Place)
1. Uttar Pradesh
A. 105th Indian Science 1. Bangalore 2. Universe
Congress 3. Prayagraj
B. 106th Indian Science 2. Pune 4. World
Congress 5. India
C. 107th Indian Science 3. Imphal Code:
Congress (a) 3, 1, 4, 5, 2 (b) 1, 3, 5, 4, 2
D. 108th Indian Science 4. Jalandhar (c) 3, 1, 5, 4, 2 (d) 3, 1, 2, 4, 5
Congress Ans. (c) : The correct order is - Prayagraj, Uttar
Code: Pradesh, India, World, Universe. Therefore option (c) 3,
1, 5, 4, 2 is correct.
A B C D
114. Consider the following statements with
(a) 1 2 3 4
reference to Atal Mission for Rejuvenation and
(b) 2 3 1 4 Urban Transformation (AMRUT) Scheme:
(c) 3 4 2 1 1. 400 cities have been covered under this
(d) 3 4 1 2 mission
UP RO/ARO (Mains) Re-exam 2016 57 YCT
CLICK HERE FOR FREE MATERIAL

2. The AMRUT Scheme covers 60% of the Code :


country's urban population A B C D
Which of the above statement(s) is/are correct? (a) 1 3 2 4
(a) Only 1 (b) Only 2 (b) 1 2 3 4
(c) Both 1 and 2 (d) Neither 1 nor 2 (c) 2 3 1 4
Ans. (b) : Atal Mission for Rejuvenation and Urban (d) 1 3 4 2
Transformation (AMRUT) scheme was launched in Ans. (b): The correct match with List-I to List-II is as
October 2015 to provide basic civic amenities like water follows-
supply, sewerage, Urban transport. This mission covers List-I - List-II
500 cities and over 60% population. Therefore, (Mission) (Launching Year)
statement (1) is incorrect and statement (2) is correct. A. National Mission for - 2009
115. Who is the founder of 'Gulabi Gang Movement'? Enhanced Energy
(a) Jaya Tiwari (b) Sampat Pal Efficiency
(c) Kamla Bhasin (d) Vandana Shiva B. National Water Mission - 2011
Ans. (b) : The Gulabi Gang is an extraordinary women's C. National Mission for - 2014
movement formed in 2006 by Sampat Pal Devi in Banda Sustaining the Himalayan
district of Uttar Pradesh as a response to widespread Ecosystem
domestic abuse and other voilence against women. D. National Mission for - 2008
116. With reference to the Hindu Nayab Wazirs of Sustainable Agriculture
Oudh Sultanate, which of the following 119. Given below are two statements, one is labelled
statement(s) is/are correct? as Assertion (A) and other as Reason (R).
1. Raja Tikait Ram was 'Artha Mantri' of Assertion (A) : Cities in western countries have
Nawab Asafudaullah more females as compared to male.
2. Usually there were certain Hindu castes Reason (R) : In western countries, agriculture
who were placed high positions in the State work is done with new technology in which men
of Nawabs of Oudh have more participation.
Select the correct answer from the code given Choose the correct answer from the code given
below : below.
Code: (a) Both (A) and (R) are true and (R) is the
correct explanation of (A)
(a) Only 1 (b) Only 2
(b) Both (A) and (R) are true and (R) is not the
(c) Both 1 and 2 (d) Neither 1 nor 2 correct explanation of (A)
Ans. (c) : Raja Tikait Rai was 'Artha Mantri' of Nawab (c) (A) is true but (R) is false
Asafudaullah. So, statement I is correct. Usually there were (d) (A) is false but (R) is true
centain Hindu castes who were placed high positions in the Ans. (b) : Cities in western countries have more
State of Nawab of Awadh. Therefore, both statements are females as compared to males. In western countries,
correct. So, option (c) is the correct answer. agriculture work is done with new technology in which
117. Rajiv Gandhi National Aviation University is men have more participation. Both Assertion (A) and
situated in U.P. at Reason (R) are true, but (R) is not the correct
(a) Noida (b) Agra explanation of (A).
(c) Fursatganj (d) Varanasi 120. Given below are two statements, one is labelled
Ans. (c) : Rajiv Gandhi National Aviation University is as Assertion (A) and other as Reason (R).
situated in Fursatganj, Uttar Pradesh. Assertion (A) : Pakistan has greater racial and
118. Match List-I with List-II and select correct ethnic diversity than Bangladesh.
answer from the code given below : Reason (R) : Punjabi is the official language of
List-I (Mission) List-II Pakistan where most people use Urdu language in
speaking.
Launching year
Choose the correct answer from the code given
A. National Mission for 1. 2009 below.
Enhanced Energy (a) Both (A) and (R) are true and (R) is the
Efficiency correct explanation of (A)
B. National Water 2. 2011 (b) Both (A) and (R) are true and (R) is not the
Mission correct explanation of (A)
C. National Mission for 3. 2014 (c) (A) is true but (R) is false
Sustaining the (d) (A) is false but (R) is true
Himalayan Ecosystem Ans. (c) : Pakistan has greater racial and ethnic
D. National Mission for 4. 2008 diversity than Bangladesh. Hence option (A) is correct.
Sustainable The official language of Pakistan is Urdu not Punjabi.
Agriculture Hence, reason (R) is incorrect.
UP RO/ARO (Mains) Re-exam 2016 58 YCT
CLICK HERE FOR FREE MATERIAL

Gòej ØeosMe meceer#ee DeefOekeâejer/meneÙekeâ meceer#ee DeefOekeâejer (cegKÙe) hejer#ee, 2016


meeceevÙe efnvoer
nue ØeMve-he$e (hejer#ee efleefLe : 22 efomecyej, 2020)
(efJeMues<eCe meefnle JÙeeKÙee)
1. efvecveefueefKele ceW mes keâewve-mee Meyo efJeMes<eCe nw? 5. efJeueesce keâer Âef° mes efvecveefueefKele ceW mes Skeâ mener Ùegice nw
(a) Oece& (b) peeefle (a) leeC[Je - ceeB[Je
(c) Ûecekeâeruee (d) Oeve (b) Deve=le-$e+le
Ans. (c) : efoÙes ieÙes efJekeâuheeW ceW Ûecekeâeruee efJeMes<eCe nw peyeefkeâ (c) cegòeâ-efJecegòeâ
Fmekeâe efJeMes<Ùe Ûecekeâ nesiee~ (d) efJeYeeJevee-meodYeeJevee
efJeMes<Ùe efJeMes<eCe Ans. (b) : ‘Deve=le’ keâe efJeueesce ‘$e+le’ nw~ cegòeâ keâe ‘yeBOee Ùee
Oece& - Oeeefce&keâ yeæ’,‘leeC[Je’ keâe ‘ueemÙe’ peyeefkeâ ‘efJeYeeJevee’ keâe efJeueesce
peeefle - peeleerÙe ‘efJeMes<eesefkeäle’ neslee nw~
Oeve - Oeveer
6. efvecveefueefKele ceW mes efJeueesce keâe DeMegæ Ùegice nw
2. efJeMes<Ùe/efJeMes<eCe keâer Âef° mes keâewve-mee Meyo Ùegice
(a) Devevle-Demeerce
DeMegæ nw?
(b) kegâMeue-DekegâMeue
(a) Âef°-Âef°Jevle (b) ieg®-efMe<Ùe
(c) DeOece&-Oece&
(c) ceefCe-jlve (d) oerhekeâ-lesue
(d) peÌ[-Ûesleve
Ans. (a) : efoÙes ieÙes efJekeâuheeW ces bmes efJekeâuhe (a) ceW ‘Âef°’ (efJeMes<Ùe)
leLee ‘Âef°Jevle’ (efJeMes<eCe) Meyo nw~ ‘Âef°’ mes yevee efJeMes<eCe ‘õ°JÙe’ Ans. (a) : ‘Devevle’ Meyo keâe efJeueesce meeble/meerefcele neslee nw~ peyeefkeâ,
neslee nw~ Mes<e efJekeâuhe ceW Ùegice ceW oesveeW Meyo ‘efJeMes<Ùe heo’ nw FveceW Demeerce ‘Devevle’ keâe meceeveeLeea Meyo nw~ Mes<e meYeer efJekeâuhe mener nw~
Skeâ Yeer efJeMes<eCe veneR nw~ 7. efJeueesce keâer Âef° mes Megæ Ùegice nw
3. efvecveefueefKele keâLeveeW ceW mes DeMegæ keâLeve nw (a) efJejeie - efÛejeie
(a) pees kesâJeue meJe&veece keâer efJeMes<elee yeleeS, Gmes efJeMes<eCe (b) mebÙegòeâ - efJeÙegòeâ
keânles nQ~ (c) mejme - Dejme
(b) efJeMes<eCe Éeje efpeme Meyo keâer efJeMes<elee Øekeâš nes, Gmes
(d) DeJee&Ûeerve - meceerÛeerve
efJeMes<Ùe keânles nQ~
Ans. (b) : efJeueesce keâer Âef° mes Megæ Ùegice mebÙegòeâ – efJeÙegòeâ nw~
(c) efpeme efJeMes<eCe kesâ Éeje efkeâmeer meb%ee DeLeJee meJe&veece kesâ
iegCe Øekeâš nes, Gmes iegCeJeeÛekeâ efJeMes<eCe keânles nQ~ DevÙe efJeJejCe efvecveJele nw-
(d) pees efJeMes<eCe efJeMes<Ùe Deewj ef›eâÙee kesâ yeerÛe DeeS, Gmes Meyo efJeueesce
efJeOesÙe efJeMes<eCe keânles nQ~ efJejeie - jeie
Ans. (a) : efoS ieS keâLeveeW ceW efJekeâuhe (a) leeefke&âkeâ ™he mes DeMegæ mebÙegòeâ - he=Lekeâ, DemebÙegòeâ/efJeÙegòeâ
nw keäÙeeWefkeâ efJeMes<eCe Skeâ Ssmee efJekeâejer Meyo nw pees meb%ee Deewj meJe&veece mejme - veerjme
(oesveeW) keâer efJeMes<elee yeleueelee nw~ DeJee&Ûeerve - ØeeÛeerve
4. ‘Jen meceejesn, efpemeceW ieg®kegâue kesâ mveelekeâeW keâes 8. efvecveefueefKele JeekeäÙeeW ceWs mes Skeâ JeekeäÙe DeMegæ nw
efJeÅeeOÙeÙeve keâj uesves kesâ Ghejeble efJeoeF& oer peeleer Leer’- (a) Úe$eeW ves hebef[le vesn™ keâes DeefYevevove he$e Deefhe&le efkeâÙee~
Fme JeekeäÙeebMe kesâ efueS Skeâ Meyo
(b) ceneosJeer ves mejmJeleer keâes ßeæe megceve Deefhe&le efkeâÙee~
(a) meceeJepe&ve (b) meceeJeeme
(c) jeveer ves yegæ keâes Deefnbmee YeeJe Deefhe&le efkeâÙee~
(c) meceeJejCe (d) meceeJele&ve
(d) Úe$eeW ves hebef[le vesn™ keâes DeefYevevove he$e Øeoeve efkeâÙee~
Ans. (d) : ‘Jen meceejesn efpemeceW ieg®kegâue kesâ mveelekeâeW keâes
efJeÅeeOÙeÙeve keâj uesves kesâ Ghejevle efJeoeF& oer peeleer Leer’ Fme JeekeäÙeebMe Ans. (d) : Øemlegle efJekeâuheeW ceW mes efJekeâuhe (d) DeMegæ JeekeäÙe nw,
kesâ efueS Skeâ Meyo ‘meceeJele&ve’ nw~ meceeJele&ve efnvogDeeW keâe 12Jeeb efpemekeâe Megæ ™he nw - ‘Úe$eeW ves hebef[le vesn™ keâes DeefYevevove he$e
mebmkeâej nw~ ØeeÛeervekeâeue ceW ieg®kegâue ceW efMe#ee hetCe& nesves kesâ heMÛeele Deefhe&le efkeâÙee’~ ‘Øeoeve’ keâe ØeÙeesie yeÌ[eW keâer Deesj mes Úesšs kesâ efueS
peye efJeÅeeLeea ieg® Éeje GheosMe oskeâj efJeoeF& keâer peeleer Leer lees Gmes neslee nw, peyeefkeâ ‘Dehe&Ce’ keâe ØeÙeesie ÚesšeW keâer Deesj mes yeÌ[eW kesâ efueS~
meceeJele&ve mebmkeâej keânles nQ~ Mes<e leerveeW efJekeâuhe Megæ JeekeäÙe nQ~
UP RO/ARO (Mains) Exam Hindi 2016 59 YCT
CLICK HERE FOR FREE MATERIAL

9. FveceW mes Megæ Jele&veer keâe Meyo nw 15. efvecveefueefKele ceW mes keâewve-mee Meyo leodYeJe nw?
(a) efnbmeÇ (b) Yee<keâj (a) hemeervee
(c) ™heÙee (d) yeoece (b) hekeäJeeve
Ans. (a) : efoÙes ieÙes efJekeâuhe ceW Megæ Meyo ‘efnbœe’ nw~ Yee<keâj keâe (c) heMÛeeleehe
Megæ ‘Yeemkeâj’, ‘™heÙee’ keâe ®heÙee Deewj ‘yeoece’ keâe yeeoece Megæ (d) heg$eJeOet
™he nw~ Ans. (a) : ‘ØeefmJeVe’ keâe leoYeJe hemeervee neslee nw~ hekeäJeeve, he§eeleehe
10. efvecveefueefKele ceW mes Skeâ JeekeäÙe DeMegæ nw Je heg$eJeOet lelmece Meyo nw efpemekeâe leodYeJe ›eâceMe: hekeâJeeve, heÚleeJee
(a) legheâeve ceW mewkeâÌ[eW ueesie cej ieS~ Je heleesnt nw~
(b) otmeje Deeoceer keâewve Lee? 16. FmeceW mes lelmece-leodYeJe keâe Skeâ Ùegice DeMegæ nw, Jen nw
(c) [e@keäšj ves Demheleeue keâe efvejer#eCe efkeâÙee~ (a) GheeOÙeeÙe-DeesPee
(d) meÌ[keâ kesâ oesveeW Deesj DeeueerMeeve Fceejles nQ~
(b) ßes‰ - mes"
Ans. (a) : Øemlegle efJekeâuheeW ceW mes ‘legheâeve ceW mewkeâÌ[eW ueesie cej ieS’ (c) Deefive‰ - DeBieer"e
DeMegæ JeekeäÙe nw~ efpemekeâe Megæ ™he ‘letHeâeve ceW mewkeâÌ[eW ueesie ceejs (d) Deeõ&keâ - Deojkeâ
ieÙes’ nw~ Mes<e meYeer JeekeäÙe Deheves Megæ ™he ceW Øemlegle nw~
Ans. (b) :
11. ‘pees legjvle keâesF& Ùegefòeâ meesÛe ues’, Gmes keânles nQ
leodYeJe lelmece
(a) meJe&%e
DeesPee - GheeOÙeeÙe
(b) otjoMeea
mes" - ßesef‰
(c) ØelÙeglheVeceefle
Debieer"er - Deefiveef‰keâe
(d) oeMe&efvekeâ
Deojkeâ - Deeo&Çkeâ
Ans. (c) : pees legjvle keâesF& Ùegefòeâ meesÛe ues- ØelÙeglheVeceefle~ efpemes meejer
Debieer"e - Deefive‰
yeeleeW keâe %eeve nes - meJe&%e~ Jen JÙeefòeâ pees otj lekeâ keâer yeele henues ner
meesÛe ues-otjoMeea~ pees oMe&ve keâe %eelee nes- oeMe&efvekeâ~ 17. ‘yeepeej ceW yengle IeesÌ[s DeeS nQ’ - Fme JeekeäÙe ceW efJeMes<eCe
12. ‘Ùen keâceerpe Gpeueer nw’ Fme JeekeäÙe ceW ‘keâceerpe’ Meyo nw keâer Âef° mes FveceW ‘yengle’ Meyo nw
(a) efJeMes<eCe (a) efveef§ele heefjceeCeyeesOekeâ efJeMes<eCe
(b) efJeMes<Ùe (b) DehetCeeËkeâyeesOekeâ efJeMes<eCe
(c) ef›eâÙee efJeMes<eCe (c) legueveelcekeâ efJeMes<eCe
(d) meJe&veece (d) Deefveef§ele heefjceeCeyeesOekeâ efJeMes<eCe
Ans. (b) : ‘Ùen keâceerpe Gpeueer nw’ JeekeäÙe ceW ‘keâceerpe’ efJeMes<Ùe nw~ Ans. (d) : efpeme efJeMes<eCe ceW efkeâmeer meb%ee keâe keâesF& efveefMÛele heefjceeCe
pees meb%ee Je meJe&veece keâer efJeMes<elee yeleeÙes, Gmes ‘efJeMes<eCe’ keâne peelee %eele ve nes~ pewmes- meYeeieej ceW yengle Deeoceer Les~ FmeceW ‘yengle’ Meyo
nw Deewj efpemekeâer efJeMes<elee yeleeF& peeÙe, Gmes ‘efJeMes<Ùe’ keâne peelee nw~ DeefveefMÛele heefjceeCeyeesOekeâ efJeMes<eCe nw~
13. ‘efpemekesâ ùoÙe ceW cecelee ve nes’ Gmes keânWies 18. ‘Yejle mejerKee YeeF& efceuevee keâef"ve nw~’ Fme JeekeäÙe ceW
(a) efveo&Ùe ‘mejerKee’ Meyo nw
(b) efveYeeakeâ (a) iegCeJeeÛekeâ efJeMes<eCe
(c) ve=Mebme (b) DehetCeeËkeâyeesOekeâ efJeMes<eCe
(d) efvece&ce (c) DeeJe=efòeJeeÛekeâ efJeMes<eCe
Ans. (d) : ‘efpemekesâ ùoÙe ceW cecelee ve nes’ Gmes efvece&ce keânles nQ~ (d) mecegoeÙeJeeÛekeâ efJeMes<eCe
Gòeâ efJeJejCe efvecve nw- Ans. (a) : ‘Yejle mejerKee YeeF& efceuevee keâef"ve nw’~ Fme JeekeäÙe ceW
JeekeäÙeebMe Skeâ Meyo mejerKee Meyo ‘iegCeJeeÛekeâ’ efJeMes<eCe nw~ peye efJeMes<eCe Jemleg Ùee JÙeefòeâ
• efpemekesâ ùoÙe ceW oÙee ve nes efveo&Ùe kesâ iegCe, jbie, Deekeâej, oMee, DeJemLee Deeefo keâe yeesOe keâjeves keâe
• pees YeÙe jefnle nes efveYeeakeâ DeeOeej yevelee nw, lees JeneB iegCeJeeÛekeâ efJeMes<eCe ØeÙegòeâ neslee nw~
GoenjCe-yeenjer, megvoj, Yeuee, uecyee Deeefo~
• DelÙeefOekeâ ›etâj ve=Mebme
19. efvecveefueefKele ceW mes Gvcetueve keâe mener efJeueesce nw
14. ‘Yeòeâ’ keâe leodYeJe keäÙee nw?
(a) Yeiele (b) ßeæe (a) DeJe®æ
(c) Øesce (d) Yeehe (b) efvecee&Ce
(c) mece=efæ
Ans. (a) : ‘Yeòeâ’ keâe leodYeJe ‘Yeiele’ nw~ ‘Yeehe’ keâe lelmece ‘Jee<he’
leLee ‘mejOee’ keâe lelmece ‘ßeæe’ nw~ (d) jesheCe

UP RO/ARO (Mains) Exam Hindi 2016 60 YCT


CLICK HERE FOR FREE MATERIAL

Ans. (d) : efJeJejCe efvecveJele nw- Ans. (b) : JeekeäÙeebMe kesâ efueS Skeâ Meyo- pees peeslee- yeesÙee ve ieÙee
Meyo efJeueesce nes- Deke=âef<ele~ efpeme Yetefce ceW kegâÚ GlheVe ve neslee nes- Tmej~
DeJe®æ - DeveJe®æ 26. ‘meye kegâÚ Keeves Jeeuee’ kesâ efueS Skeâ Meyo nw
efvecee&Ce - OJebme (a) meJe&Ye#eer
mece=efæ - efJeveeMe (b) meJe&nejer
Gvcetueve - jesheCe (c) meJe&«eener
20. ieieveÛegcyeer keâe meJe&Lee Megæ efJeueesce Meyo nw (d) GheÙeg&òeâ ceW mes keâesF& veneR
(a) Yetleue Ans. (a) : JeekeäÙeebMe kesâ efueS Skeâ Meyo nw- meye kegâÚ Keeves Jeeuee-
(b) OejeMeeÙeer
meJe&Ye#eer~
27. ‘pees Skeâ peien mes otmejer peien ve ues peeÙee pee mekesâ’ kesâ
(c) Yetefcemeeled
efueS Skeâ Meyo nw
(d) leuemheMeea
(a) meoeJele&
Ans. (b) : ieieveÛegcyeer keâe meJe&Lee Megæ efJeueesce Meyo nw- OejeMeeÙeer~
(b) mLeeJej
21. DeeIeele keâe efJeueesce Meyo nw (c) mLeeveerÙe
(a) mebIeele (d) meeLe&keâ
(b) GodIeele Ans. (b) : JeekeäÙe kesâ efueS Skeâ Meyo-
(c) ØeefleIeele
• pees Skeâ mLeeve mes otmejs mLeeve hej ve ues peeÙee pee mekesâ- mLeeJej
(d) yeueeIeele
• peneB Yeespeve cegheäle ceW efceuelee nw/efvelÙe oerve-ogefKeÙeeW keâes Yeespeve osves
Ans. (c) : Ieele/DeeIeele keâe efJeueesce ‘ØeefleIeele’ nw~ keâer JÙeJemLee - meoeJele&
22. efvecveefueefKele ceWs mes Skeâ Meyo keâer Jele&veer DeMegæ nw 28. ‘yeeBkeâe’ keâe lelmece Meyo nw
(a) nmleefueefKele (a) efleÙe&keâ
(b) kegâcegefoveer (b) Je›eâ
(c) DeepeerJekeâe (c) Ûeeketâ
(d) melÙee«en (d) JelÙe&
Ans. (c) : Jele&veer keâer Âef° mes ‘DeepeerJekeâe’ Meyo DeMegæ nw~ Fmekeâe Ans. (b) : ‘yeeBkeâe’ keâe lelmece ‘Je›eâ’ nw~
Megæ ™he ‘DeepeerefJekeâe’ nw~ Mes<e leerveeW Meyo ›eâceMe: nmleefueefKele, 29. efvecveefueefKele MeyoeW ceW mes keâewve-mee Meyo lelmece veneR nw?
kegâcegefoveer Deewj melÙee«en Megæ nw~ (a) heleve
23. keâewve-mee Meyo Jele&veer keâer Âef° mes Megæ nw? (b) hevemeejer
(a) efvejnbkeâejer (c) efheškeâ
(b) hewef$ekeâ (d) Øenej
(c) pee«ele Ans. (b) : efoÙes ieÙes efJekeâuheeW ceW mes ‘hevemeejer’ Meyo lelmece veneR nw~
(d) efveoÙeer leodYeJe lelmece
Ans. (a) : ‘efvejnbkeâejer’ Meyo Megæ nw~ efJekeâuhe kesâ Mes<e Meyo heÌ[vee - heleve
DeMegæ nQ Fvekeâe Megæ ™he ›eâceMe: hewle=keâ, peeie=le Deewj efveo&Ùeer nw~ hevemeejer - heCÙeMeeefuekeâ
24. Megæ JeekeäÙe efÛeefÖle keâjW~ efhešeje - efheškeâ
(a) Meeskeâ nw efkeâ Deeheves cesjs he$eeW keâe keâesF& Gòej veneR efoÙee~ henj - Øenj
(b) ogKe nw efkeâ Deeheves cesjs he$e keâe Gòej veneR efoÙee~ 30. FveceW mes leodYeJe-lelmece keâe Skeâ Meyo Ùegice nw
(c) #eesYe nw efkeâ Deeheves cesjs he$e keâe mener Gòej veneR efoÙee~ (a) keâewÌ[er - keâheeefo&keâer
(d) Keso nw efkeâ Deeheves cesjs he$eeW keâe keâesF& Gòej veneR efoÙee~ (b) Deheeefnpe - Deeheeo nmle
Ans. (d) : Øemlegle efJekeâuheeW ceW mes Megæ JeekeäÙe nw- Keso nw efkeâ Deeheves (c) Deesme - DeJeMÙeeÙe
cesjs he$eeW keâe keâesF& Gòej veneR efoÙee~ Mes<e leerveeW efJekeâuhe DeMegæ nw~ (d) iesnBt - ieesntce
25. ‘pees peeslee-yeesÙee ve ieÙee nes’- Fme JeekeäÙeebMe kesâ efueS Ans. (c) : efJeJejCe efvecveefueefKele nw-
Skeâ Meyo nw leodYeJe - lelmece
(a) Dekeâef<e&le keâewÌ[er - keâheefo&keâe
(b) Deke=âef<ele Deheeefnpe - Deheeonmle
(c) Tmej Deesme - DeJeMÙeeÙe
(d) Dekeâesj iesntB - ieesOetce
UP RO/ARO (Mains) Exam Hindi 2016 61 YCT
CLICK HERE FOR FREE MATERIAL

UPPSC RO-ARO (Pre) Re-Exam-2016


GENERAL STUDIES
Solved Paper [ Exam Date : 20-09-2020

1. The "Oceanic National Park" of India is 6. Which of the following systems of rocks
located in : provides maximum amount of coal in India?
(a) Sunderban (b) Chilka Lake (a) Dharwad system (b) Gondawana system
(c) Nicobar Islands (d) Kutch (c) Vindhyan system (d) Tertiary system
Ans. (d) : The Oceanic National Park of India is located Ans. (b) : In the given rocks system Gondwana rock
in the Gulf of kutch in Jamnagar district of Gujarat. system contains nearly 98% of India's coal reserves.
2. India's first semi high speed train "Vande 7. With reference to the U.S. President's visit to
Bharat" runs between : India in February. 2020, which of the following
statements is/are correct?
(a) Varanasi to New Delhi
1. U.S. first lady Melania Trump visited a
(b) New Delhi to Jammu yoga session and interacted with students
(c) New Delhi to Mumbai in a Delhi School.
(d) New Delhi to Lucknow 2. The School visited by Melania Trump is
Ans. (a) : India's first semi high speed train called - co-educational senior secondary school in
"Vande-Bharat' (formerly known as Train-18) runs Moti Bagh, Delhi.
between Varanasi to New Delhi. Select the correct answer using the codes given
3. In which of the following cities of India, the below :
first synthetic rubber plant of the country was Codes
set up? (a) Only 1 (b) Only 2
(a) Panipat (b) Sonipat (c) Both 1 and 2 (d) Neither 1 nor 2
(c) Chandigarh (d) Lucknow Ans. (c) : During the visit of U.S President in February
2020, The first lady Melania Trump visited the school
Ans. (a) : The first synthetic rubber plant of the country named as" Co-educational senior secondary school" in
was set up at Panipat in Haryana by Indian Synthetic Moti Bagh, Delhi and also she visited a yoga session
Rubber Limited (ISRL). It is a Joint venture of Indian and interacted with students in a Delhi school Hence
Oil, TSRC Corporation Taiwan and Marubeni both statements are correct.
Corporation Japan.
8. Regarding Ram Mandir Trust formed in Feb.
4. Which is the biggest refinery in India? 2020, match the list- I with list-II and select the
(a) Jam Nagar (b) Paradweep correct answer from the codes given below :
(c) Digboi (d) Tatipaka List (Name) List
Ans. (a) : The biggest refinery in India is located at (Position)
Jamnagar, Gujarat. The Jamnagar refinery is a private A. Mahant Nritya 1. Head of the
sector crude oil refinery owned by Reliance. It was Gopal Das Temple
commissioned In 1999 with an installed capacity of 27 construction
MMTPA(Million Metric Tonne Per Annum) the present committee
capacity of this refinery is 33MMTPA. while the B. Swami Govind 2. General
present Capacities of Paradip, Digboi and Tatipaka Dev Giri secretary of
refineries are 15 MMTPA, 0.65 MMTPA and 0.07 Trust
MMTPA respectively. C. Shri Champat 3. Treasurer of the
5. Which of the following state has longest coast Rai Trust
line of India? D. Shri Nripendra 4. President of Ram
(a) Andhra Pradesh (b) Gujarat Mishra Mandir Trust
(c) Tamil Nadu (d) Kerala Code:
Ans. (b) : The coastline distributed among a states and A B C D
4 union territories (UTs), of India Among the states, (a) 1 2 3 4
Gujarat has the longest coastline comprises 1214. 7 km (b) 4 3 2 1
followed by Andhra Pradesh (973.7 Km) and Tamil (c) 2 4 1 3
Nadu (906.9 Km). (d) 3 2 4 1
UP RO/ARO (Pre) Exam-2016 (Ex. Dt. 20.9.2020) 62 YCT
CLICK HERE FOR FREE MATERIAL

Ans. (b) : The correct match is- 13. "International Gandhi Award 2020" for the
Personality Position efforts against leprosy in the individual
Mahant Nritya Gopal Das President of Ram category has been awarded to :
Mandir Trust (a) Yohei Sasakawa
Swami Govind Dev Giri Treasurer of the (b) N.S. Dharmashakatu
Trust (c) Saty Nadella
Shri Champat Rai General secretary of (d) Damodar Ganesh Bapat
Trust Ans. (b) : International Gandhi Award-2020 for the
Shri Nripendra Mishra Head of the Temple effort against leprosy in the individual category has
construction been awarded to N.S. Dharmashakatu.
committee
14. Choose the correct option from the following :
9. Which of the following was conferred "Oscar 63 : 9 : : ? : 14
Award 2020" for Best Film? (a) 68 (b) 42
(a) Avenger (b) Gully Boy
(c) 96 (d) 56
(c) Parasite (d) Joker
Ans. (a) : Given 63: 9 : : ? : 14
Ans. (c) : The film 'Parasite' was conferred Oscar
Award-2020 for best film-and the winner of Oscar 63 → 9
Award 2021 was Nomadland for best film. (6+3) → 9
10. Where did the first "Khelo India University" Similarly
games take place? ? → 14
(a) Bhubaneswar (b) New Delhi (6+8) → 14
(c) Pune (d) Kolkata Hence option (a) is correct.
Ans. (a) : Khelo India university Games is a national 15. Choose the correct pair in which the words
level multi sport event held in India. The first event was bear the same relationship to each other as the
held in 2020 at Bhubaneswar, Odisha. The 2022 edition words given below pair :
of the Khelo India University is to be hosted by Dinosaur : Dragon : : ?
banglore's Jain University situated at Karnataka state. (a) Evolution : Revelation
11. Given below are two statements one labelled as (b) Gorilla : Soldier
Assertion (A) and the other as Reason (R) : (c) Snow : Ice
Assertion (A) : President Donald Trump was (d) Ancient : Medieval
acquitted by the Senate in impeachment trial
Ans. (c) : As Dinosaur is related to Dragon similarly in
against him in February 2020.
the given option Snow is related to Ice.
Reason (R) : Democrats have majority over
republicans in the senate. 16. Which number will come next in the following
Select the correct answer from the codes given series ?
below : 8, 15, 28, 53, ?
Codes: (a) 98 (b) 106
(a) Both (A) and (R) are true and (R) is the (c) 100 (d) 102
correct explanation of (A) Ans. (d) : Given
(b) Both (A) and (R) are true and (R) is not the 8, 15, 28, 53, –?–
correct explanation of (A)
(c) (A) is true but (R) is false
(d) (A) is false but (R) is true
Ans. (c) : President Donald Trump was acquitted by the
senate in impeachment trial against him in February Hence option (d) is correct answer.
2020. He belongs to Republican Party and at that time 17. In the following question, a matrix of certain
Republicans have majority over Democrats. Hence character is given. These character follow a
Assertion is correct while reason is false. certain trend, rowwise or columnwise. Find out
12. How many teams participated in ICC Women's the missing character accordingly.
T-20 World Cup that was held during 18 20 26
February – March 2020 in Australia?
5 15 11
(a) 08 (b) 10
(c) 12 (d) 14 22 40 38
Ans. (b) : The ICC women's T-20 world cup 2020 was 9 ? 13
held in Australia. In this tournament total 10 teams (a) 18 (b) 12
participated. (c) 10 (d) 14
UP RO/ARO (Pre) Re Exam 2016 (Ex. Dt. 20.9.2020) 63 YCT
CLICK HERE FOR FREE MATERIAL

Ans. (c) : Given, Ans. (b) : As per census report 2011 the district
Just as, Shravasti has lowest literacy rate (46.741) in U.P
I Coloumn = 5 + (2 + 2) + 9 = 18 followed by Bahraich (49.36) and Balrampur (49.57%)
III Coloumn = (1 + 1) + (3 + 8) + 13 = 26 while the district Gautam Buddha Nagar have highest
Similarly – literacy rate (80.12%) among in districts of U.P
II Coloumn = (1 + 5) + (4 + 0) + x = 20 followed by Kanpur Nagar (79.65) and Auraiya
6 + 4 + x = 20 (78.95%)
x = 10 23. What is the percentage of canal irrigation of
18. Mohit went 15 km to the West from his house, the total irrigated area in the state of Uttar
then turned left and walked 20 km, then he Pradesh?
turned East and walked 25 km and finally (a) 30 percent (b) 29 percent
turning left covered 20 km. How far was he (c) 28 percent (d) 27 percent
from his house? Ans. (a) : The total irrigated area under canal is about
(a) 5 km (b) 10 km 30% to the total irrigated land in the Uttar Pradesh.
(c) 40 km (d) 80 km 24. Match the List–I with List–II and select the
Ans. (b) : His home is at point O. correct answer from the codes given below :
List List
A. Brassware i. Meerut
B. Match Box ii. Saharanpur
C. Sports Goods iii. Moradabad
D. Wood Carving iv. Barailly
Codes:
Hence finally he was 10 km far from his house. A B C D A B C D
19. Which one of the following number is different (a) iv iii ii i (b) iii iv i ii
from other three? (c) ii i iii iv (d) i ii iv iii
(a) 94 (b) 67 Ans. (b) : The correct match is-
(c) 85 (d) 13 Items Place
Ans. (d) : In the given numbers number '13' is different Brassware Moradabad
from others because of the sum of digit of number 13 is Match Box Barailly
4, while other given numbers give digit sum as 13.
Sports Goods Meerut
94 = 9 + 4 = 13
Wood Carving Saharanpur
67 = 6 + 7 = 13
85 = 8 + 5 = 13 25. The main folklores of Uttar Pradesh are :
13 = 1 + 3 = 4 (a) Darun, Awadhi, Braj
(b) Karuna, Bhakti, Shaira
20. 'A' is brother of 'B', 'C' is married to 'D'. If 'C'
is the nephew of 'B', then what is the (c) Drupad, Braj, Karuna
relationship of 'A' with 'D'? (d) Birha, Chaiti, Dhola
(a) Father in law (b) Mother in law Ans. (d) : In the given options Birha, Chaiti and Dhola
(c) Son in law (d) Daughter in law are the main folklores of Uttar Pradesh Chaiti and Birha
Ans. (a) : are popular in Parvanchal region of U.P. while Dhola
is popular songs of western U.P.
26. Which are the major folk dances of Uttar
Pradesh?
(a) Bihu, Ghoomar, Garba
21. In which year "Kisan Samridhi Aayog" was (b) Charkula, Karma, Pandav
established in Uttar Pradesh? (c) Rau Nacha, Bhangra, Rouff
(a) 2016 (b) 2017 (d) Lavani, Kathak, Kathakali
(c) 2018 (d) 2019 Ans. (b) : In the given option Charkula, Karma and
Ans. (b) : Kisan Samridhi Aayog was established in Pandav are the major folk dances of U.P. In Charkula
Uttar Pradesh in the year of 2017, the aim of this dance, a veild woman dancer performs with 108 oil
commission was to double the income of farmers by lamps placed on her head and it is a folk dance of Braj
2022. The chief Minister of the state acts as the region.
Chairman of this commission. Karma is performed jointly by women and men of Kol
22. In which of the following district literacy rate is tribes and it is prevalent in Mirzapur and Sonbhadra
the lowest? districts.
(a) Bulandshahar (b) Shravasti Pandav dedicated to themes depend on Mahabharat
(c) Kushinagar (d) Unnao epic and it is prevalent in northern parts of U.P.
UP RO/ARO (Pre) Re Exam 2016 (Ex. Dt. 20.9.2020) 64 YCT
CLICK HERE FOR FREE MATERIAL

27. Trade Facilitation Centre and Craft Museum is that state, and the executive power of the Union' Shall
established in the district of Uttar Pradesh : extend to the giving of such direction to a state as may
(a) Prayagraj (b) Varanasi appear to the Government Article 257 carries the
(c) Ayodhya (d) Amethi heading 'Control of The basis of the constitution of the
Ans. (b) : Trade facilitation Centre and craft Museum country. Hence, Both A and R are true and R is the
located at Varanasi in U.P. was established in 2017 as a correct explanation of A
modern museum that showcases the traditional 31. Match the List–I with List–II and select the
handloom and handicrafts of Varanasi. correct answer from the codes given below :
28. The IFFCO urea plant is established in which List–I List–II
of the following place of U.P.? A. 7th Schedule 1. Languages
(a) Unchahar (b) Aonla B. 8th Schedule 2. Disqualification on
(c) Jamshedpur (d) Jagdishpur the ground of
Ans. (b) : IFFCO stands for Indian Farmers Fertilizer defection
Cooperative Limited. IFFCO urea plant is established at C. 9th Schedule 3. Union, State and
Aonla in Bareilly districts of U.P. Concurrent lists
29. Which one among the following is NOT D. 10th 4. Validation of
correctly matched? Schedule certain Acts
Provision Part of the Codes:
constitution A B C D A B C D
(a) Citizenship – Part–II (a) 4 2 1 3 (b) 3 1 2 4
(b) The Union Territories – Part–VII (c) 2 3 4 1 (d) 3 1 4 2
(c) The Municipalities – Part–IX A Ans. (d) : The Correct match is-
(d) Elections – Part–XV Schedules Features of Schedules
Ans. (b) : In the given among options, (b) is 7th Schedule This schedule deals with the three
legislative lists:- Union List, state
not correctly matched because the Union
List and Concurrent List.
Territories comes under part VIII of the
constitution. 8th Schedule It deals with the 22 official languages
recognized by the Indian
Part Provision Articles constitution.
Part II Citizenship 5-11 9th Schedule It deals with the state acts and
Part VIII The Union 239-241 regulations that deals with land
Territories reforms and the abolition of the
Part IX- The Municipalities 243P- zamindari system.
th
A 243ZG 10 Schedule It contains provisions relating to the
Pat XV Elections 324-329A disqualification of the members of
Parliament and state Legislatures on
30. Given below are two statements, one labelled as the ground of defection.
Assertion (A) and the other as Reason (R) :
32. Which among the following is NOT correctly
Assertion (A) : The executive powers of the state matched?
are exercised to ensure the compliance of the law
Bodies Establishment
of the Parliament and the laws applied in the state.
(a) Inter-state Council – 1990
Reason (R) : The government of India can give
necessary direction to the states by using its (b) National – 1954
executive powers. Development Council
Select the correct answer from the codes given (c) Central Vigilance – 1964
below : commission
Codes: (d) National Human – 1993
Rights Commission
(a) Both A and R are true and R is the correct
explanation of A Ans. (b): National Development Council was set up in
(b) Both A and R are true and R is not the correct 1952. It is an apex body for decisions and deliberations
explanation of A on development matters in India. It is presided by Prime
Minister. It is neither a constitutional body nor a
(c) A is true but R is false
statutory body.
(d) A is false but R is true
33. Given below are two statements, one labelled as
Ans. (a) : Article 256 of Indian Constitution carries the Assertion (A) and the other as Reason (R) :
heading 'obligation of states and the Union' which Assertion (A) : When the payments made by a
provides that "The executive power of every" 'state shall country exceeds its receipts from trade of goods,
be so exercised as to ensure compliance with the laws services, transfer and net income is called Current
made by parliament and existing laws which apply in Account Deficit (CAD)
UP RO/ARO (Pre) Re Exam 2016 (Ex. Dt. 20.9.2020) 65 YCT
CLICK HERE FOR FREE MATERIAL

Reason (R) : Current Account Deficit (CAD) 36. Given below are two statements, one labelled as
occurs when a country exports more goods, Assertion (A) and the other as Reason (R) :
services and capital. Assertion (A) : For education and culture a
Choose the correct answer from the codes Centre for Cultural Resources and Training
given below : (CCRT) was set up in 1979.
Codes: Reason (R) : The objective of CCRT was to link
(a) Both A and R are true and R is the correct education with culture.
explanation of A Choose the correct answer from the codes
given below :
(b) Both A and R are true and R is not the correct
explanation of A Codes:
(c) A is true but R is false (a) Both A and R are true and R is the correct
(d) A is false but R is true explanation of A
(b) Both A and R are true and R is not the correct
Ans. (c) : R is false because the Current Account explanation of A
Deficit occurs if the value of the goods and services (c) A is true but R is false
imported exceeds the value of those exported.
(d) A is false but R is true
The current account records exports and imports in
goods and services and transfer of payment. Ans. (a) : Centre for Cultural Resources and Training is
an Autonomous organisation under the ministry of
Current Account = Trade Gap +Net current transfers + culture. It was established in 1979 with an aim to link
Net income abroad. (Trade Gap = Exports -Imports) education with culture. It was pioneered by Smt.
34. Which of the following statement about green Kamaldevi chattopadhyay and Dr. Kapila Vatsyayan.
bond is not true? 37. "Wangla Festival" is celebrated in the
(a) Green Bond investment is only for climate following state :
friendly projects (a) Mizoram (b) Meghalaya
(b) Green bonds were first introduced by (c) Manipur (d) Tripura
European Investment fund in 2007 Ans. (b) : Wangala Festival of Meghalaya is a most
(c) Green Bonds are Financial Market Innovation popular festival among the Garo tribes of Meghalaya,
(d) Green Bonds are fixed interest loan is short India. Wangala Festival is a harvest festival held in
date maturities honor of Misi Saljong, the sun- god, for blessing the
Ans. (d) : Green Bonds are Fixed interest bonds with people with a rich harvest.
long term maturities. Green Bond is a debt instrument 38. An ultimate source of energy stored in fossil
issued by an issuer for raising funds for green projects. fuels is :
It was first issued in 2007 by the world Bank and (a) Earth (b) Sun
European Investment Bank. (c) Sea (d) Moon
35. Match the following Defence Expo and their Ans. (b) : Sun is the ultimate source of energy of fossil
venues in different years fuels. fossil fuels are nothing but decomposed plant and
Years Venue animal matter which under the action of intense
pressure and temperatures over millions of years
A. 2016 I. Chennai produced coal, oil etc.
B. 2018 II. South Goa 39. Driving force of an ecosystem is :
C. 2019 III. Lucknow (a) Carbohydrate (b) Biomass
D. 2020 IV. Hydrabad (c) Carbon (d) Solar energy
Select the correct answer from the codes given Ans. (d) : The energy obtained by the sun is called solar
below : energy. It is the driving force of the ecosystem. Solar
Codes: energy is directly utilised by plants to prepare their food.
A B C D 40. Which one of the following is essential in
corrosion of iron metal?
(a) I II IV III
(a) Oxygen only
(b) II I IV III (b) Oxygen and moisture
(c) IV III I II (c) Hydrogen only
(d) II IV I III (d) Hydrogen and moisture
Ans. (b) : The venue of different Defense Expo are as Ans. (b) : The reaction of iron and oxygen forms an
follows- iron oxide which is a reddish- brown oxide. This is
Years Venue called Rust or corrosion of iron metal.
2016 South Goa 41. Which of the following is a naturally occurring
2018 Chennai hardest substance on the earth?
2019 Hyderabad (a) Graphite (b) Wurtzite boron nitride
2020 Lucknow (c) Iron (d) Diamond
UP RO/ARO (Pre) Re Exam 2016 (Ex. Dt. 20.9.2020) 66 YCT
CLICK HERE FOR FREE MATERIAL

Ans. (d) : Diamond is the hardest naturally occurring 47. Consider the following events :
substance found on earth. Diamonds also bend and 1. Clive re-arrival in India
reflect light beam and break it up into the colours of the 2. Treaty of Allahabad
rainbow, which gives it sparkle. 3. Battle of Buxar
42. Which one of the following instrument is used 4. Warren Hastings became India's
for locating submerged objects in the ocean? Governor
(a) Audio meter (b) Galvanometer Select the correct chronological order of the
(c) Sextant (d) Sonar above events from the codes given below :
Ans. (d) : Sonar is a technique that uses sound Codes:
propagation to navigate, communicate with or to detect
objects on or under the surface of the water. (a) 3, 2, 1, 4 (b) 3, 1, 2, 4
43. "Ramsar Convention" is related to : (c) 1, 2, 3, 4 (d) 2, 1, 4, 3
(a) Climate change Ans. (b) :
(b) Pesticide pollution Event Year
(c) Ozone layer depletion Battle of Buxar 1764 Fought between British
(d) Wetland conservation and Mir Qasim
Ans. (d) : Ramsar Convention related to wetland Robert Clive's re- 1765-67, Robert Clive was
conservation. The convention on wetlands is an arrival in India the Governor of Bengal from
intergovernmental treaty that provides the frame work 1757-60 and From 1765-67
for the conservation and wise use of wetlands.
Treaty of Allahabad 1765
44. Match List–I with List–II and select the correct
answer using the codes given below : Warren Hastings 1773 to 1785
List–I List–II 48. Match the List–I with List–II and select the
A. Vitamin K 1. Wheat germ oil correct answer from the codes given below :
B. Vitamin D 2. Lemon List–I List–II
C. Vitamin E 3. Cod-liver oil A. Treaty of Allahabad 1. 1954
D. Vitamin C 4. Alfa-alfa B. Treaty of Aix -la - 2. 1746
Codes: Chapelle
A B C D A B C D C. La Bourdannaise 3. 1748
(a) 1 2 3 4 (b) 3 1 4 2 Capture of Madras
(c) 2 3 4 1 (d) 4 3 1 2 D. Duplex's dismissal 4. 1765
Ans. (d) : The correct match is- Codes:
Vitamin Source A B C D A B C D
Vitamin K Alfa-alfa (a) 2 4 1 3 (b) 4 3 2 1
Vitamin D Cod-liver oil (c) 3 1 2 4 (d) 1 4 2 3
Vitamin E Wheat germ oil
Ans. (b) :
Vitamin C Lemon
Events Year
45. Who among the following founded the "Lathi
Treaty of Allahabad 1765
Club"?
(a) Bhagat Singh (b) Lala Lajpat Rai Treaty of Aix-la - Chapelle 1748
(c) Bal Gangadhar Tilak (d) Bipin Chandra Pal La Bourdannaise Capture of 1746
Ans. (c) : Bal Gangadhar Tilak is also known as Madras
Lokmanya Tilak. He founded Lathi Club to make the Duplex's dismissal 1954
youth brave and to enable them to fight for the freedom 49. "Rajm-nama" is the Persian translation of
of their country. which Hind text?
46. "I have no time think about God because a lot (a) Ramayana (b) Mahabashya
of work has to be done on this earth" whose (c) Mahabharata (d) Asthadhyayi
statement is above? Ans. (c) : The Mahabharata was translated into Persian
(a) Swami Vivekanand during Akbar's reign, by Faizi and Abdul-al-Qadir
(b) Swami Dayanand Saraswati Bada'uni and named Rajm-nama.
(c) Swami Ramakrishna Paramhans
50. Consider the following battles and arrange
(d) Swami Ishwarchandra Vidhyasagar
these in chronological order :
Ans. (d) : The above statement is given by Ishwar
I. Battle of Kannauj
Chandra Vidyasagar, He was a social reformer of 19th
Century, The focus of his social reform was women and he II. Battle of Chanderi
spent his life's energies trying to ensure and end to practice III. Battle of Talikota
of child marriage and initiate widow remarriage. IV. Battle of Chausa
UP RO/ARO (Pre) Re Exam 2016 (Ex. Dt. 20.9.2020) 67 YCT
CLICK HERE FOR FREE MATERIAL

Select the correct answer from the code given 53. Who was the compiler of the "Imperial
below : Gazetteer"?
Codes: (a) Colin Clark
(a) I, II, IV, III (b) II, IV, I, III (b) William Wilson Hunter
(c) IV, II, I, III (d) I, IV, II, III (c) Charles Elliott
Ans. (b) : (d) William Digly
Events Year Ans. (b) : William Wilson Hunter was the compiler of
Battle of Kannauj 1540 the Imperial Gazetteer. He was also a member of the
Battle of Chanderi 1528 Indian Civil service. He was a Scottish Historian and a
Battle of Talikota 1565 Compiler.
Battle of Chausa 1539 54. With reference to the establishment of
51. Arrange the following events in chronological Supreme Court, which of the statement is/are
order and select the correct answer from the correct?
codes given below : 1. Supreme Court at Calcutta was established
1. Gandhi-Irwin agreement by the Regulating Act of 1773
2. Poona Pact 2. Lemaister was the first Chief Justice of this
court
3. Declaration of complete National
Select the correct answer using codes given
Independence
below :
4. Civil Disobedience Campaign
Codes:
Codes : (a) 1 only (b) 2 only
(a) 3, 4, 1, 2 (b) 2, 1, 4, 3 (c) Both 1 and 2 (d) Neither 1 nor 2
(c) 2, 1, 3, 4 (d) 4, 3, 1, 2
Ans. (a) : The correct answer is 1 only. The supreme
Ans. (a) : court at Calcutta was established 1774 by the
Events Year Regulating Act of 1773. Sir Elijah Impey was the first
Gandhi Irwin Pact 1931 judge of this court.
Poona Pact 1932 55. Colour of the tag used on certified seed bags is :
Declaration of Complete (a) blue (b) purple
National Independence 1929 (c) white (d) golden yellow
Civil Disobedience Campaign 1930 Ans. (a) : The colours to Breeder seeds, Foundation
52. Math the List–I with List–II and select the seeds and certified seeds tags are as Follow:-
correct answer using codes given below : • The breeder seeds have a golden yellow tag.
List List • The foundation seeds have a white tag
A. Indian 1. Anand • The Registered seeds have a purple tag.
Association Charlu • The Certified seeds have a blue tag.
B. Poona Sarvjanik 2. Surendra 56. Which one of the following is correctly
Sabha Nath Benerji matched?
C. Bombay 3. Mahadev (a) Cooperative – M.K. Gandhi
Presidency Govind Movement
Association Ranade (b) Etawah Pilot Project – Albert Mayer
D. Madrash 4. Pherozshah (c) Grow more Food – J.L. Nehru
Mahajan Sabha Mehta Campaign
Codes: (d) Sewagram Project – Vinoba Bhave
A B C D A B C D Ans. (b) : Etawah Pilot Project was started by Albert
(a) 2 3 4 1 (b) 2 4 3 1 Mayer While Robert Owen is considered as the father
(c) 3 2 4 1 (d) 3 4 2 1 of Co-operative movement, Grow more food Campaign
was started by Britishers and Sewagram project was
Ans. (a) : The correct answer in A-2, B-3, C-4, D-1 started by M.K. Gandhi
Political Founder 57. First intercity electric bus service of the
organisation country under public-private partnership
Indian Association Surendranath Banerjee model in February 2020 have been started by
Poona Sarvajanik Mahadev Govind Ranade the state :
Sabha (a) Maharashtra (b) Karnataka
(c) Gujarat (d) Tamil Nadu
Bombay Presidency Pherozshah Mehta
Association Ans. (a) : The first intercity electric bus is between
Mumbai and Pune. Union transport minister Nitin
Madras Mahajana Anand Charlu Gadkari inaugurated the first intercity electric bus
Sabha service in February 2020.
UP RO/ARO (Pre) Re Exam 2016 (Ex. Dt. 20.9.2020) 68 YCT
CLICK HERE FOR FREE MATERIAL

58. Which of the following is NOT correctly (c) (A) is true but (R) is false
matched under highest crop cultivated area of (d) (A) is false but (R) is true
Uttar Pradesh? Ans. (a) : The correct answer is Both (A) and (R) are
Crop District true and (R) is the correct explanation of (A)
(a) Mustard – Agra Operation Twist' is when the central Bank uses the
(b) Potato – Farrukhabad proceeds from the sale of short-term societies to buy
long-term government debt papers, leading to easing of
(c) Maize – Varanasi
interest rebates on the long term papers. Hence assertion
(d) Sesame – Hamirpur (A) is correct.
Ans. (c): In the given options, (c) is not correctly matched. The action of 'Operation Twist' by the RBI is
Mainpuri is the largest producer of maize in the state. encouraging for the market. This step may become a
59. With reference to Deposit Insurance and driving factor for long-term economic activity and the
Credit Guarantee Insurance Corporation, addition of new investment stock. It helps in promoting
which of the following statements is/are correct? long term investment.
1. A subsidiary of Reserve Bank of India Hence Reason (R) is correct and (R) is the correct
2. Deposite upto Rs. 5 lakh are insured by it explanation of (A).
Select the correct answer using the codes given 62. Which of the following is a social objective of
below : business?
Codes: (a) Innovation
(a) Only 1 (b) Only 2 (b) Employment generation
(c) Both 1 and 2 (d) Neither 1 nor 2 (c) Business coordination
(d) Risk management
Ans. (c) : Deposit Insurance and Credit Guarantee
Corporation (DICGC) is a wholly-owned subsidiary of Ans. (b) : The social objective of a business is to
the Reserve Bank of India, and a maximum of Rs. provide employment to society. With the development
500000 (after the budget of 2020-21) is insured for each of business, the need for employees arises. Thus, the
user for both principal and interest amount. business needs the services of good employees. So, the
business provides employment to society.
60. Match List–I with List–II and select the correct
answer from the codes given below : 63. According to 2011 census among the following
states, which one has the lowest level of
List–I (Bank) List–II (Type) urbanisation?
A. Indian Bank 1. Foreign (a) Andhra Pradesh (b) Haryana
B. ICICI Bank 2. Cooperative (c) Mizoram (d) West Bengal
C. CITI Bank 3. Private Ans. (d) : The correct answer is West Bengal.
D. Saraswat bank 4. Public State/UT Urban Pop (%)
Codes: West Bengal 31.89
A B C D A B C D Andhra Pradesh 33.49
(a) 1 2 3 4 (b) 2 3 1 4 Mizoram 51.51
(c) 4 2 3 1 (d) 4 3 1 2 Haryana 34.79
Ans. (d) : The correct match is- 64. In which of the following state/union
Bank Type of Bank territories, the "Jarawa Tribe" resides?
Indian Bank Public (a) Arunanchal Pradesh
ICICI Bank Private (b) Lakshdweep
(c) Chhattisgarh
CITI Bank Foreign
(d) Andman and Nicobar
Saraswat Bank Cooperative
Ans. (d) : The correct answer is Andaman and Nicobar.
61. Given below are two statements, one labelled as The Four major tribes of Andaman are as follows:
the Assertion (A) and other as Reason (R). Jarawa, Great Andamanese, Sentinelese and Onge.
Assertion (A) : Under Operation Twist, RBI 65. Match the List-I with List-II and select the
simultaneously sell short term securities and buys correct answer from the codes given below.
long term securities. List-I (Mining area) List–II (Mineral)
Reason (R) : The main objective of this operation
A. Guru Mahisani 1. Zinc
is to promote long term investment.
Select the correct answer from the codes given B. Talcher 2. Uranium
below : C. Jaduguda 3. Iron-Ore
Codes: D. Zawar 4. Coal
(a) Both (A) and (R) are true and (R) is the Codes:
correct explanation of (A) A B C D A B C D
(b) Both (A) and (R) are true and (R) is not the (a) 1 2 3 4 (b) 2 4 3 1
correct explanation of (A) (c) 3 4 2 1 (d) 3 2 1 4
UP RO/ARO (Pre) Re Exam 2016 (Ex. Dt. 20.9.2020) 69 YCT
CLICK HERE FOR FREE MATERIAL

Ans. (c) : The correct match is- Ans. (d) : The correct match is-
Mining Area Mineral River City
Guru Mahisani Iron-ore Shatt-al-Arab Basra
Talchir Coal Paraguay Asuncion
Jaduguda Uranium Niger Niamey
Zawar Zinc Danube Vienna
66. Monsoon originates by which of the following 70. Newly introduced Prerak Dauur Samman by
winds in India? Government of India in July 2020 is related to
(a) South–West winds (b) South–East winds which of the following?
(c) North–East winds (d) North–West winds (a) Crime status of city
Ans. (a) : The term monsoon has been derived from the (b) Electricity supply in city
Arabic word Mausim meaning 'Season'. India receives (c) Waste management and sanitation status of
south-west monsoon winds in summer and north-east city
monsoon winds in winter. (d) Vegetation cover in city
67. Tropic of cancer does NOT cross which of the Ans. (c) : The Prerak Dauur Samman is related to waste
following state? management and sanitation status of city. It was
(a) Tripura (b) Chhattisgarh introduced in July 2020 by Shri Harideep Singh Puri.
(c) Manipur (d) Mizoram 71. In which state of India, the scheduled caste
Ans. (c) : In India Tropic of Cancer passes through population is zero as per census 2011?
eight state namely Rajasthan, Gujarat, Madhya Pradesh, (a) Kerala (b) Nagaland
Chhattisgarh, Jharkhand, West Bengal, Tripura and (c) Sikkim (d) Assam
Mizoram. It does not pass through the state of Manipur. Ans. (b) : The correct answer is Nagaland.
68. Match the List-I with List-II and select the State/UT Proportion of SC Proportion of ST
correct answer from the codes given below. Population Population
List–I (Country) List–II (Capital) Nagaland 0.0 89.1
A. Myanmar 1. Hanoi Kerala 9.8 1.1
B. Cambodia 2. Vientiane Sikkim 5.0 20.6
C. Vietnam 3. Phnom Penh Assam 6.9 12.4
D. Laos 4. Yangoon 72. 'Sankalp Parva' celebrated by Government of
Codes: India during June–July 2020 is related to :
A B C D A B C D (a) Tree plantation
(a) 1 2 3 4 (b) 2 3 4 1 (b) COVID lockdown
(c) 4 3 1 2 (d) 3 4 2 1 (c) Dedication towards army
(d) Online education
Ans. (c) : The correct match is-
Ans. (a) : The Indian Prime Minister Narendra Modi
Country Capital
called for all employees to plant at least five trees in
Myanmar Yangoon their office campus or wherever possible, to ensure a
Cambodia Phnom Penh clean and healthy environment of the country.
Vietnam Hanoi 73. "Blue Revolution" is related with the following :
Laos Vientiane (a) Food grain production
69. Match the List-I with List-II and select the (b) Oil seed production
correct answer from the codes given below. (c) Fish production
List–I (River) List–II (City) (d) Milk production
A. Shatt-al-Arab 1. Vienna Ans. (c) : Blue Revolution was launched in 1985-90
during the 7th Five year Plan. The main objective is to
B. Paraguay 2. Basra develop manage and promote fisheries Industries.
C. Niger 3. Asuncion 74. Which of the following rivers the "Chitrakote"
D. Danube 4. Niamey water fall is located?
Codes: (a) Yamuna River (b) Mandakini River
A B C D A B C D (c) Indravati River (d) Narmada River
(a) 1 2 3 4 (b) 2 1 4 3 Ans. (c) : The Chitrakote Falls is situated on the
(c) 3 4 1 2 (d) 2 3 4 1 Indrāvati River. It is located in Chhattisgarh.
UP RO/ARO (Pre) Re Exam 2016 (Ex. Dt. 20.9.2020) 70 YCT
CLICK HERE FOR FREE MATERIAL

75. The Kumaun Himalaya is situated between Ans. (c) : International Mother Language Day is
which of the rivers? observed on 21st February. It is celebrated to spread
(a) Indus and Satluj awareness about the importance of linguistic, cultural
(b) Kali and Tista diversity and multilingualism.
(c) Satluj and Kali 82. "Operation Vanilla" launched by Indian Navy
(d) Tista and Brahmputra in January 2020, is related to which on of the
Ans. (c) : The Kumaun Himalayas lie in Uttarakhand following?
and extend from the satluj to the kali River. (a) Disaster Management
76. The National Afforestation and Eco (b) Oil Exploration
Development Board (NAEB) was set up by (c) Anti-Terrorist operation
Government of India in the year
(d) Coral Reef Conservation
(a) 1987 (b) 1992
(c) 1995 (d) 1998 Ans. (a) : In January 2020, the Indian Navy launched
'Operation Vanilla' to provide assistance to the cyclone-
Ans. (b) : The National Afforestation and ECO-
Development Board (NAEB) was set up in August affected population of Madagascar. Hence option (a) is
1992. It is responsible for promoting afforestation, tree the correct answer.
planting, ecological restoration and eco-development 83. Joint Military training exercise "Ajeya
activities in the country. Warior- 2020" was carried out between which
77. Which one of the following was the theme of of the following countries?
statistics Day celebrated on 29th June 2020? (a) India and United Kingdom
(a) Statistics in Trade (b) India and Israel
(b) COVID–19 (c) India and France
(c) Statistics in Governance (d) India and United States of America
(d) Sustainable Development Goals Ans. (a) : Exercise Ajeya Warior is a joint exercise
Ans. (d) : End Hunger, Achieve Food Security and conducted by the armed forces of India and the United
Improved Nutrition and promote "Sustainable Kingdom.
Development Goal" or SDG 2 of the UN is the theme of
this years National statistics Day. 84. With reference to "Ek Bharat Shrestha
Bharat" which of the following statements
78. Which Bollywood actress received the is/are correct?
"Padmashri" Award in the year 2020?
1. This campaign was carried from 10th to
(a) Taapsee Pannu (b) Shraddha Kapoor
28th February 2020
(c) Kangna Ranaut (d) Bhumi Pednekar
2. The objective is to promote the sprit of
Ans. (c) : Kangna Ranaut received the 'Padmashri'
National integration through various
Award for the Year 2022 .
cultural activities
79. In January 2020, how many "Padma Select the correct answer from the code given
Vibhushan" were conferred Posthumously?
below.
(a) 4 (b) 3
Codes:
(c) 2 (d) 1
(a) 1 only (b) 2 only
Ans. (a) : In January 2020, 4 'Padma Vibhushan' were
conferred Posthumously. These Personalities are - (c) both 1 and 2 (d) Neither 1 nor 2
(i) George Fernandes Ans. (c) : 'Ek Bharat Shrestha Bharat' Campaign has
(ii) Arun Jaitley been launched to promote cultural unity among various
(iii) Sushma Swaraj states of the country.
(iv) Vishwesha Tirtha 85. If CARPET is coded as TCEAPR, then the
80. Who has been honoured by the International code for NATIONAL would be :
Olympic Committee with the "Coaches Life (a) NLATNOIA (b) LANOITAN
Time Achievement Award 2019"? (c) LNAANTOI (d) LNOINTAA
(a) Ravi Shastri (b) Pullela Gopichand Ans. (c) : CARPET
(c) Prakash Padukone (d) Sunil Gavaskar TCEAPR
Ans. (b) : Top Indian badminton coach, Pullela Gopichand Here letters of the basic word are written in the coded
was conferred International Olympic Committee's Coaches word in such a way that last and first letters, second last
Lifetime Achievement Awards 2019 for his excellence and and second letters, third last and third letters and so on
contribution in the field of Coaching.
are written together in the coded word.
81. International Mother Language Day is
celebrated on : Then,
(a) March 20 (b) January 25 N A T I O N A L
(c) February 21 (d) December 21 L N A A N T O I
UP RO/ARO (Pre) Re Exam 2016 (Ex. Dt. 20.9.2020) 71 YCT
CLICK HERE FOR FREE MATERIAL

86. A train is running 3 minutes late and further it (a) 72 (b) 84


is getting delayed by 3 seconds per minute. (c) 68 (d) 60
After how many minutes this train will be Ans. (b) : The pattern Followed is,
delayed one hour?
(a) 1140 minute (b) 1150 minute
(c) 1160 minute (d) 1800 minute
Ans. (a) : Given,
Train is running 3 minutes late.
So, 60-3 = 57 minutes.
Train loses 3 seconds per minute = 3/60
Hence it loses 1 minute in ⇒ 1/(3/60) = 60/3 = 20 minutes ∵ 7 × 2 + 4 = 18
Thus, 57 minutes will be lost in 57×20 minutes 18 × 2 + 4 = 40
= 1140 minutes 40 × 2 + 4 = 84
87. In a certain code language if HE = 41 and SHE 84 × 2 + 4 = 172
= 49, then THEM will be equal to : 172 × 2 + 4 = 348
(a) 62 (b) 90 Hence '84' is the correct answer.
(c) 64 (d) 56 91. How many capital letters of English alphabets
Ans. (a) : The Pattern Followed is, have same mirror image?
Letter H E (a) 9 (b) 10
Place value from the Reverse side 19 22 (c) 11 (d) 12
HE = 22 +19 = 41 and Ans. (c) : The letters which appear the same after the
Letter S H E mirror image are-
Place value from the reverse side 8 19 22 A, H, I, M, O, T, U, V, W, X, Y.
Hence, 11 letters are in English Alphabets which have
SHE = 8 + 19 +22 = 49 same mirror image.
Similarly, 92. Where is "Leather Technology Park" situated?
Letter T H E M (a) Agra (b) Kanpur
Place value from the reverse 7 19 22 14 (c) Unnao (d) Noida
side Ans. (c) : Leather Technology Park is situated at Unnao
THEM = 7 + 19 + 22 + 14 = 62 in U.P.
Hence, 62 is the correct answer. 93. When was the State Higher Education council
88. If 3rd December 2000 was Sunday, then what established in U.P.?
was the day of 3rd January 2001? (a) 1990 (b) 1995
(a) Tuesday (b) Wednesday (c) 2000 (d) 2005
(c) Thursday (d) Friday Ans. (b) : In U.P. the state Higher Education council
Ans. (b) : Given, was established in the year 1995. On the
3rd December 2000 was Sunday. recommendations of National Policy on Education
10th, 17th 24th 31st December are also Sunday and 1986, Under Article 200 of Indian Constitution.
1st January 2001 is Monday 94. Which one of the following pairs is NOT
So, 3rd January 2001 is Wednesday. correctly matched?
89. "Dream" is related to "Reality" in the same (a) IIVR – Lucknow
way as "Falsehood" is related to which of the (b) IIPR – Kanpur
following? (c) IVRI – Izatnagar
(a) Untruth (b) Truth (d) IGFRI – Jhansi
(c) Fairness (d) Correctness Ans. (a) : IIVR (Indian Institute of vegetable
Ans. (b) : Dream and Reality are opposite to each other. Research) is located in Varanasi in Uttar Pradesh,
Similarly, state. Rest are correctly matched.
Truth is the opposite of Falsehood. 95. Which one of the following pairs is NOT
Hence, truth is the correct answer. correctly matched?
90. What number would come in place of? (a) Hem or lace work – Lucknow and
Varanasi
(b) Brass Idol statue – Mathura
(c) Locks – Aligarh
(d) Cane and sticks – Varanasi
Ans. (b) : The city Moradabad is known as the 'Brass
city'. The city is famous for the brass idol, statue and
other brass products.
UP RO/ARO (Pre) Re Exam 2016 (Ex. Dt. 20.9.2020) 72 YCT
CLICK HERE FOR FREE MATERIAL

96. Match the List–I with List–II and select the 101. Which of the following is NOT correctly matched?
correct answer from the codes given below : Fundamental Right Article
List–I (Fair/Mela) List–II (District) (a) Abolition of – Art 17
A. Devi Patan Mela 1. Agra Untouchability
B. Kampil Fair 2. Balrampur (b) Protection against arrest – Art 23
C. Bateshwar Mela 3. Farrukhabad and detention
D. Dadri Mela 4. Balia (c) Freedom of Religion – Art. 25
Codes: (d) Protection of Interest – Art. 29
Minorities
A B C D
(a) 1 2 3 4 Ans. (b) : In the given option, Protection against arrest
(b) 4 1 2 3 and detention are given under Article 22 of Indian
(c) 2 3 1 4 Constitution Hence, option (b) is not correctly matched
rest provision are correctly matched.
(d) 3 4 1 2
Ans. (c) : The correct match is- 102. The difference of the Government Minimum
Support Price (MSP) and market price, which is
(Fair/Mela) (District) paid directly to the farmers under W.T.O. is called
Devi Patan Mela Balrampur (a) Blue box subsidies (b) Green box subsidies
Kampil Fair Farrukhabad (c) Yellow box subsidies (d) Pink box subsidies
Bateshwar Mela Agra Ans. (a) : Blue Box refers to a category of domestic
Dadri Balia support or subsidies under the W.TO's Agreement on
97. Where are Buxa Tribes living in Uttar Agriculture. Blue box supports the subsides Like MSP
Pradesh? and market price which is paid directly to farmers.
(a) Bijnor (b) Balrampur 103. In recently announced central budget 2020–21,
(c) Shrawasti (d) Chitrakoot in which among the following budget allocation
has been decreased in comparison to budget
Ans. (a) : Buxa Tribes are found in the Bijnor district of 2019–20?
Uttar Pradesh, Where they are known as Mehra. Buxa,
also known as Buksa, are indigenous peoples living mainly (a) MGNREGA
in the Indian states of Uttrakhand and Uttar Pradesh. (b) Ayushman
(c) Mid-day Meal Scheme
98. How many main Agro-climatic zones are in
Uttar Pradesh? (d) National Ganga Cleaning Scheme
(a) 9 (b) 3 Ans. (a) : Union finance Minister Nirmala Sitharaman
(c) 5 (d) 8 announced an allocation of Rs. 61500 crore for the
Mahatma Gandhi national Rural Employment
Ans. (a) : On the basis of rainfall and soil types, Uttar Guarantee Scheme (MGNREGS) for the year 2020-21,
Pradesh has been divided into 9 agro-climatic Zones, down by more than 13% from the total estimated
they are- Terai zone, western plains, mid western plains,
expenditure for 2019-20 which was of Rs. 71001.81 crore.
western semi-dry plains, mid western south plain, south
western semi-dry plains, North Eastern plain zone 104. Consider the following measure that R.B.I. uses
Easter plain zone and Vindhyachal zone. to control inflation in the economy.
99. Acting Judges are appointed in : 1. Increase Bank Rate
(a) Supreme Court (b) District Court 2. Increase Cash Reserve Ratio
(c) High Court (d) Both (a) and (b) 3. Increase statutory liquidity Ratio
4. Purchase of government securities
Ans. (c) : Acting Judges are appointed in High court.
Acting Judges can be appointed by the President under Select the correct answer from the codes given
clause (2) of Article 224 of the Constitution. below :
Codes:
100. Which of the following is NOT correctly
matched as per Constitution of India? (a) Only 1 and 2 (b) Only 1, 2 and 3
(c) Only 2, 3 and 4 (d) Only 1, 3 and 4
(a) The Panchayats – Part–IX
Ans. (b) : Bank rate is the rate charged by the Central
(b) The Muncipalities – Part–IX A Bank for lending funds to commercial Banks. Cash
(c) The Cooperative – Part–IX B Reserve Ratio is the funds that banks have to maintain
Societies with the Reserve Bank of India at all times.
(d) Tribunals – Part–X The statutory Liquidity Ratio (SLR) is the reserve
requirement that commercial Banks are require be to
Ans. (d) : Tribunal was incorporated in the Indian
maintain in the form of Cash, Gold reserves, Government
Constitution by 42nd Amendment Act, 1976 in Part securities etc.
XIV-A of the Constitution under Article Purchase of Government security result in increased
323-A and 323-B Hence option (d) is not correctly liquidity in the market which increases the inflation in the
matched rest previsions are correctly matched economy.
UP RO/ARO (Pre) Re Exam 2016 (Ex. Dt. 20.9.2020) 73 YCT
CLICK HERE FOR FREE MATERIAL

105. Which is the first month of Indian National Ans. (a) : If food items are packed in plastic bags in an
Calendar? atmosphere of nitrogen gas it does not spoil as nitrogen
(a) Phalgun (b) Magh is an inert gas and does not react with food substances
(c) Paush (d) Chaitra present in the packet.
Ans. (d) : There are 12 months in the Hindi calendar. 111. Which one of the following process in NOT
The Hindi calendar begins with the month of Chaitra useful for purification of solid impurities?
and ends with the month of Phalgun. (a) Distillation (b) Sublimation
(c) Crystallization (d) All of the above
106. "Navarathri" is celebrated as Saraswathi
Pooja in the following state : Ans. (a) : Distillation is the process of separating
(a) Kerala (b) Karnataka components of mixture based on different boiling points.
(c) Tamil Nadu (d) Telangana 112. Which one of following colours of white light is
least deviated by the glass prism?
Ans. (*) : In the south, Saraswati Puja is observed on the
(a) Green colour (b) Red colour
ninth day of the Navarathi festival. It is followed on the
(c) Violet colour (d) Orange colour
same day as the Ayudha Puja in Kerala and Tamil Nadu.
Ans. (b) : As visible light passes through a glass prism,
107. With reference to the Indian Culture Portal, It refracted into 7 colours. These colours are-Violet
which of the following statements is/are Indigo, Blue, Green, Orange Yellow and Red. In these
correct? 7 colours. Red has highest wavelength and deviated
I. It has been developed by a team from I.I.T. least while Violent has shortest wavelength and get
Roorkee deviated most in among 7 colours.
II. It is the Government's first authorized 113. Asafoetida is obtained from
portal where knowledge and cultural of (a) Exudation from the stem
ministry of culture are available in public (b) Extraction from roots
domain on a single platform (c) Extraction from fruits
Select the correct answer from the codes given (d) Extraction from leaves
below :
Ans. (b) : Asafoetida (Heeng) is a herbaceous plant of
Codes: umbelliferae family. It is the oleo-gum resin obtained
(a) Only I (b) Only II by incising the living rhizomes and roots of Ferula assa-
(c) Both I and II (d) None of the above foetida.
Ans. (b) : The Indian Cultural portal is a digital space 114. Three chambered heart is found in :
for Indian Culture, history and heritage. It is the (a) Mammals (b) Birds
Government's first authorized portal where knowledge (c) Amphibians (d) Pisces
and cultural of ministry of culture are available in Ans. (c) : Three Chambered heart is found in
public domain on a single platform. This portal was Amphibians and reptiles, most amphibians including
developed by IIT Bombay. frogs and toads, have three chambered hearts, with two
108. In the vertical structure of the atmosphere, atria and one ventricle
which one is the lowest layer? 115. Who was the Commander in Chief of the
(a) Stratosphere (b) Mesosphere Rajput Army, besides Maharana Pratap in the
(c) Troposphere (d) Thermosphere Battle of Haldighati?
Ans. (c) : The troposphere is the lowest layer of our (a) Ibrahim Gardi (b) Tardi Beg
atmosphere. starting at ground level, the height of (c) Hakim Sur (d) Mahmood Lodi
troposphere varies 7 to 17 km from pole to equator Ans. (c) : The battle of Haldighati was a battle fought in
above sea level. Humans live in the troposphere and 1576 between Maharana paratap and the Mughal
nearly all weather occurs in this lowest layer. emperor Akbar's forces. Rana Pratap was assisted by
109. Which one of the following acids is used in the Hakim khan Suri While the Mughal forces were led by
car-batteries? Man Singh and Asaf khan.
(a) Acetic Acid (b) Hydrochloric Acid 116. With reference to "the causes of the success of
(c) Sulphuric Acid (d) Nitric Acid British and failure of the French in India"
Which of the following statement is/are
Ans. (c) : In car, acid batteries contain at least 30 to correct?
50% diluted Sulphuric acid in which only 29% 1. Getting huge wealth and manpower from
Concentrated Sulphuric acid is present. conquest of Bengal by British
110. Potato chips are packed in plastic bags in the 2. Naval superiority of the British
atmosphere of : Select the correct answer from the codes given
(a) Nitrogen Atmosphere below :
(b) Hydrogen Atmosphere Codes:
(c) Oxygen Atmosphere (a) Only 1 (b) Only 2
(d) Iodine Atmosphere (c) Both 1 and 2 (d) Neither 1 nor 2
UP RO/ARO (Pre) Re Exam 2016 (Ex. Dt. 20.9.2020) 74 YCT
CLICK HERE FOR FREE MATERIAL

Ans. (c) : The main reason behind success of British 120. Which one of the following was NOT associated
and failure of French in India were British Naval with Indian National Army (I.N.A.)?
Superiority over French and Conquest of Bengal by (a) Rashid Ali (b) Shahnawaz
British from where British were getting huge wealth (c) P.K. Sahgal (d) B.C. Dutta
and manpower . Ans. (d) : In the given option B.C. Dutta was not
117. Which one of the following is NOT correctly associated with Indian National Army (INA) while
matched? Rashid Ali, Shanawaz and P.K Sahagal, Gurbaksh singh
Dhillon these are some prominent personality who were
Acts Year
associated with INA trial.
(a) Prevention of sedition – 1908
121. Which of the following statement is NOT true
Meeting Acts
about the "Quit India Movement of 1942"
(b) Explosive Substances Act – 1908 (a) It was led by Mahatma Gandhi
(c) Indian Criminal Law – 1908 (b) Congress was declared an illegal body
Amendment Act (c) It was a non-violet movement
(d) Newspaper (Incitement to – 1908 (d) It was a spontaneous movement
Offence) Act Ans. (c) : The Quit India movement was started at
Ans. (a) : In the given option prevention of sedition Gowalia Tank. Bombay on August 8, 1942. Mahatma
meeting Act was introduced 1907 while explosive Gandhi gave the call to people to support the
substances Act, Indian criminal law amendment act and movement by the slogan "Do or Die". Since all
(newspaper incitement to offence) Act were introduced Prominent leaders of INC were arrested by British
in 1908. under operation zero, thus some scholars assume that it
was a leaderless movement. The general Public attacked
118. Match the List–I with List–II and select the symbols of authority and hoisted national flags forcibly
correct answer from the codes given below : on Public building. Hence moment was not a non-
List–I List–II violent moment.
A. Rampa Rebellion 1. 1859-60 122. By which of the following Acts was the India
B. Pabna Peasant Revolt 2. 1879-80 Council abolished?
C. Bengal Indigo Revolt 3. 1860-63 (a) Morley Minto Reform 1909
(b) The Government of India Act 1919
D. Jaintia Rebellion 4. 1873-76
(c) The Government of India Act 1935
Codes: (d) The Indian Independence Act 1947
A B C D Ans. (c) : The Government of India Act, 1935 was
(a) 2 1 3 4 passed by British parliament in 1935. It abolished India
(b) 2 4 1 3 council and dyarchy at provinces. It gave more
(c) 1 2 3 4 autonomy the Provinces, established dyarchy at centre
(d) 4 2 1 3 and bicameral federal legislature.
Ans. (b) : 123. Consider the following events and arrange
these in chronological order
List I List II
I. Cripps proposal
Rampa Rebellion 1879-80
II. August offer
Pabna Peasant Revolt 1873-76 III. Wavell plan
Bengal Indigo Revolt 1859-60 IV. C.R. Formula
Jaintia Rebellion 1860-63 Select the correct answer from the codes given
119. Which Ruler of Delhi Sultanate declared below :
himself as "Khalifa"? Codes
(a) Balban (a) II, I, III, IV (b) II, I, IV, III
(b) Alauddin Khilji (c) I, II, IV, III (d) I, II, III, IV
(c) Mubarak Khalji Ans. (b) :
(d) Muhammad Bin Tuglaq Events Years
Cripps Proposal 1942
Ans. (c) : Mubarak shah ascended the throne with the
title Qutubuddin in 1316, He was the son of Alauddin August Offer 1940
khilji. Qutb -ud-din Mubarak shah Khilji was the third Wavell Plan 1945
and last ruler of khilji dynasty. He was only ruler who CR Formula 1944
declared himself as khalifa or calipha. Hence option (b) is in correct Chronological Order.
UP RO/ARO (Pre) Re Exam 2016 (Ex. Dt. 20.9.2020) 75 YCT
CLICK HERE FOR FREE MATERIAL

124. Which of the following is NOT correctly 128. According to American based think tank
matched? population Review Report 2020, Which of the
Organisation Person following statement is/are correct?
(a) Young Bengal – Henry Vivian 1. India's GDP in 2019 was ` 209 lakh crore
Movement Derozio 2. India emerged as fifth largest economy of
(b) Bahishkrit Hitkarini – Jyotiba Phule the world
Sabha 3. India over take countries like Britain and
France
(c) Theosophical – Colonel Olcott
Society Select the correct answer using codes given
below :
(d) United Indian – Syed Ahmad
Codes:
Pariotic Association Khan
(a) 1 and 2 only (b) 1 and 3 only
Ans. (b) : Bahishkrit Hitkarini Sabha was established by
(c) 2 and 3 only (d) 1, 2 and 3 correct
Dr. B.R. Ambedkar in 1924 at Bombay with aim to
establishing a central institution for removing difficulties Ans. (d) : As per American based think tank,
of the untouchables and placing their grievances before the population Review Report 2020. India emerged as fifth
government, Hence option (b) is not correctly matched rest largest economy of the world after taking over
options are correctly matched. economies like British and France in 2019, the GDP of
India in 2019 was 209 lakh Crore.
125. Which one of the following is NOT correctly
matched? 129. Given below are two statements, one labelled as
the Assertion (A) and other as Reason (R) :
Crop Weed
Assertion (A) : Lucknow's Defence Expo 2020
(a) Wheat – Phalaris minor was the Asia' biggest Expo of defence products.
(b) Paddy – Bathua Reason (R) : The Expo was organised to portrait
(c) Pea – Piyazi India as emerging defence manufacturing hub.
(d) Berseem – Kasni Select the correct answer from the codes given
below :
Ans. (b) : The weed Bathua is related to Potato Crop
Codes :
not paddy Hence option (b) is not correctly matched rest
of all pairs are correctly matched. (a) Both (A) and (R) true and (R) is the correct
explanation of (A)
126. Match List–I with List–II and select the correct (b) Both (A) and (R) are true and (R) is not the
answer from the codes given below : correct explanation of (A)
List–I (Crop) ListII (Season) (c) (A) is true but (R) is false
A. Sunflower 1. Kharif (rainy) (d) (A) is false but (R) is true
B. Muskmelon 2. Zaid (summer) Ans. (a) : The Defence Expo which was held in 2020.
C. Cotton 3. Rabi (winter) was the Asia's biggest Expo of defence products. It was
organised to portrait India as a emerging defence
D. Linseed 4. All the season manufacturing hub.
Codes: 130. Following are the estimated source of revenue
A B C D A B C D of the Union Budget 2020–21. Match List–I
(a) 3 4 1 2 (b) 4 2 1 3 with List–II and select the correct answer from
(c) 3 4 2 1 (d) 2 3 4 1 the codes given below.
Ans. (b) : The correct match is - List–I (Source) List-II (Percentage of
Crops Season Revenue)
Sunflower All the season A. Corporate tax 1. 17 percent
Muskmelon Zaid B. Income tax 2. 18 percent
Cotton Kharif C. Customs 3. 7 percent
Linseed Rabi D. Central Excise 4. 4 percent
127. According to Fortune India List of 500 Codes:
companies, the biggest corporation/company in A B C D A B C D
2019 was : (a) 1 2 3 4 (b) 4 3 2 1
(a) Indian Oil Corporation Ltd. (c) 3 4 1 2 (d) 2 1 4 3
(b) O.N.G.C. Ans. (d) :The correct match is -
(c) Reliance Industries Ltd. Source Percentage of Revenue
(d) S.B.I. Corporate tax 18%
Ans. (c) : As per Fortune Indian List of 500 companies, Income tax 17%
the biggest corporation company in 2019 was Reliance Customs 4%
Industries Limited. Central Excise 7%
UP RO/ARO (Pre) Re Exam 2016 (Ex. Dt. 20.9.2020) 76 YCT
CLICK HERE FOR FREE MATERIAL

131. Genetic industry includes : Ans. (d) : NGT was established in 2010 by the
(a) Agriculture National Green Tribunal Act, with the aim of effective
(b) Fishing and expeditious disposal of cases relating to
(c) Hunting environment protection and conservation of forest and
(d) Mining other natural resources.
Ans. (a) : Genetic Industries means the Industries 137. Which one of the following is NOT correctly
which undertakes reproduction or multiplication of matched?
animals and plants with the aim of earning profit. It Country Iron-ore producing
includes basically Agricultural sector. area
132. Which of the following rivers of India crosses (a) Kazakhstan – Karaganda
the tropic of cancer twice?
(b) Ukrain – Krivoy Rag
(a) Mahi
(b) Chambal (c) Germany – Normandy
(c) Narmada (d) France – Pyreneas
(d) None of the above Ans. (c) : Normandy is an Iron-ore producing region
Ans. (a) : The river Mahi of India crosses the tropic of of France not Germany.
cancer twice. The river rises in Madhya Pradesh and Country Iron-ore producing area
after blowing through Rajasthan and Gujarat, it drains Kazakhstan Karaganda
into Arabian Sea.
Ukraine Krivoy Ray
133. Which of the following census years is known
Germany Ruhr
as the "Great Divide" in the demographic
history of India? France Pyrenees
(a) 1901 (b) 1921 138. Which one of the following is correctly
(c) 1931 (d) 1941 matched?
Ans. (b) : The year 1921 is known as Great Divide in Lorren Industrial – Italy
demographic history of India because before 1921 the (a) Region
population of India was not constant, sometimes it (b) Ruhr Industrial Region – Germany
increased, and sometime it decreased but after 1921 the (c) Bristal Industrial – France
population of India is increasing continuously.
Region
134. "Tulbul" project is on which river? (d) Saxony Industrial – United
(a) Satluj (b) Jhelum Region Kingndom
(c) Ravi (d) Beas
Ans. (b) :
Ans. (b) : The Tulbul Project is a navigation lock -cum
control structure at the mouth of wular lake. It is located Industrial regions Countries
at the river Jhelum. Lorren Industrial Region France
135. Consider the following statements and select Ruhr Industrial Region Germany
the correct answer from the codes given below : Bristol Industrial Region England
1. Rihand Dam is on a tributary of the Sone Saxony Industrial Region Germany
river
139. Which among the following countries NOT
2. Hirakud Dam is on Mahanadi river
located on the western coast of Africa?
3. Tungabhadra project is a joint venture of
Andhra Pradesh and Karnataka (a) Gabon (b) Botswana
4. Maithan Dam is on Barakar river, a (c) Liberia (d) Angola
tributary of Damodar river Ans. (b) : Gabon, Liberia and Angola are west coast
Codes : African countries, whereas Botswana is located at
(a) 1, 2 and 3 are correct southern part of Africa continent . It is a land locked
(b) 1, 2 and 3 are correct country.
(c) 1, 3 and 4 are correct 140. Forest Research Institute is located in :
(d) 1, 2, and 4 are correct (a) Dehradun (b) Bhopal
Ans. (*) : In the given option all statements are correct. (c) New Delhi (d) Nagpur
Hence none option is correct Ans. (a) : The forest Research Institute (FRI) is an
136. National Green Tribunal (NGT) was institute of the Indian council of Forestry Research and
established by the Government of India in : Education and is a premier institution in the field of
(a) 2007 (b) 2008 forestry research in India. It is located at Dehradun in
(c) 2009 (d) 2010 Uttarakhand.
UP RO/ARO (Pre) Re Exam 2016 (Ex. Dt. 20.9.2020) 77 YCT
CLICK HERE FOR FREE MATERIAL

Gòej ØeosMe meceer#ee DeefOekeâejer/meneÙekeâ meceer#ee DeefOekeâejer (Øeer.) hegvehe&jer#ee, 2016


meeceevÙe efnvoer
JÙeeKÙee meefnle nue ØeMve-he$e hejer#ee efleefLe : 20.09.2020

1. ‘Yeeweflekeâ’ keâe efJeueesce Meyo nw– Ans. (d) : Megæ Jele&veer Jeeuee Meyo ‘osoerhÙeceeve’ nw~ Ùen efJeMes<eCe
(a) DeeOÙeeeflcekeâ (b) oeMe&efvekeâ kesâ ™he ceW ØeÙegòeâ neslee nw, efpemekeâe DeLe& nw Ûecekeâlee ngDee, ocekeâlee
(c) meebmke=âeflekeâ (d) jepeveereflekeâ ngDee~ Ùen Meyo oerhed ± Ùe*d ± MeeveÛe ØelÙeÙe ueiee keâj yevee nw~
Ans. (a) : Yeeweflekeâ keâe efJeueesce Meyo ‘DeeOÙeeeflcekeâ’ nw~ Yeeweflekeâ SJeb 7. efvecveefueefKele ceW mes DeMegæ Jele&veer Jeeuee Meyo nw–
DeeOÙeeeflcekeâ efJeMes<eCe Meyo nQ~ efJeMes<eCe keâe efJeueesce meowJe efJeMes<eCe (a) keâewletnue (b) DeveefOekeâej
neslee nw~ efJeueesce Meyo keâe DeLe& Guše Ùee efJehejerle neslee nw~ (c) efvejhes#e (d) heewjeefCekeâ
2. ‘ØeeÛeer’ Meyo keâe efJehejerleeLe&keâ nw– Ans. (d) : ‘heewjeefCekeâ’ Meyo Jele&veer keâer Âef° mes DeMegæ nw~ Fmekeâe
(a) GoerÛeer (b) ØeleerÛeer Megæ Jele&veer ‘heewjeefCekeâ’ nw~ DevÙe Meyo keâewletnue, DeveefOekeâej, efvejhes#e
(c) veJeerve (d) meceerÛeer Jele&veer keâer Âef° mes Megæ Meyo nw~
Ans. (b) : efoÙes ieÙes MeyoeW kesâ efJehejerleeLe&keâ Meyo Fme Øekeâej nQ– 8. efvecveefueefKele JeekeäÙeeW ceW Megæ JeekeäÙe nw–
Meyo efJeueesce (a) cegPemes Ùen keâece mecYeJe veneR nes mekeâlee
(b) Ùen efyeukegâue Yeer yeele veneR keâjvee Ûeenlee
ØeeÛeer (hetjye) ØeleerÛeer (heefMÛece)
(c) Ùen keâefJelee Deveskeâ YeeJe Øekeâš keâjleer nw
GoerÛeer (Gòej) DeJeeÛeer (oef#eCe)
(d) Fve oesveeW ceW kesâJeue Ùener Deblej nw
veJeerve ØeeÛeerve
Ans. (c) : ‘Ùen keâefJelee Deveskeâ YeeJe Øekeâš keâjleer nw’ Megæ JeekeäÙe
meceerÛeer keâe DeLe& ØeMebmee nw~
nw~ FvÙe JeekeäÙeeW keâe Megæ ™he nw–
3. ‘meceef°’ keâe efJeueesce Meyo nw– • cegPemes Ùen keâece mecYeJe veneR~
(a) efJeefMe° (b) JÙeef° • Jen efyeukegâue yeele veneR keâjvee Ûeenlee~
(c) DeefMe° (d) Dehegef° • Fve oesveeW ceW Ùener Deblej nw~
Ans. (b) : meceef° keâe efJeueesce JÙeef° nw~ efJeefMe° keâe meeceevÙe, 9. efvecveefueefKele ceW mes Skeâ JeekeäÙe Megæ nw, Jen nw–
DeefMe° keâe efMe°, Dehegef° keâe efJeueesce hegef° neslee nw~ (a) Fmekesâ yeeJepeto Yeer ncemes efnvoer efueKeves ceW yengle Yetue nes
4. efvecveefueefKele ceW mes efJeueesce MeyoeW keâer Âef° mes Skeâ Ùegice peeleer nw~
ieuele nw, Jen nw– (b) Fmekesâ yeeJepeto Yeer ncemes efnvoer efueKeves ceW yengle YetueW nes
(a) efJeefOe–efve<esOe (b) Dee£eve–efJemepe&ve peeleer nw~
(c) Dee«en–efJe«en (d) DeefceÙe–nueenue (c) Fmekesâ yeeJepeto Yeer nce ueesieeW mes efnvoer efueKeves ceW yengle
Ans. (c) : efJeueesce keâer Âef° mes ieuele Ùegice ‘Dee«en–efJe«en’ nw~ mener Yetue nes peeleer nw~
efJeueesce Ùegice nQ– (d) Fmekesâ yeeJepeto nce ueesieeW mes efnvoer efueKeves ceW yengle YetueW
efJeefOe – efve<esOe nes peeleer nw~
Dee£eve – efJemepe&ve Ans. (d) : Megæ JeekeäÙe nw– ‘Fmekesâ yeeJepeto nce ueesieeW mes efnvoer
Dee«en – ogje«en efueKeves ceW yengle YetueW nes peeleer nQ~’ DevÙe JeekeäÙeeW ceW ‘yeeJepeto’ kesâ
DeefceÙe – nueenue yeeo ‘Yeer’ ueieeÙee ieÙee nw pees efkeâ DeMegæ nw~
mebefOe – efJe«en 10. DeMegæ JeekeäÙe keâe ÛeÙeve keâerefpeS~
(a) hegefueme Éeje [ekegâDeeW keâe heerÚe efkeâÙee ieÙee~
5. efJeueesce Meyo keâer Âef° mes FveceW mes mener Ùegice nw–
(b) osMe keâer Jele&ceeve ceewpetoe neueele "erkeâ veneR nw~
(a) DekeâerCe&–efJekeâerCe&
(c) ceQ hegmlekeâeueÙe ceW efvecÙe meceÙe hej hengBÛelee ntB~
(b) F&efhmele–DeYeerefhmele
(d) legce efÛevlee ve keâjes, ceQ keâesF& ve keâesF& jemlee efvekeâeuetBiee~
(c) DeeÂle–efvejeÂle
Ans. (b) : efoS ieÙes JeekeäÙeeW ceW DeMegæ JeekeäÙe nw– osMe keâer Jele&ceeve
(d) oes<e–meoes<e
ceewpetoe neueele "erkeâ veneR nw~’’ Fmekeâe Megæ JeekeäÙee nesiee– osMe keâer
Ans. (c) : efJeueesce keâer Âef° mes mener Ùegice DeeÂle–efvejeÂle nw~ DevÙe Jele&ceeve ceewpetoe neueele "erkeâ veneR nw~ ceewpetoe Deewj Jele&ceeve Meyo Skeâ
MeyoeW keâe mener efJeueesce nw– mebkeâerCe&–efJekeâerCe&, F&efhmele–Deveerefhmele, meeLe veneR ØeÙegòeâ neWies~
Deoes<e–meoes<e, oes<e–iegCe~ 11. ‘hejbheje mes Ûeueer Dee jner yeele’ kesâ efueS ØeÙegòeâ nessves
6. Megæ Jele&veer Jeeues Meyo keâe ÛeÙeve keâjW~ Jeeuee Meyo nw–
(a) owoerhÙeceeve (b) osoerheceeve (a) Øeefleßegefle (b) JÙeepemlegefle
(c) owoerheceeve (d) osoerhÙeceeve (c) Devegueesce (d) Devegßegefle
UP RO/ARO (Pre) Re Exam Hindi 2016 78 YCT
CLICK HERE FOR FREE MATERIAL

Ans. (d) : ‘‘hejcheje mes Ûeueer Dee jner yeele kesâ efueS ØeÙegòeâ nesves 17. ‘Deienve’ keâe lelmece ™he keâewve–mee nw?
Jeeuee Meyo ‘Devegßegefle’ nw~ Øeefleßegefle efkeâmeer yeele kesâ efueS efoÙee peeves (a) De«eneÙeCe (b) DeienCe
Jeeuee JeÛeve keâes keânles nQ~ JÙeepemlegefle, efveboe kesâ yeneves mlegefle keâes (c) Dee«enCe (d) De«eemeve
keânles nQ Deewj Devegueesce ÙeLee›eâce Ùee Thej mes veerÛes keâer Deesj Deeves Ans. (a) : ‘Deienve’ keâe lelmece ™he ‘De«eneÙeCe’ nw~ Deienve efnvot
Jeeues kesâ efueS ØeÙegòeâ neslee nw~ hebÛeebie keâe Skeâ cenervee nw~ efpeve MeyoeW keâes mebmke=âle mes efyevee heefjJele&ve
12. ‘efpemekesâ heeme kegâÚ ve nes’ JeekeäÙeebMe kesâ efueS Skeâ Meyo kesâ efnvoer ceW ues efueÙee ieÙee nw, lelmece Meyo keânueeles nQ~
nw– 18. efvecveefueefKele ceW mes ‘leÆJe’ Meyo nw–
(a) De#ece (b) DeefkebâÛeve (a) Ûelego&Me (b) ÛelegLe&
(c) De%e (d) DemeceLe& (c) Ûeewon (d) ÛelJeeefj
Ans. (b) : JeekeäÙeebMe kesâ efueS Skeâ Meyo nw– Ans. (c) : efoS ieÙes MeyoeW ceW ‘Ûeewon’ leÆJe Meyo nw~ peyeefkeâ
Meyo JeekeäÙeebMe Ûelego&Me, ÛelegLe& Deewj ÛelJeeefj lelmece Meyo nQ~ pees mebmke=âle Meyo
DeefkebâÛeve efpemekesâ heeme kegâÚ ve nes~ efJeefYeVe efJekeâej SJeb OJeefve heefjJele&ve kesâ meeLe efnvoer ceW ØeÙegòeâ nesles nQ,
De#ece pees kegâÚ keâjves ceW me#ece veneR/#ecelee jefnle~ GvnW leÆJe Meyo keânles nQ~
De%e/De%eeveer pees kegâÚ ve peevelee nes~ 19. ‘DeeYÙeblej’ keâe leÆJe Meyo nw–
DemeceLe& efpemeceW meeceLÙe& ve nes~ (a) Devoj (b) Yeerlej
13. ‘mJeso mes GlheVe nessves Jeeuee’ JeekeäÙeebMe kesâ efueS Skeâ Meyo (c) yeenj (d) ienje
nw– Ans. (b) : DeeYÙeblej keâe leÆJe Meyo ‘Yeerlej’ nw~ ienje keâe lelmece
(a) mJesope (b) DeC[pe ‘iecYeerj’ nw~ yeenj keâe lelmece ‘yeefnj’ neslee nw~
(c) efheC[pe (d) GYeÙepe 20. ‘keâhe&š’ keâe leÆJe ™he nw–
Ans. (a) : JeekeäÙeebMe Deewj Skeâ Meyo mes mecyeefvOele nQ– (a) keâheš (b) keâejhesš
Meyo JeekeäÙeebMe (c) keâhetj (d) keâheÌ[e
mJesope mJeso mes GlheVe nesves Jeeuee Ans. (d) : keâhe&š keâe leÆJe ‘keâheÌ[e’ neslee nw~ keâhetj, keâhet&j keâe
DeC[pe DeC[s mes GlheVe leÆJe nw~ keâheš (OeesKee), keâejhesš Demebiele Meyo nQ~
efheC[pe efpemekeâe pevce Mejerj mes nes~ 21. ‘‘Jen yengle Keelee nw~’’ Fme JeekeäÙe ceW ‘yengle’ Meyo efkeâme
14. ‘DeheefjCeerle’ Meyo efvecveefueefKele ceW mes efkeâme JeekeäÙeebMe kesâ Øekeâej keâe efJeMes<eCe nw?
efueS ØeÙegòeâ neslee nw? (a) efveefMÛele heefjceeCeJeeÛekeâ efJeMes<eCe
(a) efpemekeâe heefjCeece ve efvekeâuelee nes~ (b) efveefMÛele mebKÙeeJeeÛekeâ efJeMes<eCe
(b) efpemekeâe efJeJeen ve ngDee nes~ (c) legueveelcekeâ efJeMes<eCe
(c) efpemekeâe efJeJeen nes Ûegkeâe nes~ (d) DeefveefMÛele heefjceeCeJeeÛekeâ efJeMes<eCe
(d) pees osKeves ceW Øeereflekeâj ve nes~
Ans. (d) : ‘Jen yengle Keelee nw~’ Fme JeekeäÙe ceW ‘yengle’ Meyo
Ans. (b) : ‘efpemekeâe efJeJeen ve ngDee nes’ kesâ efueS Skeâ Meyo DeefveefMÛele heefjceeCeJeeÛekeâ efJeMes<eCe nw~ ÙeneB ‘yengle’ Meyo efkeâmeer
‘DeheefjCeerle’ nw~ efpemekeâe efJeJeen nes Ûegkeâe nes kesâ efueS Skeâ Meyo efveefMÛele heefjceeCe keâes veneR yelee hee jne nw~ DeefveefMÛele heefjceeCeJeeÛekeâ
‘heefjCeerle’ nesiee~ efJeMes<eCe kesâ DevÙe GoenjCe nQ– Lees[Ì e, DeefOekeâ, kegâÚ Deeefo~ efpeve
15. efvecveefueefKele ceW mes efkeâme JeekeäÙeebMe kesâ efueS Skeâ Meyo JemlegDeeW keâes ceehee Ùee leewuee peelee nw Gvekesâ meeLe heefjceeCeJeeÛekeâ leLee
‘ogefve&Jeej’ ØeÙegòeâ neslee nw? efpevekeâer ieCevee keâer peeleer nw Gvekesâ meeLe mebKÙeeJeeÛekeâ efJeMes<eCe keâe
(a) efpemes otj keâjvee keâef"ve nes~ ØeÙeesie efkeâÙee peelee nw~
(b) efpeme hej Jeej keâjvee keâef"ve nes~ 22. efvecveefueefKele JeekeäÙeeW ceW mes Jen JeekeäÙe ÛegefveÙes, efpemeceW
(c) efpemes osKe heevee keâef"ve nes~ iegCeJeeÛekeâ efJeMes<eCe keâe ØeÙeesie efkeâÙee ieÙee nw?
(d) efpemes peerle heevee keâef"ve nes~ (a) Jen vešKeš yeeuekeâ mebieerle veneR megve jne nw~
Ans. (a) : ‘efpemes otj keâjvee keâef"ve nes’ kesâ efueS Skeâ Meyo (b) JeneB yengle mes he#eer GÌ[ jns nQ~
‘ogefve&Jeej’ neslee nw~ efpemes peerle heevee keâef"ve nes kesâ efueS Skeâ Meyo (c) ceQves ome ceve iesntB Kejeroe nw~
‘ogpexÙe’ nesiee~ (d) Ssmes Deeoceer ueeKeeW ceW Skeâ nesles nQ~
16. efvecveefueefKele ceW mes ‘lelmece’ Meyo nw– Ans. (a) : ‘Jen vešKeš yeeuekeâ mebieerle veneR megve jne nw~’ Fme
(a) oner (b) peerCe& JeekeäÙe ceW iegCeJeeÛekeâ efJeMes<eCe (vešKeš) keâe ØeÙeesie efkeâÙee ieÙee nw~
(c) ieÙebo (d) ieenkeâ efpeve efJeMes<eCe MeyoeW mes heoeLe& kesâ iegCe, jbie, Deekeâej, oMee, DeJemLee,
Ans. (b) : efvecveefueefKele MeyoeW ceW ‘peerCe&’ lelmece Meyo nw~ peerCe& keâe ™he Deeefo keâe yeesOe neslee nw, GvnW iegCeJeeÛekeâ efJeMes<eCe keânles nQ~
leÆJe Peervee (hegjevee) Deewj ieÙebo keâe DeLe& neLeer neslee nw~ oner Deewj pewmes– Yeuee, yegje, heeheer, ueeue, heeruee, yeenjer, megvoj, uecyee, kegâ™he
ieenkeâ leÆJe Meyo nQ, Fvekeâe lelmece ›eâceMe: nw– oefOe Deewj «eenkeâ~ Deeefo~
UP RO/ARO (Pre) Re Exam Hindi 2016 79 YCT
CLICK HERE FOR FREE MATERIAL

23. ‘‘meleerMe ÛebÛeue yeeuekeâ nw’’ Fme JeekeäÙe ceW ‘ÛebÛeue’ Meyo 29. efvecveefueefKele ceW mes keâewve–mee Meyo ‘keâheÌ[e’ keâe
nw– heÙee&ÙeJeeÛeer veneR nw?
(a) efJeOesÙe efJeMes<eCe (a) JeŒe (b) heš
(b) iegCeJeeÛekeâ efJeMes<eCe (c) Jemeve (d) Jeemeve
(c) meeJe&veeefcekeâ efJeMes<eCe Ans. (d) : ‘Jeemeve’ Meyo keâheÌ[e keâe heÙee&ÙeJeeÛeer veneR nw~ JeŒe,
(d) ØeefJeMes<eCe heš, Jemeve, keâheÌ[e kesâ heÙee&ÙeJeeÛeer Meyo nQ~
Ans. (b) : ‘meleerMe ÛebÛeue yeeuekeâ nw~’ Fme JeekeäÙe ceW ‘ÛebÛeue’ 30. efvecveefueefKele ceW mes keâewve–mee Meyo ‘cesIe’ keâe
iegCeJeeÛekeâ efJeMes<eCe nw~ ÛebÛeue Meyo ÙeneB meleerMe keâe iegCe yelee jne heÙee&ÙeJeeÛeer veneR nw?
nw~ Dele: Ùen iegCeJeeÛekeâ efJeMes<eCe nw~ efJeMes<eCe kesâ Ûeej Yeso nQ– (1) (a) heÙeesOej (b) peueOej
iegCeJeeÛekeâ, (2) mebKÙeeJeeÛekeâ, (3) heefjceeCeJeeÛekeâ, (4) meeJe&veeefcekeâ~ (c) JeeefjOej (d) oeceesOej
24. ‘ceeveefmekeâ’ efJeMes<eCe efkeâme cetue Meyo mes yevee nw? Ans. (d) : ‘oeceesOej’ Meyo cesIe keâe heÙee&ÙeJeeÛeer veneR nw~ cesIe kesâ
(a) ceeve (b) ceeveme heÙee&ÙeJeeÛeer Meyo nQ– heÙeesOej, peueOej, JeeefjOej, OejeOej, Jeeefjo,
(c) ceveme (d) ceefveme yeeoue, peercetle, veerjo Deeefo~ oeceesOej Meyo DeMegæ nw, Fmekeâe Megæ
™he oeceesoj neslee nw efpemekeâe heÙee&ÙeJeeÛeer nw– peveeo&ve, Ûe›eâheeefCe,
Ans. (b) : ceeveefmekeâ efJeMes<eCe ‘cevemed’ cetue Meyo mes yevee nw~
efJeMJecYej, cegkegâvo, veejeÙeCe, kesâMeJe, ceeOeJe, ÛelegYeg&pe Deeefo~
DeeÙeesie ves Fmekeâe Gòej ‘ceeveme’ ceevee nw~ ceeveefmekeâ ceW ‘Fkeâ’ ØelÙeÙe
31. ‘DeeefJeYet&le’ keâe mener efJeueesce Meyo nw–
SJeb ‘cevemed’ cetue Meyo nw~ Fkeâ ØelÙeÙe efpeme Meyo ceW ueielee nw Gmekesâ
henues De#ej keâes Ùee lees oerIe& keâj oslee nw DeLeJee Je=efæ keâj oslee nw~ (a) DeveemLee (b) Deveskeâlee
pewmes– Meyo ± Fkeâ (ØelÙeÙe) · Meeefyokeâ, efove ± Fkeâ · owefvekeâ, (c) eflejesYetle (d) DeveeJe=ef°
Oece& ± Fkeâ · Oeeefce&keâ~ Ans. (c) : DeeefJeYet&le keâe mener efJeueesce Meyo ‘eflejesYetle nw~ DeveemLee
25. ‘‘Ûeeefjef$ekeâ’’ efJeMes<eCe Meyo keâe cetue Meyo keâewve–mee keâe efJeueesce DeemLee, Deveskeâlee keâe efJeueesce Skeâlee, DeveeJe=ef° keâe
nw? efJeueesce DeefleJe=ef° neslee nw~
(a) Ûeefj$e (b) efÛe$eCe 32. ‘#eefCekeâ’ keâe mener efJeueesce Meyo nw–
(c) Ûeefj$elee (d) Ûee®lJe (a) Deuhe (b) MeeMJele
(c) #ej (d) efJecegKe
Ans. (a) : ‘Ûeeefjef$ekeâ’ efJeMes<eCe Meyo keâe cetue Meyo ‘Ûeefj$e’ nw~
Ûeefj$e Meyo ceW ‘Fkeâ’ ØelÙeÙe ueieeves mes efJeMes<eCe Ûeeefjef$ekeâ yevee nw~ Ans. (b) : #eefCekeâ keâe efJeueesce Meyo ‘MeeMJele’ nw~ Deuhe keâe
‘Fkeâ’ ØelÙeÙe Ùegòeâ kegâÚ efJeMes<eCe nQ– Oece& ± Fkeâ · Oeeefce&keâ, cegKe ± efJeueesce Deefle, #ej keâe efJeueesce De#ej, efJecegKe keâe efJeueesce GvcegKe
Fkeâ · ceewefKekeâ, DeLe& ± Fkeâ · DeeefLe&keâ Deeefo~ neslee nw~
33. ‘De«epe’ keâe efJeueesce Meyo nw–
26. ‘ceerceebmee’ keâe mener heÙee&ÙeJeeÛeer Meyo nw–
(a) De#ele (b) Delegue
(a) mJe™he (b) megefJe%elee
(c) Devegpe (d) Dešue
(c) meceeueesÛeve (d) efveef<›eâÙe
Ans. (c) : De«epe keâe efJeueesce Meyo ‘Devegpe’ nw~ De#ele keâe efJeueesce
Ans. (c) : ‘ceerceebmee’ keâe mener heÙee&ÙeJeeÛeer Meyo ‘meceeueesÛeve’ nw~
efJe#ele leLee Dešue keâe efJeueesce ÛebÛeue neslee nw~
Fmekesâ DevÙe heÙee&Ùe nQ– meceer#ee, DeeueesÛevee, efJeJesÛevee, efve™heCe
34. efvecveefueefKele ceW mes efJeueesce MeyoeW keâer Âef° mes Skeâ Ùegice
Deeefo~
ieuele nw, Jen nw–
27. ‘efyepeueer’ keâe heÙee&ÙeJeeÛeer Meyo nw–
(a) ceewve–cegKej (b) Meeveoej–Mece&veekeâ
(a) Ûecekeâ (b) meewoeefceveer
(c) yeye&j–meYÙe (d) DevegÛej–heefjÛej
(c) ØekeâeMe (d) FveceW mes keâesF& veneR
Ans. (d) : efJeueesce MeyoeW keâer Âef° mes ieuele Ùegice ‘DevegÛej–heefjÛej’
Ans. (b) : efoÙes ieÙes MeyoeW ceW efyepeueer keâe heÙee&JeeÛeer Meyo meewoeefceveer nw~ oesveeW keâe DeLe& mesJekeâ nw~ ceewve keâe efJeueesce cegKej, Meeveoej keâe
nw~ Fmekesâ DevÙe heÙee&ÙeJeeÛeer nQ– efJeÅegle, leefÌ[le, Ûeheuee, oeefceveer~ efJeueesce Mece&veekeâ Deewj yeye&j keâe efJeueesce meYÙe mener nw~
Ûecekeâ keâe heÙee&ÙeJeeÛeer DeeYee, oerefhle, Åegefle, keâebefle, peieceieenš nw~ 35. efJeueesce Meyo keâer Âef° mes FveceW Skeâ Ùegice ieuele nw, Jen
ØekeâeMe keâe heÙee&JeeÛeer Deeueeskeâ, jesMeveer, Gpeeuee, pÙeesefle neslee nw~ nw–
28. efvecveefueefKele ceW mes keâewve–mee Meyo ‘ue#ceer’ keâe (a) Deefcele–heefjefcele (b) melkeâej–eflejmkeâej
heÙee&ÙeJeeÛeer veneR nw? (c) DeeÛÚeefole–heefjÛÚVe (d) megKe–og:Ke
(a) jcee (b) Fbefoje Ans. (c) : efJeueesce MeyoeW keâer Âef° mes ‘DeeÛÚeefole–heefjÛÚVe’ ieuele
(c) keâceuee (d) Yeejleer Ùegice nw~ DeeÛÚeefole keâe efJeueesce DeveeÛÚeefole neslee nw~ efJeueesce keâer
Ans. (d) : ‘Yeejleer’ ue#ceer keâe heÙee&ÙeJeeÛeer Meyo veneR nw~ Yeejleer Âef° mes DevÙe Ùegice mener nQ~
mejmJeleer keâe heÙee&ÙeJeeÛeer nw~ ue#ceer kesâ heÙee&Ùe nQ– jcee, Fbefoje, 36. Jele&veer keâer Âef° mes Megæ Meyo nw–
keâceuee, efJe<CegefØeÙee, keâceueemevee, ßeer Deeefo~ mejmJeleer kesâ heÙee&Ùe nQ– (a) efÛevn (b) efJeJele&
Yeejleer, JeeiesMJejer, JeerCeeJeeefoveer, JeeÛee, efJeÅee osJeer Deeefo~ (c) Deveg«enerle (d) efJejnCeer
UP RO/ARO (Pre) Re Exam Hindi 2016 80 YCT
CLICK HERE FOR FREE MATERIAL

Ans. (b) : Jele&veer keâer Âef° mes ‘efJeJele&’ Megæ Meyo nw~ DevÙe Meyo Ans. (a) : ‘Keespe keâjves Jeeuee’ JeekeäÙeebMe kesâ efueS Skeâ Meyo
SJeb Gvekeâer Megæ Jele&veer nw– ‘DevJes<ekeâ’ nw~ Deveghece– ‘efpemekeâer Ghecee ve nes’ nw, efveJesMekeâ– efveJesMe
Meyo Megæ Jele&veer keâjves Jeeuee/heBtpeer ueieeves Jeeuee nw~ DeefvJeefle keâe DeLe& ‘hejmhej
efÛevn efÛeÖ mecyeælee’ Ùee ‘Skeâlee’ neslee nw~
Deveg«enerle Devegie=nerle 43. ‘efpemekeâer «eerJee megvoj nes’ JeekeäÙeebMe kesâ efueS Skeâ Meyo
efJejnCeer efJejefnCeer nw–
37. Megæ Jele&veer Jeeues Meyo keâe ÛeÙeve keâjW~ (a) heeefLe&Je (b) meg«eerJe
(a) pÙeeseflemevee (b) pÙeeslmevee (c) megOeerj (d) megveerue
(c) pÙeeslemevee (d) pÙeeslmvee Ans. (b) : ‘efpemekeâer «eerJee megvoj nes’ kesâ efueS Skeâ Meyo ‘meg«eerJe’
Ans. (d) : ‘pÙeeslmvee’ Meyo Jele&veer keâer Âef° mes Megæ Meyo nw~ DevÙe nw~ ‘he=LJeer mes mebyebOe jKeves Jeeuee’ kesâ efueS Skeâ Meyo ‘heeefLe&Je’ nesiee~
MeyoeW keâer Jele&veer DeMegæ nw~ megOeerj keâe DeLe& ‘OewÙe&Jeeve’ ÂÌ{ neslee nw~
38. Jele&veer keâer Âef° mes keâewve–mee Meyo DeMegæ nw? 44. ‘efpemekeâe DevegYeJe FefvõÙeeW Éeje ve nes mekesâ’ Fme JeekeäÙeebMe
(a) keâesceueebieer (b) meefcceefuele kesâ efueS Skeâ Meyo nw–
(c) Glkeâ<e&lee (d) Devegie=nerle (a) peerlesvõ (b) DeleerefvõÙe
Ans. (c) : ‘Glkeâ<e&lee’ Meyo Jele&veer keâer Âef° mes DeMegæ nw~ Megæ (c) Sefvõkeâ (d) FveceW mes keâesF& veneR
Meyo Glkeâ<e& neslee nw~ keâesceueebieer, meefcceefuele Deewj Devegie=nerle Meyo Ans. (b) : ‘efpemekeâe DevegYeJe FefvõÙeeW Éeje ve nes mekesâ’ kesâ efueS Skeâ
Jele&veer keâer Âef° mes Megæ nw~ Meyo ‘DeleerefvõÙe’ nw~ FbefvõÙeeW keâes peerleves Jeeues keâes efpelesvõ Deewj
39. efvecveefueefKele ceW mes Skeâ Megæ JeekeäÙe nw, Jen nw– FbefvõÙeeW mes mecyeefvOele keâes ‘Ssefvõkeâ’ keâne peelee nw~
(a) ØelÙeskeâ efJeÅeeLeea keâes ÛeeefnS efkeâ Jen efnvoer keâer Deveskeâ 45. JeekeäÙeebMeeW Deewj Gvekesâ efueS ØeÙegòeâ MeyoeW kesâ
hegmlekeWâ heÌ{s~ efvecveefueefKele ÙegiceeW ceW mener Ùegice keâe ÛeÙeve keâerefpeS~
(b) njskeâ efJeÅeeefLe&ÙeeW keâes ÛeeefnS efkeâ Jen efnvoer keâer Deveskeâ (a) pees Œeer DeefYeveÙe keâjs – DeefYeveslee
hegmlekeWâ heÌ{s~ (b) pees JÙeekeâjCe peevelee nes – JÙeekeâjefCekeâ
(c) njskeâ efJeÅeeLeea keâes ÛeeefnS efkeâ Jen efnvoer keâer DeveskeâeW (c) DeeBKeeW mes hejs – ØelÙe#e
hegmlekeWâ heÌ{s~ (d) ueewškeâj DeeÙee ngDee – ØelÙeeiele
(d) njskeâ efJeÅeeefLe&ÙeeW kesâ ÛeeefnS efkeâ Jen efnvoer keâer DeveskeâeW Ans. (d) : Meyo Deewj JeekeäÙeebMe keâe mener ™he nw–
hegmlekeWâ heÌ{s~ JeekeäÙeebMe Meyo
Ans. (a) : Gòeâ JeekeäÙeeW ceW Megæ JeekeäÙe nQ– ‘ØelÙeskeâ efJeÅeeLeea keâes ueewškeâj DeeÙee ngDee ØelÙeeiele
ÛeeefnS efkeâ Jen efnvoer keâer Deveskeâ hegmlekeWâ heÌ{s~’ DevÙe JeekeäÙe DeMegæ nQ~ pees Œeer DeefYeveÙe keâjs DeefYeves$eer
40. efvecveefueefKele ceW mes Skeâ JeekeäÙe DeMegæ nw, Jen nw– pees JÙeekeâjCe peevelee nes JewÙeekeâjCe
(a) ceesnve iesntB efhemeJeeves Ûekeäkeâer hej ieÙee nw~ DeeBKeeW mes hejs hejes#e/DeØelÙe#e
(b) efveo&Ùe JÙeefòeâ mes efce$elee veneR keâjveer ÛeeefnS~ DeeBKeeW kesâ meeceves ØelÙe#e
(c) Gmekesâ Iej kesâ heeme Skeâ efce"eF& keâer ogkeâeve nw~
46. efvecveefueefKele ceW mes ‘lelmece’ Meyo nw–
(d) GheÙeg&òeâ keâLeve DemelÙe nw~
(a) GÚen (b) Gpeuee
Ans. (c) : efoS ieÙes JeekeäÙeeW ceW DeMegæ JeekeäÙee nQ– ‘Gmekesâ Iej kesâ
(c) Guuet (d) Dees‰
heeme Skeâ efce"eF& keâer ogkeâeve nw~’ Fmekeâe Megæ ™he nesiee– ‘Gmekesâ Iej
kesâ heeme efce"eF& keâer Skeâ ogkeâeve nw~’ DevÙe JeekeäÙe Megæ nw~ Ans. (d) : ‘Dees‰’ lelmece Meyo nw~ DevÙe MeyoeW keâe leÆJe–lelmece
™he nw–
41. ‘efpemekeâe keâesF& Me$eg veneR pevcee nw’ JeekeäÙeebMe kesâ efueS
Skeâ Meyo nw– leÆJe lelmece
(a) DepesÙe (b) Me$egpeÙeer
neW" Dees‰
(c) DepeeleMe$eg (d) Me$egefJenerve
GÚen Glmeen
Gpeuee GppJeue
Ans. (c) : JeekeäÙeebMe kesâ efueS Skeâ Meyo nw–
Meyo JeekeäÙeebMe Guuet Guetkeâ
DepeeleMe$eg efpemekeâe keâesF& Me$eg veneR pevcee nw~ 47. ‘heefjJee’ keâe lelmece ™he nw–
DepesÙe efpemes peerlee ve pee mekesâ~ (a) hejJee (b) hejsJee
Me$egpeÙeer efpemeves Me$eg keâes peerle efueÙee nes~ (c) Øeefleheoe (d) heÌ[erJee
Me$egefJenerve efpemekeâe keâesF& Me$eg ve nes~ Ans. (c) : ‘heefjJee’ keâe lelmece ‘ØeefleØeoe’ nw~ efnvoer ceen kesâ ØelÙeskeâ
42. ‘Keespe keâjves Jeeuee’ Fme JeekeäÙeebMe kesâ efueS Skeâ Meyo he#e kesâ ØeLece efleefLe keâes Øeefleheoe keânles nQ~
nw– 48. efvecveefueefKele lelmece–leÆJe MeyoeW keâe mebiele Ùegile nw–
(a) DevJes<ekeâ (b) Deveghece (a) Kehe&š – KeesheÌ[er (b) meòegâ – melÙe
(c) DeefvJeefle (d) efveJesMekeâ (c) heÙeËkeâ – heuebie (d) Ieesškeâ – IeÌ[e
UP RO/ARO (Pre) Re Exam Hindi 2016 81 YCT
CLICK HERE FOR FREE MATERIAL

Ans. (c) : ‘heÙeËkeâ–heuebie’ lelmece–leÆJe Meyo Ùegice mener nw~ DevÙe 54. ‘‘meewj’’ efkeâme cetue Meyo mes yevee efJeMes<eCe nw?
ÙegiceeW keâe Megæ ™he nw– (a) metj
lelmece leÆJe (b) megj
meòegâ meòet (c) metÙe&
melÙe meÛe (d) GheÙeg&òeâ ceW mes efkeâmeer mes veneR
Ieesškeâ IeesÌ[e Ans. (c) : ‘meewj’ Meyo ‘metÙe&’ cetue Meyo mes yevee efJeMes<eCe nw~ DevÙe
Ieš IeÌ[e Meyo Demebiele nQ~
49. efvecveefueefKele ceW mes ‘leÆJe’ Meyo nw–
55. efvecveefueefKele ceW mes keâewve–mee Meyo efJeMes<eCe nw?
(a) DeeBmet (b) Skeâ$e
(a) Deeie (b) DeeivesÙe
(c) Jeevej (d) GÛÛe
(c) Deefieve (d) GheÙeg&òeâ ceW mes keâesF& veneR
Ans. (a) : ‘DeeBmet’ leÆJe Meyo nw~ Fmekeâe lelmece Deßeg nesiee~ Skeâ$e,
Jeevej Deewj GÛÛe lelmece Meyo nQ~ Fvekeâe leÆJe ›eâceMe: nw– Fkeâªe, Ans. (b) : efoS ieÙes MeyoeW ceW ‘DeeivesÙe’ Meyo efJeMes<eCe nw~ Deeie
yevoj, TBÛee~ meb%ee Meyo nw~ Fmekeâe lelmece ‘Deefive’ neslee nw~ Deeie leÆJe Meyo nw~
50. ‘keâefhelLe’ keâe leÆJe Meyo nw– 56. ‘DeekeâeMe’ keâe heÙee&ÙeJeeÛeer Meyo nw–
(a) keâhetj (b) kewâLee (a) menkeâej (b) MetvÙe
(c) kesâuee (d) Kepeto (c) ØeYee (d) ceb[ueer
Ans. (b) : ‘keâefhelLe’ keâe leÆJe ‘kewâLee’ nw~ DevÙe MeyoeW keâe lelmece Ans. (b) : DeekeâeMe keâe heÙee&ÙeJeeÛeer Meyo– MetvÙe, ieieve, veYe,
nw– keâhetj– keâhet&j, kesâuee– keâoueer, Kepetj–Ketpe&j~ Decyej, JÙeesce Deeefo neslee nw~ ØeYee kesâ heÙee&ÙeJeeÛeer Meyo nQ– Ûecekeâ,
51. efvecveefueefKele MeyoeW ceW mes keâewve–mee Meyo ‘iegCeJeeÛekeâ’ pÙeesefle, oerefhle, keâeefvle, ÚefJe, DeeYee Deeefo~
efJeMes<eCe nw? 57. ‘meesvee’ keâe heÙee&ÙeJeeÛeer Meyo nw–
(a) GefÛele (b) heeBÛeJeeB (a) Deke&â (b) Jeke&â
(c) kegâÚ (d) leerve (c) ceeefCekeäÙe (d) keâvekeâ
Ans. (a) : ‘GefÛele’ Meyo iegCeJeeÛekeâ efJeMes<eCe nw~ heeBÛeJeeB SJeb leerve Ans. (d) : ‘meesvee’ keâe heÙee&ÙeJeeÛeer ‘keâvekeâ’ nw~ Fmekesâ DevÙe heÙee&Ùe
Meyo efveefMÛele mebKÙeeJeeÛekeâ efJeMes<eCe Deewj ‘kegâÚ’ Meyo DeefveefMÛele nQ– mJeCe&, megJeCe&, heg<keâue, nsce, neškeâ, kebâÛeve Deeefo~ Deke&â keâe
mebKÙeeJeeÛekeâ SJeb heefjceeCeJeeÛekeâ efJeMes<eCe nw~ heÙee&ÙeJeeÛeer– metÙe& jefJe, Yeeveg, efovekeâj, efoJeekeâj, Yeemkeâj Deke&â Deeefo
52. efvecveefueefKele ceW mes keâewve–mee efJeMes<eCe meb%ee mes veneR nw~ DevÙe Meyo Demebiele nQ~
yevee nw? 58. ‘Gmekesâ ØeeCe–heKes™ GÌ[ ieÙes~’ Fme JeekeäÙe ceW ‘heKes™’
(a) ßeerceeve (b) oeveer Meyo efkeâmekeâe heÙee&ÙeJeeÛeer nw?
(c) megvoj (d) Ûecekeâeruee
(a) he#eer (b) peuoer
Ans. (c) : ‘megvoj’ efJeMes<eCe Meyo meb%ee mes veneR yevee nw~ ™he jÛevee (c) heKeJeeÌ[e (d) heKeejvee
keâer Âef° mes efJeMess<eCe efJekeâejer Deewj DeefJekeâejer oesveeW nesles nQ, efpemeceW
DeefJekeâejer efJeMes<eCeeW kesâ ™heeW ceW heefjJele&ve veneR nesles~ Ùes Deheves cetue Ans. (a) : ‘Gmekesâ ØeeCe–heKes™ GÌ[ ieÙes~’ Fme JeekeäÙe ceW heKes™
™he ceW yeves jnles nQ~ pewmes– megvoj, heeruee, ieesje, meg[ewue, Yeejer, Meyo he#eer keâe heÙee&ÙeJeeÛeer nw~ Fmekesâ DevÙe heÙee&Ùe nQ– Deb[pe, efJenie,
ÛebÛeue Deeefo~ kegâÚ efJeMes<eCe meb%ee Meyo ceW ØelÙeÙe ueieekeâj yeveles nQ~ Keie, efJenbie, Mekegâefve, helebie, heefjvoe, efÛeefÌ[Ùee Deeefo~ peuoer keâe
pewmes– Oece& ± Fkeâ (ØelÙeÙe) · Oeeefce&keâ, Ûecekeâ ± F&uee (ØelÙeÙe) · heÙee&Ùe leer›e, õgle nesiee~
Ûecekeâeruee Deeefo~ 59. efvecveefueefKele ceW mes keâewve–mee Meyo ‘Ûevõcee’ keâe
53. efvecveefueefKele ceW mes efkeâme JeekeäÙe ceW GòeceeJemLee heÙee&ÙeJeeÛeer veneR nw?
iegCeJeeÛekeâ efJeMes<eCe–efJeMes<Ùe keâe ØeÙeesie efkeâÙee ieÙee nw? (a) megOeebMeg (b) megOeekeâj
(a) hejceevevo keâ#ee ceW meyemes nesefMeÙeej Úe$e nw~ (c) megOeeOej (d) meefueue
(b) ceveceesnve ves Fme ogkeâeve mes Skeâ efkeâuees šceešj Kejeroe nw~ Ans. (d) : ‘meefueue’ Meyo Ûevõcee keâe heÙee&ÙeJeeÛeer veneR nw Deefheleg
(c) Fme heewOes ceW Flevee heeveer cele [euees~ Ùen heeveer keâe heÙee&Ùe nw~ heeveer kesâ DevÙe heÙee&Ùe nQ– veerj, leesÙe, Decyeg,
(d) efJepeÙe henueJeeve Skeâ efove ceW Ûeej ueeršj otOe heerlee nw~ Jeeefj Deeefo~ megOeebMeg, megOeekeâj, megOeekeâj, ceÙebkeâ, MeefMe, Ûevõ, jekesâMe
Ans. (a) : ‘hejceevevo keâ#ee ceW meyemes nesefMeÙeej Úe$e nw~’ JeekeäÙe ceW Deeefo Ûevõcee kesâ heÙee&ÙeJeeÛeer nQ~
GòeceeJemLee iegCeJeeÛekeâ efJeMes<eCe–efJeMes<Ùe keâe ØeÙeesie efkeâÙee ieÙee nw~ 60. efvecveebefkeâle MeyoeW ceW mes Skeâ Meyo ’ceelee’ keâe
efJeMes<eCe keâer leerve DeJemLeeÙeW nesleer nQ– (1) cetueeJemLee (2) GòejeJemLee heÙee&ÙeJeeÛeer veneR nw, Jen nw–
Deewj (3) GòeceeJemLee~ cetueeJemLee– efpemeceW efkeâmeer meb%ee Ùee meJe&veece
(a) Decye (b) Decyeg
keâer meeceevÙe efmLeefle keâe keâe yeesOe nes~ pewmes– ceesnve DeÛÚe ueÌ[keâe nw~
GòejeJemLee– efpemeceW oes meb%ee Ùee meJe&veece keâer leguevee keâer peeleer nw~ (c) Decyee (d) peveveer
pewmes– censMe, jengue mes DeÛÚe nw~ GòeceeJemLee– efpemeceW oes mes Ans. (b) : ‘Decyeg’ ceelee keâe heÙee&ÙeJeeÛeer veneR nw Deefheleg ‘heeveer’ keâe
DeefOekeâ meb%ee Ùee meJe&veeceeW keâer leguevee keâjkesâ, Skeâ keâes meyemes DeÛÚe heÙee&ÙeJeeÛeer nQ Decye, Decyee, peveveer, ceele=, Oee$eer Deeefo ceelee kesâ
Ùee yegje yeleeÙee peelee nw~ pewmes– Dekeâyej meyemes DeÛÚe nw~ heÙee&ÙeJeeÛeer nQ~
UP RO/ARO (Pre) Re Exam Hindi 2016 82 YCT
CLICK HERE FOR FREE MATERIAL

UPPSC RO-ARO (Mains) Re-Exam-2017


GENERAL STUDIES
Solved Paper
1. Transport, Communication, Commerce come Ans. (a) :
under the- List-I List-II
(a) Primary activities (b) Secondary activities (Author) (Title of the Book)
(c) Tertiary activities (d) Rural activities A. V.D. Savarkar - The Indian War of
Ans. (c) : Tertiary sector includes a wide range of Independence
activities from commerce to administration, transport, B. RC. Majumdar - The Sepoy Mutiny and The
finance and real estate, communication and personal Revolt of 1857
services, education, health etc. C. S.B. Chaudhuri - Civil Rebellion in the Indian
2. 'Alha' is a popular folk song of which area- Mutinies (1857-59)
(a) Rohilkhand (b) Bundelkhand D. Ashok Mehta - The Great Rebellion
(c) Purvanchal (d) Western Uttar Pradesh 5. In which of the following state is Black Pepper
Ans. (b) : 'Alha folk song' is famous in Bundelkhand and Cardamom produced?
region of Madhya Pradesh. (a) Assam (b) Jammu and Kashmir
(c) Himachal Pradesh (d) Kerala
3. Consider the following events and arrange
them in chronological order Ans. (d) : Kerala is the largest producer of Cardamom
& Black Pepper in India. It is also known as the Spice
I. Battle of Daurah
garden of India.
II. Battle of Kannauj
6. Which of the following does not require a
III. Battle of Samugarh medium?
IV. Battle of Chausa (a) Radiation (b) Convection
Select the correct answer using the code given (c) Conduction (d) None of these
below: Ans. (a) : Radiation are electromagnetic waves that
(a) II, III, I, IV (b) I, IV, II, III travel without any medium, Rest both conduction &
(c) I, II, IV, III (d) II, I, IV, III convection takes place in specific medium.
Ans. (b) : The correct chronological order is: 7. Which work as intermediaries between biotic
Battle Year and abiotic components-
Battle of Daurah - 1532 (a) Parasite (b) Decomposers
Battle of Chausa - 1539 (c) Producers (d) Consumers
Battle of Kannauj - 1540 Ans. (c) : Producers are the connectors or intermediate
Battle of Samugarh - 1658 organisms linking the other organism (biotic) with the
abiotic components of the environment.
4. Match List-I with List-II and choose the
8. Gupta gold coin was called-
correct answer from the codes given below:
(a) Karshapana (b) Dinara
List-I List-II (c) Nishka (d) Suvarna
(Author) (Title of the Book) Ans. (b) : During Gupta period, Gold Coins were called
A. V.D. Savarkar 1. The Great Rebellion Dinara. The face of the coins generally portrayed the
B. RC. Majumdar 2. The Indian war of governing kings and carried legends, while the reverse side
Independence depict the figure of a goddess. It is the most exceptional
instances of numismatics and artistic perfection.
C. S.B. Chaudhuri 3. The Sepoy Mutiny and
9. Match List-I (Naturally Occurring Substances)
The Revolt of 1857 with List-II (Elements) and select the correct
D. Ashok Mehta 4. Civil Rebellion in the answer from the codes given below the lists:
Indian Mutinies (1857-59) List-I List-II
Codes: (Author) (Title of the Book)
A B C D A. Diamond 1. Calcium
(a) 2 3 4 1 B. Marble 2. Silicon
(b) 2 3 1 4
C. Sand 3. Aluminium oxide
(c) 3 2 4 1
(d) 3 1 2 4 D. Ruby 4. Carbon

UP RO/ARO (Mains) Exam-2017 83 YCT


CLICK HERE FOR FREE MATERIAL

Codes: 13. In West Asia, the highest Petroleum producing


A B C D country is:
(a) 1 2 3 4 (a) Iraq (b) Iran
(b) 4 1 2 3 (c) Saudi Arabia (d) Lebanon
(c) 3 1 2 4 Ans. (c) : The West Asian Countries held the world's
(d) 4 2 1 3 largest oil reserve until 2009, Thereafter, it was
Ans. (b) : The correct pairs are: Surpassed by Venezuela. Saudi Arabia is the World's
A. Diamond - Carbon third largest oil producer after the U.S. and Russia.
Saudi Arabia possesses around 15-17% of the world's
B. Marble - Calcium petroleum reserves.
C. Sand - Silicon 14. Which of the following is not among the major
D. Ruby - Aluminium oxide resources of Russia?
10. Match List-I with List-II and select the correct (a) Coal (b) Iron-ore
answer using the codes given below: (c) Diamonds (d) Uranium
List-I List-II Ans. (d) : Among the given options, Uranium is not the
A. Ashtadhyayi 1. Yaska major resources of Russia. While Australia followed by
Kazakhstan possess the major Uranium resources.
B. Mahabhashya 2. Katyayan
15. What will come in place of question mark?
C. Nirukta 3. Patanjali 9 11 99
D. Varttika 4. Panini 10 25 50
Codes: 28 35 ?
A B C D (a) 110 (b) 140
(a) 4 3 1 2 (c) 148 (d) 175
(b) 2 3 1 4 Ans. (b) : Just as –
(c) 1 2 3 4
L.C.M. of 9, 11 = 99
(d) 3 1 4 2
And L.C.M. of 10m, 25 = 50
Ans. (a) : The correct pairs are: Same as-
List-I List-II L.C.M. of 28 and 35 = 140
A. Ashtadhyayi - Panini 16. As part of New Forest Inventory Design
B. Mahabhashya - Patanjali implemented in 2016, the revisit time has been
C. Nirukta - Yaska reduced to-
(a) 5 years (b) 2 years
D. Varttika - Katyayan (c) 6 months (d) 10 years
11. Which of the following is not a greenhouse gas? Ans. (a) : The Forest Survey of India (FSI) has devised
(a) Oxygen (b) Carbon dioxide a new grid-based assessment of the national forest
(c) Chlorofluorocarbons (d) Methane inventory (NFI) which will yield data about the status of
Ans. (a) : Here, in the given option oxygen is not a forest of the country in a Span of just five years, as
greenhouse gas. Greenhouse gases are carbondioxide, compared to the 20 years time-Span in which it was
methane, chlorofluorocarbons (CFCs), nitrous oxide earlier prepared.
and ozone etc. 17. Complete the following series-
12. Which number will come at the place of? in the 2, 9, 28, 65, ?
following table/figure? (a) 121 (b) 195
1 3 7 (c) 126 (d) 103
5 12 14 Ans. (c) : The question series is as follows.
25 ? 28 2 9 28 65 126
125 192 56 ↑ ↑ ↑ ↑ ↑
(a) 40 (b) 48
1 +1 2 +1 3 +1 4 +1 53 +1
3 3 3 3
(c) 56 (d) 64
Hence the next term of the series is 126.
5 × 25 14 × 28
Ans. (b) : Just as – = 125 and = 56 18. Given below are two statements, one labelled as
1 7 Assertion (A) and other as reason (R).
Same as
Assertion (A) : The biosphere is an example of
12 × ? open system.
= 192
3 Reason (R) : There can not be reinvestment of
⇒ ? = 16 × 3 new substances in the bio-sphere but there is no
⇒ ? = 48 restriction on the investment and exit of energy.
UP RO/ARO (Mains) Exam-2017 84 YCT
CLICK HERE FOR FREE MATERIAL

Select the correct answer using the codes Ans. (d) : According to the National Capital Region
below: Planning Board Act, 1985 total 26 districts in the adjoining
(a) Both (A) and (R) are true and (R) is correctstates of Uttar Pradesh, Haryana and Rajasthan as well as
explanation of (A) National Capital Territory of Delhi comprise of the
(b) Both (A) and (R) are true but (R) is not theNational Capital Region (NCR) of India. It is an unique
correct explanation of (A) example of inter-state regional planning and development
for a region with NCT-Delhi as its core.
(c) (A) is true but (R) is false
22. At current rate of growth, the urban
(d) (A) is false but (R) is true
population of India by the year 2030 will reach:
Ans. (a) : As per the guidelines of UNESCO (MAB) (a) 575 million (b) 675 million
Programme, Biosphere Reserve is an open system. (c) 750 million (d) 900 million
In the biosphere reserves, frequent investmentAns. (*) : As per the MoHUA, At the current rate of
and exit of energy is possible but there can not be growth, urban population in India is estimated to reach a
reinvestment of new substances in the biosphere. staggering 600 million or 60 crores by 2030.
Therefore, Both (A) and (R) are true and (R) is
23. Who is the father of scientific management?
correct explanation of (A). (a) Henry Fayol (b) Elton Mayo
19. Which Union Territory of India has the lowest (c) Cheston Bernard (d) F.W. Taylor
Sex-Ratio as per the 2011 census Ans. (d) : Fredrick Winslow Taylor is known as the
(a) Chandigarh Father of Scientific Management, which also came to be
(b) Dadra and Nagar Haveli known as 'Taylorism'. Fredrick Taylor's theory is a
(c) Daman and Diu theory of management.
(d) Lakshadweep 24. In long-sightedness, image are formed at
Ans. (c) : According to the 2011 census, Daman & Diu following-
has the lowest sex ratio with 618 females per 1000 (a) On Retina (b) In front of Retina
males. Whereas, Chandigarh have 818, Dadra Nagar (c) Behind Retina (d) On blind Spot
Haveli have 774 and Lakshadweep have 946 per 1000 Ans. (c) : Far-sightedness, also known as long-
males. While among the states, Haryana has the lowest sightedness, hypermetropia or hyperopia is a condition of
sex-ratio with 879 females per 1000 males. the eye where distant objects are seen clearly but near
objects appears blurred. This blurred effect is due to
20. Match List-I with List-II and select the correct incoming light being focused behind, instead of on, the
answer using the codes given below: retina wall due to insufficient accommodation by the lens.
List-I List-II 25. Consider the following events and arrange
(Country) (Principal Primary them in chronological order:
Export) I. Cripps Mission
A. Chad 1. Cotton II. August Offer
III. Nehru Report
B. Niger 2. Uranium
IV. Wavell Plan
C. Rwanda 3. Petroleum Select correct answer from the codes given
D. Benin 4. Coffee below:
Codes: Code:
(a) III, I, II, IV (b) III, II, I, IV
A B C D
(c) II, I, III, IV (d) I, III, II, IV
(a) 4 1 3 2
Ans. (b) : The correct chronological order of the given
(b) 2 4 1 3 events are-
(c) 3 2 4 1 Events Year
(d) 1 3 2 4 1. Nehru Report - 1928
Ans. (c) : The correct pairs are: 2. August Offer - 1940
List-I List-II 3. Cripps Mission - 1942
(Country) (Principal Primary Export) 4. Wavell Plan - 1945
A. Chad - Petroleum 26. Which of the following pair is not correctly
matched.
B. Niger - Uranium
List-I List-II
C. Rwanda - Coffee (Crop/Plantation) (Largest Producer State)
D. Benin - Cotton A. Jute 1. West Bengal
21. Part of which state of the country is not B. Tea 2. Assam
included in National Capital Region.
C. Sugarcane 3. Uttar Pradesh
(a) Delhi (b) Uttar Pradesh
(c) Haryana (d) Madhya Pradesh D. Rubber 4. Himachal Pradesh
UP RO/ARO (Mains) Exam-2017 85 YCT
CLICK HERE FOR FREE MATERIAL

Ans. (d) : 31. Who translated 'Baburnama' into English?


(a) Leyden and Erskine (b) William Hawkins
List-I List-II
(c) Finch (d) William Jones
(Crop/Plantation) (Largest Producer State)
Ans. (a) : The first ones to translate Babur's memoirs
A. Jute - West Bengal into the English Language were John Leyden and
B. Tea - Assam William Erskine. This translated form of the Baburnama
C. Sugarcane - Uttar Pradesh was named as "Memoirs of Zahir-ud-Din Muhammed
Babur, Emperor of Hindustan."
D. Rubber - Kerala 32. Match List-I with List-II and select the correct
27. On the banks of which river did Mahavira answer using the codes given below:
Swami attained enlightenment? List-I List-II
(a) Swarnasikta (b) Palashini (Author) (Title of the Book)
(c) Ganga (d) Rijupalika A. Stupa 1. Bagh
Ans. (d) : At the age of 42 Mahavira attained Kaivalaya B. Painting 2. Sanchi
under a Sal tree on the bank of river Rijupalika near
C. Headless statue of 3. Deogarh
Jimbhikgrama in Modern Bihar.
Kanishka
28. A man is facing east, rotates in clockwise D. Dashvatara Temple 4. Mathura
direction 60º and then anti-clockwise direction
255º and finally clockwise direction 105º. Codes:
Which direction is he facing now? A B C D
(a) North-East (b) North (a) 1 2 3 4
(b) 2 1 4 3
(c) West (d) South-East
(c) 4 3 2 1
Ans. (b) : (d) 3 2 1 4
Ans. (b) :
List-I List-II
A. Stupa - Sanchi
B. Painting - Bagh
C. Headless statue of Kanishka Mathura
D. Dashvatara Temple - Deogarh
Hence, It is clear that the final position of man will be 33. Match List-I with List-II and select the correct
facing north direction. answer using the codes given below the lists:
29. Improvement of profit volume ratio can be List-I List-II
done by the following- A. Ballabhacharya 1. Dwaitvad
(a) Increasing selling price B. Ramanuj 2. Pushtimarg
(b) Altering sales mixture
(c) Reducing Variable cost C. Madhvacharya 3. Vishistadwait
(d) All of the above D. Shankar 4. Adwaitvad
Ans. (d) : Ways to improve profit volume ratio, Codes:
• By reducing variable cost, or A B C D
• By increasing the selling price, or (a) 1 3 4 2
(b) 2 4 1 3
• By improving sales mixture
(c) 2 3 1 4
• Reducing direct and variable costs by effectively (d) 4 1 3 2
utilizing men, machines and materials.
Ans. (c) :
• Switching the production to more profitable
products showing a higher profit volume ratio. List-I List-II
A P/V ratio indicates the rate of profitability and any A. Ballabhacharya - Pushtimarg
improvement in this ratio without increase in fixed B. Ramanuj - Vishistadwait
costs, would result in higher profits. C. Madhvacharya - Dwaitvad
30. For every action there is an equal and opposite D. Shankar - Adwaitvad
reaction. This law was given by:
34. Which Article of the Indian Constitution
(a) Newton (b) Albert Einstein
provides right to vote i.e. adult suffrage for
(c) Bohr (d) Handane electing representatives of Lok Sabha and
Ans. (a) : For every action, there is an equal and Vidhan Sabha?
opposite reaction, this law is governed by Newton's (a) Article 326 (b) Article 325
Third Law of Motion. (c) Article 324 (d) Article 323
UP RO/ARO (Mains) Exam-2017 86 YCT
CLICK HERE FOR FREE MATERIAL

Ans. (a) : The principle of Adult Suffrage is enshrined 40. Consider the following statements about
under Article 326 of the Constitution of India. Western Ghats-
35. Malampuzha dam is situated on which river? 1. It is a block mountain
(a) Bharatpuzha (b) Korapuzha 2. Its eastern slope is gentle
(c) Malampuzha (d) Pamba 3. Its northern section is covered by lava
Ans. (c) : Malampuzha Dam is built across the river (Basalt)
Malampuzha about 8 km from the Palakkad town in the Which of the above statements are correct?
state of Kerala. The construction of Malampuzha dam (a) 1 and 2 (b) 2 and 3
started in the year 1949 and was completed in the year 1955. (c) 1 and 3 (d) 1, 2 and 3
36. Which number is wrong in the given series? Ans. (d) : The Western Ghats, also known as the
1235, 2425, 3158, 4235 Sahyadri hills are well known for their rich and unique
(a) 1235 (b) 2425 assemblage of flora & fauna. The mountain of the
(c) 3158 (d) 4235 Western Ghats are Block Mountains formed due to the
Ans. (c) : 3158 is the wrong number in the given series down warping of a part of land into the Arabian Sea. Its
because it is the only number in the series which is not northern section is covered by lava (Basalt) and also has
divisible by 5. gentle slope on eastern side compared to western steep
side. Hence, all the options are correct.
37. Match List-I with List-II and select the correct
answer using the codes given below: 41. Which one is a fundamental particle?
(a) Neutron (b) Proton
List-I List-II
(News Paper) (Publisher) (c) π−meson (d) quark
A. Bengalee 1. Jogendra Nath Bose Ans. (d) : A quark is a type of elementary particle and a
fundamental constituent of matter. Quarks combine to
B. Bangbasi 2. Arvind Ghosh form composite particles called hadrons, the most stable
C. Vande Mataram 3. Surendranath of which are protons and neutrons, the components of
Banerjee atomic nuclei.
D. Amrit Bazar 4. Moti Lal Ghosh 42. Superior Lake region in North America is
Patrika famous for-
Codes: (a) iron-ore reserves (b) coal reserves
A B C D (c) gold reserves (d) uranium reserves
(a) 2 1 3 4 Ans. (a) : Superior Lake Region in North America is
(b) 3 1 2 4 famous for Iron ore reserves. The Iron Ranges of the
(c) 3 4 2 1 lake superior region are-
(d) 4 3 2 1 • Mesabi Range
Ans. (b) : • Vermillion Range
List-I List-II • Gunflint Range
A. Bengalee - Surendranath Banerjee • Cuyuna Range
• Gogebic Range
B. Bangbasi - Jogendra Nath Bose
• Marquette range and
C. Vande Mataram - Arvind Ghosh
• Menominee Range.
D. Amrit Bazar Patrika - Moti Lal Ghosh 43. Who among the following has not scored a
38. Chitrakoot waterfall is situated in: century on Test debut?
(a) Chitrakote (b) Bastar (a) Sourav Ganguly (b) Virendra Sehwag
(c) Jabalpur (d) Banda (c) Prithvi Shaw (d) Sunil Gavaskar
Ans. (b) : The famous Chitrakote falls, "Niagra of Ans. (d) : Sunil Gavaskar has not scored a century on
India" is situated on the river Indravati in Bastar district test debut among the given options.
of Chhattisgarh State of India. The height of the 44. Whom did Simona Halep defeat to win the
waterfall is 90 feet. It is the largest and most water- French Open Tennis Tournament 2018?
logged waterfall in the Chhattisgarh state.
(a) Sloane Stephens (b) Serena Williams
39. Complete the sequence with the correct option (c) Venus Williams (d) None of these
from the following codes.
Ans. (a) : Simona Halep defeated Sloane Stephens in
V, I, B,….,….,….,….
the final, to win the women's singles tennis title at the
(a) G, Y, O, R (b) G, O, Y, R 2018 french open. Halep became the second Romanian
(c) G, O, R, Y (d) Y, G, O, R Woman to win a major singles title after Virginia Ruzici
Ans. (a) : The following sequence is a ascending order in 1978.
of wavelength of dispersion of light. 45. Harnessing of nuclear energy often causes:
V, I, B, G, Y, O, R (a) Air Pollution (b) Water Pollution
Hence, option (a) is correct. (c) Thermal Pollution (d) Noise Pollution
UP RO/ARO (Mains) Exam-2017 87 YCT
CLICK HERE FOR FREE MATERIAL

Ans. (c) : Harnessing of nuclear energy often cause 49. Ozone layer of upper atmosphere is being
thermal pollution. A nuclear power plant is a thermal destroyed by which of the following?
power station in which the heat is used to generate (a) Sulphur dioxide
steam that drives a steam turbine connected to a (b) Photo-chemical oxidants
generator that produces electricity. The high (c) Chlorofluorocarbons
temperature produced during a nuclear reaction causes (d) Smog
thermal pollution.
Ans. (c) : By the action of UV rays on the oxygen
46. In the matrix given below, what are the values molecules, ozone layer is formed in the stratosphere.
of A, B and C This layer acts as absorbent for the harmful UV
9 A 12 radiations. However, Ozone layer is being destroyed by
the compounds that contain chlorine, fluorine and
B 10 7 carbon, these compound are commonly known as
8 C 11 freons. These are chlorofluorocarbons. The aerosol
sprays and refrigerants are the main source.
(a) A = 9, B = 11, C = 13
(b) A = 13, B = 9, C = 11 50. Which one of the following vitamins is formed
in skin in the presence of sunlight?
(c) A = 9; B = 13, C = 11
(a) Vitamin A (b) Vitamin D
(d) A = 13, B = 11, C = 9
(c) Vitamin K (d) Vitamin E
Ans. (C) : Ans. (b) : Vitamin D is the Sunshine Vitamin. During
9 A 12 Exposure to Sunlight 7-dehydrocholestrol in the skin
absorbs UVB radiation and is converted to previtamin
B 10 7 D3 which in turn isomerizes into vitamin D3. Vitamin
8 C 11 D is fat-soluble vitamin, responsible for increasing
On adding the numbers in the given table horizontally, intestinal absorption of calcium, magnesium and
vertically and diagonally, their sum is 30. phosphate.
∴ 9 + A + 12 = 30 ⇒ A = 30 – 21 51. Which of these is not a viral disease?
⇒A=9 (a) Measles (b) Rabies
(c) Polio (d) Tuberculosis
B + 10 + 7 = 30 ⇒ B = 30 – 17
⇒ B = 13 Ans. (d) : Tuberculosis is caused by the bacteria
Mycobacterium tuberculosis. Hence, it is not a viral
8 + C + 11 = 30 ⇒ C = 30 – 19 disease.
⇒ C = 11
52. Order of the size of Nucleus is following:
Hence, A = 9, B = 13 and C = 11 (a) 1 × 1015 meter (b) 1 × 1012 meter
47. Which of the following alternatives will come in –10
(c) 1 × 10 meter (d) None of above
place of X?
Ans. (a) : According to Rutherford the nucleus size is
27, 31, 40, 56, 81, 117, X
10–15m.
(a) 156 (b) 165
(c) 166 (d) 169 53. What kind of tax is G.S.T.?
(a) Direct Tax
Ans. (c) : The given series is a follows. (b) Indirect Tax
(c) Depends on the type of goods and services
(d) None of the above
Hence 166 will be in place of x. Ans. (b) : GST is an indirect tax for the whole nation,
48. A bill other than Money Bill which is passed by which will make India one unified Common Market.
Legislative Assembly for the first time, may GST is a Single Tax on the supply of goods and
remain pending in the legislative council for services, right from the manufacturer to the consumer.
how many months? 54. Indian and Asia's first Dolphin Research
(a) For not more than one month Centre - NDRC is being set up on the banks of
(b) For not more than two months the Ganga river at
(c) For not more than three months (a) Haridwar (b) Patna
(d) None of the above (c) Varanasi (d) Bhagalpur
Ans. (d) : An ordinary bill can be introduced in both the Ans. (b) : India's and Asia's First, National Dolphin
houses, but in case of disagreement, the Legislative Research Centre (NDRC) was set up on the banks of the
Assembly prevails. Ganga river in Patna University campus in Patna.
According to Article 197 (b) A bill other than money Gangetic river dolphin scientific name is Platanista
Bill which is passed by Legislative Assembly for the Gangetica. India's only dolphin Sanctuary Vikramshila
first time, may remain pending in the legislative council Gangetic Dolphin Sanctuary is situated in Bhagalpur
for more than three months. Bihar. It comes under schedule I of the Indian wildlife
The legislative council can delay an ordinary bill for (Protection) Act, 1972 and have been declared an
maximum 4 months. endangered species by IUCN.

UP RO/ARO (Mains) Exam-2017 88 YCT


CLICK HERE FOR FREE MATERIAL

55. Which one of the following will not have a 61. Match List-I with List-II and choose the
direct impact on human capital formation? correct answer using the code given below:
(a) Education (b) Medical Care List-I List-II
(c) Training (d) Irrigation
A. Black Pottery 1. Moradabad
Ans. (d) : The term Human Capital refers to the
economic value of a worker's experience and skills. It B. Ceramic Pottery 2. Khurja
includes assets like education, training, intelligence, C. Brass Artifact 3. Firozabad
skills, health and other things employers value such as
loyalty and punctuality. Human capital is perceived to D. Glass Artifact 4. Nizamabad
increase productivity and thus profitability. Hence, Codes:
except irrigation all others will have a direct impact on A B C D
human capital. (a) 1 3 2 4
56. Which of the following is not considered as (b) 4 2 1 3
social indicator of poverty? (c) 3 1 4 2
(a) Less number of means of transport (d) 2 4 3 1
(b) Illiteracy level
Ans. (b) : The correct answer are-
(c) Lack of access to health care
(d) Lack of job opportunities List-I List-II
Ans. (a) : Poverty line is the level of income from A. Black Pottery - Nizamabad
which a person is unable to meet his material needs due B. Ceramic Pottery - Khurja
to low income. Illiteracy level, lack of access to health
C. Brass Artifact - Moradabad
care and lack of employment opportunities are
considered social indicators of poverty whereas the less D. Glass Artifact - Firozabad
number of means of transport does not include this 62. Where in Uttar Pradesh is located the State
57. Where in Uttar Pradesh is Knowledge Park Leather Development and Marketing
being established. Corporation?
(a) Lucknow (b) Prayagraj (a) Agra (b) Kanpur
(c) Varanasi (d) Greater NOIDA (c) Moradabad (d) Ghaziabad
Ans. (d) : A large knowledge park is being built in the Ans. (a) : Uttar Pradesh State Leather Development and
city of Greater Noida. It will act as a technology hub for Marketing Corporation is an unlisted company
the companies and people in the surrounding areas. incorporated on 12 February, 1974. It is classified as a state
58. 'Flower' is related to 'Petals' in the same way govt. Company and is located in Agra, Uttar Pradesh.
as 'Book' is related to: 63. Where in Uttar Pradesh is Artificial Limbs
(a) Library (b) Author Manufacturing Corporation situated?
(c) Pages (d) Content (a) Varanasi (b) Prayagraj
Ans. (c) : In the same way, Book is related to pages. (c) Kanpur (d) Agra
59. Uruguay round pertains to- Ans. (c) : Artificial Limps Manufacturing Corporation
(a) WTO (b) IMF of India (ALIMCO) is a schedule 'C' Miniratna category
(c) GATT (d) World Bank central public sector enterprise. The corporation started
Ans. (c) : The Uruguay Round was the 8th round of manufacturing artificial aids in 1976.
Multilateral Trade Negotiations (MTN) conducted 64. Who is the Fiscal agent and advisor to
within the framework of the General Agreement on government in monetary and financial matters?
Tariffs and Trade (GATT). Spanning from 1986 to 1994 (a) NABARD (b) S.B.I.
and embracing 123 countries as "Contracting Parties". (c) R.B.I. (d) None of the above
The negotiations and process ended with the signing of
the final act of the Marrakesh Agreement in April 1994 Ans. (c) : The Reserve Bank acts as an adviser to the
at Marrakesh, Morocco. Government, whenever called upon to do so, on
60. Which of the following commodities has highest monetary and banking related matters.
export from India in 2017? 65. 'Uttar Pradesh Ratna Award' has been
(a) Agriculture and allied products conferred on which Indian - American?
(b) Engineering goods (a) Sam Pitroda (b) Sunita Williams
(c) Textiles (c) Frank Islam (d) Kalpana Chawla
(d) Chemicals Ans. (c) : India - American entrepreneur and
Ans. (b) : During June 2017, Major commodity groups philanthropist, Mr. Frank Islam (63) has been chosen
of export showing positive growth over the for the inaugural Uttar Pradesh Ratna Award for his
corresponding month of last year are Engineering achievements and outstanding contributions in a foreign
Goods (14.78%) Petroleum Products (3.60%), organic country. He was born in Uttar Pradesh's Azamgarh
& Inorganic chemicals (13.20%), Rice (27.29%) and district and studied at Aligarh Muslim University. He
Marine Products (24.27%). was honoured on U.P. Pravasi Diwas in Agra on
UP RO/ARO (Mains) Exam-2017 89 YCT
CLICK HERE FOR FREE MATERIAL

January 4, 2016. This award has been instituted to 70. Part 16 of the Indian Constitution is related to:
recognize achievements and outstanding contributions (a) Administrative Tribunal
of NRI's who hail from the state. (b) All India Services
66. Which of the following is not correctly (c) Finance Commission
matched? (d) Special provisions for certain classes
(a) Anga - Champa Ans. (d) : Part XVI of our Constitution contains the
(b) Koshala - Ahichhatra special provision relating to certain classes. It contains
(c) Vatsa - Kaushambi the article-330 to 342 (a).
(d) Matsya - Viratnagar 71. Net export equals to:
Ans. (b) : The correct pairs are (a) Export × Imports
Mahajanapadas Capital (b) Export + Imports
a. Anga - Champa (c) Export – Imports
(d) Exports of services only
b. Koshala - Shravasti (Northern)
Kushavati (Southern) Ans. (c) : Net exports are a measure of a nation's total
trade. The formula for net exports is a simple one- The
c. Vatsa - Kaushambi
value of a nation's total export of goods and services
d. Matsya - Viratnagar minus the value of all the goods and services it imports
Ahichhatra, was the capital of the Panchala Kingdom, equal its net exports.
which was one of the 16 Mahajanpadas or Great 72. Match List-I with List-II and select the correct
Republics during 6th to 4th B.C.E. answer using the codes given below:
67. Which of the following crop is transplanted? List-I List-II
(a) Mustard (b) Rice (Battles) (Year)
(c) Wheat (d) Maize A. Battle of Chandawar 1. 1398
Ans. (b) : The technique of moving a plant from a place
B. Invasion of Taimur 2. 1194
where it has been growing and replanting at another site
during the growing season is called transplantation. C. Battle of Talikota 3. 1529
Paddy (Rice) is the common example of crops mainly D. Battle of Ghaghra 4. 1565
grown by transplantation.
68. Who among the following has been appointed Codes:
Goodwill Ambassador for tourism and culture A B C D
by the Nepalese Government? (a) 1 2 3 4
(a) Jaya Prada (b) Hema Malini (b) 2 3 4 1
(c) Priyanka Chopra (d) None of these (c) 1 4 3 2
Ans. (a) : The government of Nepal appointed former (d) 2 1 4 3
member of parliament and renowned Indian actress Jaya Ans. (d) :
Prada as Goodwill Ambassador for tourism and culture List-I List-II
for four years. (Battles) (Year)
69. Consider the following statements regarding A. Battle of Chandawar - 1194
Lucknow metropolitan.
1. In the later half of 18th Century, Nawab B. Invasion of Taimur - 1398
Aasaf-ud-daula developed the town in a C. Battle of Talikota - 1565
planned manner. D. Battle of Ghaghra - 1529
2. In British period, Sir Harcourt Butler gave
attention to its shapely development. 73. Which Union Territory of India has the lowest
3. Residential area is found on 23% part of the literacy rate as per the Census of 2011?
total land of the Metropolis. (a) Chandigarh
Which is correct of the above statements? (b) Dadra and Nagar Haveli
Code: (c) Daman and Diu
(a) Only 2 (b) 1 and 2 (d) Lakshadweep
(c) 2 and 3 (d) 1, 2 and 3 Ans. (b) : Dadra and Nagar Haveli is a union territory
Ans. (b) : Nawab Aasaf-ud-Daula, who ruled from in Western India. It is a U.T. with lowest recorded
1775 to 1797, shifted the capital from Faizabad to literacy rate, according to 2011 Census 76.24% is the
Lucknow in 1775 A.D and developed the town in a total literacy rate in Dadra and Nagar Haveli.
planned manner. Sir Harcourt Butler gave attention to 74. Which of the following is not correctly
its shapely development during British period. As per matched?
the City development plan of Lucknow Municipal (a) Baikal - Russian Federation
Corporation more than 85% of the total land of the (b) Malawi - Asia
metropolis is covered under residential area. Thus, (c) Caspian Sea - Asia - Europe
statement (3) is incorrect. (d) Tanganyika - Africa
UP RO/ARO (Mains) Exam-2017 90 YCT
CLICK HERE FOR FREE MATERIAL

Ans. (b) : Malawi is a land locked country in Ans. (a) : Sugar Industry is more developed in South
Southeastern Africa, bordered by Zambia to the West, India, the reason include-
Tanzania to the north & northeast and Mozambique to • The tropical climate of peninsular India results in
the east, South and Southwest. higher yield per unit hectare of land.
75. Which country topped the medal tally in 2018 • Higher sucrose content in peninsular cane.
Commonwealth Games?
(a) Australia (b) England • Long Crushing Season in South. In North India, it
(c) Canada (d) India lasts from November to February (4 months) while
in South it lasts from October to May or even June
Ans. (a) : The 2018 Commonwealth Games (officially (nearly 8 months).
known as the XXI commonwealth games), was a
multisport event held in Gold Coast, Queensland, Also, the mills in peninsular India are larger and the
Australian. In this event Australia won the games with 78 mills are in form of cooperative sector in South, which
gold, 59 silver and 59 bronze for a total of 198 medals. are better managed.
76. Which place in Uttar Pradesh is famous for 81. Which of the following word cannot be made
'Surma' Industry? from the letters of word DETERMINATION.
(a) Varanasi (b) Prayagraj (a) TERMIDE (b) MODERN
(c) Moradabad (d) Bareilly (c) TENDER (d) MOTHER
Ans. (d) : Bareilly District of Uttar Pradesh is famous Ans. (d) : The words TERMIDE, MODERN and
for Surma Industry. TENDER can be formed from the letters at the word
77. A tariff- 'DETERMINATION' while the word MOTHER cannot
(a) increases the volume of trade be formed because there is no letter 'H' in the word.
(b) reduces the volume of trade 82. In Uttar Pradesh, which of the following tribe
(c) has no effect on volume of trade celebrates Diwali as a sorrow?
(d) both (a) and (c) (a) Sahariya (b) Baiga
Ans. (b) : A tariff is a tax imposed on the import or (c) Parahariya (d) Tharu
export of goods. In general parlance, however, a tariff Ans. (d) : In Tharu community (Tribe) traditions, this
refers to 'import duties' charged at the time goods are
day used to be observed as a day of sorrow and they
imported. So by imposing tariff a price of the product
goes up and will naturally reduce the volume of trade. used to offer a "roti" to the deceased family member.
78. Who is the winner of Nobel Prize for Literature 83. Maha Kumbha Mela is generally organised
- 2018? every-
(a) Kazuo Ishiguro (b) Bob Dylan (a) 10th year (b) 12th year
th
(c) Patrick Modiano (d) None of the above (c) 14 year (d) 16th year
Ans. (d) : The Nobel Prize in Literature 2018 was Ans. (b) : Kumbh Mela is famous for being the biggest
awarded to Olga Tokarczuk for a narrative imagination and one of the most religious gathering in India.
that with encyclopedic passion represents the crossing • It is organised every 12 years in 4 Indian cities.
of boundaries as a form of life. Prayagraj, Haridwar, Nashik and Ujjain.
79. Which film has been chosen as India's officials • Ardh Kumbh Mela is organised in every 6 years.
entry to Oscar Awards 2019.
• The Kumbh Mela is organised on the banks of
(a) Sui Dhaga
Triveni Sangam in Prayagraj, Ganga in Haridwar,
(b) Pataakha the Godavari in Nashik and Kshipra in Ujjain.
(c) Village Rockstars
• It is recognized by the UNESCO as Intangible
(d) Batti Gul Meter Chalu
Cultural Heritage of Humanity.
Ans. (c) : Village Rockstars is a 2017 Indian Assamese
language coming-of-age drama. It was selected as India's 84. Match List-I with List-II and select the correct
official entry to the 91st Academy Awards (2019). answer using the codes given below:
80. Consider the statements: List-I List-II
Assertion (A) : Sugar industry is more A. Demetrius 1. Parthian
developed in South India. B. Rudradaman 2. Kushana
Reason (R) : Per hectare production and
quantity of juice of sugarcane is higher in the C. Gondophernes 3. Indo-Greek
southern states. D. Vim 4. Scythian
Codes: Codes:
(a) Both (A) and (R) are true and (R) is the A B C D
correct explanation of (A).
(a) 1 3 2 4
(b) Both (A) and (R) are true but (R) is not the
correct explanation of (A) (b) 4 3 1 2
(c) (A) is true but (R) is false (c) 3 4 1 2
(d) (A) is false but (R) is true (d) 1 2 3 4
UP RO/ARO (Mains) Exam-2017 91 YCT
CLICK HERE FOR FREE MATERIAL

Ans. (c) : List-I List-II


List-I List-II A. Harappa 1. N.G. Majumdar (1936-37)
A. Demetrius - Indo-Greek B. Hastinapur 2. John Marshall (1913-34)
B. Rudradaman - Scythian C. Takshashila 3. Daya Ram Sahani (1923-
C. Gondophernes - Parthian 24 & 1924-25)
D. Vim - Kushana D. Kaushambi 4. B.B. Lal (1950-52)
The above are the correct match. Codes:
A B C D
85. Stainless Steel is an alloy in which following is
(a) 4 2 1 3
added along with iron.
(b) 1 3 4 2
(a) Zinc (b) Chromium (c) 3 4 2 1
(c) Tin (d) Copper (d) 4 1 3 2
Ans. (b) : Chromium is added into Iron. Chromium is a Ans. (c) : The correct pairs are-
highly reactive element and accounts for the 'passive'
List-I List-II
nature of all stainless steel. The resistance to the
A. Harappa
chemical effects of corrosion and the typical "rusting" - Daya Ram Sahani
(oxidation) that occurs with unprotected carbon steel is (1923-24 & 1924-25)
the direct result of the presence of chromium. The B. Hastinapur - B.B. Lal (1950-52)
higher the chromium level the greater the protection. C. Takshashila - John Marshall (1913-34)
86. Which of the following pair is not correctly D. Kaushambi - N.G. Majumdar (1936-37)
matched 89. Which of the following pair is not correctly
(a) Indu Prakash - M.G. Ranade matched?
(b) Native Opinion - B.N. Mandlik (a) Lahore Proposal-Fazl-ul-Haq
(c) Bombay Chronicle - Gopal Krishna Gokhale (b) Pirpur Report-Raj Mohammad Mehdi
(d) Indian Mirror - Devendranath Tagore (c) People's Plan-Vinoba Bhave
Ans. (c) : Bombay chronicle was started in 1910 by Sir, (d) Gandhian Plan-S.N. Agarwala
Ferozshah Mehta a prominent lawyer. It was an Ans. (c) : The People's plan was authored by M.N. Roy
important Nationalist newspaper of its time and an and drafted by the Post- War Re-construction
important chronicler of the political upheavals of a Committee of the Indian Federation of Labour. The
volatile preindependent India. object of the plan is to provide for the satisfaction of the
immediate basic needs of the Indian people within a
87. Match List-I with List-II and select the correct period of Ten Years.
answer using the codes given below the lists: 90. United Nation's highest environmental honour
List-I List-II - 'Champion of the Earth' award 2018 was
(Political Parties) (Year) conferred on-
A. Indian National Congress 1. 1951 (a) Narendra Modi (b) Medha Patkar
(c) Uma Bharti (d) Sonam Wangchuk
B. Bhartiya Jan Sangh 2. 1972
Ans. (a) : The Prime Minister, Narendra Modi was
C. Communist Party of India 3. 1885 conferred the champions of the Earth Award 2018 for
D. A.I.A.D.M.K. 4. 1925 Policy Leadership by UN Secretary General, Mr.
Antonio Guterres at a ceremony at Pravasi Bharatiya
Codes:
Kendra.
A B C D
91. Who among the following was called as a
(a) 3 1 4 2 'Zindapir'?
(b) 2 3 4 1 (a) Bahadur Shah I (b) Shah Alam II
(c) 4 3 2 1 (c) Aurangzeb (d) Adil Shah
(d) 2 1 4 3 Ans. (c) : Aurangzeb was the Mughal emperor who was
Ans. (a) : The correct pairs are: called 'Zinda Pir' because he lived a simple life and had
List-I List-II high thinking. He was the 6th ruler of the Mughal Empire
and is referred as Alamgir (Conqueror of the world).
A. Indian National Congress - 1885
92. Which of the following Union Territories of
B. Bhartiya Jan Sangh - 1951 India has the lowest population density as per
C. Communist Party of India - 1925 the census of 2011?
D. A.I.A.D.M.K. - 1972 (a) Andaman and Nicobar Islands
(b) Lakshadweep
88. Match List-I with List-II and select the correct (c) Daman and Diu
answer using the codes given below the lists: (d) Dadra Nagar Nagar-Haveli
UP RO/ARO (Mains) Exam-2017 92 YCT
CLICK HERE FOR FREE MATERIAL

Ans. (a) : Population Density is a measurement of (c) Welfare of only wild elephants is taken care off
population per unit area or exceptionally unit volume. (d) It was launched as a centrally sponsored scheme
Population density of respective union territories. Ans. (c) : Project Elephant was launched in 1992 by the
• Andaman & Nicobar Island : 46 Government of India. It is a centrally Sponsored
• Lakshadweep : 2149 Scheme with following objectives-
• Daman & Diu : 2191 • To Protect elephants, their habitat and corridors.
• Dadra & Nagar Haveli : 700 • To address issues of man - animal conflict.
93. Which of the following city has not been • Welfare of captive Elephants.
identified as part of National Heritage City • To promote not to harm elephants for their tusks.
Development and Augmentation Yojana? 97. Who amongst the following has been appointed
(a) Agra (b) Ajmer chief economist of International Monetary
(c) Amravati (d) Amritsar Fund?
Ans. (a) : The National Heritage City Development and (a) Gita Gopinath (b) Maurice Obstfeld
Augmentation Yojana (HRIDAY), is a central sector (c) Christine Lagarde (d) Radha Paul
scheme launched on 21st January, 2015 and was
Ans. (a) : India born Gita Gopinath has become the first
implemented in 12 cities of Ajmer, Amaravati,
Amritsar, Badami, Dwarka, Gaya, Kancheepuram and woman and second Indian to feature on the wall of
Velankanni, Mathura and Varanasi, Puri and Warangal. former chief economists of the IMF. Gopinath was
The mission has ended on 31st March, 2019 and no new appointed as IMF chief economist in October 2018 and
projects/cities were taken up after 31 March 2019. was later promoted as the IMF's first deputy managing
director last year. The present Chief Economist of IMF
94. The nomenclature of rural local governments is Pierre-Olivier Gourinchas.
as 'Panchayati Raj' was the result of the
suggestion of which Indian leader? 98. In which of the following place is Indian
(a) Jawaharlal Nehru (b) Dr. Rajendra Prasad Vegetable Research Institute situated?
(c) Sardar Patel (d) M.K. Gandhi (a) Agra (b) Kanpur
(c) Lucknow (d) Varanasi
Ans. (d) : Mahatma Gandhi advocated Panchayatiraj as
Ans. (d) : Indian Institute of Vegetable Research is
the foundation of India's political system. It would have
been a dencentralised form of government, where each located in Varanasi District of Uttar Pradesh.
Village would be responsible for its own affairs. IIVR, Varanasi is a field unit of the Indian Council of
95. Match List-I with List-II and select the correct Agricultural Research under the aegis of the
answer using the codes given below the lists: Department of Agricultural Research and Education,
Ministry of Agriculture and Farmers Welfare,
List-I List-II
Government of India.
(Committees) (Year)
99. Singrauli Small Hydro Power Project is located
A. P.K. Thungan 1. 1957
in-
B. Ashok Mehta 2. 1986 (a) Madhya Pradesh (b) Bihar
C. B.R. Mehta 3. 1977 (c) Chhattisgarh (d) Uttar Pradesh
D. L.M. Singhvi 4. 1988 Ans. (d) : Singrauli Small Hydro Power project is
located in Sonebhadra, Uttar Pradesh.
Codes:
A B C D 100. Which of the following do not have a major
(a) 1 3 4 2 reserve of Lignite.
(b) 2 1 3 4 (a) Gujarat (b) West Bengal
(c) 4 1 2 3 (c) Jharkhand (d) Puducherry
(d) 4 3 1 2 Ans. (c) : The Lignite reserves in the country are
Ans. (d) : The correct pairs are- estimated around 45.76 Billion Tonnes. The major
deposits are located in the State of Tamil Nadu followed
List-I List-II
by Rajasthan, Gujarat, Kerala, West-Bengal, Jammu
(Committees) (Year)
and Kashmir and Union Territory of Puducherry.
A. P.K. Thungan - 1988
101. Lombok strait is located between which two
B. Ashok Mehta - 1977 islands?
C. B.R. Mehta - 1957 (a) Bali and Lombok
D. L.M. Singhvi - 1986 (b) Lombok and Sumbawa
96. Which of the following statement regarding (c) Sumbawa and Java
Project Elephant launched by Government of (d) Java and Bali
India is NOT correct? Ans. (a) : The Lombok Strait, is a strait connecting the
(a) It was launched in 1991-92 Java Sea to the Indian Ocean and is located between the
(b) It addresses issues of man-animal conflict Islands of Bali and Lombok in Indonesia.
UP RO/ARO (Mains) Exam-2017 93 YCT
CLICK HERE FOR FREE MATERIAL

102. Cheruthoni Dam is situated on which river? Select the correct answer using the codes given
(a) Periyar (b) Pampar below:
(c) Bhavani (d) Kabini Codes:
Ans. (a) : Cheruthoni is a town on the banks of (a) Both (A) and (R) are true and (R) is the
Cheruthoni River, a major tributary of the Periyar River. correct explanation of (A).
The area is known by Cheruthoni Dam. Which is a part (b) Both (A) and (R) are true but (R) is not the
of Idukki reservoir, which provide hydroelectric power correct explanation of (A).
to the region. (c) (A) is true but (R) is false.
103. Which of the following is a warm current? (d) (A) is false but (R) is true.
(a) Falkland Current (b) Brazil Current Ans. (a) : The Communal Award was announced by the
(c) Labrador Current (d) Canary Current British Prime Minister, Ramsay MacDonald, on August
Ans. (b) : Warm Ocean Currents originate near the 16, 1932. It is based on the finding of the Indian
equator and move towards the poles or higher latitudes franchise committee (also called Lothian Committee)
while cold currents originate near the poles or higher which established separate electorates and reserved
latitudes and move towards the tropics or lower latitudes. seats for minorities including the depressed classes
Here, Brazil is the warm current in the given options. which were granted seventy eight reserved seats. Thus,
104. Which of the following imparts deep blue color this award accorded Separate electorates for Muslim,
to glass. Europeans, Sikhs, Indian Christians, Anglo Indians,
(a) Cobalt oxide (b) Cupric oxide depressed classes and even to the Marathas for Some
(c) Ferric oxide (d) Nickel oxide Seats in Bombay.
Ans. (a) : Cobalt is the most powerful blue colorant 108. The catchment area of which of the following
used in glassmaking producing deep blues when used in river is maximum?
potash containing mixes, but it can also gives shades of (a) Mahananda (b) Sone
pink when used in a boro-silicate mix and green when (c) Ramganga (d) Gandak
used with iodides. Ans. (b) : The catchment area of the Sone river has the
105. The Indian rupee was made fully convertible maximum catchment i.e. 64789.32 Sq Km. Whereas,
on current account in: Ramganga (30811.48 Sq. Km) Gandak (56573.83 Sq.
(a) 1994 (b) 1995 Km.), Mahananda (23700 Sq. Km.)
(c) 1996 (d) 1997 109. Ramon Magsaysay Award 2018 was conferred
Ans. (a) : Today, the rupee is fully convertible on the on which of the following?
current account, allowing it to be converted into other (a) Sonam Wangchuk (b) Aung San Suu Kyi
currencies and vice versa for transactions such as export (c) Malala Yousafzai (d) None of these
and import of goods and services. But it is only partially Ans. (a) : Sonam Wanghchuk & Bharat Vaswani are
convertible on the capital account. The rupee has been two Indians among Six individuals who were declared
convertible on the current account since 1994, meaning winners of the award, Ramon Magsaysay 2018.
it can be changed freely into foreign currency for
purposes like trade related expenses. Presently, India 110. Given below are two statements, one is labelled
has current account convertibility. as Assertion (A) and the other as Reason (R).
106. The highest coffee producing country in the Assertion (A) In Satavahana period, Sanskrit
World is: along with Prakrit and other folk languages
were developed.
(a) Bolivia (b) Brazil
Reason (R). Satavahana Kings promoted the
(c) China (d) India
use of Sanskrit and other folk languages for
Ans. (b) : The top 5 coffee producing countries are, literary writings.
1. Brazil Choose the correct answer using the codes
2. Vietnam given.
3. Colombia (a) Both (A) and (R) are true and (R) is correct
4. Indonesia explanation of (A)
5. Ethiopia (b) Both (A) and (R) are true but (R) is not the
The production of coffee has played a pivotal role in the correct explanation of (A)
development of Brazil and continues to be a driving (c) (A) is true but (R) is false
force in the country's economy. (d) (A) is false but (R) is true
107. Consider the statements: Ans. (c) : Most of the Satvahana inscriptions and Coin
Assertion (A) : The British Government legends are in a Middle Indo-Aryan language. This
announced the Communal Award in August language has been termed 'Prakrit' by some modern
1932. Scholars, but this terminology can be considered correct
Reason (R) : It allowed to each minority a only if the term "Prakrit" is defined broadly to include
number of seats in the legislature to be elected every Middle Indo-Aryan language that is not exactly
on the basis of a separate electorate. 'Sanskrit'. According to Gunadya, besides Sanskrit and
UP RO/ARO (Mains) Exam-2017 94 YCT
CLICK HERE FOR FREE MATERIAL

Prakrit there is another language called 'Desi' which Ans. (b) : The correct match are as
may mean the native language. The Satvahana also used List-I List-II
Sanskrit in political inscription. So, it is clear from the
(States) (Year of coming to
above that Sankrit, Prakrit and folk languages were
existence)
developed but use of Sanskrit & folk languages were
not promoted for literary writing. A. Nagaland - 1962
111. Where was Darashikoh buried? B. Jharkhand - 2000
(a) Delhi (b) Agra C. Telangana - 2014
(c) Aurangabad (d) Lahore D. Sikkim - 1975
Ans. (a) : Dara Sikoh also known as Darashikoh was 116. By which Act, the British Parliament had
the eldest Son and heir-apparent of the Mughal emperor
abolished the monopoly of East India
Shah Jahan. Dara was designated with the title Pad
Company's trade in India except Tea and
Shahzada-i-Buzurg Martaba. In the war of succession,
Dara was defeated by his younger brother Prince China trade?
Muhiuddin. He was executed in 1659 on Aurangzeb's (a) Charter Act of 1813 (b) Charter Act of 1833
order in a bitter struggle for the imperial throne. After (c) Charter Act of 1853 (d) Charter Act of 1873
the death, remains of Dara Sikoh were buried in an Ans. (a) : The Charter Act of 1813 ended the
unidentified grave in Humayun's Tomb in Delhi. commercial trade monopoly of the East India Company
112. Aanchal Thakur won India's first ever except for trade in tea and trade with China.
International medal in which of these sports? 117. Which country has the lowest density of
(a) Long Jump (b) Wrestling population?
(c) Skiing (d) Chess (a) Mongolia (b) Saudi Arabia
Ans. (c) : Aanchal Thakur won India's first ever (c) Iraq (d) Afghanistan
International medal in the sports of skiing. Ans. (a) : For 2015, the population density of Mongolia
113. What percentage does Uttar Pradesh contribute was 1.914 people per Square Kilometer. In 2021, the
to the Mentha oil production of India? population increase moved the population density of
(a) 60% (b) 85% Mongolia up to 2.045 people per square kilometer. This
(c) 75% (d) 90% population density is the lowest in the world, making
Ans. (d) : Uttar Pradesh share around 90% of the total Mongolia the least densely populated independent
Mentha production in India. Rising demand in the country in the world.
export market and remunerative prices have boosted 118. Loudness of sound is measured in terms of
Mentha farming in the state. following:
114. Who among the following were the 'King (a) Frequency (b) Amplitude
maker's during Mughal period? (c) Velocity (d) Wavelength
(a) Hussain Ali and Abdullah Ans. (b) : Amplitude determines the loudness of a
(b) Nasir Khan and Jakariya Khan wave. Greater the amplitude, greater is the loudness.
(c) Shah Hussain and Abdul Hussain The loudness of a sound is directly proportional to the
(d) Muhammad Zaman and Muhammad Junaidi square of the amplitude of Vibration.
Ans. (a) : Sayyid brothers is known as king makers in 119. Government of India in collaboration with
Indian history. Nawab Sayyid Hussain Ali and his Norwegian Government has established a
brother Abdullah Khan Barha were kingmakers of the centre for Biodiversity Policy and Law at-
later Mughal Period. He is best known for ordering his (a) Dehradun (b) Chennai
death of the Emperor Farrukh Siyar. (c) Shillong (d) New Delhi
115. Match List-I with List-II and select the correct Ans. (b) : Government of India in Collaboration with
answer using the codes given below the lists:
the Norwegian Government has established a "Centre
List-I List-II for Biodiversity Policy and Law (CEBPOL)" in the
(States) (Year of coming to existence) National Biodiversity Authority (NBA), Chennai to
A. Nagaland 1. 2000 develop professional expertise in biodiversity policies &
B. Jharkhand 2. 1962 laws and develop capacity building.
120. Which of the following is not correctly
C. Telangana 3. 1975
matched?
D. Sikkim 4. 2014 (a) Gold - Kolar (b) Coal - Jharia
Codes: (c) Iron - ore - Guntur (d) Diamonds - Panna
A B C D Ans. (*) : All the given option are correct, as gold is
(a) 4 1 2 3 found in Kolar mines in Karnataka. Coal is found in
(b) 2 1 4 3 Jharia in Jharkhand. Iron ore is found in small quantity
(c) 3 2 1 4 in Guntur district of Andhra Pradesh and Diamond is
(d) 1 3 4 2 found naturally in Panna district of Madhya Pradesh.
UP RO/ARO (Mains) Exam-2017 95 YCT
CLICK HERE FOR FREE MATERIAL

Gòej ØeosMe meceer#ee DeefOekeâejer/meneÙekeâ meceer#ee DeefOekeâejer (cegKÙe) hejer#ee, 2017


meeceevÙe efnvoer
nue ØeMve-he$e hejer#ee efleefLe : 17 HeâjJejer, 2019
(efJeMues<eCe meefnle JÙeeKÙee)
1. efvecveefueefKele ceW mes Megæ Jele&veer Jeeuee Meyo nw- JÙeeKÙee- efoÙes ieÙes efJekeâuheeW ceW ‘Deepe jsCeg osJeer keâe JÙeeKÙeeve
(a) Gheue#e (b) kegâMeeMeve nesiee~’ meJee&efOekeâ Megæ JeekeäÙe nw~ ‘ÙeneB ceer"s heeveer keâe Skeâ ketBâDee
(c) ØeMebmee (d) efoefhekeâe nw~’ ceW ‘kegBâDee’ nesvee ÛeeefnS; ‘Skeâ ™heÙes ceW efkeâleves hewmes nesles nQ~’
Gòej-(c) ceW ‘®heÙes’ nesvee ÛeeefnS Deewj ‘meeOet keâes efYe#ee oes’ ceW ‘meeOeg’ nesvee
JÙeeKÙee- efoÙes ieÙes MeyoeW ceW Megæ Jele&veer Jeeuee Meyo ‘ØeMebmee’ nw~ ÛeeefnS~
DevÙe MeyoeW keâe Megæ ™he efvecveefueefKele nw– 5. ‘megOeeceÙeer, JeelmeuÙeceÙeer, let ØesceceÙeer nw’ JeekeäÙe ceW
DeMegæ Megæ efJeMes<Ùe nw-
Gheue#e – Gheue#Ùe (a) megOeeceÙeer (b) JeelmeuÙeceÙeer
efoefhekeâe – oerefhekeâe (c) ØesceceÙeer (d) GheÙeg&òeâ ceW mes keâesF& veneR
kegâMeeMeve – kegâMeemeve Gòej-(d)
2. ‘Jes DeÛÚer ueÌ[efkeâÙeeB nQ’ JeekeäÙe ceW efJekeâejer efJeMes<eCe nw- JÙeeKÙee- efoÙes ieÙes efJekeâuheeW ceW mes keâesF& Yeer efJeMes<Ùe veneR nw, Dele:
(a) DeÛÚer (b) ueÌ[efkeâÙeeB efJekeâuhe (d) mener nw~ ØeMveiele JeekeäÙeebMe ceW ‘megOeeceÙeer’,
(c) Jes (d) nQ ‘JeelmeuÙeceÙeer’ leLee ‘ØesceceÙeer’ leerveeW ner efJeMes<eCe nQ leLee ‘let’
Gòej-(a) efJeMes<Ùe nw~
6. efvecveefueefKele ceW mes Skeâ JeekeäÙe Megæ nw-
JÙeeKÙee- efoÙes ieÙes JeekeäÙe ceW ‘DeÛÚer’ efJekeâejer efJeMes<eCe nw~
(a) ÛejKee keâelevee ÛeeefnS~
efJekeâejer efJeMes<eCe Jes Meyo nesles nQ, efpeveceW heefjJele&ve mecYeJe nesles nQ~
Fve efJeMes<eCeeW ceW efuebie, JeÛeve FlÙeeefo kesâ DeeOeej hej heefjJele&ve nesles (b) Gmeves cegòeâ nmle mes Oeve uegšeÙee~
nQ~ pewmes– Jes DeÛÚs ueÌ[kesâ nQ– ÙeneB ‘DeÛÚe’ Meyo hegefuebie kesâ (c) nceejs ÙeneB le™Ce veJeÙegJekeâeW keâer efMe#ee keâe DeÛÚe ØeyevOe
Devegmeej ‘DeÛÚer’ mes efYevve ™he ceW nw~ nw~
3. Deveskeâ MeyoeW kesâ efueS Skeâ Meyo kesâ efvecveefueefKele (d) cegPemes Ùen keâece mecYeJe veneR~
efJekeâuheeW ceW mes keâewve ieuele nw- Gòej-(d)
(a) pebieue keâer Deeie - oeJeeveue JÙeeKÙee- efoÙes ieÙes efJekeâuheeW ceW ‘cegPemes Ùen keâece mecYeJe veneR’
(b) Deeies keâer meesÛeves Jeeuee - De«eieeceer Megæ JeekeäÙe nw~ DevÙe JeekeäÙeeW kesâ Megæ ™he efvecveefueefKele nQ–
(c) mecegõ keâer Deeie - yeÌ[Jeeveue (a) ÛejKee keâelevee ÛeeefnS~
(d) hesš keâer Deeie - pe"jeefive Megæ– ÛejKee Ûeueevee ÛeeefnS~
Gòej-(b) (b) Gmeves cegòeânmle mes Oeve uegšeÙee~

JÙeeKÙee- efoÙes ieÙes efJekeâuheeW ceW ‘Deeies keâer meesÛeves Jeeuee– Megæ– Gmeves cegòeâ nmle Oeve uegšeÙee~
De«eieeceer’ ieuele nw keäÙeeWefkeâ ‘Deeies keâer meesÛeves Jeeuee’ kesâ efueS (c) nceejs ÙeneB le™Ce veJeÙegJekeâeW keâer efMe#ee keâe DeÛÚe ØeyevOe nw~
mešerkeâ Skeâ Meyo ‘De«emeesÛeer’ nw~ Megæ– nceejs ÙeneB veJeÙegJekeâeW keâer efMe#ee keâe DeÛÚe ØeyevOe nw~
4. efvecveefueefKele ceW Megæ JeekeäÙe nw- 7. ‘iees$e’ Meyo keâe leodYeJe ™he nesiee-
(a) ÙeneB ceer"s heeveer keâe Skeâ ketBâDee nw~ (a) ieesle (b) JebMe
(b) Skeâ ™heÙes ceW efkeâleves hewmes nesles nQ~ (c) Keeveoeve (d) ieeso
(c) Deepe jsCeg osJeer keâe JÙeeKÙeeve nesiee~ Gòej-(a)
(d) meeOet keâes efYe#ee oes~ JÙeeKÙee- efoÙes ieÙes efJekeâuheeW ceW ‘iees$e’ Meyo keâe leodYeJe ™he
Gòej-(c) ‘ieesle’ nesiee~
UP RO/ARO (Main) Hindi 2017 96 YCT
CLICK HERE FOR FREE MATERIAL

8. GÛÚdJeeme Meyo keâe leodYeJe ™he nesiee- 14. GòejeÙeCe keâe efJeueesce nw-
(a) MJeeme (b) ØeMJeeme (a) heueeÙeve (b) efJemLeeheve
(c) Gmeeme (d) efve:MJeeme (c) F&Meeve (d) oef#eCeeÙeve
Gòej-(c) Gòej-(d)
JÙeeKÙee- efoÙes ieÙes efJekeâuheeW ceW ‘GÛÚdJeeme’ keâe leodYeJe ™he JÙeeKÙee- efoÙes ieÙes efJekeâuheeW ceW ‘GòejeÙeCe’ keâe efJeueesceeLe&keâ Meyo
‘Gmeeme’ nesiee~ ‘oef#eCeeÙeve’ nw~
9. ‘efveef<eæ’ kesâ efueS mener efJeueesce nw- 15. efvecveefueefKele ceW mes efJeMes<Ùe SJeb efJeMes<eCe keâer Âef<š mes
(a) efmeæ (b) Megæ Skeâ Ùegice ieuele nw-
(c) ieefn&le (d) efJeefnle (a) ›eâesOe-›egâæ (b) Deemeefòeâ-Deemeòeâ
Gòej-(d) (c) Deekeâ<e&Ce-Deeke=â<š (d) F&<Ùee&-Ghes#ee
JÙeeKÙee- efoÙes ieÙes efJekeâuheeW ceW ‘efveef<eæ’ keâe mener efJehejerleeLe&keâ Gòej-(d)
Meyo ‘efJeefnle’ nw~ JÙeeKÙee- efoÙes ieÙes efJekeâuheeW ceW efJeMes<Ùe Deewj efJeMes<eCe keâer Âef<š
10. ‘pees Yesoe Ùee leesÌ[e ve pee mekesâ’ JeekeäÙeebMe kesâ efueS Skeâ mes ieuele Ùegice ‘F&<Ùee&–Ghes#ee’ nw, keäÙeeWefkeâ oesveeW ner meb%ee ™he nQ,
Meyo nw- efpevekeâe efJeMes<Ùe–efJeMes<eCe ™he ‘F&<Ùee&–F&<Ùee&ueg’ leLee ‘Ghes#ee–
(a) DeYesÅe (b) ogYexÅe Ghesef#ele’ nw~
(c) mebJesÅe (d) vewJesÅe 16. ‘mleyOe’ kesâ efueS mener efJeueesce nw-
Gòej-(a) (a) DevemleyOe (b) ØeejyOe
JÙeeKÙee- efoÙes ieÙes efJekeâuheeW ceW JeekeäÙeebMe ‘pees Yesoe Ùee lees[Ì e ve (c) DemleyOe (d) ueyOe
pee mekesâ’ kesâ efueS Skeâ Meyo ‘DeYesÅe’ GheÙegòeâ nw~ Gòej-(c)
11. efvecveefueefKele ceW mes keâewve-mee Meyo efJeMes<eCe nw- JÙeeKÙee- efoÙes ieÙes efJekeâuheeW ceW ‘mleyOe’ kesâ efueS meJee&efOekeâ GheÙegòeâ
(a) Gheepe&ve (b) GheefveJesMe efJeueesceeLe&keâ Meyo ‘DemleyOe’ nw~
(c) Ûe#eg (d) Iejsuet
17. ‘Jeefle&keâe’ Meyo keâe leodYeJe ™he nesiee-
Gòej-(d)
(a) ceele=keâe (b) ke=âeflekeâe
JÙeeKÙee- efoÙes ieÙes efJekeâuheeW ceW ‘Iejsuet’ Meyo efJeMes<eCe keâe (c) ueeflekeâe (d) yeòeer
meJee&efOekeâ GefÛele efJekeâuhe nw~ Gòej-(d)
12. ‘Gueenvee’ Meyo keâe lelmece ™he nesiee-
JÙeeKÙee- efoÙes ieÙes efJekeâuheeW ceW ‘Jeefle&keâe’ keâe leodYeJe ™he ‘yeòeer’
(a) Deesjnve (b) GheeuebYe
nw~
(c) efvevoe (d) PeieÌ[e keâjvee
18. ‘keâeve kesâ veerÛes ueškeâlee ngDee keâesceue Yeeie’ kesâ efueS Skeâ
Gòej-(b)
Meyo nw-
JÙeeKÙee- efoÙes ieÙes efJekeâuheeW ceW ‘Gueenvee’ Meyo keâe lelmece ™he
(a) keâCe&Hetâue (b) keâCee&YejCe
‘GheeuebYe’ nesiee~
(c) keâCe&heeueer (d) keâCe&
13. efvecveefueefKele ceW mes keâewve-mee Meyo Megæ nw-
Gòej-(c)
(a) Ye<ce (b) HeâeuiegCe
JÙeeKÙee- efoÙes ieÙes efJekeâuheeW ceW JeekeäÙeebMe ‘keâeve kesâ veerÛes ueškeâlee
(c) Yeer<ce (d) ce=vceÙe
ngDee keâesceue Yeeie’ kesâ efueS meJee&efOekeâ GheÙegòeâ Meyo ‘keâCe&heeueer’ nw~
Gòej-(c)
19. DeeJele&keâ keâe mener efJeueesce nw-
JÙeeKÙee- efoÙes ieÙes efJekeâuheeW ceW ‘Yeer<ce’ Meyo meJee&efOekeâ Megæ nw~
(a) ØeJele&keâ (b) DeveeJele&keâ
DevÙe MeyoeW kesâ Megæ ™he efvecveefueefKele nQ–
(c) meJele&keâ (d) meceLe&keâ
DeMegæ Megæ
Ye<ce – Yemce Gòej-(b)
HeâeuiegCe – Heâeuiegve JÙeeKÙee- efoÙes ieÙes efJekeâuheeW ceW ‘DeeJele&keâ’ keâe mešerkeâ efJeueesceeLe&keâ
ce=vceÙe – ce=CceÙe Meyo ‘DeveeJele&keâ’ nw~
UP RO/ARO (Main) Hindi 2017 97 YCT
CLICK HERE FOR FREE MATERIAL

20. efvecveefueefKele ceW mes keâewve-mee JeekeäÙe Megæ nw- 26. ‘ceesnve megvoj yeeuekeâ nw’ JeekeäÙe ceW efJeMes<Ùe nw-
(a) jece Deewj MÙeece mecyeOeer ÛeÛee& (a) ceesnve (b) megvoj
(b) jece Deewj MÙeece mes mecyeæ ÛeÛee& (c) yeeuekeâ (d) GheÙeg&òeâ ceW mes keâesF& veneR
(c) jece Deewj MÙeece efJe<eÙekeâ ÛeÛee& Gòej-(c)
(d) jece Deewj MÙeece mes mecyeefvOele ÛeÛee& JÙeeKÙee- efoÙes ieÙes efJekeâuheeW ceW ‘ceesnve megvoj yeeuekeâ nw’ ceW
Gòej-(c) efJeMes<Ùe ‘yeeuekeâ’ nw keäÙeeWefkeâ ‘megvoj’ Meyo Gmekeâer efJeMes<elee yeleueelee nw~
JÙeeKÙee- efoÙes ieÙes efJekeâuheeW ceW meJee&efOekeâ Megæ JeekeäÙe ‘jece Deewj 27. ‘pees heeve keâjves ÙeesiÙe veneR nw’ kesâ efueS Skeâ Meyo nw-
MÙeece efJe<eÙekeâ ÛeÛee&’ nw~ (a) DeheLÙe (b) DeYe#Ùe
21. ‘pees yengle yeesuelee nw’ JeekeäÙe kesâ efueS Skeâ Meyo nw- (c) DeeiecÙe (d) DehesÙe
(a) Jeòeâe (b) ØeJeòeâe Gòej-(d)
(c) JeeÛeeue (d) ØeKej JÙeeKÙee- efoÙes ieÙes efJekeâuheeW ceW JeekeäÙeebMe ‘pees heeve keâjves ÙeesiÙe
Gòej-(c) veneR nw’ kesâ efueS GheÙegòeâ Skeâ Meyo ‘DehesÙe’ nw~
JÙeeKÙee- efoÙes ieÙes efJekeâuheeW ceW JeekeäÙeebMe ‘pees yengle yeesuelee nw’ kesâ 28. efvecveefueefKele ceW mes efJeueesce keâer Âef<š mes mener Ùegice nw-
efueS Skeâ Meyo ‘JeeÛeeue’ nw~ (a) DeefOeef<"le-Øeefleef<"le (b) DeemLee-efJeMJeeme
(c) DemetÙee-DevemetÙee (d) ØecegKe-efJecegKe
22. efvecveefueefKele ceW mes lelmece-leodYeJe keâe mener Ùegice nw-
Gòej-(c)
(a) ueesnkeâej-ueesnej (b) ueÛÚ-ueeKe
(c) DejIešdš-jnš (d) efceMš-ceer"e JÙeeKÙee- efoÙes ieÙes efJekeâuheeW ceW efJeueesce keâer Âef<š mes meJee&efOekeâ
Gòej-(c) GheÙegòeâ Ùegice ‘DemetÙee-DevemetÙee’ nw~ DevÙe Meyo Ùegice Demebiele nQ,
efpevekeâe efJeueesce efvecveefueefKele nw-
JÙeeKÙee- efoÙes ieÙes efJekeâuheeW ceW lelmece-leodYeJe keâe mener Ùegice
Meyo efJeueesce
‘DejIešdš-jnš’ nw~ DevÙe efJekeâuheeW kesâ Megæ Ùegice ™he nQ–
DeefOeef‰le DeveefOe
ueewnkeâej-ueesnej, ue#e-ueeKe, efce<š-ceer"e~
Øeefleef‰le DeØeefleef‰le
23. ‘efmebIeeÌ[e’ keâe lelmece ™he nw-
DeemLee DeveemLee
(a) ëe=bieeškeâ (b) megIešdš efJeÕeeme DeefJeÕeeme
(c) efMebIeeÌ[e (d) ëe=bie ØecegKe meeceevÙe, ieewCe
Gòej-(a) efJecegKe GvcegKe, meccegKe
JÙeeKÙee- efoÙes ieÙes efJekeâuheeW ceW mes keâesF& Yeer efJekeâuhe Megæ veneR nw, 29. efvecveefueefKele ceW efJeueesce keâer Âef<š mes mener Ùegice nw-
keäÙeeWefkeâ ‘efmebIeeÌ[e’ keâe lelmece ™he ‘ëe=bieeškeâ’ nw~ (a) efJeMes<e-DeeMes<e (b) efJeheefòe-Deeheefòe
24. ‘efpemekeâe efveJeejCe DelÙevle keâ<š mes efkeâÙee pee mekesâ’ kesâ (c) mJelev$elee-mJeeOeervelee (d) Je›eâ-$e+peg
efueS Skeâ Meyo nw- Gòej-(d)
(a) DeefveJeej (b) efveefve&Jeej JÙeeKÙee- efoÙes ieÙes efJekeâuheeW ceW efJeueesce keâer Âef<š mes mener Ùegice
(c) ogefve&Jeej (d) kegâefveJeej ‘Je›eâ-$e+peg’ nw~ DevÙe Meyo Ùegice ØeMve mes Demebiele nQ Mes<e MeyoeW kesâ
Gòej-(c) efJeueesce efvecveJeled nQ-
JÙeeKÙee- efoÙes ieÙes efJekeâuheeW ceW JeekeäÙeebMe ‘efpemekeâe efveJeejCe Meyo efJeueesce
DelÙevle keâ<š mes efkeâÙee pee mekesâ’ kesâ efueS meJee&efOekeâ GheÙegòeâ Skeâ efJeMes<e meeceevÙe, meeOeejCe
Meyo ‘ogefve&Jeej’ nw~ efJeheefòe mecheefòe
25. efvecveefueefKele ceW mes keâewve-mee Meyo Jele&veer keâer Âef<š mes mJelev$elee hejlev$elee
ieuele nw- 30. efvecve MeyoeW ceW efJeMes<Ùe nw-
(a) heeBÛeJeeB (b) ieeBOeer (a) MJesle (b) peerCe&
(c) yeeBme (d) DeebKe (c) iejerye (d) efkeâleeye
Gòej-(d) Gòej-(d)
JÙeeKÙee- efoÙes ieÙes efJekeâuheeW ceW Jele&veer keâer Âef<š mes ‘DeebKe’ Meyo JÙeeKÙee- efoÙes ieÙes efJekeâuheeW ceW ‘efkeâleeye’ efJeMes<Ùe Meyo nw, peyeefkeâ
ieuele nw~ Fmekeâe Megæ ™he ‘DeeBKe’ nw~ DevÙe Meyo efJeMes<eCe nQ~
UP RO/ARO (Main) Hindi 2017 98 YCT
CLICK HERE FOR FREE MATERIAL

UPPSC RO-ARO (Pre) Exam-2017


GENERAL STUDIES
Solved Paper
1. In a code language SISTER and CAR are 5. Find out the missing number in the following
written as 212345 and 765 respectively, then sequence:
655423 stands for: 6, 24, 60, 120, 210,_______
(a) ARREST (b) ASRERT (a) 324 (b) 336
(c) ASSERT (d) None of the above (c) 428 (d) 440
Ans. (a) : Ans. (b) :
S I S T E R C A R
↓ ↓ ↓ ↓ ↓ ↓ ↓ ↓ ↓
2 1 2 3 4 5 7 6 5
655423 stands for ARREST.
2. How many squares are present in the following 6. If "some saints were martyrs" is true, what
figure (provided that AB = BC = CD = DE = may be inferred about the truth or falsehood of
EF = FG and all internal angles are 90º)? the proposition "some martyrs were not non-
saints?
(a) undetermined (b) true
(c) false (d) None of the above
Ans. (b) :
(a) 9 (b) 11
(c) 13 (d) 14
Ans. (c) : There are 13 squares in the given figure.
3. Find out the missing number in the following 7. What should come in the place of (?) in the
table: following sequence?
9 4 20 ADG, GJM, MPS, (?)
8 5 12 (a) SVW (b) SVX
(c) SVY (d) SWY
7 6 ?
Ans. (c) :
(a) 2 (b) 4
(c) 6 (d) 8
Ans. (b) : Just as, Column wise,
9+7 4+6
=8 =5
2 2
20 + x 8. What is the number X in the third figure given
Similarly, = 12 below:
2
x=4
4. Ram travels 1 km due east, then 5 km due
south, then 2 km due east and finally 9 km due
north. How far is he from the starting point? (a) 82 (b) 92
(a) 10 km (b) 8 km (c) 7 km (d) 5 km (c) 102 (d) 108
Ans. (d) : Ans. (d) :
Just as, 52 + 32 – 42 = 25 + 9 – 16 = 18
72 + 52 – 32 = 49 + 25 – 9 = 65
Similarly, 92 + 62 – 32 = 81 + 36 – 9 = 108
9. A school bell takes 14 seconds to ring 8 times.
how much time will it take to ring 4 times?
(a) 6 seconds (b) 7 seconds
AB = 42 + 32 = 25 = 5km (c) 8 seconds (d) None of the above
UP RO/ARO (Pre) Exam 2017 99 YCT
CLICK HERE FOR FREE MATERIAL

Ans. (a) : Time interval between two consecutive rings Ans. (c) : World Environment Day held annually on 5th
14 June since 1973, the programme is led by United
= = 2 seconds Nations Environment Programme (UNEP) and the
8 −1
theme for 2018 was beating plastic pollution. The theme
time taken to ring 4 times = 3 interval of World Environment Day 2022 is "Only One Earth"
∵ time taken = 2 × 3 = 6 seconds and Sweden is the host country for it.
10. A is taller than B, but not as tall as C.D is taller 16. Consider the following statements about
than E but not as tall as B. E is taller than F 'Hurun Global Rich List 2018':
but not as tall as D. Who among them is the 1. Indian has the third highest number of
tallest? billionaires in the world.
(a) B (b) C 2. There are 118 billionaires in India.
(c) D (d) F 3. China has the highest number of
billionaires.
Ans. (b) : Decreasing order of height:
Of the above, the correct statement/s is/are:
C>A>B>D>E>F
(a) Only 1 (b) Only 1 and 2
Hence c is the tallest. (c) Only 2 and 3 (d) Only 1 and 3
11. What is the next letter in the following Ans. (d) : The Hurun Global Rich List 2018 ranked
sequence? India third highest number of billionaire in the world.
B, D, G, K, P,______. According to human Rich list 2022 India ranked third
(a) T (b) U with 221 Billionaires.
(c) V (d) W 17. The first summit of 'International Solar
Ans. (c) : Alliance' was held in which of the following
+2 +3 +4 +5 +6
cities?
B  → D  → G  → K  → P  →V (a) New Delhi (b) New York
12. Name the first Indian woman fighter pilot who (c) Paris (d) None of the above
created history by flying solo fighter plane Ans. (a) : The first summit of the International Solar
'MIG-21 Bison'? Alliance (ISA) was held in New Delhi in 2018. It was
(a) Bhawana Kanth (b) Subhangi Swaroop held on 11 March, 2018.
(c) Mohana Singh (d) Avani Chaturvedi 18. The 'Sultan Azlan Shah Cup 2018' was won by
(a) India (b) Holland
Ans. () :
(c) Australia (d) Germany
13. Name the Indian gymnast who created history
Ans. (c) : The Sultan Azlan Shah Cup is an annual
in February 2018 by winning the first
invitational international men's field hockey
individual medal at Gymnastics World Cup?
tournament. Australia won 27th edition of the Sultan
(a) Aruna Budda Reddy (b) Dipa Karmakar Azlan Shah Cup 2018 for record 10th time. The Sultan
(c) Pranati Nayak (d) Rakesh Patra Azlan Shah Cup 2022 hockey tournament will be held
Ans. () : in Malaysia from November 16 to 25.
14. Given that A is brother of B; C is father of A; 19. Which country will host the 24th Winter
D is the brother of E and E is the daughter of Olympics, 2022?
B. Then the uncle of D is: (a) Italy (b) China
(a) A (b) B (c) Japan (d) Germany
(c) C (d) None of the above Ans. (b) :The 24th winter Olympics, 2022 took place in
Ans. (a) : February 2022 and it was hosted by China.
20. 21st FIFA World Cup, 2018 will be held in
which of the following countries?
(a) Brazil (b) Argentina
(c) Spain (d) Russia
Ans. (d) : The 2018 FIFA World Cup was an
Hence uncle of D is A. international football tournament contested by men's
national teams that took place between 14 June and 15
15. What is the theme for 'World Environment
July 2018 in Russia.
Day 2018'?
(a) Connecting People to Nature 21. Consider the following statements about 'Word
Happiness Report 2018' released by United
(b) One World, Our Environment Nations in March 2018:
(c) Beat Plastic Pollution 1. India dropped 11 spots from its ranking in
(d) Save Water for Future the previous year 2017.
UP RO/ARO (Pre) Exam 2017 100 YCT
CLICK HERE FOR FREE MATERIAL

2. Finland with 1st rank is the happiest Ans. (a) :


country. (Sector) (Budget allocation : in
3. This report includes only 155 countries. lakh crores)
Of the above, the correct statements/s is are:
Defence `2.82
(a) Only 1 (b) Only 1 and 2
(c) Only 1 and 3 (d) Only 2 and 3 Agriculture and allied activity `0.63
Ans. (b) : India was ranked at 133 in World Happiness Subsidy for food `1.69
Report 2018 which ranked 156 countries. Finland with Rural development `1.38
1st rank is the happiest country. 26. Consider the following statements related to
World Happiness Report 2022 tennis player Rozer Federer:
• Finland has been named the world's happiest 1. As of March 20, 2018 he is ranked World
country for the fifth year. No. 1 player in men's singles.
• India saw a marginal improvement in its ranking, 2. He is first-ever player to win twenty grand
jumping three spots to 136, from 139 a year ago. slam men's singles title.
22. Which among the following countries has 3. He has won Wimbledon men's single title
announced to impose 'Carbon Tax' from 2019 seven times till March 2018.
to cut its greenhouse gas emission? Of the above, the correct statement/s is/are:
(a) Thailand (b) China (a) Only 1 and 2 (b) Only 2
(c) Singapore (d) Malaysia (c) Only 2 and 3 (d) Only 1 and 3
Ans. (c) : The Singapore government imposed a carbon Ans. (a) : When the question was asked, Rozer Federer
tax from 2019 to cut its greenhouse gas emissions and was ranked World No. 1 player in men's singles. He
make companies more competitive as global agreements holded the record for most Grand Slam men's singles
on climate change effect. championships with 20 titles and has been in 30 finals.
23. The First Union Territory of India to run 100 However currently Novak Djokovic holds the record of
percent on solar energy is most Grand Slam Men's singles title with 20 wins. The
(a) Chandigarh (b) Diu first and second statement is correct but third statement
(c) Andman-Nicobar (d) Puducherry is incorrect because Federer has won Wimbledon men's
Ans. (b) : The First Union Territory of India to run 100 single title 8th times till March 2018. Therefore, option
percent on solar energy is Diu. Now Diu is a town of UT (a) is correct answer.
Dadra and Nagar Haveli and Daman and Diu [Earlier 27. Which of the following pairs is not correctly
Daman and Diu was a Separate Union Territory] matched?
24. In March 2018, which state of India has Inscription Rulers
unveiled its state flag? (a) Nasik - Gautami Putra
(a) West Bengal (b) Maharashtra (b) Hathigumpha - Kharvela
(c) Kerala (d) Karnataka (c) Bhitari - Pulkeshin II
Ans. (d) : In March 2018, the government of former (d) Girnar - Rudraman I
Chief Minister Siddaramaiah had unveiled the proposed Ans. (c) : The Bhitari pillar inscription of Skandagupta,
official state flag for Karnataka. was discovered at Bhitari, Saidpur, Ghazipur, in the
25. As per Union Budget 2018-19, match List-I state of Uttar Pradesh and dates back to the reign of
with List-II and select the correct answer from Gupta Empire ruler Skandagupta.
the codes given below the lists: 28. Match List-I with List-II and select the correct
List-I List-II answer from the codes given below:
(Sector) (Budget allocation : in List-I List-II
lakh crores)
(Acharya) (Doctrine)
A. Defence 1. `1.38
A. Lakulisha 1. Ajivaka
B. Agriculture and 2. `1.69
B. Nagarjuna 2. Sunyavada
allied activity
C. Subsidy for food 3. `0.63 C. Bhadrabahu 3. Pasupat
D. Rural development 4. `2.82 D. Gosala 4. Jain
Codes: Codes:
A B C D A B C D
(a) 4 3 2 1 (a) 2 3 4 1
(b) 1 2 3 4 (b) 3 2 4 1
(c) 4 2 1 3 (c) 1 2 3 4
(d) 3 1 4 2 (d) 3 1 4 2
UP RO/ARO (Pre) Exam 2017 101 YCT
CLICK HERE FOR FREE MATERIAL

Ans. (b) : The correct match the following of Acharya Ans. (d) : The correct match the following one :
and Doctrine are- List-I List-II
Lakulisha - Pasupat Ranthambore Hamirdeo
Nagarjuna - Sunyavada Chittor Rana Ratan Singh
Bhadrabahu - Jain Devgiri Raja Ramchandra
Gosala - Ajivaka Gujarat Karandeva
29. Which one of the following rulers established 34. Which one of the following pairs is not
Vikramasila University? correctly matched?
(a) Gopal (b) Devapal Traveller Country
(c) Mahipala I (d) Dharmapal (a) Ibn Batuta - Morocco
Ans. (d) : Vikramshila University was established by (b) Marco Polo - Italy
Dharmapala, the ruler of the Pala dynasty of Bengal (c) Abdul Razzaq - Turkey
towards the late eighth century. It was located in (d) Nuniz - Portugal
Bhagalpur district of Bihar.
Ans. (c) : Abdul Razzaq was a Persian Traveller and
30. Arrange the names of the following rulers in Islamic scholar. He visited the Vijayanagara Kingdom
correct chronological order and select the
in India at the time of Dev Rai II.
answer from the codes given below:
1. Vidyadhara 2. Dhanga 35. Consider the following events and arrange
3. Yashovarman 4. Keertiverma these in chronological order:
Codes: 1. Transfer of capital by Mohd. Tughlaq
(a) 3, 2, 1, 4 (b) 1, 3, 2, 4 2. Afganpur conspiracy
(c) 3, 1, 4, 2 (d) 2, 3, 1, 4 3. Murder of Jalal-ud-Din Khalji
Ans. (a) : These are the Chandela rulers who ruled the 4. Second Battle of Tarain
Bundelkhand region in central India. The correct order Select correct answer from the codes given
of rulers are: below:
Yashovarman 925-950 CE (a) 4, 3, 2, 1 (b) 3, 1, 2, 4
Dhangadeva 950-999 CE (c) 1, 2, 3, 4 (d) 1, 2, 4, 3
Vidyadhara 1003-1035 CE Ans. (a): The correct chronological order of events are as-
Devavarman 1050-1060 CE • In 1327, Tughlaq ordered to move his capital from
Kirtivarman 1060-1100 CE Delhi to Daulatabad (also known as Devagiri) in
31. 'Aadi Varaha' was the title of which Gurjar the Deccan region of India.
Pratihara ruler? • Jalal-ud-Din Khalji killed by his nephew and son-
(a) Vatsraja (b) Nagbhatta II in-law Alauddin in 1296.
(c) Mihira Bhoja (d) Nagbhatta I • Ghiyas-ud-Din died in 1325 as a result of the
Ans. (c) : Mihira Bhoja was a king belonging to the collapse of a pavilion hurriedly built by his son in
Gurjara-Pratihara Dynasty of India. He was a devotee of Afghanpur were known as Afghanpur conspiracy.
Lord Vishnu and adopted the title of 'Adi Varaha'. • Second Battle of Tarain was held in 1192.
32. Which district of Bihar set a record by Hence option (a) is the correct answer to this
constructing 11244 toilets in 100 hours? question.
(a) Sivan (b) Gopalganj 36. Who among the following Chisti Saints is
(c) Chhapra (d) Muzaffarpur known as 'Chirag-e-Delhi'?
Ans. (b) : Gopalganj district in Bihar has set a record by (a) Mui-ud-din (b) Farid-ud-din
constructing 11,244 toilets in 100 hours. (c) Nizam-ud-din Aulia (d) Nasiruddin
33. Match List-I with List-II and select the correct Ans. (d) : Nasiruddin Mahmud known as Chirag-e-
answer from the codes given below the lists: Delhi. He was Sufi Saint of Chisti Order, he was the last
List-I List-II important Sufi of Chisti Order belonging from Delhi.
37. Who among the following rulers circulated the
A. Ranthambore 1. Karandeva silver coin called 'Shahrukh'?
B. Chittor 2. Raja Ray Ramchandra (a) Akbar (b) Babur
C. Devgiri 3. Hamirdeo (c) Humayun (d) Shahjahan
D. Gujarat 4. Rana Ratan Singh Ans. (b) : The Mughal Emperor Babur issued standard
Timurid currency coins known as the Shahrukhi, named
Codes: after Shahrukh Mirza, Timur's eldest son. Shahrukh was
A B C D made of 72 grain silver. One side of the coin contained
(a) 4 3 2 1 the name of king along with his title and date. The coin
(b) 1 4 3 2 was in circulation till the introduction of rupee during
(c) 2 4 1 3 the reign of Akbar.
(d) 3 4 2 1 However the question deleted by the UPPSC.
UP RO/ARO (Pre) Exam 2017 102 YCT
CLICK HERE FOR FREE MATERIAL

38. What does 'Triratna' means in Buddhism? Ans. (b) : Subhash Chandra Bose was highly influenced
(a) Tripitaka by Vivekananda's teachings and considered him as his
(b) Buddha, Dhamma, Sangha spiritual Guru. His political guru was Chittaranjan Das.
(c) Sheel, Samadhi, Sangha 43. Match List-I with List-II and select the correct
(d) Satya, Ahinsa, Karuna answer from the codes given below the lists:
Ans. (b) : Triratna also called Three Jewels, in List-I List-II
Buddhism, comprises of the Buddha, The Dharma (Rebellion) (Year A.D.)
(doctrine, or teaching), and The Sangha (the monastic A. Ahom 1. 1855-56
order or community). B. Kol 2. 1828
39. 'Operation Rubicon' was the code word
C. Santhal 3. 1921
proposed to be adopted by British Government
in India with reference to which of the D. Moplah 4. 1831-32
following? Codes:
(a) Gandhi's fast unto death in Jail A B C D
(b) Activities of Jai Prakash Narayan (a) 2 4 1 3
(c) Gandhi's participation in Round Table (b) 1 3 2 4
Conference (c) 2 1 3 4
(d) None of the above (d) 3 1 4 2
Ans. (a) : 'Operation Rubicon' was the code word Ans. (a) : The correct match is-
proposed to be adopted by British Government in India Ahom - 1828
with reference Gandhi's fast unto death in Jail. Kol - 1831-32
40. Who among the following was not the member Santhal - 1855-56
of Champaran Agrarian Enquiry Committee? Moplah - 1921
(a) F.G. Slay (b) D.J. Reid 44. Who among the following rulers established
(c) Anugrah Narayan (d) Mahatma Gandhi 'Diwan-i-Amir-Kohi' department?
Ans. (c) : Anugrah Narayan was not the member of (a) Balban
Champaran Agrarian Enquiry Committee. Others (b) Allauddin Khilji
members are F.G. Sly (President), L.C. Adami, (c) Muhammad-bin-Tughlaq
Kirtyanand Singh, D.J. Reid, G. Rainy and Mahatma (d) Frizo Shah Tughlaq
Gandhi. The Champaran Agrarian Enquiry Committee Ans. (c) : Muhammad bin Tughlaq established the
was appointed by the Government of Bihar and Orissa department of Diwan-e-Amir Kohi. This department
on 10th June 1917. The report was issued by committee was related to agriculture.
on 3rd October, 1917. 45. Which one of the following persons was not
41. Consider the following events: associated with Azad Hind Fauj?
1. Gandhi-Irwin Pact (a) Major General Shahnawaz Khan
2. Communal Award (b) Colonel Prem Kumar Sahgal
3. Second Round Table Conference (c) Colonel Shaukat Ali Malik
4. Nehru Report (d) Major Kartar Singh
Arrange above events in chronological order Ans. (d) : The Indian National Army (also known as the
and select the correct answer from the codes Azad Hind Fauj) was first established by Captain
given below: Mohan Singh in 1942. It was revived by Netaji Subhas
Codes: Chandra Bose on October 21, 1943. Major Kartar Singh
(a) 3, 2, 1, 4 (b) 3, 1, 4, 2 was an Indian revolutionary, he was related to Ghadar
Movement. He was not associated with Azad Hind Fauj.
(c) 4, 3, 2, 1 (d) 4, 1, 3, 2
46. Consider the following events:
Ans. (d) : The Correct Chronological order arrangement
of following events as- 1. August Offer
2. Poona Pact
Nehru Report - 15 August 1928
3. Third Round Table Conference
Gandhi - Irwin Pact - 5 March 1931
4. Communal Award
2nd Round Table Conference -7 September 1931
Arrange the above events in chronological
Communal Award - 16 August 1932
order and select the correct answer from the
42. Who was the Political Guru of Subhash codes given below:
Chandra Bose? Codes:
(a) G.K. Gokhale (b) C.R. Das (a) 4, 3, 2, 1 (b) 4, 2, 3, 1
(c) B.C. Pal (d) B.G. Tilak (c) 2, 1, 3, 4 (d) 3, 2, 1, 4
UP RO/ARO (Pre) Exam 2017 103 YCT
CLICK HERE FOR FREE MATERIAL

Ans. (b) : The Chronological order is- 4,2,3,1 51. Which one of the following enzymes converts
1. The August Offer was an offer made by Viceroy proteins into amino acids?
Linlithgow in 1940. (a) Pepsin (b) Lactase
2. In 24 September 1932, the Poona Pact was signed (c) Urease (d) Zymase
between Mahatma Gandhi and DR B.R Ambedkar in Ans. (a) : Pepsin is a stomach enzyme that serves to
the Yerwada Central Jail, Pune. digest proteins found in ingested food. It converts
3. The third and last Round Table conference was held proteins into amino acids.
in November 1932.
4. The Communal Award was created by the British 52. Which one of the following is a female sex-
Prime Minister Ramsay MacDonald on 16 August hormone?
1932. (a) Androsterone (b) Testosterone
47. Which one of the following pairs is not (c) Estrogen (d) Thyroxin
correctly matched? Ans. (c) : The two main female sex hormones are
(a) Local self government - Lord Lytton estrogen and progesterone. Although testosterone is
(b) Subsidiary Alliance - Lord Wellesley considered a male hormone, females also produce and
(c) Doctrine of Lapse - Lord Dalhousie need a small amount of this, too.
(d) Permanent Settlement - Lord Cornwallis 53. Which one of the following polymers is not
Ans. (a) : Lord Ripon was known as the father of local biodegradable?
self-government. He had introduced local self- (a) Cellulose (b) Starch
governance in the year 1882. Hence option (a) is not (c) Protein (d) PVC
correctly paired. Ans. (d) : PVC(Poly Vinyl Chloride) are non-
48. Match List-I with List-II and select the correct biodegradable polymers. Non-biodegradable polymers
answer from the codes given below: are unaffected by natural processes and cannot be
List-I List-II disassembled or degraded after thousands of years.
A. Samachar Darpan 1. Raja Ram Mohan Roy 54. Which one of the following pairs is not
B. Mirat-ul-Akhbar 2. B.G. Tilak correctly matched?
(a) Aluminum - Bauxite (b) Copper - Cinnabar
C. Kesari 3. Mahatma Gandhi
(c) Zinc - Calamine (d) Iron - Hematite
D. Young India 4. J.C. Marshman
Ans. (b) : In the given options Cinnabar is the ore of
Codes: Mercury (Hg), hence pair given in option (b) is not
A B C D correct, rest given options are correct.
(a) 2 1 4 3
55. Sea weeds are the main and important source
(b) 4 3 2 1
of the following element:
(c) 2 3 4 1
(a) Iodine (b) Chlorine
(d) 4 1 2 3
(c) Bromine (d) None of the above
Ans. (d) : Correct match the following are-
Ans. (a) : Sea weeds are the main and important source
List-I List-II
of iodine. Sea weed is well dried and burnt in deep pits
Samachar Darpan J.C. Marshman
carefully so that iodine do not get destroyed. The obtained
Mirat-ul-Akhbar Raja Ram Mohan Roy
ash is called kalp, which contains 0.4 to 1.3% iodine.
Kesari B.G. Tilak
Young India Mahatma Gandhi 56. Which one of the following pairs is not
correctly matched?
49. Kuka Movement was founded in
(a) Vitamin D - Rickets
(a) Bengal (b) Bihar
(c) Punjab (d) Maharashtra (b) Vitamin C - Bleeding gums
(c) Vitamin A - Rheumatism
Ans. (c) : The Kuka Movement marked the first major
reaction of the people in the Punjab to the new political (d) Vitamin B1 - Beriberi
order initiated by the British after 1849. It was a phase Ans. (c) : Vitamin D deficiency has been implicated in
in Namdhari Movement. the immune diseases like rheumatic diseases, asthma,
50. The total amount of blood present in a normal psoriasis and multiple sclerosis. So, option (c) is not
human (70 kg) adult is about correctly matched.
(a) 6000 ml (b) 5000 ml 57. Which one of the following pairs is not
(c) 4000 ml (d) 3000 ml correctly matched?
Ans. (b) : The amount of blood in the human body is (a) Chloromycetin - antityphoid
generally equivalent to 7 percent of body weight. So (b) Crystal violet - antiseptic
total amount of blood present in a normal human (70 (c) Quinine - antimalarial
kg) adult is about 5000ml. (d) Aspirin - anaesthetic
UP RO/ARO (Pre) Exam 2017 104 YCT
CLICK HERE FOR FREE MATERIAL

Ans. (d) : The correct match is - Li-Fi is not operated by optical fiber network. Li-Fi is
(a) Chloromycetin - antityphoid a wireless optical networking technology that used
(b) Crystal violet - anstiseptic Light-Emitting Diodes (LED) for data transmission.
(c) Quinine - antimalarial Hence statement given in option (d ) is not correct.
(d) Aspirin - analgesic 62. The device which works on the principle of
electromagnetic induction is
58. The silk produced by spiders is called
(a) Ammeter (b) Voltmeter
(a) Tussar silk (b) Munga silk
(c) Dynamo (d) Electric motor
(c) Gossamer silk (d) Ahinsa silk
Ans. (c) : A dynamo (electric generator) is a machine
Ans. (c) : The silk produced by spiders is called
which generates electricity by using the principal of
Gossamer silk.
electromagnetic induction.
59. The ratio of kinetic energies of two bodies of
63. Which one of the following is not required for
same mass is 4 : 9, the ratio of their velocities
photosynthesis process?
will be
(a) Sunlight (b) Oxygen
(a) 4 : 9 (b) 2 : 3
(c) Water (d) Carbon dioxide
(c) 16 : 81 (d) √2 : √3 Ans. (b) : Photosynthesis is the process of synthesis of
Ans. (b) : As we know that -- food by plants using carbon dioxide and water in
Kinetic Energy = 1/2 MV2 presence of sunlight and chlorophyll. Oxygen (O2) is
(K.E) =1/2 MV2 released during the process as a product. Hence, oxygen
( K.E )1 = 1/ 2MV12 − − − −(i) is not essential for the process of photosynthesis.
64. With reference to bacteriophages, which
( K.E )2 = 1/ 2 MV22 ------ (ii) statement/s is/are correct?
( K.E )1 1/ 2mV12 1. Bacteriophages are virus that infect
= bacteria.
( K.E )2 1/ 2mV22 2. Bacteriophages are used in genetic
4 V12 engineering.
= Select the correct answer using the codes given
9 V22
below:
2
4  V1  Codes:
= 
9  V2  (a) 1 only (b) 2 only
(c) Both (1) and (2) (d) Neither (1) nor (2)
4 V1 Ans. (c) : Bacteriophages (BPs) are viruses that can
=
9 V2 infect and kill bacteria without any negative effect on
V1 2 human or animal cells. The term ''Bacteriophages" is
= coined by Felix d'Herelle in 1917. Bacteriophages are
V2 3 widely used in Biotechnology specially in genetic
V1 : V2 = 2 : 3 engineering.
65. Which one of the following scales of
60. Which one of the following is not the unit of temperature does not have a negative value?
heat? (a) Celsius (b) Fahrenheit
(a) Centigrade (b) Calorie (c) Kelvin (d) Reaumar
(c) Erg (d) Joule
Ans. (c) : The Kelvin temperature scale does not have
Ans. (a) : In the given option Centigrade is the unit of negative values. Reason : The Kelvin scale is related to
temperature not heat while the Calorie, Erg and Joule Celsius scale as follows : K = 0º C-273.15. It is physically
are the unit of heat. impossible to reach temperature of zero Kelvin (–273ºC).
61. Which one of the following statements is not Therefore Kelvin Scale is news negative.
true about Li-Fi? 66. Match List-I with List-II and select the correct
(a) The full form of Li-Fi is 'Light Fidelity' answer from the codes given below the list:
(b) The successful test of Li-Fi in India was done List-I List-II
by Ministry of Information and Broadcasting (Crop/Plantation) (Largest Producer
on 29th January, 2018. State)
(c) Li-Fi can send 10 GB/sec. data upto 1 km A. Jute 1. Kerala
circumference
(d) It is operated by optical fibre network B. Tea 2. Uttar Pradesh
Ans. (d) : The term Li-Fi( Light fidelity ) was coined by C. Sugarcane 3. Assam
Harald Haas in the year 2011. D. Rubber 4. West Bengal
UP RO/ARO (Pre) Exam 2017 105 YCT
CLICK HERE FOR FREE MATERIAL

Codes: Ans. (c) : According to data sourced from the


A B C D Directorate General of Commercial Intelligence and
(a) 4 3 2 1 Statistics, Iraq was the highest crude oil supplier to
(b) 3 1 2 4 India during 2017-18.
(c) 2 4 3 1 72. Which one of the following pair is not correctly
(d) 1 2 3 4 matched ?
Ans. (a) : Correct match the following are- (a) Cuscutta - Stem parasite
List-I List-II (b) Chandan - root parasite
(Crop Plantation) (Largest Producer State) (c) Orchids - epiphyte
(i) Jute West Bengal (d) Nepenthes - hydrophytes
(ii) Tea Assam Ans. (d) : Nepenthes is a genus of carnivorous plants,
(iii) Sugarcane Uttar Pradesh also known as tropical pitcher plants, or monkey cups,
(iv) Rubber Kerala in the monotypic family Nepenthes. Hence, option (d) is
not correctly matched. Rest all the given option is
67. Among the Indian States, Uttar Pradesh is the
correctly matched.
largest producer of which of the following
crops? 73. The share of Tertiary Sector in Indian
(a) Wheat, Potato, Groundnut economy during 2016-17 was
(b) Wheat, Potato, Sugarcane (a) `51.8 lakh crore (b) `50.6 lakh crore
(c) Potato, Sugarcane, Cotton (c) `49.0 lakh crore (d) `52.8 lakh crore
(d) Potato, Sugarcane, Paddy Ans. (d) : The share of Tertiary sector in India economy
Ans. (b) : Uttar Pradesh is the largest producer of during the 2016-17 was around 52.8 lakh crore.
Wheat, Potato and Sugarcane crops. 74. The water holding capacity of different types of
68. Which of the following pairs is not correctly soils decreases in the order:
matched? (a) Clay > Silt > Sand (b) Clay > Sand > Silt
(Crop) (Variety) (c) Sand > Silt > Clay (d) Silt > Sand > Clay
(a) Mustard Varuna Ans. (a) : The water holding capacity of different types
(b) Field Pea Sapna of soils decreases in the order: Clay > Silt > Sand.
(c) Linseed Surya 75. Which of the following pairs is not correctly
(d) Groundnut Kaushal matched?
Ans. (c) : The few popular varieties of Linseed's are K- (Institute) (Location)
2, T-397, Jawahar-17, Mayurbhans ,Hira etc while the (a) Indian Vegetable - Varanasi
variety Surya is related to Brinjal. Hence option (c) is Research Institute
not correctly matched Rest all the given option correctly (b) Central Institute of - Srinagar
match. Subtropical Horticulture
69. Which one of the following substances is a (c) Indian Institute of - Lucknow
herbicide? Sugarcane Research
(a) Chlorpyrifos (b) Carbendazim (d) Indian Institute of - Kanpur
(c) Quinolphos (d) Butachlor Pulse Research
Ans. (d) : Butachlor is a herbicide. Herbicides, also Ans. (b) : Central Institute for Subtropical Horticulture
commonly known as weedicide or " weed killers" are (CISH), is located at Rehmankhera, in Lucknow. It was
substances used to control unwanted plants. Establishment in 1972.
70. Among Indian States, which state has the 76. Which among the following statements is not
highest yield (per hectare) of rice? true?
(a) West Bengal (b) Uttar Pradesh (a) Rust is a disease found in wheat
(c) Haryana (d) Punjab (b) Early blight is a disease found in Potato
nd (c) False smut is a disease found in barley
Ans. (d) : India is the 2 largest rice producing country
and the largest exporter of rice worldwide. The top five (d) Khaira is a disease found in paddy
rice producing states in India are- West Bengal, Uttar Ans. (c) : False smut disease is found in rice. It is
Pradesh, Punjab, Tamil Nadu and Andhra Pradesh. In caused by the fungus Ustilaginoidea virens.
terms of highest yield per hectare, Punjab is at the top of 77. Consider the statements:
list in the states.
Assertion (A) : Among various types of soils,
71. Among the following countries, which was the clay retains highest amount of water.
highest crude oil supplier to India during 2017-18? Reason (R) : Clay has pore spaces of large size.
(a) Saudi Arabia (b) Iran Select the correct answer using the codes given
(c) Iraq (d) Kuwait below:
UP RO/ARO (Pre) Exam 2017 106 YCT
CLICK HERE FOR FREE MATERIAL

Codes: 82. Match List-I with List-II and select the correct
(a) Both (A) and (R) are true and (R) is the answer using the codes given below the lists:
correct explanation of (A) List-I List-II
(b) Both (A) and (R) are true, but (R) is not the (Dam) (River)
correct explanation of (A) A. Dulhasti 1. Chambal
(c) (A) is true, but (R) is false B. Gandhisagar 2. Chenab
(d) (A) is false, but (R) is true
C. Ukai 3. Tapi
Ans. (c) : Clay soil retains the highest amount of water
because the porosity in Clay soils are less and has pore D. Tawa 4. Tawa
spaces minimum in size while Sandy soil will have Codes:
larger porosity than Clay soil. So statement (1) is A B C D
correct while the statement (2) is incorrect. (a) 1 2 3 4
78. First 'Deendayal Handicraft Sankul' trade (b) 3 2 1 4
facilitation centre is located at (c) 2 3 1 4
(a) Agra (b) Varanasi (d) 2 1 3 4
(c) Kanpur (d) Gorakhpur Ans. (d) : Correct match the following of list-I and II
Ans. (b) : First 'Deendayal Handicraft Sankul' trade are-
facilitation centre is located at Varanasi. The Trade List-I List-II
Facilitation Centre and Crafts Museum, was inaugurated (Dam) (River)
by the Hon'ble Prime Minister of India in 2017 at Dulhasti Chenab
Varanasi. It is dedicated to the nation as "Deendayal Gandhisagar Chambal
Hastkala Sankul (Trade Centre and Museum)". Ukai Tapi
79. Which among the following is not a speculator Tawa Tawa
in the stock exchange? 83. Match List-I with List-II and select the correct
(a) Broker (b) Bull answer using the codes given below the lists:
(c) Bear (d) Stag List-I List-II
Ans. (a) : Speculators are sophisticated investors or (Grass-Lands) (Country)
traders who purchase assets for short periods of time A. Pampas 1. Australia
and employ strategies in order to profit from changes in B. Pustaz 2. South Africa
its price. The 4 main types of speculators are a bull,
C. Veld 3. Argentina
bear, stag and lame duck. Broker is not a speculator in
the stock exchange. D. Downs 4. Hungary
80. Which of the following commodities has highest Codes:
export from India in 2017? A B C D
(a) Agriculture and allied products (a) 3 4 2 1
(b) Engineering goods (b) 3 2 1 4
(c) Textiles (c) 4 1 2 3
(d) Chemicals (d) 4 2 1 3
Ans. (b) :The commodities that has highest export from Ans. (a) : Correct match of Grass land and related
India in 2017 was Engineering goods. country are-
Grassland Country
81. Which of the following agencies is not engaged Pampas Argentina
in exporting agricultural goods from India?
Pustaz Hungary
(a) NAFED
Veld South Africa
(b) State Trading Corporation Downs Australia
(c) IFFCO
84. 'TRAI' is a regulatory body associated with
(d) MMTC which of the following sectors?
Ans. (c) : IFFCO is not engaged in exporting (a) Transport (b) Tourism
agriculture good from India. Indian Farmers Fertilizer (c) Technical Education (d) Telecom
Cooperative(IFFCO) Limited founded in 1967 to enable Ans. (d) : The Telecom Regulatory Authority of India
Indian farmers to prosper through timely supply of (TRAI) was, established with effect from 20th February
reliable, high quality agricultural inputs and services in 1997 by an Act or Parliament, called the Telecom
an environmentally sustainable manner and to undertake Regulatory Authority of India Act, 1997, to regulate
other activities to improve their welfare. It is one of telecom services, including fixation/revision of tariffs
India's biggest cooperative societies which is wholly for telecom services which were earlier vested in the
owned by Indian Cooperatives. Central Government.
UP RO/ARO (Pre) Exam 2017 107 YCT
CLICK HERE FOR FREE MATERIAL

85. Which of the following represents the correct Ans. (c) : Correct match the following of National Park
north to south sequence of following four and state is-
islands of Japan? National Park State
(a) Hokkado, Honshu, Kyushu, Shikoku Intanki Nagaland
(b) Hokkado, Shikoku, Honshu, Kyushu Betla Jharkhand
(c) Hokkado, Honshu, Shikoku, Kyushu Sirohi Manipur
(d) Hokkado, Kyushu, Honshu, Shikoku Guindy Tamil Nadu
Ans. (c) : The correct North to South Sequence of 90. The place on earth's surface which records the
islands of Japan is- seismic waves first is called
(a) Focus (b) Epicentre
Hokkado, Honshu, Shikoku, Kyushu
(c) Seismosite (d) None of the above
86. Chiquicamata (Chili) is worldwide famous for
Ans. (b) : The place on earth's surface which records
which of the following minerals? the seismic waves first is called epicenter.It is the point
(a) Copper (b) Iron on the earth's surface vertically above the focus point in
(c) Silver (d) Manganese the crust where a the earthquake begins.
Ans. (a) : Chuquicamata is the largest open pit copper 91. The number of coastal states in India is
mine in terms of excavated volume in the world. It is (a) 06 (b) 07
located in the northem of Chile. (c) 08 (d) 09
87. Which of the following rock systems is the Ans. (d) : India has nine coastal states. These are--
major source of metallic minerals in India? Gujarat, Maharashtra, Goa, Karnataka, Kerala, Tamil
(a) Tertiary system Nadu, Andhra Pradesh, Orissa and West Bengal.
(b) Vindhyan system 92. As per Census 2011 of India, which among the
(c) Gondwana system following states recorded highest density of
population?
(d) Dharwar system
(a) Uttar Pradesh (b) Bihar
Ans. (d) : Dharwar rock system is the major source of (c) Punjab (d) Tamil Nadu
metallic minerals in India. Dharwar rocks are the oldest
Ans. (b) As per Census 2011 of India, Bihar is
metamorphic rocks of India which formed between 2-5
recorded as most densely populated state with
billion to 1.8 billion years ago. They are formed due to the population density 1106 (persons/sq km) followed by
metamorphosis of the sediments from Archean rocks. West Bengal-1028 and Kerala-860.
88. According to "India, State of Forest Report 93. According to Malthus, which one of the
2017" what percentage of total geographical following is the most effective measure of
area of the country is under forest cover? population control?
(a) 21.04 (b) 21.54 (a) War (b) Misery
(c) 20.54 (d) 20.04 (c) Birth control (d) Vices
Ans. (b) : According to ISFR 2017 the total forest cover Ans. (c) : According the Malthus the most effective
was 7,08,273 sq. km, which was 21.54% of the total measures of population control is preventive measures
geographical area of the country. The total forest cover such as birth control by means of late marriage, self
in India (2022) is 7,13,789 square kilometers which is control etc. because preventive measures not only check
21.71% of the total geographical area. the population growth but can also prevent the harmful
effect of positive checks such as famine, natural
89. Match List-I with List-II and select the correct disasters etc.
answer from the codes given below the lists: 94. As per Census 2011, which among the following
List-I List-II Indian states recorded lowest female literacy?
(National Parks) (States) (a) Uttar Pradesh (b) Rajasthan
A. Intanki 1. Jharkhand (c) Kerala (d) Andhra Pradesh
B. Betla 2. Tamil Nadu Ans. (b) : As per census 2011 report, The lowest female
literacy rate recorded in Bihar (51.50) followed by
C. Sirohi 3. Nagaland Rajasthan (52.12) and Jharkhand (55.42).
D. Guindy 4. Manipur 95. Which of the following states recorded a
Codes: decline in its population in Census 2011?
A B C D (a) Nagaland (b) Kerala
(a) 2 1 3 4 (c) Sikkim (d) Manipur
(b) 4 3 2 1 Ans. (a) : Nagaland was the only state in the country to
(c) 3 1 4 2 record a negative growth in population with -0.58%
(d) 3 4 1 2 decadal population growth as per 2011 census.
UP RO/ARO (Pre) Exam 2017 108 YCT
CLICK HERE FOR FREE MATERIAL

96. As per Census 2011, which among the following 1. Gujarat 2. Andhra Pradesh
states recorded highest literacy? 3. Maharashtra 4. West Bengal
(a) Manipur (b) Punjab Codes:
(c) Assam (d) Madhya Pradesh (a) 4, 2, 3, 1 (b) 3, 4, 2, 1
Ans. (a) :In the given option Literacy rate of Manipur (c) 2, 3, 1, 4 (d) 1, 2, 4, 3
is highest (76.94%) as per 2011 census report. Ans. (*) : The correct descending order of urbanization
97. As per 2011 Census, the percentage of urban as per Census 2011 of the given states is -
population to total population in India was Maharashtra (45.22%) > Gujarat (42.60%)
(a) 28.50 (b) 31.16 West Bengal (31.87%) > Andhra Pradesh (37.7 %)
(c) 37.60 (d) 39.20 However UPPSC considered option (b) as correct
Ans. (b) : As per the Census report 2011, the answer.
percentage of urban population to total population in 102. Consider the following statements about eco-
India was 31.16%. system:
98. Match List-I with List-II and select the correct 1. The production at the autotroph level is
answer using the codes given below the lists: said to be primary productivity.
List-I List-II 2. The secondary productivity refers to the
(Hills) (States) production at the heterotrophy level.
A. Garo 1. Meghalaya Of the above, the correct statement/s is/are:
(a) Only 1 (b) Only 2
B. Miri 2. Tamil Nadu (c) Both 1 and 2 (d) Neither 1 nor 2
C. Kollai Mallai 3. Arunachal Pradesh Ans. (c) : Both the statement are correct.
D. Dalma 4. Jharkhand The production at the autotrophs level is said to be the
Codes: primary productivity and the secondary productivity
A B C D refers to the production at the heterotrophy level. Hence
(a) 1 3 2 4 both statements are correct.
(b) 1 2 3 4 103. The amount of energy during transfer from
(c) 1 3 4 2 one trophic level to another in an eco-system
(d) 2 1 3 4 (a) Increases
(b) Decreases
Ans. (a) : Correct match the following of hills and
related state are- (c) Remains constant
Hills State (d) May increase or decrease
Garo Meghalaya Ans. (b) : As per Lindemans rule, only 10% of energy
Miri Arunachal Pradesh is transferred from one trophic level to another and 90%
Kollai Mallai Tamil Nadu of the energy is lost during transfer, respiration and
digestion processes. Hence amount of energy decreses
Dalma Jharkhand
from one trophic level to another trophic level in an
99. As per Census 2011, which among the following ecosystem.
states recorded lowest percentage of urban
104. In which of the following types of forests
population?
maximum plant diversity is found?
(a) Tripura (b) Sikkim
(a) Tropical moist deciduous forests
(c) Arunachal Pradesh (d) Himachal Pradesh
(b) Sub-tropical mountain forests
Ans. (d) : According to the 2011 Census,only 10.03
(c) Temperate moist forests
percent of the population of Himachal Pradesh resides
in urban areas, while 89.97 percent lived in rural areas. (d) Tropical evergreen forests
100. As per Census 2011, which of the following Ans. (d) : The maximum plant diversity is found in
states is second in India in its literacy rate and tropical evergreen forest. In India, these types of forests
the level of urbanization? are found in southern India along the Western Ghats,
(a) Mizoram (b) Maharashtra Andaman and Nicobar Islands and north-eastern region.
(c) Tamil Nadu (d) Tripura 105. As per 2011 Census, the percentage of
Ans. (a) : As per Census 2011, Mizoram is the second population of metropolitan cities to the total
state after Kerala (93.91) in literacy rate and the level of urban population of India was
urbanization in India. (a) 31.16 (b) 36.48
(c) 42.61 (d) 49.20
101. Arrange the following States in descending
order of urbanization as per Census 2011 and Ans. (c) : As per 2011 census the percentage of
select the correct answer from the codes given population of metropolitan city to total urban population
below: of India was 42.61%.
UP RO/ARO (Pre) Exam 2017 109 YCT
CLICK HERE FOR FREE MATERIAL

106. As per 2011 Census, the literacy rate in Uttar 112. Which district of Uttar Pradesh is the largest
Pradesh from 2001 to 2011, rose up to producer of Dushari Mango?
(a) 11.45% (b) 12.45% (a) Agra (b) Lucknow
(c) 13.45% (d) 14.45% (c) Varanasi (d) Pratapgarh
Ans. (a) : As per census report 2011,during 2001-2011, Ans. (b) : Lucknow district of Uttar Pradesh is the
the literacy of the Uttar Pradesh increased by 11.45%. largest producer of Dushari Mango.The Dushari Mango
Note:-As per UPPSC Commission answer was 13.45%. is a Variety of mango which originated near Kakori in
107. In U.P., which one of the following cities is not Lucknow district of U.P.
an export development centre? 113. Which one of the following is the largest eco-
(a) Agra (b) Allahabad system of the earth?
(c) Khurja (d) Meerut (a) Hydrosphere (b) Biome
Ans. (b) : In the given option, all 3 cities Agra, Khurja (c) Lithosphere (d) Biosphere
and Meerut are an export development centres of U.P Ans. (d) : Biosphere is the largest ecosystem of the
except Allahabad (Presently known as Prayagraj) earth because it will contain all types of ecosystem and
108. Pandit Deen Dayal Upadhyay Pashu Chikitsa surrounded by the atmosphere to balance the life on the
Vigyan Vishawa Vidhyalaya Evam Go Raksha earth.
Anushanthan Sansthan is situated at 114. Which one of the following is the eastern most
(a) Lucknow (b) Meerut town of Uttar Pradesh?
(c) Ayodhya (d) Mathura (a) Ballia (b) Azamgarh
Ans. (d) : Pandit Deen Dayal Upadhyay Pashu Chikitsa (c) Barabanki (d) Basti
Vigyan Vishawa Vidhyalaya Evam Go Raksha
Ans. (a) : Ballia is the easternmost town of the State
Anushanthan Sansthan is situated at Mathura, Uttar Pradesh.
Uttar Pradesh and It borders with Bihar State.
109. 'NOIDA' is located in which district of Uttar
115. Which one of the following pairs is not
Pradesh?
correctly matched?
(a) Gautam Buddha Nagar (b) Ghaziabad
(City) (Characteristic)
(c) Meerut (d) Shahjahanpur
(a) Varanasi - Culture and Religion
Ans. (a) : NOIDA is located in Gautam Buddha Nagar
(b) Lucknow - State Capital
district of Uttar Pradesh.
(c) Jhansi - Industrial Hub
110. What is the size of the Budget of Uttar Pradesh
(d) Moradabad - Brass Industry
for 2018-19?
(a) `824384.53 crores (b) `428354.53 crores Ans. (c) : Jhansi is famous for soft toys not for
(c) `428384.52 crores (d) `824254.52 crores industrial Hub. Hence except this all the pairs are
correctly matched.
Ans. (c) : `428384.52 crores was the size of the Budget
of Uttar Pradesh for 2018-19. 116. The tenure of the office of the Lokayukta of
Uttar Pradesh is
111. Consider the following statements:
(a) 5 years (b) 6 years
Assertion (A) : 'Bhojpuri' is lingua franka of
eastern Uttar Pradesh. (c) 7 years (d) 8 years
Reason (R) : People in eastern Uttar Pradesh Ans. (d) : Lokayukta tenure was fixed to be 6 years, but
are not at home with other languages. was extended to 8 years in March 2012. Justice
Select the correct answer from the codes given Vishwambhar Dayal was the 1st Lokayukta of Uttar
below: Pradesh (1977-1982).
Codes: 117. Consider the statements:
(a) Both (A) and (R) are true and (R) is the Assertion (A) : Kannauj is a 'city of perfumes'.
correct explanation of (A). Reason (R) : All residents of Kanauj city have
(b) Both (A) and (R) are true, but (R) is not the perfume industries.
correct explanation of (A) Select the correct answer from the codes given
(c) (A) is true, but (R) is false below:
(d) (A) is false, but (R) is true Codes:
Ans. (c) : Bhojpuri is lingua franka of eastern Uttar (a) Both (A) and (R) are true and (R) is the
Pradesh. Assertion is correct statements but reason is correct explanation of (A)
incorrect because the peoples of Uttar Pradesh love (b) Both (A) and (R) are true, but (R) is not the
every languages and are not uncomfortable with other correct explanation of (A)
languages. (c) (A) is true, but (R) is false
Therefore, (A) is true, but (R) is false. (d) (A) is false, but (R) is true
UP RO/ARO (Pre) Exam 2017 110 YCT
CLICK HERE FOR FREE MATERIAL

Ans. (c) : Due to the key role of perfume production in Ans. (d) : According to the Article 170 of the Indian
Kannauj, the city is known as "the perfume capital of constitution the legislative assembly consists of
India" but all the residents of Kannauj city does not representatives directly elected by the people on the
have perfume industry. Hence (A) is true while (R) is basis of universal adult franchise. Its maximum strength
wrong statement. is fixed at 500 and minimum strength at 60. On the
118. Consider the following objectives of 'Single basis of the population size of the state.
Window Clearance department' in U.P. and 122. The two Union Territories which have been
select the correct answer from the codes given given right to take part in the election of the
below: President of India by the 70th Amendment of
1. Industrial waste management Indian Constitution, are:
2. Industrial approval (a) Delhi and Chandigarh
Codes:
(b) Delhi and Puducherry
(a) Only 1 is correct
(c) Delhi and Daman & Diu
(b) Only 2 is correct
(d) Chandigarh and Puducherry
(c) Both 1 and 2 are correct
(d) Neither 1 nor 2 is correct Ans. (b) : Delhi and Puducherry are the two Union
Territories which have been given right to take part in
Ans. (b) : The main objectives of Single window
the election of the President of India by the 70th
clearance department in U.P are to enable ease of doing
business through facilitating entrepreneurs, with the Amendment of Indian Constitution.
electronic based transparent system and industrial approval. 123. Which of the following pairs is not correctly
119. In which city the famous 'Thumari' singer matched?
Girija Devi was born? Institution Year of Establishment
(a) Lucknow (b) Aligarh (a) Central Bureau of - 1963
(c) Moradabad (d) Varanasi Investigation
Ans. (d) : Girija Devi was born at Varanasi, in 1929, (b) Central Vigilance - 1964
She belonged to Banaras gharana. Commission
120. With reference to the reservation of seats for (c) Prevention of - 1985
women in Panchayati Raj Institutions, which of Corruption Act
the following statements is/are correct? (d) Enforcement - 2000
1. It is provided for in Article 243D(3) of the Directorate
Indian Constitution.
Ans. (c) : Correctly match the following institution
2. The reservation shall cease to have effect on and establishment are -
the expiration of the period specified in
Article 334. Central Bureau of - 1 April 1963
Investigation
Select the correct answer from the codes given
below: Central Vigilance - February 1964
(a) 1 only (b) 2 only Commission
(c) Both 1 and 2 (d) Neither 1 nor 2 Prevention of - 5 September1988
Ans. (a) : Article 243D provides that one-third of the Corruption Act
total number of seats and offices of the Chairpersons in Enforcement - 1 May 1956
PRIs at each level shall be reserved for women to be Directorate
allotted by rotation to different constituencies in a Hence pairs given in option (c) and (d) are not
Panchayat. The provision of reservation of seats for correctly matched.
women in Pachayati Raj Institutions under Article
The UPPSC has deleted the question.
243(D) is an exception of clause (5) of the Article
243(D). 124. The Administrative Reforms Commission 1967,
Hence 2nd statement is incorrect. which for the first time recommended for the
setting up of Lokpal and Lokayukta, was
121. According to Article 170 of the Indian
headed by
Constitution, the minimum and maximum
strength of members of Legislative Assembly in (a) Morarji Desai
a state can be respectively: (b) K. Santhanam
(a) 40 and 400 (b) 50 and 450 (c) Chaudhary Charan Singh
(c) 50 and 500 (d) 60 and 500 (d) Babu Jagjivan Ram
UP RO/ARO (Pre) Exam 2017 111 YCT
CLICK HERE FOR FREE MATERIAL

Ans. (a) : In 1966, the first Administrative reforms Ans. (a) : Fiscal deficit of Indian Government as a
Commission was formed under the Chairmanship of percentage of GDP was higher in 2017-18 as compared
Morarji Desai (later on K. Hanumanthaiya). The to budget estimates. Growth in indirect tax collection
Commission recommended the creation of two special was relatively lower during 2017-18 on account of
authorities designated as 'Lokpal' and 'Lokayukta' for introduction of GST. Reason is the explanation for the
redressal of citizens' grievances. assertion. Hence, both (A) and (R) are true and (R) is
the correct explanation of (A).
125. In which of the following Article of the Indian
Constitution, the abolition of untouchability 128. Which of the following continued to be the
has been guaranteed? major component of India's external credit till
2017?
(a) Article - 14 (b) Article - 15
(a) NRI deposits
(c) Article - 16 (d) Article - 17
(b) Short term debt
Ans. (d) : Part III of the Indian Constitution provides (c) Trade Credit
Fundamental Rights, in which Article 17 has been
(d) Commercial borrowing
provided the abolition of "Untouchability".
Ans. (d) : Commercial borrowing continued to be the
126. Which one of the following statements is/are major component of Indian external credit till 2017.
correct with reference to India?
129. Which of the following statements is/are
1. The fiscal deficit target for the year 2017- correct about 73rd Amendment to the Indian
18 was raised to 3.5 percent of GDP. Constitution?
2. The fiscal deficit target for the year 2018- 1. It recognized Panchayats as institutions of
19 has been placed at 3.3 percent of GDP. self-government.
3. The fiscal deficit target for the year 2020- 2. It recognized urban local governments as
21 is projected at 3.1 percent of the GDP. institutions of self government.
Select the correct answer from the codes given Select the correct answer using the codes given
below: below:
Codes: Codes:
(a) 3 only (b) 1 and 3 only (a) Both 1 and 2 (b) Only 2
(c) 1 and 2 only (d) 2 and 3 only (c) Only 1 (d) Neither 1 nor 2
Ans. (c) : India expected to trim the fiscal deficit to Ans. (c) : The Constitution (73rd Amendment) Act was
3.3% of GDP in the fiscal year (2017-18) but it revised passed in 1992 and it came into effect on 24 April 1993.
fiscal deficit target of 3.5% of G.D.P in 2017-18. It provided constitutional status to Panchayati Raj
Former finance minister Arun Jaitly had set the fiscal Institutions.
deficit target for 2018-19 at 3.3% of G.D.P. Therefore, Hence only 1st statement is correct.
option 1 and 2 are correct but 3rd option is incorrect 130. As in December 2017, Indian government's
because fiscal deficit was targeted at 3.5% of GDP in holding in which of the following banks was
2020-21 not 3.1%. maximum?
Hence, option (c) is correct answer. (a) Central Bank of India
127. Consider the following statements: (b) United Bank of India
Assertion (A) : Fiscal deficit of Indian (c) Bank of India
Government as a percentage of GDP was (d) State Bank of India
higher in 2017-18 as compared to Budget Ans. (b) : As of December 2017, Indian government
estimates. holding in United Bank of India was maximum.
Reason (R) : Growth in indirect tax collection 131. As per Census of India 2011, which among the
was relatively lower during 2017-18 on account following districts of Uttar Pradesh has the
of introduction of GST. highest literacy rate?
Select the correct answer using the codes given (a) Allahabad (b) Varanasi
below: (c) Bareilly (d) Ghaziabad
Codes: Ans. (d) : As per census of India 2011 Ghaziabad
(a) Both (A) and (R) are true and (R) is the district has the highest literacy rate (78.07) in Uttar
correct explanation of (A) Pradesh.
(b) Both (A) and (R) are true, but (R) is not the 132. Which city of Uttar Pradesh is famous for
correct explanation of (A) 'wood carving cottage industry'?
(c) (A) is true, but (R) is false (a) Gorakhpur (b) Saharanpur
(d) (A) is false, but (R) is true (c) Meerut (d) Kanpur
UP RO/ARO (Pre) Exam 2017 112 YCT
CLICK HERE FOR FREE MATERIAL

Ans. (b) : The Saharanpur city of Uttar Pradesh is 137. Looking at a portrait of a man, Sanjay said
world-famous for its "Wood carving cottage industry". "His mother is the wife of my father's son. I
133. In Uttar Pradesh, North Central Zone Cultural have no brothers and sisters". At whose
Centre has been established at: portrait was Sanjay looking?
(a) Agra (b) Varanasi (a) His son
(c) Allahabad (d) Bareilly (b) His nephew
Ans. (c) : In Uttar Pradesh, North Central Zone Culture (c) His cousin
Centre has been established at Allahabad. (Now (d) His father
Prayagraj) Ans. (a) :
134. Consider the following statements to explain
why Uttar Pradesh is called the 'Sugar-bowl of
India:'
1. U.P. has the highest production of
sugarcane.
2. U.P. has the sugarcane engineering
education institutes. Hence sanjay was looking his son's portrait
3. U.P. has the largest number of sugar mills. 138. Which one of the following is the correct
4. U.P. has the highest number of sugarcane symbolization of the proposition "some
growers. peddlers are millionaires" by means of Venn
Which of the above explanations are correct? diagram?
(a) Only 1 and 3 (b) Only 1 and 2
(c) Only 1, 2 and 4 (d) Only 2, 3 and 4
(a) (b)
Ans. (a) : Uttar Pradesh is called the sugar bowl of
India. This is because it is the largest producer of
sugarcane in India. Uttar Pradesh accounts for almost
39% of sugarcane production in the country and having (c) (d)
largest number of sugar mills.
135. Producer Price Index measures: Ans. (c) :
(a) The average change in the prices of produced
goods and services.
(b) The marginal change in the prices of
produced goods and services.
(c) The total change in the prices of produced
goods and services.
139. A monkey climbs 16 m up a slippery pole. It
(d) None of the above rises 1 m in 1 minute and slips 1/4 m in the next
Ans. (a) : Producer Price Index (PPI) measures average minute. It will reach the top in:
change in prices received by producers of domestically (a) 30 minutes (b) 32 minutes
produced goods and services. It is calculated by (c) 36 minutes (d) None of the above
dividing the current prices received by the sellers of a
representative basket of goods by their prices in some 1 3
Ans. (d) : Monkey rises in 2 minutes = 1 – = m
base year multiplied by 100. 4 4
136. Which one of the following did not take place 3
Monkey rises in 40 minutes = × 20 = 15 m
in the Union Budget for 2017-18? 4
(a) Elimination of the classification of Monkey will reach the top in 40 + 1 = 41 minutes.
expenditure into 'Plan' and 'Non-Plan'. 140. "Tharu Tribe" is residing in which of the
(b) Increase in the number of centrally sponsored following states of India?
schemes.
(a) Bihar and Madhya Pradesh
(c) Bringing Railway finances into the
(b) Jharkhand and Bihar
mainstream budgeting.
(d) Advancing the date of Union Budget almost (c) Chhattisgarh and Himachal Pradesh
by a month. (d) Uttarakhand and Uttar Pradesh
Ans. (b) : Increase in the number of centrally sponsored Ans. (d) : The Tharu Tribe live in both India and
schemes did not take place in the Union Budget for Nepal. They live mostly in Uttarakhand, Uttar Pradesh,
2017-18. and Bihar in terai belt of India.
UP RO/ARO (Pre) Exam 2017 113 YCT
CLICK HERE FOR FREE MATERIAL

Gòej ØeosMe meceer#ee DeefOekeâejer/meneÙekeâ meceer#ee DeefOekeâejer (Øeer.) hejer#ee, 2017


meeceevÙe efnvoer
nue ØeMve-he$e hejer#ee efleefLe : 08 DeØewue, 2018
(DeOÙeeÙeJeej efJeMues<eCe meefnle JÙeeKÙee)
efJeueesce .
JÙeeKÙee– efoÙes ieÙes efJeueesce MeyoeW ceW DeMegæ Ùegice ‘GÅece-efve®Åece’
nw~ peyeefkeâ Fmekeâe Megæ efJeueesce Meyo ‘GÅece-DeeuemÙe’ nesiee Deewj
1. ‘efve<eæ’ kesâ efueS mener efJeueesce Meyo nw-
Mes<e efJeueesce Ùegice Meyo Megæ nw~
(a) efmeæ (b) Deefveef<eæ
7. efvecveefueefKele ceW efJeueesce keâer Âef<š mes Megæ Ùegice nw-
(c) efJeefnle (d) Ie=efCele
(a) Deuhe%e - yeng%e (b) mebÙegòeâ - mebefOeÙegòeâ
Gòej– (c)
(c) mecemle - DeYÙemle (d) De%e - DeveefYe%e
JÙeeKÙee–‘efve<eæ’ kesâ efueS meJe&Lee GheÙegòeâ efJehejerleeLe&keâ Meyo
Gòej– (a)
‘efJeefnle’ neslee nw peyeefkeâ ‘efmeæ’ keâe efJeueesce Meyo ‘Deefmeæ’ neslee nw~
JÙeeKÙee– Meyo efJeueesce
2. efvecveefueefKele ceW Skeâ Meyo ‘Meg<keâ’ keâe efJeueesce nw-
mecemle Skeâue
(a) G<Ce (b) Deeõ&
mebÙegòeâ DemebÙegòeâ, efJeÙegòeâ
(c) Meerle (d) efMe<š
De%e efJe%e
Gòej– (b)
DeefYe%e DeveefYe%e
JÙeeKÙee–‘Meg<keâ’ keâe GheÙegòeâ efJehejerleeLe&keâ Meyo ‘Deeõ&’ neslee nw
8. efJeueesceeLe&keâ Âef<š mes Skeâ Ùegice DeMegæ nw-
peyeefkeâ ‘efMe<š’ keâe efJeueesce ‘DeefMe<š’ neslee nw~ ‘Meerle’, ‘G<Ce’
(a) meÛeue - Ûeue (b) efvejhes#e - meehes#e
hejmhej efJehejerleeLeea Meyo nw~
(c) ceewefuekeâ - Devetefole (d) yebOeve - cees#e
3. efvecveefueefKele ceW mes Skeâ Meyo ‘mLetue’ keâe efJeueesce veneR nw-
Gòej– (a)
(a) met#ce (b) levJeer
(c) ogye&ue (d) MeeMJele
JÙeeKÙee– ‘Ûeue’ keâe GheÙegòeâ efJehejerleeLe&keâ Meyo ‘DeÛeue’ neslee nw
Gòej– (d) ve efkeâ meÛeue, Mes<e Ùegice Megæ nw~
veesš- DeeÙeesie ves Fmekeâe Gòej efJekeâuhe (a) Deewj (d) oesveeW keâes ceevee nw~
JÙeeKÙee– efoÙes ieÙes MeyoeW ceW ‘MeeMJele’ Meyo mLetue keâe efJeueesce
veneR nw peyeefkeâ Ùen #eefCekeâ keâe efJeueesce nw~ met#ce, levJeer SJeb ogye&ue 9. efvecveefueefKele ceW mes efJeueesce keâer Âef<š mes mener Meyo Ùegice nw-
mLetue kesâ efJeueesce nQ~ (a) Dee«en - efJe«en
(b) Dehekeâ<e& - Ghekeâ<e&
4. ‘he=Lekeâd’ keâe mener efJeueesce nw-
(c) peejpe - Deewjme
(a) Skeâ$e (b) mebÙegòeâ
(d) Glmepe&ve - efJemepe&ve
(c) Leefkeâle (d) megIeefšle
Gòej– (b) Gòej– (c)
JÙeeKÙee– ‘he=Lekeâd’ keâe GheÙegòeâ efJehejerleeLe&keâ Meyo ‘mebÙegòeâ’ neslee nw JÙeeKÙee–‘peejpe’ keâe efJeueesce Deewjme, ‘Dehekeâ<e&’ keâe efJeueesce Glkeâ<e&
peyeefkeâ ‘Skeâ$e’ keâe efJeueesce Meyo ‘efJekeâerCe&’ neslee nw~ peyeefkeâ ‘Dee«en’ keâe efJeueesce ogje«en ‘efJemepe&ve’ keâe efJeueesce DeJeenve neslee nw~
5. efvecveefueefKele ceW Skeâ Meyo ‘GheÙeesie’ keâe efJeueesce nw- 10. efvecveefueefKele ceW efJeueesce MeyoeW keâe mener Ùegice nw-
(a) mecegheÙeesie (b) efve®heÙeesie (a) F&efhmele - DeYeerefhmele (b) Deefcele - meefcele
(c) meogheÙeesie (d) DevegheÙeesie (c) nÙee - yesnÙee (d) DeIeer - efvejIe
Gòej– (d) Gòej– (d)
JÙeeKÙee– ‘GheÙeesie’ keâe GheÙegòeâ efJehejerleeLe&keâ Meyo ‘DevegheÙeesie’ JÙeeKÙee– efoÙes ieÙes efJeueesce MeyoeW ceW Megæ Ùegice ‘DeIeer-efvejIe’ nw~
neslee nw~ Mes<e efJekeâuhe Demebiele SJeb $egefšhetCe& nQ~ peyeefkeâ Mes<e Ùegice DeMegæ nw~ efpevekeâe Megæ ™he Fme Øekeâej nesiee–
veesš- DeeÙeesie ves Fmekeâe Gòej efJekeâuhe (b) Deewj (d) oesveeW keâes ceevee nw~ F&efhmele – Deveerefhmele
6. FveceW mes efJeueesce MeyoeW keâe Skeâ ieuele Ùegice nw- Deef c ele – heef jefcele
(a) oef#eCe - Jeece (b) GÅece - efve®Åece nÙee – yesnÙee
(c) efJeefOe - efve<esOe (d) yeye&j - meYÙe vees š – ef Jekeâuhe (c) leLee (d) oes
veeW mener nw efkeâvleg DeeÙeesie ves Fmekeâe
Gòej– (b) Gòej (d) ceevee nw~
UP RO/ARO (Pre) Hindi 2017 114 YCT
CLICK HERE FOR FREE MATERIAL

heÙee&ÙeJeeÛeer JÙeeKÙee– KeÌ[ie, Deefme, ke=âheeCe, lesie, keâjJeeue Deewj MeceMeerj


11. ‘efJeefMeKe’ efkeâme Meyo keâe heÙee&ÙeJeeÛeer nw ? Deeefo leueJeej kesâ heÙee&ÙeJeeÛeer Meyo nQ~ DevÙe Demebiele nQb~
19. efvecveefueefKele ceW mes ‘keâeceosJe’ keâe heÙee&ÙeJeeÛeer Meyo nw-
(a) yegOe (b) yeeCe
(c) le® (d) mejesJej (a) efJe[ewpee (b) efheMegve (c) ceej (d) DeMce
Gòej– (b) Gòej– (c)
JÙeeKÙee– ‘efJeefMeKe’ Meyo yeeCe keâe heÙee&ÙeJeeÛeer nw~ yeeCe kesâ DevÙe JÙeeKÙee– Devebie, kebâohe&, cevceLe, ceove, Deosn, jefleheefle Deewj ceej
heÙee&ÙeJeeÛeer Meyo leerj, Mej, MeeÙekeâ Deeefo nw~ ‘keâeceosJe’ kesâ heÙee&ÙeJeeÛeer Meyo nw~ ‘efye[ewpee’ Fvõ keâe, ‘efheMegve’
12. efvecveefueefKele ceW ‘Kej’ keâe heÙee&Ùe Meyo veneR nw -
keâewDee keâe leLee ‘DeMce’ helLej keâe heÙee&ÙeJeeÛeer Meyo nw~
20. ‘OevebpeÙe’ keâe heÙee&Ùe nw-
(a) og<š (b) ieone
(c) eflevekeâe (d) lespe (a) ieg[ekesâMe (b) OevegOe&j (c) ce=lÙebpeÙe (d) meejLeer
Gòej– (a) Gòej– (a)
JÙeeKÙee–‘ieone, eflevekeâe Deewj lespe’ leerveeW Meyo ‘Kej’ kesâ heÙee&Ùe nQ JÙeeKÙee– Depeg&ve, heeLe&, keâewvlesÙe, ieg[ekesâMe, ieeb[erJeOej Deeefo
peyeefkeâ ‘og<š’ Meyo ‘Keue’ keâe heÙee&Ùe neslee nw~ IevebpeÙe kesâ heÙee&ÙeJeeÛeer Meyo nw~
13. keâewve-mee Meyo ‘IeesÌ[e’ keâe heÙee&ÙeJeeÛeer veneR nw ? JeekeäÙe SJeb Jele&veer mecyevOeer DeMegefæÙeeB
(a) yeeefpe (b) legjbie (c) Meeot&ue (d) nÙe
21. efvecveefueefKele MeyoeW ceW Jele&veer keâer Âef<š mes keâewve-mee Meyo
Gòej– (c)
DeMegæ nw ?
JÙeeKÙee– yeeefpe, legjbie, nÙe, DeMJe, Ieesškeâ Deeefo IeesÌ[e kesâ (a) heg<heebpeefue (b) efvejehejeOe (c) Yeemkeâj (d) GppJeue
heÙee&ÙeJeeÛeer Meyo nQ peyeefkeâ Meeot&ue, Mesj keâe heÙee&ÙeJeeÛeer Meyo nw~ Gòej– (b)
14. ‘šerkeâe’ Meyo keâe heÙee&Ùe nw-
JÙeeKÙee– ØeMveiele efJekeâuhe ceW GefuueefKele ‘efvejehejeOe’ Jele&veer keâer
(a) JÙeeKÙee (b) DeeuesKe (c) šskegâDee (d) lekeâueer
Âef<š mes DeMegæ nw, Fmekeâer Megæ Jele&veer ‘efvejhejeOe’ neslee nw, peyeefkeâ
Gòej– (a) Mes<e Jele&veer keâer Âef<š mes Megæ nQ~
JÙeeKÙee– JÙeeKÙee, Yee<Ùe, Je=efòe, Yee<eeblejCe, efJeJe=efòe Deeefo šerkeâe 22. efvecveefueefKele ceW DeMegæ Jele&veer keâe Meyo nw-
kesâ heÙee&ÙeJeeÛeer Meyo nQ~ peyeefkeâ DeeuesKe, šskegâDee leLee lekeâueer leerveeW (a) Jee*dceÙe (b) Glkeâ<e&
Demebiele nQ~ (c) JewcevemÙe (d) efceLeuesMekegâceejer
15. efvecveefueefKele ceW ‘Ùecegvee’ keâe heÙee&ÙeJeeÛeer Meyo veneR nw- Gòej– (d)
(a) nbmemeglee (b) Deke&âpee (c) ke=â<Cee (d) ketâuebkeâ<ee JÙeeKÙee– ØeMveiele efJekeâuhe ceW GefuueefKele ‘efceLeuesMekegâceejer’ DeMegæ
Gòej– (d) Jele&veer Ùegkeäle Meyo nw, efpemekeâer Megæ Jele&veer efceefLeuesMekegâceejer’ nw
JÙeeKÙee– nbmemeglee, Deke&âpee leLee ke=â<Cee leerveeW Ùecegvee kesâ peyeefkeâ Mes<e Jele&veer keâer Âef<š mes Megæ nw~
heÙee&ÙeJeeÛeer Meyo nQ~ peyeefkeâ ‘ketâuebkeâ<ee’ Meyo Ùecegvee keâe heÙee&Ùe veneR 23. efvecveefueefKele ceW Skeâ Meyo DeMegæ nw-
nw yeefukeâ veoer keâe heÙee&ÙeJeeÛeer Meyo nw~ (a) owefnkeâ (b) veeskeâjer (c) ØeewÌ{ (d) heew®<e
16. efvecveefueefKele ceW mes ‘ceerve’ keâe meceeveeLeea nw- Gòej– (b)
(a) PeKe (b) efMeueercegKe (c) Deefue (d) <ešdheo JÙeeKÙee– ØeMve ceW GefuueefKele ‘veeskeâjer’ DeMegæ Jele&veer keâe Meyo nw
Gòej– (a) efpevekeâer Megæ Jele&veer ‘veewkeâjer’ neslee nw, peyeefkeâ Mes<e Jele&veer keâer Âef<š
JÙeeKÙee– ‘ceerve’ keâe meceeveeLeea Meyo ‘PeKe’ nw peyeefkeâ efMeueercegKe, mes Megæ nw~
Deefue, <ešdheo, efceefuebo, Yeücej, ÛebÛejerkeâ, ceOegkeâj, ceOeghe Deeefo 24. FveceW mes DeMegæ Jele&veer keâe Meyo nw-
YeBJeje kesâ heÙee&ÙeJeeÛeer Meyo nw~ (a) Ûejceeslkeâ<e& (b) Jee*dceÙe
17. efvecveefueefKele ceW mes ‘kesâleg’ keâe heÙee&Ùe veneR nw- (c) hegveg®lLeeve (d) efnjCÙekeâMÙeheg
(a) Peb[e (b) heleekeâe Gòej– (*)
(c) efveMeeve (d) «en JÙeeKÙee– Ûejceeslkeâ<e&, Jee*dceÙe oesveeW Megæ Jele&veer Jeeues Meyo nQ
Gòej– (d) peyeefkeâ hegveg™lLeeve Meyo DeMegæ nw efpemekeâe Megæ ™he nw– hegve®lLeeve
JÙeeKÙee– Peb[e, heleekeâe, efveMeeve, kesâleve, OJepe Deeefo ‘kesâleg’ kesâ leLee efnjCÙekeâMÙeheg keâe Megæ ™he efnjCÙekeâefMeheg nw, efkeâvleg DeeÙeesie
heÙee&ÙeJeeÛeer Meyo nw~ peyeefkeâ ‘«en’ kesâleg keâe heÙee&Ùe veneR nw~ ves Fmes cetuÙeebkeâve mes yeenj keâj efoÙee nw~
18. ‘leueJeej’ keâe heÙee&ÙeJeeÛeer Meyo nw- 25. efvecveefueefKele ceW mes Megæ Jele&veer Jeeuee Meyo nw-
(a) nueJeej (b) keâjJeeue (a) ØeefleoMe& (b) Â<še
(c) OejJeej (d) Oeejoej (c) jÛeFlee (d) Denej
Gòej– (b) Gòej– (a)
UP RO/ARO (Pre) Hindi 2017 115 YCT
CLICK HERE FOR FREE MATERIAL

JÙeeKÙee– ‘ØeefleoMe&’ Megæ Jele&veer keâe Meyo nw peyeefkeâ Denej, JÙeeKÙee– efoÙes ieÙes JeekeäÙeeW ceW Megæ JeekeäÙe ‘efvejhejeOe JÙeefòeâ keâes
jÛeFÙee Deewj Â<še DeMegæ Jele&veer kesâ Meyo nQ efpevekeâer Megæ Jele&veer oC[ keäÙeeW efceuee? peyeefkeâ Mes<e efJekeâuhe DeMegæ nQ~ efpemekeâe Megæ
›eâceMe: Deenej, jÛeefÙelee Deewj õ<še nesieer~ ™he Fme Øekeâej nesiee–
26. efvecveefueefKele ceW DeMegæ JeekeäÙe nw- DeMegæ JeekeäÙe Megæ JeekeäÙe
(a) ieewlece $e+ef<e keâer helveer keâe veece DenuÙee Lee~
1. mebmeeefjkeâ mecemÙeeDeeW ceW 1. meebmeeefjkeâ mecemÙeeDeeW ceW
meYeer HeBâmes nQ~ meYeer HeBâmes nQ~
(b) Fme keâeÙe& ceW yengle efJeuecye nes ieÙee~
2. efnvoer osJeveeiejer efueheer ceW 2. efnvoer osJeveeiejer efueefhe ceW
(c) jmeieguuee yengle mJeeefo<š nw~
efueKeer peeleer nw~ efueKeer peeleer nw~
(d) Skeâ iegueeye keâer ceeuee Kejero uesvee~ 3. DelÙeeefOekeâ og:Ke mene 3. DelÙeefOekeâ og:Ke mene
Gòej– (d) veneR peelee~ veneR peelee~
JÙeeKÙee– ØeMve ceW GefuueefKele JeekeäÙe ‘Skeâ iegueeye keâer ceeuee Kejero
uesvee’ DeMegæ nw keäÙeeWefkeâ FmeceW ‘Skeâ iegueeye keâer ceeuee’ kesâ mLeeve hej
Deveskeâ MeyoeW kesâ Skeâ Meyo
‘iegueeye keâer Skeâ ceeuee’ ØeÙegòeâ nesvee ÛeeefnS~ Mes<e JeekeäÙeebMe Megæ 31. ‘Deheves menejs hej jnves Jeeues’ kesâ efueS Skeâ Meyo nw-
nw~ (a) mJeeJeuebyeer (b) DeelceefveYe&jer (c) yeefue<" (d) DeelceefJeMJeemeer
27. efvecveefueefKele ceW mes Skeâ JeekeäÙe DeMegæ nw-
Gòej– (a)
(a) Fme yeele keâe mhe<šerkeâjCe keâjvee DeeJeMÙekeâ nw~
JÙeeKÙee– JeekeäÙeebMe ‘Deheves menejs hej jnves Jeeues’ kesâ efueS Skeâ
(b) hÙeepe leerKee neslee nw~
Meyo ‘mJeeJeuebyeer’ ØeÙegòeâ efkeâÙee peelee nw~ Mes<e efJekeâuhe Demebiele nQ~
32. ‘DeØecesÙe’ efkeâme JeekeäÙeebMe kesâ efueS Skeâ Meyo nw ?
(c) Deehe Fleveer osj ceW keäÙeeW DeeÙes ?
(a) pees leewuee Ùee veehee ve pee mekesâ~
(d) ØelÙeskeâ ßeefcekeâ keâes oes ™heÙes efceues~
(b) pees DeJeMÙe nesves Jeeuee nes~
Gòej– (a)
(c) pees ØeceeCe mes efmeæ ve nes mekesâ~
JÙeeKÙee–‘mhe<šerkeâjCe’ kesâ meeLe ‘keâjvee’ keâe ØeÙeesie $egefšhetCe& nw Fme (d) pees meceÙe hej mebYeJe ve nes~
efueS efJekeâuhe (a) ceW GefuueefKele JeekeäÙe DeMegæ nw, Mes<e JeekeäÙe Megæ nQ~ Gòej– (*)
28. efvecveefueefKele ceW Megæ JeekeäÙe nw- JÙeeKÙee– JeekeäÙeebMe ‘pees ØeceeCe mes efmeæ ve nes mekesâ’ kesâ efueS Skeâ
(a) Yeejle keâYeer efyeÇšsve kesâ DeeOeerve Lee~ Meyo ‘DeØecesÙe’ ØeÙegòeâ efkeâÙee peelee nw~ leLee ‘pees leewuee Ùee veehee ve
(b) efvejhejeOeer JÙeefòeâ keâes oC[ veneR efceuevee ÛeeefnS~ pee mekesâ~’ kesâ efueS ‘DeØecesÙe’ Meyo keâe ØeÙeesie neslee nw~
(c) yeeoMeen ves cegòeânmle oeve efoÙee~ veesš- DeeÙeesie ves Fmekeâe Gòej (a) Deewj (c) oesveeW keâes ceevee Lee~
(d) Gmes hewef$ekeâ mecheefòe ceW efnmmee veneR efceuee~ yeeo ceW DeeÙeesie kesâ DeeefOekeâeefjkeâ Gòej he$ekeâ ceW Ùen ØeMve cetuÙeebkeâve
Gòej– (c) mes yeenj keâj efoÙee ieÙee nw~
JÙeeKÙee– ØeMve ceW GefuueefKele JeekeäÙeebMe ‘yeeoMeen ves cegòeâ nmle 33. ‘efkeâmeer hej efJepeÙe Øeehle keâjves keâer FÛÚe jKeves Jeeuee’ -
oeve efoÙee’ kesâ Deefleefjòeâ Mes<e meYeer DeMegæ nw keäÙeeWefkeâ ‘DeeOeerve’ kesâ JeekeäÙeebMe kesâ efueS Skeâ Meyo nw-
(a) efJeefpele (b) efJepeslee (c) efpeieer<eg (d) efpe%eemeg
mLeeve hej ‘DeOeerve’ ‘efvejhejeOeer’ JÙeefòeâ kesâ mLeeve hej ‘efvejhejeOe’ Deewj
Gòej– (c)
‘hewef$ekeâ’ kesâ mLeeve hej ‘hewle=keâ Meyo ØeÙegòeâ nesvee ÛeeefnS~
JÙeeKÙee– JeekeäÙeebMe ‘efkeâmeer hej efJepeÙe Øeehle keâjves keâer FÛÚe jKeves
29. efvecveefueefKele ceW keâewve-mee JeekeäÙe DeMegæ nw ?
Jeeuee’ kesâ efueS Skeâ Meyo ‘efpeieer<eg’ ØeÙegòeâ efkeâÙee peelee nw~
(a) Jen DeveskeâeW ceeceueeW ceW ieJeen Lee~
34. ‘pees Œeer metÙe& Yeer ve osKe mekesâ’ JeekeäÙeebMe kesâ efueS Skeâ Meyo nw-
(b) vethegj keâer OJeefve ceveceesnkeâ nw~
(a) MeeŒe%ee (b) DemetÙe&cheMÙee (c) ŒewCe (d) ogefve&Jeej
(c) Deehekeâer ceve:keâecevee hetjer nes~ Gòej– (b)
(d) ceveeref<eieCe ! cesjer yeele hej OÙeeve oW~
JÙeeKÙee– JeekeäÙeebMe ‘pees Œeer metÙe& Yeer ve osKe mekesâ’ kesâ efueS Skeâ
Gòej– (a) Meyo ‘DemetÙe&cheMÙee’ ØeÙegòeâ efkeâÙee peelee nw~
JÙeeKÙee– ‘Jen DeveskeâeW ceeceueeW ceW ieJeen Lee’~ JeekeäÙe DeMegæ nw~ 35. ‘kegâÚ Keeme MeleeX hej efkeâmeer keâeÙe& keâes keâjves keâe mecePeewlee’
efpemekeâe Megæ ™he nesiee– ‘Jen Deveskeâ ceeceueeW ceW ieJeen Lee,’ peye kesâ efueS Skeâ Meyo nw-
efkeâ Mes<e efJekeâuhe Megæ nw~ (a) keâce&"lee (b) yesueoejer (c) "skesâoejer (d) mebefJeoe
30. efvecveefueefKele ceW mes Megæ JeekeäÙe keâe ÛeÙeve keâerefpeS– Gòej– (d)
(a) mebmeeefjkeâ mecemÙeeDeeW ceW meYeer HeBâmes nQ~ JÙeeKÙee– JeekeäÙeebMe ‘kegâÚ Keeme MeleeX hej efkeâmeer keâeÙe& keâes keâjves
(b) efvejhejeOe JÙeefòeâ keâes oC[ keäÙeeW efceuee? keâe mecePeewlee’ kesâ efueS Skeâ Meyo ‘mebefJeoe’ ØeÙegòeâ efkeâÙee peelee nw~
(c) efnvoer osJeveeiejer efueheer ceW efueKeer peeleer nw~ 36. ‘efpemekeâer henues mes keâesF& DeeMee ve nes’ kesâ efueS Skeâ Meyo nw-
(d) DelÙeeefOekeâ og:Ke mene veneR peelee~ (a) DeØelÙeeefMele (b) DeeMeeleerle (c) DeeMeepevekeâ (d) ØelÙeeefMele
Gòej– (b) Gòej– (a)
UP RO/ARO (Pre) Hindi 2017 116 YCT
CLICK HERE FOR FREE MATERIAL

JÙeeKÙee– JeekeäÙeebMe ‘efpemekeâer henues mes keâesF& DeeMee ve nes’ kesâ efueS 43. ‘ceesleer’ keâe lelmece ™he nw-
Skeâ Meyo ‘DeØelÙeeefMele’ ØeÙegòeâ efkeâÙee peelee nw~ (a) ceewleer (b) ceewefòeâkeâ
37. ‘jepÙe Éeje efvekeâeuee ieÙee DeeefOekeâeefjkeâ DeeosMe’ – (c) cegòeâkeâ (d) GheÙeg&òeâ ceW mes keâesF& veneR
JeekeäÙeebMe kesâ efueS Meyo nw- Gòej– (b)
(a) DeefOeefveÙece (b) DeOÙeeosMe (c) DeefOeÙeeÛevee (d) DeefOemetÛevee JÙeeKÙee– leodYeJe Meyo ‘ceesleer’ keâe lelmece ‘ceewefòeâkeâ’ neslee nw~
Gòej– (b) 44. ‘efceóer’ keâe lelmece Meyo nw-
JÙeeKÙee– efoÙes ieÙes JeekeäÙeebMe ‘jepÙe Éeje efvekeâeuee ieÙee (a) ce=efòekeâe (b) ceefšdškeâe
(c) efceüefókeâe (d) ce=óer
DeeefOekeâeefjkeâ DeeosMe’ JeekeäÙe kesâ efueS Skeâ Meyo ‘DeOÙeeosMe’ nesiee~
Mes<e efJekeâuhe DemebYeJe nw~ Gòej– (a)

38. ‘Jen Œeer efpemekeâe heefle hejosMe (efJeosMe) ieÙee nes’ –


JÙeeKÙee– leodYeJe Meyo ‘efcešdšer’ keâe lelmece ‘ce=efòekeâe’ neslee nw~
JeekeäÙe kesâ efueS Skeâ Meyo nw- 45. efvecveefueefKele ceW mes leodYeJe Meyo nw-
(a) veJeesÌ{e (b) ØeJelmÙeleheeflekeâe (c) Øeesef<eleheeflekeâe (d) Deeieleheeflekeâe (a) Guetkeâ (b) ketâhe
Gòej– (c) (c) Dees P ee (d) hegmlekeâ
Gòej– (c)
JÙeeKÙee– ‘Jen Œeer efpemekeâe heefle hejosMe (efJeosMe) ieÙee nes’ JeekeäÙe
kesâ efueS Skeâ Meyo ‘Øeesef<eleheeflekeâe’ ØeÙegòeâ efkeâÙee peelee nw peyeefkeâ JÙeeKÙee– ØeMveiele efJekeâuheeW ceW ‘DeesPee’ leodYeJe Meyo nw efpemekeâe
‘Deeieleheeflekeâe’ keâe DeLe& neslee nw ‘efpemekeâe heefle hejosMe mes DeeÙee lelmece Meyo ‘GheeOÙeeÙe’ neslee nw~ Mes<e Meyo lelmece nw efpevekeâe
nes’~ leoddYeJe efvecveefueefKele nw-
lelmece leoddYeJe
39. ‘hewj mes efmej lekeâ’ JeekeäÙeebMe kesâ efueS Skeâ Meyo nw-
Guetkeâ Guuet
(a) efMeKe-veKe (b) efMejeshej
ketâhe kegâDeeB
(c) heeocemlekeâ (d) Deeheeocemlekeâ
hegmlekeâ heesLeer
Gòej– (d)
46. efvecveefueefKele ceW mes Skeâ Meyo lelmece nw-
JÙeeKÙee– ‘hewj mes efmej lekeâ’ JeekeäÙeebMe kesâ efueS Skeâ Meyo (a) mehetle (b) mejerme=he
‘Deeheeocemlekeâ’ ØeÙegòeâ efkeâÙee peelee nw~ Mes<e efJekeâuhe Demebiele nQ~ (c) meebkeâue (d) meebPe
40. ‘DevegefÛele yeele kesâ efueS Dee«en keâjves Jeeuee’ – JeekeäÙeebMe Gòej– (b)
kesâ efueS Skeâ Meyo nw- JÙeeKÙee– efJekeâuhe ceW GefuueefKele ‘mejerme=he’ lelmece Meyo nw peyeefkeâ
(a) ogje«en (b) Devee«ener mehetle, meebkeâue Deewj meebPe leodYeJe Meyo nQ efpevekesâ lelmece ›eâceMe:
(c) ogje«ener (d) kegâDee«ener megheg$e ëe=bKeuee Deewj mebOÙee nesles nw~
Gòej– (c) 47. efvecveefueefKele ceW mes leodYeJe Meyo nw-
JÙeeKÙee– JeekeäÙeebMe ‘DevegefÛele yeele kesâ efueS Dee«en keâjves Jeeuee’ (a) pÙeeslmvee (b) ßes<"er
kesâ efueS Skeâ Meyo ‘ogje«ener’ ØeÙegòeâ efkeâÙee peelee nw peyeefkeâ ‘DevegefÛele (c) hejer#ee (d) Iej
yeele kesâ efueS Dee«en’ kesâ efueS ‘ogje«en’ Meyo ØeÙegòeâ efkeâÙee peelee nw~ Gòej– (d)
JÙeeKÙee– ØeMve ceW GefuueefKele ‘Iej’ leodYeJe Meyo nw efpemekeâe
lelmece SJeb leodYeJe
lelmece ™he ‘ie=n’ neslee nw, peyeefkeâ Mes<e lelmece Meyo nw~
41. efvecveefueefKele ceW mes lelmece keâer Âef<š mes Megæ nw- 48. ‘DeKejesš’ keâe lelmece ™he nw-
(a) Meeefškeâe (b) jmmeer (a) De#eJeš (b) De#eJeeš
(c) jwve (d) mejmeeW (c) De#eesš (d) De#eÙeJeš
Gòej– (a) Gòej– (c)
JÙeeKÙee– leodYeJe lelmece JÙeeKÙee– leodYeJe Meyo ‘DeKejesš’ keâe lelmece Meyo ‘De#eesš’ neslee
meeÌ[er Meeefškeâe nw, peyeefkeâ ‘DeKeeÌ[e’ keâe lelmece De#eJeeš neslee nw~
jmmeer jefMce 49. efvecveefueefKele ceW mes lelmece-leodYeJe keâe mener Ùegice nw-
jwve jpeveer (a) centkeâ - cengDee (b) De#eJeeš - DeKeeÌ[e
mejmeeW me<e&he (c) Deeõkeâ - Deojkeâ (d) F#e - F&Ke
42. ‘yejele’ keâe lelmece ™he nw- Gòej– (b)
(a) Jeje&le (b) yejele (c) yeÇele (d) JejÙee$ee JÙeeKÙee–lelmece Meyo ‘De#eJeeš’ keâe leodYeJe Meyo ‘DeKeeÌ[e’ neslee
Gòej– (d) nw, Fme Øekeâej ØeMveiele efJekeâuhe (b) mener Ùegice nw peyeefkeâ Mes<e
JÙeeKÙee– leodYeJe Meyo ‘yejele’ keâe lelmece ‘JejÙee$ee’ neslee nw~ DeMegæ nw, efpevekeâe Megæ ™he efvecveefueefKele nw-
UP RO/ARO (Pre) Hindi 2017 117 YCT
CLICK HERE FOR FREE MATERIAL

lelmece leoddYeJe 55. ‘ogMÛeefj$e JÙeefòeâ mes mecyevOe veneR jKevee ÛeeefnS~’ Fme
ceOetkeâ cengDee JeekeäÙe ceW ØeÙegòeâ ‘ogMÛeefj$e’ Meyo JÙeekeâjCe keâer Âef<š mes
Deeõ&keâ Deojkeâ efkeâme mebJeie& ceW nw ?
F#eg F&Ke (a) meb%ee (b) meJe&veece
(c) efJeMes<eCe (d) ef›eâÙeeefJeMes<eCe
50. efv ecveefueefK ele ceW mes lelmece keâer Âef<š mes Skeâ Ùegice
Gòej– (c)
DeMegæ nw-
JÙeeKÙee– JeekeäÙe ‘ogMÛeefj$e JÙeefòeâ mes mecyevOe veneR jKevee ÛeeefnS’
(a) oer#ee - hejer#ee (b) Metkeâj - yeueerJeo&
ceW ‘ogMÛeefj$e’ Meyo JÙeekeâjCe keâer Âef<š mes efJeMes<eCe mebJeie& ceW Deelee
(c) keâhetle - mehetle (d) Jeefj<" - keâefve<"
Deelee nw~
Gòej– (c)
56. ‘ueeKeeW ueesieeW ves Fmes cenmetme efkeâÙee nw’ JeekeäÙe ceW efJeMes<eCe nw-
JÙeeKÙee– ØeMveiele efJekeâuhe ceW GefuueefKele ‘keâhetle-mehetle’ lelmece keâer (a) menmetme (b) ueesieeW
Âef<š mes DeMegæ Ùegice nw, efpemekeâe Megæ Ùegice ‘kegâheg$e-megheg$e’ nesiee (c) ueeKeeW (d) Fmes
peyeefkeâ Mes<e lelmece keâer Âef<š mes Megæ Ùegice nw~ Gòej– (c)
efJeMes<Ùe SJeb efJeMes<eCe JÙeeKÙee– JeekeäÙeebMe ‘ueeKeeW ueesieeW ves Fmes cenmetme efkeâÙee nw’ ceW
mebKÙeeJeeÛekeâ efJeMes<eCe ‘ueeKeeW’ ØeÙegòeâ efkeâÙee ieÙee nww~ efpeme efJeMes<eCe
51. ‘Gmekeâe ueÌ[keâe uecyee nw’ ceW efJeMes<eCe keâe ÛeÙeve keâerefpeS~ mes mebKÙee keâe yeesOe nes, Gmes mebKÙeeJeeÛekeâ efJeMes<eCe keâne peelee nw~
(a) ueÌ[keâe (b) uecyee Fmekesâ heeBÛe Yeso nesles nw– ieCeveeJeeÛekeâ, ›eâceJeeÛekeâ, DeeJe=efòeJeeÛekeâ,
(c) Gmekeâe (d) GheÙeg&òeâ ceW mes keâesF& veneR mecegoeÙeJeeÛekeâ Deewj ØelÙeskeâyeesOekeâ~
Gòej– (b) 57. ‘efJeMes<Ùe’ Jen Meyo neslee nw-
JÙeeKÙee– JeekeäÙeebMe ‘Gmekeâe ueÌ[keâe uecyee nw’ ceW iegCe JeeÛekeâ (a) efpeme Meyo keâer efJeMes<elee yeleeF& peeleer nw~
efJeMes<eCe ‘uecyee’ ØeÙegòeâ efkeâÙee ieÙee nw~ GuuesKeveerÙe nw efkeâ efpeme (b) efpeme Meyo kesâ Éeje efJeMes<elee yeleeÙeer peeleer nw~
efJeMes<eCe mes efkeâmeer meb%ee Ùee meJe&veece keâe iegCe-oes<e, ™he-jbie, (c) efpeme nsleg efJeMes<elee yeleeÙeer peeleer nw~
Deekeâej-Øekeâej, mecyevOe oMee Deeefo keâe yeesOe nes Gmes iegCe JeeÛekeâ (d) GheÙeg&òeâ ceW mes keâesF& veneR~
efJeMes<eCe keâne peelee nw~ Gòej– (a)
52. ØeÙeesie kesâ DeeOeej hej ‘heeC[g’ Meyo nesiee- JÙeeKÙee– meb%ee Ùee meJe&veece keâer efJeMes<elee yeleeves Jeeues Meyo keâes
(a) kesâJeue efJeMes<eCe (b) kesâJeue efJeMes<Ùe efJeMes<eCe keâne peelee nw Deewj efpeme Meyo keâer efJeMes<elee yeleeÙeer peeleer
(c) efJeMes<eCe Deewj efJeMes<Ùe oesveeW(d) GheÙeg&òeâ ceW mes Skeâ Yeer veneR
nw Gmes ‘efJeMes<Ùe’ keâne peelee nw~ ‘keâeuee IeesÌ[e’ ceW keâeuee efJeMes<eCe
leLee ‘IeesÌ[e’ efJeMes<Ùe heo nw~
Gòej– (c)
58. ‘ceesnve Skeâ DeÛÚe efJeÅeeLeea nw~’ JeekeäÙe ceW efJeMes<Ùe nw-
JÙeeKÙee– ØeÙeesie kesâ DeeOeej hej ‘heeC[g’ Meyo efJeMes<Ùe Deewj (a) ceesnve (b) Skeâ
efJeMes<eCe oesveeW ™heeW ceW ØeÙegòeâ efkeâÙee peelee nw~ ‘heeC[g’ keâe ØeÙeesie (c) DeÛÚe (d) efJeÅeeLeea
‘nukesâ jbie keâe’ efJeMes<eCe kesâ DeLe& ceW efkeâÙee peelee nw peyeefkeâ ceneYeejle Gòej– (d)
keâeueerve Decyeeefuekeâe Deewj $e+ef<e JÙeeme kesâ heg$e keâe veece ‘heeC[g’ Lee~ JÙeeKÙee– ØeMveiele JeekeäÙe ceW efJeÅeeLeea efJeMes<Ùe nw leLee ‘DeÛÚe’
Dele: ‘heeC[g’ meb%ee nesves kesâ keâejCe efJeMes<Ùe Yeer nesiee~ efJeMes<eCe heo nw~
53. ‘Ûelegj efJeÅeeLeea mes ØeMve hetÚes~’ JeekeäÙe ceW efJeMes<eCe nw- 59. ‘Dee" yeÌ[s Ûeesj hekeâÌ[s ieÙes Les, hegefueme keâer ueehejJeener mes
(a) efJeÅeeLeea (b) ØeMve DeeOes Ûeesj Yeeie ieÙes~’ JeekeäÙe ceW efkeâve MeyoeW ceW efJeMes<eCe-
(c) Ûelegj (d) hetÚes efJeMes<Ùe mecyevOe veneR nw ?
Gòej– (c) (a) yeÌ[s Ûeesj (b) DeeOes Ûeesj
JÙeeKÙee–JeekeäÙe ‘Ûelegj efJeÅeeLeea mes ØeMve hetÚes’ ceW efJeMes<eCe ‘Ûelegj’ nw, (c) hegefueme keâer ueehejJeener (d) Dee" yeÌ[s Ûeesj
pees efJeÅeeLeea keâer efJeMes<elee yelee jne nw~ Dele: ‘Ûelegj’ efJeMes<eCe Meyo nw~ Gòej– (c)
54. ‘‘Jen Deeoceer pees keâue Deehekesâ Iej DeeÙee Lee, yengle yeÌ[e
JÙeeKÙee– JeekeäÙeebMe ‘Dee" yeÌ[s Ûeesj hekeâÌ[s ieS Les, hegefueme keâer
ueehejJeener mes DeeOes Ûeesj Yeeie ieS’ ceW ‘hegefueme keâer ueehejJeener’ ceW
"ie nw~’’ GheÙeg&òeâ JeekeäÙe ceW efkeâleves efJeMes<eCe nQ ? efJeMes<eCe-efJeMes<Ùe mecyebOe veneR nw~
(a) leerve (b) oes
60. efvecveefueefKele MeyoeW ceW keâewve-mee efJeMes<eCe nw ?
(c) Ûeej (d) heeBÛe
(a) ›eâesOeer (b) keâCškeâ
Gòej– (c)
(c) Ûegveewleer (d) jener
JÙeeKÙee– efoÙes ieÙes JeekeäÙe ceW ‘Jen’ leLee ‘Deehekesâ’ oesveeW Meyo
meeJe&veeefcekeâ efJeMes<eCe nw, ‘yengle’ ØeefJeMes<eCe nw leLee ‘yeÌ[e’ Meyo Gòej– (a)
efJeMes<eCe nw~ Dele: Ûeej efJeMes<eCe Jen, Deehekesâ, yengle SJeb yeÌ[e nQ~ JÙeeKÙee– efJekeâuhe ceW GefuueefKele MeyoeW ceW ‘›eâesOeer’ efJeMes<eCe heo nw
peyeefkeâ leerve efJeMes<Ùe Deeoceer, Iej Deewj "ie nQ~ peyeefkeâ Mes<e keâCškeâ, Ûegveewleer Deewj jener efJeMes<Ùe heo nw~
UP RO/ARO (Pre) Hindi 2017 118 YCT
CLICK HERE FOR FREE MATERIAL

UPPSC RO-ARO (Pre) (Cancelled) Exam-2016


GENERAL STUDIES
Solved Paper
1. Which one of the following pairs is not Ans. (b) : The correct matches are :
correctly matched? National Environmental Engineering Nagpur
(a) Vitamin A - Night Blindness Research Institute
(b) Vitamin B3 - Pellagra
Central Food Technology Research Mysuru
(c) Vitamin D - Color Blindness
(d) Folic acid - Anemia Central Potato Research Institute Shimla
Ans. (c) : The correct match are :- Central Tobacco Research Institute Rajahmundry
Vitamin A - Night Blindness 4. Which one of the following is not correctly
Vitamin B3 - Pellagra matched?
Vitamin D - Rickets (a) Yeast - Fermentation
Folic Acid - Anemia (b) Casein - Milk Protein
2. Consider the following statements: (c) Aloe Vera - Angiosperm
Assertion (A): Cuscuta (Amarbel) is an (d) Nepenthes - Parasitic angiosperm
example of parasitic angiosperm. Ans. (d) : The correct matches are :
Reason (R) : It gets its nutrition from the leaves Yeast Fermentation
of the host plant. choose your answer from the Casein Milk Protein
codes given below:
Aloe Vera Angiosperm
(a) Both (A) and (R) are true and (R) is the
correct explanation of (A). Nepenthes Carnivorous
(b) Both (A) and (R) are true but (R) is not the 5. Which of the following is responsible for 'Bird
correct explanation of (A). Flu'?
(c) (A) is false, but (R) is true. (a) H5N1 (b) H1N1
(d) (A) is true, but (R) is false. (c) Zika (d) Retro
Ans. (d) : Cuscuta commonly known as Amarbel is a Ans. (a) : Avian influenza, sometime called bird flu or
genus of over 201 Species of Yellow orange or red avian flu, is a disease of birds, usually carried by wild
Parasitic plants. It is example of parasitic angiosperm. ducks and other water birds. Bird's flu viruses do not
Cuscuta does not have chlorophyll; it cannot synthesize normally infect humans. It is caused by a virus H5N1
its own food by photosynthesis. It gets its nutrition from capable of spreading to humans. Therefore correct
the roots and stem of the host plant without contributing answer should be option (a).
to the benefit of the host. Therefore correct answer 6. For which of the following rulers "Eka-
should be (d) (A) is true and (R) is False Brahmana" has been used?
3. Match List-I with List-II and select correct (a) Pushyamitra Shunga
answer from the code given below the lists: (b) Kharavela
List-I List-II (c) Gautamiputra Shatakarni
A. National Environmental 1. Rajahmundry (d) Susharman
Engineering Research Ans. (c) : Gautamiputra Shatakarni (106-130 AD) was
Institute the greatest ruler of Satavahana was also known for
B. Central Food Technology 2. Nagpur Eka-Brahmana. Hence, the correct answer in option (c).
Research Institute
7. The pinnacle of which of the following is built
C. Central Potato Research 3. Shimla in Dravida style?
Institute (a) Bhitargaon Temple
D. Central Tobacco 4. Mysuru (b) Teli Temple of Gwalior
Research Institute
(c) Kandaria Mahedeo Temple
Codes: (d) Osiya Temple
A B C D Ans. (b) : The pinnacle of the Teli Temple of Gwalior
(a) 2 3 4 1 is built in Dravida style. Dravidian architecture style
(b) 2 4 3 1 emerged in the Southern part of the Indian subcontinent
(c) 3 4 2 1 but can also be found in parts of North India. Hence,
(d) 1 2 3 3 the correct answer is option (b).
UP RO/ARO (Cancelled) Exam 2016 119 YCT
CLICK HERE FOR FREE MATERIAL

8. Who amongst the following was the last person 13. Who among the following translated the
converted by the Buddha? Mahabharata into Persian?
(a) Anand (b) Vasumitra (a) Abdul Qadir Badauni
(c) Goshal (d) Subhadda (b) Abul Fazl
Ans. (d) : Subhadda was the last convert and disciple of (c) Nizamuddin Ahmad
Buddha. Kondanna was the first Convert and disciple of (d) Shaikh Mubarak
Buddha. Ans. (a) : The Mahabharata was translated into Persian
9. An inscription by which of the following is at Akbar order, by 'Faizi' and 'Abdul Qadir Badauni and
found on the pillar containing Prayag Prasasti named Razamama means "book of war". It was
completed by the Maktab Khana, a translation bureau
of Samudragupta?
set up by Akbar to translate important Sanskrit book into
(a) Jahangir (b) Shahjahan Persian. Badauni also translated the Ramayana, and many
(c) Aurangzeb (d) Dara Shikoh Sanskrit tales and the Hindu epics in Persian in accordance
Ans. (a) : The Prayag Prasasti also known as the with the wish of Akbar. Hence, option (a) is correct.
Allahabad Pillar Inscription was composed by Harisena 14. Which one of the following kings was defeated
in Sanskrit. The Allahabad Prasasti was originally by Muhammad Ghori in the battle of
engraved on the Asokan pillor instituted at Kaushambi Chandawar?
near Allahabad in Uttar Pradesh. It was re-erected by (a) Prithviraj Chauhan (b) Jai Chand
the Mughal emperor Jahangir in 1605. There are three (c) Kumarpala (d) Bhima II
sets of inscriptions on the column belonging to three Ans. (b) : The last powerful King of the Gahadavala
important emperors, namely Ashoka Maurya, dynasty, Jai Chand was defeated and killed in 1194 AD
Samudragupta and Jahangir. by Muhammad Ghori in the battle of Chandawar. The
10. Which one of the following is NOT correctly battle between took place near Chandawar on the river
matched? Yamuna between Etawah and Kannauj.
List-I List-II 15. Among the following whose tomb is situated
(Dynasties) (Metals of coin) outside India?
A. Kushanas 1. Gold and copper (a) Humayun (b) Aurangzeb
B. Guptas 2. Gold and Silver (c) Jahandar Shah (d) Jahangir
Ans. (d) : The Tomb of the Jahangir, is outside India. It is
C. Satavahanas 3. Gold situated in Shahdara Bagh at Lahore along the banks of Ravi
D. Kalachuris 4. Gold, silver and River. The Tomb of Aurangzeb is situated in Aurangabad ,
copper Maharashtra. Humayus tomb is situated in Delhi.
Ans. (c) : The correct matched are : 16. Match List- I with List -II and select the
List-I (Dynasties) List-II (Metals of correct answer from the code given below the
Coins) lists:
Kushanas Gold and Copper List-I List-II
Guptas Gold and Silver A. Hawkins 1. 1615-1619
Satavahans Silver and Copper B. Thomas Roe 2. 1608-1611
Kalachuris Gold, Silver and Copper C. Manucci 3. 1585-1586
11. Which amongst the following families was the D. Ralph Fitch 4. 1653-1708
first to have matrimonial alliance with Akbar? Codes:
(a) Rathor (b) Sisodiya A B C D
(c) Kachwaha (d) Chauhan (a) 1 2 3 4
Ans. (c) : Raja Bharmal (Rajput Kachwaha ruler of (b) 2 1 4 3
Amber) was the first to have matrimonial alliance with (c) 3 4 2 1
Akbar in 1562. Raja Bharmal proposed Akbar to marry (d) 2 1 3 4
his daughter Harkhabai (Jodha). Akbar accepted the Ans. (b) :
offer and it was the first marriage of Akbar with any List - I List-II
Rajput girl. Hence, option (c) is correct answer. Hawkins – 1608 - 1611
12. Which one of the following was the capital of Thomas Roe – 1615- 1619
the Sultanate during the reign of lltutmish? Manucci – 1653 - 1708
(a) Agra (b) Lahore Ralph Fitch – 1585- 1586
(c) Badaun (d) Delhi 17. Who among the following was bestowed with
Ans. (d) : Delhi was the capital of the Sultanate during the title of Saheb-e-Alam Bahadur by Bahadur
the reign of Iltutmish. After Aibak's death, Iltutmish Shah during the uprising of 1857?
dethroned his unpopular successor Aram Shah in 1211, (a) Azimullah (b) Birijis Qadr
and set up his capital at Delhi. (c) Bakht Khan (d) Hasan Khan
UP RO/ARO (Cancelled) Exam 2016 120 YCT
CLICK HERE FOR FREE MATERIAL

Ans. (c) : Bakht Khan was bestowed with the title of 21. Consider the following passage:
'Saheb-e-Alam Bahadur' by Bahadur Shah during the "Born in 1853 he was a Parsi from Western
uprising of 1857. Bakht Khan was an Indian freedom India. He was the editor of "Indian Spectator"
fighter and commander chief of Indian rebel forces in the and "Voice of India". He was a social reformer
Indian Rebellion of 1857 against the East India Company. and was the chief crusader for the Age of
18. Match List-I with List-II and select the correct Consent Act 1891."
answer from the code given below the lists
Who is being described in the above
List-I List-II paragraph?
A. Pondicherry 1. Dutch (a) Dadabhai Naoroji
B. Goa 2. French (b) B.M. Malabari
C. Tranquebar 3. Portuguese (c) B.P. Wadia
(d) Naoroji Furdon Ji
D. Sadras 4. Danish (Dane)
Ans. (b) :The mentioned lines describes about B.M.
Codes:
Malabari. He was advocate of women's social reform in
A B C D
India and a champion of women's suffrage in India.
(a) 2 3 4 1
(b) 1 2 3 4 22. Consider the following statements:
(c) 3 4 1 2 Assertion (A) : Jawaharlal Nehru represented the
(d) 4 1 2 3 Indian National Congress in the Second Round
Ans. (a) : The correct matched are : Table conference (1932).
List-I List-II Reason (R): It was implicit in the Gandhi Irwin
Pondicherry – French Pact (1931) that the Indian National Congress will
Goa – Portuguese participate in the Second Round Table Conference
Tranquebar – Danish (Dane) (1932).
Sadras – Dutch Choose the correct answer from the code given
19. Match List-I with List-II and select the correct below:
answer using the code given below the lists (a) Both (A) and (R) are true and (R) is the
List-I List-II correct explanation of (A).
(Name of the (Place) (b) Both (A) and (R) are true but (R) is not the
Revolutionaries) correct explanation of (A).
A. Nana Saheb 1. Delhi (c) (A) is true, but (R) is false.
B. Nawab Hamid Ali Khan 2. Kanpur (d) (A) is false, but (R) is true.
Ans. (d) : Mahatma Gandhi represented the Indian
C. Maulavi Anmaddullah 3. Lucknow
National Congress in the Second Round Table
D. Mani Ram Diwan 4. Assam Conference (1932). Hence, Assertion is wrong. It was
Codes: implicit in the Gandhi Irwin Pact (1931) that the Indian
A B C D National Congress will participate in the Second Round
(a) 1 2 3 3 Table Conference (1932). Hence, Reason is correct.
(b) 1 2 3 4 Therefore, option (d) is correct.
(c) 2 1 4 3 23. Consider the following organizations;
(d) 2 1 3 4
1. Bangabhasha Prakasika Sabha
Ans. (d) : The correct match are : 2. Landholders Society
List-I List-II
3. Bengal British India Society
(Name of the Revolutionaries) (Place)
Nana Saheb – Kanpur 4. Indian League
Nawab Hamid Ali Khan – Delhi Select the correct chronological order of
Maulvi Ahmadullah – Lucknow founding of these organizations from the code
Mani Ram Diwan – Assam given below:
20. When was the first tea company in Assam (a) 1,2,3,4 (b) 1,3,2,4
established? (c) 2,1,3,4 (d) 2,3,4,1
(a) 1835 (b) 1837 Ans. (a) : The correct chronological order organization
(c) 1839 (d) 1841 are :
Ans. (c) : The tea industry in Assam is about 172 years Organization Year
old. Robert Brue in 1823 discovered tea plants growing Bangabhasha Prakasika Sabha - 1836
wild in upper Brahmaputra Valley. The Assam Landholders Society - 1838
Company was formed in 1839, to cultivate tea in
Assam, India. It was the first tea company in India and Bengal British India Society - 1843
it was the world's first privately owned tea company. Indian League - 1875
UP RO/ARO (Cancelled) Exam 2016 121 YCT
CLICK HERE FOR FREE MATERIAL

24. Which one of the following pairs is not 28. Match List-I with List-II and select the correct
correctly matched? answer from the code given below the lists:
A. V.D Savarkar 1. The Indian War of List-I List-II
Independence (Author) (Title of the Book)
A. 7th Schedule 1. Languages
B. R.C. Majumdar 2. The Sepoy Mutiny and th
Revolt of 1857 B. 8 Schedule 2. Disqualification on
ground of defection
C. Radha Kamal 3. Awadh in Revolt (1857- th
Mukherjee 1858) C. 9 Schedule 3. Distribution of
legislative powers
D. S.B. Chaudhuri 4. Civil Rebellions in the
D. 10th Schedule 4. Validation of certain
Indian Mutinies, 1857-
Acts
1859
Ans. (c) : The correct match are :
Ans. (c) : Awadh in Revolt (1857-1858) was authored List-I List-II
by Rudrangshu Mukerjee not by Radha Kamal 7th Schedule - Distribution of legislative
Mukherjee. Hence option (c) is not correctly matched. power
Rest of the option is correctly match. 8th Schedule - Languages
25. Consider the following provinces of British 9th Schedule - Validation of certain Act
India and identify those where the Indian 10th Schedule - Disqualification on
National Congress did not form ministry in ground of defection
1937? 29. Which one of the following cannot be the
(1) Central Provinces (2) Orissa ground for proclamation of Emergency under
(3) Bengal (4) Punjab the Constitution of India?
Select the correct answer from the code given (a) War
below: (b) External aggression
(a) 1 & 2 (b) 3 & 4 (c) Armed rebellion
(c) 2 & 3 (d) 1 & 4 (d) Internal disturbance
Ans. (b) : Provincial elections were held in British India Ans. (d) : As per Article 352, ground for proclamation
of emergency is external aggression, war and armed
in the winter of 1936 – 1937 as mandated by the
rebellion. Ground of internal disturbance was repealed
Government of India Act 1935. In the 1937 elections, by 44th Constitutional Amendment.
the Indian National Congress emerged in the power of
eight of the provinces the three exceptions being 30. The authoritative text of the Indian
Bengal, Punjab, Sindh. Congress formed Ministry in 8 Constitution in Hindi was authorized to be
out of the 11 provinces. Hence option (b) is correct published which of the following constitutional
amendments?
answer. The All India Muslim League failed to form the
(a) 57th Amendment, 1987
government in any province.
(b) 58th Amendment, 1987
26. Who among the following was the president of
(c) 59th Amendment, 1988
the Central legislative Assembly in August
(d) 60th Amendment, 1988
1925?
(a) C.R. Das (b) Motilal Nehru Ans. (b) : The Constitution (58th Amendment Act),
1987, provided for an authoritative text of the
(c) M.R. Jayakar (d) Vitthal Bhai Patel
constitution in Hindi language and gave the Hindi
Ans. (d) : Vitthal Bhai Patel was elected President version of the constitution the same legal sanctity. It
(Speaker) of the Central Legislative Assembly in inserted article 394-A in Part XXII of the constitution. It
August 1925. He was the co-founder of Swaraj Party empowered the President to publish the translation of
and elder brother of Sardar Vallabh Bhai Patel. He the constitution in hindi language.
played an important role in Rowlatt Satyagraha of 1919
31. Which of the following is not correctly matched
and Non Co-operation Movement of 1920. under the Constitution of India?
27. Who amongst the following wrote the book A. The Panchayats - Part IX
"Politics in India"?
B. The Municipalities - Part IX-A
(a) Rajni Kothari (b) M.N Srinivas
(c) Ashish Nandy (d) D.L. Sheth C. The Co- operative - Part IX-B
Societies
Ans. (a) : Rajni Kothari wrote the book "Politics in
India". Kothari also wrote, Caste in Indian Politics and D. Tribunals - Part X
Rethinking Democracy. He was an Indian political Ans. (d) : Part XIVA of the Indian Constitution consists
scientist, political theorist, academic and writer. He was of Tribunals. Tribunals include Article 323 A and 323
the founder of Centre for the Study of Developing B. Hence, option (d) is not correctly matched. Rest of
Societies (CSDS) in 1963. the option is correctly matched.
UP RO/ARO (Cancelled) Exam 2016 122 YCT
CLICK HERE FOR FREE MATERIAL

32. Consider the following statements in the Ans. (b) : The Eighth five Year plan commenced in
context of Indian Constitution: 1992 in background of LPG reforms It's main objective
1. The Union Council of Ministers shall be were:-
collectively responsible to the Lok Sabha. • Creation of employment, check population growth,
2. Union Ministers shall hold office during and overall human development.
the pleasure of the Prime Minister. • Primary health facilities, Drinking Water &
(a) Only 1 is correct. Vaccination in all villages
(b) Both 1 and 2 are correct. • Growth and diversification of agricultural activates
(c) Neither 1 nor 2 is correct. • Strengthen the basic infrastructure.
(d) Only 2 is correct. • The plan was a plan for managing the transition from
a centrally planned economy to market led economy
Ans. (a) : Article 75 in of the constitution provides for through indicative planning.
following provisions • By 1992, India was a party to WTO and decision of
• The Prime Ministers shall be appointed by the opening of India Economy was taken to correct the
President and the other Ministers shall be appointed increasing deficit and foreign debt.
by the President on the advice of the Prime Minister.35. Consider the following statements:
• The Ministers shall hold office during the pleasure of 1. The GST council is chaired by the Union
the president. Hence, statement (2) is wrong. Finance Minister and the Minister of State-
• The Council of Ministers shall be collectively in-charge of Revenue of Finance at the
responsible to the House of the People (Lok Sabha). centre is a member.
Hence statement (1) is correct. 2. The GST Council will decide the tax rate,
exempted goods and the threshold under
33. Match List- I with List-II and select the correct the new taxation regime.
answer from the code given below the lists: 3. State Governments will have the option to
List-I List-II levy VAT, if they so decide.
(a) Only 1 is correct
(Programmes) (Year of
(b) Only 2 is correct.
Launching)
(c) Only 2 and 3 are correct.
A. Swarna Jayanti Gram 1. 1997 (d) Only 1 and 2 are correct.
Swarozgar Yojana
Ans. (d) : The GST council is a joint forum of the
B. Swarna Jayanti Shahari 2. 1989 centre and the states, Set up by the President as per
Rozgar Yojana Article 279 (A) of the Constitution
C. Jawahar Rozgar Yojana 3. 1999 The members of the council include the Union Finance
D. National Rural Health 4. 2005 Minister (Chairperson), the Union Minister of State
Mission (finance) from the centre. Each state can nominate a
minster in-charge of finance or taxation or any other
Codes: minister as member. The council is meant to "made
A B C D recommendation to the Union and the states on
(a) 1 2 3 4 important issues related to GST, like the goods and
(b) 2 1 3 4 service that may be subjected or exempted from GST. It
(c) 3 1 2 4 also decides on various rate slabs of GST.
(d) 4 3 2 1 36. Which among the following States was the
largest producer of wheat in India in the year
Ans. (c) : The correct match are : 2015-16?
List-I List-II (a) Haryana (b) Uttar Pradesh
(Programmes ) (Years of Launching) (c) Punjab (d) Bihar
Swarna-Jayanti Gram Ans. (b) : Uttar Pradesh was the largest producer of
Swarozgar Yojana- 1999 wheat in India in the year 2015-16. Most of the wheat
Swarna Jayanti Shahari Rozgar Yojana– 1997 produced in the country comes from Uttar Pradesh,
Jawahar Rozgar Yojana – 1989 Punjab, Haryana, Madhya Pradesh, Rajasthan, Bihar
and Gujarat. In the year 2021-22, total production of
National Rural Health Mission – 2005 wheat :-
34. Which of the following Five Year Plans of India Uttar Pradesh (33.94 MT)
recognized human development as the core of Madhya Pradesh (22.41MT)
developmental efforts? Punjab (14.82 MT)
(a) Seventh Five Year Plan 37. Consider the following statements about
(b) Eighth Five Year Plan NABARD:
(c) Sixth Five Year Plan 1. NABARD was set up by an Act of
(d) Fifth Five Year Plan Parliament in 1982.
UP RO/ARO (Cancelled) Exam 2016 123 YCT
CLICK HERE FOR FREE MATERIAL

2. It facilitates credit for agriculture only. Of these:


Of these: (a) Only 1 is correct
(a) only 1 is correct (b) Only 2 is correct
(b) only 2 is correct (c) 1 and 2 both are correct
(c) 1 and 2 both are correct (d) Neither 1hor 2 is correct
(d) Neither 1 nor 2 is correct Ans. (b) : The Kisan Credit Card (KCC) Scheme was
Ans. (a) : National Bank for Agriculture and Rural introduced in 1998 for issue of KCC to farmers on the
Development (NABARD) was established on 12 July basis of their holding for uniform adoption by the banks
1982 by an Act of the Parliament. NABARD, as a so that farmers may use them to readily purchase
Development Bank, is mandated for providing and agriculture inputs such as seeds, fertilizers, pesticides
regulating Credit and other facilities for the promotion etc. Hence, statement (1) is incorrect. The scheme aims
and development of agriculture, small-scale industries, at providing and timely credit support from the banking
cottage and village industries, handicrafts and other system under a single window with flexible and
rural crafts and other allied economics activities in rural simplified procedure to the farmers for their cultivation
areas with a view to promoting integrated rural and other needs indicated below:-
development and securing prosperity of rural areas, and
a. To meet the short term credit requirements for
for matters connected therewith or incidental there to.
cultivation of crops;
Therefore, statement (1) is correct and statement (2) is
incorrect. b. Post -harvest expenses;
38. Which one of the following signifies the folk c. Produce marketing loan;
culture? d. Consumption requirements of farmer household;
(a) Cultural practices of more civilized people. e. Working Capital for maintenance of farm assets and
(b) Cultural practices of urban people. activities allied to agriculture;
(c) Cultural practices of common people. f. Investment credit requirement for agriculture and
(d) Cultural practices of people of modern society. allied activities.
Ans. (c) : Folk culture can be defined as a rich culture 42. The net crop sown area in India is about:
which have been developed by common people and (a) 12 crore ha (b) 16 crore ha
have become popular over time. Songs, dance and (c) 14 crore ha (d) 17 crore has
storytelling are some of the forms of folk culture Ans. (c) : The physical extent of land on which crops
through which common people have expressed are sown and harvested in known as 'Net Sown Area'.
themselves. Folk culture focused primarily on a The net crop sown area is India is about 14 crore ha. As
traditions practiced by small homogeneous, rural per the land use statistics 2016-17, the total
groups living in relative isolation from other groups. geographical area of the country is 328.7 million
Hence, option (c) is correct. hectares, of which 139.4 million hectares is the reported
39. The Rig Veda is the net sown area and 200.2 million hectares is the gross
(a) Collection of hymns (b) Collection of stories cropped area with a cropping intensity of 143.6%. The
(c) Collection of words (d) Text of war net area sown works out to 42.4% of the total
Ans. (a) : The Rigveda is a collection of 1,028 hymns geographical area.
divided into 10 Mandalas. They are the earliest 43. Which one of the following particles has less
compositions and depict the life of the early Vedic than 0.002mm diameter?
people in India. (a) Clay (b) Silt
40. Which one of the following is not correctly (c) Fine sand (d) None of the above
matched? Ans. (a) : Clay has the smallest size of particles, less
(a) Vilayat Khan - Sitar than 0.002 mm in diameter. Single clay particle cannot
(b) Alla Rakah - Table be seen with a naked eye. The best top soil for growing
(c) Hari Prasad Chaurasia - Flute plants is loam soil. Loamy soil is a mixture of sand, silt
(d) Amjad Ali Khan - Pakhawaj and clay. Silt occurs as a deposit on riverbeds. The size
Ans. (d) : Amjad Ali Khan is an eminent Indian of the silt particles is between those of sand and clay.
classical sarod player, he is popularly known as the 44. Which of the following is not correctly
'Sarod Samrat'. He was awarded India's second highest matched?
civilian honor Padma Vibhushan in 2001, Rest of the (a) Jawaharlal Nehru Krishi - Madhya Pradesh
given options are correctly matched. Vishwavidyalaya
41. Consider the following statements about Kisan (b) Sardar Vallabh Bhai Patel - Uttar Pradesh
Credit Card Scheme: University of Agriculture
1. The Kisan Credit Card Scheme was and Technology
launched in the year 2001. (c) Indira Gandhi Krishi - Rajasthan
2. The Scheme provides adequate and timely Vishwavidhyalaya
credit support to the famers for their (d) Acharya NG Ranga - Telangana
cultivation needs. Agricultural University
UP RO/ARO (Cancelled) Exam 2016 124 YCT
CLICK HERE FOR FREE MATERIAL

Ans. (c) : Indira Gandhi Krishi Vishwavidyalaya Codes:


situated in the capital of Raipur, Chhathisgarh (IGKV) A B C D
was established on 20th January 1987 after bifurcating (a) 4 2 3 1
from Jawahar Lal Nehru Krishi Vishwavidyalaya, (b) 1 3 2 4
Jabalpur, Madhya Pradesh. Rest of given option are
(c) 3 1 4 2
correctly matched.
(d) 2 4 1 3
45. Karnal Bunt is a disease of
(a) Barley crop (b) Wheat crop Ans. (a) : The correct matches are :
(c) Bajra crop (d) Jowar crop List-I List-II
Ans. (b) :Karnal bunt is a fungal disease of wheat and Capital Gain Tax – Sale of Property
triticale. The smut fungus Tilletia Indica invades kernels Central Excise Duty – Factory Produce
and obtains nutrients from the endosperm, leaving Custom Duty – Import
behind waste products with disagreeable odor. Corporate Tax – Income
46. Which one of the following is not correctly 50. The Negotiable Instruments Act came into
matched? force from:
Plant Nutrients Role (a) 1881 A.D. (b) 1882 A.D.
(a) Nitrogen - Growth and Development (c) 1883 A.D. (d) 1884 A.D.
(b) Phosphorus - Root development Ans. (b) : The Negotiable Instruments Act, 1881 (NI
Act) was enacted on 9th December 1881 and came into
(c) Potassium - Regulating stomatal
opening and closing force on 1st March, 1882 with a view to regulate the
exchange of negotiable instruments such as promissory
(d) Boron - Disease resistance notes, bill exchange and cheques. However, some other
Ans. (d) : Boron is one of the essential nutrients for the documents are also recognized as negotiable
optimum growth, development, yield , and quality of instruments on the basis of custom and usage, like
crops. It performs many important functions in plants hundis, treasury bills, share warrants, etc, provided they
and is mainly involved in cell wall synthesis and are featured with negotiability.
structural integration.
51. Micro, Small and Medium Enterprises
47. A consumer co- operative store is set up by Development Act (MSMED) was passed in the
(a) Members year
(b) Registrar of co- operative societies (a) 2005 A.D. (b) 2006 A.D.
(c) Central Government (c) 2007 A.D. (d) 2008 A.D.
(d) State Government
Ans. (b) : Micro, Small and Medium Enterprises
Ans. (a) : A consumer co-operative store is an Development Act was passed in the year of 2006 The
organization owned, managed and controlled by definition of micro, small and medium enterprises in as
consumers themselves. The main motive of consumer
cooperative stores is to provide supreme quality goods under :
and services to consumers at reasonable prices and (i) A Micro enterprise is an enterprise where the
protect themselves from the exploitation of middleman. investment in plant and machinery or equipment
does not exceed Rs. 1 crore and turnover does not
48. SEBI Act was passed in
exceed Rs. 5 crore.
(a) 1956 (b) 1962
(c) 1992 (d) 2013 (ii) A Small enterprise is an enterprise where the
investment in plant and machinery as equipment
Ans. (c) : The Securities and Exchange Board of India does not exceed Rs. 10 crore and turnover does
(SEBI) was constituted as a non-statutory body in April
not exceed Rs. 50 crore.
12, 1988 through a resolution of Government of India.
SEBI was established as a statutory body in the year (iii) A Medium enterprise is an enterprise where the
1992 and the provisions of the Securities and Exchange investment in plant and machinery as equipment
Board of India Act, 1992 (15 of 1992) come into force does not exceed Rs. 50 crore and turnover does
on January 30, 1992. The basic function the SEBI as "to not exceed Rs. 250 crore.
protect the interests of investors in securities and to 52. Investment in Plant and Machinery in a small
promote the development of, and to regulate the scale manufacturing unit should be:
securities market." Hence, option (c) is correct. (a) Not more than ` 10 lakh
49. Match List-I with List -II and select the correct (b) Above ` 10 lakh but not more than ` 25 lakh
answer from the code given below the lists (c) Above ` 25 lakh but not more than 5 crore
List-I List-II (d) Above ` 5 crore but not more than ` 10 crore
A. Capital Gain Tax 1. Income Ans. (c) : When question was asked, A small and
B. Central Excise Duty 2. Factory Produce medium enterprises was defined as an enterprise where
the investment in plant and machinery in a small scale
C. Custom Duty 3. Import manufacturing unit should be above Rs. 25 lakh but not
D. Corporate Tax 4. Sale of Property more than Rs. 5 Crore.
UP RO/ARO (Cancelled) Exam 2016 125 YCT
CLICK HERE FOR FREE MATERIAL

Note :- As per latest published by Ministry of Micro, 57. Which among the following was used as the
Small and Medium Enterprises notification on 1st June, motto for Census of India 2011?
2020, the following criteria for a small enterprises, (a) Our Future, Our Country
where the investment in Plant and Machinery or (b) Our Country, Our Census
Equipment : not more than Rs. 10 crore and annual (c) People of India, Our Census
turnover not more than Rs. 50 crore. (d) Our Census, Our Future
53. Based on 2011 Census which of the following Ans. (d) : "Our Census Our Future" was used as the
figures in respect of Uttar Pradesh is not motto for Census of India 2011. It was released on 31st
correctly matched? March 2011 by Union Home Secretary and RGCCI
Characteristics Value (Registrar General and Census Commissioner of India)
(a) Percentage population of U.P. - 18.5% of India. Census 2011 was the 15th census of India and
to population of India 7th census after independence.
(b) Density per sq. km. - 829 58. Which among the following sanctuaries is not
(c) Decadal population growth of - 20.09% correctly matched?
U.P. during 2001-2011 Sanctuary Place
(d) Female literacy - 57.18% (a) Samaspur - Rae Barelli
Ans. (a) : According to the census of 2011, 16.51% of (b) Nawabganj - Allahabad
the total population of the whole country lives in Uttar (c) Bakhira - Sant Kabir Nagar
Pradesh. Hence, option (a) is not correctly matched. (d) Sandi - Hardoi
Rest of the option correctly matched.
54. Which of the following states has the largest Ans. (b) : Nawabganj Bird Sanctuary, renamed in 2015
population? Shahid Chandra Shekhar Azad Bird Sanctuary, is a bird
(a) Maharashtra (b) Bihar sanctuary located in Unnao district on the Kanpur-
Lucknow highway in Uttar Pradesh. Rest of all the
(c) Gujarat (d) Uttar Pradesh correctly matched.
Ans. (d) : The most populated states of India as per 59. The measurement unit of the ecological
census 2011 is Uttar Pradesh followed by Maharashtra footprint is:
and Bihar while Sikkim is the least populous state in
(a) Global hectares (b) Nanometer
India.
(c) Hoppus cubic foot (d) Cubic ton
55. Consider the following Statements:
Assertion (A): The census of India is carried out Ans. (a) : Ecological foot print is the impact of human
every 10 years. activities measured in terms of the area of biologically
productive land and water required to produce the
Reason (R): The population of India has largely goods consumed and to assimilate the waste generated.
remained unchanged over the period of ten years. It is measured in global hectares.
Select the correct answer from the code given
below: 60. Consider the following statements:
(a) Both (A) and (R) are true and (R) correctly Assertion (A) The loss of forest cover in India has
explains \(A). been directly associated with Demographic
Transition in India
(b) Both (A) and (R) are true but (R) does not
Reason (R): The forest cover is generally
correctly explain (A). negatively related with population growth.
(c) (A) is true, but (R) is false. (a) Both (A) and (R) are true and(R) explains (A)
(d) (A) is false, but (R) is true. (b) Both (A) and (R) are true but (R) does not
Ans. (c) : The Census of India is conducted every 10 explain (A).
years. The responsibility of conducting the decadal (c) (A) is true, but (R) is false.
census rests with the office of the Registrar General and (d) (A) is false, but (R) is true.
Census Commissioner of India, Ministry of Home
Affairs. Government of India. Ans. (a) : The loss of forest cover in India has been
directly associated with Demographic Transition in
The total population of India in 2011 was 121 crore
India. Hence, assertion (A) is correct. The forest cover
(approx) which is an increase of approx 19 crore
is generally negatively related with population growth
compared to 102 crore population in 2001. Thus
as due to increased population, pressure on existing
Assertion (A). is correct and Reason (R) is wrong.
resources increases which results into resource
56. Which among the following has the lowest depletion and pollution. Hence Reason (R) is correct
population density as per Census of 2011? explanation of assertion (A). Therefore, Both Assertion
(a) Uttar Pradesh (b) Bihar and Reason are correct and Reason is the correct
(c) West Bengal (d) Kerala explanation for Assertion.
Ans. (a) : In the given option, Uttar Pradesh has the 61. Which one of the following is located in
lowest population density As per census of (829) 2011 Chhattisgarh?
829, West Bengal (1028), Bihar (1106), and Kerala (a) Anshi National Park
(860). Population density. (b) Betla National Park
UP RO/ARO (Cancelled) Exam 2016 126 YCT
CLICK HERE FOR FREE MATERIAL

(c) Indravati National Park (c) (A) is true, but (R) is false.
(d) Gugamal National Park (d) (A) is false, but (R) is true.
Ans. (c) : Indravati National Park is located in the Ans. (b) : Urbanization refers to the movement of
Bijapur district of Chhatisgarh. It derives its name from people and human settlements from the rural areas to
the nearby Indravati River. It is the home of rare Wild the urban areas. factors like employment opportunities,
Buffalo. Hence, option (c) is correct. a higher standard of living, better facilities and
62. Match List-I with List-II and select the correct infrastructure push people to migrate from rural areas
answer from the code given below the lists: to urban areas. Urbanization in India has increased
rapidly after 2001 Hence, statement (1) is correct.
List-I List-II A revolution in mobile communication has been taking
(Biosphere Reserve) (States) place in India because of liberalization in government
A. Similipal 1. Himachal Pradesh policy and the entry of private players, India has
B. Dihang- Dibang 2. Uttarakhand become one of the fastest growing economy in terms of
mobile communication, therefore, both (A) and (R) are
C. Nokrek 3. Arunachal Pradesh true but (R) does not explains (A).
D. Cold Desert 4. Odisha 65. Arrange the following states in descending
5. Meghalaya order of urbanization as per 2011 Census and
Codes: select the correct answer from the code given
below the states:
A B C D
1. Kerala 2. Mizoram
(a) 1 3 5 4
3. Goa 4. Tamil Nadu
(b) 1 5 2 4
(a) 3,2,4,1 (b) 1,2,3,4
(c) 4 5 2 1
(c) 3,4,1,2 (d) 4,1,2,3
(d) 4 3 5 1
Ans. (a):- According to census 2011, the state in
Ans. (d) : The correctly matched are : descending order of degree of Urbanization are;-
List-I List-II Goa (62.17%)
(Biosphere Reserve) (States) Mizoram (52.11%)
Similipal – Odisha Tamil Nadu (48.4%)
Dihang-Dibang – Arunachal Pradesh Kerala (47.7%)
Nokrek – Meghalaya
66. Which one of the following states does not have
Cold Desert – Himachal Pradesh a million plus urban agglomeration as per 2011
63. Consider the following statements about Census?
Bhitarkanika Mangroves: (a) Haryana (b) Jammu & Kashmir
1. It is located in the deltaic region of (c) Odisha (d) Jharkhand
Vamsadhara and Subarnarekha rivers. Ans. (c) : As per the Census of 2011, there are 53 urban
2. It lies in the state of West Bengal. agglomerations in India with a population of 1 million
Of these: or more. The maximum number of million cities are in
(a) Only 1 is correct. Kerala and Uttar Pradesh with seven such cities in each
(b) Only 2 is correct. of these two states. There is no million city in Odisha.
(c) 1 and 2 both are correct. 67. As per Census of India, 2011 which of the
(d) Neither 1 nor 2 is correct. following pairs of cities recorded the highest
Ans. (d) : The Bhitarkanika Mangroves are a mangrove population?
wetland in state of Odisha. The Bhitarkanika (a) Kolkata and Delhi
Mangroves are located in the delta of river Brahmani (b) Delhi and Bengaluru
and Baitarani rivers. The Mangroves harbor one of (c) Mumbai and Kolkata
India's largest populations of saltwater crocodiles. (d) Mumbai and Delhi
Hence both given statements are incorrect. Therefore, Ans. (d) : As per census of India, 2011 Mumbai and
neither (1) nor (2) is correct. Delhi pairs of cities recorded the highest population.
64. Consider the following statements: The population of Mumbai (12, 442, 373) and Delhi
Assertion (A): Urbanization in India has (16, 787, 941).
increased rapidly after 2001. 68. The percentage of India's population in the
Reason (R): A revolution in mobile total population of the world as per 2011
communication has been taking place in India. Census is :
Select the correct answer from the code given (a) 17.31 (b) 18.50
below: (c) 18.90 (d) 19.05
(a) Both (A) and (R) are true and (R) truly Ans. (a) : The percentage of India's population in that
explains (A). population of the world as per 2011 census is 17.5%.
(b) Both (A) and (R) are true but (R) does not The population of India as per 2011 census is 1210.19
explain (A) million.
UP RO/ARO (Cancelled) Exam 2016 127 YCT
CLICK HERE FOR FREE MATERIAL

69. The UNESCO declared which one of the (a) ` 29,365 crore (b) ` 55,250 crore
following National Parks of India as the World (c) ` 64,275 crore (d) ` 65,250 crore
Heritage Site in July, 2016? Ans. (d) : The former Union Finance Minister Arun
(a) Kaziranga National Park Jaitley announced in October, 2016 that under the
(b) Kanchenjunga National Park Income Declaration Scheme (IDS), 2016 the amount of
(c) Corbett National park declared black money till September 30, 2016 is Rs.
(d) Valley of Flowers National Park 65,250 crore. The IDS, 2016 was launched by Income
Ans. (b) : Khangchendzonga (Kanchenjunga) National Tax Department, Government of India. The scheme
Park is located in Sikkim. It was inscribed to the provides an opportunity to citizen who has not paid full
UNESCO World Heritage Site list in July 2016, taxes in the past to come forward and declare the
becoming the first "Mixed Heritage" site of India. It was undisclosed income and pay tax, surcharge and penalty
included in the UNESCO Man and the Biospere totaling in all to forty-five percent of such undisclosed
Programme. income declared.
70. Solar Impulse-2 aircraft completed its round 75. The position of India in the Sustainable
the world journey in July 2016 at which one of Development Goal Index declared in 2016 is
the following places? (a) 110th (b) 115th
th
(a) Cairo (b) Aden (c) 120 (d) 130th
(c) Muscat (d) Abu Dhabi Ans. (a) : The position of India in the Sustainable
Ans. (d) : Solar Impulse-2 aircraft completed its first Development Goal Index declared in 2016 is 110 out of
round-the world journey on 26th July 2016 after it 149 nations. In 2022, India was ranked 121 out of the
landed successfully in Abu Dhabi. The Solar Impulse-2 163 countries.
has become the first aircraft to circle the globe using 76. As of July 2016, UNESCO has enlisted how
solar energy. Hence, option (d) is correct answer. many world Heritage Sites in India?
71. Which one of the following countries has been (a) 37 (b) 36
declared by the W.H.O. as Malaria- free nation (c) 35 (d) 34
in September, 2016? Ans. (c) : At present (in 2022), there are 40 UNESCO
(a) Bangladesh (b) Sri Lanka World Heritage Sites in India, Among these 40 sites, 32
(c) Nepal (d) India are culture, seven are natural and one is mixed. When
th
Ans. (b) : Sri Lanka, on 5 September 2016 was question was asked, In the July 2016, there were 35
certified by WHO for having eliminated malaria, a life- world Heritage sites enlisted by UNESCO in India.
threatening disease which long affected the island Hence, option (c) is correct answer.
country. Sri Lanka was the second country in the WHO 77. The "Make- in India Conference" is scheduled
South-East Asia Region to eliminate malaria after to take place in December, 2016 at:
Maldives. (a) Chennai (b) Bhubaneshwar
72. In the Paralympic of Rio-2016, which of the (c) Kolkata (d) New Delhi
following Indian Players won Gold medial? Ans. (b) : Make -in India Conference in 2016 was
(a) Devendra Jhanjharia hosted by Bhubaneshwar.
(b) Parvez Rasool 78. Which of the following Congressmen moved in
(c) Mariyappan Thangavelu September 2016 a bill entitled "Pakistan State
(d) Deepa Karmakar Sponsor of Terrorism Designation Act (HR-
Ans. (c) : Tamil Nadu's Mariyappan Thangavelu, won 6069)"
a gold medal in the men's high jump F42 event at the (a) Ted Poe and Joe Barton
Rio 2016 Paralympics. He jumped 1.89 meters. He was (b) Joe Barton and Dana Rohrabacher
honoured with the Padma Shri and Arjuna award. He (c) Ed Royce and Sam Johnson
received the Major Dhyan Chand Khel Ratna, the (d) Ted Poe and Dana Rohrabacher
highest sports honor in India. Ans. (d) : In September 2016, 2 two congressmen Ted
73. In the final match of under 18 Asia Cup Poe and Dana Rohrabacher introduced a bill in US
Hockey Tournament held in September 2016, Congress calling for a declaration of Pakistan as a
Indian defeated: "State Sponsor of Terrorism".
(a) Bangladesh (b) China 79. On September 21, 2016 the President of the
(c) Oman (d) Pakistan United Nations General Assembly convened a
Ans. (a) : In the Final match of under 18 Asia Cup high-level meeting on health issue pertaining
Hockey Tournament held in September 2016, India to:
defeated Bangladesh by 5–4 in the final match. (a) Antimicrobial resistance (b) H.I.V
74. The Union Finance Minister Arun Jaitley (c) Ebola (d) Diabetes
announced in October, 2016 that under the Ans. (a) : On 21 September 2016, the President of UN
Income Declaration Scheme (IDS), 2016 the General Assembly convened an one-day high level
amount of declared black money till September meeting at the UN Headquarters in New York on
30, 2016 is about "Antimicrobial Resistance".
UP RO/ARO (Cancelled) Exam 2016 128 YCT
CLICK HERE FOR FREE MATERIAL

80. In which of the following divisions of Uttar Ans. (b) : Blue Nile River originates from Lake Tana
Pradesh the first Samajwadi Abhinav School in Ethiopia and merges with the smaller tributary White
was established? Nile at Khartoum. the capital of Sudan to from the Nile
(a) Allahabad (b) Aligarh River. Hence, option (b) is correct answer.
(c) Agra (d) Varanasi
86. Which one of the following is NOT correctly
Ans. (a) : In April 2016, Former Chief Minister of Uttar matched?
Pradesh, Akhilesh Yadav inaugurated the states first
Samajwadi Abhinav Vidyalaya in Dandupur, Shipbuilding Centre State
Allahabad. The Samajwadi Abhinav Vidyalaya was to (U.S.A)
be set up by the state government in 18 divisional A. Houston - Texas
headquarters. Hence, option (a) is correct answer. B. Sparrows Point - Maryland
81. Indian Army carried out a surgical strike C. New Orleans - Louisiana
against terrorists in P.O.K. on:
(a) September 27, 2016 D. Camden - Florida
(b) September 28, 2016 Ans. (d) : The correct matche are :
(c) September 29, 2016 Shipbuilding Centre State
(d) September 30, 2016 (U.S.A.)
Ans. (b) : In September 2016, the Indian Army Houston – Texas
launched surgical strikes against terrorist camps in Sparrows Point – Maryland
Pakistan Occupied Kashmir (P.O.K.). The strike on New Orleans – Louisiana
September 28, 2016, was in response to an attack by
Pakistan-based terrorists on an army base in Kashmir's Camden – New Jersey
Uri on September 18, 2016 in which 19 Soldier's were 87. The correct sequence of the following seas from
killed. Hence, option (b) is correct answer. West to East is :
82. In August 2016, China developed a system (a) Black Sea, Mediterranean Sea, Caspian Sea,
nick-named "Micius" which is: Aral Sea
(a) an ancient Chinese musical system. (b) Caspian Sea, Mediterranean Sea, Black Sea,
(b) a quantum satellite communication system. Aral Sea
(c) a computer game. (c) Mediterranean Sea, Black Sea, Caspian Sea,
(d) none of the above. Aral Sea
Ans. (b) : "Micius" is the first quantum communication (d) Black Sea, Mediterranean Sean Aral Sea,
satellite launched by China in 2016. Caspian Sea
83. For the first time in the history of Paralympics, Ans. (c) : The correct sequence of seas from West to
two Indians stood on the podium together in East is- Mediterranean Sea, Black Sea, Caspian Sea,
Rio Paralympics 2016 after participating in the Aral Sea. Hence option (c) is correct answer.
event: 88. Match List-I with List-II and select the correct
(a) Shooting (b) Swimming answer from the code given below the lists:
(c) Javelin Throw (d) High Jump
List-I List-II
Ans. (d) : For the first time in the history of
Paralympics, Mariyappan Thangavelu and Varun Singh (Rivers) (Flowing into)
Bhati winning gold and bronze respectively, stood on A. Lena 1. Persian Gulf
the podium together in Rio Paralympics 2016 after B. Amur 2. Arctic Ocean
participating in the men high jump event. Hence, option
C. Tigris 3. Arabian Sea
(d) is correct answer.
84. "LEMOA" has been signed on August 29, 2016 D. Mahi 4. Pacific Ocean
between Codes:
(a) India and U.S.A. (b) India and France A B C D
(c) India and Israel (d) India and Russia (a) 2 4 1 3
Ans. (a) : India and the United States have signed the (b) 3 2 4 1
Logistics Exchange Memorandum Of Agreement (c) 4 1 3 2
(LEMOA) in Washington DC, USA on 29th August, (d) 1 3 2 4
2016. LEMOA is a facilitating agreement that
establishes basic terms, conditions, and procedures for Ans. (a) : The correct match are :
reciprocal provisions of logistic support, supplies, and (List-I) (List-II)
services between the armed forces of India and the (Rivers) (Flowing into)
United States. Lena – Arctic Ocean
85. The Blue Nile river originates from Amur – Pacific Ocean
(a) Lake Victoria (b) Lake Tana Tigris – Persian Gulf
(c) Lake Edward (d) Lake Albert Mahi – Arabian Sea
UP RO/ARO (Cancelled) Exam 2016 129 YCT
CLICK HERE FOR FREE MATERIAL

89. In their book "Introducing Cultural Note : As per the report of India State of Forest Report
Geography", J.E. Spencer and W.L. Thomas 2021, in terms of forest cover as percentage of total
have divided the world into how many Cultural geographical area, the top five states are Mizoram
Worlds? (84.53%), Arunachal Pradesh (79.33%), Meghalaya
(a) 9 (b) 10 (76.00%), Manipur (74.34%) and Nagaland (73.90%).
(c) 11 (d) 12 94. The correct sequence in descending order of
Ans. (c) : In their book "Introducing Cultural the following states in terms of the proved coal
Geography". J.E. Spencer and W.L. Thomas have reserves, as power the report of the Ministry of
divided the world into 11 cultural worlds. Hence, option Coal as on 1.4.2014 is:
(c) is correct answer. (a) Chhattisgarh, Odisha, Jharkhand, Madhya
90. What is the correct North-South sequence of Pradesh
the following civilizations? (b) Odisha, Chhattisgarh, Madhya Pradesh,
(a) Maya- Aztec- Muisca - Inca Jharkhand
(b) Maya - Muisca - Inca - Aztec (c) Jharkhand, Chhattisgarh, Odisha, Madhya
(c) Aztec - Muisca- Maya- Inca Pradesh
(d) Aztec- Maya- Muisca- Inca (d) Jharkhand, Odisha, Chhattisgarh, Madhya
Ans. (d) : The correct sequence of the following Pradesh
civilization from North to South is Aztec-Maya-Muisca- Ans. (d) : As on 1.4.2018 the details of state wise
Inca. The Aztec civilization was located in Central geological resources of coal are given as under.
Mexico. Maya Civilization was located in a region that State Proved Total Reserves
is today Southern Mexico. The Muisca civilization was Reserves (Million Tonnes)
located of the central highlands of present-day
Colombia Eastern Range. The Inca Empire was Jharkhand 45563 83152
centered in Peru, West Coast of South America. Hence, Odisha 37391 79295
option (d) is correct answer. Chhattisgarh 20428 57206
91. A gradual change in the manner of population West Bengal 14156 31667
growth occurring over a long period of time is Madhya Pradesh 11958 27987
known as
95. Consider the following statements:
(a) Demographic transition
Assertion (A): Natural resources are those that
(b) Population explosion
exist in a region and can be used also in future.
(c) Demographic dynamism
Reason (R): The development of an actual natural
(d) Demographic transformation
resource depends upon the technology and cost of
Ans. (a) : A gradual change in the manner of population production.
growth occurring over a long period of times is known Select the correct answer from the code given
as Demographic transition. A Demographic transition
below:
refers to a population cycle that begins with a fall in the
(a) Bothe (A) and (R) are true and (R) correctly
death rate, continues with a phase of rapid population
growth and concludes with a decline in the birth rate. explains (A).
(b) Both (A) and (R) are true but (R) does not
92. Kariba dam has been built on the river:
correctly explain (A).
(a) Kafue (b) Zambezi
(c) (A) is true, but (R) is false.
(c) Chambeshi (d) Kabompo
(d) (A) is false, but (R) is true.
Ans. (b) : The Kariba dam is a double curvature
Ans. (a) : Natural resources are those that exists in a
concrete arch dam in the Kariba gorge of the Zambezi
river basin between Zambia and Zimbabwe. region and can be used also in future, Hence assertion is
Construction of the dam began on 6th November 1956, correct. The development of an actual natural resource
and was completed in 1959. The structure is 128 m high depends upon the technology and cost of production.
and 579 m long. Hence, Reason is correct. Therefore, both assertion and
93. Which of the following States of India has the reason are correct and reason is the correct explanation
largest percentage of geographical area under for assertion.
forest as per the report of the Forest Survey of 96. Consider the following sources of power
India? generation:
(a) Manipur (b) Meghalaya 1. Hydro 2. Natural gas
(c) Mizoram (d) Nagaland 3. Nuclear 4. Diesel
Ans. (c) : When question was asked, Mizoram states Arrange them in descending order of their
has the largest percentage of the geographical area percentage in the total installed capacity of power
under forest. generation in India as on 30th September,2016:
UP RO/ARO (Cancelled) Exam 2016 130 YCT
CLICK HERE FOR FREE MATERIAL

(a) 1,3,4,2 (b) 3,2,1,4 Ans. (d) : The chronological order from the earliest to
(c) 1,2,3,4 (d) 2,1,4,3 the latest established National Park are:-
Ans. (c) : As per data of Power ministry percentage National Parks Year of Establishment
contribution of different sources in India's Power Jim Corbett 1936
production is as follows- Kanha 1955
Coal- 51.9% Kaziranga 1974
Solar- 14.6% Silent Valley 1984
Wind- 10.2%  1 1 1 1
Hydro- 12.7% 101. 1 + 11 + 111 + 1111  is equal to
 2 2 2 2
Natural Gas- 6.1%
1
Nuclear- 1.7% (a) 1236 (b) 1234
Diesel- .1% 2
1
97. Which one of the following is not a biotic (c) 1235 (d) 1238
resource? 2
(a) Fresh water (b) Coal 1 1 1 1
Ans. (a) : 1 + 11 + 111 + 1111
(c) Petroleum (d) Fish 2 2 2 2
Ans. (a) : Biotic and abiotic factors make up the 1 1 1 1
Ecosystem. Biotic factors are living things within an = 1 + 11 + 111 + 1111 +  + + + 
2 2 2 2
ecosystem; such as plants, animals, and bacteria, while
= 1234 + 2 = 1236
abiotic are non-living compounds; such as water, soil
and atmosphere. Coal and Petroleum are formed by 102. The missing letters in order in the following
biotic components which is vegetation and marine letter series are ab_ _ a_ _ b_ _ cb
animals. Fresh water is not a biotic resource. (a) bcbcbb (b) abcabc
(c) cbbcab (d) accacc
98. Which of the following has not been
categorized as forest under the National Forest Ans. (c) : a b c b /a b c b / a b c b
Policy (1952)? Hence option (c) is true.
(a) National Forests (b) National Parks 103. The missing number in the following is
(c) Protected Forests (d) Village Forests
Ans. (b) : National Forest Policy (1952) proposed the
categorization of forests on a functional basis into
protected forests, National forests and village forests.
The policy aimed at bringing one-third of the total land
area with 65% in hilly areas and 25% in plains under (a) 29 (b) 39
the forest cover. National Parks are not categorized as (c) 37 (d) 49
forest under the National Forest Policy 1952. Ans. (b) : Series is as follows:
99. According to the Indian State of Forest Report, 3×2–1=5
2015 released by the Ministry of Environment, 5×2–2=8
what is the percentage of total geographical 8 × 2 – 3 = 13
area of India under forest and tree cover? 13 × 2 – 4 = 22
(a) 22.48 (b) 23.00 22 × 2 – 5 = 39
(c) 24.16 (d) 24.48 104. Shailesh travels 4km towards north. He turns
Ans. (c) : According to the India State of Forest Report to the left and travels 6km. Then he turns right
(ISFR) 2015, the total forest and tree cover were 79.42 and travels 4km. How far is he from his
million hectares, which was 24.16% of the total starting point?
geographical area. As forest Survey Report 2021, the (a) 5 km (b) 6 km
total forest and tree cover in India is 8,09,537 square (c) 14 km (d) 10km
kilometers, which is 24.62% of the geographical area of Ans. (d) :
the country, Hence, option (c) is correct answer.
100. Arrange the following National Parks in their
chronological order from the earliest to the
latest and select the correct answer from the
from the codes given below :
1 Silent Valley 2 Jim Corbett
3 Kaziranga 4 Kanha
Codes:
Hence required distance
(a) 1,3,2,4 (b) 4,2,1,3
(c) 3,2,4,2 (d) 2,4,3,1 AB = 82 + 62 = 100 = 10km
UP RO/ARO (Cancelled) Exam 2016 131 YCT
CLICK HERE FOR FREE MATERIAL

105. ABZY : CDWX :: EFVU: ? 109. In throwing fair dice twice, the probability that
(a) GHST (b) HGST the sum of two numbers obtained is six will be
(c) GHTS (d) HGTS 5 1
(a) (b)
Ans. (a) : 36 6
Just as 1 1
(c) (d)
4 2
Ans. (a) : Total number of possibilities of throwing two
dice simultaneously = 6 × 6 = 36
Possible conditions of getting sum 6
= (1, 5), (2, 4), (3, 3), (5, 1), (4, 2)
Similarly, 5
Required probability =
36
110. If each side of a cube is reduced by 10% its
value will decrease by :
(a) 27.1% (b) 17.2%
(c) 30.0% (d) 10.0%
106. if SISTER is coded as 212345 and ARC as 658, Ans. (a) : Percentage in volume
then ARREST will be coded as xy + yz + zx xyz
(a) 655243 (b) 655423 =x+y+z+ +
( )
2
100 100
(c) 655324 (d) 655325
100 + 100 + 100 1000
Ans. (b) : = – 10 – 10 – 10 + +
S I S T E R A R C 100 100 × 100
= – 30 + 3 – 01
↓ ↓ ↓ ↓ ↓ ↓ ↓ ↓ ↓ = – 27.1% = 27.1% decrease
2 1 2 3 4 5 6 5 8 111. In a row P is in the 10th position from the left
Hence ARREST will be coded as 655423. and Q is in the 12th position from the right. If
position of P and Q are interchanged, then P
107. If 3×4 = 1609 becomes 18th from the left. The number of
5×6 = 3625 persons in the row other than P and Q is:
and 1×2 = 401 (a) 29 (b) 27
(c) 26 (d) 24
then 4×7 is equal to
(a) 1649 (b) 74 Ans. (b) :
(c) 148 (d) 4916
Ans. (d) : Here the square of the given digits is written
in reverse order. After changing position of P and Q, P becomes 18th
from the left.
Total number of persons = 18 + 12 –1 = 29
Total number of persons other than P and Q = 27
112. A's brother is B,B's wife is C, E is the daughter
of A, D is the son of B, F is the brother of E and
G is the sister of D. A is what of G?
Similarly 4 × 7 → 72 42 = 4916 (a) Father (b) Brother
(c) Uncle (d) Cousin
108. Find the missing number in the following chart
Ans. (c) :
2 3 5
14 12 8
23 - 38
(a) 25 (b) 27
(c) 30 (d) 32 A is the uncle/aunt of G. Because gender of A is not
Ans. (c) : In the given chart the number series is known.
running in the following way– 113. A boy was asked to divide one half of a number
2+1 = 3, 3+2= 5, 5+3= 8, 8+4 = 12, by 6, the other half by 10 and add the results.
In order to save time, he divided the number by
12+5 = 17, 17+6 = 23, 23+7 = 30 , 1
30+8 = 38 ( 6 + 10 ) . From actual results his obtained
2
Hence, the missing number is 30. result will be:
UP RO/ARO (Cancelled) Exam 2016 132 YCT
CLICK HERE FOR FREE MATERIAL

(a) less Ans. (d) : Among all the states of India, Uttar Pradesh
(b) more is at 4th rank in terms of area. Rajasthan is the largest
(c) same state in India in terms of area followed by Madhya
(d) depends on the number Pradesh, Maharashtra, Uttar Pradesh.
Ans. (a) : Let number be x.
118. In which year Bharatendu Natya Academy was
x x 8x 2x
+ = = = 0.13x established?
2 × 6 2 × 10 60 15 (a) 1975 (b) 1963
In order to save time,
(c) 1985 (d) 1995
x x
= = 0.125x Ans. (a) : In memory of Bhartendu Harish Chandra,
1
( 6 + 10 ) 8 father of Modern Hindi Drama, Department of Culture,
2 Government of Uttar Pradesh established Bhartendu
Hence obtained result will be less than initial results.
Natya Kendra in August 1975 at Lucknow, Uttar
114. A sum of money is divided between A,B,C and Pradesh. Hence option (a) is correct answer.
D in such a way that A and B may get as 3:5,B
and C as 6: 7 and C and D as 8:9. The ratio in 119. Which one of the following places is known for
which A and D get is the Imdadkhani Gharana of sitar and
(a) 36 : 35 (b) 19 : 21 Surbahar music?
(c) 33 : 37 (d) 16 : 35 (a) Agra (b) Lucknow
Ans. (d) : A : B = 3 : 5 (c) Etawah (d) Varanasi
A 3 Ans. (c) : The Etawh Gharana is a North Indian School
=
B 5 of Sitar and Surbahar music and named after a small
B 6 town close to Agra where Imad Khan (1848-1920)
=
C 7 lived. It is also known as Imdadkhani Gharana in the
C 8 honor of its founder, Imdad Khan.
=
D 9 120. Which one of the following is not correctly
on multiplying, matched?
A B C 3 6 8 Crop Variety
× × = × ×
B C D 5 7 9 A. Paddy - Govind
A 16 B. Wheat - Shusk Samrat
= = 16 : 35
D 35 C. Arhar - Paras
115. The sum of two numbers is 10 and the D. Pea - Prakash
difference of their squares is 40. which is the
larger of the two numbers? Ans. (b) : Shusk Samrat is not a variety of wheat. It is a
(a) 5 (b) 7 variety of Rice developed for drought prone rain fed
(c) 9 (d) 11 upland and lowland areas of eastern Uttar Pradesh,
Ans. (B) : Let the two numbers are a and b. Bihar and Chhattisgarh. Hence, option (b) is not
a + b = 10 ______(1) correctly matched. Rest of given options are correctly
a2 – b2 = 40 matched.
(a – b) (a + b) = 40 121. In which of the following districts 'Mar- soil' is
(a – b) = 4 ______(2) found"
Adding the equations, (a) Kanpur (b) Pratapgarh
2a = 14, 7+ b = 10 ⇒ b = 3 (c) Sitapur (d) Jhansi
a=7
Hence larger number = 7 Ans. (d) : Lal, Mar, Rakar and Parwa are the varities of
soil found in Bundelkhand. Mar Soil is found in Jhansi
116. The literacy rate in U.P. as per 2011 Census
India is district.
(a) 70.05% (b) 68.21% 122. Which one of the following is not correctly
(c) 67.68% (d) 65.41% matched?
Ans. (c) : As per Census 2011, the literacy rate of Uttar Place Product
Pradesh is 67.68%. Male and Female literacy rate of A. Lakhimpur - Ornamental wood craft
Uttar Pradesh are 77.28% and 57.18% respectively. Kheri
117. Amongst all the States of India, what is the B. Khurja - Ceramics
rank of U.P. area- wise ?
(a) First (b) Second C. Meerut - Sports goods
(c) Third (d) Fourth D. Jalesar -. Cast bells
UP RO/ARO (Cancelled) Exam 2016 133 YCT
CLICK HERE FOR FREE MATERIAL

Ans. (*) : The correct matched are : 127. Who among the following taught Hinustani
List-I (Place) List-II (Product) Classical Music to Shambhu Maharaj?
Lakhimpur Kheri – Ornamental wood craft (a) Natthu Khan
Khurja – Ceramics (b) Rahimuddin Khan
Meerut – Sports goods (c) Sujan Khan
Jalesar – Cast bells (d) Abdul Karim Khan
Note:- The Uttar Pradesh State Public Service Ans. (b) : Shambhu Maharaj was born in Lucknow
Commission removed this question because all given (1910) as Shambhunath Mishra. He was a Kathak artist
options are correctly matched. of the Lucknow Gharana. He learned Hindustani
123. In which of the following places, "Mudia Classical Music from Ustad Rahimuddin Khan.
Purnima Mela" is organized? 128. Who among the following is NOT a Param
(a) Bateshwar (b) Kannauj Veer Chakra Awardee?
(c) Govardhan (d) Sardhana (a) Sanjay Kumar
Ans. (c) : Mudia Purnima Mela is organized in (b) Manoj Kumar Pnadey
Govardhan, Mathura. It is organized at occasion of (c) Neeraj Kumar Singh
Guru Purnima. Hence, option (c) is correct answer. (d) Yogendra Singh Yadav
124. Match List-I with List-II and select your Ans. (c) : Param Vir Chakra is India's highest military
correct answer from the codes given below the decoration, which is given for displaying distinguished
lists. acts of valor during War. Sanjay Kumar (1999), Manoj
Kumar Pandey (1999), Yogendra Singh Yadav (1999)
List-I List-II
were awarded Param Vir Chakra.
(Districts) (Product)
A. Aligarh 1. Powerloom products 129. Who among the following was the first women
Chief minister of Uttar Pradesh?
B. Firozabad 2. Locks (a) Smt. Rajendra Kumari Bajpayee
C. Moradabad 3. Brass utensils (b) Smt. Sucheta Kripalani
D. Tanda 4. Bangles (c) Ms. Mayawati
Codes: (d) Smt. Sarojini Naidu
A B C D Ans. (b) : Sucheta Kripalani was an Indian women
(a) 1 2 3 4 freedom fighter and politician. She was India's first
(b) 2 4 3 1 female Chief Minister, serving as the head of the Uttar
(c) 2 3 4 1 Pradesh government from 1963 to 1967. Hence, option
(d) 4 3 2 1 (b) is correct answer.
Ans. (b) : The correct match are : 130. Consider the following statements:
List-I List-II Assertion (A) : Light Colored clothes are
preferred in summer.
(Districts) (Product)
Reason (R): Light colors are poor absorbers of
Aligarh – Locks
heat. Select correct answer from the code given
Firozabad – Bangles below:
Moradabad – Brass Utensils (a) Both (A) and (R) are correct and (R) correctly
Tanda – Powerloom products explains (A).
125. Who among the following was born in Agra? (b) Both (A) and (R) are correct but (R) does not
(a) Meer (b) Amir Khusro correctly explain (A).
(c) Firaq (d) Ghalib (c) (A) is true, but (R) is false.
Ans. (d) : Mirza Ghalib was born in Agra in the state of (d) (A) is false, but (R) is true.
Uttar Pradesh. He was a Poet. He was also known as Ans. (a) : Light colored clothes are preferred in summer
Mirza Asadullah Khan. He is one of the prominent because light colors are poor absorbers of heat. Light
Persian and Urdu language poet during decline of colors reflect heat which helps our body to keep cool.
Mughal Empire. He was court poet in Bahadur Shah Hence, option (a) both assertion and reason are correct
Zafar II court. Hence, option (d) is correct answer. and reason is the correct explanation for assertion.
126. In which of the following cities the State Lalit 131. Which of the following is correctly matched?
Kala Akademi, U.P. is located?
A. Thermoresistor - Electronic
(a) Lucknow (b) Varanasi Thermometer
(c) Mathura (d) Allahabad
B. Capacitor - Thermometer
Ans. (a) : State Lalit Kala Akademi, Uttar Pradesh is
located in Lucknow. It was established on 8th February, C. Bipolar junction - Rectifier
1962 under the Department of Culture, Government of Transistor
Uttar Pradesh. Hence, option (a) is correct answer. D. Junction Diode - Amplifier
UP RO/ARO (Cancelled) Exam 2016 134 YCT
CLICK HERE FOR FREE MATERIAL

Ans. (a) : A thermoresistor is a resistance thermometer, 136. Which one of the following is used in
or a resistor whose resistance is dependent on production of explosives?
temperature. Hence option (a) is correctly matched. (a) Glycerol (b) Methanol
Other all given options are incorrect. (c) Urea (d) Oxalic acid
132. Stratollite is the: Ans. (c) : Urea used to produce urea nitrate is a
(a) uppermost band of stratosphere powerful explosive material. It is frequently used by
(b) lowermost band of stratosphere terrorists. It was also used in the first World Trade
(c) high altitude airship positioned in the Center bombing in New York in 1993. Hence, option
stratosphere
(c) is correct.
(d) natural satellite of Saturn
Ans. (c) : Stratollite are high altitude balloon (airships) 137. Which one of the following is used for
similar to satellites, employed for remote sensing, manufacturing safety matches?
navigation and communication. Stratollite are positioned in (a) Di-phosphorus (b) Black phosphorus
the stratosphere. Hence, option (c) is correct answer. (c) Red phosphorus (d) Scarlet phosphorus
133. Consider the following statements about Ans. (c) : Red Phosphorus are used for manufacturing
nuclear fusion reactors : safety matches. Phosphorus is of the two types- Red and
1. They work on the principle of fission of White, White phosphorus is highly reactive and catches
heavy nuclei. fire at normal room temperature but red phosphorus
2. They usually have Tokamak design. does not show such nature.
3. They operate at very high temperature. 138. Which one of the following is NOT correctly
Of these matched?
(a) only 1 and 3 are correct.
(b) only 1 and 2 are correct. A. Autoimmune - Rheumatoid arthritis
(c) only 2 and 3 are correct. disease
(d) all the three 1, 2 and 3 are correct. B. Genetic disease - Tuberculosis
Ans. (c) : Fission and Fusion are nuclear reactions that C. Scabies - Mite
produce energy. Fission is the splitting of heavy D. Mental disease - Schizophrenia
unstable nucleus into two smaller nuclei, while fusion
reactions involve combination of two light nuclei together. Ans. (b) : Tuberculosis (TB) is not a Genetic disease. It
Hence, statement (1) is incorrect. Nuclear fusion reactor is caused by a bacterium called Mycobacterium
have Tokomak design. It operates at very high Tuberculosis. The bacteria usually attack the lungs, but
temperature. Hence statements (2) & (3) are correct. TB bacteria can attack any part of the body such as the
134. Which one of the following is an important kidney, spine and brain. Rest all the given options are
component of a transistor? correct.
(a) Arsenic (b) Germanium 139. Poisonous fangs of a snake are modified form
(c) Osmium (d) Radium of:
Ans. (b) : Germanium semiconductor is an important (a) Mandibular teeth (b) Palatine teeth
component of a transistor. (c) Canine teeth (d) Maxillary teeth
135. Match List-I with List -II and select the correct Ans. (d) : Poisonous fangs of a snake are modified form
answer using the code given below the lists:
of maxillary teeth. The fangs are long pointed teeth.
List-I List-II Hence, option (d) is correct answer.
A. Compound present in 1. Lactic acid 140. Consider the following statements about
Bones mosquitoes:
B. Acid present in vinegar 2. Ethylene 1. Only males suck blood
C. Souring of milk 3. Acetic acid 2. Only females suck blood
D. Ripening of fruits 4. Calcium 3. Females suck plant juices
phosphate 4. Males have bigger wings than females.
Codes: 5. Females have bigger wings than males
A B C D Of these
(a) 1 2 3 4 (a) Only 2 and 3 are correct.
(b) 2 1 4 3 (b) Only 1 and 2 are correct.
(c) 3 4 2 1 (c) Only 3 and 5 are correct.
(d) 4 3 1 2 (d) Only 2 and 5 are correct.
Ans. (d) : List-I List-II Ans. (d) : Mosquitoes are infamous for spreading
Compound present in Bones – Calcium Phosphate diseases such as malaria, dengue, brain fever, cerebral
Acid present in vinegar – Acetic Acid malaria and Filariasis. Only female mosquitoes suck
Souring of Milk – Lactic acid blood. Females have bigger wings than males. Hence,
Ripening of fruits – Ethylene option (d) is correct answer.
UP RO/ARO (Cancelled) Exam 2016 135 YCT
CLICK HERE FOR FREE MATERIAL

Gòej ØeosMe meceer#ee DeefOekeâejer/meneÙekeâ meceer#ee DeefOekeâejer (Øeer.) hejer#ee, 2016


meeceevÙe efnvoer
nue ØeMve-he$e (efvejmle) hejer#ee efleefLe : 27 veJecyej, 2016
(DeOÙeeÙeJeej efJeMues<eCe meefnle JÙeeKÙee)
efJeueesce JÙeeKÙee- ‘menÙeesieer’ keâe meJe&Lee GheÙegkeäle efJehejerleeLe&keâ Meyo
‘ØeefleÙeesieer’ neslee nw peyeefkeâ ‘Øeefleketâue’ keâe efJehejerleeLe&keâ Meyo
1. ‘megueYe’ keâe GheÙegkeäle efJehejerleeLe&keâ Meyo nw ‘Devegketâue’ neslee nw~
(a) og<ØeehÙe (b) DeueyOe 7. ‘peesÌ[’ keâe GheÙegkeäle efJehejerleeLe&keâ Meyo nw
(c) DeØeehÙe (d) ogue&Ye (a) iegCee (b) Yeeie
Gòej (d) (c) IešeJe (d) yeekeâer
JÙeeKÙee- ‘megueYe’ keâe GheÙegkeäle efJehejerleeLe&keâ Meyo ‘ogue&Ye’ nw peyeefkeâ Gòej (c)
og<ØeehÙe DeLeJee DeØeehÙe keâe efJeueesce Meyo ØeehÙe nw Deewj ‘ueyOe’ keâe JÙeeKÙee- ‘peesÌ[’ keâe GheÙegkeäle efJehejerleeLe&keâ Meyo ‘IešeJe’ neslee nw
efJehejerleeLe&keâ Meyo ‘DeueyOe’ DeLeJee ‘Øeoòe’ neslee nw~ peyeefkeâ ‘iegCee’ keâe efJehejerleeLe&keâ Meyo ‘Yeeie’ neslee nw~ Mes<e efJekeâuhe
2. ‘oerIee&Ùeg’ keâe efJeueesce nesiee Demebiele nw~
(a) efÛejeÙeg (b) DeuheeÙeg
8. ‘le=<Cee’ keâe GheÙegkeäle efJehejerleeLe&keâ Meyo nw
(a) efJele=<Cee (b) efvemhe=n
(c) veMJej (d) #eefCekeâ
(c) meblegef<š (d) efJele=hle
Gòej (b) Gòej (a)
JÙeeKÙee- ‘oerIee&Ùeg’ DeLeJee ‘efÛejeÙeg’ keâe efJeueesce Meyo ‘DeuheeÙeg’ nw JÙeeKÙee- ‘le=<Cee’ keâe GheÙegkeäle efJehejerleeLe&keâ Meyo ‘efJele=<Cee’ nw
peyeefkeâ ‘veMJej’ DeLeJee ‘#eefCekeâ’ keâe efJehejerleeLe&keâ Meyo ‘MeeMJele’ nw~ peyeefkeâ ‘meblegef<š’ keâe efJehejerleeLe&keâ Meyo ‘Demeblegef<š’ nw~ Mes<e efJekeâuhe
3. ‘JÙeef<š’ keâe efJehejerleeLe&keâ Meyo nw Demebiele nw~
(a) meceeme (b) meceJesle 9. ‘efveef<eæ’ keâe meJe&Lee GheÙegkeäle efJehejerleeLe&keâ Meyo nw
(c) meceef<š (d) mecemle (a) efveefMÛele (b) efJeefnle
Gòej (c) (c) GefÛele (d) efveefce&le
JÙeeKÙee- ‘JÙeef<š’ keâe efJehejerleeLe&keâ Meyo ‘meceef<š’ nw peyeefkeâ Gòej (b)
‘meceeme’ keâe efJeueesce Meyo ‘JÙeeme’, ‘meceJesle’ keâe efJeueesce Meyo JÙeeKÙee- ‘efveef<eæ’ keâe meJe&Lee GheÙegkeäle efJehejerleeLe&keâ Meyo ‘efJeefnle’
nw, peyeefkeâ ‘efveefMÛele’ keâe efJeueesce ‘DeefveefMÛele’, ‘GefÛele’ keâe efJeueesce
‘DemeceJesle’ neslee nw~
‘DevegefÛele’ Deewj ‘efveefce&le’ keâe efJeueesce ‘Deefveefce&le’ neslee nw~
4. ‘GhecesÙe’ keâe meJe&Lee GheÙegkeäle efJehejerleeLe&keâ Meyo nw 10. ‘hees<ekeâ’ keâe GheÙegkeäle efJehejerleeLe&keâ Meyo nesiee
(a) DeveghecesÙe (b) DelegueveerÙe (a) Ûet<eCe (b) Devehees<ekeâ
(c) Deveghece (d) Devegheefcele (c) Mees<ekeâ (d) #eerCekeâejer
Gòej (a) Gòej (c)
JÙeeKÙee- ‘GhecesÙe’ keâe meJe&Lee GheÙegkeäle efJehejerleeLe&keâ Meyo ‘DeveghecesÙe’ JÙeeKÙee- ‘hees<ekeâ’ keâe GheÙegkeäle efJehejerleeLe&keâ Meyo ‘Mees<ekeâ’ neslee nw~
nw~ DelegueveerÙe keâe leelheÙe& nw efpemekeâer leguevee ve keâer pee mekesâ peyeefkeâ Dele: ØeMveiele efJekeâuhe (c) melÙe nw~
Deveghece keâe DeLe& nw efpemekeâer keâesF& Ghecee ve nes~ heÙee&ÙeJeeÛeer
5. ‘Yetieesue’ keâe efJehejerleeLe&keâ Meyo nw
11. heÙee&ÙeJeeÛeer keâer Âef<š mes Skeâ Jeie& Megæ nw
(a) Fefleneme (b) Debleefj#e
(a) efmebn, hebÛeeveve, veenj, ce=ieeefj
(c) OeÇgJeieesue (d) Keieesue (b) metÙe&, jefJe, efovekeâj, lejCeer
Gòej (d) (c) Deefive, heeJekeâ, Deveue, efheMegve
JÙeeKÙee- ‘Yetieesue’ keâe efJehejerleeLe&keâ Meyo ‘Keieesue’ neslee nw~ Mes<e (d) censMe, jcesMe, YetlesMe, meleerMe
efJekeâuhe Demebiele nw~ Gòej (a)
6. ‘menÙeesieer’ keâe meJe&Lee GheÙegkeäle efJehejerleeLe&keâ Meyo nw JÙeeKÙee- efmebn, kesâmejer, veenj, Jevejepe, ce=iesvõ, ce=ieeefj, hebÛeeveve
(a) ØeefleÙeesieer (b) ØeefleÉvÉer Deeefo Mesj kesâ heÙee&ÙeJeeÛeer Meyo nQ~ jefJe, efovekeâj, efoJeekeâj metÙe& kesâ
(c) ØeeflejesOeer (d) Øeefleketâue heÙee&ÙeJeeÛeer nQ peyeefkeâ lejCeer veewkeâe keâe heÙee&ÙeJeeÛeer nw~ Fmeer Øekeâej
Gòej (a) Deefive, heeJekeâ Deewj Deveue Deeie kesâ heÙee&ÙeJeeÛeer Meyo nQ~ Fme Øekeâej
ØeMveiele efJekeâuhe (a) melÙe nw~
UP RO/ARO (Pre) Hindi 2016 136 YCT
CLICK HERE FOR FREE MATERIAL

12. efvecveefueefKele ceW mes Skeâ ‘helLej’ keâe heÙee&ÙeJeeÛeer veneR nw: 18. heÙee&ÙeJeeÛeer keâer Âef<š mes Skeâ Jeie& Megæ nw
(a) Øemlej (b) Gheue (a) Fvõ, jleerMe, menœee#e, ceIeJee
(c) heMece (d) heenve (b) Iej, efvekesâleve, DeeÙeleve, efveueÙe
Gòej (c) (c) Ùecegvee, keâeefuevoer, YeevegleveÙee, peeÖJeer
(d) Jem$e, heeškeâ, Decyej, Ûeerj
JÙeeKÙee- helLej kesâ heÙee&ÙeJeeÛeer Meyo nQ- Øemlej, Gheue, heenve, hee<eeCe, Gòej (b)
DeMce Deewj efMeuee~ Fme Øekeâej ØeMveiele efJekeâuhe (c) DemelÙe nw~ JÙeeKÙee- Iej, efvekesâleve, DeeÙeleve, efveueÙe, ie=n kesâ heÙee&ÙeJeeÛeer nQ~
13. Skeâ ‘Ùecegvee’ keâe heÙee&ÙeJeeÛeer veneR nw ceOeJee Deewj menm$ee#e Fvõ kesâ heÙee&ÙeJeeÛeer nQ »eyeefkeâ jleerMe keâeceosJe
(a) jefJepee (b) levetpee keâe heÙee&Ùe nw~ keâeefuevoer Deewj YeevegleveÙee Ùecegvee kesâ heÙee&ÙeJeeÛeer nQ
(c) metÙe&pee (d) Deke&âpee peyeefkeâ peeÖJeer iebiee keâe heÙee&ÙeJeeÛeer nw~ peyeefkeâ Decyej Je Ûeerj Jem$e
Gòej (b) kesâ heÙee&Ùe nQ Deewj heeškeâ efkeâveeje keâe heÙee&Ùe nw~ Fme Øekeâej ØeMveiele
JÙeeKÙee- Ùecegvee kesâ heÙee&ÙeJeeÛeer jefJepee, metÙe&pee, metÙe&meglee, Deke&âpee efJekeâuhe (b) melÙe nw~
ke=â<Cee Deewj keâeefuevoer nw peyeefkeâ levetpee heg$eer keâe heÙee&ÙeJeeÛeer Meyo nw~ 19. efvecveefueefKele ceW mes Skeâ ‘mecegõ’ keâe heÙee&ÙeJeeÛeer Meyo
14. efvecveefueefKele MeyoeW ceW mes Skeâ Meyo ‘Mebkeâj’ keâe
veneR nw :
(a) peueefOe (b) peueOeece
heÙee&ÙeJeeÛeer veneR nw :
(c) peueefveefOe (d) peueeefOehe
(a) ueueeše#e (b) iebieeOej Gòej (d)
(c) ef$eueesÛeve (d) MeMeOej
JÙeeKÙee- peueefOe, peueOeece, peueefveefOe Deewj meeiej mecegõ kesâ
Gòej (d) heÙee&ÙeJeeÛeer Meyo nQ peyeefkeâ peueeefOehe Jewefokeâ osJelee Je®Ce keâe
JÙeeKÙee- Mebkeâj kesâ heÙee&ÙeJeeÛeer Meyo ueueeše#e, iebieeOej, ef$eueesÛeve, heÙee&ÙeJeeÛeer Meyo nw~ mecegõ keâer DevÙe heÙee&ÙeJeeÛeer Meyo efvecveefueefKele
censMe, efMeJe, ceneosJe Deeefo nQ peyeefkeâ MeMeOej, Ûevõcee leLee keâhetj nQ~ meeiej, efmevOeg, GoefOe, veoerMe, JeejerMe, veerjefveefOe, jlveekeâj,
oesveeW keâe heÙee&ÙeJeeÛeer Meyo nw~ Fme Øekeâej ØeMveiele efJekeâuhe (d) DeCe&Je, heÙeesefveefOe, leesÙeefveefOe, meefjlheefle Deeefo~
‘MeMeOej’ Mebkeâj keâe heÙee&ÙeJeeÛeer veneR nw~ 20. efvecveefueefKele ceW mes Skeâ ‘veoer’ keâe heÙee&ÙeJeeÛeer Meyo
15. efvecveefueefKele ceW mes Skeâ Meyo ‘Deece’ keâe heÙee&ÙeJeeÛeer veneR nw :
veneR nw : (a) efvecveiee (b) ef$eheLeiee
(c) ketâuebkeâ<ee (d) ketâueJeleer
(a) DeeflemeewjYe (b) menkeâej
Gòej (b)
(c) Debyeg (d) jmeeue
Gòej (c) JÙeeKÙee- efvecveiee, ketâuebkeâ<ee, ketâueJeleer, meefjlee, veefoÙee, Deeheiee,
uenjer Deewj heÙeefmJeveer veoer kesâ heÙee&ÙeJeeÛeer Meyo nQ peyeefkeâ ef$eheLeiee
JÙeeKÙee- DeeflemeewjYe, menkeâej, jmeeue, Deeceü, heâueßes<" Deewj iebiee keâe heÙee&ÙeJeeÛeer Meyo nw~
Dece=leheâue Deece kesâ heÙee&ÙeJeeÛeer Meyo nQ peyeefkeâ Debyeg peue keâe
heÙee&ÙeJeeÛeer Meyo nw~ JeekeäÙe SJeb Jele&veer mecyevOeer DeMegefæÙeeB
16. efvecveefueefKele ceW mes Skeâ ‘efJe<Ceg’ keâe heÙee&ÙeJeeÛeer Meyo 21. Skeâ keâer Jele&veer Megæ nw
veneR nw : (a) efKeJewÙee (b) vÙeewÚeJej
(c) Devevveeme (d) efvehex#e
(a) keâceuesMe (b) keâceueekeâevle
Gòej (c)
(c) keâceueeheefle (d) keâceueemeve
JÙeeKÙee- Devevveeme keâer Jele&veer Megæ nw~ Mes<e efvecveJele nw-
Gòej (d)
DeMegæ Megæ
JÙeeKÙee- keâceuesMe, keâceueekeâevle, keâceueeheefle, ßeerheefle, ue#ceerheefle efKeJewÙee KesJewÙee
Ûe›eâheeefCe, peveeo&ve, nefj Deewj veejeÙeCe, efJe<Ceg kesâ heÙee&ÙeJeeÛeer Meyo nQ vÙeewÚeJej vÙeesÚeJej
peyeefkeâ keâceueemeve, yeÇÿee keâe heÙee&ÙeJeeÛeer Meyo nw~ efvehex#e efvejhes#e
17. efvecveefueefKele ceW mes Skeâ ‘mJeCe&’ keâe heÙee&ÙeJeeÛeer Meyo 22. Skeâ keâer Jele&veer Megæ nw
veneR nw : (a) <ešdoMe&ve (b) Deveehesef#ele
(c) efkeâefue<š (d) heÙe&Jemeeve
(a) nsce (b) neškeâ
Gòej (d)
(c) efnjCÙee (d) keâueOeewle
JÙeeKÙee- heÙe&Jemeeve keâer Jele&veer Megæ nw~ Mes<e efvecveJele nw-
Gòej (c) DeMegæ Megæ
JÙeeKÙee- kebâÛeve, keâvekeâ, meesvee, nsce, neškeâ, efnjCÙe, keâueOeewle <ešdoMe&ve <e[oMe&ve
leecejme peele™he mJeCe& kesâ heÙee&ÙeJeeÛeer Meyo nQ~ Fme Øekeâej ØeMveiele Deveehesef#ele Devehesef#ele
efJekeâuhe (c) DemelÙe nw~ efkeâefue<š efkeäue<š
UP RO/ARO (Pre) Hindi 2016 137 YCT
CLICK HERE FOR FREE MATERIAL

23. Skeâ keâer Jele&veer Megæ nw 28. Skeâ JeekeäÙe Megæ nw


(a) GvevÙeve (b) GvveÙeve (a) Kesle peesleves kesâ heejcheefjkeâ Gheeoeve nue keâe mLeeve Deye
(c) GvÙeve (d) GvvÙeve šw^keäšj ves ues efueÙee nw
Gòej (b) (b) efvejhejeOeer kesâ efJe®æ ob[elcekeâ keâeÙe&Jeener keâe efkeâÙee peevee
JÙeeKÙee- ‘GvveÙeve’ keâer Jele&veer Megæ nw, Mes<e keâer Jele&veer DeMegæ nw~ ogYee&iÙehetCe& nw
GvveÙeve keâe leelheÙe& nw Thej keâer Deesj G"vee Ùee ues peevee~ (c) ßeerceleer js[d[er efpeme mlej keâer ve=lÙeebievee nw Gvekesâ heefle Gme
24. Skeâ keâer Jele&veer Megæ nw
mlej kesâ ve=lÙeebieve veneR nw
(d) efMe#ee ØeCeeueer peveesheÙeesieer nesveer ÛeeefnS
(a) DeeefJe<keâej (b) osJeeef<e&
Gòej (d)
(c) efveMeyo (d) peceelee
Gòej (a) JÙeeKÙee- ‘efMe#eeØeCeeueer peveesheÙeesieer nesveer ÛeeefnS’ Skeâ Megæ JeekeäÙe
nw~ efJekeâuhe (a) ceW ‘Gheeoeve’ Meyo keâe ØeÙeesie DeMegæ nw~ Gheeoeve keâe
JÙeeKÙee- ‘DeeefJe<keâej’ keâer Jele&veer Megæ nw~ Mes<e efvecveJele nw- leelheÙe& JÙeJenej ceW Deeves Jeeueer JemlegDeeW kesâ yeveeves keâer meece«eer mes nw~
DeMegæ Megæ efJekeâuhe (b) ceW ØeÙegkeäle ‘efvejhejeOeer’ kesâ mLeeve hej ‘efvejhejeOe’ Meyo
osJeeef<e& osJeef<e& ØeÙegkeäle nesvee ÛeeefnS peyeefkeâ efJekeâuhe (c) ceW ‘ve=lÙeebieve’ Meyo kesâ mLeeve hej
efveMeyo efve:Meyo ‘vele&keâ’ Meyo ØeÙegkeäle nesvee ÛeeefnS~ Dele: ØeMveiele efJekeâuhe (d) melÙe nw~
peceelee peeceelee 29. Skeâ JeekeäÙe Megæ nw
25. Skeâ JeekeäÙe Megæ nw (a) YeejleJe<e& kesâ Deleerle ceW keâF& efveo&Ùeer Meemekeâ ngS nQ
(a) heJe&leerÙe ØeosMe ceW Øeele:keâeue keâe ÂMÙe yengle efÛeòeekeâ<e&keâ (b) Skeâef$ele YeerÌ[ peÙepeÙekeâej keâj jner nw
neslee nw (c) ieeBOeerpeer ÛejKee Ûeueeles Les
(b) ØelÙeskeâ Oece& kesâ efueS DeÛÚe meodYeeJe jKevee nceeje (d) Skeâ-Skeâ keâjkesâ ØelÙeskeâ Úe$e keâ#ee mes efvekeâue ieS
keâòe&JÙe nw Gòej (c)
(c) cesjs Jesleve keâe DeefOekeâebMe Yeeie yeÛÛeeW keâer heÌ{eF& ceW ner JÙeeKÙee- heüMveiele efJekeâuhe c ces ‘ieeBOeerpeer ÛejKee Ûeueeles Les’ JeekeäÙe
KeÛe& nes peelee nw Megæ nw~ efJekeâuhe (a) ces ‘efveo&Ùeer’ keâs mLeeve hej ‘efveo&Ùe’ Meyo heÇÙegkeäle
(d) Fme yeele keâe mhe<šerkeâjCe keâjvee DeeJeMÙekeâ nw nesvee ÛeeefnS efJekeâuhe (b) ces ‘Skeâef$ele’ Meyo keâe heÇÙeesie DevegefÛele nw
Gòej (a) peyeefkeâ efJekeâuhe (d) ceW ‘Skeâ-Skeâ’ Deewj ‘heÇlÙeskeâ’ oesveeW keâe Skeâ meeLe
JÙeeKÙee- ‘heJe&leerÙe ØeosMe ceW Øeele:keâeue keâe ÂMÙe yengle efÛeòeekeâ<e&keâ heÇÙeesie DevegefÛele nw~
neslee nw’ Skeâ Megæ JeekeäÙe nw~ efJekeâuhe (b) ceW ‘DeÛÚe’ kesâ meeLe 30. Skeâ JeekeäÙe Megæ nw
‘meodYeeJe’ Meyo ØeÙegkeäle ngDee nw pees efkeâ DeMegæ nQ efJekeâuhe (c) ceW (a) mJeeceer efJeJeskeâevebo keâe Yee<eCe megvekeâj Gheneme GÌ[eves
‘DeefOekeâebMe’ kesâ meeLe ‘Yeeie’ Meyo keâe ØeÙeesie DeMegæ nw Fmeer Øekeâej Jeeues DecejerefkeâÙeeW hej IeÌ[eW heeveer heÌ[ ieÙee~
efJekeâuhe (d) ceW ‘mhe<šerkeâjCe’ kesâ meeLe ‘keâjvee’ Meyo ØeÙegkeäle ngDee nw, (b) yeepeej keâe meehleeefnkeâ DeJekeâeMe meesceJeej keâes jnlee nw
pees efkeâ DeMegæ nw~ Fme Øekeâej ØeMveiele efJekeâuhe (a) melÙe nw~ (c) Jen ogKeer m$eer JewOeJÙelee keâe peerJeve efyelee jner nw
26. Skeâ keâer Jele&veer Megæ nw (d) ceQ meheefjJeej meeveefvole ntB
(a) jepÙe cenue (b) keâesceueebefieveer Gòej (a)
(c) efvejesie (d) De#eewefnCeer JÙeeKÙee- heÇMveiele efJekeâuhe (a) ceW heÇÙegkeäle JeekeäÙe Megæ nw peyeefkeâ
Gòej (d) efJekeâuhe (b) ces ‘DeJekeâeMe’ keâs mLeeve hej ‘yeboer’ Meyo heÇÙegkeäle nesvee
JÙeeKÙee- ‘De#eewefnCeer’ Meyo keâer Jele&veer Megæ nw~ Mes<e efvecveJele nw- ÛeeefnS, efJekeâuhe (c) ces ‘JewOeJÙelee keâs meeLe ‘m$eer’ keâe heÇÙeesie efvejLe&keâ
DeMegæ Megæ nw peyeefkeâ efJekeâuhe (d) ces ‘meeveeefvole’ keâs mLeeve hej ‘Deeveefvole’ Meyo
jepÙe cenue jepecenue heÇÙegkeäle nesvee ÛeeefnS~ Fme Øekeâej efJekeâuhe (a) Megæ JeekeäÙe nw, peyeefkeâ
keâesceueebefieveer keâesceueebieer DeeÙeesie ves Fmekeâe Gòej (b) ceevee nw~
efvejesie veerjesie Deveskeâ MeyoeW kesâ Skeâ Meyo
27. Skeâ JeekeäÙe Megæ nw
31. ‘ceefoje heerves keâe hÙeeuee’ kesâ efueS Skeâ Meyo nw
(a) Ùen lees DeÛÚe ngDee efkeâ ÛeesjeW keâe heoehe&Ce nesles ner ceQ
(a) ÛeMce (b) ÛeMekeâ (c) ÛeMkeâ (d) Ûe<ekeâ
peeie ieÙee Gòej (d)
(b) DeÛÚs uesKeve kesâ efueS Meyo mebÙece DeeJeMÙekeâ nw
(c) keâ#e ceW DemebKÙe pevemecetn GheefmLele Lee
JÙeeKÙee- ‘ceefoje heerves keâe hÙeeuee’ kesâ efueS Skeâ Meyo ‘Ûe<ekeâ’
(d) ‘jeceÛeefjle ceeveme’ leguemeer keâer meyemes megvojlece ke=âefle nw
ØeÙegkeäle neslee nw~ Ûe<ekeâ Skeâ efJeMes<e Øekeâej keâe hÙeeuee neslee nw efpememes
Gòej (b) ceefoje Ùee DevÙe õJeeW keâes efheÙee peelee nw~ FmeceW Dekeämej yeerÛe ceW Skeâ
lebie ieo&ve nesleer nw Deewj veerÛes Skeâ ÛeewÌ[e efnmmee neslee nw efpeme hej Fmes
JÙeeKÙee- ‘DeÛÚs uesKeve kesâ efueS Meyo mebÙece DeeJeMÙekeâ nw’ Ùen Skeâ efškeâeÙee peelee nw~
Megæ JeekeäÙe nw~ efJekeâuhe (a) ceW ‘heoehe&Ce’ kesâ meeLe ‘nesles ner’ DeMegæ 32. pees Œeer kesâ JeMeerYetle nw, kesâ efueS Skeâ Meyo nw
nw~ efJekeâuhe (c) ceW DemebKÙe’ kesâ meeLe ‘pevemecetn’ Meyo keâe ØeÙeesie (a) Œeer Øesceer (b) ŒewCe
DeMegæ nw peyeefkeâ efJekeâuhe (d) ceW ‘meyemes’ kesâ meeLe ‘megvojlece’ keâe (c) efm$eÙeesefÛele (d) ef$eÙeeJeMeer
ØeÙeesie DeMegæ nw~ Fme Øekeâej ØeMveiele efJekeâuhe (b) melÙe nw~ Gòej (b)
UP RO/ARO (Pre) Hindi 2016 138 YCT
CLICK HERE FOR FREE MATERIAL

JÙeeKÙee- ‘pees Œeer kesâ JeMeerYetle nw’ kesâ efueS Skeâ Meyo ‘ŒewCe’ ØeÙegkeäle JÙeeKÙee- ‘yegjs GösMÙe kesâ meeLe keâer ieÙeer ieghle ceb$eCee’ kesâ efueS Skeâ
neslee nw~ efm$eÙeesefÛele keâe leelheÙe& efŒeÙeeW kesâ meceeve iegCe Jeeues mes nw Meyo ‘ogjefYemebefOe’ ØeÙegkeäle efkeâÙee peelee nw~ efkeâmeer keâes mebkeâš ceW [eueves
peyeefkeâ pees m$eer mes Øesce keâjlee nw Gmes ‘Œeer Øesceer’ keâne peelee nw~ kesâ efueS yeveeF& peeves Jeeueer Ùeespevee keâes ‘ogjefYeÙeespeve’ keâne peelee nw
33. ‘meJeeue-peJeeye’, ‘yenme-ngppele’ Ùee ‘efoS ieS Gòej hej peyeefkeâ efpemekeâe Deefle›eâceCe Ùee GuuebIeve menpe ceW ve nes mekesâ kesâ efueS
Gòej’ kesâ efueS Skeâ Meyo nw Skeâ Meyo ‘ogjefle›eâce’ ØeÙegkeäle efkeâÙee peelee nw~
(a) Gòejehes#eer (b) GòejCe 39. ‘Ùegæ keâer FÛÚe jKeves Jeeuee’ kesâ efueS Skeâ Meyo nw
(c) ØelÙegòej (d) Gòejesòej (a) Ùeesæe (b) Ùegæjle (c) ÙegæJeerj (d) ÙegÙeglmeg
Gòej (c) Gòej (d)
JÙeeKÙee- ‘meJeeue-peJeeye’, ‘yenme-ngppele’ Ùee ‘efoS ieÙes Gòej hej JÙeeKÙee- ‘‘Ùegæ keâer FÛÚe jKeves Jeeues’ kesâ efueS Skeâ Meyo ‘ÙegÙeglmeg’
Gòej’ kesâ efueS Skeâ Meyo ‘ØelÙegòej’ ØeÙegkeäle efkeâÙee peelee nw~ ‘GòejCe’ ØeÙegkeäle efkeâÙee peelee nw peyeefkeâ ‘pees Ùegæ ceW ueiee ngDee nes’ kesâ efueS
keâe leelheÙe& lewjkeâj Ùee veeJe Deeefo kesâ Éeje peueeMeÙe heej keâjvee neslee Skeâ Meyo ‘Ùegæjle’ ØeÙegkeäle efkeâÙee peelee nw~
nw peyeefkeâ ‘Gòejesòej keâe leelheÙe& nw ›eâceeiele ™he mes Deeies yeÌ{les peevee~ 40. ‘lewjves Ùee heej nesves keâe FÛÚgkeâ’ kesâ efueS Skeâ Meyo nw
34. ‘Jen keâvÙee efpemekesâ meeLe efJeJeen keâe JeÛeve efoÙee ieÙee (a) lewjekeâ (b) efleleer<eg& (c) leejkeâ (d) leejCekeâ
nw’- kesâ efueS Skeâ Meyo nw Gòej (b)
(a) Jeeioòee (b) Jeeioeve JÙeeKÙee- ‘lewjves Ùee heej keâjves keâe FÛÚgkeâ’ kesâ efueS Skeâ Meyo
(c) Jeeiyeæ (d) JeeefiJeodiOe
‘efleleer<eg&’ ØeÙegkeäle neslee nw peyeefkeâ ‘lewjekeâ’ Gmekesâ efueS ØeÙegkeäle neslee nw
Gòej (a) pees Ketye DeÛÚer lejn lewjvee peevelee nes Deewj ‘leejkeâ’ keâe leelheÙe& ‘DeeBKe
JÙeeKÙee- ‘Jen keâvÙee efpemekesâ meeLe efJeJeen keâe JeÛeve efoÙee ieÙee nw’ keâer hegleueer’ DeLeJee ‘leejeW mes Yeje ngDee’ nw~
kesâ efueS Skeâ Meyo ‘Jeeioòee’ ØeÙegkeäle efkeâÙee peelee nw~ ‘Jeeioeve’ keâe
DeLe& nw keâvÙee kesâ efJeJeen keâer yeele efkeâmeer mes hekeäkeâer keâjvee Deewj Gmes lelmece SJeb leodYeJe
keâvÙeeoeve keâe JeÛeve osvee peyeefkeâ ‘Jeeiyeæ’ keâe leelheÙe& ‘ceewve’ DeLeJee 41. leodYeJe ‘Ûeyetleje’ keâe lelmece ™he nw
‘JeÛeveyeæ’ nesves mes nw~ (a) ÛeJe&lej (b) ÛeJeg&leje
35. ‘Ssmeer peerefJekeâe efpemekeâe kegâÚ "erkeâ-ef"keâevee ve nes’ kesâ (c) ÛeeJe&tleje (d) ÛelJeeue
efueS Skeâ Meyo nw Gòej (d)
(a) DeekeâeMe kegâmegce (b) DeekeâeMeJe=efòe JÙeeKÙee- ‘Ûeyetleje’ Skeâ leodYeJe Meyo nw efpemekeâe lelmece Meyo
(c) DeekeâeMe meefueue (d) DeekeâeMeheâue ‘ÛelJeeue’ DeLeJee ‘ÛelJej’ neslee nw~ Fme Øekeâej efJekeâuhe (d) melÙe nw~
Gòej (b) 42. efvecveefueefKele ceW mes Skeâ leodYeJe Meyo nw
JÙeeKÙee- ‘Ssmeer peerefJekeâe efpemekeâe kegâÚ "erkeâ-ef"keâevee ve nes’ kesâ efueS (a) hetbpeer (b) veÙeve (c) cetKe& (d) Mekegâve
Skeâ Meyo ‘DeekeâeMeJe=efòe’ ØeÙegkeäle neslee nw~ ‘DeekeâeMe heâue’ Meyo Gòej (a)
‘mebleeve’ DeLeJee ‘mebleefle’ kesâ efueS ØeÙegkeäle neslee nw peyeefkeâ ‘Devenesveer’ JÙeeKÙee-‘hetbpeer’ Skeâ leodYeJe Meyo nw efpemekeâe lelmece Meyo heg_pe
Ùee DemebYeJe yeele kesâ efueS ‘DeekeâeMe kegâmegce’ Meyo ØeÙegkeäle neslee nw~
neslee nw~ Dele: ØeMveiele efJekeâuhe (a) melÙe nw~
36. cejCeemevve DeJemLee Jeeuee kesâ efueS Skeâ Meyo nw
43. ‘yeWle’ keâe lelmece ™he nw
(a) cewcelÙe& (b) cegcet<eg&
(a) Jesvle (b) Jes$e (c) Jesv$e (d) JeWle=
(c) ce=leieeceer (d) efveefJe&keâejer
Gòej (b)
Gòej (b)
JÙeeKÙee- ‘cejCeemevve DeJemLee Jeeuee kesâ efueS Skeâ Meyo’ ‘cegcet<e&’ JÙeeKÙee- ‘yeWle’ Skeâ leodYeJe Meyo nw efpemekeâe lelmece Meyo ‘Jes$e’ neslee
ØeÙegkeäle neslee nw~ ce=leieeceer keâe leelheÙe& nw pees ce=lÙeg keâes Øeehle keâj Ûegkeâe nw~ Fme Øekeâej ØeMveiele efJekeâuhe (b) melÙe nw~
nes peyeefkeâ efveefJe&keâejer keâe DeeMeÙe efJekeâej mes jefnle nesves mes nw~ 44. ‘Ûevee’ keâe lelmece ™he nw
37. efpemes Deheveer peien mes Deueie keâj efoÙee ieÙee nes, kesâ efueS (a) ÛeCekeâ (b) ÛeCee (c) ÛeeCekeâ (d) ÛeCeekeâ
Skeâ Meyo nw Gòej (a)
(a) efJemLeeefhele (b) DeJemLeeefhele JÙeeKÙee- Ûevee Skeâ leodYeJe Meyo nw efpemekeâe lelmece Meyo ‘ÛeCekeâ’
(c) mebmLeeefhele (d) mebmLeeefpele neslee nw Dele: ØeMveiele efJekeâuhe (a) melÙe nw~ Mes<e efJekeâuhe DeMegæ nw~
Gòej (a) 45. ‘Tve’ keâe lelmece ™he nw
JÙeeKÙee- ‘efpemes Deheveer peien mes Deueie keâj efoÙee ieÙee nes’ kesâ efueS Skeâ (a) TvÙe (b) TCÙe& (c) TjCe (d) TCe&
Meyo ‘efJemLeeefhele’ ØeÙegkeäle neslee nw peyeefkeâ ‘mebmLeeefhele’ keâe DeeMeÙe ‘mebefÛele’ Gòej (d)
Ùee ‘Fkeâªe keâjvee’ DeLeJee ‘ÛeueeÙee Ùee ØeÛeefuele’ efkeâÙee ngDee mes nw~ JÙeeKÙee- ‘Tve’ Skeâ leodYeJe Meyo nw efpemekeâe lelmece Meyo ‘TCe&’
38. ‘yegjs GösMÙe mes keâer ieÙeer ieghle ceb$eCee’ kesâ efueS Skeâ Meyo nw neslee nw~ Dele: ØeMveiele efJekeâuhe (d) melÙe nw~ Mes<e efJekeâuhe DeMegæ nw~
(a) ogjefle›eâce (b) ogjefOeiece 46. ‘Leeueer’ keâe lelmece ™he nw
(c) ogjefYemebefOe (d) ogjefYeÙeespeve (a) mLeeueer (b) Leeefuekeâe (c) LÙeeueer (d) Leeefue
Gòej (c) Gòej (a)
UP RO/ARO (Pre) Hindi 2016 139 YCT
CLICK HERE FOR FREE MATERIAL

JÙeeKÙee- ‘Leeueer’ Skeâ leodYeJe Meyo nw efpemekeâe lelmece Meyo ‘mLeeueer’ 54. ‘FefvõÙe’ keâe efJeMes<eCe Meyo nw
neslee nw~ Dele: ØeMveiele efJekeâuhe (a) melÙe nw~ (a) FvoÇerÙe (b) Fefvõkeâ (c) Ssefvõ (d) SsefvõÙe
47. ‘peecegve’ keâe lelmece ™he nw Gòej (d)
(a) Ùeecegve (b) ÙeecegCe (c) pebyeg (d) peebyetCe JÙeeKÙee- ‘FefvõÙe’ Meyo keâe efJeMes<eCe heo ‘SsefvõÙe’ nw~ peyeefkeâ DevÙe
Gòej (c) Meyo Demebiele nw~
JÙeeKÙee- ‘peecegve’ Skeâ leodYeJe Meyo nw efpemekeâe lelmece Meyo ‘pebyeg’ 55. Skeâ JeekeäÙe ceW efJeMes<eCe keâe ØeÙeesie veneR ngDee nw
neslee nw~ Dele: ØeMveiele efJekeâuhe (c) melÙe nw~ (a) Jen efJeÅeeLeea nw (b) Jen ueÌ[keâe efJeÅeeLeea nw
48. Skeâ lelmece Meyo nw (c) Jen Øeyegæ efJeÅeeLeea nw (d) Jen heefjßeceer Yeer nw
(a) Deveefnle (b) efJeveleer Gòej (a)
(c) [eefkeâveer (d) Dešejer JÙeeKÙee- JeekeäÙeebMe ‘Jen efJeÅeeLeea nw’ ceW efJeMes<eCe keâe ØeÙeesie veneR
Gòej (c) ngDee nw peyeefkeâ DevÙe efJekeâuheeW ceW ›eâceMe: Jen, Øeyegæ Deewj heefjßeceer
JÙeeKÙee- ‘Deveefnle’, ‘efJeveleer’ Deewj Dešejer leodYeJe Meyo nw peyeefkeâ efJeMes<eCe ØeÙegkeäle ngDee nw~
‘[eefkeâveer’ Skeâ lelmece Meyo nw efpemekeâe leodYeJe ‘[eFve’ neslee nw~ Dele: 56. ‘Yekeäle keâer keâ®Ce hegkeâej megvekeâj Yekeäle-Jelmeue YeieJeeved
ØeMveiele efJekeâuhe (c) melÙe nw~ oÙeeõ& nes G"s’- JeekeäÙe ceW ØeÙegkeäle efJeMes<ÙeeW keâer Âef° mes
49. Skeâ leodYeJe Meyo nw keâewve mee Ùegice Megæ nw?
(a) hebpej (b) hebÛeeueer (c) hebef[le (d) hebpeerjer (a) Yekeäle leLee YeieJeeved (b) keâ®Ce leLee oÙeeõ&
Gòej (d) (c) Yekeäle-Jelmeue leLee YeieJeeved (d) hegkeâej leLee YeieJeeved
JÙeeKÙee- hebpej, hebÛeeueer Deewj hebef[le lelmece Meyo nw peyeefkeâ ‘hebpeerjer’ Gòej (d)
leodYeJe Meyo nw~ Dele: ØeMveiele efJekeâuhe (d) melÙe nw~ JÙeeKÙee- JeekeäÙeebMe ‘Yekeäle keâer keâ®Ce hegkeâej megvekeâj Yekeäle-Jelmeue
50. Skeâ leodYeJe Meyo nw YeieJeeve oÙeeõ& nes G"s’ ceW hegkeâej leLee YeieJeeve efJeMes<Ùe heo nw peyeefkeâ
(a) Dešue (b) Deelegj (c) DeefleefLe (d) Deefpej keâ®Ce Deewj Yekeäle-Jelmeue efJeMes<eCe heo nw~
Gòej (a) 57. Yeesues yeeuekeâ ves ›etâj [eketâ kesâ keâ"esj ùoÙe ceW keâesceue
JÙeeKÙee- ‘Deelegj, DeefleefLe Deewj Deefpej lelmece Meyo nw peyeefkeâ YeeJevee peiee oer’- JeekeäÙe ceW efkeâleves efJeMes<Ùe nQ?
‘Dešue’ Skeâ leodYeJe Meyo nw~ Dele: ØeMveiele efJekeâuhe (a) melÙe nw~ (a) Ûeej (b) leerve (c) oes (d) Ún
Gòej (a)
efJeMes<Ùe SJeb efJeMes<eCe JÙeeKÙee- JeekeäÙeKeC[ ‘Yeesues yeeuekeâ ves ›etâj [eketâ kesâ keâ"esj ùoÙe ceW
51. Skeâ efJeMes<eCe Meyo veneR nw keâesceue YeeJevee peiee oer’ ceW Ûeej efJeMes<Ùe heo yeeuekeâ, [eketâ, ùoÙe
(a) uepeeruee (b) ueeÌ[uee Deewj YeeJevee nw peyeefkeâ efJeMes<eCe heo Yeer Ûeej nQ- Yeesues, ›etâj, keâ"esj
(c) ueebÚve (d) ueehelee Deewj keâesceue~
Gòej (c) 58. efvecveefueefKele ceW mes Skeâ efJeMes<eCe Meyo veneR nw :
JÙeeKÙee- meb%ee DeLeJee meJe&veece keâer efJeMes<elee yeleeves Jeeues Meyo (a) Jejoeve (b) Jejo (c) JejoeÙekeâ (d) JejCeerÙe
efJeMes<eCe keânueeles nQ Deewj efJeMes<eCe kesâ ØeÙeesie mes efpeme meb%ee DeLeJee Gòej (a)
meJe&veece keâe iegCe DeLeJee Oece& Øekeâš neslee nw, efJeMes<Ùe keânueelee nw~ JÙeeKÙee-Jejo, JejoeÙekeâ Deewj JejCeerÙe efJeMes<eCe Meyo nw peyeefkeâ
ØeMveiele mevoYe& ceW uepeeruee, uee[uee Deewj ueehelee efJeMes<eCe nw peyeefkeâ Jejoeve efJeMes<Ùe heo nw~ Fme Øekeâej ØeMveiele efJekeâuhe (a) melÙe nw~
ueebÚve efJeMes<Ùe nw~
59. efvecveefueefKele ceW mes Skeâ efJeMes<eCe Meyo nw :
52. ‘Ùegæ osKekeâj DeMeeskeâ keâe keâ"esj ùoÙe ceesce pewmee
(a) ueeIeJe (b) cenòJe (c) ueIeglee (d) cenled
efheIeue ieÙee’- JeekeäÙe ceW efJeMes<Ùe nw Gòej (d)
(a) keâ"esj (b) DeMeeskeâ
JÙeeKÙee- ueeIeJe, cenòJe Deewj ueIeglee efJeMes<Ùe heo nQ peyeefkeâ cenled
(c) ùoÙe (d) ceesce
Gòej (c) efJeMes<eCe Meyo nw~ Fme Øekeâej ØeMveiele efJekeâuhe (d) melÙe nw~
60. efvecveefueefKele ceW mes Skeâ efJeMes<eCe Meyo veneR nw :
JÙeeKÙee- ‘Ùegæ osKekeâj DeMeeskeâ keâe keâ"esj ùoÙe ceesce pewmee efheIeue
ieÙee’, JeekeäÙeebMe ceW ùoÙe efJeMes<Ùe nw peyeefkeâ keâ"esj efJeMes<eCe nw~ (a) yeng™efheÙee (b) yeeletveer
(c) yeeoeceer (d) yeebkeâe
53. efvecveefueefKele MeyoeW ceW mes Skeâ efJeMes<eCe veneR nw :
Gòej (*)
(a) ßeJÙe (b) meJe&
(c) ieJe& (d) YeJÙe JÙeeKÙee- efoÙes ieÙes MeyoeW ceW ‘yeng™efheÙee’ leLee ‘yeebkeâe’ oesveeW Meyo
Gòej (c) efJeMes<eCe nw peyeefkeâ DeeÙeesie ves Fmekeâe Gòej efJekeâuhe (a) ceevee nw~
JÙeeKÙee- ßeJÙe, meJe& Deewj YeJÙe efJeMes<eCe heo nw peyeefkeâ ieJe& efJeMes<Ùe yeng™efheÙee Deewj yeebkeâe Meyo efJeMes<eCe Deewj efJeMes<Ùe oesveeW ™heeW ceW
heo nw~ ieJe& keâe efJeMes<eCe heo ieJeeauee neslee nw~ ØeÙegòeâ nesles nQ~

UP RO/ARO (Pre) Hindi 2016 140 YCT


CLICK HERE FOR FREE MATERIAL

UPPSC RO-ARO (Mains) Exam-2014


GENERAL STUDIES
Solved Paper
1. The ivory scale is found in Harappan context :- (a) Balban (b) Kaiqubad
(a) Kalibangan (b) Lothal (c) Jalaluddin Khilji (d) None of the above
(c) Dholavira (d) Banawali Ans. (a) : Ghiyasuddin Balban called himself 'helper of
Ans. (b) : In Harappan context the ivory scale is found Caliph' in mosque inscription of Garhmukteshwar.
in Lothal. Lothal was ancient Indus valley civilisation 7. Whose disciple was Khwaja Moinuddin Chisti?
site located in Gujarat. Lothal was discovered by (a) Abdul Qadir Jilani
Archaeological Survey of India (ASI) in 1954. Lothal (b) Khwaja Abu Yusuf
has the earliest known dock. (c) Khwaja Usman Harooni
2. Champa was capital of which mahajanapada? (d) Khwaja Maudood
(a) Magadha (b) Vajji Ans. (c) : Khwaja Moinuddin Chisti was disciple of
(c) Kosala (d) Anga Khwaja Usman Harooni, Khwaja Moinuddin Chisti was
Ans. (d) : Champa was capital of Anga. Anguttar a persian sunni muslim preacher who settled in India in
Nikaya, a Buddhist scripture mentions 16 Mahajanpada early 13th - century, he promulgated the famous Chisti
in India at the beginning of 6th century BCE. Capital of order of sunni mysticism. Khwaja Usman Harooni was
Magadha, Vajji and Kosala were Girivraja/Rajagriha, a sufi saint of Islam, who visited India during the rule of
Vaishali and Shravasti/Ayodhya respectively. Sultan Iltutmish.
3. Where was second Buddhist council took 8. Which one of the following hindi composition
place? was written first ?
(a) Rajgriha (b) Vaishali (a) Indravati (b) Padmavati
(c) Patliputra (d) Kashi (Varanasi) (c) Madhumalati (d) Mrigavati
Ans. (b) : Second Buddhist Council took place in Ans. (d) : Out of given hindi compositions Mrigavati
Vaishali. I was under the patronage of Kalasoka and was written first. It was written in 1501 by a Persian
was presided over by Sabakami. It was held in 383 BC. writer Kutuban in Awadhi.
There were six Buddhist Council in total. First council Note:-
was held in 483 BC, third in 250 BC, fourth in 72 AD, Indravati - K. Nur Muhammad (1743)
fifth in 1871 and sixth in 1954. Padmavat - Malik Muhammad Jayasi
4. Chandragupta Maurya defeated Seleucus in (1540)
which year? Madhumalati - Mir Sayyid Manjhan Sattari
(a) 317 BC (b) 315 BC (1545)
(c) 305 BC (d) 300 BC 9. Who among the following was first woman
Ans. (c) : Chandragupta Maurya defeated Seleucus in president of Indian National Congress?
during the Seleucid-Mauryan War which took place (a) Sarojini Naidu (b) Annie Besant
between 305 to 303 BCE. The war ended in 303 BCE (c) N. Sengupta (d) None of the above
by a peace treaty. Seleucus offered a Macedonian Ans. (b) : Annie Besant was the first woman president of
princes for marriage to Chandragupta and several the Indian National Congress, who presided over the 1917
territories of Kamboja, Gandhar and Balochistan. session of Congress at Calcutta. In 1916, along with
5. Which of the following cities is not mentioned Lokmanya Tilak, she launched the Home Rule League.
in Rabatak inscription of Kanishka? She founded the Central Hindu College at 1898 in
(a) Shrawasti (b) Kaushambi Varanasi. She joined the Theosophical Society in 1889.
(c) Pattiputra (d) Champa 10. Who among the following was the first
Ans. (a) : In Rabatak inscription of Kanishka following president of Indian National Congress?
cities are mentioned - Saket, Champa, Pataliputra, (a) A.O. Hume (b) S.N. Banerjee
Ujjain and Kaushambi. Shrawasti was not mentioned in (c) W.C. Bonnerjee (d) Dadabhai Naoroji
Rabatak inscription of Kanishka. Kanishka was the ruler Ans. (c) : First President of Indian National Congress
of Kushana empire and was the great grandson of was W.C. Bonnerjee. INC party was established in
Kujula Kadphises I, the founder of the empire. 1885. First session was held at Bombay. It was started
Kanishka's Rabatak inscription was found in 1993 at by a retired British civil servant A.O. Hume.
Rabatak in Afghanistan. 11. Who among the following was chairman of
6. Who among the following sultans called drafting committee?
himself as Caliph's Assistant' on the walls of (a) A.K. Azad (b) B.R. Ambedkar
Garhmukteshwar Mosque? (c) J.L. Nehru (d) Dr. Rajendra Prasad
UP RO/ARO (Mains) Exam 2014 141 YCT
CLICK HERE FOR FREE MATERIAL

Ans. (b) : The Drafting Committee of the Constitution of Ans. (d) : The correct matched as -
India was chaired by Dr. B.R. Ambedkar. Other 6 members List - I List - II
of the committee were : K.M. Munshi, Muhammad (President) (Place of session of
Saadulla, Alladi Krishnaswamy lyer, Gopal Swami
Ayyangar, N. Madhava Rao (He replaced B.L. Mitter who Indian National
resigned due to ill-health), T.T. Krishnamachari (He Congres)
replaced D.P. Khatan who died in 1948). Abul Kalam Azad Ramgarh, 1940
12. 'Indian Home Rule Society' was established in - Sarojini Naidu Kanpur, 1925
(a) 1900 (b) 1901 Motilal Nehru Amritsar, 1919
(c) 1902 (d) 1905 Dr. Rajendera Prasad Bombay, 1934
Ans. (d) : 'Indian' Home Rule Society' was established 16. Who gave the idea of a separate muslim state in
in 1905 in London by freedom fighter Shyamji Krishna India for the first time?
Verma. It sought to promote the cause of self - rule in (a) Agha Khan (b) M.A. Jinanah
British India. Bhikaji Cama, Dadabhai Naoroji and S.R.
Rana also supported the society. (c) Liaquat Ali Khan (d) Mohammad Iqbal
13. Who among the following was the only jewish Ans. (d) : Mohammad Iqbal gave the idea of a separate
viceroy in India muslim state in India for the first time. Iqbal was
(a) Lord Curzon (b) Lord Canning elected president of the muslim league in 1930 at its
(c) Lord Irwin (d) Lord Reading session in Allahabad. Iqbal outlined a vision of an
independent state for muslim - majority provinces in
Ans. (d) : Lord Reading was the only Jewish viceroy in north western India in 1930 session of Muslim League.
India. Lord Chelmsford was succeeded by Lord Reading in
1921 as Governor - General and Viceroy of India. His 17. In the context of Interim Government (1946) of
tenure was from 1921-25. The Non- Cooperation India, who among the following was holding
Movement reached its climax and was withdrawn during the protfolio of 'Finance'?
his tenure, in 1922. Other event that happened during his (a) Fazlul Haq (b) Khizar Hayat Khan
time are - Mopla rebellion (1921), simultaneous (c) Liaquat Ali khan (d) Ghazanfar Ali Khan
examinations of civil services in London and Delhi (1923), Ans. (c) : In the context of the Interim Government
start of Vishwa Bharti University (1922), foundation of
(1946) of India, Liaquat Ali Khan was holding the
Communist Party of India (1921, formation of Swaraj
Party (1923), Kakori Train Action (1925). portfolio of 'Finance'. Interim Government was headed
by Jawahar Lal Nehru.
14. When did the British Government called the
Indian leaders for first Round Table 18. Who wrote the book 'India Wins Freedom'?
Conference? (a) A.K. Azad (b) M.K. Gandhi
(a) 1931 (b) 1929 (c) M.A. Ansari (d) Sarojini Naidu
(c) 1930 (d) 1932 Ans. (a) : Maulana Abul Kalam Azad wrote the book
Ans. (c) : First Round Table Conference was called in 'India Wins Freedom'. He was independent India's first
1930 by British Government. In response to the Education Minister. The book offers a glimpse of what
inadequacy of the Simon Report, the Labour Party really happened along the path to independence and
Government, which came to power under Ramsay provide insight into the Indian Freedom struggle.
MacDonald in 1929, decided to hold a series of Round 19. Which of the following is not correctly
Table Conference in London. The first one convened
from 12 November 1930 to 19 January 1931, the second matched?
conference in September 1931, and third round table (Hot spring) (Indian State)
conference in November, 1932. (a) Lasundra – Rajasthan
15. Match list-I to list-II and choose the correct (b) Akloli – Maharashtra
option from below : (c) Manikaran – Himachal Pradesh
List - I List - II (d) Sohna – Haryana
(President) (Place of session of Ans. (a) : Lasundra hot spring is situated in Gujarat.
Indian National Akloli hotspring is in Maharashtra. Manikaran hot spring
Congres) is in Himachal Pradesh. Sohna hotspring is in Haryana.
A. Abul Kalam Azad 1. Amritsar, 1919 20. Which of the following river cuts-across all the
B. Sarojini Naidu 2. Bombay, 1934 ranges of the Himalayas?
C. Motilal Nehru 3. Kanpur, 1925 (a) Ravi (b) Satluj
D. Dr. Rajendera 4. Ramgarh, 1940 (c) Beas (d) Chenab
Prasad Ans. (b) : The Satluj river originates from the
A B C D Rakshastal Lakes. It takes a north - westerly course up
(a) 1 3 2 4 to the Shipki La on the Tibet - Himachal Pradesh
(b) 2 4 3 1 boundary. Out of its total length of 1450 m, it flows for
(c) 3 2 4 1 1050 km in Indian territory. It cuts-across all the ranges
(d) 4 3 1 2 of the Himalayas.
UP RO/ARO (Mains) Exam 2014 142 YCT
CLICK HERE FOR FREE MATERIAL

21. Largest river of peninsular India is– Ans. (c) : Following is correctly matched.
(a) Narmada (b) Godavari Industry Centre
(c) Mahanadi (d) Krishna Cement – Porbandar
Ans. (b) : Godavari is the longest peninsular river in Petro Chemical – Nagothane
India. It originates in the western Maharashtra and Sugar – Bettiah
flows through Andhra Pradesh before draining into the Iron and Steel – Raurkela
Bay of Bengal. It is also know as 'Dakshin Ganga' and
26. Which of the following is not correctly
is second largest river after Ganga. Its total length is
matched?
1465 km.
Centre Energy
22. Match List - I to List - II and choose Correct (a) Patratu – Thermal
option given below:
(b) Jhakri – Hydro - Electric
List-I List-II
(c) Kalpakkam – Nuclear
A. Jute 1. Assam (d) Korba – Wind
B. Tea 2. Kerala Ans. (d) : Correctly matched :-
C. Rubber 3. West Bengal (a) Patratu – Thermal
D. Sugar Cane 4. Uttar Pradesh (b) Jhakri – Hydro - Electric
A B C D (c) Kalpakkam – Nuclear
(a) 3 1 2 4 (d) Korba – Coal
(b) 4 3 1 2 27. Jharia coal region is in which state?
(c) 2 4 3 1 (a) West Bengal (b) Odisha
(d) 1 2 4 4 (c) Jharkahand (d) Chattisgarh
Ans. (a): Correct matching - Ans. (c) : Jharia coal region is in Jharkhand state.
List - I List - II Jharia's economy is heavily dependent on the local coal
Jute - West Bengal field. The coal field lies in the Damodar River valley
Tea - Assam and produces bituminous coal suitable for coking. Most
of India's coal comes from Jharia.
Rubber - Kerala
28. What is the right sequence of energy source in
Sugarcane - Uttar Pradesh
India. -
23. Name two states between which an agreement (a) Thermal > Hydro > Nuclear > Wind
was signed on 20 June, 2015 on 'Kishau Dam (b) Thermal > Nuclear > Hydro > Wind
Project' -
(c) Hydro > Nuclear > Thermal > Wind
(a) Uttarakhand and Uttar Pradesh
(d) Nuclear > Hydro > Wind > Thermal
(b) Rajasthan and Uttarakhand
(c) Uttarakhand and Himachal Pradesh Ans. (a) : Out of the given options the correct sequence
(d) Himachal Pradesh and Rajasthan should be - Thermal energy > Hydro electric > Nuclear
energy > Wind energy.
Ans. (c) : Uttarakhand and Himachal Pradesh signed an Note :- As per Central Electricity Authority (CEA),
agreement on 'Kishau Dam Project'. It is on the Tons total installed capacity (As per 30.09.2022), this correct
River and its primary purpose is power generation and sequence are :-
downstream water supply. MoU was signed between
both government for joint ventures formation in 2015. Thermal energy > Hydro > Wind > Nuclear.
Approval was granted by CWC in 2018. Completion is 29. Which of these countries is not located in
slated for 2023. central - America?
24. In which industrial regions of India Shivkashi (a) Panama (b) Sierra leone
is located? (c) Honduras (d) Guatemala
(a) Chhota Nagpur region Ans. (b) : Countries located in Central America are
(b) Ahmedabad - Vadodara region Mexico, Guatemala, Honduras, Nicaragua, EI Salvador,
(c) Madurai - Coimbatore - Bengaluru region Costa Rica, Panama and Belize. Sierra - Leone is not in
(d) Kolkata - Hugli region Central - America but it is a country in West Africa.
Ans. (c) : Shivakashi is located in Madurai - 30. Which of the following mountain is in Italy?
Coimbatore - Bengaluru Industrial region. It is famous (a) Apennine (b) Pyrenees
for its fireworks and safety match industries. (c) Dinaric Alps (d) Jura
25. Which of the following is not correctly Ans. (a) : Apennine – Italy.
matched? Pyrenees – France and Spain
Industry Centre Dinaric Alps – Mountain range in Bosnia
(a) Cement – Porbandar and Herzegovina
(b) Petro Chemical – Nagthen Jura – Mountain range in Europe
(c) Sugar – Silvasa (France, Switzerland,
(d) Iron and Steel – Rourkela Germany)
UP RO/ARO (Mains) Exam 2014 143 YCT
CLICK HERE FOR FREE MATERIAL

31. Which of the following is the world's second 36. Which of the following continent had least
most deepest and longest lake? population density according to 2011 census?
(a) Tanganyika Lake (b) Baikal Lake (a) South America (b) North America
(c) Superior Lake (d) Great Bear Lake (c) Europe (d) Africa
Ans. (a) : Lake Tanganyika is the 2nd largest fresh water Ans. (a) : According to 2011 census.
Lake in the world and the second deepest Lake of any Continent Population Density
kind. Asia 4.27 Billion 86.7
Lake Length (Km) Deepness. Africa 1.7 Billion 32.7
Tanganyika 673 1470 m Europe 0.74 Billion 70
Baikal 636 1642 m North America 0.34 Billion 22.9
Superior 563 147 m South America 0.39 Billion 21.4
Great Bear 320 446 m Oceania 0.4 Billion 4.25
32. Which of the following is not correctly Antarctica 4490 0.00
matched. 37. What was the name of the campaign which was
(a) Lake Titicaca – Bolivia - Peru started and celebrated by Uttar Pradesh
(b) Lake Urmia – Iran Government on the occasion of population day
(c) Lake Reindeer – Russia from 11 to 24 July 2015.
(d) Lake Athabasca – Canada. (a) Population stability fortnight
Ans. (c) : Reindeer Lake is situated in Canada. Rest are (b) Happy family fortnight
correctly matched. (c) Population control fortnight
33. In which of the following rock system in India, (d) Population awareness fortnight
iron - ore reserves are found in plenty? Ans. (a) : On the eve of World Population Day (11th
(a) Vindhyan (b) Cuddapah July), Uttar Pradesh government had celebrated 11 to 24
(c) Gondwana (d) Dharwar July 2015 as Population Stability Fortnight.
Ans. (d) : The highest iron - ore reserves in Indian rock 38. According to 2001 census which state had
system is found in Dharwar rock systems. Jharkhand lowest child death rate.
has the highest reserves of haematite ore in India. (a) Goa (b) Kerala
Dharwar rocks are rich in iron ore, maganese, gold, (c) Maharashtra (d) Tamil Nadu
Zinc, lead, silver etc. They are named after the Dharwar Ans. (b) : According to 2001 census Kerala had lowest
region of Karnataka. child death rate. According to 2011 census lowest child
34. Which of the following is main characteristics death rate is recorded in Goa followed by Kerala.
of Indian Society– 39. Which state has highest density of population
1. Abundance of Indian Villages according to 2011 census
2. Various Religions (a) Bihar (b) Kerala
3. Various Castes (c) West Bengal (d) U.P.
4. Poverty Ans. (a) : Highest populated State in terms of density as
Choose the right option: - per census 2011 are :-
(a) 1, 2, 3 (b) 1, 3, 4 Bihar – 1106
(c) 3, 4 (d) 2, 4 West Bengal – 1028
Ans. (a) : Indian Society is a pluralistic society with Kerala – 860
Complex societal order. Salient features are : - UP – 829
1. Abundance of Indian Villages Haryana – 573
2. Various religion 40. In which of the following State sex ratio is
3. Various castes highest as per 2011 census.
4. Patriarchal Society. Census -
Poverty is not the main characteristics but it is one of (a) Tamil Nadu (b) Andhra Pradesh
the features of Indian society. (c) Chhattisgarh (d) Kerala
35. Which one of the following method will be Ans. (d) : State with highest sex ratio as per 2011
suitable for stable population structure census :-
(a) Increasing Birth rate and stable death rate Kerala – 1084
(b) Decreasing Birth rate and increasing Death Tamil Nadu – 996
rate Andhra Pradesh – 993
(c) Stable Birth and Death rate Manipur – 985
(d) Stable Birth rate and decreasing Death rate Chhattisgarh – 991
Ans. (c) : For stable population structure, there should 41. Which of the following State has highest
be stable Birth and Death rate. The distribution of population of Scheduled Caste according to
people in a population according to designated 2011 census :-
demographic traits such as age, sex, cast etc. is known (a) West Bengal (b) Uttar Pradesh
as population structure. (c) Punjab (d) Maharashtra
UP RO/ARO (Mains) Exam 2014 144 YCT
CLICK HERE FOR FREE MATERIAL

Ans. (b) : According to 2001 and 2011 census highest 46. How many members are there in Public
schedule caste population resides in Uttar Pradesh. Accounts Committee.
According to 2011 census SC population constitute Lok Sabha Rajya Sabha Total
20.69% of its total population. (a) 11 05 16
42. Which of the following statement is incorrect? (b) 15 07 22
(a) Some cities in Uttar Pradesh has status of (c) 10 05 15
Municipal Corporation. (d) 17 10 27
(b) Status of Municipal Corporation is given due Ans. (b) : The Committee on Public Accounts is
to political arrangement and reasons.
constituted by Parliament each year for examination of
(c) There are some principles on the basis of accounts showing the appropriation of sums granted by
which municipal corporations are created.
Parliament for expenditure of Government of India. It is
(d) U.P. has separate law for Municipal oldest Parliamentary committee and was first
Corporation.
constituted in 1921. It consists of 22 members, 15
Ans. (b) : The Constitution (74th Amendment) Act 1992 members are elected by Lok Sabha and 7 members of
added Part IX-A to the constitution of India which Rajya Sabha are associated with it.
provides details regarding organization of
Municipalities in India. According to the constitution 47. In which years first communist government
there are 3 types of Municipalities mainly based on size was made in Kerala.
of the urban area. These are - (a) 1953 (b) 1955
• Nagar Panchayats (for areas in transition from Rural (c) 1957 (d) 1959
to Urban area). Ans. (c) : The first democratically elected communist
• Municipal Councils (for smaller urban areas). government was formed in Kerala in 1957 under the
• Municipal Corporations (for larger urban areas) leadership of Namboodiripad. First General Secretary of
Therefore statement under option (b) is incorrect. the Communist Party of India was S.U. Ghate.
43. Arrange the following south Asian countries in 48. Who has final authority and say on the report
descending order in terms of Urbanisation of Comptroller and Auditor General of India.
percentage in their total population - (a) Presidend of India
1. Bangladesh (b) Supreme Court
2. India (c) Parliament
3. Pakistan (d) National Development Council
3. Sri Lanka Ans. (c) : According to Article 148 of Indian
(a) 2, 3, 1, 4 (b) 3, 2, 1, 4 constitution - There shall be a Comptroller and Auditor
(c) 2, 1, 4, 3 (d) 4, 2, 3, 1 - General of India who shall be appointed by the President
Ans. (b) : Urbanisation in SAARC countries :- by warrant under his hand and seal and shall only be
Country Urban population(%) removed from office in like manner and on the like
2014 2018 grounds as a Judge of the Supreme Court of India. The
Pakistan 36.2 36.7 final say on CAG report lies on the Parliament of India.
India 31.16 34 49. Legislative power to create new state in India
Bangladesh 28.4 36.6 lies in hand of
Sri Lanka 15.0 18.5 (a) Lok Sabha (b) Parliament
44. How many Fundamental Duties are there in (c) Political Parties (d) Union Government
Indian Constitution - Ans. (b) : Article 2 and 3 of the Indian constitution
(a) 7 (b) 8 provides for admission or establishment of new states
(c) 9 (d) 11 and formation of new states and alteration of areas,
Ans. (d) : There are total 11 Fundamental Duties in boundaries or name of existing states respectively.
Indian constitution which were added by 42nd Legislative power to create new state lies in hand and
Constitutional Amendment Act of 1976, on authority of the Parliament of India.
recommendation of Swarn Singh Committee. It is was
added in Part IV-A of the Indian constitution under 50. In how many days Rajya Sabha had to give its
Article 51A. acceptance on Money Bill?
45. TK Oommen has differentiated Urban families (a) 14 days (b) 18 days
on the basis of :- (c) 21 days (d) 30 days
(a) Source of Income and changing values Ans. (a) : Money Bill is defined under Article 110 of
(b) Political structure the Indian Constitution and it can be only introduce in
(c) Urban social environment and social ecology Lok Sabha. Rajya Sabha can delay or have to give its
(d) All of the above acceptance within 14 days.
Ans. (d) : Tharailath Koshy Oommen is an Indian 51. If any state has 42 Lok Sabha seats, How may
author and sociologist. He differentiated the urban seats for Sheduled Caste will be reserved.
families on the basis of source of Income, traditions, (a) 21 (b) 14
politics, urban environment etc. (c) 7 (d) 6
UP RO/ARO (Mains) Exam 2014 145 YCT
CLICK HERE FOR FREE MATERIAL

Ans. (c) : If any state has 42 Lok Sabha seats then 7 Ans. (c) : Among the given option groundnut is
seats for Scheduled Caste will be reserved. However in dicotyledonous leguminous oil seed Crop.
West Bengal 10 seats of Lok Sabha are reserved for 57. Yellow Revolution is related to.
Scheduled Castes. (a) Oil seeds (b) Fish
52. Which of the following has authority to (c) Rice (d) Fruits
establish state contingency fund Ans. (a) : Yellow Revolution is related to oil seeds
(a) Legislature of State (b) Parliament production such as mustard, groundnuts, etc. It was
(c) President (d) None of the above started in 1986 - 87. Sam Pitroda is credited for the
Ans. (a) : According to Article 267(2), the legislature of launch of Yellow Revolution in India.
a state has authority to establish state contingency fund 58. National Research Centre for Banana is
at the disposal of Governor of the state for the purpose situated :-
of meeting unforeseen expenditure. (a) Tiruchirappalli (b) Bhusawal
53. On Whose recommendation the work of a State (c) Saharanpur (d) Pune
Public Service Commission can be entrusted to Ans. (a) : ICAR National Research Centre for Banana
Union Public Service Commission. was established on 21st August 1993 at Tiruchirappalli,
(a) Chief Justice of India Tamil Nadu by ICAR, New Delhi with an aim to
(b) Prime Minister increase the production and productivity of banana and
(c) Speaker of Lok Sabha plantains through mission mode basic and strategic
research approaches.
(d) President of India
59. Which of the following states produces
Ans. (d) : According to Article 315(4) of the maximum pulses in India -
Constitution of India, on the request of the Governor of (a) UP (b) Bihar
the state and after approval from the President of India,
(c) MP (d) Punjab
Union Public Service Commission can be entrusted
with the duties of the Public Service Commission of Ans. (c) : Currently (2019-20) Pulse production in India
that particular state. in sequence -
1) Rajasthan
54. Consider the following statement regarding
Attorney - General of India- 2) Madhya Pradesh
(1) He is appointed by President of India 3) Maharashtra
(2) He should have same qualification as a judge 60. Leading bank of agriculture sector is
of the Supreme Court (a) Punjab National Bank
(3) He should be member of any of the two house (b) State Bank of India
of Parliament (c) NABARD
(4) He can be removed by impeachment process (d) Central Bank of India
in Parliament Ans. (c) : NABARD came into existence on 12 July
Code: 1982 by transferring the agriculture credit functions of
(a) 1 and 2 (b) 1 and 3 RBI and refinance functions of the then Agricultural
(c) 2,3 and 4 (d) 3 and 4 Refinance and Development Corporation (ARDC). It is
leading bank in India for agricultural sector.
Ans. (a) : According to Article 76(1) of the Indian
61. Indian Patent Act came into force in which
Constitution the President shall appoint a person who is year :-
qualified to be appointed a Judge of the Supreme Court
(a) 1970 (b) 1971
as Attorney General of India. He shall hold office
during the pleasure of the President and shall receive (c) 1972 (d) 1973
such remuneration as the President may determine. Ans. (a) : The Patent Act 1970 is the legislation that
Therefore only statements 1 and 2 are correct. governs the patents in India. It came into force in 20th
April 1972 by repealing the Indian Patent and Design
55. Kisan Credit Card Scheme for formers was Act 1911. There are three types of Patent -
started in - 1) Design Patents 2) Utility Patent 3) Plant Patent
(a) 1998 - 1999 (b) 1999 - 2000
62. When Special Economic Zone Policy was
(c) 2000 - 2001 (d) 2001 - 2002 Launched in India?
Ans. (a) : The Kisan Credit Card (KCC) Scheme was (a) April 2000 (b) April 2001
introduced in 1998 for issue of Kisan Credit Cards to (c) April 2002 (d) April 2003
farmers on the basis of their holdings for uniform
Ans. (a) : The Special Economic Zones (SEZs) Policy
adoption by the banks so that farmers may use them to was launched in April, 2000. The main objective of the
readily purchase agriculture inputs such as seeds, policy was to enhance foreign investment and provide
fertilizers, pesticides etc and draw cash for their an internationally competitive and hassle free
production needs. environment for exports.
56. Which of the following is dicotyledonous 63. World Trade Organisation (WTO) is situated
leguminous oil seed crop ? in?
(a) Sunflower (b) low pea (a) New York (b) Geneva
(c) Groundnut/Peanut (d) Maize (c) Washington D.C. (d) London
UP RO/ARO (Mains) Exam 2014 146 YCT
CLICK HERE FOR FREE MATERIAL

Ans. (b) : WTO was established on 1st January 1995. 70. Which of the following is a vector quantity?
Its headquarter is situated at Geneva (Switzerland). (a) Time (b) Speed
WTO is the only international organization dealing with (c) Distance (d) Displacement
the global rules of trade. Its main function is to ensure Ans. (d) : Vector quantity is that physical quantity
that trade flows as smoothly, predictably and freely as which has both a) Direction and b) Magnitude.
possible. WTO is successor to the General Agreement Examples of Vector quantity are - Linear momentum,
on Tariffs and Trade (GATT). Force, Displacement, Angular velocity, Acceleration.
64. Long term vision of the Department of Scalar quantity is that physical quantity which has only
commerce to make India a major player in the magnitude. Examples are. Time, Speed, Distance,
global trade is till the year - Volume, Density, Temperature.
(a) 2018 (b) 2019 71. A Weightless balloon is filled with 200 gm
(c) 2020 (d) 2021 water. What will be its apparent weight when
Ans. (c) : Commerce Ministry had long term vision to weighed in water?
make India biggest player in foreign trade till 2020. (a) zero (b) 100 gram
(c) 200 gram (d) 400 gram
65. What percentage increase in share of foreign
trade as part of GDP was seen from 2004 - 05 Ans. (a) : When the weightless balloon is filled with
to 2012 - 13? 200gms of water, its total weight becomes equal to weight
of water. When the water filled balloon is place inside the
(a) 30.6 (b) 40.6 water, it displaces equal amount of water i.e. 200gms,
(c) 45.6 (d) 50.6 resulting the apparent weight of balloon to be zero.
Ans. (c) : In 2004 - 05 trade to GDP ratio was 29.5% 72. Which is not a minor nutrients?
which was increased to 45.6% in 2012 - 13. In 2021 the (a) Iron (b) Zinc
trade to GDP ratio was 43.68% and in 2020 it was (c) Phosphorous (d) Manganese
37.81%.
Ans. (c) : Minor nutrients are those nutrients which are
66. India's external trade figure reached found in the soil and they are required by plants in tiny
approximately in 2011-12 - quantities. Examples of minor nutrients : - Cobalt (Co),
(a) ` 71000 crore (b) ` 81000 crore Iron (Fe), Copper (Cu), Zinc (Zn), Manganese (Mn),
(c) ` 91000 crore (d) ` 101000 crore Sodium (Na), Boron (B), Aluminium (Al). Major
Ans. (d) : India's external trade figure reached ` 101000 nutrients are those nutrients which are required in large
crore in 2011-12. quantities. Examples : - Nitrogen (N) Phosphorous (P),
Potassium (K) Magnesium (Mg).
67. Which of the following commodity is exported
most in India? 73. Which one of the following is not correctly
(a) Agricultural Commodity matched :-
(a) Chlorophyll - Magnesium
(b) Engineering Goods
(b) Hemoglobin - Iron
(c) Clothes
(c) Edible Salt - Sodium
(d) Medicines
(d) Washing Soap - Potassium
Ans. (b) : When the question was asked during that
Ans. (d) : Sodium Carbonate is also known as washing
time engineering goods were largest export commodity. soda or washing soap. (Na2CO3.10H2O). It does not
Engineering goods are still largest export commodity of contains Potassium.
India during fiscal year 2022. It includes products made
from iron-steel, non-ferrous metals, industrial 74. Which one of the following chemicals helps in
machinery and automobiles etc petroleum products fruit ripening?
came in second position. (a) Carbon dioxide (b) Sulpher dioxide
(c) Nitrogen (d) Ethylene
68. World Smallest Light Bulb made by using?
(a) Tungsten (b) Graphene Ans. (d) : Ethylene is a gas which is also known as
"fruit ripening hormone." Most fruit's produce a gaseous
(c) Chromium (d) Graphite compound called ethylene that starts the ripening
Ans. (b) : Scientists Develop world's smallest Light process. Its level in under ripe fruit is very low, but as
Bulb using Graphene. Researchers and Scientists used the fruits develop, they produce larger amounts of the
the wonder material known as graphene to create the chemical that speeds up the ripening process.
world's smallest light bulb. 75. Which of the following is not stored in the
69. Which color of the light has the shortest body?
wavelength - (a) Vitamin A (b) Vitamin C
(a) Red (b) Yellow (c) Vitamin D (d) Vitamin E
(c) Blue (d) Violet Ans. (b) : Vitamin B and C are water - soluble vitamins.
Ans. (d) : Wavelength of VIBGYOR in ascending order Water soluble vitamins are not stored in the body. They get
Violet < Indigo < Blue < Green < Yellow < Orange < excreted from the body with sweat or urine.
Red. Violet has the shortest wavelength and largest 76. Which of the following vitamin contain cobalt?
frequency. Red has the longest wavelength and smallest (a) Vitamin K (b) Vitamin B12
frequency. (c) Vitamin B6 (d) Vitamin B2
UP RO/ARO (Mains) Exam 2014 147 YCT
CLICK HERE FOR FREE MATERIAL

Ans. (b) : Vitamin B12 which is also known as Ans. (a) : The National Biodiversity Authority (NBA)
cobalamin is water - soluble vitamin. It contains cobalt. was established by the central government in 2003
It is involved in metabolism. It is important in the (Chennai) to implement India's Biological Diversity Act
normal functioning of the nervous system and (2002). NBA is a statutory body and it performs
circulatory system (in the maturation of RBC in the facilitative, regulatory and advisory functions for the
bone narrow). Government of India on issues of conservation
77. Golden Rice is an effective source of which sustainable use of biological resources and fair and
Vitamin? equitable sharing of benefits arising out of the use of
(a) Vitamin A (b) Vitamin B12 biological resources.
(c) Vitamin C (d) Vitamin D 83. Which is most important and harmful, air
Ans. (a) : Genetically Engineered "Golden Rice" pollutant -
contains up to 354 g β - carotene per gram of rice. It is (a) Ozone (b) Hydrogen sulphide
an effective source of Vitamin A. The intake of Vitamin (c) CO2 (d) CO
A provides humans with an important nutrient for Ans. (d) : Six most common Air pollutants are - CO,
vision, growth, reproduction and integrity of the NO2, Ozone, Particulate Matter, Lead, SO2. Among
immune system. these most harmful is Carbon Monoxide (CO). CO is
78. Opium is obtained from often called a silent killer. It is an odorless, invisible gas
(a) Dried leaves (b) Root than can cause sudden illness and death if inhaled.
Potential CO sources include generators, charcoal and
(c) Unripe Fruit's Latex (d) Riped fruit Latex.
gas grills, gas stoves, oil and gas fired furnaces, fire
Ans. (c) : Opium is dried latex obtained from its unripe places, space heaters and motor vehicles.
fruits. Its active ingredients, are - Morphine, Codeine,
84. Aquatic fern, which is used as green manure is
Papaverine, thebaine.
(a) Azolla (b) Salvania
79. What do you mean by A.T.M.
(c) Teridium (d) Marsilia
(a) Automatic Transaction Machine
Ans. (a) : Azolla is used as green manure. Azolla is
(b) Automatic Transfer Machine
very rich in proteins, essential amino acids, vitamins (A,
(c) Automated Teller Machine
B12), growth promoter intermediaries. Azolla is also
(d) Advance Transaction Machine.
rich in iron, copper, magnesium. It is acquatic fern and
Ans. (c) : ATM means Automated Teller Machine which found in symboitic relation with Anabena Algae. It
helps to perform financial transaction such as withdrawal belong to plant family Azollacea which is grown
and deposit of money and other banking operations. primarily as fodder for livestock.
80. Indian Institute of Ecology and Environment is 85. Oxygen, which is obtained from
situated - photosynthesis, comes from -
(a) New Delhi (b) Kolkata (a) Water (b) CO2
(c) Jodhpur (d) Kerala (c) Chlorophyll (d) Phosphoglyceric Acid
Ans. (a) : The Indian Institute of Ecology and Ans. (a) : Photosynthesis is the process by which plants
Environment is situated in New Delhi. It was use sunlight, water and CO2 to create oxygen and
established on the occasion of the World Environment energy in the form of sugar. The process is carried out
Day on the 5th June 1980. by plants, algae and some types of bacteria, which
81. Biodiversity is defined as :- capture energy from sunlight to produce oxygen (O2)
(a) Range of different species in an environment and chemical energy. During photosynthesis oxygen
(b) Diurnal and seasonal changes in an comes from water. Photosynthesis occurs most in Red
environment light and least in Violet light.
(c) Variation of one species from another 86. Which light affect the most on the process of
(d) Effect of physical components on an photosynthesis -
environment (a) Red (b) Blue
Ans. (a) : Biodiversity refers to the variety of life in an (c) Green (d) Violet
ecosystem including plants, animals, bacteria and fungi. Ans. (a) : Photosynthesis occurs most in Red light and
It is variation among living organisms from different least in Violet light.
sources including terrestrial, marine and desert 87. Match the following :
ecosystems and the ecological complex of which they
District Minerals
are a part.
A. Sonbhadra 1. Non-Plastic Fire Clay
82. National Biodiversity Authority was
established in - B. Allahabad 2. Copper
(a) 2003, Chennai (b) 2003, Bengaluru C. Lalitpur 3. Glass Sand
(c) 2003, Hyderabad (d) 2003, Kerala D. Mirzapur 4. Limestone
UP RO/ARO (Mains) Exam 2014 148 YCT
CLICK HERE FOR FREE MATERIAL

A B C D Ans. (c) : Gorakhpur is mainly associated with services


(a) 1 2 3 4 Gorakhpur is famous for handloom clothes manufacturing.
(b) 4 3 2 1 It is also famous for Terracotta item manufacturing. Sports
(c) 4 3 1 2 product are mainly manufactured at Meerut.
(d) 2 1 4 3 94. Match the following :-
Ans. (b): Trade/Business Centre
District Minerals A. Wooden Toys 1. Meerut
A. Sonbhadra 1. Limestone B. Sports Item 2. Bareilly
B. Allahabad 2. Glass Sand C. Brass Idols 3. Varanasi
C. Lalitpur 3. Copper D. Matchstick 4. Mathura
D. Mirzapur 4. Non - Plastic Fire Clay A B C D
88. Guru Purnima festival in Uttar Pradesh is (a) 1 4 3 2
dedicated to which saint - (b) 3 2 1 4
(a) Vyas (b) Vashisht (c) 2 1 4 3
(c) Valmiki (d) Bhrigu (d) 3 1 4 2
Ans. (a) : Hindus observe Guru Purnima as the birth Ans. (d): The correct mater as -
anniversary of Maharishi Ved Vyasa in Uttar Pradesh. Trade Centre
He is the author of Mahabharata. Therefore, the day is
also marked as Vyasa Purnima. A. Wooden Toys Varanasi
89. Soyabean is cultivated mainly on which region B. Sports Item Meerut
of Uttar Pradesh C. Brass Idols Mathura
(a) East (b) Central D. Matchstick Bareily
(c) Vindhayan (d) Bundelkhand 95. Find the missing word in the series -
Ans. (d) : In Uttar Pradesh soyabean is cultivated LK – NL – MN – KM –
mainly in Bundelkhand region. Areas of Bundelkhand (a) IJKL (b) JKMN
where soyabean is cultivated are - Jalaun, Jhansi, (c) MKLN (d) LKMN
Lalitpur, Hamirpur, Banda and Mahoba. Ans. (c) : The answer letters must complete the
90. Central Agriculture University is established in question series such as - LKMN/LKMN/LKMN. Thus
Uttar Pradesh at - missing word are MKLN.
(a) Varanasi (b) Jhansi 96. By Pointing towards Geeta, Ramesh said, "I
(c) Banda (d) Bareilly am the only son of the son of her mother" How
Ans. (b) : In Uttar Pradesh Central Agricultural University is Geeta related to Ramesh?
is established in Jhansi. The University is named as Rani (a) Mother (b) Aunty (Mausi)
Lakshmi Bai Central Agricultural University. It was (c) Aunty (Buwa) (d) None of the above
established in 2014 by an Act of Parliament. Ans. (c) :
91. Oldest University of Uttar Pradesh
(a) Banaras Hindu University
(b) Allahabad University
(c) Chaudhary Charan Singh University
(d) Lucknow University
Ans. (b) : Oldest University of Uttar Pradesh is
University of Allahabad. It was established in 1887.
Banaras Hindu University was established in 1916. Geeta is the aunty (Buwa) of Ramesh.
Chaudhary Charan Singh University was established in
97. Arrange the following in sequence
1965 and Lucknow University was established in 1921.
1. Rectangle 2. Line
92. Dr. Ambedkar Institute of Technology for 3. Triangle 4. Point
Handicapped (AITH) is situated at -
5. Angle
(a) Agra (b) Allahabad (Prayagraj)
(c) Kanpur (d) Lucknow (a) 42315 (b) 42531
(c) 13542 (d) 1345256
Ans. (c) : AITH was established in 1997 at Kanpur (UP,
India) by Government of Uttar Pradesh with the Ans. (b) : Point have no lines, then a single line, two
assistance from World Bank. lines make an angle, three lines make a triangle while
four lines make a rectangle. Therefore the correct
93. Which one of the following is not correctly sequence should be 4, 2, 5, 3, 1.
matched -
98. Find the missing word -
(a) Noida - Software and IT business
(b) Sitapur - Plywood trade A D H
(c) Gorakhpur - Items relating to sports F I M
(d) Varanasi - Silk trade ? N R
UP RO/ARO (Mains) Exam 2014 149 YCT
CLICK HERE FOR FREE MATERIAL

(a) K (b) N Ans. (d) :


(c) O (d) P
Ans. (a) : Sequence
ADH 1 4 8 102. If Z = 26, NET = 39, Then UNT is equal to -
F I M 6 9 13 (a) 50 (b) 53
? N R ? 14 18 (c) 55 (d) 56
Second column - 4 + 5 = 9 Ans. (c) : Z = 26 (26th number of alphabet)
9 + 5 = 14 ∴ NET = 14+5+20 = 39
Third column- 8 + 5 ∴ UNT = 21+14+20 = 55
13 + 5 = 18 103. What will be in place of (?) in given question
First column - 1 + 5 = 6
6 + 5 = 11
So, ? = 11
99. Find the odd one out :-
(a) AZCX (b) LOMN (a) 7 (b) 6
(c) EVFU (d) GTHR (c) 2 (d) 0
Ans. (d) : Ans. (a) : 1st figure : (5 + 6) – (4 + 7) = 11 – 11 = 0
Opposite
(a) A ←→ Z
Opposite
(b) L ←→ O 2nd figure : - (7 + 6) – (8 + 4)
= 13 – 12 = 1
Opposite Opposite
C ←→ X M ←→ N 3rd figure = (11 + 5) – (6 + 3)
= 16 – 9 = 7
Opposite
(c) E ←→ V
Opposite
(d) G ←→ T 104. Which among the following is largest -
(a) 41/3 (b) 61/4
Opposite Opposite 1/6
F ←→ U H ←→ S (not R) (c) 15 (d) 2451/12
100. On 18 -9 - 1977 was Sunday. One couple were Ans. (a) : 4 = 4 = (4 ) = (256)1/12
1/3 4/12 4 1/12

married on this day. In next 15 years, how 61/4 = 63/12 = (63)1/12 = (216)1/12
many times their marriage anniversary will fall 151/6 = 52/12 = (152)1/12 = (225)1/12
on Sunday. 2451/12 = (245)1/12
(a) 1 (b) 2 Thus largest number is (256)1/12 = 41/3
(c) 5 (d) 9 105. Which hurricane in 2012 hit North east and
Ans. (b) : Odd days from 1977 to 15 years ahead eastern coastal states of USA -
Year Odd day (a) Katrina (b) Vilma
1978 1 (c) Rita (d) Sandy
1979 1 Ans. (d) : Sandy was the 18th tropical storm and the 10th
1980 2 hurricane of the 2012 Atlantic regions and became one
of the most significant storm in northeast US history. It
1981 1 reached on the coast of USA on 30th October 2012.
1982 1
106. Who has hit double century during 2015 Ashes
1983 1 Cricket Series between England and Australia.
1984 2 (a) Cook (b) Root
1985 1 (c) Smith (d) Strokes
1986 1 Ans. (c) : Steve Smith scored 215 in Ashes Series, in
1987 1 match played at Lords. He became just second
1988 2 Australian to score two double hundreds in an Ashes
1989 1 Series in England.
1990 1 107. Which is most cheapest travel destination
1991 1 around the world according to Trip Advisor
1992 2 Report 2015.
Sum of odd days till 1983 = 7 ≅ 0 odd day (a) Hanoi (b) Mumbai
Sum of odd days till 1988 = 14 ≅ 0 odd day (c) Warsaw (d) Toronto
That is why their marriage anniversary will fall on Ans. (a) : According to Trip Advisor Report of 2015.
Sunday for two times in next 15 years. The most cheapest travel destination around the world
was Hanoi (Vietnam), followed by Warsaw (Poland).
101. Which word will come in place of (?) in the The most expensive destination was Cancun (Mexico),
series Zurich (Switzerland) According to Travel Advisor
D, F, I, M, ?, X Report, 2021, most popular travel destination around
(a) S (b) T the world are - (1) Dubai, UAE (2) London, United
(c) Q (d) R Kingdom (3) Cancun, Mexico.
UP RO/ARO (Mains) Exam 2014 150 YCT
CLICK HERE FOR FREE MATERIAL

108. Recently Japan had reduced the Voting age - 114. Who had recently made his 1st One day
(a) From 20 to 18 years International century?
(b) From 21 to 18 years (a) Monoj Tiwari (b) Manish Pandey
(c) From 25 to 20 years (c) Kedar Jadhav (d) Robin Uthappa
(d) From 25 to 18 years. Ans. (c) : Kedar Jadhav scored 105 runs during India
Ans. (a) : On 17th June, 2015 Japan had reduced the Vs Zimbabwe one day international cricket match and
voting age from 20 to 18 years. Previously in 1945 it it was his first century.
was reduced from 25 to 20 years. 115. In this century, Naba Kalebar Rath Yatra of
Voting Age in India is 18 years which was done by 61st Bhagvan Jaganath was of which number?
Constitutional Amendment. (a) 1st (b) 2nd (c) 3rd (d) 4th
109. Which developed nation was recently declared Ans. (a) : The century's first Naba Kalebar Rath Yatra
as debtor of IMF. of Lord Jagannath was held in Puri in 2015. It was 9
(a) Argentina (b) Greece day journey of the new Idols of Lord Jagannath, Lord
(c) UK rain (d) Jamaica Balabhadra and Devi Subhadra to Gundicha Temple
Ans. (b) : Greece became the first developed country on and back to their abode.
29th June 2015 to default on it's debt to the IMF after 116. In 2015 - 16, Union government has restarted
missing a 1.5 billion euro payment. which Pension Scheme -
110. What is India's rank in World Peace Index of (a) Ram Manohar Lohiya Pension Scheme
2015. (b) Pradhanmantri Pension Yojana
(a) 154 (b) 114 (c) Atal Pension Yojana
(c) 84 (d) 143 (d) Nirantar Pension Yojana
Ans. (d) : Global Peace Index is released by Institute of Ans. (c) : Atal Pension Yojana was launched in 2015 to
Economics and Peace (IEP). In 2015, India was placed create a universal social security systems for all Indians,
at 143th position. In 2022, India position was at 135th especially the poor, the under privileged and the workers in
position whereas Iceland was at 1st position. the unorganized sector. It is administered by Pension Fund
111. Which country launched BX-1E, the world's Regulatory and Development Authority (PFRDA).
first electric passenger plane 117. Which State government in July 2015 tabled
(a) Japan (b) USA the resolution to remove the role of Chief
(c) Switzerland (d) China Justice in appointment of Lokayukta
(a) MP (b) Karnataka
Ans. (d) : China had produced world's first electric
passenger aircraft which has been named BX-1E. The (c) Uttar Pradesh (d) Rajshtan
aircraft was designed by Shenyang Aerospace Ans. (c) : On 27th August 2015, Uttar Pradesh
University and Liaoning General Aviation Academy government passed a resolution and tabled it to remove
located in the north eastern Liaoning Province of China. the role of Chief Justice in appointment of Lokayukta.
It has speed of 160 Km per hour. Flight time is 45 Sanjay Mishra is present Lokayukta of Uttar Pradesh.
minutes to one hour. 118. Which of the following bank launches India's
112. Which of the following Statement is correct first mobile ATM
regarding Indian President visit to Sweden. (a) ICICI (b) IDBI
1. Shri Pranab Mukherjee became first (c) HDFC (d) SBI
Indian President to visit Sweden. Ans. (a) : ICICI which is India's one of the largest
2. He had visited Sweden during last week of Private sector bank launched ATM on wheels and
June 2015 country's first mobile ATM in Mumbai.
3. Total 6 agreements were signed related to 119. Which of the following is not a cause of
cooperation in the areas of urban 'cultural lag'
development, medium and small scale (a) Religion (b) Politics
enterprises, polar research and medicine. (c) Law (d) Tradition
(a) Only 1 & 2 (b) Only 1 & 3 Ans. (b) : Difference between material culture and non-
(c) Only 3 (d) All of the above (1, 2, 3) material culture is called cultural lag. It is seen as an
Ans. (b) : Shri Pranab Mukherjee became 1st Indian issue because failure to develop broad social consensus
President to visit Sweden and he started his visit during on appropriate applications of modern technology may
last week of May 2015. Total 6 agreements lead to breakdowns in social solidarity and rise of social
(Memorandum of Understanding) were signed such as conflict. It is affected by Religion, Law and Traditions
on Sustainable Urban Development, Micro, Small and and customs of the society.
Medium Enterprises etc. 120. Which coast of India was most affected by the
113. Which state had banned the use of disposable Tsunami of 2004
thermocol plates. (a) Malabar coast (b) Konkan coast
(a) Chattisgarh (b) Jharkhand (c) Coromondal coast (d) Norther lircar coast
(c) MP (d) Bihar Ans. (c) : Coromandal coast was most affected by the
Ans. (b) : In June 2015 Jharkhand Government banned 2004 Tsunami (26th December 2004). Andhra Pradesh,
sale of thermocol plates with aim to promote leaf plates Tamil Nadu and Pondicherry were states which were
and generate employment for poor and tribal people. mostly affect by the Tsunami waves.
UP RO/ARO (Mains) Exam 2014 151 YCT
CLICK HERE FOR FREE MATERIAL

Gòej ØeosMe meceer#ee DeefOekeâejer/meneÙekeâ meceer#ee DeefOekeâejer (cegKÙe) hejer#ee, 2014


meeceevÙe Meyo %eeve SJeb JÙeekeâjCe
nue ØeMve-he$e
efJeueesce JeekeäÙe SJeb Jele&veer mecyevOeer DeMeggefæÙeeB
1. `Deentle' keâe efJeueesce Meyo nw– 7. efvecve ceW mes Megæ Jele&veer Jeeuee Meyo nw :
(a) ntle (b) Deventle (a) GppJeue (b) JewcevemÙelee
(c) Dehentle (d) Deveentle (c) keâefJe$eer (d) ØeceeefCekeâ

Gòej─(d) Gòej─(a)
JÙeeKÙee – `GppJeue' Megæ Jele&veer Jeeuee Meyo nw~ peyeefkeâ keâefJe$eer keâe
JÙeeKÙee – `Deentle' keâe efJeueesce Meyo `Deveentle' nw~ Mes<e efJekeâuhe leke&â mebiele
`keâJeefÙe$eer' leLee ØeceeefCekeâ keâe ØeeceeefCekeâ' SJeb JewcevemÙelee keâe `JewcevemÙe'
veneR nQ~
Megæ Jele&veer' nesiee~
2. `DeLe' keâe efJeueesce Meyo nw– 8. efvecveefueefKele MeyoeW ceW efkeâmekeâer Jele&veer Megæ nw?
(a) DeLekeâ (b) he=Lekeâ (a) mebvÙeeme (b) ØelÙethekeâej
(c) Fefle (d) Deefle (c) ogjeJemLee (d) kewâuee<e
Gòej─(c) Gòej─(a)
JÙeeKÙee –`DeLe' keâe efJeueesce Meyo `Fefle' nesiee~ Mes<e efJekeâuhe leke&â mebiele JÙeeKÙee –`mebvÙeeme' Meyo keâer Jele&veer Megæ nw peyeefkeâ DevÙe efJekeâuhe ceW efoS
veneR nQ~ ieÙes MeyoeW keâer Megæ Jele&veer nesieer~
DeMegæ – Megæ
3. ‘Jeeoer’ keâe efJeueesce Meyo nw–
ØelÙethekeâej – ØelÙeghekeâej
(a) DeJeeoer (b) efJeJeeoer
ogjeJemLee – ogjJemLee
(c) ØeefleJeeoer (d) DeveeJeeoer kewâuee<e – kewâueeme
Gòej─(c) 9. efvecve ceW mes DeMegæ Jele&veer Jeeuee Meyo nw :
JÙeeKÙee – `Jeeoer' keâe efJeueesce Meyo `ØeefleJeeoer' nesiee~ Mes<e efJekeâuhe leke&â (a) GodIees<e (b) Skeâlee
mebiele veneR nQ~ he=Lekeâ keâe efJeueesce mebÙegòeâ leLee Deefle keâe efJeueesce vÙetve nesiee~ (c) leelkeâeefuekeâ (d) Ghejeskeäle
4. `DeJej' keâe efJeueesce Meyo nw– Gòej─(d)
(a) ueIeg (b) ØeJej JÙeeKÙee –`Ghejeskeäle' DeMegæ Jele&veer Jeeuee Meyo nw~ Fmekeâe Megæ Jele&veer
(c) megJej (d) keâefve‰ `GheÙeg&keäle' nesiee~ Mes<e efJekeâuhe Jele&veer keâer Âef„ mes Megæ nQ~
Gòej─(b) 10. efvecveefueefKele JeekeäÙeeW ceW mes Skeâ JeekeäÙe DeMegæ nw :
(a) keâeMeer meowJe mes YeejleerÙe mebmke=âefle keâe kesâvõ jne nw~
JÙeeKÙee – `DeJej' keâe efJeueesce Meyo `ØeJej' nesiee~ peyeefkeâ `ueIeg' keâe efJeueesce
(b) ieeBOeerpeer keâe ÛejKee Ûeueevee `mJeosMeer' keâe Øeleerkeâ Lee~
`ieg®' leLee `keâefve‰' keâe efJeueesce `Jeefj‰' SJeb `megJej' keâe efJeueesce `kegâJej' nesiee~
(c) Jeve-peerJeve kesâ keâ„eW keâe YeÙe Yeer meerlee keâes jece kesâ Devegieceve mes jeskeâ
5. `peefšue' keâe efJeueesce Meyo nw– veneR mekeâe~
(a) mejue (b) kegâefšue (d) Deheveer kegâMeue jCeveerefle mes efMeJeepeer ves efJeheef#eÙeeW kesâ Úkeäkesâ ÚgÌ[e
(c) menpe (d) Depeefšue] efoÙes Les~
Gòej─(a) Gòej─(a)
JÙeeKÙee – `peefšue' keâe efJeueesce Meyo `mejue' nw~ peyeefkeâ kegâefšue keâe Yeer JÙeeKÙee –`keâeMeer meowJe mes YeejleerÙe mebmke=âefle keâe kesâvõ jne nw~' Ùen DeMegæ
efJeueesce `mejue' nesiee leLee `menpe' keâe efJeueesce `Demenpe' nesiee~ JeekeäÙe nw~ keäÙeeWefkeâ Fme JeekeäÙe ceW `keâeMeer' Meyo keâlee& nw pees meowJe m$eereEueie
ceW ØeÙegkeäle neslee nw~ Dele: `jne nw' kesâ mLeeve hej `jner nw' nesvee ÛeeefnS~ Mes<e
6. efJeueesce keâer Âef„ mes Skeâ Ùegice DeMegæ nw–
meYeer Megæ JeekeäÙe nQ~
(a) DeeÙeele-efveÙee&le (b) ÂMÙe-DeÂMÙe
11. efvecveefueefKele JeekeäÙeeW ceW mes Megæ JeekeäÙe nw :
(c) ØelÙe#e-hejes#e (d) Deeefce<e-meeefce<e
(a) Jes meye Øekeâej kesâ oes<eeW mes efJenerve nQ~
Gòej─(d) (b) Jes meye oes<eeW mes nerve nQ~
JÙeeKÙee – efJeueesce keâer Âef„ mes `Deeefce<e-meeefce<e' Ùegice DeMegæ nw~ Fmekeâe (c) Jes meye Øekeâej kesâ oes<eeW mes cegkeäle nQ~
Megæ Ùegice `Deeefce<e-efvejeefce<e' nesiee~ peyeefkeâ DeeÙeele-efveÙee&le, ÂMÙe-DeÂMÙe, (d) Jes meYeer ner oes<eeW mes cegkeäle nQ~
ØelÙe#e-hejes#e efJeueesce keâer Âef„ mes Megæ Ùegice nQ~ Gòej─(c)
UP RO/ARO (Main) General Hindi 2014 152 YCT
CLICK HERE FOR FREE MATERIAL

JÙeeKÙee –`Jes meye Øekeâej kesâ oes<eeW mes cegkeäle nQ~' Ùen Megæ JeekeäÙe nw~ peyeefkeâ 15. `pees he=LJeer mes mecyeæ nw'– JeekeäÙeebMe kesâ efueS Skeâ Meyo nw–
`Jes meye Øekeâej kesâ oes<eeW mes efJenerve nQ~' leLee `Jes meye oes<eeW mes nerve nQ~' JeekeäÙeeW (a) he=Lee (b) ueewefkeâkeâ
ceW ›eâceMe: `efJenerve' SJeb `nerve' MeyoeW keâe ØeÙeesie DevegheÙegkeäle nw~ Fmeer Øekeâej (c) YetieesueerÙe (d) heee|LeJe
`Jes meYeer ner oes<eeW mes cegkeäle nQ~' JeekeäÙe ceW `meYeer ner' Meyo keâe ØeÙeesie DevegheÙegkeäle Gòej─(d)
nw ÛetBefkeâ `meYeer→ meye + ner' neslee nQ~ mhe„ nw efkeâ `ner' keâe ØeÙeesie Deefleefjkeäle JÙeeKÙee –`pees he=LJeer mes mecyeæ nw'–JeekeäÙeebMe kesâ efueS Skeâ Meyo `heee|LeJe' nw~
nw~ Dele: JeekeäÙe $egefšhetCe& nw~ Fme ueeskeâ mes mebyebOe jKeves Jeeuee `ueewefkeâkeâ' neslee nw~ Mes<e efJekeâuhe Demebiele nQ~
12. efvecveefueefKele JeekeäÙeeW ceW mes Skeâ JeekeäÙe Megæ nw : 16. `pees henues Lee, hej Deye veneR nw' kesâ efueS Skeâ Meyo nw–
(a) he=LJeerjepe Ûeewneve ves Ùeefo jepeveereflekeâ ÛeelegÙe& keâe heefjÛeÙe efoÙee neslee
(a) hetJe&pe (b) DeblÙepe
lees Deepe Yeejle keâe Fefleneme kegâÚ Deewj ner neslee~
(c) YetlehetJe& (d) Deveeiele
(b) efieueeme Thej lekeâ otOe mes "mee"me Yeje ngDee Lee~
Gòej─(c)
(c) Øeyegæ efJeÅeee|LeÙeeW keâes efMe#ekeâ keâe kesâJeue mebkesâle cee$e ner heÙee&hle
neslee nw~ JÙeeKÙee –`pees henues Lee, hej Deye veneR nw' kesâ efueS Skeâ Meyo–`YetlehetJe&
neslee nw~ Mes<e Fme Øekeâej nQ–
(d) efMe#ekeâ ves efJeÅeee|LeÙeeW keâes DeelebkeâJeeo kesâ Thej efveyevOe efueKeves keâe
DeeosMe efoÙee~ pees henues pevcee nes –hetJe&pe

Gòej─(a) pees efvecve peeefle ceW hewoe ngDee nes –DeblÙepe


pees ve DeeÙee nes –Deveeiele
JÙeeKÙee –efJekeâuhe (a) ceW efveefnle JeekeäÙe Megæ nw~ peyeefkeâ `efieueeme Thej
lekeâ otOe mes "mee"me Yeje ngDee Lee~ JeekeäÙe ceW `"mee"me' keâe ØeÙeesie DevegheÙegkeäle 17. `pees keâef"veeF& mes efceuelee nw'– kesâ efueS Skeâ Meyo nw–
nw~ `Øeyegæ efJeÅeee|LeÙeeW keâes efMe#ekeâ keâe kesâJeue mebkesâle cee$e ner heÙee&hle neslee (a) ogue&Ye (b) DeueYÙe
nw~' JeekeäÙe ceW oes Meyo `kesâJeue' SJeb `cee$e' meceeveeLeea' nQ~ Fme lejn JeekeäÙe (c) DeØeehle (d) DevegheueyOe
ceW kesâJeue Ùee cee$e ceW mes efkeâmeer Skeâ Meyo keâe ØeÙeesie keâjvee ÛeeefnS~ JeekeäÙe Gòej─(a)
`efMe#ekeâ ves efJeÅeee|LeÙeeW keâes DeelebkeâJeeo kesâ Thej efveyevOe efueKeves keâe DeeosMe JÙeeKÙee – `pees keâef"veeF& mes efceuelee nw' kesâ efueS Skeâ Meyo `ogue&Ye' neslee
efoÙee~' DeMegæ nw keäÙeeWefkeâ kesâ Thej' Meyo keâe ØeÙeesie DevegheÙegkeäle nw~ Fmekesâ nw~ Mes<e efvecve Øekeâej nQ–
mLeeve hej `hej' Meyo ØeÙeesie nesvee ÛeeefnS~
efpemes Øeehle ve efkeâÙee pee mekesâ – DeueYÙe
Deveskeâ MeyoeW kesâ Skeâ Meyo pees Øeehle ve nes – DeØeehÙe
13. `efpemes peeveves keâer FÛÚe nw'– kesâ efueS Skeâ Meyo nw– pees GheueyOe ve nes – DevegheueyOe
(a) %eeveeLeea (b) efpe%eemeg 18. `yengle meer Yee<eeDeesb keâes peeveves Jeeuee'– kesâ efueS Skeâ Meyo nw–
(c) efpeieer<eg (d) efpepeerefJe<eg (a) yengYee<eeefJeod (b) yengYee<eeYee<eer
Gòej─(b) (c) ogYeeef<eÙee (d) yengYeeef<eÙee

JÙeeKÙee – efpemes peeveves keâer FÛÚe nw' kesâ efueS Skeâ Meyo `efpe%eemeg' nesiee~ Gòej─(a)
peyeefkeâ efkeâmeer hej efJepeÙe heeves keâer FÛÚe jKeves Jeeuee' efpeieer<eg leLee `DeefOekeâ JÙeeKÙee –`yengle meer Yee<eeDeesb keâes peeveves Jeeuee' kesâ efueS Skeâ Meyo `yeng
meceÙe lekeâ peerles jnves keâes FÛÚgkeâ' `efpepeerefJe<eg' Skeâ Meyo nesiee~ Yee<eeefJeod' nw~ Mes<e efJekeâuhe Demebiele nQ~
14. `pees keâef"veeF& mes mecePeves ÙeesiÙe nw'– kesâ efueS Skeâ Meyo nw– lelmece SJeb leodYeJe
(a) Deiece (b) eqkeäue„ 19. `{er"' keâe lelmece ¤he nw–
(c) ogyeexOe (d) ogie&ce
(a) Oetle& (b) Oe=„
Gòej─(c) (c) Göb[ (d) Oe=ef<ele
JÙeeKÙee –`pees keâef"veeF& mes mecePeves ÙeesiÙe nw' kesâ efueS Skeâ Meyo `ogyeexOe' Gòej─(b)
nw~ JeekeäÙeebMeeW kesâ efueS kegâÚ Meyo Fme Øekeâej nQ– JÙeeKÙee –`{er"' keâe lelmece ¤he `Oe=„' neslee nw~
pees iece jefnle nes – Deiece
20. efvecve MeyoeW ceW keâewve mee Meyo lelmece veneR nw?
peneB Ùee efpemekesâ Deboj ve peeÙee pee mekesâ – DeiecÙe
(a) ce=efòekeâe (b) efJekeâej
pees mecePeves ceW keâef"ve nes (Yee<ee kesâ mecyevOe ceW) – eqkeäue„
(c) keâheeš (d) keâhetj
peneB peevee keâef"ve nes – ogie&ce Gòej─(d)
UP RO/ARO (Main) General Hindi 2014 153 YCT
CLICK HERE FOR FREE MATERIAL

JÙeeKÙee –`keâhetj' lelmece Meyo veneR nw~ Ùen leodYeJe Meyo nw~ `keâhetj' keâe (c)ef›eâÙee (d) DeJÙeÙe
lelmece Meyo `keâhet&j' neslee nw~ `ce=efòekeâe' keâe leodYeJe `efcešdšer' SJeb `keâheeš' Gòej─(b)
keâe leodYeJe `efkeâJeeÌ[' nw~ JÙeeKÙee –efpeme efJekeâejer Meyo mes meb%ee keâer JÙeeefhle ceÙee&efole nesleer nw, Gmes
21. efvecve ceW mes keâewve mee Meyo leodYeJe nw? ‘efJeMes<eCe’ keânles nw~ GoenjCe–IeesÌ[e oewÌ[ jne nQ~ ÙeneB `IeesÌ[e' Meyo mecemle
(a) keâesÙeue (b) DeeMÛeÙe& IeesÌ[e peeefle keâe yeesOe keâje jne nw~ peyeefkeâ Skeâ IeesÌ[e oewÌ[ jne nw~ ÙeneB `Skeâ'
(c) GppJeue (d) kebâškeâ
mebKÙee JeeÛekeâ efJeMes<eCe nw~ pees IeesÌ[e (meb%ee) keâer JÙeeefhle keâes ceÙee&efole keâj
jne nw~
Gòej─(a)
27. `jcesMe keâer hegmlekeâ hegjeveer nw'–Fme JeekeäÙe ceW `hegmlekeâ' Meyo nw–
JÙeeKÙee –`keâesÙeue' Meyo leodYeJe nw efpemekeâe lelmece `keâesefkeâue' neslee nw~ peyeefkeâ
(a) efJeMes<Ùe (b) efJeMes<eCe
`DeeMÛeÙe&' keâe leodYeJe `DeÛejpe', `GppJeue' keâe leodYeJe `Gpeeuee' leLee `kebâškeâ'
keâe leodYeJe `keâeBše' neslee nw~ (c) ef›eâÙee-efJeMes<eCe (D) meJe&veece

22. `Deóeefuekeâe' keâe leodYeJe ¤he nw– Gòej─(a)


(a) Dešeuee (b) Dešue JÙeeKÙee –`jcesMe keâer hegmlekeâ hegjeveer nw' JeekeäÙe ceW hegmlekeâ Meyo `efJeMes<Ùe'
(c) Dešeje (d) Dešejer nw~ efpeme meb%ee Meyo keâer efJeMes<eCe Éeje efJeMes<elee yeleueeÙeer peeleer nw Gmes efJeMes<Ùe
keânles nw~
Gòej─(d)
28. `Gme «ebLe ceW 500 he=‰ nQ'– Fme JeekeäÙe ceW `Gme' Meyo nw–
JÙeeKÙee –`Deóeefuekeâe' keâe leodYeJe ¤he `Dešejer' nw~ Mes<e efJekeâuhe Demebiele
(a) iegCeJeeÛekeâ efJeMes<eCe
nQ~
(b) heefjceeCeJeeÛekeâ efJeMes<eCe
23. efvecve ceW keâewve-mee Meyo leodYeJe nw?
(c) mebkesâleJeeÛekeâ efJeMes<eCe
(a) peue (b) veive
(d) JÙeeqkeäleJeeÛekeâ efJeMes<eCe
(c) leerve (d) YeÇcej
Gòej─(c)
Gòej─(c)
JÙeeKÙee –`Gme «ebLe ceW 500 he=‰ nQ' JeekeäÙe ceW `Gme' Meyo ceW mebkesâleJeeÛekeâ
JÙeeKÙee –`leerve' Meyo leodYeJe nw~ Fmekeâe lelmece ¤he `$eerefCe' neslee nw~ Mes<e
efJeMes<eCe nw~ Ùen meeJe&veeefcekeâ efJeMes<eCe keâe Yeeie nw~ heg®<eJeeÛekeâ Ùee efvepeJeeÛekeâ
lelmece Meyo nw~ peue, veive SJeb YeÇcej keâe leodYeJe ›eâceMe: heeveer, vebiee leLee meJe&veeceeW keâes ÚesÌ[keâj DevÙe meJe&veece peye efkeâmeer meb%ee keâer efJeMes<elee yeleueeSB,
YeeQje neslee nw~ lees GvnW `meeJe&veeefcekeâ efJeMes<eCe’ keânles nw~
24. `keâhe&š' keâe leodYeJe ¤he nw– 29. `Ùen ÛeeBoer Keesšer meer efoKeleer nw'–Fme JeekeäÙe ceW Keesšer meer' efJeMes<eCe
(a) keâhetj (b) keâheÌ[e keâe Øekeâej nw–
(c) keâke&âš (d) Kehhej (a) iegCeJeeÛekeâ efJeMes<eCe
Gòej─(b) (b) mebKÙeeJeeÛekeâ efJeMes<eCe
JÙeeKÙee –‘keâhe&š’ keâe leÆJe ™he keâheÌ[e neslee nw peyeefkeâ keâhetj keâe lelmece (c) heefjceeCe yeesOekeâ efJeMes<eCe
™he keâhet&j nesiee~ Mes<e efJekeâuhe Demebiele nQ~ (d) hetCeeËkeâ yeesOekeâ efJeMes<eCe
Gòej─(a)
efJeMes<Ùe SJeb efJeMes<eCe
JÙeeKÙee –`Ùen ÛeeBoer Keesšer meer efoKeleer nw' JeekeäÙe ceW `Keesšer meer' `iegCeJeeÛekeâ
25. efvecveefueefKele ceW mes keâewve mee Meyo efJeMes<eCe nw?
efJeMes<eCe' keâe Øekeâej nw~ efpeme efJeMes<eCe mes efkeâmeer meb%ee Ùee meJe&veece keâe iegCe-oes<e,
(a) yeveejmeer (b) YeejleJe<e&
®he-jbie, Deekeâej-Øekeâej, mecyevOe, oMee Deeefo keâe helee Ûeues, Gmes iegCeJeeÛekeâ
(c) yeepeej (d) Kesleer efJeMes<eCe’ keânles nw~
Gòej─(a) 30. `oveeceesue' ceW efkeâme Øekeâej keâe efJeMes<eCe nw?
JÙeeKÙee –`yeveejmeer' Meyo efJeMes<eCe nw~ pees Meyo meb%ee Ùee meJe&veece keâer efJeMes<elee (a) heefjceeCe yeesOekeâ
yeleueeles nw GvnW efJeMes<eCe keânles nw~ GoenjCeeLe&- jeOee yeveejmeer meeÌ[er henveer (b) DehetCeeËkeâ yeesOekeâ
nw~ ÙeneB `meeÌ[er' efJeMes<Ùe keâer yeveejmeer efJeMes<eCe kesâ Éeje efJeMes<elee yeleueeÙeer
(c) DeefveeqMÛele mebKÙeeJeeÛekeâ
pee jner nw~
(d) efveeqMÛele mebKÙeeJeeÛekeâ
26. efpeme efJekeâejer Meyo mes meb%ee keâer JÙeeefhle ceÙee&efole nesleer nw, Gmes Gòej─(b)
keânles nQ–
JÙeeKÙee –`oveeceesue' ceW DehetCeeËkeâ yeesOekeâ efJeMes<eCe nQ~
(a) meJe&veece (b) efJeMes<eCe
UP RO/ARO (Main) General Hindi 2014 154 YCT
CLICK HERE FOR FREE MATERIAL

UPPSC RO-ARO (Pre) Exam-2014


GENERAL STUDIES
Solved Paper
1. Betwa river is a tributery of - 8. Sargasso Sea is characterized by
(a) Ganga (b) Yamuna (a) very cold water
(c) Brahma putra (d) Son (b) very warm water
Ans. (b) : The Betwa or Betravati is a river in Northern (c) highly saline water
India, and a tributary of the Yamuna. The Betwa river (d) typical marine vegetation
rises in the vindhya range just north of Hoshangabad in Ans. (b) : The Sargasso sea in a region in the gyre in
Madhya Pradesh and flows north-east through Madhya the middle of the North Atlantic ocean, only sea on the
Pradesh and Orchha in Uttar Pradesh. Earth that has no coastline. The highest salinity 37%
2. Which continent has maximum concentration due the high temperature and evaporation, of water.
of tropical deciduous of forests - Hence option (b) is the correct answer.
(a) Africa (b) Asia 9. Which of the following lakes is a world heritage
(c) Australia (d) South America site?
Ans. (b) : In Asia the tropical deciduous forests are (a) Red Lagoon of Bolivia
most extensive. (b) Boiling Lake of Dominica
3. Mexico is largest producer of - (c) Five Flower Lake of China
(a) Gold (b) Copper (d) Plitvice Lake of Croatia
(c) Zinc (d) Silver Ans. (d) : Plitvice Lakes National Park, a UNESCO
Ans. (d) : Mexico is the largest producer of silver. The world hesitate site, is located in Croatia. It is one of the
country produced 5,600 metric tons in 2021. oldest and largest national parks in Croatia
4. Which city is located on Lake Superior - 10. Which one of the following States of India is
(a) Chikago (b) Detroit the largest producer of coconut?
(c) Baffalo (d) Duluth (a) Kerala (b) Karnataka
Ans. (d) : Duluth is the second largest city that is (c) Tamil Nadu (d) Andhra Pradesh
located on lake Superior after thunder Bay, Ontario, and Ans. (b) : As of 2021-22 Karnataka has the largest
has the largest metropolitan area on lake Superior. production of coconut 4,210.87 tonnes.
5. Hazira-Bijaypur - Jagdishpur (HBT) Gas 11. Mithi river of Mumbai originates from which
pipeline was built by which authority - of the following lakes?
(a) Gas Authority of India Ltd (a) Tulsi Lake (b) Vihar Lake
(b) Dil India Limited (c) Powai Lake (d) None of the above
(c) Indian Oil Company
Ans. (*) : The Mithi River is one of the four rivers
(d) Hindustan Petroleum Ltd flowing through the Mumbai city. It originates from the
Ans. (a) : The Hazira - Bijaypur - Jagdishpur (HBT) Vihar Lake and Powai Lake.
project - started operations in 1987 and is owned by
GAIL. HBJ is a 2887 km long on-shore, shallow water • It flow for a total of 18 km before it meets the
pipeline project, with a maximum diameter of 36 inches, Arabian sea at Mahim Creek.
starts at Arabian Sea (India) and ends in Uttar Pradesh. 12. The historic statue of Mahatma Gandhi has
6. Which if the following States is not known for been unveiled at which country's Parliament
the production of cardamom? Square?
(a) Kerala (b) Karnataka (a) Japan (b) USA
(c) Tamil Nadu (d) Odisha (c) Britain (d) France
Ans. (d) : Odisha is not a cardamom Producing state Ans. (c) : A new statue of Mahatma Gandhi unveiled in
while Kerala, Karnataka and Tamil Nadu are cardamom London's parliament square in 2015 to mark the 100th
producing states. Highest production of cardamom in anniversary of his return to India to start the struggle for
2021-22 is from Kerala 15.54 tonnes. independence from British rule
7. How many States in India are coastal? 13. Which university has topped in Times World
(a) 7 (b) 8 Reputation Rankings, 2015?
(c) 9 (d) 10 (a) Combridge University
Ans. (c) : India has a 9 Coastal States. These are - (b) Oxford University
Gujarat, Maharashtra, Goa, Karnataka, Kerala, Tamil (c) Harvard University
Nadu, Andhra Pradesh, Odisha and West Bengal. (d) Stanford University
UP RO/ARO (Pre) Exam 2014 155 YCT
CLICK HERE FOR FREE MATERIAL

Ans. (c) : According to Times World Reputation rankings Ans. (a) : Mithivirdi Nuclear Power Plant is setup in
2015 Harvard University topped the list while in 2022 Gujarat Bhavnagar district in collaboration with USA.
University of Oxford United Kingdom topped the list. 21. According to the latest report, what is the rank
14. ‘NAINA’ is a new city to be set up around of India in steel production in the world?
which of the following airports? (a) 4th (b) 3rd
(a) Indira Gandhi International Airport, New Delhi (c) 2nd (d) 1st
(b) Rajeev Gandhi International Airport, Hyderabad Ans. (b) : India was the third-largest producer of steel
(c) Navi Mumbai International Airport, Mumbai in the world in 2015 leaving behind the US as the fourth
(d) None of the above largest producer. As of 2021 India is at second position
behind China.
Ans. (c) : NAINA is a city planned around the Navi
Mumbai International Airport. The acronym unfolds to 22. ‘Agenda-21’ is related to which field?
"Navi Mumbai Airport Influence Notified Area". (a) Sustainable development
15. According to the latest Supreme Court (b) Nuclear disarmament
Judgments on Armed Forces Tribunal (AFT), (c) Patent protection
which of the following is true? (d) Agricultural subsidies
(a) High Courts cannot entertain writ petitions Ans. (a) : Agenda 21 sets out a plan of action to guarantee
against verdict of AFT that life in the next millennium will change sustantially for
(b) High Courts can entertain writ petitions the better. It was endorsed by the world's governments at
against verdict of AFT the UN Conference on Environment and Development, in
(c) High Courts can entertain writ petitions Rio Janeiro in June 1992.
against verdict of AFT concerning officials 23. ‘Mission Indradhanush’ of the Union
below the rank of Major only Government is related to
(d) None of the above (a) pulse production
Ans. (a) : Supreme Court in its judgment on Armed (b) immunization of children against seven
Forces Tribunal (AET) that high court cannot entertain vaccine-preventable diseases
writ petitions against verdict of AFT. (c) skill development
16. North Central Zone Cultural Centre (NCZCC) (d) tourism promotion
would organize Chaiti and Kajri centres in Ans. (b) : Mission Indradhanush (M I) was launched by
which city? the Ministry of Health and family welfare on 25th
(a) Mirzapur (b) Bhojpur December 2014 with the aim to ensure high coverage of
(c) Mathura (d) Varanasi children and pregnant women with all available
vaccines throughout the country.
Ans. (d) : North Central Zone Cultural Centre
(NCZCC), Prayagraj would organize Chaite and Kajri 24. Prestigious ‘Tyler Prize’ is given in the field of
centers in the city of Varanasi. (a) drama (b) women rights
17. The book Lucknow Boy was written by (c) child rights (d) environmental protection
(a) Kuldeep Nayar (b) Muzaffar Ali Ans. (d) : The Tyler prize for Environmental
(c) Vinod Mehta (d) Noorul Hasan Achievement is are annul award for environment science,
Ans. (c) : The book Lucknow Boy was written by environmental health and energy. Tyler Laureates receive a
$ 200,000 cash prize and a medallion. In 2022 it was
Vinod Mehta.
awarded to Professor Sir Andy Haines.
18. Which of the following National Parks has 25. According to recent Judgement of the Supreme
started to use drone or unmanned aerial Court, Section 66A of IT Act is a violation of
vehicle for wildlife management?
(a) Article 20 (b) Article 19(1)
(a) Bandipur Tiager Reserve
(c) Article 14 (d) Article 25
(b) Corbett Tiger Reserve
(c) Ranthambore Tiger Reserve Ans. (b) : The supreme court struck down section 66A
of the Information Technology Act, 2000 relating to
(d) Periyar Tiger Reserve restriction on grounds of violating the freedom of
Ans. (a) : Bandipur Tiger Reserve has started to use drone speech guaranteed under Article 19 (1) (a) of the
or unmanned aerial vehicle for wildlife management. Constitution of India.
19. Who amongst the following has been appointed 26. How many of the recently approved 17 mega
as Editor-in-Chief of the Guardian newspaper? food parks for food processing have been
(a) Cheryl Cole (b) Katharine Viner allotted to States agencies?
(c) Joya Vincent (d) Samantha Rice (a) 7 (b) 10
Ans. (b) : Katharine Viner is editor-in-chief of the (c) 11 (d) 13
Guardian newspaper, a position she held since June 2015. Ans. (a) : 7 Mega food parks have been allocated to
20. ‘Mithivirdi’ Nuclear Power Plant will be set up state governments and 10 to private players. Under the
in collaboration with which of the following scheme (2008-09) of mega food parks, the food
countries? processing ministry had sanctioned 42 projects
(a) USA (b) Canada throughout the country of there, 25 parks have already
(c) Russia (d) France been allocated.
UP RO/ARO (Pre) Exam 2014 156 YCT
CLICK HERE FOR FREE MATERIAL

27. ‘Stockholm Water Prize, 2015’ winner 32. Which one of the following sets is different
Rajendra Singh is associated with which NGO? from others?
(a) Bandhan (b) Tarun Bharat Singh (a) 3, 11, 13, 17 (b) 7, 14, 18, 37
(c) Save Water (d) Boond (c) 3, 5, 41, 47 (d) 3, 7, 17, 71
Ans. (b) : Renowned water conservationist and Ans. (b) : Set of option (b) is odd. Because others are
environment activist Rajendra Singh of Tarun Bharat the set of prime numbers.
Singh (TBS) NGO was conferred with 2015 Stockholm 33. A prime number between 10 and 50 remains
Water Prize. unchanged if its digits are reversed.
Professor Emeritus Wiltried Brutsraert has been named Then the square of such a number is
as the Stockholm Water Prize Laureate 2022 for ground (a) 121 (b) 484
breaking work to quantify environmental evaporation. (c) 1089 (d) 1936
28. The Seventh Asian Ministerial Conference, on Ans. (a) : Required prime number = 11
Disaster Risk Reduction to be held in 2016 will Square of this prime number = 121
be hosted by 34. The missing letter in the letter arrangement is
(a) Pakistan (b) Thailand ARENIJM?
(c) Myanmar (d) India (a) V (b) H
Ans. (d) : The 7th edition of Asian Ministerial (c) G (d) F
Conference on Disaster Risk Reduction (AMCDRR) Ans. (d) :
has inaugurated by prime minister Narendra Modi in
New Delhi on 3-5 November, 2015
29. The First Counter-Terrorism Conference, 2015,
organized by India Foundation in association
with Sardar Patel University of Police, Security 35. In the word ‘=MATHEMATICS’, if M is
and Criminal Justice, was held in replaced by P, A by D, T by W and so on, then
(a) Jaipur (b) Jodhpur in the new arrangement of the letters, the letter
(c) Udaipur (d) Rawatbhata of the 9th place is
(a) K (b) L
Ans. (a) : The Center for Strategy and Security, Indian
Foundation in collaboration with Sardar Patel (c) F (d) W
University of Police, Security are Criminal Justice, Ans. (b) :
+3
Jodhpur organized the first counter Terrorism M  →P
conference 2015 at Jaipur. +3
A → D
2 T +3
→W
30. 40% of 50% of of 2000 will be
5 Hence MATHEMATICS should be written as
(a) 400 (b) 360 PDWKHPDWLFV
(c) 200 (d) 160 Hence the letter of the 9th place = L (from left)
2 40 50 36. The number at the question mark (?) in the box
Ans. (d) : = 2000 × × × is
5 100 100
= 160
31. The greatest fraction among
9 17 28 33
, , and is
13 26 39 52 (a) 40 (b) 32
9 17 (c) 36 (d) 42
(a) (b) Ans. (b) : Just as, (5 – 1) × 9 = 36
13 26
28 33 (7 – 1) × 6 = 36
(c) (d) Similarly, (9 – 1) × 4 = 32
39 52
37. The sum of the natural numbers which are
9 divisors of –100 is
Ans. (c) : = 0.692
13 (a) 116 (b) 117
17 (c) 216 (d) 217
= 0.654
26 Ans. (d) : Divisors of 100 are 1, 2, 4, 5, 10, 20, 25, 50,
28 100
= 0.718 Sum = 1 + 2 + 4 + 5 + 10 + 20 + 25 + 50 + 100 = 217
39
33 38. Interchanging + and ÷ and also the numbers 2
= 0.635 and 5, find the value of
52 8+4×5÷2–3
28 (a) 2 (b) 4
Hence is the greatest.
39 (c) 6 (d) 8
UP RO/ARO (Pre) Exam 2014 157 YCT
CLICK HERE FOR FREE MATERIAL

Ans. (c) : 8 + 4 × 5 ÷ 2 – 3 New Area = 180


After interchanging, 180 9
Hence required fraction = =
=8÷4×2+5–3 100 5
=4+5–3=6 43. If looking at a mirror the time in a
39. Ram walks 4 km north-west and then 3 km clock appears to be 30 minutes past 9,
south-west. How far is he from his starting then the actual time is
point? (a) 2:30 (b) 4:30
(a) 12 km (b) 7 km
(c) 6 km (d) 5 km (c) 6:30 (d) 6:10
Ans. (d) : Ans. (a) : Actual time = 12 : 00 – 9:30
= 11 : 60 – 9 : 30
= 2 : 30
44. What is the missing number in the
following box?

Required distance = 32 + 42 = 25 = 5km


40. The next term in the sequence
XBZ, VDY, RHW,? (a) 10 (b) 12 (c) 15 (d) 16
is Ans. (c) : Just as, 5 + 17 + 9 = 31,
(a) PLT (b) PJV
(c) PJT (d) PLV 3 + 21 + 7 = 31
Ans. (c) : and 7 + 16 + 8 = 31
same as, 6 + 10 + 15 = 31
45. If the average of 5, 10, 15 and X is 20,
what is the value if X?
(a) 20 (b) 25 (c) 45 (d) 50
sum of terms
41. By changing one letter at a time, in how many Ans. (d) : Average =
Number of terms
minimum steps, the word ‘LOCK’ can be
5 + 10 + 15 + x
changed to ‘BANK’? Each change should 20 =
result in a meaningful word. 4
(a) 6 (b) 5 80 = 30 + x
(c) 4 (d) 3 x = 50
Ans. (d) : 46. Which one amongst the following
Re place States has declared ‘house sparrow’ as
Step-1 LOCK O A LACK its State bird?
by
(a) Haryana (b) Punjab
Replace
Step-2 LACK L B BACK (c) Delhi (d) Rajasthan
by
Ans. (c) : The good old house sparrow, which
Step-3 BACK C
Re place
N BANK
is becoming rarer by the day has been declared
by State Bird of Delhi, by then Chief Minister
Hence, Total Step-3 Sheela Dixit as part of New campaign to save
42. If in a rectangular field the length is increased the species and enhance awareness about their
by 50% and breadth by 20%, its area will life and habitat.
become 47. Famous personality Yusuf Ali Kechery,
5 5 who passed away, was a famous
(a) times (b) times
9 3 (a) Telugu poet (b) Urdu poet
9 (c) Malayalam poet (d) Marathi poet
(c) times (d) 3 times
5 Ans. (c) : Yusufali Kechery was a post, film
Ans. (c) : Let Initial area = 100 lyricist, film producer and direct from Kerala.
xy He wrote during the modern era of Malayalam
Percentage change in Area of Rectangle = x + y +
100 Poetry and won the Odakkuzhal Award, the
= 50 + 20 +
50 × 20
= 80%
Kerala Sahitya Academy Award and the
100 Vallathol Award.
UP RO/ARO (Pre) Exam 2014 158 YCT
CLICK HERE FOR FREE MATERIAL

48. The book Indian Parliamentary 53. ‘Jan Rath’ is a name given
Diplomacy-Speaker's Perspective has (a) for expanding rural communication in UP
been written by (b) for boosting transportation facilities in the
country
(a) Meira Kumar
(c) by Indian Government to initiate bus service
(b) Sumitra Mahajan between India and Bangladesh
(c) B. S. Shakhawat (d) to low-cost AC bus service started by UP
(d) Somnath Chatterjee Government
Ans. (a) : Meira Kumar former Lok Sabha Ans. (d) : Jan Rath' , low-cost AC bus service started by
UP Government on the lines of Indian Railways Gareeb
Speaker is the author of "Indian Parliamentary Rath.
Diplomacy- Speaker's Perspective". The book 54. Which of the following statements is incorrect
features on account of the parliamentary diplomacy and for 2013?
its role in furthering foreign policy and strengthening (a) The birth rate (per 1000 population) in Uttar
relations. Pradesh was less than the birthrate in India
49. Famous personality Lia van Leer has passed (b) The birth rate (per 1000 population) in Uttar
away recently. She was Pradesh was more than the birthrate in India
(a) the pioneer of Italian cinema (c) The birth rate in Uttar Pradesh was more than 25
(b) the pioneer of Israeli cinema (d) The birth rate in Uttar Pradesh was less than 30
(c) the pioneer of American cinema Ans. (a) : As per Sample Registration System Bulletin
(d) a photographer 2013 published in September 2014. The birth rate of
Ans. (b) : Lia Van Leer, pioneer of Israeli cinema Uttar Pradesh on each 1000 population was 27.2 and
passed away on 13 march 2015 in Jerusalem, Israel She that of entire India was 21.4. Therefore, the birth rate in
was 90. She was the founder of Israel Film Archive, Uttar Pradesh was more than the birth rate of India.
Jerusalem Cinematheque, Haifa Cinematheque and the Hence, correct answer is option (a).
Jerusalem Film Festival. 55. UP Government has recently decided to
50. Who has written the book Shree Ramayana celebrate 2015-16 as
Mahanveshanam? (a) Yuva Year
(a) P. K. Verma (b) Vinod Mehta (b) Mahila Suraksha Year
(c) Shashi Tharoor (d) M. Veerappa Moily (c) Kisan Year
Ans. (d) : Dr. M. Veerappa Moily's Shree Ramayana (d) Environment Conservation Year
Mahanveshanam is a highly laudable work, which, Ans. (c) : While presenting the state budget 2015-16,
though rooted in the contemporary Indian. Indian context, Akhilesh Yadav, the then chief minister declared it to be
addresses universal concerns and presents a unique vision the 'KisanVarsh' of Uttar Pradesh, the "year of the farmer".
of past, present and future all clubbed into one. 56. In the Census of 2011 the highest literacy rate
51. What is a ‘Solar Impulse 2’? was in which district of UP?
(a) Solar-powered mobile phone (a) Ghaziabad (b) Mahoba
(b) Solar-powered missile (c) Kanpur (d) Varanasi
(c) Solar-powered submarine Ans. (c) : The highest literacy rate district of UP as per
(d) Solar-powered plane census 2011 is Gautam Buddha Nagar (80.12%) while
Ans. (d) : The Solar Impulse 2 has become the first as per question Kanpur Nagar (79.65%) literacy is at
aircraft to circle the globe using solar energy after second position.
landing in Abu Dhabi. In its journey the long-range 57. The largest share of revenue of UP
solar powered aircraft has travelled 26744 miles across Government comes from
four continents, three seas and two oceans since setting (a) registration fess
off from Abu Dhabi in March 2015. (b) trade tax
52. Which State has launched ‘Bhagyashree’ (c) land revenue
scheme for girl child? (d) share of Union excise duty
(a) Delhi (b) Punjab Ans. (b) : The largest share of revenue of UP
(c) Maharashtra (d) Haryana Government comes from trade tax.
Ans. (c) : On 8 March 2015, Maharashtra Government 58. In Uttar Pradesh, the 'Knowledge Park'
launched Bhagyashree scheme in place of the Sukanya (a) Varanasi (b) Noida
scheme which caters to girls from below poverty line (c) Lucknow (d) Greater Noida
(BPL) families. The state government was supposed to Ans. (d) : In Uttar Pradesh, the 'Knowledge Park' is
deposit an amount of Rs 21,200 in bank for a girl child being setup at Greater Noida.
born in a BPL family. The scheme was aimed at 59. The logo of 'Make in India' programme is
providing Rs 1 Lakh on maturity after the girl (a) Lion (b) Elephant
completes 18 years of age. (c) Panther (d) Kangaroo
UP RO/ARO (Pre) Exam 2014 159 YCT
CLICK HERE FOR FREE MATERIAL

Ans. (a) : Lion is the symbol of make in India. This Ans. (c) : On the night of 2 December, 1984, chemical,
logo was inspired by the Ashoka Chakra, to represent methyl isocyanate (MIC) spilt out from Union Carbide
India's success in all spheres. India pesticide factory turned the city of Bhopal into a
60. Which one of the following groups of colossal gas chamber. It was India's first major
compounds is called 'accessory dietary factor'? industrial disaster. At least 30 tonnes of methyl
(a) Fats (b) Hormones isocyanate gas killed more than 15,000 people and
effected ones 600,000 workers. Bhopal gas tragedy is
(c) Proteins (d) Vitamins known as world's worst industrial disaster.
Ans. (d) : Vitamins are groups of compounds called as 67. The chemical used as fixer in the photography is
accessory dietary factor. (a) sodium thiousulphate
61. Which one of the following is biodegradable? (b) borax
(a) Plastic (b) Polythene (c) sodium tetrathionate
(c) Mercury (d) Rubber (d) ammonium molybdate
Ans. (d) : Natural rubber is biodegradable Natural Ans. (a) : Photographic fixer is a mix of chemicals used
rubber decomposes in well-aerated moist soils ones a in the final step in the photographic processing of film
period of at least one year. or paper. By fixation the film or paper is insentive to
Synthetic rubber cannot be used as fuel because it further action by light. Popular salts are sodium
contains additives that might react with oxygen during thiosulfate and ammonium thiosulfate.
burning. It can however, be burned safely to dispose of 68. Percentagewise which gas is the most abundant
it in controlled conditions. in the atmosphere?
62. Which one of the following gases is responsible (a) Nitrogen (b) Oxygen
for decrement in ozone layer? (c) Hydrogen (d) Carbon dioxide
(a) Nitrous oxide Ans. (a) :The most abundant gas in the Earth's
(b) Chlorofluorocarbon atmosphere is nitrogen. The Earth's atmosphere is
(c) Carbon dioxide composed of approximately 78% nitrogen, 21% oxygen
(d) Carbon monoxide 1% argon and trace amounts of other gases that include
Ans. (b) : Chlorofluorocarbons (CFCS) and other carbon dioxide and neon.
halogenated ozone-depleting substances (ODS) are 69. Which one of the following proteins is present
mainly responsible for man made chemical ozone in human hair and nails?
depletion. While other are greenhouse gasses. Hence, (a) Oxytocin (b) Keratin
the correct answer is option (b) (c) Vasopressin (d) Trypsin
63. The gas used for inflating the tyres of Ans. (b) : Keratinocytes make Keratin a type of protein
aeroplanes is that's a basic component of hair, skin and nails. Keratin in
(a) hydrogen (b) helium the skin' s outer layer helps create a protective barrier.
(c) nitrogen (d) neon 70. Which one of the following salts should not be
Ans. (c) : Aircraft tyres are usually inflated with used as dining table salt?
nitrogen to minimize expansion and contraction from (a) lodized salt (b) Sea salt
extreme changes in ambient temperature and pressure (c) Rock salt (d) Processed common salt
experienced during flight. Only nitrogen is used since it Ans. (c) : Both rock salt and table salt have the same
absorbs moisture and is appropriate for high pressure chemical formula, NaCl , and both dissolve in water.
and temperature conditions. The main difference between them it that rock salt
64. An essential constituent of diet is granules are larger so they don't dissolve as fast. Another
(a) starch (b) glucose problem with rock salt is that it is unrefined and may
contain insolvable impurities and may contain insoluble
(c) carbohydrate (d) cellulose
impurities. Hence the correct answer is option (c).
Ans. (c) : Carbohydrates are the Chief source of energy 71. Natural camphor is obtained from
in our diet. They are the Chemical Compound
(a) high altitude rocks if Himalayan mountains
containing Carbon, Hydrogen and Oxygen. Source of
Carbohydrates are- Rice, Wheat, Maize, beetroot etc. (b) foams of Angel Falls in Venezuela
(c) a tree native to China and Japan
65. 'Red ink' is prepared from
(d) sediments of tides of sea near Andaman
(a) phenol (b) aniline
Ans. (c) : Camphor oil is extracted from the camphor
(c) Congo red (d) eosin
tree cinnamomum camphora which is indigenous to
Ans. (d) : Dyes are basically organic compounds used Vietnam and an area extending from southern China to
to impart colour to papers, textile and leather etc. Red southern Japan.
ink is also prepared from one of the dye named eosin 72. Cow milk is yellowish white in colour due to
Hence, the correct option in (d). the presence of
66. The compound associated with Bhopal Gas (a) casein
tragedy was (b) lactose
(a) methyl alcohal (b) phosphazene (c) carotene together with casein
(c) methyl isocyanate (d) methylamine (d) butyric acid together with lactose
UP RO/ARO (Pre) Exam 2014 160 YCT
CLICK HERE FOR FREE MATERIAL

Ans. (c) : Cow milk is light yellow in color due to 78. 'Gopatha Brahmana' is associated with
presence of carotene. Buffalo milk is white is color due (a) Yajurveda (b) Samveda
to casein (absence of carotene). Cow milk contains 3% (c) Atharvaveda (d) Rigveda
casein white buffalo milk contains 4.3% casein. Ans. (c) : The Gopatha Brahmana is a genre of the prose
73. Consider the following statements: texts describing the Vedic rituals, associated with the
Assertion (A): Atharavaveda. The text is associated with both the
During sleep, bright light is undesirable. Shaunaka and the paippalada recension of the
Reason (A): Atharavaveda. It is considered that this Brahmana is
associated with both Shaunaka and paippalada Shakha of
In bright light, production of melatonin the Atharvaveda.
in the body is considerably increased.
79. Originally the Mahabharata was known as
Select your answer using the codes given below.
(a) Brihatkatha (b) Brahmanas
Codes: (c) Brihatsamhita (d) Jayasamhita
(a) Both (A) and (R) are correct, and (R) is the
correct explanation of (A) Ans. (d) : The Mahabharata in its original version was
called Jayasamhita and it was writter down by Ganesha
(b) Both (A) and (R) are correct, but (R) is not
and narrated by Maharishi ved vyasa. The original epic
the correct explanation of (A)
had only 8800 shlokas and was called Jaya. Then it was
(c) (A) is true but (R) is false called Vijaya, then Bharata and finally Mahabharata.
(d) (A) is false but (R) is true
80. From which inscription it is known that
Ans. (a) : During sleep bright light is undesirables is Skandagupta defeated Hunas?
correct statement. (a) Bhitari Pillar Inscription
In bright light production of melatonin in the body is (b) Allahabad Pillar Inscription
considerably increased is correct and is also a correct (c) Mandsaur Inscription
explanation of Assertion (A). Hence the correct answer (d) Udayagiri Inscription
is option (a)
Ans. (a) : Skandagupta ruled the Gupta Empire from 455-
74. Who among the following was given the title of 467 AD. Skandagupa's Bhitari pillar imcription was
'Shah Buland Iqbal' by Shahjahan? discovered in Bhitari, Saidpur, Ghazipur, Uttar Pradesh
(a) Dara Shikoh (b) Shuja According to this inscription Skandagupta defeated the
(c) Aurangzeb (d) Murad Huns. The Huns are the Central Asian Tribe who invaded
Ans. (b) : Dara Shikoh has been given the title of Sultan from the North - West in the middle of the 5th century AD.
Buland Iqbal or Shahzarda-e-Buland Iqba by Shah 81. Vallabhi University was situated in
Jahan. He was the eldest son of mughal emperor shah (a) Bihar (b) Uttar Pradesh
Jahan. He was appointed as Governor of Multan and (c) Bengal (d) Gujarat
Kabul in 1652. He was defeated by Aurangzeb and Ans. (d) : Vallabhi University was situated in Gujarat.
executed in 1659 on his order. Vallabhi is also known as Vallabhipura and was the
75. Who were the 'Nayanars'? capital of the ancient maitraka dynasty. It is one of the
(a) Vaishnavites (b) Shaivites most important centers of Buddhist learning.
(c) Shaktas (d) Sun worshippers 82. Which of the following works were authored by
Ans. (b) : Nayanars are shaivites. Nayanars were Harsha?
devoted to Lord Shiva and his avatars Nayanars along (1) Priyadarshika (2) Nagananda
with the alvars who are the worshiper of Vishnu, they (3) Harshacharita (4) Ratnavali
participated in the bhakti movement in the 6th - 8th Select your answer using the codes given below.
century. The Nayanars were from various backgrounds Codes:
including Brahmins, Harijan and nobles. (a) 1, 2, 3 and 4 (b) 1, 2 and 4
76. 'Saptaparni Cave' is situated in (c) 1, 2 and 3 (d) 2 and 3
(a) Sanchi (b) Nalanda Ans. (b) : Priyadarshika, Nagananda and Ratnavali are
(c) Rajgriha (d) Pavapuri books authored by Harsha while Harshacharita he also
Ans. (c) : Saptaparni cave is a Buddhist cave situated commissioned the writing of his biography, which was
on vaibhavgiri Hill in Rajgir, Bihar. It is also referred t as written by the court poet Banabhata. Therefore, The
Sapta Parni Gufa, literally seven-leaves-cave is an important correct answer is option (b).
Buddhist cave site as it is believed to be the place where 83. The largest and the most evolved rock-cut
Lord Buddha spent some time before his death. Chaitya hall of the Hinayana phase is situated at
77. 'Dwarasamudra' was the capital of which (a) Pitalkhora (b) Junnar
dynastic power? (c) Karle (d) Bedsa
(a) Ganga (b) Kakatiya Ans. (c) : The largest and the most evolved rock-cut
(c) Hoysala (d) Kadamba chaitya hall of the Hinayana phase is situated at karle.
Ans. (c) : Dwarasamudra is capital of Hoysala dynasty The main Chaityagriha is one among the largest in
who ruled in Karnataka area of south India between 10th India. Great Chaitya at karle, is the largest rock-cut
and 14th centuries. The year 1116 AD, the chaitya in India, measuring 45 meters (148 ft) long and
Dwarasamudra was officially declared as the capital up to 14 meters (46 ft) high.
was officially declared as the capital of Hoysala dynasty There is a temple of a local goddess right at the entrance
by the then king Vishnuvardhana. of the main chaitya of Goddess Elvira.
UP RO/ARO (Pre) Exam 2014 161 YCT
CLICK HERE FOR FREE MATERIAL

84. In whose Viceroyalty the 'Rowlatt Act' was Ans. (c) : Dadabhai Naoroji popularly known as the
passed? Grand Old Man of India. He is also called the
(a) Lord Hardinge II (b) Lord Reading Unofficial Ambassador of India and has played a huge
(c) Lord Chelmsford (d) Lord Minto II role in India gaining independence from the British.
Ans. (c) : Lord Chelmsford was the Viceroy when the 91. Who among the following was the President of
Rowlatt Act was passed. It was passed in 1919, despite the Indian Constituent Assembly?
opposition from Indians. The act authorized the (a) Dr. Rajendra Prasad
government to impression any suspect without trial and (b) Pt. Jawaharlal Nehru
conviction in the court of law. It enables the (c) Sardar Patel
government to supress the habeas corpus. It was
repealed by Viceroy Lord Reading. (d) Dr. B. R. Ambedkar
85. The Indian Independence Act, 1947 came into Ans. (a) : Dr. Sachchidananda Sinha was the first
force on elected chairman (temporary) of Constituent Assembly.
(a) July 4, 1947 (b) July 10, 1947 Later Dr. Rajendra Prasad was first elected as the
president.
(c) July 18, 1947 (d) August 14, 1947
Ans. (c) : The Indian Independence Act, 1947 received 92. When was newly adopted tricolour flag of
the royal assent and entered into force on 18th July freedom first hoisted?
1947. This Act put into action the Mountbatten Plan for (a) 31st December, 1928
the Independence and partition of India. (b) 31st December, 1929
86. Boycott of British goods was adopted as a (c) 31st December, 1930
national policy in (d) 31st December, 1931
(a) 1899 (b) 1901 Ans. (b) : The tricolour flag was unfurled for the first
(c) 1903 (d) 1905 time in the Lahore Congress Session of Indian National
Ans. (d) : In August 1905, at Calcutta Town hall a Congress on 31st December 1929. This session was
massive meeting was held and the formal proclamation presided by Jawaharlal Nehru who unfurled the flag of
of the swadeshi Movement was made. The message was India's independence on the bank of river Ravi in
propagated to boycott goods such as Manchester cloth Lahore.
and Liverpool Salt. 93. In Which State the Bhoodan Movement was
87. The strategy of 'Divide and Rule' was adopted first started?
by (a) Andhra Pradesh (b) Karnataka
(a) Lord Curzon (b) Lord Minto (c) Tamil Nadu (d) Uttar Pradesh
(c) Lord Dalhousie (d) Lord Wellesley Ans. (a) : Bhoodan Movement was started by Mahatma
Ans. (b) : The strategy of divide and rule was adopted Gandhi's disciple Vinoba Bhave in April 1951. It was
by Lord Minto. It intended to divide people into groups started from Pochampali village in Nalgonda district,
based on religion and turn them against each other. The Andhra Pradesh (now Telangana). It was a voluntary
conflict was majorly fostered between the Hindu and land reform movement in India.
Muslim Communities. 94. The Right to Equality is granted by 5 Articles
88. The headquarters of the Ghadar Party was in in the Indian Constitution. These are
(a) San Francisco (b) New York (a) Articles 13–17 (b) Articles 14–18
(c) Madras (d) Calcutta (c) Articles 15–19 (d) Articles 16–20
Ans. (a) : Ghadar party was founded in 1913 by Lala Ans. (b) : The Right to Equality (Article 14-18) is one
Har Dayal, Sohan Singh Bhakna, initially called as of the six fundamental rights which are guaranteed to
Pacific Coast Hindustan Association. It was the citizens by the Constitution of India. These Right to
headquartered at San Francisco, USA. It published its Equality articles ensures citizens, equal treatment before
newspaper called The Ghadar. the law and equal protection of law, equal opportunity
89. Who among the following was not an in public employment and prohibit discrimination and
outstanding leader of Militant Nationalism? untouchability which are the social evils.
(a) Gopal Krishna Gokhale 95. The framers of the Constitution of India
(b) Bipin Chandra Pal borrowed the concept of Fundamental Rights
(c) Lokmanya Tilak from the
(d) Lala Lajpat Rai (a) Constitution of USA
Ans. (a) : Gopal Krishna Gokhale was one of the leader (b) Constitution of Ireland
of moderates bloc. Bipin Chandra Pal, Lokmanya Tilak (c) Constitution of Canada
and Lala Lajpat Rai were leaders of militart nationalism (d) Constitution of USSR
in India. Hence, the correct answer is option (a) Ans. (a) : The framers of the Constitution of India
90. Who among the following is known by the title borrowed the concept of Fundamental Rights from the
of The Grand Old Man? constitution of USA.
(a) Khan Abdul Ghaffar Khan 96. The Sarkaria Commission was set up for
(b) W. C. Bannerjee reviewing the relations between the
(c) Dadabhai Naoroji (a) Prime Minister and President
(d) Motilal Nehru (b) Legislature and Executive
UP RO/ARO (Pre) Exam 2014 162 YCT
CLICK HERE FOR FREE MATERIAL

(c) Executive and Judiciary Ans. (b) : Judicial Activism signifies the proactive role
(d) Center and States of the judiciary in protecting the rights of citizens. In
Ans. (d) :The Sarkaria Commission was set up on 9th India the Supreme Court and the High Court are vested
June 1983 by the Central government of India. The with the power to examine the constitutionality of any
Sarkaria Commission's charter was to examine the law, and if such a law is found to be inconsistent with
relationship and balance of power between state and the provisions of the constitution, the court can declare
the law as unconstitutional.
central governments in the country and suggest changes
within the framework of constitution of India. The 101. In the Union Budget of 2015-16 which one of
commission was so named as it was headed by Justice the following pension schemes has been
Ranjit Singh Sarkaria. reintroduced?
(a) Ram Manohar Lohia Pension Yojana
97. In which Part of the Constitution of India (b) Pradhan Mantri Pension Yojana
relations between the Union and States have
(c) Atal Pension Yojana
been mentioned?
(d) Nirantar Pension Yojana
(a) Part X (b) Part XI
(c) Part XIV (d) Part XV Ans. (c) : Announced in the budget 2015-16, the Atal
Pension Yojana, is a central government scheme that
Ans. (b) : Part XI of the Indian Constitution describes after income security to individuals involved in the
relations between the Union and the States. unorganized sector, post their retirement. It was
98. If the President and the Vice President of India launched on 9 May, 2015. Atal pension yojana is
both have resigned, who will act as the President? administered by Pension Fund Regulatory and
(a) The Speaker of the Lok Sabha Development Authority (PFRDA).
(b) The Prime Minister 102. In the Union Budget of 2015-16, the FDI limit
(c) The Home Minister of equity in case of insurance companies has
(d) The Chief Justice of the Supreme Court been raised to
(a) 49% (b) 76%
Ans. (d) : Discharge of President's functions in certain (c) 81% (d) 99%
contingencies:
Ans. (a) :The 2015 amendments raised the FDI cap to
In the event of the Occurrence of vacancies in the 49% from 26% led to foreign capital inflows of more
offices of both the President and the Vice-President, by than Rs 26,000 crore into the Insurance sector.
reason in each case of death, resignation or removal or
103. What is 'Shadow Banking'?
otherwise, the Chief Justice of India or, in his absence,
the senior most judge of the Supreme Court of India (a) Outsourcing of banking work by a bank
available shall discharge the functions of the President (b) Financial transactions and other activities of
until a new President elected in accordance with the nonbanking financial intermediary
provisions of the constitution to fill the vacancy in the (c) Domestic banks' foreign operations
office of the President enters upon his office or a new Vice (d) Foreign bank operating in another country for
President so elected begins to act as president under article banking and other activities
65 of the Constitution, whichever is earlier. Ans. (b) : The Shadow Banking System is a group of
financial intermediaries which facilitate the creation of
99. Which of the following Directive Principles of credit across the global financial system. But whose
State Policy was added to the Constitution at a members are not subject to regulatory oversight. These
later date? companies are often known as non-bank financial
(a) Organization of Village Panchayats companies (NBFCs). The Shadow banking system also
(b) Prohibition of Cow Slaughter refers to unregulated activities by regulated institutions.
(c) Free Legal Aid 104. Which among the following are multinational
(d) Uniform Civil Code corporations working in India?
Ans. (c) : The 42nd amendment Act of 1976 added four (1) Hindustan Lever Limited
new Directive principle to the original list. They require (2) Samsung Corporation
the state, (3) Guest Keen and Williams Pharma
To secure opportunists for healthy development of (4) LG
children (Article 39). Select your answer using the codes given below.
To promote equal justice and to provide free legal aid to Code:
the poor (Article 39A). (a) 1, 2, 3 and 4 (b) 2 and 4
To take steps to secure the participation of workers in (c) 2, 3 and 4 (d) 1 and 3
the management of Industries (Article 43A). Ans. (b) : Of the given options, Samsung Corporation
To protect and improve the environment and to and LG is the Multinational Corporation (MNC)
safeguard forests and wildlife (Article 48A). working in India.
100. 'Judicial Activism' in India is related to 105. The density of population in India
(a) Committed Judiciary (a) has continuously increased
(b) Public Interest Petition (b) is approximately constant
(c) Judicial Review (c) has mildly declined
(d) Judicial Independence (d) has first increased and then declined since 1991
UP RO/ARO (Pre) Exam 2014 163 YCT
CLICK HERE FOR FREE MATERIAL

Ans. (a) : Density of population, is expressed as Ans. (b) : Central potato research Institute is situated in
number of persons per unit area. It helps in getting a shimla (H . P). It's seven regional research stations are
better understanding of the spatial distribution of located in different potato producing zones such as-
population in relation to land. The density of population Kufri faagu (H. P), Modipuram (Meerut, U.P),
in India (2011) is 382 persons per Sq.km. There has Jalandhar (Punjab). Gwalior (M.P), Patna (Bihar),
been a steady increase of more than 200 persons per Sq. shillong (Meghalaya) and Ooty (T.N).
km over the last 50 years as the density of population
increased from 117 persons / Sq. km in 1951 to 382 112. Which one of the following is the most
persons Sq.km in 2011. productive soil in India?
106. Which one of the following decades in Indian (a) Red soil (b) Black soil
Planning deserves to be called 'the decade of (c) Alluvial soil (d) Calcareous soil
jobless growth'? Ans. (c) : It is highly fertile and mostly available soil in
(a) 1971–1980 (b) 1991–2000 India (about 43%). Mostly found in northern plain and
(c) 2001–2010 (d) 2005–2015 river valleys while in deltas and estuaries in Peninsular
Ans. (b) : The prospects of a jobless growth economy India. e.g: Indus - Ganga - Brahmaputra plain Narmada
has ramifications for everyone. An economy that is - Tapi plain etc.
growing without showing concomitant growth in the They are Rich in Humus, potash, lime and organic
number of jobs challenges investors, employees, and matters, But poor in phosphorous.
industries to adapt to the new economic order when 113. Generally Orobanche weed is found in
growth is coupled with high unemployment, it means
that the economy experiencing structural changes. (a) tabacoo field (b) gram field
Employment increase of approximately 1% per year (c) rice field (d) wheat filed
from 1994 to 2000 earned this period (1991-2000) the Ans. (a) : Orobanche is popularly known as broomrape
epithet of a decade of jobless growth. in english speaking countries. Tokra in North Indian. It
107. The folk dance of Meghalaya is is a flowering parasite on tobacco roots and occurs site
(a) Nati (b) Loho and draws its nourishment from tobacco by means of
(c) Bamboo dance (d) Khantum haustoria attached to the roots of tobacco.
Ans. (b) : Loho dance belongs to the state of 114. The highest protein content is found in the
Meghalaya in India. Both men and women take part in grain of
this dance performance. It is performed during the (a) black gram (b) pigeon pea
Behdienkhlam festival. (c) field pea (d) soya bean
108. Who was awarded Dada Saheb Phalke Award Ans. (a) : Black gram hold 25.21 gm or 45% of
for the year 2014? recommended daily values of protein. Though their
(a) Gulzar (b) Shashi Kapoor caloric value is comparatively lesser, the beans carry
(c) Pran (d) Lata Mangeshkar relatively high protein composition than Pigeon pea,
Ans. (b) : Dada Sahab Phalke Award, highest award in field pea and Soya bean.
cinema is presented Annually at the National Film 115. Which one of the following is not included in
Awards ceremony by the directorate of film festivals. the working capital of a farm?
The award was first presented in 1969. Shashi Kapoor
was 2014. Pran received award in 2012, Dilip Kumar in (a) Seed (b) Fertilizer
1994 and Amitabh Bachchan in 2018. (c) Irrigation water (d) Land revenue
109. UP Council of Agricultural Research is located Ans. (d) : Working capital is the liquid funds that a
at farm operation has available to meet short - term
(a) Meerut (b) Bulandshahar financial obligations. It is the difference in value
(c) Lucknow (d) Gorakhpur between current assets and current liabilities. Farmers
Ans. (c) : The Uttar Pradesh Council of Agricultural cannot reinvest in their crops without working capital.
Research is (UPCAR) was established on June 14, 1989 So, it clear that land revenue is not included in the
with its headquarter at Lucknow. working capital of a farm.
110. Central Institute of Medicinal and Aromatic 116. The proper seed rate for transplanting of
Plants is located at Basmati rice is
(a) Chitrakut (b) Saharanpur (a) 45–50 kg/ha (b) 20–30 kg/ha
(c) Kanpur (d) Lucknow (c) 15–20 kg/ha (d) 5–10 kg/ha
Ans. (d) : The Central Institute of Medicinal and Ans. (c) : Seed Supply and Seed rate :
Aromatic Plant is situated in Lucknow, Uttar Pradesh. It Farmers generally sow their own seed or procure from
is a frontier plant research laboratory of the Council of private seed agencies, Govt- agencies and from other
Scientific and Industrial Research (CSIR). farmers. The percent breakup of different seed sources
It was established originally as a Central Indian Medicinal is as follows:
Plants Organisation (CIMO) in the year 1959. It has been amassed that all of the farmers use less seed
111. Potato Research Institute in UP is located in than recommended, which is 20 kg seed for sowing
the district nursery for transplanting one ha field. Almost 85%
(a) Farrukhabad (b) Meerut farmers use 12-14 kg and 15% use 8-10 kg/ha seed rate
(c) Kanpur (d) Allahabad for transplanting of Basmati Rice.
UP RO/ARO (Pre) Exam 2014 164 YCT
CLICK HERE FOR FREE MATERIAL

117. Devaluation of 'Mudra' 124. A 'Pan' has initially a five-alphabet series like
(a) increases the imports and exports of a country AFZPK 7190 K. Here P stands for
(b) decreases the imports and exports if a country (a) individual
(c) increases the exports and decreases the (b) firm
imports of a country
(c) Hindu undivided family
(d) increases the imports and decreases the
exports of a country (d) association of persons
Ans. (c) : Due to devaluation of 'Mudra', there is an Ans. (a) : PAN is a ten digit unique alphanumeric
increase in the export and decreases the imports of a number issued by the Income Tax Department. Here is
country. There is a typically a better balance of given Example AFZPK 7190 K, the fourth character of
payments because the trade deficit shrinks. In short, a PAN represents the status of the PAN holder i.e. "P"
country that devolves its currency can reduce its deficit Stands for Individual.
because there is greater demand for cheaper exports.
125. In 2011, micro-finance institutions were
118. Which one of the following NBFCs has been installed in the recommendation of
granted licence by the RBI to open commercial
bank by March 31, 2015? (a) Malegam Committee
(a) Reliance Capital (b) Bajaj Capital (b) Goiporia Committee
(c) IDFC (d) IFCI (c) Rangarajan Committee
Ans. (c) : IDFC (earlier NBFC) has been granted license (d) Banking Reforms Committee
by the RBI to open commercial bank by 31 March, 2015. Ans. (a) : Regulatory approach towards microfinance
119. Long-term fiscal policy was announced by has been largely based on the recommendations of the
which Finance Minister of India? Malegam Committee. The key recommendation of the
(a) V. P. Singh (b) P. Chidambaram committee was as follows.
(c) Dr. Manmohan Singh (d) Yashwant Sinha Creation of Seperate Category of NBFC operating in the
Ans. (a) : Finance Minister, Vishwanath Pratap Singh has microfinance Sector.
announced the long term fiscal policy. He made budgetary Criteria for defining 'microfinance loans' classified as
history by introducing a long term fiscal policy and 'qualifying assets'.
through this policy. Government revenue and expenditure
was framed with a long term framework. During 1987-88, Prudential norms on Capital adequacy and provisioning
the resource projections have been announced and this requirements.
projection was made with three months advance. Establishment of a proper System of grievance redressal.
120. Which one of the following is a direct tax? Measure to address multiple lending, over-borrowing
(a) Sales tax (b) Excise duty me and coercive methods of recovery.
(c) Customs duty (d) Wealth tax Transparency in interest charges as well as other terms
Ans. (d) : Wealth tax was a charge levied on the total or and conditions of the loan.
market value of personal assets. Also known as capital tax Prescription related to pricing of credit in terms of a
or equity tax. The wealth tax was not a part of the income margin cap and interest rate celling on individual loans.
tax return (ITR) and was a form of direct tax required to be 126. Which one of the following is true for 'Pradhan
paid separately at the end of each financial year.
Mantri Jan-Dhan Yojana?
121. RBI does not transact the business of which
State Government? Accident Insurance Life Insurance
(a) Nagaland (b) Jammu and Kashmir (Rs) (Rs)
(c) Punjab (d) Assam (a) 1,00,000 30,000
Ans. (b) : When the question was asked. Reserve Bank (b) 30,000 1,00,000
of India (RBI) does not transact the business with (c) 30,000 10,000
Jammu and Kashmir. Now, the status has been changed. (d) 1,30,000 1,00,000
122. Income tax in India was introduced by Ans. (a) : Pradhan Mantri Jan-Dhan Yojana (PMJDY)
(a) Sir Charles Wood (b) Lord MacHale is National Mission for Financial Inclusion to ensure
(c) James Wilson (d) Assam access to financial Services which has a life cover of Rs
Ans. (c) : James Wilson, the Scotsman who created 30,000/- for those subscribed to a bank account with a
India's first Budget, introduced the income tax act in 1860. Rupay debit card before 26th January 2015 to
123. What is the maximum limit of gratuity payable complement the Rs 1 lakh accident insurance cover.
under the Payment of Gratuity Act, 1972? 127. In India the effective literacy rates are ..........
(a) Rs 3 lakh (b) Rs 7.50 lakh from
(c) Rs 10 lakh (d) Rs 10.50 lakh (a) ....... population
Ans. (c) : Gratuity is a type of after service payment to (b) child population
the employers who have fulfilled certain conditions of (c) adult population
the job. According to the payment of the Gratuity Act,
1972 the maximum amount can be paid to a employer is (d) population above 7 years of age
10 lakh rupees. A Recently approved amendment by the Ans. (d) : Effective Literacy Rate is calculated by
centre has increased the maximum limit of gratuity, dividing the number of literate persons aged 7 and above
now it is tax exempt up to Rs 20 lakh. by the population aged 7 and above, multiplied by 100.
UP RO/ARO (Pre) Exam 2014 165 YCT
CLICK HERE FOR FREE MATERIAL

128. Which of the following is an act that disturbs (a) Increasing use of high quality seeds
the ecological balance? (b) Development of agro-service centres
(a) Lumbering (b) Social forestry (c) Agro-credit facility
(c) Vanamahotsav (d) Afforestation (d) Kisan Credit Card Scheme
Ans. (a) : Lumbering is process by which wood is Ans. (a) : Quality seeds must be considered as a major
selected, faller and transported from the forest to the component in the package of inputs required to improve
sawmill where the logs are cut into planks for the crop production and as a catalyst for exploitation of other
making of all wood products. In this way human technological improvements in crop production. Quality
interference with the natural resources of nature exploit seed increases production of crops by about 20%.
or disturb the ecological balance. Response of other inputs in crop production largely
129. 'Humus' is an example of depends on the type of seed material used. Better the
(a) soil structure (b) crystalloids seed, better will be the crop produce.
(c) organic colloids (d) None of the above 135. As per the Census of 2011, what is the
Ans. (c) : Humus is an example of organic colloids. percentage of urban population in the total
Organic Colloids are tiny particle of organic matter that population of India?
can not decay further or that are resistant to decay. (a) 31.8 percent (b) 30.8 percent
130. Xeric condition refers to (c) 32.2 percent (d) 31.2 percent
(a) low temperature Ans. (*) : The level of urbanization increased from
(b) low humidity 27.81% in the 2001 census to 31.6% in the 2011 census,
while the proportion of rural population decline from
(c) high evaporation
72.19% to 68.84%.
(d) extreme temperature
136. UP Government has decided to have metro rail
Ans. (b) : Xeric environment means dry conditions or in various cities. Which of the following cities
low atmospheric humidity and lack of precipitation. Also does not find place in the list?
the temperature variations depends on several other (a) Kanpur (b) Allahabad
variables like altitude, latitude and hemisphere. A high rate (c) Agra (d) Varanasi
of vaporization is not a characteristic of environment but a
consequence of low atmospheric humidity. Ans. (b) : When the question was asked, Allahabad is
right answer. As now its name is in the list of proposed
131. Ecology is the study of relationship between metro project.
(a) organism and environment
137. Which State is the largest producer of coal in
(b) man and forest India?
(c) soil and water (a) West Bengal (b) Odisha
(d) husband and wife (c) Andhra Pradesh (d) Madhya Pradesh
Ans. (a) : Ecology is the study of the relationships Ans. (*) : According to recent data Chhattisgarh is
between living organism, including humans and their highest producer of Coal and Jharkhand has the highest
physical environment, it seeks to understand the vital reserve is India.
connections between plants and animals and the world
around them. 138. The second largest continent in the world (in
terms if area) is
132. Soil erosion can be prevented by (a) Asia (b) South America
(a) heavy rains (b) deforestation (c) North America (d) Africa
(c) afforestation (d) overgrazing
Ans. (d) : Asia is the second largest continent in the
Ans. (c) : Soil erosion can be prevented through world. (in terms of area).
afforestation as it is a process of planting new trees and
establishment of a forest in an area where there were no 139. In which State the Aravalli ranges are located?
tress previously. Trees prevent soil erosion by (a) Rajasthan (b) Himachal Pradesh
strengthening the soil. (c) Odisha (d) Andhra Pradesh
133. The boundary of Lucknow district does not Ans. (a) : The Aravalli Range is a mountain Range in
touch the boundary of which of the following Northwestern India, the length of the range is
districts? approximately 692 km in a south-west direction.
(a) Ballia (b) Unnao The Range is starting near Delhi, passing through
(c) Hardoi (d) Barabanki southern Haryana and Rajasthan and ending in Gujarat
Its highest peak is Gurushikhar (1722 m) situated in
Ans. (a) : Lucknow is bounded on the east by
Mount Abu.
Barabanki district, on the west by Unnao and Hardoi
district, on the south by Raebareli district and in the 140. Which river forms its delta in Odisha?
north by Sitapur district. Ballia lies in the extreme (a) Godavari (b) Mahanadi
eastern part of the Uttar Pradesh state. So, option (a) is (c) Narmada (d) Tapti
the right answer. Ans. (b) : The Mahanadi is a major river in east central
134. Which of the following factors is most effective India. It drains an area of around 1,32,100 Sq.km and has a
in the increase of agricultural production in total length of 900 km. Mahanadi is also known for the
Uttar Pradesh? Hirakud Dam. Its delta lies in the state of Odisha.
UP RO/ARO (Pre) Exam 2014 166 YCT
CLICK HERE FOR FREE MATERIAL

Gòej ØeosMe meceer#ee DeefOekeâejer/meneÙekeâ meceer#ee DeefOekeâeje (Øeerefuecme) hejer#ee, 2014


meeceevÙe efnvoer
nue ØeMve-he$e
7. keâewve-mee Meyo `YeÇcej' keâe heÙee&ÙeJeeÛeer veneR nw?
heÙee&ÙeJeeÛeer (a) MeueYe (b) ÛebÛejerkeâ (c) efMeueercegKe (d) efceefuevo
1. keâewve-mee Meyo `IeesÌ[e' keâe heÙee&ÙeJeeÛeer veneR nw? Gòej-(a)
(a) yeeefpe (b) legjbie (c) Meeot&ue (d) nÙe JÙeeKÙee – ‘MeueYe’ Yeücej keâe heÙee&heJeeÛeer Meyo veneR nw~ Yeücej kesâ
Gòej-(c) heÙee&ÙeJeeÛeer nw– ÛebÛejerkeâ, Deefue, efceefuevo, efMeueercegKe, meejbie FlÙeeefo
JÙeeKÙee – Meeot&ue Meyo IeesÌ[e keâe heÙee&ÙeJeeÛeer veneR nw, peyeefkeâ nw~ peyeefkeâ ‘MeueYe’ keâe heÙee&Ùe heeflebiee neslee nw~
yeeefpe, legjbie Deewj nÙe IeesÌ[e kesâ heÙee&ÙeJeeÛeer Meyo nQ~ Meeot&ue keâe heÙee&Ùe 8. keâewve-mee Meyo `Fvõ' keâe heÙee&ÙeJeeÛeer veneR nw?
efmebn neslee nw~ Meeot&ue kesâ DevÙe heÙee&ÙeJeeÛeer Meyo nQ- ce=iejepe, kesâMeer, (a) hegjboj (b) Me›eâ (c) ceOeJee (d) ieCeeefOehe
veenj, kesânjer Deeefo~ Gòej-(d)
2. `ceervee#eer' keâe heÙee&ÙeJeeÛeer Meyo nw – JÙeeKÙee – Gòeâ efJekeâuheeW ceW Fvõ keâe heÙee&ÙeJeeÛeer ieCeeefOehe veneR nw,
(a) megvojer (b) ogiee& (c) ceÚueer (d) ue#ceer peyeefkeâ DevÙe leerveeW efJekeâuhe hegjboj, Me›eâ Deewj ceOeJee Fvõ kesâ
Gòej-(a & b) heÙee&ÙeJeeÛeer nQ~ ieCeeefOehe, ieCesMe keâe heÙee&ÙeJeeÛeer nw~
JÙeeKÙee – ceervee#eer keâe heÙee&ÙeJeeÛeer Meyo megvojer leLee ogiee& oesveeW 9. `efnjCÙeieYe&' keâe heÙee&ÙeJeeÛeer Meyo nw –
neslee nw~ ceervee#eer keâe DeeMeÙe ceerve keâer lejn DeebKe Jeeueer neslee nw~ (a) efJe<Ceg (b) yeÇÿee (c) censMe (d) ieCesMe
veesš - DeeÙeesie ves Fmekeâe Gòej (a) Deewj (b) DeLee&le megvojer Deewj ogiee& Gòej-(b)
oesveeW keâes ceevee nw~ JÙeeKÙee – efnjCÙeieYe& keâe heÙee&ÙeJeeÛeer Meyo yeÇÿee nw~ yeÇÿee kesâ DevÙe
3. keâewve-mee Meyo `veeie' keâe heÙee&ÙeJeeÛeer veneR nw? heÙee&ÙeJeeÛeer Meyo nw – efheleecen, mJeÙebYet, Ûelegjeveve, œe°e, Øepeeheefle,
(a) mehe& (b) Deefn (c) efJe<eOej (d) legjbie keâceueemeve, DeelceYet~ efJe<Ceg keâe heÙee&Ùe DeÛÙegle, peveeo&ve, Ûe›eâheeefCe,
Gòej-(d) cegkegâvo, veejeÙeCe, ie®Ì[OJepe Deeefo nw~ censMe keâe heÙee&Ùe efMeJe,
JÙeeKÙee – veeie keâe heÙee&ÙeJeeÛeer Meyo legjbie veneR nw~ legjbie keâe heÙee&Ùe Gceeheefle, ceneosJe, ef$ehegjeefj, osJeeefOeosJe, ceoveeefj, Ûevõceewefue Deeefo nw~
IeesÌ[e neslee nw~ legjbie kesâ DevÙe heÙee&Ùe–DeMJe, Ieesškeâ, nÙe, Jeeefpe, ieCesMe keâe heÙee&Ùe nsjcye, efJeIveefJeveeMekeâ, ieCeheefle, uecyeesoj, Skeâovle,
jefJemegle, meQOeJe Deeefo~ peyeefkeâ mehe&, Deefn Deewj efJe<eOej veeie kesâ heÙee&Ùe efJeveeÙekeâ, iepeeveve Deeefo nw~ veesš–DeeÙeesie ves Fme ØeMve keâe Gòej (a)
nQ~ Deewj (b) DeLee&led yeÇÿee SJeb efJe<Ceg oesveeW ceevee nw~
10. keâewve-mee Meyo `oeme' keâe heÙee&ÙeJeeÛeer veneR nw?
4. keâewve-mee Meyo `owlÙe' keâe heÙee&ÙeJeeÛeer veneR nw?
(a) DevegÛej (b) heefjkeâj (c) Ye=lÙe (d) mesJekeâ
(a) je#eme (b) oeveJe (c) Yetmegj (d) efveMeeÛej
Gòej-(*)
Gòej-(c)
JÙeeKÙee – Gòeâ efJekeâuheeW ceW oeme keâe heÙee&ÙeJeeÛeer heefjkeâj Yeer neslee nw,
JÙeeKÙee – efoÙes ieÙes efJekeâuheeW ceW owlÙe keâe heÙee&Ùe Yetmegj veneR neslee nw,
peyeefkeâ DevÙe leerveeW DevegÛej, Ye=lÙe Deewj mesJekeâ oeme kesâ heÙee&ÙeJeeÛeer nQ~
peyeefkeâ DevÙe leerveeW efJekeâuhe owlÙe kesâ heÙee&Ùe nQ~ Yegmetj keâe heÙee&Ùe yeÇeÿeCe
veesš – Fme ØeMve keâes DeeÙeesie ves nše efoÙee nw~
neslee nw~ Yetmej kesâ DevÙe heÙee&ÙeJeeÛeer Meyo–efÉpe, cenermegj, YetosJe, efJeØe
Deeefo nQ~ efJeueesce
5. `®Ke' keâe heÙee&ÙeJeeÛeer Meyo nw –
11. `pebiece' keâe efJeueesce Meyo nw –
(a) efJešhe (b) Øemetve (c) leÌ[keâe (d) nsjcye
(a) Deiece (b) ogie&ce (c) mLeeJej (d) ÛebÛeue
Gòej-(a)
Gòej-(c)
JÙeeKÙee - ®Ke keâe heÙee&ÙeJeeÛeer Meyo efJešhe neslee nw~ ®Ke keâe DevÙe JÙeeKÙee – ‘pebiece’ keâe efJeueesce Meyo ‘mLeeJej’ neslee nw~ ‘ogie&ce’ keâe
heÙee&ÙeJeeÛeer Meyo nw – Je=#e, hesÌ[, õgce, heeohe, efJešhe Deeefo~ nsjcye efJeueesce megiece neslee nw, ‘Deiece’ keâe efJeueesce ‘megiece’ leLee ‘ÛebÛeue’ keâe
ieCesMe keâe heÙee&ÙeJeeÛeer Meyo nw~ ‘efmLej’ neslee nw~
veesš – Fme ØeMve keâes DeeÙeesie ves nše efoÙee nw~ 12. `me=ef„' keâe efJeueesce Meyo nw –
6. keâewve-mee Meyo `yeÇÿee' keâe heÙee&ÙeJeeÛeer veneR nw? (a) efJeme=ef„ (b) ØeueÙe (c) JÙeef„ (d) meceef„
(a) keâceueemeve (b) Ûelegjeveve (c) Ûelegceg&Ke (d) ÛelegYeg&pe Gòej-(b)
Gòej-(d) JÙeeKÙee – me=ef° keâe efJeueesce ‘ØeueÙe’ neslee nw peyeefkeâ JÙeef° keâe
JÙeeKÙee – Gòeâ efJekeâuheeW ceW yeÇÿee keâe heÙee&ÙeJeeÛeer ÛelegYeg&pe veneR nw~ efJeueesce meceef° neslee nw~ Mes<e efJekeâuhe Demebiele nw~
ÛelegYeg&pe keâe heÙee&Ùe efJe<Ceg neslee nw~ efJe<Ceg kesâ DevÙe heÙee&ÙeJeeÛeer Meyo 13. `F&eqhmele' keâe efJeueesce Meyo nw –
nQ – veejeÙeCe, nefj, Ûe›eâheeefCe, cegkegâvo, peveeo&ve, ieesefJevo, ue#ceerheefle (a) DeefYeeqhmele (b) Deveereqhmele
Deeefo~ yeÇÿee keâe heÙee&ÙeJeeÛeer Meyo nw–keâceueemeve, ÛelegYeg&pe, efJejbefÛe, (c) hejeseqhmele (d) megveereqhmele
efheleecen, Ûelegjeveve, Ûelegceg&Ke, Øepeeheefle, mJeÙebYet Deeefo~ Gòej-(b)
UP RO/ARO (Pre) General Hindi, 2014 167 YCT
CLICK HERE FOR FREE MATERIAL

JÙeeKÙee – ‘F&efhmele’ keâe efJeueesce ‘Deveerefhmele’ neslee nw~ Mes<e efJekeâuhe 22. efvecveefueefKele JeekeäÙeeW ceW Megæ JeekeäÙe nw –
leke&âmebiele veneR nQ~ (a) ßeerke=â<Ce kesâ DeveskeâeW veece nQ~
14. `meenÛeÙe&' keâe efJeueesce Meyo nw – (b) YeieJeeve ßeerke=â<Ce kesâ DeveskeâeW veece keâe GuuesKe efceuelee nw~
(c) ßeerke=â<Ce keâes DeveskeâeW veeceeW mes hegkeâeje peelee nw~
(a) JewcevemÙe (b) DemenÙeesie
(d) ßeerke=â<Ce kesâ Deveskeâ veece nQ~
(c) efJeefveÙeesie (d) DeueieeJe
Gòej-(d) Gòej-(d)
JÙeeKÙee – ‘meenÛeÙe&’ keâe efJeueesce ‘DeueieeJe’ neslee nw~ ‘DemenÙeesie’ JÙeeKÙee – efoÙes ieÙes ÛeejeW efJekeâuheeW ceW Megæ JeekeäÙe nw – ßeerke=â<Ce kesâ
Deveskeâ veece nQ~
keâe efJeueesce ‘menÙeesie’ neslee nw~ ‘JewcevemÙe’ keâe efJeueesce ‘meewcevemÙe’ neslee nw~
23. efvecveefueefKele JeekeäÙeeW ceW Megæ JeekeäÙe nw –
veesš- DeeÙeesie ves Fme ØeMve keâe Gòej efJekeâuhe (a) ceevee nw~
(a) osKees! HetâueeW hej YeeQjs efYeveefYevee jns nQ~
15. `efJeješd' keâe efJeueesce Meyo nw –
(b) osKees! HetâueeW hej YeeQjs iegbpeej keâj jns nQ~
(a) Je=nod (b) Je=nled (c) Úesšeheve (d) #egõ (c) osKees! HetâueeW kesâ Thej YeeQjs iegbpeejles nQ~
Gòej-(d) (d) osKees! HetâueeW kesâ Thej YeeQjs efYeveefYevee jns nQ~
JÙeeKÙee – ‘efJeješd’ keâe efJeueesce ‘#egõ’ neslee nw~ ‘Je=nled’ keâe efJeueesce Gòej-(b)
‘ueIeg’ SJeb ‘Úesšeheve’ keâe ‘yeÌ[eheve’ neslee nw~ JÙeeKÙee – efoÙes ieÙes ÛeejeW efJekeâuheeW ceW Megæ JeekeäÙe nw – osKees!
16. `mhe=MÙe' keâe efJeueesce Meyo nw – HetâueeW hej YeeQjs iegbpeej keâj jns nQ~
(a) mhe=mÙe (b) Demhe=mÙe 24. efvecveefueefKele JeekeäÙeeW ceW Megæ JeekeäÙe nw –
(c) DeMhe=<Ùe (d) Demhe=MÙe (a) Gmekeâer Mebkeâe keâe efveJeejCe nes ieÙee nw~
Gòej-(d) (b) Gmekeâer Mebkeâe meceehle nes ieÙeer nw~
JÙeeKÙee – ‘mhe=MÙe’ keâe efJeueesce ‘Demhe=MÙe’ neslee nw~ Mes<e efJekeâuhe (c) Gmekeâer Mebkeâe keâe meceeOeeve nes ieÙee nw~
Demebiele nQ~ (d) GvnW Deye Mebkeâe veneR jner nw~
Gòej-(c)
17. `De%e' keâe efJeueesce Meyo nw –
(a) efJe%e (b) Ùe%e (c) meJe&%e (d) Deve%e
JÙeeKÙee – Gòeâ ÛeejeW efJekeâuheeW ceW mener efJekeâuhe nw- Gmekeâer Mebkeâe keâe
Gòej-(a) meceeOeeve nes ieÙee nw~
veesš – DeeÙeesie ves Fme ØeMve keâe Gòej (a) Deewj (c) oesveeW ceevee nw~
JÙeeKÙee – ‘De%e’ keâe efJeueesce Meyo ‘efJe%e’ nw~ ‘meJe&%e’ keâe efJeueesce
25. Megæ `mebÙegkeäle JeekeäÙe' keâe GoenjCe nw –
‘Deuhe%e’ neslee nw~ Mes<e efJekeâuhe Demebiele nQ~
(a) ceQ hejer#ee osves kesâ efueS Fueeneyeeo pee jne ntB~
18. `ieewjJe' keâe efJeueesce Meyo nw – (b) cegPes hejer#ee osveer nw, Dele: efouueer pee jne ntB~
(a) ueeIeJe (b) ueIeglJe (c) ueIeglee (d) ueIeglece (c) cegPes hejer#ee osveer nw Deewj ceQ ieesjKehegj pee jne ntB~
Gòej-(a) (d) ceQ JeejeCemeer pee jne ntB keäÙeeWefkeâ cegPes hejer#ee osveer nw~
JÙeeKÙee – ‘ieewjJe’ keâe efJeueesce ‘ueeIeJe’ neslee nw~ ‘ueIeg’ keâe efJeueesce Gòej-(c)
‘oerIe&’ neslee nw~ Fmeer Øekeâej ‘ueIeglee’ keâe ‘oerIe&lee’ neslee nw~ JÙeeKÙee – Gòeâ ÛeejeW efJekeâuheeW ceW mebÙegòeâ JeekeäÙe keâe mener GoenjCe
19. `yeefnjbie' keâe efJeueesce Meyo nw – nw- cegPes hejer#ee osveer nw Deewj ceQ ieesjKehegj pee jne ntB~
(a) meJee&” (b) Deblejbie (c) Ûelegjbie (d) DeYÙe%e 26. efvecveefueefKele ceW mes Megæ Jele&veer Jeeuee Meyo nw –
Gòej-(b) (a) keâues<e (b) keäues<e (c) keäuesMe (d) keäuesme
JÙeeKÙee–‘yeefnjbie’ keâe efJeueesce ‘Deblejbie’ nw~ Mes<e efJekeâuhe Demebiele nQ~ Gòej-(c)
20. `«emle' keâe efJeueesce Meyo nw – JÙeeKÙee – efoÙes ieÙes efJekeâuheeW ceW Megæ Jele&veer Jeeuee Meyo nw - keäuesMe
(a) meghle (b) «ee¢e (c) cegkeäle (d) ueghle 27. efvecveefueefKele ceW mes Megæ Jele&veer Jeeuee Meyo nw –
Gòej-(c) (a) ße=bieej (b) ßebieej (c) ëe=bieej (d) me=bieej
JÙeeKÙee – ‘«emle’ keâe efJeueesce ‘cegòeâ’ nw~ ‘«ee¢e’ keâe efJeueesce Gòej-(c)
‘De«ee¢e’ Deewj ‘ueghle’ keâe efJeueesce ‘Øekeâš’ neslee nw~ Fmeer Øekeâej ‘cegòeâ’ JÙeeKÙee – efoÙes ieÙes efJekeâuheeW ceW Megæ Jele&veer Jeeuee Meyo nw– ëe=bieej
keâe efJeueesce ‘yeæ’ neslee nw~ 28. efvecveefueefKele ceW mes Megæ Jele&veer Jeeuee Meyo nw –
(a) GppeJeue (b) GppJeue (c) GlpeJeue (d) GlpeJeue
JeekeäÙe SJeb Jele&veer DeMegefæÙeeB Gòej-(b)
21. efvecveefueefKele JeekeäÙeeW ceW Megæ JeekeäÙe nw – JÙeeKÙee – efoÙes ieÙes efJekeâuheeW ceW Megæ Jele&veer Jeeuee Meyo nw –
(a) cesjs Iej kesâ heeme Skeâ nueJeeF& keâer ogkeâeve nw~ GppJeue~ Mes<e meYeer Meyo DeMegæ nQ~
(b) cesjs Iej kesâ heeme Skeâ nueJeeF& keâer ogkeâeve eqmLele nw~ 29. efvecveefueefKele ceW mes Megæ Jele&veer Jeeuee Meyo nw –
(c) cesjs Iej kesâ heeme Skeâ nueJeeF&ÙeeW keâer ogkeâeve nw~ (a) efnjCÙekeâMÙeheg (b) efnjCÙekeâefMeheg
(d) cesjs Iej kesâ heeme nueJeeF& keâer Skeâ ogkeâeve nw~ (c) efnjCÙekeâMÙehe (d) efnjCÙekeâmÙehe
Gòej-(d) Gòej-(b)
JÙeeKÙee – Gòeâ efoÙes ieÙes ÛeejeW JeekeäÙeeW ceW Megæ JeekeäÙe nw – cesjs Iej JÙeeKÙee–efoÙes ieÙes efJekeâuheeW ceW Megæ Jele&veer Jeeuee Meyo nw –
kesâ heeme nueJeeF& keâer Skeâ ogkeâeve nw~ efnjCÙekeâefMeheg~
UP RO/ARO (Pre) General Hindi, 2014 168 YCT
CLICK HERE FOR FREE MATERIAL

30. efvecveefueefKele ceW mes Megæ Jele&veer Jeeuee Meyo nw – JÙeeKÙee – ‘efpemekesâ ùoÙe hej DeeIeele ngDee nes’ JeekeäÙeebMe kesâ efueS
(a) keâJeefÙe$eer (b) keâefJe$eer (c) keâefJeefÙe$eer (d) keâefJeÙe$eer GheÙegòeâ Meyo nesiee – cecee&nle~
Gòej-(a) 38. `pees Deheves heo mes nšeÙee ieÙee nes' kesâ efueS Skeâ GheÙegkeäle
JÙeeKÙee – efoÙes ieÙes efJekeâuheeW ceW Megæ Jele&veer Jeeuee Meyo nw – Meyo nesiee –
keâJeefÙe$eer~ Mes<e meYeer Meyo DeMegæ nQ~ (a) heoYeÇ„ (b) heoeveJele (c) heoevegiele (d) heoÛÙegle
Gòej-(d)
Deveskeâ MeyoeW kesâ Skeâ Meyo JÙeeKÙee – ‘pees Deheves heo mes nšeÙee ieÙee nes’ kesâ efueS Skeâ GheÙegòeâ
31. `keâefveef‰keâe Deewj ceOÙecee kesâ yeerÛe keâer DeBiegueer' keâes keânles nQ – Meyo nesiee – heoÛÙegle, pees Deheves heo keâe og®heÙeesie keâjlee nw Gmes
(a) Deveeceer (b) Deefveefcekeâe heoYeü° keâne peelee nw~
(c) Deveeceerkeâe (d) Deveeefcekeâe 39. `efpemes efkeâmeer Jemleg keâer mhe=ne ve nes' kesâ efueS GheÙegkeäle Meyo
Gòej-(d) nesiee –
JÙeeKÙee – keâefveef‰keâe Deewj ceOÙecee kesâ yeerÛe keâer DeBiegueer keâes (a) efve:mhe=ne (b) efve:mhe=n
Deveeefcekeâe keânles nQ~ (c) efvemhe=n (d) efvemhe=nerve
32. `efpemekesâ hesš ceW ceeB ves jmmeer (oece) yeeBOe oer nes', Gmes keânles nQ – Gòej-(b)
(a) oeceeo (b) oeceeo Flej
JÙeeKÙee – ‘efpemes efkeâmeer Jemleg keâer mhe=ne ve nes’ kesâ efueS GheÙegòeâ
(c) oece (d) oeceesoj
Gòej-(d) Meyo nesiee – efve:mhe=n~ veesš – DeeÙeesie ves Fme ØeMve keâe Gòej (b)
Deewj (c) DeLee&led efve:mhe=n SJeb efvemhe=n oesveeW keâes ceevee nw~
JÙeeKÙee – ‘efpemekesâ hesš ceW ceeB ves jmmeer (oece) yeeBOe oer nes’ Gmes
40. `efpemekeâer hetJe& mes keâesF& DeeMee ve nes' JeekeäÙeebMe kesâ efueS
keânles nQ – oeceesoj~
33. `heerÚs-heerÚs Ûeueves Jeeuee' JeekeäÙeebMe kesâ efueS Skeâ Meyo nw –
GheÙegkeäle Meyo nesiee –
(a) ØelÙeeefMele (b) DeØelÙeeefmele
(a) DevegÛej (b) Devegieeceer
(c) DevegJeleea (d) DevegieceveerÙe (c) DeØelÙeeef<ele (d) DeØelÙeeefMele
Gòej-(b) Gòej-(d)
JÙeeKÙee – ‘heerÚs-heerÚs Ûeueves Jeeuee’ JeekeäÙeebMe kesâ efueS Skeâ Meyo nw JÙeeKÙee – ‘efpemekeâer hetJe& mes keâesF& DeeMee ve nes’ JeekeäÙeebMe kesâ efueS
– Devegieeceer~ veesš – DeeÙeesie ves Fme ØeMve keâe Gòej (a) Deewj (b) GheÙegòeâ Meyo nesiee – DeØelÙeeefMele~
DeLee&led Devegieeceer Deewj DevegÛej oesveeW ceevee nw~ lelmece SJeb leodYeJe
34. `efpemekeâer keâesF& keâercele ve nes mekesâ' JeekeäÙeebMe kesâ efueS Skeâ
Meyo nw – 41. `cekeäKeer' Meyo keâe lelmece ¤he nw –
(a) keâerceleer (b) DecetuÙe (a) ceeqÛÚkeâe (b) ceeÚer
(c) yengcetuÙe (d) GheÙeg&keäle ceW mes keâesF& veneR (c) ceÛÚer (d) ceef#ekeâe
Gòej-(b) Gòej-(d)
JÙeeKÙee – ‘efpemekeâer keâesF& keâercele ve nes mekesâ’ Skeâ Meyo nw-DecetuÙe~ JÙeeKÙee – ‘cekeäKeer’ Meyo keâe lelmece ™he nw – ceef#ekeâe
pees yengle keâerceleer nes, Gmes yengcetuÙe keâne peelee nw~ veesš- DeeÙeesie ves Fme ØeMve keâe Gòej efJekeâuhe (a) ceevee nw~
35. `LeesÌ[e vehee-leguee Yeespeve keâjves Jeeuee' JeekeäÙeebMe kesâ efueS 42. `nuoer' Meyo keâe lelmece ¤he nw –
GheÙegkeäle Meyo nw – (a) njefõkeâe (b) njerõe (c) nefjõe (d) nefueõe
(a) efceleJÙeÙeer (b) efceleJÙeÙe (c) efceleenejer (d) efcelenejerve Gòej-(c)
Gòej-(c)
JÙeeKÙee – ‘nuoer’ Meyo keâe lelmece ™he nw – nefjõe~ Mes<e efJekeâuhe
JÙeeKÙee – ‘LeesÌ[e vehee-leguee Yeespeve keâjves Jeeuee’ JeekeäÙeebMe kesâ efueS leke&âmebiele veneR nw~
GheÙegòeâ Meyo nw – efceleenejer, peyeefkeâ ‘keâce KeÛe& keâjves Jeeuee’ kesâ efueS
43. `ngueeme' Meyo keâe lelmece ¤he nw –
Skeâ Meyo – ‘efceleJÙeÙeer’ neslee nw~
(a) efnueeme (b) efJeueeme
36. `ieg® kesâ meceerhe jnkeâj DeOÙeÙeve keâjves Jeeuee' kesâ efueS Skeâ
(c) nemÙe (d) Guueeme
Meyo nw –
(a) efMe<Ùe (b) DeeßeceJeemeer
Gòej-(d)
(c) efJeÅeeLeea (d) DevlesJeemeer JÙeeKÙee – ‘ngueeme’ Meyo keâe lelmece ™he nw – Guueeme~ nBmeer keâe
Gòej-(d) lelmece ‘nemÙe’ neslee nw~
JÙeeKÙee – ‘ieg® kesâ meceerhe jnkeâj DeOÙeÙeve keâjves Jeeuee’ kesâ efueS 44. `veeefjÙeue' Meyo keâe lelmece ¤he nw –
Skeâ Meyo nw – DevlesJeemeer~ Fmeer Øekeâej pees efJeÅee keâes Ûeenves Jeeuee nes, (a) veeefjkesâue (b) veeefjkesâefue
Jen efJeÅeeLeea nw~ ieg® keâe DevegkeâjCe keâjves Jeeuee efMe<Ùe neslee nw~ (c) veejerkesâue (d) veeefjkesâuee
37. `efpemekesâ ùoÙe hej DeeIeele ngDee nes' JeekeäÙeebMe kesâ efueS Gòej-(a)
GheÙegkeäle Meyo nesiee – JÙeeKÙee – ‘veeefjÙeue’ Meyo keâe lelmece ™he ‘veeefjkesâue’ neslee nw~
(a) cecee&efnle (b) cecee&nle (c) cecee&ngle (d) cecee&ntle veesš – DeeÙeesie ves Fme ØeMve keâe Gòej (a) Deewj (c) DeLee&led veeefjkesâue
Gòej-(b) SJeb veejerkesâue oesveeW ceevee nw~
UP RO/ARO (Pre) General Hindi, 2014 169 YCT
CLICK HERE FOR FREE MATERIAL

45. `meeRie' Meyo keâe lelmece ¤he nw – 54.efvecveefueefKele ceW mes keâewve-mee Meyo efJeMes<Ùe nw?
(a) ße=bie (b) eEMeie (c) ëe=bie (d) eEmeie (a) Deveemeeqkeäle (b) Deveemekeäle
Gòej-(c) (c) DevegMebefmele (d) Deheceeefvele
JÙeeKÙee – ‘meeRie’ Meyo keâe lelmece ™he nw – ëe=bie~ Mes<e efJekeâuhe Gòej-(a)
Demebiele nw~ JÙeeKÙee – Gòeâ efJekeâuheeW ceW efJeMes<Ùe Meyo Deveemeefòeâ nw~ Ùen meb%ee
46. efvecveefueefKele ceW mes keâewve-mee Meyo leodYeJe veneR nw? Meyo nw, peyeefkeâ Deveemeòeâ Meyo Fmekeâe efJeMes<eCe nw~
55. efvecveefueefKele ceW mes keâewve-mee Meyo efJeMes<Ùe nw?
(a) ogDeej (b) meeÙeb (c) efieefjmleer (d) heeBJe
(a) heewef„keâ (b) hee"keâerÙe
Gòej-(b)
(c) YeeJegkeâ (d) efJe<eeo
JÙeeKÙee – efoÙes ieÙes efJekeâuheeW ceW leodYeJe Meyo meeÙeb veneR nw~ peyeefkeâ Gòej-(d)
ogDeej, efieefjmleer SJeb heeBJe leodYeJe Meyo nQ~ JÙeeKÙee – efoÙes ieÙes efJekeâuheeW ceW efJeMes<Ùe Meyo efJe<eeo nw, Fmekeâe
47. `Skeâue' Meyo keâe leodYeJe ¤he nw – efJeMes<eCe efJe<eCCe nw; peyeefkeâ heewef°keâ, YeeJegkeâ Deewj hee"keâerÙe efJeMes<eCe
(a) Dekeâue (b) Deefkeâue Meyo nQ~
(c) Dekesâuee (d) GheÙeg&keäle ceW mes keâesF& veneR 56. `OegBOeuee' Meyo ceW efJeMes<eCe nw –
Gòej-(c) (a) mebKÙeeJeeÛekeâ efJeMes<eCe (b) iegCeJeeÛekeâ efJeMes<eCe
JÙeeKÙee – ‘Skeâue’ Meyo keâe leodYeJe ™he nw – Dekesâuee~ (c) meeJe&veeefcekeâ efJeMes<eCe (d) GheÙeg&keäle ceW mes keâesF& veneR
Gòej-(b)
48. `keâhe&š' Meyo keâe leodYeJe ¤he nw –
JÙeeKÙee – ‘OegBOeuee’ Meyo ceW iegCeJeeÛekeâ efJeMes<eCe nw~ iegCeJeeÛekeâ
(a) keâšjvee (b) keâheeš (c) keâheÌ[e (d) keâheš
efJeMes<eCe kesâ DevÙe GoenjCe nQ– keâeuee, mehesâo, ceesše, heleuee, iejeryeer,
Gòej-(c) Keóe, ceer"e Deeefo~
JÙeeKÙee – ‘keâhe&š’ Meyo keâe leodYeJe ™he nw – keâheÌ[e~ 57. `mejerKee' Meyo ceW efJeMes<eCe nw –
49. `kewâJeòe&' Meyo keâe leodYeJe ¤he nw – (a) ieCeveeJeeÛekeâ efJeMes<eCe (b) iegCeJeeÛekeâ efJeMes<eCe
(a) ceuueen (b) kesâJeš (c) veeefJekeâ (d) kesâJeue (c) mecegoeÙeJeeÛekeâ efJeMes<eCe (d) ›eâceJeeÛekeâ efJeMes<eCe
Gòej-(b) Gòej-(b)
JÙeeKÙee – ‘kewâJeòe&’ Meyo keâe leodYeJe ™he nw - kesâJeš~ JÙeeKÙee–‘mejerKee’ ceW ‘iegCeJeeÛekeâ efJeMes<eCe’ nw~ iegCeJeeÛekeâ
50. `ie=OeÇ' Meyo keâe leodYeJe ¤he nw –
efJeMes<eCe–efpeme efJeMes<eCe mes efkeâmeer meb%ee Ùee meJe&veece keâe iegCe-oes<e,
¤he-jbie, Deekeâej-Øekeâej, mecyevOe, oMee Deeefo keâe helee Ûeues, Gmes
(a) ieerOevee (b) ie=Oe (c) ieerOeer (d) ieerOe
iegCeJeeÛekeâ efJeMes<eCe keânles nQ~
Gòej-(d)
58. ``Jen veewkeâj veneR DeeÙee~'' JeekeäÙe ceW `Jen' keâewve-mee efJeMes<eCe nw?
JÙeeKÙee – ‘ie=OeÇ’ Meyo keâe leodYeJe ™he ‘ieerOe’ neslee nw~ (a) meeJe&veeefcekeâ efJeMes<eCe (b) iegCeJeeÛekeâ efJeMes<eCe
(c) mebKÙeeJeeÛekeâ efJeMes<eCe (d) heefjceeCeyeesOekeâ efJeMes<eCe
efJeMes<Ùe SJeb efJeMes<eCe Gòej-(a)
51. efJeMes<eCe efpeme meb%ee keâer efJeMes<elee yeleelee nw, Gmes keäÙee JÙeeKÙee – Gòeâ ØeMve ceW ‘Jen’ meeJe&veeefcekeâ efJeMes<eCe nw~ meb%ee kesâ
keânles nQ? yeoues pees Meyo Deelee nw, Gmes meJe&veece keânles nQ~ pewmes- Jen, Jes, ceQ,
(a) mebKÙeeJeeÛekeâ efJeMes<eCe (b) iegCeJeeÛekeâ efJeMes<eCe legce Deeefo, peyeefkeâ pees meJe&veece Meyo meb%ee kesâ henues Deekeâj meb%ee keâer
(c) efJeMes<Ùe (d) meeJe&veeefcekeâ efJeMes<eCe efJeMes<elee yeleueeles nQ, Gmes meeJe&veeefcekeâ efJeMes<eCe keânles nQ~
Gòej-(c) 59. `leermeje' Meyo ceW efJeMes<eCe nw –
(a) hetCeeËkeâyeesOekeâ efJeMes<eCe (b) DeeJe=efòeJeeÛekeâ efJeMes<eCe
JÙeeKÙee – efJeMes<eCe efpeme meb%ee keâer efJeMes<elee yeleelee nw Gmes efJeMes<Ùe
(c) ieCeveeJeeÛekeâ efJeMes<eCe (d) ›eâceJeeÛekeâ efJeMes<eCe
keânles nQ~ pees Meyo meb%ee Ùee meJe&veece keâer efJeMes<elee yeleueelee nw Gmes Gòej-(d)
efJeMes<eCe keânles nQ~ JÙeeKÙee – ‘leermeje’ Meyo ceW ›eâceJeeÛekeâ efJeMes<eCe nw~ GoenjCeeLe&-
52. efvecveefueefKele ceW mes keâewve-mee Meyo efJeMes<Ùe nw? henuee, otmeje, leermeje Deeefo, peyeefkeâ ieCeveeJeeÛekeâ efJeMes<eCe Skeâ, oes,
(a) Deemeerve (b) Deeqive (c) ceOegj (d) keâce&" leerve, Ûeej Deeefo neslee nw SJeb DeeJe=efòe JeeÛekeâ efJeMes<eCe ogiegvee, leeriegvee,
Gòej-(b) Ûeewiegvee Deeefo neslee nw~
JÙeeKÙee – Gòeâ ØeMveebleie&le mener efJekeâuhe Deefive nw~ 60. `ÛeewLeeF&' Meyo keâe efJeMes<eCe nw –
53. ``"C[e heeveer "C[ hewoe keâjlee nw~'' Fme JeekeäÙe ceW keâewve-mee (a) DeeJe=efòeJeeÛekeâ efJeMes<eCe (b) ieCeveeJeeÛekeâ efJeMes<eCe
(c) ›eâceJeeÛekeâ efJeMes<eCe (d) DehetCeeËkeâyeesOekeâ efJeMes<eCe
Meyo efJeMes<Ùe nw?
Gòej-(d)
(a) "C[e (b) "C[ (c) heeveer (d) GheÙeg&keäle ceW mes keâesF& veneR
JÙeeKÙee – ‘ÛeewLeeF&’ Meyo ceW ‘DehetCeeËkeâyeesOekeâ efJeMes<eCe’ nw~
Gòej-(c)
‘DehetCeeËkeâyeesOekeâ efJeMes<eCe’–efpememes hetCe& mebKÙee kesâ efkeâmeer Skeâ
JÙeeKÙee – "C[e heeveer "C[ hewoe keâjlee nw, ceW efJeMes<Ùe Meyo heeveer Yeeie keâe yeesOe nes, Gmes DehetCeeËkeâyeesOekeâ keâne peelee nw~ pewmes–heewves oes,
nw; peyeefkeâ "C[e Meyo efJeMes<eCe nw~ meeÌ{s ome, ÛeewLeeF&, efleneF& Deeefo~
UP RO/ARO (Pre) General Hindi, 2014 170 YCT
CLICK HERE FOR FREE MATERIAL

UPPSC RO-ARO (Mains) Exam-2013


GENERAL STUDIES
Solved Paper
1. Bone ornaments in the Mesolithic context in Ans. (b) : Porus was brave Indian king, defeated by
India have been obtained from: Alexander on the banks of Jhelum River. The Battle of
(a) Sarai Nahar Rai (b) Mahadaha the Hydaspes River was fought by Alexander the Great
(c) Lekhahia in 326 BC against King Porus of the Hindu Paurava
(d) Chopani Mando
kingdom on the banks of the Hydaspes River (Jhelum
Ans. (b) : Bone ornaments in the Mesolithic context in
India have been obtained from Mahadaha. A large River). Alexander defeated Porus but he was
number of implements made of bones and horns have impressed by his bravery and courage. Alexander
restored his kingdom to him and made him his ally.
been found from this Mesolithic site. Mahadaha is located
6.
in Patti tehsil in Pratapgarh district of Uttar Pradesh. Saka Calender Era begins from:
2. Which Harappan city was divided into three (a) 78 AD (b) 58 BC
parts? (c) 120 AD (d) 320AD
(a) Lothal (b) Kalibangan Ans. (a) : The 'Saka Era' begins from 78 AD. The
(c) Dholavira (d) Surkotda Government of India adopted it as Indian National
Calendar in 1957.
Ans. (c) : Dholavira is one of the most remarkable and
7.
well preserved urban settlements in South Asia. It was In which of the following sources mentioned
discovered in 1968 by archaeologist Jagat Pati Joshi. It that there was no slavery in ancient India?
is located on Khadir bet island in Great Rann of (a) Arthshastra (b) Mudrarakshasa
Kachchh, Gujarat. The city was divided into 3 parts (c) Indica of Megasthenes (d) Vayupuran
different from other sites of Harappan civilization as
Ans. (c) : In ‘Indica’, it is mentioned that there was no
they were divided into 2 parts. slavery in ancient India. Indica was written by
3. In the Vedic rituals, HOTA is associated with Megasthenes, the Greek ambassador to India around
the: 300 BC. Megasthenes was an ancient Greek historian,
(a) Rigveda (b) Yajurveda diplomat and Indian ethnographer and explorer. He
(c) Samaveda (d) Atharvaveda was an ambassador of Seleucus at the Mauryan court
in Pataliputra. Indian Caste system as conceived by
Ans. (a) : In the Vedic rituals, HOTA is associated with
Megasthenes consisted of seven different castes.
the Rig Veda. Hota is the main Rigvedic priest and his
assistants are Maitravaruna, Acchavaka, Gravastut. 8. Which one of the following Tamil Texts has
Each of the four Vedas is assigned a specific Hindu been called ‘Laghuveda’?
priest. Hota for Rig Veda, Adhvaryu for Yajur Veda, (a) Nandikalambakam (b) Kalingattuparni
Udgata for Sama Veda and Brahma for Atharva Veda. (c) Periya puranam (d) Kural
4. Fourth Buddhist council held in reign of Ans. (d) : In tamil texts, Kural has been called
Emperor: ‘Laghuveda’. Thiruvalluvar was the author of the book
(a) Kanishka (b) Asoka ‘Thirukkural (also known as the Kural). It is a classical
(c) Harshvardhan (d) Minander Tamil sangam literature consisting of 1330 couplets
organized into 133 chapters. The book is also called as
Ans. (a) : The fourth Buddhist Council was the fifth Veda or ‘Bible of the Tamil Land’.
convened under the patronage of Emperor 9. During the rule of the Chola kings, which of the
Kanishka in Kundalvan, Kashmir in 72 AD. following Variyam looked after garden
Fourth Buddhist council was presided over by administration?
Vasumitra and Asvaghosha. The agenda of this (a) Pon Variyam (b) Airi Variyam
Buddhist council was the reconciliation of various (c) Totta Variyam (d) Samwatsar Variyam
conflicts between different schools of thought. Ans. (c) : During the rule of the Chola kings, Totta
Hinayana and Mahayana sects of Buddhism Wariyam looked after garden administration. Cholas
diverged after this council. had good administrative system. They had a
5. Name that brave Indian king who was defeated centralised administrative system. A council of
by Alexander on the banks of Jhelum River. Ministers assisted the king. Elected representatives had
to work in the Annual Committee, Garden Committee
(a) Ambhik and Tank Bund Committee. These committees were
(b) Puru (Porus) called “Variyam”. The representatives were called
(c) Chandragupta Mourya “Variya Perumakkal”. These committees worked for
(d) Mahapadamanand 360 days. The village committees performed duties
UP RO/ARO (Mains) Exam 2013 171 YCT
CLICK HERE FOR FREE MATERIAL

like the protection of village property, collection of marriage. It had a limited impact as the Act was
taxes and the protection of temples, lakes, groves and applicable to only Indian Christians The relentless
forests etc. The resolutions of the Committee were efforts of a Parsi reformer, B.M. Malabari, were
written. The central administration did not interfere in rewarded by the enactment of the Age of Consent Act
the village administration unnecessarily. (1891) which forbade the marriage of girls below the
10. Firishtah, the author of Shah-nama was age of 12. In free India, the Child Marriage Restraint
associated with the court of: (Amendment) Act, 1978 raised the age of marriage for
(a) Subuktagin girls from 15 to 18 years and for boys from 18 to 21.]
(b) Mahmud Ghazanavi 15. Who was the governor general of India during
(c) Mohammad Ghori the first war of independence 1857?
(d) Alaptagin (a) Lord Harding (b) Lord Canning
(c) Lord Mayo (d) Lord Dufferin
Ans. (*) : Given question is wrong because Shah-
nama, was written by Abu Al-Qasim Firdausi, not Ans. (d) : Lord Charles Canning was the Governor
Farista. Abu Al-Qasim Firdausi was prominent courtier General of India during the first war of independence
poet of Mahmud Ghazanavi. Shah-nama is an epic, 1857. Lord Canning served as Governor General of
written in 1010 AD, containing about 60,000 couplets. India from 1856 to 1862. During his tenure, the
Government of India Act, 1858 was passed which
11. At the time of Aurangzeb's death, the Maratha created the office of Viceroy to be held by the same
leadership was in the hands of: person who was Governor General of India. Thus, Lord
(a) Shambha ji (b) Rajaram Canning also served as first Viceroy of India.
(c) Jijabai (d) Tarabai 16. Which of the following famines of the 19th
Ans. (d) : At the time of Aurangzeb's death, the century in India has been described as ‘Sea of
Maratha leadership was in the hands of Tarabai. After Calamity’?
death of Chhatrapati Rajaram, Tarabai took over the (a) Bihar Famine of 1873
reins of the Maratha kingdom as regent for her 4-year- (b) Rajputana Famine of 1868-69
old son, Shivaji II in the year 1705. (c) Madras Presidency Famine of 1866-67
12. Who among the following Viceroy of India (d) Orissa Famine of 1866-67
assassinated by a convict, when he was on tour
Ans. (b) : Orissa Famine of 1866-67 in India has been
of Andman-Nicobar?
described as ‘Sea of Calamity.’ It affected the east coast
(a) Lord Rippon (b) Lord Curzon of India. The impact of the famine, however, was
(c) Lord Mayo (d) Lord Minto greatest in Orissa, which at that time was quite isolated
Ans. (c) : Lord Mayo was Governor-General of India from the rest of India. In Orissa alone, a third of the
between 1869 and 1872, and assassinated in the population died in 1866 due to starvation and diseases.
Andaman. Lord Mayo was assassinated by Sher Ali 17. Who among the following was not a minister in
Afridi, an Afghan convict in 1872. Lord Mayo was the interim government formed on 2 September,
only Viceroy of India to be murdered in office. Lord 1946?
Mayo had supported the policy of Sir John Lawrence in (a) Jawaharlal Nehru
reference of Afganistan. (b) Liyaqat Ali
13. First elected European President of Indian (c) Abul Kalam Azad
National Congress was: (d) Dr. Rajendra Prasad
(a) A.O. Hume (b) George Yule
Ans. (c) : Abul Kalam Azad was not a minister in
(c) Alfred Webb (d) Annie Besant interim government formed on 2 September, 1946. The
Ans. (b) : First elected European President of Indian Interim Government was formed as a provisional
National Congress was George Yule. George Yule, a government between an imperial structure and a
Scottish merchant in England and India, who served as the democratic structure. This Interim Government was
fourth President of the Indian National Congress in 1888 at formed from the Constituent Assembly which was
Allahabad. He was the first non-Indian and European to elected in August 1946.
hold that office. He served as Sheriff of Calcutta and as 18. Who used first the word 'Swaraj'?
President of the Bengal Chamber of Commerce.
(a) Bal Gangadhar Tilak
14. Sharda Act had determined the minimum age (b) Lala Lajpat Rai
of girls and boys respectively as:
(c) Subhas Chandra Bose
(a) 12 and 16 years (b) 14 and 18 years
(d) Mahatma Gandhi
(c) 15 and 21 years (d) 16 and 22 years
Ans. (b) : The Child Marriage Restraint Act was a Ans. (a) : Among the given options, Bal Gangadhar Tilak
legislative act passed on 28 September 1929. The act used the word ‘Swaraj’ first time. Bal Gangadhar Tilak,
fixed the marriageable age for girls at 14 years and 18 extremist face of Congress, had announce his famous
years for boys. This act is popularly known as 'Sharda slogan- “Swaraj is my Birth Right and I shall have it”.
Act' on the name of its sponsor, famous educationist 20. Which was the only session of Indian National
Harvilas Sharda. Congress, presided by Mahatma Gandhi?
[Note- Native Marriage Act (or Civil Marriage Act), (a) Allahabad Session- 1921
1872 signified legislative action in prohibiting child (b) Gaya Session - 1922
UP RO/ARO (Mains) Exam 2013 172 YCT
CLICK HERE FOR FREE MATERIAL

(c) Belgaum Session- 1924 Ans. (a) : Subhas Chandra Bose announced the
(d) Karachi Session - 1930 establishment of the Provisional Government of Free
Ans. (c) : Belgaum Session- 1924(Karnataka) was the India on 21 October 1943 in Singapore. He had
only session of Indian National Congress, Presided by launched a struggle to free India from British rule under
Mahatma Gandhi. It was 39th session of Indian National the banner of the provisional government-in exile
Congress, held in December,1924. during the latter part of the Second World War.
21. Which one of the following pair is not correctly 25. Who is regarded as father of extreme Indian
matched? nationalism?
(a) Navjivan - M.K. Gandhi (a) Gopal Krishna Gokhale
(b) Swaraj - T. Prakasham (b) Bal Gangadhar Tilak
(c) Prabhat - N.C. Kelkar (c) Surendra Nath Banerjee
(d) Qaumi Awaz - Abul Kalam Azad (d) None of the above
Ans. (d) : Abul Kalam Azad was not associated with Ans. (b) : Bal Gangadhar Tilak is regarded as father of
Qaumi Awaz. 'The National Herald' newspaper was extreme Indian nationalism.
published by Jawaharlal Nehru from Lucknow. Hindi 26. Consider the following statements and Select
edition of this newspaper was known as 'Navjivan' correct answer by using the codes given below:
while urdu edition was named as 'Qaumi Awaz'. It is Assertion (A): The rivers of West Coast of
notable that Mahatma Gandhi had served as editor for India do not form the delta.
'Young India', 'Harijan' and 'Navjivan'. Rests are Reason (R): They flow through short courses
correctly matched. on relatively harder rocks.
22. Match List-I to List-II and select correct (a) (A) and (R) are true statements and (R) is
answer by using codes given below: correct explanation of (A).
List-I List-II (b) (A) and (R) are true statements but (R) is not
A. Bharat Mitra 1. Bengali correct explanation of (A).
B. Rashtramat 2. Gujarati (c) (A) is true but (R) is false.
(d) (R) is true but (A) is false.
C. Prajamitra 3. Hindi
Ans. (a) : The rivers of West Coast of India do not form
D. Nayak 4. Marathi the delta because of the high gradient and they don't
Codes have to travel much distance to drain into the sea as a
A B C D result they don't carry much sediments required to form
(a) 1 2 4 3 deltas. Thus, (A) and (R) are true statements and (R) is
(b) 2 3 1 4 correct explanation of (A).
(c) 3 4 2 1 27. The state of Chhattisgarh does not share
(d) 4 1 3 2 boundary with:
Ans. (c) : The correct match is as follows: (a) Maharashtra (b) Bihar
List – I List – II (c) Orisha (d) Andhra Pradesh
Bharat Mitra - Hindi Ans. (b) : The state of Chhattisgarh does not share
Rashtramata - Marathi boundary with Bihar. Chhattisgarh borders the states of
Prajamitra - Gujarati Madhya Pradesh in the northwest, Uttar Pradesh in the
Nayak - Bengali north, Jharkhand in northeast, Maharashtra in the
southwest, Telangana and Andhra Pradesh in the south,
23. In which of the following session of Indian and Orisha in the southeast.
National Congress, Mahatma Gandhi was 28. The origin source of Cauvery river is:
apprised of problems of Champaran peasants?
(a) Brahmagiri Hills (b) Sahyadri Hills
(a) Banaras Session 1905
(c) Gwaligarh Hills (d) Amarkantak Hills
(b) Calcutta Session 1906
(c) Surat Session 1907 Ans. (a) : Cauvery River rises at Talakaveri on the
Brahmagiri range in the Western Ghats in the state of
(d) Lucknow Session 1916
Karnataka. It flows for about 800 km before it falls into
Ans. (d) : In Lucknow Session 1916 of Indian National the Bay of Bengal at Poompuhar.
Congress, Mahatma Gandhi was apprised of problems
of Champaran peasants. In December 1916, Rajkumar 29. Gorakhpur is situated on the bank of which of
Shukla, a Champaran farmer no longer able to stand the the following rivers?
oppression, went to see Mahatma Gandhi at an Indian (a) Ghaghra (b) Gandak
National Congress meeting. Shukla insisted that Gandhi (c) Rapti (d) Sarayu
move a resolution condemning the situation and Ans. (c) : Gorakhpur is a city along the banks of the
treatment of Champaran tenant farmers. Rapti river in the north-eastern part of the state of Uttar
24. Subhash Chandra Bose had announced the Pradesh. Rapti is major tributary of Ghaghra.
establishment provisional government of 30. Govind Ballabh Pant Sagar Reservoir is
Independent India on: located in:
(a) 21 October, 1943 (b) 22 October, 1943 (a) Uttar Pradesh (b) Chhattisgarh
(c) 23 October, 1943 (d) 24 October, 1943 (c) Jharkhand (d) Uttarakhand
UP RO/ARO (Mains) Exam 2013 173 YCT
CLICK HERE FOR FREE MATERIAL

Ans. (a) : Govind Ballabh Pant Sagar Dam also known Codes
as Rihand dam, is located near Pipri village in A B C D
Sonbhadra district of Uttar Pradesh. Govind Ballabh (a) 3 4 1 2
Pant Sagar is the largest artificial reservoir in India. (b) 2 3 4 1
31. Which of the following state has been declared (c) 4 2 3 1
best in implementation of rural tourism projects? (d) 1 3 2 4
(a) Sikkim (b) Kerala Ans. (a) : The correct match is as follows:
(c) Goa (d) Assam List – I List – II
Ans. (a) : Sikkim has been declared as best state in the Monsoon Forest - Sal and Teak
country in implementation of rural tourism projects by Equatorial forest - Mahogany and Rosewood
the Union Ministry of Tourism in 2014. Mediterranean forest - Plum and Olive
32. Which of the following has not been listed in Conifer Forest - Pine and Fur
‘excellent towns’ for export till March,2014?
37. Which of the following country was favorite
(a) Panipat (b) Ludhiana
tourist destination (on base of numbers of
(c) Tripur (d) Madurai tourist) of Indian tourists in the year 2010?
Ans. (*) : (a) Australia
Towns of Export Excellence – Year of recognition (b) Singapore
Panipat – 2003 (c) United States America (U.S.A)
Ludhiana – 2003 (d) United Kingdom(U.K)
Tripur – 2003 Ans. (b) : Singapore was favorite tourist destination
Madurai – 2004 (on base of numbers of tourist) of Indian tourists in the
33. Which of the following is not correctly year 2010. As per report of Singapore Tourism Board, a
matched? total of 829,000 Indian tourists visited Singapore in
(a) Iron Ore - Kudremukh 2010. India was the fifth largest tourist generating
(b) Manganese - Koraput market for Singapore.
(c) Copper - Khetri 38. The idea of 'Folk-Urban Continuum ' has been
(d) Coal - Singreni developed that were studied in:
Ans. (b) : Koraput is known for bauxite production. (a) Mexico (b) Brazil
Orisha is one of the largest producers of Manganese, (c) Indonesia (d) India
here it is found mainly in Sundergarh district and Ans. (a) : The idea of 'Folk-Urban Continuum ' has
Keonjhar district. been developed by Robert Redfield in 1941 on the basis
34. Navgram Oilfield is located in: of his studies of Mexican Communities.
(a) Assam (b) Gujarat 39. The boundary of Zambia is not common with:
(c) Maharashtra (d) Andhra Pradesh (a) Namibia (b) Tanzania
Ans. (b) : Navgram Oilfield is located in Gujarat, India. (c) Angola (d) Botswana
Gujarat maintains a variety of industries such as Ans. (*) : Zambia is the landlocked country in Africa. It
agriculture, pharmaceutical, automobile, industrial share its borders with Democratic Republic of Congo at
production, oil and gas. Ankleshwar, Lunej, Kalol, north, Tanzania at northeast, Malawi at east,
Navgram, Kosamba, Kathana, Barkol, Mehsana, and Mozambique to the southeast, Zimbabwe and Botswana
Sanand are the important oil fields in the state of Gujarat. at south, Namibia at the southwest and Angola to the
35. Mahaweli Ganga is a river of: west.
(a) Bangladesh (b) India 40. Which one of the following statement is not
(c) Sri Lanka (d) Nepal correct about Jawaharlal Nehru National
Ans. (c) : The longest river in Sri Lanka is Mahaweli Urban Renewal Mission?
Ganga. It has a length of 335 kilometres and has a (a) It is started in 2005.
drainage basin that covers almost one-fifth of the island. (b) It is ten year programme.
The origin point of the river is found in Horton Plains (c) It was initiated for qualitative growth of life
National Park and the river ends in the Bay of Bengal. in Indian cities.
36. Match List-I to List-II and select correct (d) It was for inclusive growth.
answer by using codes given below: Ans. (d) : Jawahar Lal Nehru Urban Renewal Mission
List-I List-II was a big urban modernization scheme. It envisaged a
A. Monsoon Forest 1. Plum and Olive total investment of over $ 20 billion over seven years.
It started in 2005 with aim to improve quality of life in
B. Equatorial 2. Pine and Fur cities, and inclusive via efforts like development of
forest infrastructure, providing civic amenities.
C. Mediterranean 3. Sal and Teak 41. Which one of the following is largest district (in
forest terms of population) of Uttar Pradesh?
D. Conifer Forest 4. Mahogany and (a) Muradabad (b) Lucknow
Rosewood (c) Allahabad (d) Ghaziabad
UP RO/ARO (Mains) Exam 2013 174 YCT
CLICK HERE FOR FREE MATERIAL

Ans. (c) : Allahabad (Prayagraj) district is the most Ans. (c) : In census year 1951, sex ratio of India was
populous district in Uttar Pradesh. Its population as per 946.
the 2011 census is 59.54 lakhs. Top five populous Census Year Sex Ratio (India)
districts of Uttar Pradesh are as follow: 1921 955
Cities Population 1941 945
(a) Prayagraj 59,54,391 1951 946
(b) Moradabad 47,72,006 1971 930
(c) Ghaziabad 46,81,645 47. In duration of year 2001-2011, state with
(d) Azamgarh 46,13,913 minimal percentage growth in total population
(e) Lucknow 45,89,838 is:
42. Which of the following state has sex-ratio above (a) Nagaland (b) Goa
to national average? (c) Kerala (d) Tamil Nadu
(a) Gujarat (b) Jharkhand Ans. (a) : States/UTs with minimal decadal
(c) Maharashtra (d) Madhya Pradesh percentage growth in total population are as follow:
Ans. (b) : As per data of Census 2011, Jharkhand States/UTs Percentage of
(948) has sex-ratio above to national average(943). population growth rate
Sex-ratio of other states are as follow: Nagaland -0.6%
Gujarat - 919 Kerala 4.91%
Maharashtra – 929 Lakshadweep 6.30%
Madhya Pradesh - 931 Andman &Nicobar 6.86%
43. Which of the following state of India has least Goa 8.23%
difference between male and female literacy 48. According to Redfield and Singer, primary
rate? urbanization development process has been
(a) Kerala (b) Meghalaya characterized by :
(c) Nagaland (d) Mizoram (a) Folk tradition (b) Elite tradition
Ans. (b) : Among the given options, Meghalaya has (c) Great Tradition (d) Small tradition
least difference between male and female literacy rate. Ans. (c) : Social scientists Robert Redfield and Milton
As per latest data of Census- 2011, states and their male Singer have identified two processes of urbanization
– female literacy are as follow: Primary and Secondary. The primary urbanization is
State Total literacy Female Male characterized by the development of a 'Great Tradition.
Kerala 94.00% 92.07% 96.11% 49. What is correct descending order of the
Meghalaya 74.43% 72.89% 75.95% following districts of Uttar Pradesh in terms of
Nagaland 79.55% 76.11% 82.75% geographical area?
Mizoram 91.33% 89.27% 93.35% (a) Varanasi, Baghpat, Sant Ravidas
44. Which of the following state has highest female Nagar,Gautam Buddha Nagar
literacy rate in 2001? (b) Sant Ravidas Nagar, Varanasi,Gautam
(a) Tamil Nadu Buddha Nagar, Baghpat
(b) Maharashtra (c) Gautam Buddha Nagar , Varanasi , Sant
(c) Mizoram Ravidas Nagar , Baghpat
(d) Chandigarh (d) Varanasi, Gautam Buddha Nagar, Baghpat,
Ans. (c) : According to Census –2001, state of Mizoram Sant Ravidas Nagar
had highest female literacy rate i.e. 86.75%. Now, Ans. (d) : Geographical areas of above mentioned
Kerala state has highest female literacy rate i.e. 92.07%, districts of Uttar Pradesh are as follows:
as per latest data of Census – 2011. Districts Geographical area (km2)
45. According to Census – 2001 , literacy rate in Varanasi 1535
Uttar Pradesh was: Gautam Buddha Nagar 1442
(a) More than average literacy rate of India Baghpat 1321
(b) More than Rajasthan Sant Ravidas Nagar 1056
(c) Less than Orisha 50. Who of the following is Head of Indian
(d) Less than Bihar Republic?
Ans. (c) : According to Census – 2001 , literacy rate in (a) President of India
Uttar Pradesh (56.27%) was less than Orissa (63.08%). (b) Prime Minister of India
Also in Census – 2011, literacy rate in Uttar Pradesh (c) Ministerial Council
(67.7%) is less than Orisha (72.87%). (d) Ministerial Council with Political Head
46. Which of the following census year had sex Ans. (a) : India has adopted parliamentary form of
ratio of 946 in India? government which is characterized by presence of real
(a) 1921 (b) 1941 executive and nominal executive. President is head of
(c) 1951 (d) 1971 the State and Prime Minister is head of the government.
UP RO/ARO (Mains) Exam 2013 175 YCT
CLICK HERE FOR FREE MATERIAL

51. Match List-I to List-II and select correct Ans. (a) : The correct order of formation of above
answer by using codes given below: mentioned States is Nagaland, Meghalaya, Sikkim and
List-I List-II Arunachal Pradesh because Nagaland was formed in
1963; Meghalaya in 1972; Sikkim in 1975 and
A. XV part of 1. Emergency Arunachal Pradesh in 1987.
Constitution provisions
54. Which of the following state was constituted in
B. XVI part of 2. Official language the year 1948?
Constitution (a) West Bengal (b) Assam
C. XVII part of 3. Special provisions in (c) Punjab (d) Himachal Pradesh
Constitution regarding of some Ans. (*) : This question is wrong because all the above
classes mentioned states were constituted in following years:
D. XVIII part of 4. Election States Constituted Year
Constitution West Bengal 26th January, 1950
Codes Assam 26th January, 1950
A B C D Punjab 1st November, 1966
(a) 3 4 1 2 Himachal Pradesh 25th January, 1971
(b) 4 3 2 1 55. Who presides over joint sitting of both houses
(c) 1 2 3 4 of the Parliament?
(d) 2 1 4 3 (a) President of India
Ans. (b) : The correct match is as follows: (b) Chairperson of Rajya Sabha
Parts of Constitution Related Provisions (c) Speaker of Lok Sabha
(d) Prime Minister
XV part of Constitution Election
XVI part of Constitution Special provisions in Ans. (c) : As per Article 108 of The Constitution joint
regarding of some sitting is called in order to solve the deadlock between
Loksabha and Rajyasabha. The joint sitting is called by
classes the President. The Speaker presides over a joint sitting.
XVII part of Constitution Official language In the absence of the Speaker, the Deputy Speaker of
XVIII part of Emergency the Lok Sabha presides over it, and in his absence, the
Constitution provisions sitting is presided over by the Deputy Chairman of the
52. Which of the following is not objective of Rajya Sabha. If any of the above-mentioned people are
Directive Principles of State Policy? not available, any Member of Parliament (MP) can
(a) To establish a welfare state. preside over the sitting by consensus of both Houses.
(b) To ensure socio-economic justice. 56. Consider the following statements and select
the correct answer by using codes given below:
(c) To establish a religious state.
Assertion (A): The urban poverty is rooted in
(d) To establish a secular state. rural areas.
Ans. (c) : Part IV of the Indian Constitution deals with Reason (R): The education level is low in rural
Directive Principles of State Policy (DPSP). The areas.
provisions contained in this Part cannot be enforced by (a) (A) and (R) are true statements and (R) is
any court, but these principles are fundamental in the correct explanation of (A).
governance of the country and it shall be the duty of the (b) (A) and (R) are true statements but (R) is not
State to apply these principles in making laws. They are correct explanation of (A).
covered in Articles 36 to 51. Directive Principles of (c) (A) is true but (R) is false.
State Policy aim to create social and economic (d) (R) is true but (A) is false.
conditions under which the citizens can lead a good life. Ans. (b) : The root cause of urban poverty lies in rural
They also aim to establish social and economic area due to uncontrolled migration form rural areas to
democracy through a welfare state. To establish a urban areas. The education level is low in rural areas
religious state is not an objective of Directive Principles due to the lack of teachers, school and poor sanitation.
of State Policy. Thus both (A) and R are individually true statements
Construction of States in Independent India but R is not correct explanation of A.
53. The correct ascending order of formation of 57. The poverty level in India is determined on the
following States is: basis of:
(a) Nagaland, Meghalaya, Sikkim, Arunachal (a) Per capita income of various regions.
Pradesh. (b) Average domestic income
(b) Meghalaya, Arunachal Pradesh, Nagaland, (c) Expenditure of domestic consumer
Sikkim. (d) Slum population in the country
(c) Arunachal Pradesh, Nagaland, Sikkim, Ans. (c) : The poverty level in India is determined on
Meghalaya. the basis of consumption expenditure of domestic
(d) Sikkim, Nagaland, Arunachal Pradesh, consumer. Poverty estimation in India is now carried
Meghalaya. out by NITI Aayog’s task force through the calculation
UP RO/ARO (Mains) Exam 2013 176 YCT
CLICK HERE FOR FREE MATERIAL

of poverty line based on the data collected by the National 62. Which of the following is not a constitutional
Sample Survey Office (NSSO) under the Ministry of body?
Statistics and Programme Implementation (MoSPI). (a) Finance Commission
58. Water (Prevention and Control of Pollution) (b) Public Service Commission
Cess Act was enacted in: (c) Planning Commission
(a) 1973 (b) 1975 (d) Election Commission
(c) 1977 (d) 1979 Ans. (c) : Planning Commission, established in 1950
Ans. (c) : The Water (Prevention and Control of was non-constitutional, non-statutory body. It was
Pollution) Cess Act was enacted in 1977 to provide for established by a resolution of government. Rest all
the levy and collection of a cess on water consumed by options are constitutional body.
persons operating and carrying on certain type of 63. In which of the following year, growth rate of
industrial activities. G.D.P on factor cost (at fixed prices) was
59. India ranks ………. in cement production maximum?
(2010) at world level: (a) 2005-06 (b) 2006-07
(a) First (b) Second (c) 2003-04 (d) 2007-08
(c) Third (d) Fourth Ans. (b) : In the year 2006-07, growth rate of G.D.P on
Ans. (b) : India ranks second in cement production factor cost (at fixed prices) was maximum, i.e, 9.6%. The
(2010) at worldwide. In 2021 Indian produced 330 growth in GDP during 2021-22 is estimated at 8.7 percent
as compared to a contraction of 6.6 percent in 2020-21.
million metric tones cement and got 2nd rank.
64. Consider the following statements and select
60. Peanut is main crop of: the correct answer by using codes given below:
(a) Georgia (b) Gambia Assertion (A) : Copper rods are generally
(c) Ghana (d) Guatemala preferred to iron rods for making lightning
Ans. (b) : Peanut is main crop of Gambia. In countries conductors.
such as Gambia, Niger, Senegal, and Sudan peanuts Reason (R) : Copper is a better conductor of
comprise a large part of the gross national product. electricity to Iron and does not oxidized easily
Peanuts are one of the major cash income crops for in atmospheric conditions.
many small-scale farmers in these and other developing (a) (A) and (R) are true statements and (R) is
countries. correct explanation of (A).
61. Agro-ecological zones in India are: (b) (A) and (R) are true statements but (R) is not
(a) 15 (b) 17 correct explanation of (A).
(c) 18 (d) 20 (c) (A) is true but (R) is false.
(d) (R) is true but (A) is false.
Ans. (d) : There are 20 agro-ecological zones in India
as following: Ans. (a) : Copper is a better conductor of electricity
1. Western Himalayas compared to Iron and does not oxidized easily in
atmospheric conditions that is why copper rods are
2. Western Plain, Kachchh, and part of Kathiwara generally preferred to iron rods for making lightning
Peninsula conductors. Thus, (A) and (R) are true statements and
3. Deccan Plateau (R) is correct explanation of (A).
4. Northern Plain and Central Highlands including 65. Jet engine works on the principle of:
Aravallis (a) Linear Momentum conservation
5. Central Malwa Highlands, Gujarat Plains, and (b) Angular Momentum conservation
Kathiawar Peninsula (c) Energy conservation
6. Deccan Plateau, hot semi-arid ecoregion (d) Mass conservation
7. Deccan (Telengana) Plateau and Eastern Ghats
8. Eastern Ghats, Tamil Nadu Plateau and Deccan Ans. (a) : Jet engine works on the principle of linear
(Karnataka) momentum conservation. In a jet engine, a large
volume of gas is produced through the combustion of
9. Northern Plain, hot sub-humid (dry) eco region fuel which gets escaped from the back side of the jet.
10. Central Highlands (Malwas, Budelkhand, and A large momentum is gained by this gas since it has a
Eastern Satpura) high velocity. Therefore, there is an equal and opposite
11. Eastern Plateau (Chattisgarh), hot sub-humid momentum. Thus, the jet moves in the forward direction
ecoregion with high speed. The conservation of linear momentum
12. Eastern (Chotanagpur) Plateau and Eastern Ghats states that the body in motion will retain its total
13. Eastern Plain momentum unless an external force is applied to it.
14. Western Himalayas 66. Match List-I to List-II and select correct
15. Bengal and Assam Plains answer by using codes given below:
16. Eastern Himalayas List- I List- II
17. North Eastern Hills (Purvanchal) (A) Acetone 1. Extinguisher
18. Eastern Coastal Plain (B) Carbon tetrachloride 2. Nail Enamel Remover
19. Western Ghats and Coastal Plain (C) Hydrogen Paraoxide 3. Wound dressing
20. Island of Andaman Nicobar and Lakshadweep (D) Liquid Ammonia 4. Refrigerant
UP RO/ARO (Mains) Exam 2013 177 YCT
CLICK HERE FOR FREE MATERIAL

Code: 72. Triple antigen DPT is given to children to


A B C D protect them against:
(a) 4 3 2 1 (a) Diphtheria, Polio and Measles
(b) 3 4 1 2 (b) Polio, Diphtheria and Tetanus
(c) 2 1 3 4 (c) Measles, Whooping Cough and Tetanus
(d) 1 3 4 2 (d) Tetanus, Diphtheria and Whooping Cough
Ans. (c) : The correct match is as follows: Ans. (d): Triple antigen vaccine is a combination of
List-I List-II diphtheria, tetanus and whooping cough (pertussis). The
Acetone - Nail Enamel Remover vaccine stimulates the production of antibodies to
Carbon tetrachloride - Extinguisher immunize the body against the causative agents of the
Hydrogen Paraoxide - Wound dressing three viruses. Infants and young children are recommended
Liquid ammonia - Refrigerant to receive a 5-dose series of diphtheria and tetanus toxoids
67. Which of the following element deficiency compel and acellular pertussis (DTaP) vaccines.
insectivorus plants to trap and eat insects? 73. Which one of the following gives maximum
(a) Manganese (b) Nitrogen energy during metabolic process?
(c) Magnesium (d) Sulphur (a) Carbohydrate (b) Fat
Ans.(b): Insectivorous plants trap and digest insects and (c) Protein (d) Lactic Acid
other small animals to obtain the nitrogen that is
required for their growth. Most plants absorb enough Ans. (b): Fats give maximum energy during metabolic
nitrogen from nitrates in the soil but as the insectivorous processes. Lipids get converted into fatty acids on
plants live in nitrate-deficient bogs, they need to fulfill reduction. Therefore, the energy content of fat is over
their nitrogen demand by digesting the prey instead. twice that of glycogen and proteins. Both glycogen and
68. The process by which plants prepare own food, proteins are hydrophilic, and bind water. The result is
is called as: that the stored glycogen and protein have a much lower
(a) Osmosis (b) Respiration energy density than the hydrophobic lipids. While fats
(c) Photosynthesis (d) Diffusion have 9 calories per gram of energy, carbohydrates and
proteins have only 1−4 calories per gram.
Ans. (c): The process by which plants prepare own
food, is called as Photosynthesis. Photosynthesis is a 74. The first agriculture university was established
biological process used by plants to prepare their food in India in the year:
with the help of sunlight water and carbon dioxide. This (a) 1955 (b) 1960
process occurs when plants use light energy to convert (c) 1965 (d) 1970
carbon dioxide and water into glucose and oxygen gas.
Ans. (b): The first Agriculture University of India was
69. Which of the following organelle is known as G.B. Pant University of Agriculture and Technology,
‘Powerhouse of the Cell’? established in 1960. The India Council of Agricultural
(a) Ribosome (b)Mitochondria Research recognizes 3 Central Agricultural Universities
(c) Cellular membrane 4 Deemed Universities and 63 Agricultural Universities
(d) Centrosome as of January, 2021.
Ans. (b): Mitochondria is known as the “Powerhouse of 75. Which one of the following states is known as
the Cell.” The organelle that is responsible for the
production of energy in a cell is the mitochondria. This ‘Garden of Spices’?
is the most vital part of the cell as it takes the food and (a) Kerala (b) Tamil Nadu
makes energy for the other parts of the cell. (c) Karnataka (d) Gujarat
Mitochondria produce energy through cellular Ans. (a): Kerala is known as 'Garden of Spices' in India
respiration, more specifically aerobic respiration. as variety of spices are produced which are popular for
70. Which of the following is edible part of Potato? their taste worldwide. Black pepper is regarded as ‘King
(a) Stem (b) Seed of Spices’ and cardamom as ‘Queen of Spices’. Kerala
(c) Root (d) Fruit produces 94% black pepper and 60% cardamom alone
Ans. (a): Potatoes grow underground, but the part we in total production.
eat is not a root. It is an underground stem called as 76. The largest wheat producing state is:
tuber. Potatoes are rich in fibres, potassium,vitamin C (a) Punjab (b) Haryana
and vitamin B6.
(c) Uttar Pradesh (d) Madhya Pradesh
71. Which one of the following is responsible for
muscle fatigue? Ans. (c): The largest wheat producing state is Uttar
(a) Uric Acid (b) Pyruvic Acid Pradesh in India. As per data of 2021-22, the leading wheat
(c) Benzoic Acid (d) Lactic Acid producing states in India are Uttar Pradesh (33.94 MT),
Madhya Pradesh (22.41 m MT), Punjab (14.82 MT).
Ans. (d): Lactic Acid is responsible for muscle fatigue.
Lactic acid is formed and accumulated in the muscle 77. Which one of the following sources is not
under conditions of high energy demand, rapid commercial source of energy?
fluctuations of the energy requirement and insufficient (a) Petroleum (b) Nuclear Energy
supply of O2. (c) Natural Gas (d) Biogas
UP RO/ARO (Mains) Exam 2013 178 YCT
CLICK HERE FOR FREE MATERIAL

Ans. (d): Among the above options, biogas is not Ans. (a) :
commercial source of energy. The main components of +2 +2
biogas are methane (50-75%), carbon dioxide (25-50%), A  → C  →E
−3 −3
Trace levels of hydrogen sulfide, ammonia, hydrogen, N  → K  →H
and various volatile organic compounds are also present +4 +4
in biogas depending on feedstock. R  → V  →Z
78. L.P.G used as domestic fuel, consists mainly: Hence Z will come in place of ?
(a) Methane (b) Acetylene 83. If the code of PERFECT is RGTHGEV then in
(c) Ethylene (d) Butane this code how will BROWN be written?
Ans. (d): Liquefied Petroleum Gas (LPG) mainly (a) CSPXO (b) DSQYP
consists of butane and propane. Besides this, traces of (c) CTQXP (d) DTQYP
ethane and pentane are also found. The smell in L.P.G Ans. (d) : Just as,
is caused by ethyl mercaptan.
79. Which one of the following is largest UNESCO
accredited biosphere reserve (in terms of area)
of India?
(a) Nilgiri (b) Nanda Devi
(c) Sundarbans (d) Gulf of Mannar
Ans. (d): Gulf of Mannar (Tamil Nadu) is largest Same as, BROWN → DTQYP
UNESCO accredited biosphere reserve (in terms of 84. How many rectangles are their in given figure
area) of India. Other UNESCO certified biosphere
reserves (in terms of area) of India are as followed:
Biosphere Area (Km2) Year of
Reserves accreditation
Nilgiri 5520 2000
Nanda Devi 5860.69 2004
Sundarbans 9630 2001
Gulf of Mannar 10500 2001 (a) 8 (b) 10
80. Which one of the following cities of India is (c) 11 (d) 12
specialized in the plantation? Ans. (c) : Total no. of rectangles –
(a) Vijayawada (b) Chandigarh ABCD, DEFG, GHIJ, JKLA, ADGJ, LEFK, BCHI,
(c) Shillong (d) Valparai LDGK, JBCG, AEFJ, DHIA = 11 rectangeles.
Ans. (d): Valparai city of India is specialized in the Tea 85. Find the odd figure out –
plantations. Valparai is a hill station in the district of
Coimbatore, in the state of Tamil Nadu. It is a part of the
Anamalai hills in the Western Ghats. Valparai is
surrounded by Anamalai Tiger Reserve, Parambikulam
Tiger Reserve, Eravikulam National Park and the forest
division of Vazhachal. This city is known as 'Seventh
Heavan'. It is rich in terms of green vegetation and forests.
81. A cube of dimensions 4 cm × 4 cm ×4 cm has
been painted green on all its faces and has been Ans. (c) : Both the smaller parts of option (a), (b), (d),
cut into 64 unit cubes. How many unit cubes together form half of the figure while both the smaller
are there whose two faces are green? parts of option (c) don not form half of the figure.
(a) 8 (b) 16 86. If HORSE = IQUWJ, then TIGER = ?
(c) 24 (d) 32
(a) GJKHF (b) UKGSJ
Ans. (c) : Number of cubes colored in two faces (c) UKJIW (d) SJKHW
= (n – 2)12
Ans. (c) :
∴n = 4 = (4 – 2)12
= 24 Just as,
So the number of two green faces will be 24.
82. Find out the missing letter in the following box
A C E
N K H Same as,
R V ?

(a) Z (b) T
(c) L (d) I
UP RO/ARO (Mains) Exam 2013 179 YCT
CLICK HERE FOR FREE MATERIAL

87. The missing letter in the following series is- 94. Which of the following country has signed an
AZEVJ? agreement for peaceful use of nuclear energy
(a) Q (b) B with India on 5 September, 2014?
(c) F (d) L (a) France (b) U.S.A
Ans. (a) : (c) Japan (d) Australia
Ans. (d): India- Australia civil nuclear cooperation
agreement was signed in 2014. Australia is third major
country in terms of Uranium reserves and exports 7000
tons in single year approximately.
88. The missing number in the series 95. Commonwealth games, 2018 will be held at:
4, 9, 25, ?, 121, 169, 289 is- (a) Wellington (b) Kuala Lumpur
(a) 49 (b) 64 (c) Gold Coast (d) Hamilton
(c) 81 (d) 91 Ans. (c): The 2018 Commonwealth Games, officially
Ans. (a) : 4, 9, 25, 49, 121, 169, 289 known as the XXI Commonwealth Games, were an
The above series is a square of prime number. international multi-sport event for members of the
Hence, 72 = 49 will be placed at ? Commonwealth that were held on the Gold Coast,
Queensland, Australia. The Commonwealth games
89. Select the combination of letters that will 2026 will be held in state of Victoria, Australia.
complete the series :-
aba.....bca....ac....cab....cbc 96. Former Navy Chief Admiral D.K.Joshi had
resigned in February, 2014 after which of the
(a) cbbb (b) cbba following submarine accident?
(c) ccba (d) caac (a) INS Sindhurakshak (b) INS Sindhushastra
Ans. (b) : abacbcabacbcabacbc (c) INS Sindhuraj (d) INS Sindhuratna
aba, cbc, aba, cbc, aba, cbc Ans. (d):Former Navy Chief Admiral D.K.Joshi had
90. If A = 2, B = 4, C = 6 and so on, then the resigned in February, 2014 after accident of submarine
number value of PREVENT will be - INS Sindhuratna. Admiral D.K. Joshi is the first Indian
(a) 196 (b) 198 military commander to have resigned since General
(c) 200 (d) 202 Kodandera Subayya Thimayya in 1959 and the only to
Ans. (c) : have his resignation accepted by the government.
97. Who has been declared 'Man of the Series' in
one day international series held in August-
September, 2014 between India-England?
(a) Jimmy Anderson
91. Agriculture contributes in national income of (b) Bhuwaneshwar Kumar
India is approximately: (c) Shikhar Dhawan
(a) 20% (b) 25%
(d) Suresh Raina
(c) 30% (d) 35%
Ans. (d): Suresh Raina has been declared 'Man of the
Ans. (a): The share agriculture in national income of Series' in one day international series held in August-
India is approx 20%. In 2020-2021 it contributed 20.2% September, 2014 between India-England.
in GDP.
98. With which of the following country India has
92. In February, 2014 the Indo-Pakistan signed a Memorandum of Understanding (MoU)
International Trade Fair was organized at:
on defense co-operation in February, 2014?
(a) Faridabad (b) Jalandhar
(a) Maldives (b) Sri Lanka
(c) New Delhi (d) Noida
(c) Saudi Arabia (d) Mauritius
Ans. (*): In February, 2014 the Indo-Pakistan Ans. (c): India has signed a Memorandum of
International Trade Fair was organized at Ludhiana. Understanding (MoU) on defense co-operation in
39th India International Trade Fair was held at Pragati February, 2014 with Saudi Arabia.
Maidan, New Delhi in November 2019 in which
Pakistan did not participate 99. Who was appointed as Governor of Kerala
after resignation of Sheela Dixit?
93. According to UN Habitat Report on condition
of cities of the world, which one of following is (a) Vajubhai Wala (b) E.S.L Narsimhan
not factor to determine the prosperity of cities? (c) Padamnabh Acharya (d) P. Sathsivam
(a) Productivity (b) Quality of life Ans. (d): Former Chief Justice of India P. Sathasivam
(c) Optimum Population (d) Equality was appointed as Governor of Kerala after resignation
Ans. (c): According to UN Habitat Report on condition of Sheela Dixit. Presently, Arif Mohammad Khan (1
of cities of the world, optimum population is not factor September,2019 – till now ) is Governor of Kerala.
to determine the prosperity of cities. Productivity, 100. Nanda, died in March 2014, was a:
quality of life and equality are factors to determine the (a) Bollywood actress (b) Freedom fighter
prosperity of cities. (c) Social worker (d) Sitar player
UP RO/ARO (Mains) Exam 2013 180 YCT
CLICK HERE FOR FREE MATERIAL

Ans. (a): Nanda, died in March 2014, was a bollywood 105. The oldest man of the world, Arturo Licata,
actress. She began her career as a child artist in the had died in age of 111 years on 28th February,
1950s and later went on to act in a number of films as a was resident of:
lead actress and supporting actress. (a) Italy (b) France
101. India and Fiji have signed ‘Double Taxation (c) Japan (d) England
Avoidance Agreement’ on: Ans. (a): The oldest man of the world, Arturo Licata
(a) 27 January,2014 (b) 28 January,2014 (2nd May 1902 – 28th February 2014), died at age of 111
(c) 29 January,2014 (d) 30 January,2014 years , was resident of Italy.
Ans. (d): Double Taxation Avoidance Agreement was 106. Which of the following sub-sector has negative
signed between India and Fiji on 30th January 2014 for growth rate in the year 2009-10 compare to the
the avoidance of double taxation and the prevention of year 2008-09?
fiscal evasion with respect to taxes on income. (a) Agriculture and related areas
(b) Construction
102. In which city, 12th edition of Pravasi Bhartiya
(c) Mining and quarrying
Divas, 2014 has organized?
(d) Electricity, gas and water supply
(a) Mumbai (b) Kolkata
(c) New Delhi (d) Hyedrabad Ans. (a): Agriculture and allied sector has negative
growth rate in the year 2009-10 compare to the year
Ans. (c): 12th edition of Pravasi Bhartiya Divas, 2014 2008-09. According to economic survey 2009-10,
was organized at New Delhi. Pravasi Bharatiya Divas agriculture growth rate was 1.6 during 2008-2009 while
(PBD) is celebrated on 9th January every year since it has negative growth (- 4%) in year 2009-10.
2003 to mark the contribution of Overseas Indian
107. Which one of the following country has been
community in the development of India. January 9 was visited by Chief Minister of Uttar Pradesh in
chosen as the day to celebrate this occasion since it was September,2014 for purpose to attract overseas
on this day in 1915 that Mahatma Gandhi, the greatest capital investment in state?
Pravasi, returned to India from South Africa, led India’s (a) Denmark (b) The Netherlands
freedom struggle and changed the lives of Indians
(c) Spain (d) China
forever. Since 2015, its format has been revised to
celebrate the PBD once every two years and to hold Ans. (b): The Chief Minister of Uttar Pradesh, Akhilesh
theme-based PBD Conferences during the intervening Yadav, visited Netherlands in September,2014 aiming
period with participation from overseas diaspora to attract overseas capital investment in state.
experts, policy makers and stakeholders. Pravasi 108. How many legislative assembly constituencies
Bhartiya Divas, 2022 was organized at New Delhi. of Uttar Pradesh has held by-polls in
September, 2014?
103. Eminent leader, Bangaru Laxman died in
(a) 12 (b) 11
March, 2014 at Hyderabad, was related to:
(c) 13 (d) 14
(a) Telegu Desham Party
(b) Indian National Congress Ans. (b): Eleven (11) legislative assembly
constituencies of Uttar Pradesh has held by-polls in
(c) Bhartiya Janta Party
September, 2014.
(d) Communist Party of India
109. Which one of the following book was written by
Ans. (c): Eminent leader, Bangaru Laxman died on 1st Khushwant Singh, died in March-2014?
March, 2014 at Hyderabad, was related to Bhartiya (a) India@ risk
Janta Party. He was the first leader of Scheduled Castes
(b) Night Broken
to head the BJP.
(c) A God In Every Stone
104. Who among the following has been selected for (d) The Good, The Bad and The Ridiculous
Jnanpith Award,2013 in June, 2014?
Ans. (d): Khushwant Singh died on 20th March,2014 at
(a) Dr. Kedarnath Singh
New Delhi. Some famous books written by him are as
(b) Dr.Vishwanath Tripathi follows:-
(c) Chetan Bhagat The Good, The Bad and The Ridiculous
(d) Amit Tripathi Women and Men in My Life
Ans. (a): Noted Hindi poet Dr. Kedarnath Singh has A History of the Sikhs
been chosen for the prestigious Jnanpith award for Train to Pakistan
2013. Dr. Kedarnath Singh is the 10th Hindi writer to Delhi: A Novel
receive the honour. Jnanpith award is the oldest Indian The End of India
literary award which is annually presented by the Bhartiya Death at My Doorstep.
Jnanpith to an author who has an outstanding contribution 110. The numbers of central universities in Uttar
to literature. The most recent Jnanpith award winners in Pradesh is:
2021 and 2022 are Assamese poet Nilmani Phookan and (a) 3 (b) 4
Konkani novelist Damodar Mauzo. (c) 2 (d) 1
UP RO/ARO (Mains) Exam 2013 181 YCT
CLICK HERE FOR FREE MATERIAL

Ans. (b): When this question was asked, there were 117. Prime Minister Shram Puraskar is presented
four (4) central universities in Uttar Pradesh which are to:
as follows: (a) Public sector undertaking employees of State
governments only.
1) Aligarh Muslim University, Aligarh
(b) Public sector undertaking employee of
2) Allahabad University, Prayagraj Central government only,
3) Banaras Hindu University, Varanasi (c) Public undertaking employee of State and
4) Dr. Bhimrao Amebkar University, Lucknow Centre governments only.
Recently, Rajiv Gandhi National Aviation University (d) Working employee of selected manufacturing
is included in this list. units of Central and State Public Sector
111. The first co-operative dairy committee was Undertakings and Private sector.
established in Uttar Pradesh at: Ans. (d): Prime Minister's Shram Puruskar scheme was
(a) Saharanpur (b) Varanasi instituted in the year 1985. The award has been
instituted to recognize the outstanding contributions
(c) Lucknow (d) Allahabad made by workmen as defined in the Industrial Dispute
Ans. (d): The first co-operative dairy committee in the Act, 1947 in organizations both in public and private
country started with the organization of 'Katra sector and who have distinguished record of
Cooperative Milk Society' in 1917 at Allahabad (Uttar performance, devotion to duty of a high order, specific
Pradesh). It was first cooperative dairy committee of contribution in the field of productivity, proven
Uttar Pradesh as well as India. innovation abilities, presence of mind and exceptional
courage and also to the workmen who have made
112. Who among of the following was not Chief supreme sacrifice of laying down their lives in the
Minister of Uttar Pradesh? conscientious discharge of their duties.
(a) Govind Ballabh Pant (b) T.N. Singh 118. Identify the incorrect pair of the following:
(c) Nityanand Swami (d) Ram Naresh Yadav (a) Kannauj - Perfumes and oils
Ans. (c): Nityanand Swami was the first Chief Minister (b) Mirzapur - Carpet
of Uttarakhand. He had never been Chief Minister of (c) Muradabad - Brass utensils
Uttar Pradesh. (d) Agra - Knives & scissors
113. In which of the following district of Uttar Ans. (d): Agra is famous for leather, handicrafts, stone
Pradesh, Kalinjar Fort is located? carving and carpets. Rampur district of Uttar Pradesh is
(a) Jhansi (b) Banda famous for knives & scissors.
(c) Chitrakoot (d) Jalaun 119. Match List-I to List-II and select correct
Ans. (b): Kalinjar Fort is located in Banda district in answer by using codes given below:
Bundlekhand region of Uttar Pradesh .This fort built by List - I List - II
the Chandelas is an example of the grand architecture of (A) Bharatnatyam 1. Tamil Nadu
the reign of the Chandela dynasty. (B) Kathak 2. Kerala
114. Prominent centers of leather industry in Uttar (C) Kuchipudi 3. Andhra Pradesh
Pradesh are: (D) Mohiniyattam 4. Uttar Pradesh
(a) Agra, Kanpur (b) Agra, Allahabad Code:
(c) Kanpur, Allahabad (d) Kanpur, Merrut A B C D
Ans. (a): Prominent centers of leather industry in Uttar (a) 2 1 4 3
Pradesh are Agra and Kanpur. (b) 1 4 3 2
115. Uttar Pradesh State Tourism Development (c) 4 3 2 1
Cooperation Limited had established in: (d) 3 4 1 2
(a) 1974 (b) 1978 Ans. (b) : The correct match is as follows:
(c) 1984 (d) 1990 List- I List- II
Bharatnatyam - Tamil Nadu
Ans. (a): Uttar Pradesh State Tourism Development
Kathak - Uttar Pradesh
Cooperation Limited was established in the year 1974.
Kuchipudi - Andhra Pradesh
116. The name of Ebola virus has been taken from: Mohiniyattam - Kerala
(a) The city (b) The province
120. Central Arid Zone Research Institute is located
(c) The river (d) The mountain at:
Ans. (c): Ebola Virus Disease (EVD) is a severe disease (a) Jhansi (b) Hyderabad
that is caused by Ebola virus. Ebola is named after the (c) Jodhpur (d) Jabalpur
river in Africa where the disease was first recognized in Ans. (c): Central Arid Zone Research Institute (CAZRI)
1976. The virus is transmitted to people from wild , one of the biggest research institutes of the Indian
animals and spreads in the human population through Council of Agricultural Research (ICAR), was
human-to-human transmission. established at Jodhpur by Government of India in 1959.
UP RO/ARO (Mains) Exam 2013 182 YCT
CLICK HERE FOR FREE MATERIAL

Gòej ØeosMe meceer#ee DeefOekeâejer/meneÙekeâ meceer#ee DeefOekeâejer (cegKÙe) hejer#ee, 2013


meeceevÙe efnvoer
nue ØeMve-he$e
1. FveceW mes keâewve mee JeekeäÙe DeMegæ nw? JÙeeKÙee – Jele&veer keâer Âef„ mes efoÙes ieÙes efJekeâuheeW ceW Megæ Meyo
(a) nj Skeâ efKeueeÌ[er cewoeve ceW hengBÛe ieS~ Devegie=nerle nw~ DevÙe leerveeW efJekeâuhe DeMegæ nQ~
(b) meÌ[keâ hej yengle YeerÌ[ nw~
8. Jele&veer keâer Âef„ mes Megæ Meyo nw–
(c) DeOÙeÙeve kesâ efueS DeÛÚer hegmlekeWâ DeeJeMÙekeâ nQ~
(d) meeefnlÙe Deewj mebmke=âefle keâe ienje mebyebOe nw~ (a) vejkeâ (b) veke&â
Gòej– (a) (c) veÇkeâ (d) ve&keâ
JÙeeKÙee – efoÙes ieÙes efJekeâuheeW ceW meÌ[keâ hej yengle YeerÌ[ nw, DeOÙeÙeve Gòej–(a)
kesâ efueS DeÛÚer hegmlekeWâ DeeJeMÙekeâ nQ leLee meeefnlÙe Deewj mebmke=âefle keâe JÙeeKÙee – Jele&veer keâer Âef„ mes Megæ Meyo vejkeâ nw, peyeefkeâ DevÙe
ienje mecyevOe nw~ Ùes leerveeW JeekeäÙe JÙeekeâjCe keâer Âef„ mes Megæ nQ; peyeefkeâ Meyo veke&â, veÇkeâ Deewj ve&keâ DeMegæ nQ~
nj Skeâ efKeueeÌ[er cewoeve ceW hengBÛe ieÙes, ceW ieÙes keâer peien ieÙee nesvee 9. Jele&veer keâer Âef„ mes Megæ keâe ÛeÙeve keâerefpeS~
GefÛele nw~ DeleSJe efJekeâuhe (a) DeMegæ nw~ (a) efšheÌ[er (b) efšhheefCe
2. megmebiele Meyo ØeÙeesie ceW keâewve mee JeekeäÙe meJee&efOekeâ GheÙegkeäle nw? (c) šerhheCeer (d) efšhheCeer
(a) og„ keâes mepee efceueveer ÛeeefnS~ Gòej–(d)
(b) DehejeOeer keâes mepee efceueveer ÛeeefnS~
(c) DeefMe„ keâes mepee efceueveer ÛeeefnS~
JÙeeKÙee – Jele&veer keâer Âef„ mes Megæ Meyo efšhheCeer nw~ DevÙe leerveeW
(d) Me$eg keâes mepee efceueveer ÛeeefnS~ efJekeâuhe DeMegæ nQ~
Gòej–(b) 10. `heefle-Ùegkeälee m$eer' keâes Skeâ Meyo ceW keânles nQ?
JÙeeKÙee – efoÙes ieÙes efJekeâuheeW ceW og„ keâes mepee efceueveer ÛeeefnS Ùee DeefMe„ keâes (a) ocheleer (b) peesÌ[er
mepee efceueveer ÛeeefnS Ùee Me$eg keâes mepee efceueveer ÛeeefnS, leerveeW JeekeäÙe DeMegæ nQ, (c) meOeJee (d) heefle-helveer
peyeefkeâ `DehejeOeer keâes mepee efceueveer ÛeeefnS’ JeekeäÙe Megæ nw~ Gòej–(c)
3. Megæ JeekeäÙe keâe ÛeÙeve keâerefpeS~ JÙeeKÙee – heefle-Ùegkeälee m$eer keâes Skeâ Meyo ceW ‘meOeJee’ keâne peelee nw~
(a) ieg¤ Dee jne nw~ (b) ieg¤ Dee jns nw~ 11. `PeieÌ[e ueieeves Jeeuee ceveg<Ùe' Skeâ Meyo ceW keâne peelee nw?
(c) ieg® Dee jns nQ~ (d) ieg¤peer Dee jns nQ~ (a) peÙeÛevo (b) Mekegâveer
Gòej–(c) (c) efJeYeer<eCe (d) veejo
JÙeeKÙee–‘ieg®’ Megæ Jele&veer nw FmeefueS ‘ieg® Dee jns nQ’ mener JeekeäÙe nw~ Gòej–(d)
4. Meyoeveg›eâce ceW mener JeekeäÙe nw– JÙeeKÙee – ‘PeieÌ[e ueieeves Jeeues ceveg<Ùe’ kesâ efueS Skeâ Meyo ceW ‘veejo’
(a) Jen iejerye Deeoceer Lee~ (b) Jen Deeoceer iejerye Lee~ keâne peelee nw~
(c) Lee Jen iejerye Deeoceer~ (d) Lee iejerye Jen Deeoceer~
12. `cenue kesâ Yeerlejer Yeeie' keâes efkeâme Meyo ceW peeveles nQ?
Gòej–(a)
(a) ieYe&ie=n (b) Yeerlejer leue
JÙeeKÙee – efoÙes ieÙes JeekeäÙeeW ceW Meyoeveg›eâce keâer Âef° mes ‘Jen iejerye
(c) Devle:hegj (d) jefveJeeme
Deeoceer Lee~’ mener nw keäÙeeWefkeâ JeekeäÙe ceW efJeMes<eCe keâe ØeÙeesie efJeMes<Ùe mes
Gòej–(c)
henues neslee nw~ Mes<e meYeer efJekeâuhe Meyoeveg›eâce keâer Âef° mes ieuele nw~
5. Jele&veer keâer Âef„ mes Megæ Meyo nw– JÙeeKÙee – cenue kesâ Yeerlejer Yeeie keâes ‘Devle:hegj’ kesâ veece mes peevee
(a) Deveg<eebefiekeâ (b) Devegmeebefiekeâ (c) Deeveg<ebefiekeâ (d) Deevegmeebefiekeâ peelee nw~ Dele: efJekeâuhe (c) Megæ nw~ DevÙe efJekeâuhe $egefšhetCe& nw~
Gòej–(c) 13. FveceW keâewve mee Meyo leodYeJe nw?
JÙeeKÙee– ØeMveiele efoÙes ieÙes efJekeâuheeW ceW ‘Deeveg<ebefiekeâ’ mener Jele&veer Jeeuee (a) ceOeghe (b) ceOegkeâj
Meyo nw, peyeefkeâ DevÙe leerveeW Meyo Jele&veer keâer Âef„ mes DeMegæ nQ~ (c) YeÇcej (d) YeBJeje
6. Jele&veer keâer Âef„ mes Megæ Meyo nw– Gòej– (d)
(a) heefjÛÚe (b) hejerÛÚe JÙeeKÙee – efoÙes ieÙes efJekeâuheeW ceW ceOeghe, ceOegkeâj SJeb YeÇcej Megæ lelmece
(c) hejer#ee (d) heefj#ee Meyo nQ, peyeefkeâ efJekeâuhe (d) ceW ØeÙegkeäle Meyo YeBJeje leodYeJe Meyo nw~
Gòej–(c) 14. `cegojer' keâe lelmece ¤he nw–
JÙeeKÙee – Jele&veer keâer Âef„ mes heefjÛÚe, hejerÛÚe, heefj#ee leerveeW DeMegæ (a) cegõer (b) cegvojer
Meyo nQ, peyeefkeâ ‘hejer#ee’ Megæ Meyo nw~ (c) cegoefjkeâe (d) cegefõkeâe
7. Jele&veer keâer Âef„ mes Megæ keâe ÛeÙeve keâerefpeS~ Gòej–(d)
(a) Deveg«enerle (b) Devegie=nerle
JÙeeKÙee – efoÙes ieÙes efJekeâuheeW ceW cegojer Meyo keâe lelmece ¤he
(c) Deveg«eefnle (d) Deevegie=nerle
‘cegefõkeâe’ neslee nw~ Mes<e efJekeâuhe Demebiele nQ~
Gòej–(b)
UP RO/ARO (Main) General Hindi 2013 183 YCT
CLICK HERE FOR FREE MATERIAL

15. `oeref"' keâe lelmece ¤he nw– JÙeeKÙee – efoÙes ieÙes efJekeâuheeW ceW meele Meyo mebKÙeeJeeÛekeâ efJeMes<eCe nw,
(a) õef„ (b) efoef„ peyeefkeâ keâeuee leLee Keóe iegCeJeeÛekeâ efJeMes<eCe Meyo nwb~
(c) oeref„ (d) Âef„ 23. `Glkeâ<e&' Meyo keâe efJeueesce efueefKeS~
Gòej–(d) (a) heleve (b) Dehekeâ<e&
JÙeeKÙee – oeref" Meyo keâe lelmece ¤he ‘Âef„’ neslee nw~ DevÙe leerveeW (c) DeheYeÇ„ (d) efJekeâ<e&
efJekeâuhe kesâ Meyo lelmece ¤he keâer Âef„ mes DeMegæ nQ~ Gòej–(b)
16 keâewve mee Meyo lelmece nw? JÙeeKÙee – Glkeâ<e& Meyo keâe efJeueesce Meyo `Dehekeâ<e&' neslee nw, ve efkeâ
(a) Megßet<ee (b) megVej efJekeâ<e&, heleve Deewj DeheYeÇ„~
(c) Dehepeme (d) DeÛÚj 24. `Devleceg&Keer' keâe efJeueesce efueefKeS~
Gòej–(a) (a) peielcegKeer (b) JeeÛeeue
JÙeeKÙee – efoÙes ieÙes ÛeejeW efJekeâuheeW ceW lelmece Meyo ‘Megßet<ee’ nw, (c) Ûelegceg&Keer (d) yeefnceg&Keer
peyeefkeâ DevÙe efJekeâuhe megVej, DeÛÚj Deewj Dehepeme leerveeW Meyo leodYeJe Gòej–(d)
nQ~ JÙeeKÙee – Debleceg&Keer Meyo keâe efJeueesce Meyo ‘yeefnceg&Keer’ neslee nw, ve
17. `eEmeieej' Meyo keâe lelmece nw– efkeâ peielecegKeer, JeeÛeeue, Ûelegceg&Keer~
(a) ëe=bieej (b) ßebieej 25. `DeefleJe=ef„' keâe efJeueesce Meyo nw–
(c) ße=bieej (d) eEMeieej (a) DeuheJe=ef„ (b) ueIegJe=ef„
Gòej–(a) (c) DeveeJe=ef„ (d) vÙetveJe=ef„
JÙeeKÙee – eEmeieej Meyo keâe lelmece ¤he ‘ëe=bieej’ neslee nw~ Mes<e Gòej–(c)
efJekeâuheeW kesâ Meyo $egefšhetCe& nQ~ JÙeeKÙee – DeefleJe=ef„ keâe efJeueesce Meyo ‘DeveeJe=ef„’ neslee nw, ve efkeâ
18. `ceer"e Dece¤o' ceW `ceer"e' efJeMes<eCe efkeâme keâesefš keâe nw?
DeuheJe=ef„, ueIegJe=ef„ Deewj vÙetveJe=ef„~
26. `mebÙeesie' keâe efJeueesce Meyo nw–
(a) heefjceeCeJeeÛekeâ (b) iegCeJeeÛekeâ
(a) DeÙeesie (b) efJeÙeesie
(c) JÙeeqkeäleJeeÛekeâ (d) mebKÙeeJeeÛekeâ
(c) efJejn (d) Ùeesienerve
Gòej–(b)
Gòej–(b)
JÙeeKÙee – ceer"e Dece¤o ceW ceer"e ‘iegCeJeeÛekeâ’ efJeMes<eCe nw~ Ùen
JÙeeKÙee – mebÙeesie keâe efJeueesce Meyo ‘efJeÙeesie’ neslee nw, ve efkeâ efJejn,
Dece¤o kesâ ceer"e nesves kesâ iegCe keâer ÛeÛee& keâjlee nw~ DeÙeesie Deewj Ùeesienerve~
19 `legce keâneB heÌ{les nes'–ceW efkeâme keâesefš keâe efJeMes<eCe ØeÙegkeäle ngDee nw?
27. `jepee' Meyo keâe efJeueesce efueefKeS~
(a) iegCeJeeÛekeâ (b) ØeMveJeeÛekeâ (a) iejerye (b) oefjõ
(c) mebKÙeeJeeÛekeâ (d) mebkesâleJeeÛekeâ (c) jbkeâ (d) efYeKeejer
Gòej–(b) Gòej–(c)
JÙeeKÙee – legce keâneB heÌ{les nes - ceW ‘ØeMveJeeÛekeâ efJeMes<eCe’ ØeÙegkeäle JÙeeKÙee – jepee Meyo keâe efJeueesce Meyo ‘jbkeâ’ neslee nw, ve efkeâ iejerye,
ngDee nw, ve efkeâ iegCeJeeÛekeâ, mebKÙeeJeeÛekeâ Ùee mebkesâleJeeÛekeâ efJeMes<eCe~ oefjõ Deewj efYeKeejer~
20. `ueesYeer' efkeâme efJeefOe mes efvee|cele efJeMes<eCe nw? 28. `DeekeâeMe' Meyo keâe efJeueesce nw?
(a) meb%ee-efJeefOe (b) meJe&veece-efJeefOe (a) Oejleer (b) veeieueeskeâ
(c) ef›eâÙee-efJeefOe (d) ØelÙeÙe-efJeefOe (c) heeleeue (d) mecegõ
Gòej– (d) Gòej–(c)
JÙeeKÙee – ‘ueesYeer’ ØelÙeÙe-efJeefOe Éeje efveefce&le efJeMes<eCe nw~ ‘ueesYe’ meb%ee JÙeeKÙee – DeekeâeMe Meyo keâe efJeueesce Meyo `heeleeue' neslee nw, ve efkeâ
Meyo nw efpemeceW ‘F&’ ØelÙeÙe-efJeefOe Éeje ‘ueesYeer’ Meyo yevee nw~ Dele: Oejleer, mecegõ Deewj veeieueeskeâ~
efJekeâuhe (d) mener nw~ 29. `meiegCe' Meyo keâe efJeueesce nw?
21. FveceW keâewve mee Ùegice efJeMes<eCe veneR nw? (a) DeJeiegCe (b) DeiegCe
(a) Úesše-yeÌ[e (b) nje-heeruee (c) ogieg&Ce (d) efveieg&Ce
(c) oes-leerve (d) jece-ue#ceCe Gòej–(d)
Gòej–(d) JÙeeKÙee – meiegCe Meyo keâe efJeueesce Meyo ‘efveieg&Ce’ neslee nw, ve efkeâ
JÙeeKÙee – efoÙes ieÙes efJekeâuheeW ceW ‘jece-ue#ceCe’ Ùegice efJeMes<eCe veneR DeJeiegCe, DeiegCe Deewj ogieg&Ce~
nw, yeefukeâ Ùen meb%ee Ùegice Meyo nw; peyeefkeâ Úesše-yeÌ[e, nje-heeruee Deewj 30. `DeheMekegâve' keâe efJeueesce nw–
oes-leerve Ùegice efJeMes<eCe nQ~ (a) DeMekegâve (b) Mekegâve
(c) hegCÙe (d) heeJeve
22. FveceW mebKÙeeJeeÛekeâ efJeMes<eCe keâewve mee nw?
Gòej–(b)
(a) meele (b) keâeuee
(c) jeJeCe (d) Kešdše JÙeeKÙee – DeheMekegâve keâe efJeueesce Meyo ‘Mekegâve’ neslee nw, ve efkeâ
Gòej–(a) heeJeve, hegCÙe Deewj DeMekegâve~
UP RO/ARO (Main) General Hindi 2013 184 YCT
CLICK HERE FOR FREE MATERIAL

UPPSC RO/ARO (Pre) Exam-2013


GENERAL STUDIES
Solved Paper
1. Who among the following has written a book Ans. (d) : 'Operation Polo' is associated with military
on imaginary scientific instruments? action in the Hyderabad state. It was launched on
(a) Bhoja (b) Govindraj September 13,1948 to liberate Hyderabad.
(c) Chandra Varman (d) Mahipal 7. 'India Wins Freedom' is a book concerning
Ans. (a) : Bhoja was the ruler of Paramar dynasty and autobiography of?
he ruled from (1010-1055) C.E. He is best known as a (a) Dr. Rajendra Prasad
patron of arts, literature and sciences. (b) Jawahar Lal Nehru
Samarangana - Sutradhara - a treatise on architecture, (c) Maulana Abul Kalam Azad
iconography and mechanical machines, It gives
detailing on self-operating machines, construction of (d) Humayun Kabir
building, forts, temples etc. Ans. (c) : 'India Wins Freedom' is a book concerning
2. The third Anglo-Maratha War is related to? autobiography of Maulana Abul Kalam Azad.
(a) Sir John Shore (b) Lord Wellesley 8. In which country the 'Ghadar Party' was
(c) Lord Hastings (d) Lord Cornawallis established?
Ans. (c) : The third Anglo-Maratha war is related to (a) U.S.A. (b) Germany
Lord Hastings (1813-1823). (c) Spain (d) France
The third Anglo-Maratha war (1817-1819) fought Ans. (a) : 'Ghadar Party' was established on 15 July 1913
between the East India Company and the Maratha in the United States under the leadership of Lala Har
Empire (Baji Rao II) in India. The British won the war Dayal, Sant Baba Wasakha Singh Dadehar, Santokh Singh
against Marathas. and Sohan Singh Bhakna as its president.
The war concluded with the treaty of Poona between 9. Chauri Chaura is situated in which District?
British and Peshwa and the treaty of Gwalior between (a) Deoriya (b) Gorakhpur
British and Scindia and treaty of Mandasor between (c) Maharajganj (d) Kushinagar
British and Holkar.
Ans. (b) : Chauri-Chaura is situated in the district
3. Who was the last ruler/king of Sikh state?
Gorakhpur of U.P. The place is known for incident
(a) Kharak Singh (b) Sher Singh occured at Chauri Chauria in 1922, when a large group
(c) Nav Nihal Singh (d) Duleep Singh of protesters participating in the Non-Cooperation
Ans. (d) : The last ruler of the Sikh state was Duleep movement, clashed with police and set on fire (ablaze)
Singh of Lahore. Later in life he was given a nickname - to police station which killed all of its occupants.
"Black Prince of Perthshire."
10. During the Quit India Movement, in which of
4. The merger of Awadh State into British the following districts of Uttar Pradesh, a
Empire took place in- parallel government was formed?
(a) 1855 (b) 1854 (a) Allahabad (b) Lucknow
(c) 1856 (d) 1853 (c) Ballia (d) Faizabad
Ans. (c) : The merger of Awadh state into British Ans. (c) : During Quit India Movement, parallel
Empire took place in 1856. In 1856, Lord Dalhousie governments were established at four places. These
ordered to depose Wajid Ali Shah on the account of
were Ballia (Uttar Pradesh), Tamluk (Bengal) Satara
alleged misgovernance. This was inline with
Dalhousie's Doctrine of Lapse. (Maharashtra) and Talcher (Odisha) under the
leaderships of Chittu Pandey (Ballia), local people
5. Bhagat Singh threw bombs in the Central (Tamluk) Y.B. Chavan & Nana Patil (Satara) and
Assembly alongwith- Lakshman Nayak (Talcher) respectively.
(a) Chandrasekhar (b) Sukhdev
11. Consider the following events-
(c) Batukeshwar Dutt (d) Rajguru
1. Lucknow Pact
Ans. (c) : Bhagat Singh threw bombs in the Central
Assembly alongwith Batukeshwar Dutt in 1929. This 2. Champaran Satyagraha
incident is known as the Central Assembly Bombing 3. Khilafa movement
case in Modern Indian History. 4. Jallianwala Bagh Massacre
6. 'Operation Polo' is associated with Arrange the above events in their correct
(a) Quit Indian movement chronological order from the codes given
(b) Civil Disobedience movement below-
(c) Military action/intervention in Junagarh state (a) 1, 2, 4, 3 (b) 1, 2, 3, 4
(d) Military action in the Hyderabad state (c) 4, 3, 2, 1 (d) 1, 3, 2, 4
UP RO/ARO (Pre) Exam 2013 185 YCT
CLICK HERE FOR FREE MATERIAL

Ans. (a) : The chronological order of the above events 18. Which is the most polluted river among the
are as follows- following?
Lucknow Pact - December, 1916 (a) Gomti (b) Periyar
Champaran Styagraha - April, 1917 (c) Damodar (d) Mahanadi
Jallianwala Bagh Massacre - April 1919 Ans. (c) : Damodar is the most polluted river among the
Khilafat Movement - August 1920. given rivers. Damodar river flows through the Indian
12. The book - 'Springing Tiger' is a biography of? State of Jharkhand and West Bengal.
(a) Bhagat Singh 19. On the banks of which of the following rivers
(b) Subhash Chandra Bose the famous Badrinath Temple is located?
(c) Chandrasekhar Azad (a) Alaknanda (b) Bhagirathi
(d) Ram Prasad Bismil (c) Mandakini (d) Ganges
Ans. (b) : The book- 'Springing Tiger' is a biography of Ans. (a) : The famous Badrinath temple is located on
Subhash Chandra Bose written by Hugh Toye. the banks of Alaknanda river in Chamoli district of
13. In which of its session, The Indian National Uttarakhand.
Congress, had rejected the Government of 20. Which of the following river flows through a
India Act, 1935? Rift Valley?
(a) Ramgarh Session (b) Lucknow Session (a) Krishna (b) Godavari
(c) Faizpur Session (d) None of them (c) Tapi (d) Kaveri
Ans. (b) : The INC, had rejected the GoI Act, 1935 in Ans. (c) : In the given options river Tapi flows through
the Lucknow Session, 1936, under the Presidentship of a Rift Valley. The river flows through the Indian state
Jawahar Lal Nehru. of M.P., Maharashtra and Gujarat. It originates from the
14. Who among the following was an extremist? Multai reserve forest in the Betul district of M.P and
(a) Firozshah Mehta drained into Arabian Sea.
(b) Gopal Krishna Gokhale 21. Which of the following pairs is not correctly
(c) Bipin Chandra Pal matched?
(d) None of them (a) Mount Abu - Aravalli mountains
Ans. (c) : Bipin Chandra Pal among the following was (b) Kodaikanal - Annamalai hills
an extremist while Firoz Shah Mehta, Gopal Krishna (c) Ootacamund - Nilgiri hills
Gokhale are the moderates. Moderates believes in the (d) Shimla - Pir Panjal Range
method of constitutional measures and non-violence to Ans. (b/d) : Not correctly matched is Kodaikanal and
attain the goal of freedom whereas, extremists do not Shimla.
believe in the method of constitutional measures but the
The correct match is-
radical measures to attain independence.
Mount Abu - Aravalli mountains
15. The Paper namely______ Commonweal is
Kodaikanal - Palani Hills
connected to?
Ootacamund - Nilgiri hills
(a) B.G. Tilak (b) Annie Besant
(c) G.K. Gokhale (d) None of them Shimla - Dhauladhar Range
Ans. (b) : The Commonweal was a weakly magzine 22. The Meghalay a Plateau is a part of?
which deals issues of national reform. This magzine (a) Himalayan Range
was published by Annie Besant. She also published a (b) Peninsular region
newspaper named as New India. (c) Eastern Ghat Mountains
16. Who was the chairman of the first All India (d) Satpura Range
Samajvadi Yuva Congress? Ans. (b) : The Meghalaya plateau is a part of Peninsular
(a) Subhash Chandra Bose region. It is a type of block mountain, formed when
(b) Acharya Narendra Dev horizontal movement of the waves within the earth's
(c) J.L. Nehru crust moved away from each other creating tension on
(d) J.B. Kripalani the surface finally creating faults.
Ans. (c) : J.L. Nehru was the chairman of the first All 23. Match List-I with List-II and choose the
India Samajvadi Yuva Congress. correct answer using the codes given below:
17. Which among the following is the highest List-I List-II
mountain Peak of South India? (Place) (Confluence of Rivers)
(a) Anamudi (b) Doda Betta A. Rudraprayag 1. Bhagirathi - Alaknanda
(c) Mahendra Giri (d) Dhupgarh
B. Nand Prayag 2. Alaknanda - Mandakini
Ans. (a) : Anamudi is the highest mountain peak of
South India. It is located in the Indian State of Kerala C. Karna Prayag 3. Alaknanda - Pindar
with an elevation of 2695 metres. D. Dev Prayag 4. Alaknanda & Nandakini
UP RO/ARO (Pre) Exam 2013 186 YCT
CLICK HERE FOR FREE MATERIAL

Codes: 30. Which of the following state ranks first in


A B C D terms of wind energy production?
(a) 1 3 2 4 (a) Gujarat (b) Maharashtra
(b) 2 4 3 1 (c) Karnataka (d) Rajasthan
(c) 3 2 4 1 Ans. (a) : The state of Gujarat ranks first in terms of wind
(d) 4 1 3 2 energy production. The country currently has the fourth
Ans. (b) : highest wind installed capacity in the world with total
(Place) (Confluence of Rivers) installed capacity of 39.25 GW (as on 31st March, 2021).
Rudraprayag - Alaknanda & Mandakini 31. The largest centre for Petro-chemical
Nand Prayag - Alaknanda & Nandakini production is located at?
Karna Prayag - Alaknanda & Pindar (a) Jamnagar (b) Ankaleshwar
Dev Prayag - Alaknanda & Bhagirathi (c) Nunamati (d) Trambay
24. Ganga River is an example of- Ans. (a) : The largest centre for petro-chemical
(a) Antecedent Drainage production is located at Jamnagar. The Jamnagar
(b) Consequent Drainage refinery is the private sector crude oil refinery owned by
Reliance Industries Limited in Jamnagar, Gujarat India.
(c) In sequent Drainage
(d) Subsequent Drainage 32. The share of nuclear power in India's total
electricity/power generation is approximately-
Ans. (a) : River Ganga is an example of Antecedent
Drainage. An antecedent stream is a stream that (a) 2% (b) 3%
maintains it's original course and pattern despite the (c) 4% (d) 5%
changes in underlying rock topography. Ans. (b) : The share of nuclear power in India's total
25. Which is the largest glacier among the electricity/power generation is approximately 3%. while
coal is the largest producer of India's total
following?
electricity/power generation.
(a) Siachen (b) Baltoro
(c) Chogolungma (d) Biafo 33. Which of the following iron & steel production
centre of India is remotely located from the
Ans. (a) : Siachen is the largest glacier among the coal production zones?
following it is a 78 km long and the source is the Nubra
River that waters the Nubra valley near Leh in the Trans (a) Bokaro (b) Durgapur
Himalayas. (c) Kulti-Asansol (d) Bhadravati
26. Which of the following rivers is infamous for Ans. (d) : Bhadravati is the iron and steel production
changing its course of flow? centre of India is remotely located from the coal
(a) Ganges (b) Kosi production zones. It is situated in the Shivamogga
(c) Damodar (d) Gomti district of Karnataka.
Ans. (b) : Kosi river flows in the Indian state of Bihar. 34. Which of the following pair is not matched
is infamous for changing its course of flow. It is also correctly?
called the "Sorrow of Bihar." (a) Sardar Sarovar Dam - Narmada River
27. Leh is located on the- (b) Telaiya Dam - Konar River
(a) Right bank of Jhelum river (c) Gandhi Sagar Dam - Chambal River
(b) Left bank of Jhelum river (d) Nagarjuna Sagar Dam - Krishna River
(c) Right bank of Sindhu (Indus) river Ans. (b) : Telaiya Dam is situated on River Barakar in
(d) Left bank of Sindhu (Indus) river the Jharkhand state of India. While rest are correctly
Ans. (c) : Leh is located on the right bank of Sindhu matched-
(Indus) river. Leh is one of the most remotest areas of Sardar Sarovar Dam - Narmada River
the Indian administered portion of the Ladakh region. Gandhi Sagar Dam - Chambal River
28. Which of the following state is a major Cashew Nagarjuna Sagar Dam - Krishna River
producing state? 35. Largest lignite coal reserves in India are found
(a) Goa (b) Maharashtra in?
(c) Kerala (d) Karnataka (a) Jharkhand (b) Odissa
Ans. (b) : Maharashtra is leading cashew producing (c) Jamu & Kashmir (d) Tamil Nadu
state in India, followed by Andhra Pradesh and Odisha. Ans. (d) : Largest Lignite coal reserves in India are
29. In India maximum natural gas is produced found in Tamil Nadu. Lignite is also known as brown
from. coal that has a carbon content around 40-55% and is
(a) Andhra Pradesh considered as the lowest rank of coal after peat due to
(b) Coastal region of Gujarat its relatively low heat content.
(c) Bombay High 36. In India, the first Hydro-Electric plant was
(d) Coastal region of Tamil Nadu established in-
Ans. (c) : In Indian maximum natural gas is produced in (a) Darjeeling (b) Shivasamudram
Bombay High. It is controlled by ONGC. (c) Mohra (d) Khopoli
UP RO/ARO (Pre) Exam 2013 187 YCT
CLICK HERE FOR FREE MATERIAL

Ans. (a) : In India, the first Hydro-electric plant was 44. Which of the following Hydro Electricity
established in Darjeeling West Bengal. It was installed in generation project utilizes water of Pong Dam
the year 1897. It is also the first hydroelectric plant in Asia. of River Beas?
(a) Indira Gandhi Canal Project
37. In India the first crude oil well was dug near-
(b) Pochampad project
(a) Digboi (b) Makum (c) Mayurakshi Project
(c) Narkatia (d) Lakwa (d) Nagarjunsagar Project
Ans. (b) : In India, the first crude oil well was dug at Ans. (a) : Indira Gandhi Canal Project is the Hydro
Makum near Margherita area of upper Assam. Electricity generation project which utilizes water of
38. Out of the following crops in India which crop Pong Dam of River Beas.
has the largest percentage share of irrigated area It is the longest canal in India which starts at the Harike
in contrast to the net gross agricultural area? Barage at the confluence of the Sutlej and Beas rivers in
(a) Wheat (b) Rice the Indian Punjab.
(c) Oilseeds (d) Sugarcane 45. Which of the following Coast is associated with
Ans. (d) : Sugarcane crop has the largest percentage share Kochi Sea Port?
of irrigated area in contrast to the net gross agricultural (a) Malabar Coast
area which is followed by wheat, rice and oil-seeds. (b) Konkan Coast
(c) Coromandal Coast
39. Which of the following minerals is found in (d) Northerm Circars Coast
abundant quantity in Kerala?
Ans. (a) : Malabar coast is associated with Kochi Sea
(a) Tin (b) Manganese
Port located in Kerala.
(c) Mica (d) Monazite
46. Out of the following states in India, which state
Ans. (d) : Monazite minerals is found in abundant is deprived of Rail Services?
quantity in Kerala. Monazite is an important source of (a) Tripura (b) Meghalaya
Thorium, Cerium and other rare elements. (c) Arunachal Pradesh (d) Mizoram
40. In order to reduce the Cargo handling pressure Ans. (b) : Meghalaya, state of India was deprived of Rail
at Mumbai Seaport which port has been Services at the time when this question was asked but now the
constructed? work of rail services has been going on in Meghalaya.
(a) Europe (b) Haldia Currently all the states of India are covered under rail services.
(c) Paradeep (d) Nhava Sheva 47. Konkan Railways connects-
Ans. (d) : Nhava Sheva port has been constructed to (a) Mumbai to Manmad (b) Roha to Mangalore
reduce the cargo handling pressure at Mumbai Seaport. It (c) Mangalore to Kochi (d) Mumbai to Pune
is also known as Jawaharlal Nehru Port Trust or JLN Port. Ans. (b) : Konkan Railway connects Roha near
41. The Krishna River water dispute is between Mumbai to Mangalore. It passes through four states
which two states? Maharashtra, Goa, Karnataka and Kerala. The strech of
(a) Karnataka & Maharashtra its 740 Km comprises of difficult and rugged terrain.
(b) Karnataka & Andhra Pradesh 48. Which of the following is a Coral island?
(c) Andhra Pradesh and Tamil Nadu (a) New Moore (b) Car Nicobar
(c) Andaman (d) Lakshadweep
(d) Andhra Pradesh & Maharashtra
Ans. (d) : Lakshadweep is a coral Island. Corals are
Ans. (*) : The Krishna River water dispute is between skeletons of tiny marine animals called polyps. When
the states of Andhra Pradesh, Telangana, Maharashtra they die their remains accumulate on the sea which
and Karnataka. grows higher and higher to form coral islands.
42. Among the following states of India, the gross 49. The mountain which forms a boundary
length of which state highway is longest? between France and Spain is?
(a) Gujarat (b) Rajasthan (a) Apennines (b) Alps
(c) Uttar Pradesh (d) Maharashtra (c) Jura (d) Pyrenees
Ans. (d) : The gross length of Maharashtra state's Ans. (d) : Pyrenees mountain forms a boundary
highway is the longest which is followed by Karnataka, between France and Spain. The Pyrenees range
Gujarat, Rajasthan and Tamil Nadu. stretches along the entire southern border of France.
Pyrenees is a 490 km long.
43. Which of the following ports is located on the
Odisha Coast? 50. Match List-I with List-II and choose the
(a) Haldia (b) Mumbai correct answer from the codes given below:
(c) Paradip (d) Visakhapatnam List-I List-II
Ans. (c) : Paradip port is located on the Odisha Coast. A. Alps 1. Block Mountains
While port Haldia is located in West Bengal, Port B. Vosges 2. Volcano Mountain
Visakhapatnam in Andhara Pradesh and port Mumbai is
located in Maharashtra. At present there are 13 major ports C. Vindhya 3. Folded Mountains
and 200 non-major ports (minor ports) in the country. D. Fujiyama 4. Residual Mountains
UP RO/ARO (Pre) Exam 2013 188 YCT
CLICK HERE FOR FREE MATERIAL

Codes: 56. Which of the following nation forms the largest


A B C D terrestrial boundary with India.
(a) 2 4 1 3 (a) China (b) Pakistan
(b) 3 1 4 2 (c) Bangladesh (d) Nepal
(c) 1 3 2 4 Ans. (c) : Bangladesh forms the largest terrestrial
(d) 1 4 3 2 boundary with India. Bangladesh and India share a
Ans. (b) : The correct match is as follows- 4,096.70 km long international border. It shares its
border with the Indian states of Assam (263 km),
Mountain Type Tripura (856 km), Mizoram (318 km) Meghalaya (443
Alps Folded Mountains km) and W.B. (2,216.70 km).
Vosges Block Mountains 57. The dead valley located in Southern California
of U.S.A. is an example of?
Vindhya Residual Mountains
(a) Anticlinal Valley (b) Synclinal Valley
Fujiyama Volcano Mountains (c) Antecedent Valley (d) Rift Valley
51. Blind Valleys are located in? Ans. (d) : The dead valley located in Southern
(a) Karst Region (b) Arid Region California of U.S.A. is an example of Rift Valley.
(c) Glacier Region (d) Tundra Region 58. From which of the following coast India's
Ans. (a) : Blind valleys are located in Karst Region. Blind average sea surface is measured?
Valley is a deep, narrow, flat bottomed valley with an (a) Mumbai (b) Chennai
abrupt ending. Karst is a landscape which is underlain by (c) Cochin (d) Vishakhapattanam
limestone which has been eroded by dissolution of soluble Ans. (b) : India's average sea surface is measured from
rocks (e.g. limestone, dolomite and Gypsum). the Chennai coast of India.
52. Which of the following country is located/found 59. Which of the following country is not drained
in the south of Arctic Circle? by Nile River?
(a) Finland (b) Iceland (a) Chad (b) Ethiopia
(c) Norway (d) Sweden (c) Sudan (d) Uganda
Ans. (b) : Arctic Circle is the most northerly of the five Ans. (a) : The Nile River flows from south to north
major circles of latitude (Other fours are - Tropic of through eastern Africa. It flows through 11 African
Cancer, Tropic of Capricorn, Antarctic Circle and countries namely, Egypt, Burundi, Tanzania, Rwanda,
Equator). Iceland is found in the South of Arctic Circle. the Democratic Republic of the Congo, Kenya, Uganda,
53. The chemical element which is found in Sudan, Ethiopia, South Sudan and Eritrea.
abundant quantity in the earth's crust is? Therefore, Chad is NOT drained by Nile River.
(a) Oxygen (b) Aluminium 60. Which is the longest river among the rivers of
(c) Iron (d) Silicon peninsular India?
Ans. (a) : The chemical element which is found is (a) Krishna (b) Cauvery
abundant quantity in the earth's crust is oxygen. (c) Godavari (d) Mahanadi
The most common elements in the crust by weight are Ans. (c) : Godavari is the longest river among the rivers
oxygen (46.1%) silicon (28.2%), Aluminium (8.2%), of peninsular India. It is India's second longest river
Iron (5%), Calcium (4.1%), Sodium (2.3%), Potassium after the river Ganga.
(2.9%) and Magnesium (2.3%). These eight elements River Godavari originates from Trimbakeshwar in the
accounts for about 98.5% of the weight of the crust. Nashik districts of Maharashtra draining the states of
54. Which of the following island comes under the Maharashtra, Telangana, A.P., Chhattisgarh and Odisha
remotest islands category of Indian Coast Line? and ultimately drains into the Bay of Bengal.
(a) Bhatkal (b) Arnala 61. Which of the following countries does not come
(c) Minicoy (d) Henry under the Mediterranean climate region?
Ans. (c) : Minicoy island comes under the remotest (a) Congo (b) Ethiopia
islands category of Indian coast line. Minicoy, locally (c) Gabon (d) Zaire
known as Maliku is an island in Lakshadweep India. Ans. (b) : Ethiopia does not come under the
55. Which of the following country is not larger Mediterranean climate region. The country has a diverse
than India in terms of Area? climatic conditions ranging from equatorial rainforest
(a) Russia (b) Brazil with high rainfall and humidity in the south and
(c) Canada (d) France southwest to the Afro alpine on the summits of the
Simian and Bale mountains to desert like conditions in
Ans. (d): France is NOT larger than India in terms of area. the north-east east and south-east lowlands.
Country Area
62. Which of the following pair is not correctly
India - 3.287 Million Km2
matched?
France - 551,695 km2
(a) Bolivia - Tin
Russia - 17.09 Million km2
(b) Brazil - Iron
Brazil - 8.51 Million Km2
(c) Mexico - Silver
Canada - 9.70 Million Km2 (d) Peru - Gold
UP RO/ARO (Pre) Exam 2013 189 YCT
CLICK HERE FOR FREE MATERIAL

Ans. (c) : At the time this question was asked, all the Ans. (d) : U.S.A. has the largest share in the
options were correctly matched. International trade.
As per the data released in 2017-18 the mineral 68. In Europe what is the place of France among
producing countries are as follows- Iron-ore production countries?
Tin- China, Indonesia, Peru etc. (a) First (b) Second
Iron are - Australia, Brazil, China, India (c) Third (d) Fourth
Silver - Mexico, China, Peru etc. Ans. (d) : In Europe, France ranked fourth among the
Gold - China, Australia, Russia etc. Iron-Ore production countries.
63. Which of the following nation is the world's 69. The twin ports which are located at both the
largest producer of Iron-ores. ends of Suez Canal are?
(a) China (b) India (a) Cairo and Alexandria
(c) Brazil (d) France (b) Suez and Cairo
Ans. (a) : China is the world's largest producer of Iron- (c) Cairo and Port Saed
ores in the world at the time this question was asked. (d) Port Said and Suez
Currently, as per the latest data the largest iron ore
producing country is Australia (900 Mnt.) followed by Ans. (d) : The twin ports which are located at both the
Brazil (400Mnt.) which is followed by China (340 Mnt.). ends of Suez Canal are Port Saed and Suez.
The Canal extends from the northern terminus of Port
64. Which of the following woolen clothes
manufacturing centers is associated to Saed to the Southern Terminus of Port Tewfik at the
Germany? City of Suez
(a) St. Petersburg (b) Wuppertal 70. In which country market oriented Iron & Steel
(c) Bradford (d) Prato Industries are found?
Ans. (b) : Woolen clothes manufacturing centre (a) China (b) India
associated to Germany is Wuppertal. (c) Japan (d) United Kingdom
65. Which is the largest Bauxite producing country Ans. (c) : Japan is market oriented Iron and steel
in the world? industry. It is having all facilities required for the
(a) Australia (b) Brazil growth of iron-steel industry.
(c) China (d) India 71. Which of the following country is the largest
Ans. (a) : Australia is the largest Bauxite producing producer of wool in the world?
country in the world. While Odisha is India's largest (a) China
bauxite producer state. (b) United States of America
66. Match List-I with List-II and choose the (c) Australia
correct answer from the codes given below: (d) United Kingdom
List-I List-II Ans. (a) : China is the largest producer of wool in the
(River) (Dam) world, which is followed by Australia.
A. Colorado 1. Aswan 72. Match List-I with List-II and choose the
correct answer from the codes given below:
B. Damodar 2. Kariba
List-I List-II
C. Nile 3. Panchet Hill (Crop) (Nation)
D. Zambezi 4. Hoover A. Rubber 1. Mauritius
Codes: B. Coffee 2. Italy
A B C D
C. Olive 3. Columbia
(a) 4 3 1 2
(b) 1 2 3 4 D. Sugarcane 4. Malaysia
(c) 3 4 1 2 Codes:
(d) 1 3 4 2 A B C D
Ans. (a) : The correct match is as follows- (a) 1 2 3 4
List-I List-II (b) 4 3 2 1
River Dam (c) 4 3 1 2
A. Colorado - Hoover (d) 3 4 2 1
B. Damodar - Panchet Hill Ans. (b) : The correct match is as follows-
C. Nile - Aswan List-I List-II
(Crop) (Nation)
D. Zambezi - Kariba
A. Rubber - Malaysia
67. Which country has the largest share in the B. Coffee - Columbia
International trade?
(a) China (b) France C. Olive - Italy
(c) Germany (d) U.S.A. D. Sugarcane - Mauritius
UP RO/ARO (Pre) Exam 2013 190 YCT
CLICK HERE FOR FREE MATERIAL

73. Which of the following country is the world's Ans. (b) : The correct decadal growth rate of Indian
largest producer of diamonds? population during the 2001-11 decade is 17.64%
(a) Australia (b) Venezuela 80. As per the 2001 census the literacy rate was
(c) Russia (d) Botswana highest in which of the following Union
Ans. (c) : Russia is the world's largest producer of Territory of State?
diamonds. (a) Delhi (b) Chandigarh
In India Diamond is found only in the Panna district of (c) Lakshadweep (d) Pondicherry
Madhya Pradesh. Ans. (c) : As per 2001 census. The literacy rate was
74. Which of the following tribes are found in the highest in the UT of Lakshadweep. As per 2011 census.
Kerala State? Highest literacy rate - Lakshadweep (91.85%).
(a) Chenchu (b) Lepcha Highest Literacy Rate State - Kerala (94%)
(c) Dafla (d) Darfur Least Literacy Rate State - Bihar (61.8%)
Ans. (a) : Chenchu tribes are found in the Kerala state. Least Literacy Rate UT - Dadra and Nagar Haveli
They are part of Particularly Vulnerable Tribal Group. 81. In the demographic history of India, the year of
(PVTG) Great Divide was?
Lepcha - Sikkim (a) 1921 (b) 1947
Darfur and Dafla - Arunachal Pradesh (c) 1951 (d) 1982
75. If the birth and Death rate are same and there Ans. (a) : In the demographic history of India, the year
is no Internal or External Migration then there of Great Divide was 1921. This year was the defining
is no change in the population. This is called as- year for Indian demography.
(a) Fixed Population Constitutional Growth of India
(b) Zero Population Growth 82. Who said that- "Constituent Assembly was
(c) Dynamic Population Congress and Congress was India?
(d) Stability of Population
Ans. (b) : Zero population growth is the absence of (a) Austin
population growth in which equal birth and death rates (b) C.R. Atlee
create a stable human population. (c) Winston Churchill
76. According to the 2001 Census, the infant (d) Lord Mountbatten
mortality rate was minimum in which state?
Ans. (a) : Granville Austin said "Constituent Assembly
(a) Kerala was Congress and Congress was India.
(b) Himachal Pradesh
(c) Karnataka 83. Who is the author of the book- "Politics in
India"?
(d) Jammu & Kashmir
(a) Vidyut Chakravarti
Ans. (a) : According to the 2001 census, the infant (b) Rajni Kothari
mortality rate was minimum in Kerala. (c) Romilla Thapar
Infant mortality rate is defined as the infant deaths per (d) A.K. Dubey
thousand live births.
The IMR for India as per 2011 census was 44. Ans. (b) : The author of the book- "Politics in India" -
Rajni Kothari.
77. Which state in India started Death census first?
(a) Karnataka (b) Kerala Legislature : Cabinet
(c) Andhra Pradesh (d) Himachal Pradesh 84. The Chairman of Rajya Sabha-
Ans. (a) : The Karnataka state of India has started the (a) Appointment by President
Death Census first. (b) Elected by the Parliament
(c) Vice-President is the ex-officio Chairman
78. Which of the following pairs is not correctly
matched? (d) Is elected by the members of state legislature
council
(a) 1951 - 946
(b) 1971 - 930 Ans. (c) : The Vice President is the ex-officio Chairman
(c) 1991 - 927 of the Rajya Sabha.
(d) 2011 - 925 Currently, Jagdeep Dhankar is the VP and the ex-officio
Chairman of Rajya Sabha.
Ans. (d) : The sex ratio of India in various census-
1951 - 946 Election Commission, Official Language
1971 - 930 85. In India, the right to vote and right to be
1991 - 927 elected is a-
2011 - 943 (a) Fundamental Right
(b) Constitutional Right
79. Which of the following was the correct decadal
(c) Natural Right
growth rate of Indian population during the
2001-11 decade? (d) Legal Right
(a) 17.11% (b) 17.64% Ans. (d) : In India, the right to vote and right to elected
(c) 16.74% (d) 16.05% is a legal right.
UP RO/ARO (Pre) Exam 2013 191 YCT
CLICK HERE FOR FREE MATERIAL

86. An amendment in the constitution can be Ans. (c) : India had constituted a committee in order to
initiated- set new standards of living of population living below
(a) Only in Lok Sabha the poverty line. This committee was chaired by Suresh
(b) Only in Rajya Sabha Tendulkar.
(c) Only in State Legislative Assemblies The Tendulkar committee in 2011-12 estimated poverty
(d) In any house of Parliament lines and poverty ratio based on the monthly per capita
Ans. (d) : An amendment in the Constitution can be consumption expenditure. Accordingly, the poverty line
initiated in any house of the parliament. The bill can be at all India level was estimated as monthly per capita
introduced either by a minister or by a private member consumption expenditure of Rs. 816 for rural areas and
and does not require prior permission of the President. Rs. 1000 for urban areas.
87. From which of the following, an increase in the 92. The Tendulkar Committee estimated which of
inclusive growth cannot be expected? the following ratio of Indian population living
(a) High growth rate in National Income below the Poverty Line?
(a) 27.2 (b) 37.2
(b) Rural Development
(c) Agricultural Development (c) 22.7 (d) 32.7
(d) Adequate credit facility to farmers Ans. (b) : As per the Tendulkar committee report, the
percentage of the population below the poverty line in
Ans. (a) : High growth rate in National income is not 2004-05 was 37.2%. The percentage of poor in rural
the clear indicator of increase in the inclusive growth areas was estimated at 41.8% and 25.7% in urban areas.
while Rural Development, Agricultural Development &
adequate credit facility to farmers enhances inclusive 93. The percentage of the poverty ratio which is
growth. Inclusive growth is the economic growth that is armed to be reduced by the year 2012 in the
distributed fairly across society and creates eleventh five year plan is-
opportunities for all. (a) 2.0% (b) 2.5%
88. Which among the following, is an obstacle in (c) 10.0% (d) 15.0%
the process of Financial Inclusion? Ans. (c) : Eleventh Five Year Plan is the economic and
(a) Low Income (b) Illiteracy development plan of India for the period 2007-12. The
(c) Lack of Banking branch (d) All of them percentage of the poverty ratio which is armed to be
reduced by the year 2012 in the eleventh five year plan
Ans. (d) : According to the definition of World Bank, is 10.0%.
"Financial inclusion means that individuals and
businesses have access to useful and affordable 94. In the year 2010-11 at the 2004-05 price level
financial products and services that meet their needs, what was the per capita income in India?
transactions, Payments, savings, credit and insurance (a) `30,525 (b) `33,626
delivered in a responsible and sustainable way." (c) `34,443 (d) `35,993
Low income, illiteracy, lack of banking branch is an Ans. (d) : In the year 2010-11 at the 2004-05 price
obstacle in the process of financial inclusion. level. The per capital income in India is Rs. 35,993.
89. In Indian model of Development Safeguards 95. Consider the following statements related to
the interests of? Public Distribution System:
(a) Person (b) State 1. Its main aim was to provide essential
(c) Person and State both (d) None of them commodities at subsidized rates.
Ans. (c) : In the Indian Model of development, it 2. This was started as a price stablisation
safeguards the interests of both the person and the state. programme.
90. In India what is the basis of poverty 3. Food subsidy is granted/provide by state
estimation? governments.
(a) Per Capita Which of the above statements are true?
(b) Per Capita Age (a) 1, 2 & 3 (b) 1 & 2
(c) Consumption Expenditure of Family (c) 1 & 3 (d) Only 3
(d) None of them Ans. (b) : PDS is operated under the joint responsibility
Ans. (c) : In India, the basis of poverty estimation is of the Central and the State/UT Governments. Food
consumption expenditure of family. The Planning subsidy is granted/provided by the central government.
Commission estimates levels of poverty in the country While the operational responsibility including
on the basis of consumer expenditure survey by the identification of eligible families, issue of Ration Cards
National Sample Survey Office (NSSO) of the Ministry etc. lies with the state governments. Rest of the
of Statistics and Programme Implementation. statements are true regarding PDS.
91. In order to set new standards of living of 96. Which one among the following is exhibited as
population living below the Poverty Line Govt. a percentage of G.D.P. against the public
of India had constituted a committee who was expenditure on health for the year 2009-10 in
the chairman of this committee? India?
(a) Nirmala Deshpande (b) V. Sidharth (a) 0.59% (b) 1.09%
(c) Suresh Tendulkar (d) Prof. Jankiraman (c) 1.59% (d) 2.09%
UP RO/ARO (Pre) Exam 2013 192 YCT
CLICK HERE FOR FREE MATERIAL

Ans. (b) : The percentage of G.D.P. against the public Ans. (c) : The National Rural Health Mission (NRHM)
expenditure on health for the year 2009-10 in India is was launched on 12th April 2005, to provide accessible,
exhibited as 1.09%. affordable and quality health care of the rural
97. Which among the following does not begin and population, especially the vulnerable groups.
manage the mid-day meal scheme? The National Rural Health Mission was started in the
(a) School Management Tenth five year plan (2002-2007).
(b) Panchayati Raj Sanstha (System) 101. Aam Admi Bima Yojana (A.I.B.Y.) was
(c) Voluntary Service Group initiated on-
(d) Contractor (a) 15th March, 2009 (b) 15th Jan, 2008
th
Ans. (d) : The Mid-day Meal Scheme is a school meal (c) 15 Aug, 2007 (d) 2nd October, 2007
programme in India designed to better the nutritional Ans. (d) : Aam Aadmi Bima Yojana has been launched
standing of school age children nationwide. under the ministry of finance. It is a social security
It was launched in the year 1995. The name of the scheme that was launched on 2nd October 2007.
scheme has been changed to PM-POSHAN (Pradhan The scheme is administered through the LIC that
Mantri Poshan Shakti Nirman) Scheme, in September provides death and disability cover to persons between
2021, by MoE (Ministry of Education), which is nodal the age group of 18 years to 59 years.
ministry for the scheme. The scheme is Executed and 102. Which of the following Public Sector
managed by the school management, Panchayati Raj undertaking has not so far attained the status
System, voluntary service group. contractors are not of 'Maharatna', which hitherto been granted to
involved in this. four Navratnas?
98. Match List-I with List-II and choose the (a) SAIL (b) B.E.L.
correct answer from the codes given below: (c) O.N.G.C. (d) N.T.P.C.
List-I List-II Ans. (b) : "Maharatna" companies are those whose, (1)
A. Sarva Shiksha Abhiyan 1. 1987 Net profit for there consecutive years is Rs. 5,000 crore
(2) average annual turnover of Rs. 25,000 crore for
B. Saakshar Bgarat Mission 2. 1988 three years or (3) should have an average annual net
C. Operation Black Board 3. 2001 worth of Rs. 15,000 crore for three years.
D. National Literacy 4. 2009 As of 2022, India has 11 Maharatna Companies Viz.,
Mission NTPC, ONGC, SAIL, BHEL, IOCL, HPCL, CIL, GAIL,
BPCL, POWERGRID & PFC Ltd. However, BEL is an
Codes: Indian Govt. owned aerospace and defence electronics
A B C D company, not included in the "Maharatna" List.
(a) 3 4 1 2 103. In 1818, the first cotton Textile Factory, was
(b) 4 3 2 1 started in-
(c) 1 2 3 4 (a) Fort Gloster in West-Bengal
(d) 1 2 4 3 (b) Mumbai of Maharashtra
Ans. (a) : The correct match is as follows- (c) Ahmedabad in Gujarat
List-I List-II (d) Kanpur of Uttar Pradesh
A. Sarva Shiksha Abhiyan - 2001 Ans. (a) : In 1818, the first cotton textile factory, was
started in Fort Glaster Nar and Kolkata in West Bengal.
B. Saakshar Bharat Mission - 2009 It was a commercial failure.
C. Operation Black Board - 1987 104. Who among the following was the chairman of
D. National Literacy Mission - 1988 the committee formed on 'Financial Inclusion'
in Jan 2005?
99. Which one among the following does not come (a) C. Rangarajan (b) D. Subba Rao
under the Minimum Need Programme?
(c) M.S. Ahluwalia (d) Kamalnath
(a) Rural Water Supply
(b) Social Forestry Ans. (a) : The chairman of the financial inclusion
(c) Primary Education committee which was formed in 2005 was C. Rangarajan.
The concept of financial inclusion was given by R.B.I.
(d) Improvement of slum settlements in Cities
with the objective of establishing proper financial
Ans. (b) : The minimum needs programme was institutions to cater the needs of the poor people.
introduced in the first year of the fifth five year plan.
105. Indian Diamond Institute is established at?
The objective of the programme is to establish a
network of basic services and facilities of social (a) New Delhi (b) Surat
consumption in all the areas upto nationally accepted (c) Mumbai (d) Jaipur
norms, within a specified time frame. Social forestry Ans. (b) : Indian Diamond Institute is established at
does not come under the minimum Need programme. Surat in Gujarat state of India.
100. The National Rural Health Mission was started 106. Which of the following is an example of 'Foot
in which five-year plan? Loose' industry?
(a) Sixth (b) Ninth (a) Oil Refining (b) Sugar
(c) Tenth (d) Eleventh (c) Software (d) Aluminium
UP RO/ARO (Pre) Exam 2013 193 YCT
CLICK HERE FOR FREE MATERIAL

Ans. (c) : Foot loose industry are those that can be placed 114. The Light Year is a unit of measuring-
and located at any location without effect from factors of (a) Intensity of light (b) Time
production such as resources, land, labour and capital. (c) Distance (d) Velocity of Light
'Software' is an example of 'foot loose' industry. Ans. (c) : Light year is the distance light travels in one
107. One Nanometer is- year. 1 light year is about 6 trillion miles (9 trillion km).
(a) 10–6 cm (b) 10–7 cm 115. The element found in highest quantity in
–8
(c) 10 cm (d) 10–9 cm human body is-
Ans. (b) : One Nanometer is 10–9m or 10–7 cm. (a) Iron (b) Sodium
108. 'Mirage' is an example of? (c) Phosphorus (d) Calcium
(a) Refraction Ans. (d) : The human body is approximately 99%
(b) Total Internal Reflection comprised of 6 elements- Oxygen, Hydrogen, Nitrogen,
(c) Scattering Carbon, Calcium and Phosphorous. Another 5 elements
(d) Diffraction make up to 0.85% of the remaining mass- Sulphur,
potassium, All of these 11 elements are essential elements.
Ans. (b) : Mirage, in optics is the deceptive appearance Therefore, among the following the element found in
of a distant object or objects caused by the bending of highest quantity is calcium.
light rays (refraction) in layers of air or varying density.
Mirage is an example of Total Internal Reflection. 116. The sky appears blue because-
(a) Blue light is scattered most
109. Sodium metal is stored by dipping into which
of the following? (b) Red light is scattered most
(a) Kerosene (b) Air (c) Blue light is absorbed least in Atmosphere
(c) Water (d) Ammonia (d) Red light is absorbed least in atmosphere
Ans. (a) : Sodium metal is very reactive and thus reacts Ans. (a) : Blue light is scattered most in the sky which
vigorously with Oxygen and Water. Therefore, Sodium is the reason that sky appears blue.
metal is stored by dipping into the kerosene. 117. A polluted atmosphere is cleared by which of
110. Which among the following is a by product of the following?
Petroleum Refining? (a) Oxygen (b) Rain
(a) Petrol (b) Charcoal (c) Nitrogen (d) Air
(c) Coal (d) Asphalt Ans. (b) : A polluted atmosphere is cleared by Rain.
Ans. (d) : A petrol refinery is an industrial process plant As a raindrop falls through the atmosphere, it can attract
where petroleum is transformed and refined into useful tens to hundreds of tiny aerosol particles to its surface
products such as gasoline diesel fuel, asphalt base, fuel before hitting the ground.
oils, heating oil, Kerosene, LPG and petroleum. The process by which droplets and aerosols attract is
Therefore, asphalt is a by product of petroleum refining. coagulation, a natural phenomenon that can act to clear the
air of pollutants like soot, sulfates and organic particles.
111. Dry Ice is a-
(a) Vapour (b) Ice at 0ºC 118. Which among the following is not helpful in
(c) Solid CO2 (d) Ice treated with an restoring the ecological balance?
hydrous calcium chloride (a) Deforestation
Ans. (c) : Dry ice is the solid form of Carbon-dioxide (b) A forestation
(CO2). It's called "dry ice" because it does not melt like (c) Rain water management
wet ice. Instead dry ice converts into carbon dioxide gas. (d) Biosphere reserve
112. Which of the following process occurs in a Bio- Ans. (a) : Deforestation is the purposeful clearing of
Gas Plant? forested land. It will not be helpful in restoring
(a) Fermentation (b) Reduction ecological balance. While, Aforestation, Rain water
(c) Hydrogenation (d) Polymerisation management and biosphere reserve are helpful in
restoring ecological balance.
Ans. (a) : Anaerobic fermentation is the process which
occurs in a Bio-Gas Plant. 119. In India, Environment (Conservation) Act was
passed in?
It occurs in two phases-
(a) 1981 (b) 1986
(1) Hydrolysis Phase - Transformation of organic matter
into CO2, hydrogen and fatty acids. (c) 1995 (d) 2000
(2) Methanogenic Phase - Here fatty acids decompose Ans. (b) : Environment Conservation Act was enacted
to become methane. with the main objective to provide the protection and
113. For the preservation of food substances which improvement of the environment and for matters
chemical is used? connected therewith. It was launched in the year 1986.
(a) Sodium (b) Caustic Soda 120. Which of the following is not a social
(c) Sodium Benzoate (d) Sulphuric Acid conservation programme?
Ans. (c) : Sodium Benzoate is used for the preservation (a) Aam Admi Bima Yojana
of food substances. It is the sodium salt of benzoic acids (b) National Health Insurance Scheme
which appears as a white crystalline chemical with the (c) Swarn Jayanti Urban Employment Scheme
formula (C6H5COONa). (d) Unorganised Workers Social Security Act
UP RO/ARO (Pre) Exam 2013 194 YCT
CLICK HERE FOR FREE MATERIAL

Ans. (c) : Swarna Jayanti Urban Employment Scheme 100


was an employment generation programme and NOT a ⇒ T=
5
social conservation programme.
It is a centrally sponsored scheme which came into ⇒ T = 20 years.
effect on 1 December 1997. 127. The population of any village reduces at the
121. Which one among the following is not related rate of 20% per year. It is population was
to Environmental Security? 10,000 2 years ago, then what is its present
(a) Sustainable Development population?
(b) Reduction in Poverty (a) 10,000/64 (b) 7600
(c) Air Conditioning (c) 6000 (d) 6400
(d) Use of Paper bags Ans. (d) : Let present population of village = x
Ans. (b) : Environmental security is comprised of According to the question-
2
restoration, environmental security technology and  20 
international activities. x = 10000  1 − 
 100 
Sustainable development, Air condition and use of
16
paper bags are related to environmental security. While ⇒ x = 10000 ×
reduction in poverty is not related to it. 25
122. Permanent Development is an event of Inter- ⇒ x = 6400
Generation Sensibity in context of use of?
(a) Natural Resources (b) Physical Resources 128. Which of the following countries was affected
(c) Industrial Resources (d) Social Resources by the hurricane - 'Sandy' in 2012?
(a) Mexico (b) Columbia
Ans. (a) : Permanent Development is an event of Inter-
Generation sensibility in context of use of Natural Resources. (c) U.S.A. (d) Brazil
123. In which of the following states highest Human Ans. (c) : U.S.A. was affected by the hurricane "Sandy"
Development Index was recorded in year 2008? in 2012.
(a) Uttar Pradesh (b) Jharkhand 129. Recently scientists have managed to discover
(c) Odissa (d) Bihar that protein which produces the dangerous
Alzheimer Disease of human brain?
Ans. (a) : Highest Human Development in the year
2008 was recorded in the state of U.P. (a) Amylide Monoprotein
In HDI 2022, India ranked 132/191 and Kerala ranked (b) Amylide Multiprotein
first among the states. (c) Amylide Precursor Protein
124. The Uttar Pradesh Government issued a Card (d) Amylide Leptic Protein
to facilitate easy access into its state govt. Ans. (c) : The dangerous Alzheimer Disease of human
offices for the industrialists and industrial brain is produced by the Amylide Precursor Protein.
organizations. The name of that Card is- It is a progressive disease that destroys memory and
(a) Silver Card (b) Gold Card other important mental functions. Brain cell connection
(c) Platinum Card (d) Industrial Card and the cells themselves degenerate and die, eventually
Ans. (b) : Gold Card is issued by U.P. Govt. to destroying memory and other important mental
facilitate easy access to its State Government Offices functions.
for the industrialists and industrial organization. 130. The well known God particle is a-
125. In Uttar Pradesh a Lion Safari is being (a) Neutrino (b) Leptons
established at? (c) Higgs Boson (d) Mesons
(a) Balrampur (b) Maharajganj Ans. (c) : The well known God Particle is a Higgs
(c) Etawah (d) Pilibhit Boson is the fundamental particle associated with the
Ans. (c) : In U.P. a Lion Safari is being established at Higgs field, a field that gives mass to other fundamental
Etawah. It is one of the biggest safari in Asia. particles such as electrons and quarts.
126. An amount will become two times in how many 131. Match List-I with List-II and choose the
years at 5% simple interest rate annually? correct answer from the codes given below:
(a) 10 (b) 15 List-I List-II
(c) 20 (d) 30
A. Curiosity Rover 1. Tablet
Ans. (c) : Let amount = x
B. Messenger 2. An Unmanned air craft
Amount × Rate × Time
∵Simple Interest = of D.R.D.O.
100
C. Rustam-1 3. A mars planet probe
According to the question- space shuttle of U.S.A.
x ×5× T D. Aakash-2 4. A mercury planet probe
(2x – x) =
100 space shuttle of NASA
UP RO/ARO (Pre) Exam 2013 195 YCT
CLICK HERE FOR FREE MATERIAL

Codes: 136. Chernobyl Nuclear disaster occurred in?


A B C D (a) France
(a) 3 4 2 1 (b) Japan
(b) 4 3 2 1 (c) Ukraine
(c) 3 4 1 2 (d) Germany
(d) 1 2 3 4 Ans. (c) : Chernobyl Nuclear disaster occurred in
Ans. (a) : The correct match is as follows- Ukraine. It took place in the year 1986 on April 26.
List-I List-II 137. In India which of the following places was most
A. Curiosity Rover - A mars planet probe space severely affected by the Tsunami of 2004?
shuttle of U.S.A. (a) Picchavaram
B. Messenger - A mercury planet probe space (b) Bhutarkanika
shuttle of NASA (c) Muthupet
C. Rustam-1 - An Unmanned air craft of (d) Nagapattinam
D.R.D.O. Ans. (d) : In India, Nagapattinam, in Tamil Nadu was
D. Aakash-2 - Tablet most severely affected by the Tsunami of 2004.
132. The modern Brahmos supersonic missile 138. According to the report of United Nations
technology has been developmented by India in Development Programme, published in 1990
joint collaboration of which of the following? which of the following is not included in the
(a) U.S.A. (b) Russia estimation of Human Development Index?
(c) France (d) Japan (a) Per Capita Income
(b) Urbanisation
Ans. (b) : Brahmos missile is a joint collaboration of
Indian and Russia. It is a medium-range stealth (c) Literacy
supersonic cruise missile. It is the fastest supersonic (d) Life Expectancy
cruise missile in the world. Ans. (b) : In 1990, the UN Development Programme
133. India was not affected by which of the launched the first Human Development Report.
following cyclones? This report is about people and about how development
(a) Aayla (b) Nargis enlarges their choices.
(c) Thane (d) Neelam HDI depends on four factors-mean years of schooling,
expected years of schooling, life expectancy at birth,
Ans. (b) : Nargis cyclone was a severe cyclonic storm.
and gross national income (GNI) per capita.
It affected Myanmar, badly. Whereas cyclones Aayla,
Uranisation is NOT included in the estimation of HDI.
Thane and Neelam affected India.
139. Who among the following focused its attention
134. Lakhs of people died of hunger due to leap
first towards the humanitarian side of
forward Policy in?
development?
(a) Russia (b) Vietnam
(a) United Nations
(c) China (d) Germany
(b) Integrated Rural Development Plan
Ans. (c) : Lakhs of people died of hunger due to leap (c) World Development Report
forward policy in China. The central idea behind the (d) Community Development Scheme
great leap was that rapid development of China's
Ans. (a) : United Nations focused its attention first
agricultural and industrial sectors should take place in
towards the humanitarian side of development. It has
parallel.
done so by releasing Human Development Index.
135. Which of the following is correctly matched? The UNDP releases HDI, which is a static composite index
(a) Nag - Anti Tank Missile of life expectancy, education and per capita income.
(b) Trishul - Surface to surface missile 140. When did Indian Postal Department
(c) Prithvi - Surface to Air Missile introduced/started the "Fast Postal Service" in
(d) Pinaka - Light Air fighter Aircraft order to compete with Courier Service?
Ans. (a) : Nag - Anti Tank Missile is correctly matched. (a) 1988 (b) 1987
While rest of the correct match are as follows- (c) 1989 (d) 1986
Trishul - Surface to Air Ans. (d) : The Indian Postal Department started the fast
Prithvi - Surface to Surface postal service in order to compete with courier service
Pinaka - Multi Barrel rocket launcher in the year 1986.
UP RO/ARO (Pre) Exam 2013 196 YCT
CLICK HERE FOR FREE MATERIAL

Gòej ØeosMe meceer#ee DeefOekeâejer/meneÙekeâ meceer#ee DeefOekeâejer (Øeer.) hejer#ee, 2013


meeceevÙe efnvoer
nue ØeMve-he$e
heÙee&ÙeJeeÛeer JÙeeKÙee – lejbie Meyo keâe heÙee&ÙeJeeÛeer Meyo Te|ce nw ve efkeâ heg<keâj, ketâue Deewj
peuepe~ Fmekesâ DevÙe heÙee&ÙeJeeÛeer Meyo - uenj, efnueesj, JeerefÛe Deeefo nQ~ peyeefkeâ
1. `Ùecegvee' keâe heÙee&ÙeJeeÛeer nw – ‘peueefOe’ meeiej keâe , ‘heg<keâj’ leeueeye keâe leLee ‘ketâue’ efkeâveeje keâe heÙee&Ùe nw~
(a) keâeefueeqvoveer (b) YeeieerjLeer (c) Ùeeefceveer (d) keâeefuevoer 8. `pebieue' Meyo keâe heÙee&ÙeJeeÛeer nw –
Gòej – (d)
(a) Øeceeso (b) efJeßeeeqvle (c) keâevleej (d) efoJe
JÙeeKÙee – efoÙes ieÙes efJekeâuheeW ceW Ùecegvee keâe heÙee&ÙeJeeÛeer `keâeefuevoer' nw~ peyeefkeâ Gòej – (c)
Ùeeefceveer, jeef$e keâe heÙee&ÙeJeeÛeer nw leLee ‘YeeieerjLeer’ iebiee keâe heÙee&ÙeJeeÛeer JÙeeKÙee – pebieue Meyo keâe heÙee&ÙeJeeÛeer Meyo ‘keâevleej’ nw, ve efkeâ Øeceeso,
nw~ Ùecegvee kesâ DevÙe heÙee&ÙeJeeÛeer Meyo nQ ─ metÙe&leveÙee, Yeevegpee, metÙe&meglee,
efJeßeeeqvle Deewj efoJe~ pebieue kesâ DevÙe heÙee&ÙeJeeÛeer Meyo nQ - efJeefheve, keâeveve,
lejefCelevegpee Deeefo~
Jeve, DešJeer, keâevleej Deeefo~
2. efvecveefueefKele ceW keâewve mee Meyo `efveMeerLe' keâe heÙee&ÙeJeeÛeer veneR nw?
9. `›eâesOe' Meyo keâe heÙee&ÙeJeeÛeer nw –
(a) jeef$e (b) jpeveer (c) lece (d) efveMee
(a) mebleehe (b) Dece<e& (c) JewcevemÙe (d) Yeerefle
Gòej – (c)
Gòej – (b)
JÙeeKÙee – efvee|o„ efJekeâuheeW ceW ‘lece’ DebOekeâej keâe heÙee&Ùe nw peyeefkeâ efveMeerLe
kesâ heÙee&Ùe jeef$e, jpeveer Deewj efveMee nw~ Dele: efJekeâuhe (c) lece efveMeerLe keâe JÙeeKÙee – ›eâesOe Meyo keâe heÙee&ÙeJeeÛeer Meyo ‘Dece<e&’ nw, ve efkeâ mebleehe, JewcevemÙe
heÙee&ÙeJeeÛeer Meyo veneR nw~ Deewj Yeerefle~ peyeefkeâ mebleehe keâe DeeMeÙe ‘og:Ke’, JewcevemÙe keâe DeeMeÙe ‘F&<Ùee&’
leLee Yeerefle keâe DeeMeÙe ‘[j’ neslee nw~
3. `metÙe&' keâe heÙee&ÙeJeeÛeer nw –
10. `efyepeueer' Meyo keâe heÙee&ÙeJeeÛeer veneR nw –
(a) YeemJej (b) ceele&C[ (c) ØekeâeMe (d) lespe
Gòej – (b) (a) efJelegb[e (b) oeefceveer (c) ÛebÛeuee (d) leefÌ[le
Gòej – (a)
JÙeeKÙee – metÙe& keâe heÙee&ÙeJeeÛeer ‘ceele&C[’ nw ve efkeâ ØekeâeMe, lespe SJeb
YeemJej nw~ metÙe& kesâ DevÙe heÙee&ÙeJeeÛeer Meyo nQ – lejefCe, DeeefolÙe, efoJeekeâj, JÙeeKÙee – efyepeueer Meyo keâe heÙee&ÙeJeeÛeer Meyo ‘efJelegb[e’ veneR nw~ oeefceveer,
ØeYeekeâj, efovekeâj, Yeemkeâj Deeefo~ ÛebÛeuee Deewj leefÌ[le efyepeueer Meyo kesâ heÙee&ÙeJeeÛeer nQ~ Fmekesâ DevÙe heÙee&ÙeJeeÛeer
Meyo - Ûeheuee, #eCeØeYee, meewoeefceveer Deeefo nQ~ peyeefkeâ ‘efJelegb[e’ neLeer keâe
4. efvecveefueefKele ceW keâewve mee Meyo `DejefJevo' keâe heÙee&ÙeJeeÛeer veneR nw? heÙee&ÙeJeeÛeer Meyo nw~
(a) efceefuevo (b) hebkeâpe (c) peuepe (d) Decyegpe
Gòej – (a) efJeueesce Meyo
JÙeeKÙee –efoÙes ieÙes efJekeâuhe ceW ‘efceefuevo’ Meyo DejefJevo keâe heÙee&ÙeJeeÛeer 11. `ØelÙe#e' Meyo keâe efJeueesce nw –
Meyo veneR nw~ DejefJevo keâe DeeMeÙe keâceue mes nw~ DejefJevo kesâ heÙee&ÙeJeeÛeer
(a) Dehejes#e (b) hejes#e (c) megboj (d) ØelÙeÙe
Meyo nQ– keâceue, hebkeâpe, peuepe, Decyegpe, mejefmepe, jepeerJe, DecYeespe,
Gòej – (b)
mejespe, veefueve Deeefo~ peyeefkeâ ‘efceefuevo’ YeeQje keâe heÙee&ÙeJeeÛeer nw~
JÙeeKÙee – ØelÙe#e keâe efJeueesce Meyo ‘hejes#e’ nw ve efkeâ Dehejes#e, megboj Deewj
5. `heÙee&ÙeJeeÛeer' Meyo keâe DeLe& nw –
ØelÙeÙe nw~ Dehejes#e keâe DeLe& Yeer ØelÙe#e nw~
(a) efJeueesceJeeÛeer (b) ØeefleefJeueesceJeeÛeer
(c) meceeveeYeeme (d) meceeveeLeea 12. `meeceevÙe' Meyo keâe efJeueesce nw –
Gòej – (d) (a) ßes‰ (b) meJe&%e (c) meeOeejCe (d) efJeefMe„
JÙeeKÙee – heÙee&ÙeJeeÛeer Meyo keâe DeeMeÙe ‘meceeveeLeea’ Meyo mes neslee nw~ Jes Gòej – (d)
Meyo efpevekesâ DeLeeX ceW meceevelee nes, heÙee&ÙeJeeÛeer Meyo keâns peeles nQ~ Fve MeyoeW JÙeeKÙee – meeceevÙe Meyo keâe efJeueesce Meyo ‘efJeefMe„’ nw~ ßes‰, meJe&%e Deewj
keâes meceeveeLeea Meyo Yeer keâne peelee nw~ meeOeejCe Fmekesâ efJeueesce Meyo veneR nQ, peyeefkeâ meeOeejCe keâe efJeueesce DemeeOeejCe,
6. `ØeeÛeer' keâe heÙee&ÙeJeeÛeer Meyo nw –
meJe&%e keâe efJeueesce De%e Deewj ßes‰ keâe efJeueesce efvecve neslee nw~
(a) ØeeÛeerve (b) Øeke=âle (c) hetJe& (d) Øe%ee 13. `Decej' Meyo keâe efJeueesce nw –
Gòej – (c) (a) ce=lekeâ (b) ce=lÙeg (c) cejCe (d) celÙe&
JÙeeKÙee – ØeeÛeer keâe heÙee&ÙeJeeÛeer Meyo hetJe& nw ve efkeâ ØeeÛeerve, Øeke=âle Deewj Gòej – (d)
Øe%ee nw~ JÙeeKÙee – Decej Meyo keâe efJeueesce ‘celÙe&’ nw ve efkeâ ce=lekeâ, ce=lÙeg Deewj cejCe nw~
7. `lejbie' Meyo keâe heÙee&ÙeJeeÛeer nw – 14. `Ghekeâej' Meyo keâe efJeueesce nw –
(a) heg<keâj (b) ketâue (c) peueefOe (d) Te|ce (a) efJekeâej (b) Deveghekeâej (c) Dehekeâej (d) eflejmkeâej
Gòej – (d) Gòej – (c)
UP RO/ARO (Pre) General Hindi 2013 197 YCT
CLICK HERE FOR FREE MATERIAL

JÙeeKÙee – Ghekeâej Meyo keâe efJeueesce ‘Dehekeâej’ nw ve efkeâ efJekeâej, Deveghekeâej JÙeeKÙee – efoÙes ieÙes efJekeâuheeW ceW ‘Mejledkeâeue ceW Ûebõcee keâer MeesYee osKeves
Deewj eflejmkeâej nw~ ÙeesiÙe nesleer nw’ Megæ JeekeäÙe nw~ DevÙe efJekeâuhe JÙeekeâjCe keâer Âef° mes ieuele
15. `efleefcej' Meyo keâe efJeueesce nw – nw, keäÙeeWefkeâ keâeue Deewj efove keâe ØeÙeesie Skeâ meeLe veneR neslee nw~
(a) Deeueeskeâ (b) efkeâjCe (c) jbieerve (d) jbienerve 23. keâewve mee JeekeäÙe Megæ nw?
Gòej – (a) (a) oner Kešdšer nw~ (b) oner Kešdše nw~
JÙeeKÙee – efleefcej Meyo keâe efJeueesce ‘Deeueeskeâ’ nw ve efkeâ efkeâjCe, jbieerve, (c) oner Kešeme nw~ (d) oner KešeF& nw~
Deewj jbienerve nw~ Gòej – (b)
16. `Gkeäle' Meyo keâe efJeueesce nw – JÙeeKÙee – efoÙes ieÙes efJekeâuheeW ceW efJekeâuhe (b) oner Kešdše nw, JeekeäÙe Megæ
(a) Devegkeäle (b) GheÙegkeäle (c) DevegheÙegkeäle (d) GheÙeg&keäle nw~ OÙeeleJÙe nes efkeâ oner Meyo hegefubueie kesâ DeLe& ceW ØeÙeesie efkeâÙee peelee nw~
Gòej – (a) otOe SJeb otOe mes yeves heoeLe& ØeeÙe: hegefubueie nesles nQ~
JÙeeKÙee – Gkeäle keâe efJeueesce `Devegkeäle' neslee nw, peyeefkeâ GheÙegkeäle keâe efJeueesce 24. keâewve mee JeekeäÙe Megæ nw?
`DevegheÙegkeäle' neslee nw~ (a) megjsMe keâes Skeâ heeleer efueKevee nw~
17. `efveo&Ùe' Meyo keâe efJeueesce nw – (b) megjsMe ves Skeâ heeleer efueKeveer nw~
(c) megjsMe kesâ efueS Skeâ he$e efueKeveer nw~
(a) mendÙe (b) meùoÙe (c) meoÙe (d) meYeÙe
Gòej – (c) (d) megjsMe kesâ efueS Skeâ he$e efueKevee nw~
Gòej – (d)
JÙeeKÙee – efveo&Ùe keâe efJeueesce `meoÙe' neslee nw ve efkeâ meùoÙe, meYeÙe Deewj
mendÙe~ JÙeeKÙee – efvee|o„ efJekeâuheeW ceW efJekeâuhe (d) megjsMe kesâ efueS Skeâ he$e efueKevee
nw, Megæ JeekeäÙe nw~ peyeefkeâ DevÙe efJekeâuhe JÙeekeâjCe keâer Âef„ mes $egefšhetCe& nQ~
18. `Gvceerueve' Meyo keâe efJeueesce nw –
(a) Devegceerueve (b) efveceerueve
25. keâewve mee JeekeäÙe Megæ nw?
(c) DeJeceerueve (d) ceerueve (a) Ùen DeeBKeeW mes osKeer Iešvee nw~
Gòej – (b) (b) Ùen DeeBKeeW osKeer Iešvee nw~
(c) Ùen DeeBKeeW Éeje osKeer ieF& Iešvee nw~
JÙeeKÙee–‘Gvceerueve’ Meyo keâe efJeueesce efveceerueve nesiee~ Mes<e efJekeâuhe Demebiele nw~
(d) Ùen DeeBKeeW Éeje osKeer megveer-megveer Iešvee nw~
19. `Glkeâ<e&' Meyo keâe efJeueesce nw– Gòej – (b)
(a) Dekeâ<e& (b) Deveglkeâ<e& (c) Dehekeâ<e& (d) Deekeâ<e& JÙeeKÙee – efJekeâuhe (b) Ùen DeeBKeeW osKeer Iešvee nw, Megæ JeekeäÙe nw~ OÙeeleJÙe
Gòej – (c) nes efkeâ DeeBKeeW keâer GheÙeesefielee osKeves ceW ner nesleer nw~ DeleSJe Fmekesâ meeLe efkeâmeer
JÙeeKÙee–‘Glkeâ<e&’ Meyo keâe efJeueesce ‘Dehekeâ<e&’ nesiee~ Mes<e efJekeâuhe Demebiele nQ~ Yeer keâejkeâ efJeYeefòeâ keâe ØeÙeesie GefÛele veneR nw~
20. `DeeefJeYee&Je' Meyo keâe efJeueesce nw – 26. DeMegæ Jele&veer Jeeuee Meyo nw –
(a) DeveeefJeYee&Je (b) efJeYeeJe (a) heejueewefkeâkeâ (b) efve<Ûes„ (c) ogOe&<e& (d) DeeMeerJee&o
(c) DeefJeYee&Je (d) eflejesYeeJe Gòej – (b)
Gòej – (d)
JÙeeKÙee – DeMegæ Jele&veer Jeeuee Meyo ‘efve<Ûes°’ nw, peyeefkeâ Fmekeâe Megæ ™he efveMÛes°
JÙeeKÙee – ‘DeeefJeYee&Je ’ keâe efJeueesce Meyo ‘eflejesYeeJe’ neslee nw, peyeefkeâ ‘efJeYeeJe’ neslee nw~ Mes<e efJekeâuheeW kesâ Meyo Jele&veer keâer Âef° mes Megæ nQ~
keâe efJeueesce DevegYeeJe neslee nw~
27. Megæ Jele&veer Jeeuee Meyo nw –
JeekeäÙe SJeb Jele&veer DeMegefæÙeeB (a) Oeesefyeve (b) Oeesefyeveer (c) Oeesyeveer (d) Oeesyeerve
Gòej – (a)
21. keâewve mee JeekeäÙe Megæ nw?
(a) DeeF&vmšerve kesâ heeme efJeue#eCe yegefæ Leer~
JÙeeKÙee – Megæ Jele&veer Jeeuee Meyo `Oeesefyeve' nw~ Mes<e efJekeâuheeW kesâ Meyo Demebiele
(b) DeeF&vmšerve kesâ heeme efJeefÛe$e yegefæ Leer~ nQ~
(c) DeeF&vmšerve kesâ heeme Deueewefkeâkeâ yegefæ Leer~ 28. Megæ Jele&veer Jeeuee Meyo nw –
(d) DeeF&vmšerve kesâ heeme owJeerÙe yegefæ Leer~ (a) YeieerjLeer (b) YeeieerjLeer (c) YeefiejLeer (d) YeeefiejLeer
Gòej – (a) Gòej – (b)
JÙeeKÙee – efoÙes ieÙes efJekeâuheeW ceW ‘DeeF&vmšerve kesâ heeme efJeue#eCe yegefæ Leer’ JÙeeKÙee – efoÙes ieÙes efJekeâuheeW ceW efJekeâuhe (b) `YeeieerjLeer' Megæ efJekeâuhe
Megæ JeekeäÙe nw~ Mes<e efJekeâuhe $egefšhetCe& nQ~ nw~ DevÙe leerveeW $egefšhetCe& nQ~ Mes<e efJekeâuheeW kesâ Meyo Demebiele nQ~
22. keâewve mee JeekeäÙe Megæ nw – 29. Megæ Jele&veer Jeeuee Meyo nw –
(a) Mejledkeâeueerve efoveeW ceW Ûebõcee keâer MeesYee osKeves ÙeesiÙe nesleer nw~
(a) efJeefjefnCeer (b) efJejnCeer (c) efJejefnCeer (d) efJeefjnCeer
(b) Mejod keâeue kesâ efoveeW ceW Ûevõcee keâer MeesYee osKeves ÙeesiÙe nesleer nw~
Gòej – (c)
(c) Mejled keâeue ceW Ûevõcee keâer MeesYee osKeves ÙeesiÙe nesleer nw~
(d) Mejled keâeue kesâ efoveeW ceW Ûevõcee keâer MeesYee osKeves ÙeesiÙe nesleer nw~ JÙeeKÙee – Megæ Jele&veer Jeeuee Meyo `efJejefnCeer' nw~ Mes<e efJekeâuheeW kesâ Meyo
Gòej – (c) $egefšhetCe& nQ~
UP RO/ARO (Pre) General Hindi 2013 198 YCT
CLICK HERE FOR FREE MATERIAL

Deveskeâ MeyoeW kesâ Skeâ Meyo `efpemekeâe pevce Úesšer peeefle (efveÛeues JeCe&) ceW ngDee nes' – kesâ efueS
39.
GheÙegkeäle Meyo nw –
30. DeMegæ Jele&veer Jeeuee Meyo nw – (a) DeC[pe (b) Metõ (c) DevlÙepe (d) DeÚtle
(a) ØepJeefuele (b) mecegppJeue Gòej – (c)
(c) GppJeue (d) mecegpJeue JÙeeKÙee – efpemekeâe pevce Úesšer peeefle ceW ngDee nes, kesâ efueS GheÙegkeäle Meyo
Gòej – (d) `DevlÙepe' neslee nw~ ‘efpemekeâe pevce DeC[s mes ngDee nes’ kesâ efueS Skeâ Meyo
JÙeeKÙee – DeMegæ Jele&veer Jeeuee Meyo ‘mecegpJeue’ nw efpemekeâe Megæ Jele&veer ‘DeC[pe’ neslee nw~
mecegppJeue nesiee DevÙe Meyo ØepJeefuele GppJeue Megæ Jele&veeriele Meyo nw~ 40. `pees meye kegâÚ peevelee nw' – kesâ efueS GheÙegkeäle Meyo nw –
31. `efpemekeâe Fueepe ve nes mekesâ' – Gmekesâ efueS GheÙegkeäle Meyo nw – (a) ef$ekeâeue%e (b) yeng%e (c) %eeveer (d) meJe&%e
(a) DemeeOÙe (b) og:meeOÙe Gòej – (d)
(c) meeOevenerve (d) ßecemeeOÙe JÙeeKÙee – pees meye kegâÚ peevelee nes Gmes `meJe&%e' keâne peelee nw~ peyeefkeâ
Gòej – (a) efpemes leerveeW keâeueeW keâe %eeve nes – ef$ekeâeue%e leLee ‘pees yengle kegâÚ peevelee nes’
JÙeeKÙee – ‘efpemekeâe Fueepe ve nes mekesâ' kesâ efueS GheÙegkeäle Meyo nw - `DemeeOÙe'~ kesâ efueS GheÙegòeâ Meyo meJe&%e nesiee~
32. `efpemes peerlee ve pee mekesâ' – Gmekesâ efueS GheÙegkeäle Meyo nw – lelmece SJeb leodYeJe
(a) DepesÙe (b) De%esÙe (c) DehejepesÙe (d) ogpexÙe
Gòej – (a) 41. keâewve mee lelmece Meyo veneR nw?
(a) Fvog (b) efovesMe (c) ceveespe (d) jele
JÙeeKÙee – ‘efpemes peerlee ve pee mekesâ’ kesâ efueS GheÙegkeäle Meyo nw – ‘DepesÙe’~
Gòej – (d)
peyeefkeâ ‘efpemes peevee ve pee mekesâ’ kesâ efueS Skeâ Meyo ‘De%esÙe’ nesiee~
JÙeeKÙee – efoÙes ieÙes efJekeâuheeW ceW Fvog, efovesMe Deewj ceveespe lelmece Meyo
33. `efpeme m$eer keâe heefle peerefJele nw' – Gmekesâ efueS GheÙegkeäle Meyo nw–
nQ peyeefkeâ jele leodYeJe Meyo nw~ Fmekeâe lelmece ¤he jeef$e neslee nw~
(a) heefleJeÇlee (b) meOeJee
(c) efJeJeeefnlee (d) Devegjkeälee 42. `eflekeäle' Meyo keâe leodYeJe nw –
Gòej – (b) (a) leerlee (b) leerKee (c) eflekeälee (d) efleKeve
Gòej – (a)
JÙeeKÙee – ‘efpeme m$eer keâe heefle peerefJele nw’ kesâ efueS GheÙegkeäle Meyo ‘meOeJee’ nw~
JÙeeKÙee – eflekeäle Meyo keâe leodYeJe ‘leerlee’ neslee nw, peyeefkeâ ‘leerKee’ Meyo
34. `pees #eerCe ve nes mekesâ' – Gmekesâ efueS GheÙegkeäle Meyo nw – keâe lelmece ‘leer#Ce’ neslee nw~
(a) Deefceš (b) Deheej (c) De#eÙe (d) Deveble
43. `memegj' keâe lelmece Meyo nw –
Gòej – (c)
(a) memJej (b) mJemegj (c) MJemegj (d) MJeßeg
JÙeeKÙee – ‘pees #eerCe ve nes mekesâ’ kesâ efueS GefÛele Meyo ‘De#eÙe’ nw~ Gòej – (c)
35. `yengle DeefOekeâ yeesueves Jeeuee JÙeeqkeäle' – JÙeeKÙee – memegj keâe lelmece ‘MJemegj’ neslee nw~ Mes<e efJekeâuhe Demebiele nQ~
(a) Jekeälee (b) DeefOeJekeälee (c) ØeJekeälee (d) JeeÛeeue
44. `leodYeJe' Meyo efvee|o„ keâerefpeS –
Gòej – (d)
(a) DeeOee (b) ketâhe (c) efJeÅee (d) JÙeesce
JÙeeKÙee – ‘yengle DeefOekeâ yeesueves Jeeuee JÙeeqkeäle’ `JeeÛeeue' keânueelee nw~ Gòej – (a)
36. `efpemekesâ efmej hej Ûebõ nes' – kesâ efueS GheÙegkeäle Meyo nw – JÙeeKÙee – efoÙes ieÙes efJekeâuheeW ceW `ketâhe', `efJeÅee' Deewj `JÙeesce' leerveeW efJekeâuhe
(a) ÛevõefMeKej (b) ÛevõMesKej lelmeced Meyo kesâ nQ, peyeefkeâ efJekeâuhe (a) ceW ØeÙegkeäle Meyo DeeOee leodYeJe Meyo
(c) Ûe›eâOej (d) Ûevõ«enCe nw~ Fmekeâe lelmece ¤he Deæ& (DeOe&) nesiee~
Gòej – (b) 45. `Ûetjve' keâe lelmece Meyo nw –
JÙeeKÙee – ‘efpemekesâ efmej hej Ûebõ nes’ kesâ efueS GheÙegkeäle Meyo `ÛevõMesKej' nw~ (a) Ûeewj (b) ÛetCe& (c) Ûece& (d) Ûe#eg
37. `efpemekesâ ùoÙe ceW cecelee veneR nw' – kesâ efueS GheÙegkeäle Meyo nw – Gòej – (b)
(a) cecee&nle (b) ›etâj (c) efvece&ce (d) efveo&Ùe JÙeeKÙee – Ûetjve Meyo keâe lelmece Meyo ‘ÛetCe&’ neslee nw, ve efkeâ Ûe#eg, Ûeewj
Gòej – (c) Ûece&~
JÙeeKÙee – ‘efpemekesâ ùoÙe ceW cecelee veneR nw’ kesâ efueS GheÙegkeäle Meyo `efvece&ce' 46. `{er"' Meyo keâe lelmece nw –
neslee nw, peyeefkeâ efpemekesâ ùoÙe ceW oÙee ve nes, Gmekesâ efueS ‘efveo&Ùe’ Meyo keâe (a) „ (b) heg„ (c) ÂMÙe (d) Oe=„
ØeÙeesie neslee nw~ Gòej – (d)
38. efleuekeâ ueieeves ceW efkeâme DeVe keâe ØeÙeesie GheÙegkeäle neslee nw? JÙeeKÙee – {er" Meyo keâe lelmece Meyo ‘Oe=„’ neslee nw~ Mes<e efJekeâuhe Demebiele
(a) iesntB (b) De#ele (c) peew (d) GÌ[o nQ~
Gòej – (b) 47. `lelmece' Meyo keâe ÛeÙeve keâerefpeS –
JÙeeKÙee – efleuekeâ ueieeves ceW ‘De#ele’ keâe ØeÙeesie efkeâÙee peelee nw~ De#ele kesâ (a) Mesj (b) yeyej Mesj (c) JÙeeIeÇ (d) yeeIe
efueS ÛeeJeue kesâ oeves keâe ØeÙeesie neslee nw~ Gòej – (c)
UP RO/ARO (Pre) General Hindi 2013 199 YCT
CLICK HERE FOR FREE MATERIAL

JÙeeKÙee – efoÙes ieÙes efJekeâuheeW ceW ‘JÙeeIeÇ’ Meyo lelmece nw, peyeefkeâ Mesj, yeyej JÙeeKÙee – veerueer meeÌ[er ceW ØeÙegkeäle efJeMes<eCe iegCeJeeÛekeâ efJeMes<eCe nw~
Mesj Deewj yeeIe leodYeJe Meyo nQ~ mebKÙeeJeeÛekeâ efJeMes<eCe mebKÙee Øekeâš keâjles nQ~ JeneR heefjceeCeJeeÛekeâ efJeMes<eCe
48. `ceeB' Meyo keâe lelmece nw – õJÙe (Jemleg) mes mecyeeqvOele nw~
(a) ceelee (b) ceele=keâe (c) ceele= (d) Deccee 55. `efJeMes<eCe' Meyo keâe ÛeÙeve keâerefpeÙes –
Gòej – (a) (a) mejhebÛe (b) heeBÛeJeeB (c) ØehebÛe (d) hengBÛe
Gòej – (b)
JÙeeKÙee – ceeB Meyo keâe lelmece Meyo ‘ceelee’ neslee nw~ Mes<e efJekeâuhe kesâ Meyo
JÙeeKÙee – efoÙes ieÙes efJekeâuheeW ceW heeBÛeJeeB Meyo efJeMes<eCe nw pees efkeâ efveefMÛele
Demebiele nQ~
mebKÙee (›eâcemetÛekeâ) keâer DeefYeJÙeefòeâ keâjlee nw peyeefkeâ DevÙe Meyo mejhebÛe,
49. leodYeJe Meyo nw – ØehebÛe Deewj hengBÛe ner efJeMes<eCe Meyo veneR nQ~
(a) ceeveJe (b) ceveF& (c) ceveg<Ùe (d) ceevees 56. `efJeMes<Ùe' Meyo nw –
Gòej – (b) (a) jeceueuee (b) yegefæceleer m$eer
JÙeeKÙee – efoÙes ieÙes efJekeâuheeW ceW ‘ceveF&’ Meyo leodYeJe Meyo nw~ (c) jceeheefle (d) meerlee-jece
Gòej – (d)
50. `lelmece' Meyo nw –
JÙeeKÙee – efJeMes<Ùe heo meowJe meb%ee nesles nQ~ meerlee-jece oesveeW meb%ee Meyo
(a) eqkeäue‰ (b) keâ"esj (c) keâef"ve (d) cepeyetle nQ~ ‘meerlee-jece’ efJeMes<Ùe Meyo nw~ meerlee-jece meb%ee Meyo nw, meb%ee Meyo ncesMee
Gòej – (b) & (c) efJeMes<Ùe kesâ ™he ceW ner ØeÙegòeâ nesles nQ~ ‘jeceueuee Je jceeheefle’ yeng›eerefn meceeefmekeâ
JÙeeKÙee – ueeskeâ mesJee DeeÙeesie ves Fme ØeMve keâe Gòej efJekeâuhe (a) ceevee Meyo nw pees efJeMes<eCe kesâ ™he ceW ØeÙeesie nesles nQ~ ‘yegefæceleer m$eer’ ceW efJeMes<eCe
Lee, peyeefkeâ ceeveveerÙe GÛÛe vÙeeÙeeueÙe Éeje efJekeâuhe (b) Deewj (c) keâes Fmekeâe Je efJeMes<e keâe ØeÙeesie ngDee nw~ peyeefkeâ ØeMve ceW kesâJeue efJeMes<Ùe hetÚe ieÙee nw
FmeefueS meerlee-jece Meyo efJeMes<Ùe nw~
mener Gòej ceevee ieÙee nw~ %eele nes efkeâ efkeäue‰ Meyo keâer Jele&veer DeMegæ nw,
Fmekeâe Megæ ™he ‘efkeäue°’ neslee nw~ GheÙeg&òeâ efJekeâuheeW ceW ‘keâ"esj’ Deewj ‘keâef"ve’ 57. efvecveefueefKele efJeMes<Ùe-efJeMes<eCe ÙegiceeW ceW Skeâ ieuele nw –
lelmece Meyo nw~ (a) meJe&-megueYe (b) leepeer-jesšer
(c) keâce&-efve‰ (d) YeeJe-efJendJeue
efJeMes<Ùe SJeb efJeMes<eCe Gòej – (a)
JÙeeKÙee – meJe&-megueYe oesveeW ner Meyo efJeMes<eCe heo nQ~ Dele: ØeMve keâer Âef°
51. `$e+ef<e' meb%ee Meyo mes efJeMes<eCe Meyo keäÙee yevesiee?
mes Ùen $egefšhetCe& nw~ OÙeeleJÙe nes efkeâ [e@. njosJe yeenjer Meyo keâesMe kesâ Devegmeej
(a) Dee<e& (b) $e+ef<ekeâuhe keâce&, YeeJe SJeb jesšer meb%ee nw~
(c) $e+ef<eleguÙe (d) $e+ef<eJeled
58. `heefjceeCeJeeÛekeâ ef›eâÙee-efJeMes<eCe' keâe JeekeäÙe nesiee –
Gòej – (a) (a) Jen yengle Lekeâ ieÙee nw~
JÙeeKÙee – $e+ef<e meb%ee Meyo mes efJeMes<eCe Meyo ‘Dee<e&’ yevesiee~ DevÙe efJekeâuhe (b) Jen DeYeer-DeYeer ieÙee nw~
$e+ef<ekeâuhe, $e+ef<eJele Deewj $e+ef<eleguÙe Demebiele nQ~ (c) Jen Deboj yew"e nw~
(d) Jen Deye Yeueer-YeeBefle veeÛe ueslee nw~
52. JÙeekeâjCe keâer Âef„ mes keâewve mee Meyo efJeMes<eCe veneR nw? Gòej – (a)
(a) YeÙeYeerle (b) efveYeeakeâ JÙeeKÙee – heefjceeCeJeeÛekeâ ef›eâÙee efJeMes<eCe keâe JeekeäÙe nw – ‘Jen yengle Lekeâ
(c) Yeer¤ (d) YeÙe ieÙee nw’~ ‘yengle’ Meyo heefjceeCeJeeÛekeâ ef›eâÙee efJeMes<eCe kesâ Devleie&le Deelee
Gòej – (d) nw~ DevÙe efJekeâuhe Jen DeYeer-DeYeer ieÙee nw, Jen Deboj yew"e nw, Jen Deye Yeueer-YeeBefle
JÙeeKÙee – JÙeekeâjCe keâer Âef„ mes YeÙeYeerle, efveYeeakeâ Deewj Yeer® Meyo efJeMes<eCe veeÛe ueslee nw, Gkeäle ØeMve mes he=Lekedâ DeeMeÙe jKelee nw~
nQ peyeefkeâ YeÙe Meyo efJeMes<eCe veneR Deefheleg efJeMes<Ùe nw~ Fmekeâe efJeMes<eCe YeÙeevekeâ 59. efvecveefueefKele ceW efJeMes<Ùe heo nw –
neslee nw~ (a) Devegjeieer (b) DeveeÂle
(c) Deheceeefvele (d) Deeqive
53. `Skeâ ØeefleYeemecheVe Úe$e' keâe efJeMes<eCe nw –
Gòej – (d)
(a) kegâMeue (b) Ûelegj
JÙeeKÙee – OÙeeleJÙe nes efkeâ ‘Deefive’ meb%ee nw pees efJeMes<Ùe heo nw~ Fmekeâe
(c) DeOÙeÙeveMeerue (d) cesOeeJeer
efJeMes<eCe `DeeivesÙe' nesiee~
Gòej – (d)
60. `yeÌ[e Iej' `Úesše Deeoceer' Deewj `veeruee Jem$e' ceW efJeMes<Ùe
JÙeeKÙee – Skeâ ØeefleYeemecheVe Úe$e keâe efJeMes<eCe efoÙes ieÙes efJekeâuheeW ceW cesOeeJeer
keâewve-keâewve heo nQ?
nw, ve efkeâ Ûelegj, kegâMeue Deewj DeOÙeÙeveMeerue~
(a) veeruee, Úesše (b) yeÌ[e, Úesše
54. `veerueer meeÌ[er' ceW keâewve mee efJeMes<eCe nw? (c) Iej, Deeoceer, Jem$e (d) Úesše, veeruee
(a) mebKÙeeJeeÛekeâ (b) heefjceeCeJeeÛekeâ Gòej – (c)
(c) iegCeJeeÛekeâ (d) meeJe&veeefcekeâ JÙeeKÙee – yeÌ[e Iej, Úesše Deeoceer Deewj veeruee Jem$e ceW efJeMes<Ùe heo nQ -
Gòej – (c) Iej, Deeoceer Deewj Jem$e~ peyeefkeâ yeÌ[e, Úesše leLee veeruee efJeMes<eCe nQ~
UP RO/ARO (Pre) General Hindi 2013 200 YCT
CLICK HERE FOR FREE MATERIAL

UPPSC RO-ARO (Mains) Exam-2010


GENERAL STUDIES
Solved Paper
1. Match List-I and List-II and choose the correct Ans. (b) : Mahakasyapa presided over the first Buddhist
answer using the codes given below :- Council.
List-I List-II Buddhists Year Venue Chairman Patron
(Colony of Harappan (River on which Council
Civilisation) located) 1st Buddhists 483 BC Rajgariha Mahakasapa Ajatashtru
Council
(a) Harappa 1. Bhogavo
2nd Buddhist 383 BC Vaishali Sabbakami Kalashoka
(b) Kalibangan 2. Ghaggar Council
(c) Lothal 3. Ravi 3rd Buddhist 250 BC Pataliputra Magliputta Ashoka
(d) Ropar 4. Sutlej Council Tissa
Code : 4th Buddhist 72 AD Kundalvan Vasumitra/ Kanishka
Council Ashavaghosh
A B C D
5. Gujarat was included in the Chandragupta
(a) 3 2 1 4
Maurya Kingdom. It is certified from ?
(b) 3 4 1 2 (a) Greek descriptions
(c) 4 2 3 1 (b) From the Rock Edicts of Junagarh of
(d) 1 3 2 4 Rudradaman
Ans. (a) : The correct match as follow are- (c) Jain Traditions
List-I List-II (d) From the Pillar Edict-II of Ashoka
(Colony of Harappan (River on Which Ans. (b) : It is proved from the Junagadh inscription of
Civilisation) Located) Rudradaman that Gujarat was included in the empire of
A. Harappa - Ravi Chandragupta Maurya and he had built Sudarshan Lake there.
B. Kalibangan - Ghaggar 6. Which of the following Ashoka Inscriptions
confirms the tradition that Gautam Budha was
C. Lothal - Bhogavo born at Lumbini ?
D. Ropar - Sutlej (a) Basarh Pillar Edict
2. Which among the following was not a dynasties (b) Nigali Sagar Pillar Edict
of Sangam Age of a Tamil State ? (c) Rampurva Pillar Edict
(a) Chera (b) Chola (d) Rummindei Pillar Edict
(c) Pallava (d) Pandya Ans. (d) : The birth of Lord Buddha in Lumbini was
found from Rummindei pillar inscription. It is smallest
Ans. (c) : The Cholas, Cheras and Pandyas are the pillar inscription of Ashoka. Rummindei village is
dynasties of the Sangam age, while Pallava was not one Lumbani which is situated in Terai of Nepal. This
of the rulling dynasties of South Indian in the Sangam inscription was discovered by Fuihrer.
Age of a Tamil State. The Pallava dynasty was an
7. Agra City was established by
Indian dynasty that existed from 275 CE to 897 CE,
(a) Bahlol Lodi (b) Firoz Tughlaq
ruling a significant portion of the Deccan.
(c) Khizr Khan (d) Sikandar Lodi
3. Who among the following was the author of
Ans. (d) : Modern Agra was established by Sikandar
Tamil Ramayanam or Ramavataram ? Lodhi in the 16th century and shifted the capital from
(a) Kambar (b) Kuttan Delhi to Agra in 1506. He introduced the authentic scale
(c) Nannay (d) Tikkan Gaj-i- Sikandari for the measurement of the land and
Ans. (a) : Ramavataram, popularly referred to as Kama also wrote Persian poems under the title 'Gulrukhi'.
Ramayanam, is a Tamil epic that was written by the 8. Abdul Razzak came to Vijaynagar During -
Tamil poet Kambar during the 12th century. Based on (a) Ruling period of Dev Raya-I
Valmiki's Ramayana (which is in Sanskrit), the story (b) Ruling period of Dev Raya-II
describes the life of King Rama of Ayodhya. (c) Ruling period of Krishna Deva Raya
4. Who among the following presided over the (d) Ruling period of Veer Vijaya.
first Budhist Council ? Ans. (b) : Abdul Razzak was an Islamic scholar and a
(a) Anand (b) Mahakasyapa historian who visited the Vijayangar Kindom in India at
(c) Moggaliputta Tissa (d) Upali the time of Dev Raya-II, the greatest ruler of the
UP RO/ARO (Mains) Exam 2010 201 YCT
CLICK HERE FOR FREE MATERIAL

Sangama dynasty. He says of the city. "The city of Ans. (a) : Prarthana Samaj was founded by Atmaram
Vijayanagar is such that the pupil of the eye has never Pandurang in 31st March 1867 when Keshub Chandra
seen a place like it and the ear of intelligence has never Sen visited Maharashtra, with aim to make people
been informed that there existed anything to equal it in believe in one God and worship only one God. It
the world. become popular after Mahadev Govind Ranade joined.
9. A Dutch tourist, who gave a vivid description 15. Which of the following Governor - General
of Jahangir's ruling period was :- abolished the Slavery Custom in India ?
(a) Francisco Pelsaert (b) Hawkins (a) Lord Cornawallis
(c) Niccolao Manucci (d) Peter Mundy (b) Lord Ellenborough
Ans. (a) : Francisco Pelsaert was a Dutch traveler gave (c) Lord William Bentinck
a vivid description of Jahangir ruling period from 1620- (d) Sir John Shor
1627. Ans. (b) : It was Lord Ellenborough, who abolished
10. Who laid the foundation of Mughal Painting ? slavey custom in India. The Indian Slavery Act, 1843,
(a) Babar (b) Humayun also Act V of 1843, was an act passed in British India
under East India Company rule, which outlawed many
(c) Akabar (d) Jahangir
economic transactions associated with slavery. Lord
Ans. (b) : The foundation of Mughal pointing was laid Ellenborough was the governor of India from 1842-44 AD
by Humayun during his stay in Persia and Afghanistan. and the Anglo-Afghan was also ended during his reign.
In which he received the services of two Persian 16. In India the success of Theosophical Society
painters Mir Sayyid Ali and Abdus Samad. Humayun was mainly due to-
started Mughal painting with the help of two famous (a) Annie Beasants's efforts
Persian painters, Mir Sayyid Ali and Abdus Samad. (b) Efforts of Colonel H.S. Alcott
11. Which of the following pair is not correctly (c) Efforts of Sir William Crook
matched ? (d) Efforts H.S of M.M Malviya
(a) Hector Munro Battle of Buxar Ans. (a) : The credit for the success of Theosophical
(b) Lord Hastings Anglo - Nepal War Society in India goes to Annie Besant. He had played an
(c) Lord Wellesley Fourth Anglo- important role in making effective and connecting
Mysore War people by joining this organization in India. Adyar
(d) Lord Conrawallis Third Maratha War (Tamil Nadu), was the international office of the
Ans. (d) : Lord Hastings served as the Governor- Theosophical Society.
General of India from 1813 to 1823. It was during this 17. Who established the 'Independent League' ?
time the third Anglo-Maratha War was fought between (a) Lala Lajpat Rai (b) Mahatma Gandhi
the East India Company and the Maratha. His tenure is (c) Motilal Nehru (d) Ras Bihari Bose
known for the policy of intervention and war. Rest of all Ans. (d) : The 'Independent League' was founded in the
the given option is correctly match. year 1942 by Rash Bihari Bose in Tokyo, Japan. It aim
12. Baba Ramchandra had organised peasants of ? was to end the British rule in India.
(a) Awadh region (b) Bihar province 18. Arrange the following events related to Indian
(c) Andhra region (d) Bengal province National Movement in a chronological order
Ans. (a) : Baba Ram Chandra was on Indian trade and choose the correct answer from the codes
unionist who organised the farmers of Awadh, India given below :
into farming a united front to fight against the abuses of 1) Rowlatt Act, 1919
landlords in the 1920s and 1930s. 2) Rowlatt Satyagarh, 1919
3) Amritsar Session of Indian National
13. The first session of All India Kisan Sabha was Congress, 1919
held in -
4) Jallianwala Bagh Massacre, 1919
(a) Allahabad (b) Calcutta
Code :
(c) Lucknow (d) Patna A B C D
Ans. (c) : After the end of the Civil Disobedience (a) 1 4 3 2
Movement, the All India Kisan Sabha was established (b) 1 2 4 3
in Lucknow in 1936. The first All India Kisan Sabha (c) 4 2 1 3
whose conference was held in Lucknow on April 11th, (d) 3 1 4 2
1936, was presided over by Swami Sahajanand
Saraswati, the president of Bihar Congress. Andhra Ans. (b) : The correct chronological order of Indian
Pradesh farmer's movement N.G. Ranga was appointed National Movement are-
as the general secretary of the Kisan Sabha. 1. Rowlatt Act - 18 March, 1919
14. Who among the following laid the foundation 2. Rowlatt Satyagraha - 6 April, 1919
of 'Prarthana Samaj' in Maharashtra ? 3. Jallianwala Bagh Massacre - 13 April, 1919
(a) Atmaram Pandurang (b) Jyotibha Phule 4. Amritsar Session of the - 26-30 December, 1919
(c) M.G Chandravarkar (d) M.G. Ranade Indian National Congress
UP RO/ARO (Mains) Exam 2010 202 YCT
CLICK HERE FOR FREE MATERIAL

19. Arrange the following events related to Indian 23. Which one of the following place is closest to
National Movement in a chronological order the Indian Standard Time Meridian ?
and choose the correct answer from the codes (a) Bilaspur (b) Faizabad
given below :- (c) Koraput (d) Mirzapur
1. Dandi March of Mahatma Gandhi. Ans. (d) : Mirzapur, near Allahabad in the state of Uttar
2. Poona Agreement. Pradesh is closest to the Indian Standard Time at 82.50
3. Communal Award. ºE longitute. The standard meridian of India passes
4. Gandhi-Irwin Pact through five states of Indian, namely, Uttar Pradesh,
(a) 1, 4, 3, 2 (b) 1, 3, 4, 2 Madhya Pradesh, Chhattisgarh, Orissa and Andhra
(c) 3, 4, 2, 1 (d) 4, 2, 3, 1 Pradesh.
Ans. (a) : The correct chronological order of Indian 24. India is largest producer of -
National Movement are- (a) Coffee (b) Sugarcane
On 12th March, 1930, Mahatma Gandhi started the (c) Oil Seeds (d) Tobacco
Dandi March. The Gandhi-Irwin Pact was signed an 5th Ans. (*) : In 2020, the major countries in sugarcane
March, 1931. The communal award was announced on production are Brazil, India and China respectively.
16th August 1932 and the Poona Pact was signed on 24th India ranks first in production of pulses, jute, spices and
September 1932. milk, while the country is the second largest producer of
20. Who among the following had called Rabindra wheat, rice, groundnut, sugarcane, potato.
Nath Tagore - "The Great Sentinel" ? 25. Consider the following statements and choose
(a) Maulana Abul Kalam Azad the correct answer from the codes given
(b) Mahatma Gandhi below:-
(a) In India, Karnataka is the largest producer of
(c) Dr. Rajendra Prasad
Raw Silk.
(d) Subhash Chandra Bose
(b) In India, Andhra Pradesh is the largest
Ans. (b) : Mahatma Gandhi called Rabindranath Tagore producer of mulberry silk.
as 'The Great Sentinel'. Rabindranath Tagore was a poet, (c) In India, Jharkhand is the largest producer of
playwright, essayist and painter. He was awarded the Tussor silk
Nobel Prize for literature in 1913 for his work Gitanjali. (d) In India, Meghalaya is the largest producer of
21. Hindustan Republican Association was Eri-Silk.
established in ? Ans. (*) : The South Indian state of Karnataka was the
(a) Allahabad (b) Kanpur leading raw silk producer, with over 8,483 metric tons
(c) Lucknow (d) New Delhi of raw silk produces in fiscal year 2021. In India,
Ans. (b) : In 1924, A revolutionary organisation named Karnataka is the largest producer of mulberry silk
'Hindustan Republican Association, was founded in followed by Andhra Pradesh, West Bengal. Jharkhand
Kanpur, Uttar Pradesh by Ram Prasad Bismil, Jogesh is largest producer of Tasar Silk followed by
Chandra Chatterjee and Sachin Sanyal. Later in the year Chhattisgarh and Orissa. In India, Assam is the largest
1928, after the joining of other revolutionaries at the producer of Eri Silk. Hence, option (1) and (3) are
Firoz Shah Kotla Maidan in Delhi, the name of this correct answer.
organisation was changed to 'Hindustan Socialist 26. In India factory made clothes largest
Republican Association.' production is obtained from -
22. Consider the following statements :- (a) Dadra and Nagar Haveli (b) Gujarat
Assertion (A) : Mahatma Gandhi had (c) Maharashtra (d) Punjab
adjourned the Non-Cooperation Movement in Ans. (*) : Tamil Nadu has the largest production of
1922. Mill-Made clothes in India. According to the Ministry
Reason (R) : The adjourned of the movement of Textiles Report (2017), Tamil Nadu has the largest
was opposed by C.R. Das and Motilal Nehru. number of textile mills with 752 textile mills, followed
Codes : by Maharashtra (135) and Andhra Pradesh (112).
(a) Both (A) and (R) are true and (R) is the true Andhra Pradesh, Telangana, Haryana, Jharkhand and
explanation (A). Gujarat are the top textile and clothing manufacturing
(b) Both (A) and (R) are true but (R) is not the states in India.
correct explanation of (A). 27. Match List-I & List-II and choose the correct
(c) (A) is true but (R) is false. answer from the codes given below :-
(d) (A) is false but (R) is true. List-I List-II
Ans. (b) : In the Bardoli Conference of Congress in (Iron-Ore Region) (State)
1922, the Non-Cooperation Movement was adjourned (a) Badam Pahar 1. Jharkhand
by Mahatma Gandhi, whose C.R. Das and Moti Lal
(b) Dalli Rajahra 2. Odisha
Nehru opposed. But even through both the above
mentioned events are true, reason (R) is not a proper (c) Kudremukh 3. Chhattisgarh
explanation of assertion (A). (d) Noamundi 4. Karnataka
UP RO/ARO (Mains) Exam 2010 203 YCT
CLICK HERE FOR FREE MATERIAL

Code : Ans. (d) : The ocean surrounded by Italy, Sicily,


A B C D Sardinia and Corsica is the Tyrrhenian Sea. The
(a) 4 2 3 1 Tyrrhenian Sea is located along the western shorline of
(b) 2 3 4 1 Italy and is considered part of the larger Mediterranean
(c) 1 3 2 4 Sea, which is situated over the boundary between the
(d) 2 1 4 3 Eurasian and Africa tectonic plates.
Ans. (b) : The correct match as follows are- 34. The largest natural reserves of crude oil
List-I List-II (mineral oil) are located in ?
(Iran-Ore Region) (State) (a) Canada (b) Iran
Badam Pahar - Odisha (c) Saudi Arabia (d) Venezuela
Dalli Rajahra - Chhattisgarh Ans. (d) : Venezuela is the leading country in terms of
Kudremukh - Karnataka oil reserves with over 304 billion barrels of oil beneath
Noamundi - Jharkhand its surface. Saudi Arabia is a close second with 298
28. Which one among the following is a port city of billion and Canada is third with 170 billion barrels of oil
Andhra Pradesh ? reserves. Top 5 countries with the largest oil reserves
(a) Kakinada (b) Nellore (in thousand million barrels):
(c) Ongole (d) Vijaynagram Rank Country Reserves % of World
Ans. (a) : Kakinada is a port city and a municipal 1. Venezuela 303.8 17.5%
corporation in the Indian state of Andhra Pradesh. It is 2. Saudi Arabia 297.5 17.2%
the headquarters of the East Godavari district. Kakinada
3. Canada 168.1 9.7%
is port of the mighty River Godawari Delta Region.
4. Iran 157.8 9.1%
29. National Highway No. 4 does not pass through
5. Iraq 145.0 8.4%
(a) Andhra Pradesh (b) Karnataka
(c) Maharashtra (d) Tamilnadu 35. In terms of Ship load tonnage capacity which is
Ans. (*) : National Highways Number 4 connects from the busiest port in the world ?
Thane in Maharashtra to Chennai in Tamil Nadu. Its (a) Busan (b) Rotterdam
national highway passes through the four states of (c) Singapore (d) Shanghai
Maharashtra, Karnataka, Andhra Pradesh and Tamil Ans. (d) : The World's top four ports in terms of Cargo
Nadu. The total length is 1235 km. tonnage are as follows-
30. Prime Meridan does not pass through - 1. Shanghai (47 TEUs)
(a) Algeria (b) France 2. Singapore (37.47 TEUs)
(c) Niger (d) Spain 3. Ningbo - Zhoushan (31.08 TEUs)
Ans. (c) : The prime meridian does not pass through 4. Shenzhen (28.76 TEUs)
Niger. While it passes through London, France, Spain, Where, TEUs are Throughput in million.
in Europe; Algeria, Mali, Durkina Faso, Togo and
Ghana in Africa and Antarctica at the South Pole. It 36. In terms of Monetary Value, which is the
intersects the Equator (0' latitude) in the Atlantic Ocean. world's largest ship manufacturing country ?
31. Which among the following is a Land locked (a) China
nation of Africa ? (b) Japan
(a) Angola (b) Chad (c) South Korea
(c) Kenya (d) Senegal (d) United States of America
Ans. (b) : A land locked country is a country that does Ans. (c) : South Korea is the world's largest ship
not have territory connected to an ocean or whose building country in terms of monetary value. China,
coastline lie on endorheic basins. Out of Africa's 55 South Korea and Japan are the world's largest
countries, 16 of them are land locked: Bostswana, Burundi, shipbuilders, with South Korea and China accounting
Central African Republic, Chad, Lesotho, Malawi, Mali, for the lion's share of the market.
Niger, Zimbabwe, Rwanda, Swaziland, Uganda, Burkina
Faso, Zambia, Ethiopia and South Sudan. 37. Which is the correct descending order of
32. Which is the most densely propulated nation of States/Union Territories (as per the last census
South America ? data of 2011) which have registered highest sex
(a) Bolivia (b) Colombia (Gender) Ratio ?
(c) Ecuador (d) Venezuela (a) Kerala, Puducherry, Tamil Nadu, Andhra Pradesh
Ans. (c) : According to 2022 data, the population (b) Andhra Pradesh, Kerala, Tamil Nadu,
density of the above states is Ecuador (71 per km2), Puducherry
Colombia (46 per km2), Venezuela (32 per Km2), (c) Tamil Nadu, Andhra Pradesh, Puducherry, Kerala
Bolivia (11 per Km2). (d) Puducherry, Kerala, Andhra Pradesh, Tamil Nadu
33. The name of the Sea enclosed by Ans. (a) : The descending order of the states/UTs as per
Italy, Sicily, Sardinia and Corsica is - the final census data of 2011, the sex ratio are Kerala
(a) Adriatic Sea (b) Aegean Sea (1084), Puducherry (1037), Tamil Nadu (996), Andhra
(c) Ligurain Sea (d) Tyrrhenian Sea Pradesh (993).
UP RO/ARO (Mains) Exam 2010 204 YCT
CLICK HERE FOR FREE MATERIAL

38. As per last census figures of 2011, which of the 41. As per the last Census figures of 2011, which of
following states, have registered higher number the given statements is/are true? Choose the
of males than females of 7 years age or more correct answers from the codes given below :-
(above) ? (1) Among the Union-Territories, the highest
(1) Bihar (2) Maharashtra made literacy rate is in Lakshwadeep.
(3) Uttar Pradesh (4) West Bengal (2) Among the states highest male literacy rate
Choose the correct answer from the codes below: is in Mizoram.
Codes: (3) Among the Union Territories the highest
(a) Only 1 & 2 (b) Only 1, 2 & 3 literacy rate of women has been recorded
(c) Only 2, 3 & 4 (d) All the four in Lakshwadeep.
Ans. (d) : Bihar, Maharashtra, Uttar Pradesh and West (4) Kerala is the state where the women
Bengal have more males than females in all the four states. literacy rate is highest.
Bihar - Male (54278157), Female (49821295) Codes :-
Maharashtra - Male (58243056), Female (54131277) (a) only 1 & 2 (b) only 1, 2 & 3
Uttar Pradesh - Male (87565369), Female (78632552) (c) only 2, 3 & 4 (d) All the four
West Bengal - Male (46809027), Female (44467088) Ans. (*) : Among the Union Territories, Lakshadweep
39. Consider the following statements :- has the highest literacy rate is 91.85%. Hence, first
Assertion (A) :- In Uttar Pradesh literacy rate statement correct. The highest male literacy rate among
has been steadily increasing. in states in Kerala (96.11%). Hence second statement is
Reason (R) : The growth of literacy rate among incorrect. Among the Union Territories the highest
women has not been consistent with the growth literacy rate of women has been recorded in
rate among men. Lakshadweep. Hence, third statement is correct. Kerala
is the highest female literacy rate (92.07%) among the
Choose the correct answer from the codes
states. Hence fourth statement correct.
given below :-
Codes :- 42. Arrange the following states as per the
(a) Both (A) & (R) are true and (R) is the correct descending order of their urban population
explanation of (A). (2011). Choose the correct answer using the
(b) Both (A) & (R) are true but (R) is not the codes given below :-
correct explanation of (A). 1. Maharashtra 2. Tamilnadu
(c) (A) is true but (R) is false. 3. Uttar Pradesh 4. West Bengal
(d) (A) is false but (R) is true. Code :
Ans. (b) : The growth of literacy rate among women A B C D
has not been consistent with that of men and the literacy (a) 1 3 2 4
in Uttar Pradesh has been increasing at a steadily rate. (b) 1 2 3 4
Both Assertion (A) and Reason (B) are true but Reason (c) 4 3 2 1
(R) is not the correct explanation of Assertion (A). Uttar (d) 2 1 4 3
Pradesh literacy rate is 67.68%. In which male literacy Ans. (a) :
rate is 77.3% and female literacy rate is 57.2%. State Rural Urban % of Rural % of
40. Consider the following statements :- Population Population Population Urban
Assertion (A) : Uttar Pradesh remains the Population
highest populated state of country [as per the Mahara- 6,15,56,074 5,08,18,259 54.78% 45.22%
provisional data of census 2011] shtra
Reason (R) : Bihar, West-Bengal and Tamil 3,72,29,590 3,49,17,440 51.6% 48.4%
Maharashtra are consecutively placed in the Nadu
descending order after it. Uttar 15,53,17,278 4,44,95,063 77.73% 22.27%
(a) Both (A) and (R) are true and (R) is the Pradesh
correct explanation of (A). West 6,21,83,113 2,90,93,002 68.13% 31.87%
(b) Both (A) and (R) are true but (R) is not the Bengal
correct explanation of (A). So the descending order of urban population is as follows-
(c) (A) is true but (R) is false. Maharashtra > Uttar Pradesh > Tamil Nadu > West Bengal.
(d) (A) is false but (R) is true. 43. Which of the following statements are true?
Ans. (c) : According to the provisional data of census of Choose the correct answer using the codes
2011, Uttar Pradesh has the highest population of given below:-
19,9812, 341. Uttar Pradesh is followed by the 1) For the first time after Independence the
population of Maharashtra, Bihar and West Bengal in total (net) increase in population has been
descending, order respectively. Hence Assertion (A) is more in urban areas in contrast to rural
true and Reason (R) is false. Top states with highest areas.
population are Uttar Pradesh (19.981 crore), 2) In India (as per 2011 Census) the rural
Maharashtra (11.237 crore), Bihar (10.409 crore), West population was 67.84% and urban
Bengal (9.127 crore). population was 32.16%
UP RO/ARO (Mains) Exam 2010 205 YCT
CLICK HERE FOR FREE MATERIAL

3) In 2001, the level of urbanization is (c) A Fundamental Duty


27.81%. (d) An Organ of Directive Principles of State
4) In 2001, the percentage of rural population Policy
was 71.19%. Ans. (d) : 'Equal pay for equal work' is included in the
(a) 1 & 2 only (b) 2 & 3 only Directive Principles of State Policy in the constitution
(c) 3 & 4 only (d) 1 & 3 only of India. Article 39(d) provides that both men and
Ans. (d) : After independence, between 2001 and 2011, women should have equal pay for equal work. The
the growth of population of urban areas was higher than parliament has passed the Equal Remuneration Act,
that of rural areas. Rural population was 68.84% and 1976 in pursuance of Article 39(d).
urban population was 31.15% in the year 2011. In the 48. The speaker of which of the following House is
year 2001, the level of urbanization in India was 27.81% a non-member ?
while the percentage of rural population in that year was (a) Lok Sabha
72.19%. Hence statements first and third are true. (b) Sate Legislative Assembly
44. Arrange the following Union Territories of (c) Rajya Sabha
India in the descending order of their (d) None of these
urbanisation level and choose the correct Ans. (c) : According to the article 63 of the constitution,
answer using the codes given below :- there will be a Vice President of India. The Vice-
(1) Chandigarh (2) Daman & Diu (3) Delhi President of India is the ex-officio chairman of Rajya
(4) Lakshadweep Sabha. It is not a member of Rajya Sabha. Therefore, it
Code :- does not have the right to vote, but as the chairman, he
(a) 2, 1, 4, 3 (b) 3, 1, 4, 2 has the right to cast a casting vote.
(c) 4, 1, 2, 3 (d) 4, 3, 1, 2 49. Among the following politicians, who adorned
Ans. (b) : The descending order of their urbanization of the post of Prime Ministership more than once
Union Territories as per Census 2011 are as follows- in their political career ?
Delhi (97.50%)→Chandigarh (97.25%)→Lakshadweep 1) Jawahar Lal Nehru
(78.07%) → Daman & Diu (75.17%). 2) Lal Bahadur Shastri
45. Arrange the following states in their increasing 3) Gulzarilal Nanda
order of urban population and choose the 4) Indira Gandhi
correct answer using the codes given below :- 5) Atal Behari Vajpayee
1) Arunachal Pradesh 2) Mizoram Choose the correct answer using the codes
3) Nagaland 4) Sikkim given below :-
Code : (a) 1, 2 & 3 only (b) 1, 3 & 4 only
A B C D (c) 3, 4 & 5 only (d) 1, 2, 4 & 5
(a) 1 4 2 3 Ans. (c) : Pandit Jawahar Lal Nehru was the Prime
(b) 3 1 4 2 Minister from 1947 to 1964 without any break and Lal
(c) 4 1 2 3 Bahadur Shashtri from 1964 to 1966, India Gandhi from
(d) 4 3 1 2 1966-77 and 1980-84. Gulzari Lal Nanda was the
Ans. (*) : As per the census of 2011 data- caretaker in 1964 and 1966 and Atal Bihari Vajpayee
was the Prime Minister for 13 days in 1996 and second
State Urban Population
terms from 1998 to 2004.
Arunachal Pradesh 3,17,369
Mizoram 5,71,771 50. Which of the following statements related to
Chief Minister is not correct ?
Nagaland 5,70,966
(a) He is normally selected by the Governor
Sikkim 1,53,578
(b) He is formally appointed by the Governor
Ascending order - Sikkim < Arunachal Pradesh <
(c) He is elected by the members of the majority
Nagaland < Mizoram.
party
46. According to the Census 2011, the total number (d) His stay on his post depends on many factors
of Census towns in India is -
Ans. (a) : According to article 164(1) of the Indian
(a) 3894 (b) 4041 Constitution states that the Chief Minister is appointed by
(c) 5161 (d) 7935 the Governor but he appoints the leader of the majority party
Ans. (a) : According to the census 2011, the total number in the Legislative Assembly. Therefore, his appointment is
of towns in India is 7935 out of which the number of only a formality. Along with this, the Chief Ministers
census towns is 389 and the number of statutory towns is continuance in the post also depends on others factors.
4,041. The number of census towns in the year 2001 was 51. The source of the "theory of basic structure of
1362 while the total number of towns was 5161. constitution" in India is -
47. "Equal Pay for Equal Work" which has been (a) Constitution
instituted/vested in Indian Constitution is a - (b) Judicial Interpretation
(a) A Fundamental Right (c) Opinion of Jurists
(b) An Economic Right (d) Parliamentary laws
UP RO/ARO (Mains) Exam 2010 206 YCT
CLICK HERE FOR FREE MATERIAL

Ans. (b) : Judicial Interpretation is the source of the 56. The Weather Based Crop Insurance Scheme
'theory of the basic structure of the constitution' in was first implemented in -
India. The principle of basic structure has been explained (a) Haryana (b) Karnataka
by the Supreme Court in Kesavananda Bharti Vs. Union of (c) Punjab (d) Tamilnadu
India (1973) and in several subsequent cases. Ans. (b) : In the year 2007, Weather Based Crop
52. The 73rd & 74th constitutional amendments are Insurance Scheme (WBCIS) was first implemented on
responsible for the formation of which of the pilot basis in the state of Karnataka by the central
following ? government. Pradhan Mantri Fasal Bima Yojana was
(1) State Election Commission launched in the year 2016 for Kharif Crop. The
premium rates in this scheme are 1.5% for rabi, 2% for
(2) District Planning Committee
kharif, 5% for cash and horticulture crops.
(3) State Finance Commission
57. Which of the following Agricultural produce
(4) State Wealth Tax cost fixing
fetches maximum foreign exchange for India
Choose the correct answer from the codes through its product export ?
given below :- (a) Coffee (b) Basmati Rice
(a) 1 & 2 only (b) 1, 2 & 3 only (c) Raw Cotton (d) Tea
(c) 2, 3 & 4 only (d) All the four Ans. (b) : Most of the foreign exchange for the country
Ans. (b) : Constitutional recognition was given to the comes from the export of basmati rice. Basmati rise
State Election Commission by the 73rd and 74th ranks highest in the export of agricultural products with
Constitutional Amendment Act 1992 of the USD 4712.44 million for the year 2018-19.
constitutional. The provision of financial for panchayats 58. Which of the following is used as an antifreeze
and the provision of District Planning Committee has in automatic engines ?
also been made in this amendment. The chairman of the (a) Ethanol (b) Ethylene Glycol
State Finance Commission is appointed by the (c) Methanol (d) Propyl Alcohol
Governor.
Ans. (b) : Use of ethylene glycol for automatic engines.
53. Which of the following statements relating to It is used as an-anti-fueze. It is used in radiation. It
"Globalisation" is not correct ? lowers the freezing point in cold countries.
(a) It has motivated the Indian Industrialists to 59. Which of the following is known as "Laughing
harness/mobilise foreign investments. Gas"
(b) It has motivated/inspired the large business (a) Methane (b) Nitric Oxide
houses to acquire large foreign companies. (c) Nitrons oxide (N2O) (d) Ozone
(c) It has enticted foreign companies to
Ans. (c) : Nitrous oxide, commonly known as laughing
purchase/acquire Indian companies. gas or happy gas, is a colorless, non-flammable gas.
(d) It has instilled a sense of optimism in small This gas is used in medical and dental procedures as a
entrepreneurs. sedative. It helps to relieve anxiety before the procedure
Ans. (d) : Globalization has threatened the existence of and allow the patient to relax.
small entrepreneurs in India rather than a sense of 60. To construct bullet proof material, which of the
optimism because the concentration of capital makes it following polymer is used ?
difficult to complete with the big companies. (a) Polymide (b) Polycarbonate
54. Which of the following programmes was meant (c) Polyethylene (d) Polyvinyl chloride
for eradicating poverty in Rural India ? Ans. (b) : Polycarbonates are used to make bullet proof
(a) I.R.D.P. (b) TRYSEM materials. Polyvinyl chloride makes films, thin sheets,
(c) N.R.E.P (d) All of them raincoats and thick causes, while polyethylene makes
Ans. (d) : IRDP, TRYSEM and NREP are to eradicate wire covers, polyamide is used for making silk, wool
poverty in rural India. and nylon etc.
Integrated Rural Development Programme (IRDP) was 61. Stem is not an organ of storage of which of the
launched in 1978. On 1st April, 199 this scheme was following plant ?
merged with Swarna Jayanti Gram Swarozgar Yojana. (a) Sugarcane (b) Ginger
Training Rural Youth for Self Employment Programme (c) Potato (d) Sweet Potato
(TYSEM) was started on 15th August, 1979 as an auxiliary Ans. (d) : Sugarcane, potato and ginger are the stem
component programme of IRDP. The National Rural while sweet potato in the root which is eaten.
Employment Program (NREP) was launched in 1980.
62. Match the List-I and List-II and choose the
55. The first Industrial Policy of Independent India
correct answer using the codes given below :-
was announced in ?
(a) 1947 (b) 1948 List-I List-II
(c) 1950 (d) 1956 (a) Encephalitis 1. Blood cells
Ans. (b) : The first Industrial Policy in India was (b) Filaria 2. Brain
announced post-Independence in 1948. It was presented (c) Leukemia 3. Lymph nodes
by Dr. Shyama Prasad Mukherjee (d) Malaria 4. Bone-Marrow
UP RO/ARO (Mains) Exam 2010 207 YCT
CLICK HERE FOR FREE MATERIAL

Code : Ans. (d) : In June 2009, research from the University of


A B C D Pittsburgh and the Buffalo Museum of Science made it
(a) 4 2 3 1 clear that the orangutan primate is the closest to modern
(b) 2 1 4 3 humans. The orangutan is mostly found; Malaysia and
(c) 1 3 2 4 Indonesia.
(d) 2 3 4 1 68. Rh-factor name is related to a type of ?
Ans. (d) : The correct match is- (a) Monkey (b) Human
(c) Ape (d) Rat
List-I List-II
Ans. (c) : In 1940, Landsteiner and Weiner detected
A. Encephalitis - Brain another type of antigen is blood. He detected his
B. Filaria - Lymph nodes element in rhesus monkey. That's why it is called the Rh
C. Leukemia - Bone-Marrow factor people who have this element in their blood.
Their blood is called Rh positive and these in which it is
D. Malaria - Blood cells not found, their blood is called Rh negative.
63. Which of the following pairs is correctly 69. The substance which affects the qualities of
matched ? chapati making is ?
(a) Vitamin B1 - Retinol (a) Gluten (b) Globulin
(b) Vitamin B2 - Riboflarin (c) Glycine (d) Lysine
(c) Vitamin D - Niacin Ans. (a) : Gluten is a substance that affects the
(d) Vitamin C - Calciferol properties of the wheat flour.
Ans. (b) : The chemical name of Vitamin B1 is 70. A wax solution is used in fruits and vegetables
thiamine. Its deficiency causes a disease called beri- to -
beri. Vitamin B2 is called riboflavin, Vitamin B6 is (a) To produce shine on fruits & vegetables
called Pyridoxine and Vitamin C is called ascorbic acid. (b) To enhance their shelflife (storage life)
64. Which of the following is not a insect ? (c) to increase the speed of their ripening
(a) Bedbug (b) House fly (d) None of these
(c) Mosquito (d) Spider Ans. (b) : Wax solution is used in fruits and vegetables
Ans. (d) : Bedbug, mosquitoes and house fly are insects to increase their storage life. Paraffin and carnauba wax
while spiders are not counted among insects. Insects are are used for this artificial solution. The storage time
the only invertebrates (animal without-backbones) with increases because it is anti-bacterial.
wings, but spiders do not have wings. Hence, the spider 71. Pegging is a beneficial technique/process used
not an insects. in?
65. Which of the following pair is not correctly (a) Sugarcane (b) Sweet potato
matched ? (c) Ground nut (d) Tapioca
(a) Niacin - Pellagra Ans. (c) : The budding ovary in peanut is called peg.
(b) Thiamine - Beri-Beri The peg enlarges and grow down and away forms the
(c) Vitamin- D - Rickets plant forming a small stem which that extends to the
soil. The peanut embryo is in the tip of the peg, which
(d) Vitamin - K - Sterility penetrates (pegging) the soil. When pegging stops
Ans. (d) : Vitamin K deficiency does not cause blood to embryo beings to mature taking the form of a pod.
Clot. Beri-Beri disease due to thiamine deficiency of 72. The length of a word in computers is measured
Vitamin B1, Vitamin D causes rickets and deficiency of in?
niacin causes pellagra disease. Infertility in related to (a) Bits (b) Bytes
vitamin E.
(c) Millimeters (d) Metre
66. The centre of satiety and thirst in human brain Ans. (a) : The Hindi name of computer is Sanganak.
is located in which part ? Charles Babbage is called the father of the computer.
(a) Frontal brain (b) Hypothalamus Modern computer was first discovered in 1946 AD. A
(c) Medulla (d) Optic lobe computer's memory is usually expressed in terms of kilo
Ans. (b) : The centers of 'satiety and 'thirst' are located bytes or megabytes. A byte is made up of eight binary digits.
in the hypothalamus part of the human brain. The The length of words in a computers is measured in bits.
hypothalamus controls the harmones recreated from the 73. Which is an latest substance/material which are
endocrine glands. The forebrain is divided into being used in the production of Chips ?
cerebrum and diencephalon. The cerebrum is the centre (a) Carbon (b) Gallium Arsenide
of consciousness, intelligence and memory. (c) Gallium Sclicate (d) Silicon
67. Which of the following primate is the closest Ans. (d) : Silicon is the most abundant element on earth
relative of Modern Man ? except for oxygen and is used in chip manufacturing
(a) Gibbon (b) Gorilla because it is a natural semiconductor. Silicon
(c) Langoor (d) Orangutan compounds have a wide variety of useful properties,
UP RO/ARO (Mains) Exam 2010 208 YCT
CLICK HERE FOR FREE MATERIAL

mostly because they can bind other atoms very tightly 79. India's largest Tiger Reserve is in -
and in complex arrangements. Silicon is also the major (a) Andhra Pradesh (b) Karnataka
structural component of the synthetic material silicone. (c) Madhya Pradesh (d) Tamilnadu
74. The body of mouse used in operating Ans. (a) : Nagarjunasagar Srisailam Tiger Reserve
computers was densed/constructed about 40 (NSTR) is the largest tiger reserve in India. It is
years ago. At that time it was made of ? spreaded over five districts in Andhra Pradesh and
(a) Aluminuim (b) Plastic Telangana. It was notified in the year of 1978 and came
(c) Steel (d) Wood under the protection of Project Ttiger in 1983. In 1992 it
Ans. (d) : The mouse was invented by Douglas was retitled as Rajiv Gandhi Wildlife Sanctuary.
Engelbart in 1964 and consisted of wooden shell, circuit 80. In India the total number of bio-geographic
board and two metal wheels that came into contact with zone -
the surface it was being used OA. (a) 7 (b) 8
75. The Indian Institute Pulses Research is located (c) 10 (d) 13
in ? Ans. (c) : India has been divided into 10 biogelographic
(a) Allahabad (b) Faizabad zones as follows- Andaman and Nicobar Island (0.3%)
(c) Kanpur (d) Indore Coastal Region (2.5%), North East Region (5.2%),
Ans. (c) : Indian Institute of Pulses Research (IIPR) is a Gangetic Plains (10.8%), Deccan Plateau (42%),
government institute located in Kanpur, Uttar Pradesh. Western Ghats (4%), Semi Arid Regions (16.6%),
Indian Desert (6.6%), Himalayan Zone (6.4%) and
It was established in 1983 by the Indian Council of
Tans-Himalayan Region (5.6%).
Agricultural Research (ICAR) to carry out basic
strategic and applied research on major pulse crops. 81. The Biodiversity Act was passed by the Indian
Parliament in -
76. The National Academy of Agriculture
(a) May 2000 (b) December 2002
Research Management is located in ?
(c) January 2004 (d) October 2008
(a) Bengaluru (b) Hyderabad
(c) Kolkata (d) New Delhi Ans. (b) : The Biodiversity Act of India was passed by
the Parliament in December 2002. It is an Act enacted
Ans. (b) : The National Academy of Agricultural by the Parliament of India for the preservation of
Research Management (NAARM) was established by biological diversity in India.
the Indian Council of Agricultural Research (ICAR) at
Hyderabad, in 1976 to address issues related to 82. Which of the following pair is matched
agricultural research and education management. correctly ?
(a) Bhotia - Barabanki
77. Which of the following tree is not an eco-
(b) Buksha - Bijnor
friendly tree ?
(c) Razi - Gorakhpur
(a) Acacia (b) Eucalyptus
(d) Tharu - Banda
(c) Neem (d) Ficus
Ans. (b) : The correct match as follow-
Ans. (b) : Eucalyptus is called the enemy of ecology
because it absorbs large amount of water from the soil. List-I List-II
A eucalyptus tree consumes 90 litres of water a day. (Tribes) (Districts)
During summers and times of drought, its roots can go A. Bhotia - Garwal
down upto soft. B. Buksha - Bijnor
78. Consider the following statements :- C. Razi - Pihoragarh
Statement (A) : Ozone is imperative for plant
D. Tharu - Gorakhpur
life.
Reason (R) : Ozone layer protects the Earth 83. 'Karma' is the folk-dance of ?
from high energy radiations. (a) Mahoba (b) Purvanchal
(a) Both (A) and (R) are true and (R) is the (c) Sonbhadra (d) Braj
correct explanation of (A). Ans. (c) : Karma folk dance is performed by Kharwar
(b) Both (A) and (R) are true but (R) is not the tribal people. Its area is Sonbhadra/Mirzapur District of
correct explanation of (A). Uttar Pradesh which is the world-famous folk dance.
(c) (A) is true but (R) is false. 84. In Uttar Pradesh a famous pilgrim site of both
(d) (A) is false but (R) is true. Budhists and Jains is located in ?
Ans. (a) : The ozone layer protects the earth from high (a) Sarnath (b) Devipatan
energy radiation and ozone is essential for biological (c) Kaushambi (d) Kushinagar
life. The ozone layer is between 32 km and 60 km from Ans. (c) : Kaushambi is a holy pilgrims place for both
the surface. The ozone layer protects the earth from Buddhists and Jains in Uttar Pradesh. It was the capital
high energy in ultraviolet-rays. Hence, both Assertion of Vatsa district during the Mahajanpada period. As per
(A) and Reason (R) are true and Reason (R) is the Jain religious texts, it is the birthplace of 6th
correct explanation of Assertion (A). Teerthankara 'Padma Prabhu'.
UP RO/ARO (Mains) Exam 2010 209 YCT
CLICK HERE FOR FREE MATERIAL

85. Uttar Pradesh Janhit Guarantee Act, 2011 Ans. (a) : The correct match as follow.
provides for the availability of services related
List-I List-II
to daily needs of people in a/of/on
(1) Easy way (Event) (Location)
(2) A time bound way A. Magh Mela - Allahabad (Prayagraj)
(3) 13 notified services B. Jhoola Mela - Vrindavan
(4) The basis of break even principle C. Cattle Fair - Bateshwar
Choose the correct answer from the codes (Pashu Mela)
given below :- D. Deva Mela - Barabanki
(a) only 1 & 2 (b) only 1, 2 & 3
(c) only 2, 3 & 4 (d) All the four 89. In Uttar Pradesh first bio-tech park has been
established in -
Ans. (d) : The 13 services have been identified in the
(a) Agra (b) Allahabad
Uttar Pradesh Janhit Guarantee Act, 2011. These
services are based on the break-even economic principle (c) Lucknow (d) Varanasi
and will enable people to get services related to their Ans. (c) : The Lucknow Bio-Technology Park has been
needs in a smooth and time bound manner every day. established a Lucknow with the support from the
86. Which among the following is not correctly department of Biotechnology. It will contribute to mass
matched - production of organic acids, biofertilizers, antibiotics. It
(a) Ganga - Varanasi (b) Gomti - Sultanpur has started its operations since the year 2006.
(c) Kosi - Gorakhpur (d) Saryu - Ayodhya 90. Which one of the following schemes being
Ans. (c) : The correct match as follow. implemented in Uttar Pradesh is not a central
government sponsored scheme ?
List-I List-II
(a) Ganga Express Way
(River) (Districts)
(b) Mahatma Gandhi National Rural
A. Ganga - Varanasi Employment Guarantee Act Programme
B. Gomti - Sultanpur (c) Rural Health Mission
C. Rapti - Gorakhpur (d) Jawahar Lal Nehru National Urban Renewal
D. Saryu - Ayodhya Mission
87. Which of the following Institutions are Ans. (a) : In the given option Ganga Expressway is not
currently located at Lucknow ? a central government scheme. The foundation stone of
Ganga Expressway was laid by Prime Minister Modi on
(1) Jal Sansthan
18th December 2021 in Shahjahanpur. It has been
(2) Nagar Nigam [Municipal Corporation]
designed to connect over 12 district of Uttar Pradesh
(3) Development Authority (Meerut, Jyotibaphule Nagar, Hapur, Sambhal, Badaun,
(4) District Urban Development Authority Shahjahanpur, Farrukhabad, Hardoi, Unnav, Raebareli,
(DUDA) Pratapgarh and Prayagraj).
Choose the correct answer from the codes
given below : 91. The total number of sides in a regular polygon
whose every angle is 156º are :-
(a) only 1 and 2 (b) only 1, 2 and 3
(c) only 2, 3 and 4 (d) All the four (a) 10 (b) 12
(c) 14 (d) 15
Ans. (d) : Municipal Corporation, Water Institution (Jal
Sansthan), Development Authority and District Urban Ans. (d) : No. of sides in regular polygon
Development Authority are all four situated in the
=
( n − 2 ) × 180 = 156
Districts of Lucknow, Uttar Pradesh. n
88. Match List-I with List-II and choose the = 180n – 360 = 156n
correct answer using the codes given below :- = 24m = 360
List-I List-II 360
(Event) (Location) =n=
24
(a) Magh Mela 1. Barabanki = n = 15
(b) Jhoola Mela 2. Bateshwar
92. The age of B is two times that of A, but half of
(c) Pashu Mela 3. Vrindawan F. Age of C is half of the Age of A but twice
(d) Deva Mela 4. Allahabad than that of D. Which two persons age is
Code : maximum and minimum ?
A B C D (a) F & A (b) F and D
(a) 4 3 2 1 (c) F and B (d) F and C
(b) 4 2 3 1 Ans. (b) : According to the question
(c) 1 3 4 2 B = 2A, F = 2B, C = 1/2 A= 2D
(d) 3 1 2 4 So, F > B > A > C > D
UP RO/ARO (Mains) Exam 2010 210 YCT
CLICK HERE FOR FREE MATERIAL

93. If 15778 is written as XTZAL is a coded Ans. (d) :


language and 2346 as NPSU, then how will be
the number 2549 written in same language ?
(a) NPTSL (b) PNTSL
(c) NPTUL (d) NBTSL
Ans. (a) : Given –
XT Z A L → 15789
and NPSU → 2346 Hence, Arun is father in law of this girl.
then from the given codes of words. 99. Which is the number out of the following.
Which gives a remainder 4 when divided by 13
NPTSL → 23549 and gives a remainder 11 when divided by 17 ?
So, option (a) is correct. (a) 578 (b) 589
94. Which one among the following is not identical (c) 654 (d) 760
to rest of them ? Ans. (b) : By option–
(a) CHM (b) DIN 589 when divided by 13 leaves a remainder of 4 and
(c) LPU (d) KOT when divided by 17 leaves remainder of 11.
Ans. (a) : CHM does not contain any vowel letter So, 589 is correct.
whereas DIN, LPU, KOT contain vowel letter I,U,O 100. What is the value of X which will make QCP a
respectively. straight live ?
95. Which of the following number will occupy the
vacant space in the following number series :-
2-3, 4-6, 7-10, 11-15, 16-21, 22.... (a) 34º (b) 60º
(a) 25 (b) 27 (c) 68º (d) 120º
(c) 28 (d) 29 Ans. (a) : After making QCP a straight line –
Ans. (c) : 3x + 10 + 2x = 180
5x = 180 – 10
5x = 170
170
x=
Hence, option (c) is correct. 5
x = 34º
96. The given diagram portrays : Doctor, Father 101. A sum of Rs. 425 is divided among 4 men, 5
and man. Which figure depicts doctors - women and 6 children in the ratio of 9:8:4, then
what will be the share of each men (in Rs.)?
(a) 39.75 (b) 38.25
(c) 35.75 (d) 35.25
Ans. (b) : According to the question -
(a) Circle (b) Square 9x + 4 + 8x × 5 + 4x × 6 = 425
(c) Rectangle (d) None of these = 36x + 40x + 24x = 425
Ans. (c) : In given figure rectangle represent the doctor = 100x = 425
because a doctor can be both male or female but a man x= 4.25
can be only father. Share of 4 men's = 9x × 4
Hence (c) is correct option. = 9 × 4.25 × 4 = 153
153
97. The next letter in the letter series AGLPS .... So, the share of one man = = 38.25
will be - 4
(a) B (b) R 102. In the new series of Industrial Production
Index (IPI) which was released by the Union
(c) T (d) U Government. on 10th June 2011, the base year
Ans. (d) : Given series– has been changed from 1993-94 to -
(a) 2001-02 (b) 2004-05
(c) 2008-09 (d) 2009-10
Ans. (b) : The new category of Index of Industrial
98. Arun says that - "this girl is the wife of the Production (IIP) was released by the Government of
grandson of my mother". What is the relation India on 10th June 2011, the base year has been change
of Arun with the girl ? to 2004-05, while earlier it was 1993-94. At present, the
(a) Grand father (b) Husband base year for the IIP is 2011-12, which was determined
(c) Father (d) Father-in-law in May 2017.
UP RO/ARO (Mains) Exam 2010 211 YCT
CLICK HERE FOR FREE MATERIAL

103. Which of the following statements with regard Ans. (b) : The correct match as follow-
to Lokpal Act, 2011 is not correct ?
List-I List-II
(1) It was reintroduced into the Lok Sabha on 4th
Aug, 2011. (Book) (Author)
(2) It was restored/reinstituted by the Home A. Ghar aur Adalat - Leela Seth
Minister Shri. P. Chidambaram. B. Jhopdi se Rashtrapati - Mahendra Kulshrestha
(3) It has been sent to the Standing Committee Bhawan Tak
(4) It has been dubbed as a weak legislation by C. Imagining India - Nandan Nilekani
civil society members.
D. Journey Through - T.S.R. Subramanian
Codes :
Babudam and Neta
(a) only 1 & 2 (b) only 1, 2 & 3
(c) only 2 & 4 (d) only 1, 3 & 4 Land
Ans. (d) : The Lokpal Bill 2011 was re-introduced in 107. "These (Americans) are living much beyond
the Lok Sabha by P. Chidambaram on 22 December, their status, they are living as parasite aloof
2011 and has been referred to the select committee. from the world economy". These comments
Civil society members called it weak as its provisions were made by ?
were not able to prevent political corruption. According (a) Chinese Prime Minister
to the Bill passed on 18 December 2013, the jurisdiction (b) Russian Prime Minister
of the Lokpal includes the Prime Ministers, Ministers, (c) Frances President
Members of Parliament and officers and employees of (d) Communist Party of India
Groups A, B, C and D of the Central Governmental. Ans. (b) : In the given statement in question, the
104. Which of the following bank has constituted a remarks were accused in August 2011 in the context of
"Farmers Club" to facilitate easy access to an assessment of the global economy by the then
farmers ? Russian Prime Minister Vladimir Putin.
(a) Allahabad Bank
108. Film star - Shammi Kapoor, debut film name
(b) Punjab National Bank who dead on 14th August, 2011 was?
(c) Regional Gramin Bank (a) Tumsa Nahi Dekha (b) Rail ka Dabba
(d) State Bank of India
(c) Laila-Majnu (d) Jeevan Jyoti
Ans. (d) : In a significant move to help the farmers of
Uttar Pradesh, the State Bank of India and NABARD have Ans. (d) : Film actor Shammi Kapoor died on 14th
joined hands to establish a "Farmers Club." The club aims August 2011. His full name was Shamsher Raj Kapoor.
at enhancing the bargaining power of the farmers and His first film of acting was Jeevan Jyoti released in the
artisans besides reaching out to the rural masses. year 1953.
105. Who among the following defeated Saina Nehwal 109. Who among the following has been selected for
in the Womens singles final of the match of 2011, Ramon Magsaysay Award
Indonesian an Open Super Series 2011 ? (a) Niket Mishra (b) Nileema Mishra
(a) Yihan Wang (b) Wong Shin (c) Mishika Singh (d) Chetan Bhagat
(c) Tyne Brown (d) Cheng Shao Chia Ans. (b) : Raman Magsaysay Award 2011 was
Ans. (a) : In the 2011, Indonessia Open Surper Series, presented to Nileema Mishra and Harish Hande. This
Yihan Wang defeated Saina Nehwal. Ratchanok Ramon Magsaysay Award, instituted in the year 1957,
Intanon won the women's singles semifinal of the is considered highest honor. In the 2022, Raman
Simlnvest Indonesia Open 2011 held in year 2021 by Magsaysay Awardees are Sothearna Chhim
defeating P.V. Sindhu of India. (Cambodia), Bernadette Madrid (Philippines), Tadashi
106. Match List-I with List-II and choose the Hattori (Japan) and Gary Bencheghib (Indonesia).
correct answer using the codes given below : 110. Which of the following Sea port has been
List-I List-II included in the work plan for Nibhar Jal
(Book) (Author) Prabandhan (management) by the National
(a) Ghar aur Adalat 1. T.S.R. Subramanian Ocean Science Institute or (NIOT) ?
(a) Chennai (b) Cochin
(b) Jhopdi se Rashtrapati 2. Nandan Nilekani
Bhawan Tak (c) Kolkata (d) Vishakahapattanam
(c) Imagining India 3. Mahendra Ans. (a) : Chennai has been included in the action plan
Kulshrestha prepared by the National Institute of Oceanography for
(d) Journey Through 4. Leela Seth the management of Nibhar (Jal) Waters.
Babudam and Neta 111. Which of the following states has not been
Land included in the list of "highly concentrated
Code : states" for the family planning programme ?
A B C D (a) Haryana (b) Jammu & Kashmir
(a) 4 1 3 2 (c) Punjab (d) Rajasthan
(b) 4 3 2 1 Ans. (b) : Jammu and Kashmir has not been included in
(c) 1 3 2 4 the list of 'Highly Concentrated States' for family
(d) 3 2 4 1 planning.
UP RO/ARO (Mains) Exam 2010 212 YCT
CLICK HERE FOR FREE MATERIAL

112. The Tenth Conference of members/teams of Ans. (d) : The correct match as follows-
United Nations Conference on bio-diversity List-I List-II
was organised in -
(Arjun Khel Award (Sports)
(a) Beijing (b) Copenhagen Winner-2011)
(c) Nagoya (d) New York
A. Ashish Kumar - Gymnastics
Ans. (c) : The 10th Conference of Parties to the United
Nations Convention on Biodiversity was held in B. Jwala Gutta - Badminton
Nagoya. The 15th COP will be held on 7-19 December C. Ravi Kumar - Weight Lifting
2022 in Montreal, Canada. D. Tejasvini Sawant - Shooting
113. The interests which have been pladged to be 117. Who among the following has taken over the
protected in the I.L.O's 100th annual meet are- charge of Tibetan government in exile new
(a) Child labour Prime Minister in August, 2011 ?
(b) Domestic servants (a) Lobsang Sangay (b) Dhondup Sodane
(c) Landless agricultural labourer (c) Jigme Thinle (d) Galpon tsange
(d) Women labourers Ans. (a) : In August 2011, Lobsang Sangay assumed
Ans. (b) : In June 2011, the 100th meeting of the the position of the new Prime Minister of Tibet's
International Labour Organization (ILO) was held in government in-exile. The chairman of the Central
Geneva Switzerland, in which a decision was taken to Tibetan Administration (CTA) and the current Prime
protect the interests of domestic workers. Minister of the Tibetan Government-in-Exile is
114. Which of the following statements relating to Lobsang Sangay.
South Sudan are true? Use the codes given 118. Locusts enter India through -
below to choose the right answer. (a) Bangladesh (b) Nepal
(I) It is land locked country (c) Pakistan (d) Srilanka
(II) Its capital is in Akoba Ans. (c) : Locusts enter India from Pakistan. Their
(III) Its main river is While Nile origin mainly comes from the coastal parts of the Red
(IV) It chief religion is Islam Sea and around the month of December from the
Codes : Persian Gulf via Pakistan reaches India.
(a) I & II only (b) II and III only 119. Who among the following translated the
(c) I & III only (d) II & IV only famous composition of Kalidas - "Shakuntala"
Ans. (c) : South Sudan came into existence as a country into English for the first time ?
in July 2011, it is landlocked country and its capital is (a) Charles Wilkins
'Juba'. The main following river here is 'White Nile'. (b) Henry Colebrook
Where the people of Christianity are in the majority. (c) Johan Woltgang Von Goethe
115. Rajiv Gandhi Khel Ratna Award, 2011 has (d) Sir Willion Jones
been conferred on- Ans. (d) : Kalidas famous book Abhigyan Shakuntalam
(a) Abhinav Bindra (b) Gagan Narang was first translated into English by 'Sir William Jones',
(c) Pankaj Advani (d) Sushil Kumar while 'Gita' was translated by Charles Wilkins.
Ans. (b) : Rajiv Gandhi Khel Ratna Award was given to 120. Consider the following statements :-
shooter Gagan Narang in 2011. Major Dhyan Chand Khel Assertion (A) : The Right to Information Act
Ratna Award, formerly known as Rajiv Gandhi Khel has normally been responsible for increasing
Ratna Award. It was introduced in the year 1991-1992. the sence of accountability in the Bureaucracy.
116. Match List-I with List-II and choose the Reason (R) : It has miles to cover for
correct answer using the codes given below :- generating the desired impact.
List-I List-II Choose the correct answer using the codes
(Arjun Khel Award (Sports) given below :
Winner-2011) (a) Both (A) & (R) are correct and (R) is the right
(a) Ashish Kumar 1) Badminton explanation of (A).
(b) Jwala Gutta 2) Shooting (b) Both (A) & (R) are correct but (R) is not the
(c) Ravi Kumar 3) Gymnastics right explanation of (A).
(c) (A) is true but (R) is false.
(d) Tejasvini Sawant 4) Weight Lifting
(d) (A) is false but (R) is true.
Code :
Ans. (b) : The Right to Information Act has been
A B C D responsible for inculcating a sense of responsibility in
(a) 3 4 2 1 the bureaucracy in general and it has yet to travel for
(b) 4 1 3 2 miles to produces the desired effect. Hence, both the
(c) 1 2 4 3 above statement are truss but reason is not the correct
(d) 3 1 4 2 explanation of the assertion.
UP RO/ARO (Mains) Exam 2010 213 YCT
CLICK HERE FOR FREE MATERIAL

Gòej ØeosMe meceer#ee DeefOekeâejer/meneÙekeâ meceer#ee DeefOekeâejer (cegKÙe) hejer#ee, 2010


meeceevÙe efnvoer
nue ØeMve-he$e
meeceevÙe Meyo-%eeve SJeb JÙeekeâjCe 6. efvecveefueefKele ceW lelmece Meyo nw
(a) keâjesÌ[ (b) lesue
1. `efpeme heg®<e keâer helveer meeLe veneR nw' JeekeäÙeebMe kesâ efueS Skeâ
(c) heerheue (d) efove
Meyo nw
Gòej–(d)
(a) Dehelveerkeâ (b) efJeÙeesieer
(c) efJeOegj (d) efJehelveerkeâ JÙeeKÙee- efvecveefueefKele ceW efove lelmece Meyo nw~ peyeefkeâ keâjeWÌ[, lesue,
Gòej–(d) heerheue Deeefo leÆJe Meyo nQ~
7. efvecveefueefKele MeyoeW ceW mes Skeâ leodYeJe Meyo veneR nw
JÙeeKÙee – ‘‘efpeme heg®<e keâer helveer meeLe veneR nw’’ Gmekesâ efueS
GheÙegòeâ Meyo nesiee – efJehelveerkeâ~ efpemekeâer helveer cej Ûegkeâer nes Gmes (a) legjble (b) Deepe
efJeOegj keâne peelee nw, efpemekeâe efkeâmeer mes efJeÙeesie nes ieÙee nes, Gmes (c) Oeerjpe (d) Kehe&j
efJeÙeesieer keâne peelee nw~ Gòej–(d)
2. ‘efpeme hesÌ[ kesâ heòes PeÌ[ ieÙes neW'– kesâ efueS Skeâ Meyo nw JÙeeKÙee- Deepe, legjble Deewj Oeerjpe leodYeJe Meyo nw peyeefkeâ Kehe&j
(a) ØeheCe& (b) DeheCee& lelmece Meyo nw~
(c) he$enerve (d) Dehele 8. veerÛes efoÙes `lelmece-leodYeJe' MeyoeW kesâ Ùegice ceW mes keâewve-mee
Gòej–(b) Ùegice $egefšhetCe& nw?
JÙeeKÙee- efpeme hesÌ[ kesâ heòes PeÌ[ ieÙes nes, Gmekesâ efueS Skeâ Meyo (a) #eerj – Keerj (b) oefn – oner
‘DeheCee&’ nw~Mes<e efJekeâuhe Demebiele nQ~ (c) ogiOe – otOe (d) Ie=le – Ieer
3. `lewjves keâer FÛÚe' keâes keânles nQ Gòej–(b)
(a) lejCesÛÚe (b) efleleer<ee& JÙeeKÙee- efoÙes ieÙes efJekeâuheeW ceW #eerj keâe Keerj, ogiOe keâe otOe Deewj
(c) meblejCesÛÚe (d) peueeJelejCesÛÚe Ie=le keâe Ieer lelmece neslee nw, peyeefkeâ oner keâe oefOe lelmece ™he neslee nw
ve efkeâ oefn~ Dele: efJekeâuhe (b) $egefšhetCe& nw~
Gòej–(b)
9. veerÛes efoÙes lelmece-leodYeJe Meyo Ùegice ceW mes keâewve-mee Ùegice
JÙeeKÙee- ‘‘lewjves keâer FÛÚe’’ kesâ efueS GefÛele Meyo nw – ‘efleleer<ee&’~
$egefšhetCe& nQ?
Mes<e efJekeâuheeW kesâ Meyo $egefšhetCe& nwb~
(a) JeÛeve – yewve (b) keâheeš – keâheÌ[e
4. `osJej' keâe lelmece Meyo nw
(c) hegjeCe – hegjevee (d) iecYeerj – ienje
(a) osJeJej (b) efÉJej
Gòej–(b)
(c) ogJej (d) efÉleerÙeJej
JÙeeKÙee- efoÙes ieÙes lelmece-leodYeJe Meyo Ùegice ceW JeÛeve keâe yewve,
Gòej–(b)
hegjeCe keâe hegjevee, iecYeerj keâe ienje mener Ùegice nw~ peyeefkeâ keâheeš keâe
JÙeeKÙee- osJej keâe lelmece Meyo nw – efÉJej~ Mes<e efJekeâuheeW kesâ Meyo keâheÌ[e ieuele nw~ Fmekeâe mener lelmece-leodYeJe Ùegice nesiee – ‘keâheeš-
$egefšhetCe& nQ~ efkeâJeeÌ[’ nesiee~
5. `cekeäKeve' keâe lelmece Meyo nw 10. `mebÙeesie' Meyo keâe efJeueesce nw
(a) ceeKeve (b) cee#eCe (a) ogÙeexie (b) efJeÙeesie
(c) ce<e#e (d) ce=#eCe (c) menÙeesie (d) kegâÙeesie
Gòej–(d) Gòej–(b)
JÙeeKÙee- cekeäKeve keâe lelmece Meyo ce=#eCe nw, peyeefkeâ DevÙe leerveeW JÙeeKÙee- mebÙeesie Meyo keâe efJeueesce efJeÙeesie nesiee~ Mes<e efJekeâuheeW kesâ
efJekeâuhe ieuele nw~ Meyo Demebiele nQ~
UP UDA/LDA (Main) General Hindi 2010 214 YCT
CLICK HERE FOR FREE MATERIAL

11. veerÛes efoÙes Meyo Ùegice ceW keâewve-mee $egefšhetCe& nw? 17. veerÛes efoÙes MeyoeW ceW efkeâmekeâer Jele&veer Megæ nw?
(a) Deveg«en – Dee«en (b) Devevle – meevle (a) mebvÙeemeer (b) Deekeâeue
(c) peÌ[ – Ûesleve (d) Deeke=â„ – efJeke=â„ (c) Deveg«enerle (d) DeepeerJekeâe
Gòej–(a) Gòej–(a)
JÙeeKÙee- Devevle keâe meevle, peÌ[ keâe Ûesleve Deewj Deeke=â° keâe efJeke=â° JÙeeKÙee- efoÙes ieÙes efJekeâuhe ceW ‘mebvÙeemeer’ Megæ Meyo nw peyeefkeâ
efJeueesce nesiee~ Deveg«en keâe Dee«en ve neskeâj efJe«en efJeueesce nesiee~ Deveg«enerle keâe Devegie=nerle, Deekeâeue keâe Dekeâeue, DeepeerJekeâe
‘DeepeerefJekeâe’ Megæ Meyo nesiee~
12. `$e+peg' Meyo keâe efJeueesce nw
18. efvecveefueefKele ceW mes Megæ Jele&veer Jeeuee Meyo keâewve-mee nw?
(a) Je›eâ (b) le›eâ
(a) Devlee#ejer (b) DevlÙee#ejer
(c) meerOee (d) efJejue
(c) Devleeef#ejer (d) Devleeef#eCeer
Gòej–(a)
Gòej–(b)
JÙeeKÙee- $e+peg Meyo keâe efJeueesce Je›eâ neslee nw~ meerOee keâe efJeueesce
JÙeeKÙee-efoÙes ieÙes efJekeâuheeW ceW Megæ Jele&veer Jeeuee Meyo DevlÙee#ejer nw~
šsÌ{e neslee nw Deewj efJejue keâe efJeueesce meIeve neslee nw~
Mes<e efJekeâuheeW kesâ Meyo $egefšhetCe& nQ~
13. `Deekeâueve' Meyo keâe efJeueesce nw
19. efvecveefueefKele ceW keâewve-mee JeekeäÙe Megæ nw?
(a) efJekeâueve (b) mebkeâueve
(a) Ùen Deehekeâer DeveeefOekeâej Ûes„e nw~
(c) meceekeâueve (d) Øeekeäkeâueve
(b) Ùen Deehekeâer DeveefOekeâej Ûes„e nw~
Gòej–(a)
(c) Ùen Deehekeâer Ûes„e DeveeefOekeâej nw~
JÙeeKÙee- Deekeâueve keâe efJeueesce efJekeâueve neslee nw~ Mes<e efJekeâuheeW kesâ (d) Ùen Ûes„e Deehekeâe DeveeefOekeâej nw~
Meyo Demebiele nQ~ Gòej–(b)
14. efvecveefueefKele ceW keâewve-mee efJeueesce-Ùegice $egefšhetCe& nw? JÙeeKÙee- DeveeefOekeâej kesâ mLeeve hej DeveefOekeâej Megæ Meyo nw~ ‘‘Ùen
(a) #ej – De#ej (b) meceeme – JÙeeme Deehekeâer DeveefOekeâej Ûes°e nw~’’ Megæ JeekeäÙe nw~ Ùeneb hej Jele&veeriele oes<e nw~
(c) mJeuheeÙeg – efÛejeÙeg (d) Deenej – efJenej 20. efvecveefueefKele ceW keâewve-mee JeekeäÙe Megæ nw?
Gòej–(d) (a) Gmeves Deheveer keâceeF& keâe DeefOekeâebMe Yeeie ieBJee efoÙee~
JÙeeKÙee- #ej keâe efJeueesce De#ej, meceeme keâe JÙeeme Deewj mJeuheeÙeg keâe (b) Jen Deheveer keâceeF& keâe DeefOekeâebMe Yeeie ieBJee yew"e~
efÛejeÙeg neslee nw Jener Deenej keâe efJeueesce efJenej ve neskeâj efvejenej neslee nw~ (c) Gmeves keâceeF& keâe DeefOekeâebMe Yeeie ieBJee [euee~
15. efvecveefueefKele ceW keâewve-mee efJeueesce-Ùegice $egefšhetCe& nw? (d) Gmeves Deheveer keâceeF& keâe DeefOekeâebMe ieBJee efoÙee~

(a) Deeefce<e – meeefce<e (b) DeeOeej – DeeOesÙe Gòej–(d)


(c) efJevele – Gæle (d) efJeheefòe – mebheefòe JÙeeKÙee- ‘‘Gmeves Deheveer keâceeF& keâe DeefOekeâebMe Yeeie ieBJee efoÙee~’’
Gòej–(a) Fme JeekeäÙe ceW DeefOekeâebMe ceW DeefOekeâ DebMe Meyo pegÌ[e ngDee nw efpemekeâe
DeLe& neslee nw – Yeeie~ Dele: DeefOekeâebMe kesâ yeeo Yeeie Meyo ueieevee
JÙeeKÙee- DeeOeej keâe DeeOesÙe, efJevele keâe Gæle Deewj efJeheefòe keâe
ieuele nw~ Megæ JeekeäÙe nesiee – Gmeves Deheveer keâceeF& keâe DeefOekeâebMe
mecheefòe mener efJeueesce Meyo nw peyeefkeâ Deeefce<e keâe meeefce<e ve neskeâj ieBJee efoÙee~
efvejeefce<e efJeueesce neslee nw~
21. efvecveefueefKele ceW mes keâewve-mee JeekeäÙe mener nw?
16. veerÛes efoÙes MeyoeW ceW mes efkeâmekeâer Jele&veer Megæ nw? (a) ßeer jece Ûeewon Je<e& kesâ yeeo Jeve mes Jeeheme ueewšs~
(a) Éejkeâe (b) hetpÙeeveerÙee (b) ßeerjece Ûeewon Je<e& kesâ yeeo JeveJeeme mes Jeeheme ueewšs~
(c) DevleOÙee&ve (d) DeefnequÙeÙee (c) ßeerjece Ûeewon Je<e& yeeo Jeve mes Jeeheme ueewšs~
Gòej–(a) (d) ßeer jece Ûeewon Je<e& kesâ yeeo Jeve mes ueewšs~
JÙeeKÙee- efoÙes ieÙes efJekeâuheeW ceW Megæ Jele&veer nw – Éejkeâe~ Mes<e Gòej–(d)
efJekeâuheeW keâer Jele&veer DeMegæ nw~ Fvekeâe Megæ ™he hetpÙeeveerÙee keâe JÙeeKÙee- ‘‘ßeer jece Ûeewon Je<e& kesâ yeeo Jeve mes ueewšs~’’ Megæ JeekeäÙe
‘hetpeveerÙe’, DevleOÙee&ve keâe ‘DevleOee&ve’ leLee DeefnefuÙeÙee keâe ‘DenuÙee’ nw~ ‘Jeeheme ueewšs’ oesveeW Meyo Skeâ meceeveeLe&keâ YeeJeeW keâes JÙeòeâ keâjles
nesiee~ nQ~ Dele: Ùeneb hej oesveeW MeyoeW keâe meeLe-meeLe ØeÙeesie hegve®efòeâ oes<e nesiee~
UP UDA/LDA (Main) General Hindi 2010 215 YCT
CLICK HERE FOR FREE MATERIAL

22. efvecveefueefKele ceW keâewve-mee Meyo efJeMes<eCe veneR nw? JÙeeKÙee- megieefvOele keâmletjer kesâ ueesYeer efMekeâejer jepemLeeveer efnjCeeW keâe
(a) kegâefšuelee (b) peefšuelee DeJewOe efMekeâej keâjles nQ~ FmeceW Ûeej efJeMes<eCe Deewj Ûeej efJeMes<Ùe nQ~ pees
(c) keâewefšuÙe (d) kegâ¤helee meb%ee Ùee meJe&veece keâer efJeMes<elee yeleeS, Gmes ‘efJeMes<eCe’ keânles nQ~
Gòej– (*) efpemekeâer efJeMes<elee yeleeF& peeS Jen ‘efJeMes<Ùe’ keânueelee nw~
JÙeeKÙee- kegâefšuelee, peefšuelee, kegâ™helee, keâewefšuÙe ÛeejeW YeeJeJeeÛekeâ 26. `mecegõer meeBhe ceW Ieelekeâ hejvleg yengle keâerceleer ]penj heeÙee peelee
meb%ee nQ~ Fvekeâe efJeMes<eCe-efJeMes<Ùe ™he efvecve nw– nw'–JeekeäÙe ceW nQ–
efJeMes<Ùe efJeMes<eCe (a) leerve efJeMes<eCe Deewj oes efJeMes<Ùe
(b) oes efJeMes<eCe Deewj leerve efJeMes<Ùe
kegâefšuelee kegâefšue
(c) oes efJeMes<eCe Deewj oes efJeMes<Ùe
peefšuelee peefšue
(d) Ûeej efJeMes<eCe Deewj oes efJeMes<Ùe
keâewefšuÙe kegâefšue
kegâ®helee kegâ®he Gòej–(d)
Dele: efJekeâuhe ceW keâesF& Gòej mener veneR nw~ JÙeeKÙee- mecegõer meeBhe ceW Ieelekeâ hejvleg yengle keâerceleer penj heeÙee peelee
23. veerÛes efoÙes efkeâme JeekeäÙe ceW efJeMes<eCe keâe ØeÙeesie ngDee nw? nw~ FmeceW Ûeej ‘efJeMes<eCe’ Deewj oes ‘efJeMes<Ùe’ nw~ pees meb%ee meJe&veece keâer
efJeMes<elee yeleeÙes nQ Gmes efJeMes<eCe keânles nQ~ efpemekeâer efJeMes<elee yeleeF&
(a) jece Deewj MÙeece YeeF& nw
peeS Gmes Gmes efJeMes<Ùe keânles nQ~
(b) jece ves ceeB mes heeveer Deewj Keevee ceeBiee~
27. `oMejLe kesâ ØeeCe jece kesâ efueS Deekegâue Les'–JeekeäÙe ceW cegKÙe
(c) ceeB ves Keeves ceW oeue Deewj jesšer hejesmeer~
efJeMes<Ùe nw–
(d) jece ves jesšer Keekeâj Deewj jesšer ceeBieer~
(a) oMejLe (b) jece
Gòej–(d)
(c) ØeeCe (d) Deekegâue
JÙeeKÙee- ‘‘jece ves jesšer Keekeâj Deewj jesšer ceeBieer~’’ JeekeäÙe ceW ‘Deewj’ Gòej–(c)
Meyo Deefveef§ele mebKÙeeJeeÛeer efJeMes<eCe kesâ ™he ceW ØeÙegòeâ ngDee nw~
JÙeeKÙee- oMejLe kesâ ØeeCe jece kesâ efueS Deekegâue Les~’’ JeekeäÙe ceW cegKÙe
24. veerÛes efoÙes efkeâme JeekeäÙe ceW efJeMes<eCe keâe ØeÙeesie ngDee nw? efJeMes<Ùe ØeeCe nw~ efJeMes<eCe efpemekeâer efJeMes<elee yeleueelee nw, Gmes efJeMes<Ùe
(a) ueÌ[keâe DeeÙee nw~ keânles nQ~
(b) Jen iegueeye kesâ Heâtue ueeÙee nw~ 28. `pees m$eer metÙe& Yeer ve osKe mekesâ' JeekeäÙeebMe kesâ efueS Skeâ Meyo nw–
(c) iegueeye kesâ Hetâue cegPes hemebo nQ~
(a) DemetÙe&oMe&vee (b) DemetÙe&„e
(d) Jen ueÌ[keâe Hetâue oskeâj Ûeuee ieÙee~
(c) DemetÙe&mheMee& (d) DemetÙe&heMÙee
Gòej–(d) Gòej–(d)
JÙeeKÙee- veerÛes efoÙes JeekeäÙeeW ceW efJeMes<eCe keâe ØeÙeesie ‘Jen ueÌ[keâe hetâue JÙeeKÙee- ‘‘pees Œeer metÙe& Yeer ve osKe mekesâ’’ Gmekesâ efueS ‘DemetÙe&heMÙee’
oskeâj Ûeuee ieÙee~’ JeekeäÙe ceW nw~ GheÙeg&òeâ JeekeäÙe cebs ‘Jen’ meeJe&veeefcekeâ Meyo GheÙegòeâ nw~
efJeMes<eCe nw~
29. `pees Ùegæ ceW eqmLej jnlee nw' Gmes keânles nQ–
meeJe&veeefcekeâ efJeMes<eCe–Ssmes meJe&veece pees meb%ee mes henues ueiekeâj Gme
(a) Ùegæ-mLeefJej (b) Ùegæ-efLejkeâ
meb%ee Meyo keâer efJeMes<eCe keâer lejn efJeMes<elee yeleeles nQ Jes Meyo
(c) ÙegefOeef‰j (d) ÙegæmLeeÙeer
meeJe&veeefcekeâ efJeMes<eCe keânueeles nQ, pewmes–Jen hegmlekeâ, Jen Deeoceer, Jen
ueÌ[keâer Deeefo~ Gòej–(c)
25. `megiebefOele keâmletjer kesâ ueesYeer efMekeâejer jepemLeeveer efnjCeeW keâe JÙeeKÙee- ‘‘pees Ùegæ ceW efmLej jnlee nw’’ Gmes ÙegefOeef‰j keânles nQ~ Mes<e
DeJewOe efMekeâej keâjles nQ'–JeekeäÙe ceW nQ efJekeâuheeW kesâ Meyo Demebiele nQ~
(a) leerve efJeMes<eCe Deewj leerve efJeMes<Ùe 30. `pees ncesMee jnves Jeeuee nw' Gmekeâes keânles nQ–
(b) oes efJeMes<eCe Deewj oes efJeMes<Ùe (a) MeeMJele (b) DeveJejle

(c) Ûeej efJeMes<eCe Deewj Ûeej efJeMes<Ùe (c) DeØeeflenle (d) DeepeerJekeâ

(d) leerve efJeMes<eCe Deewj Ûeej efJeMes<Ùe Gòej–(a)


Gòej–(c) JÙeeKÙee- ‘‘pees ncesMee jnves Jeeuee nw’’ Gmekeâes ‘MeeMJele’ keânles nQ~
UP UDA/LDA (Main) General Hindi 2010 216 YCT
CLICK HERE FOR FREE MATERIAL

UPPSC RO/ARO (Pre) Exam-2013


GENERAL STUDIES
Solved Paper
1. The number of Bhakti forms in the Bhagvat Ans. (d) : Firoz Shah Tughlaq was born in 1309 and
Sect is? ascended the throne of Delhi after the demise of his
(a) 7 (b) 8 cousin Muhammad-bin-Tughlaq. He was the third ruler
(c) 9 (d) 10 of Tughlaq dynasty and Firuz Shah Tughlaq was in
Ans. (c) : The number of forms of bhakti in the power from 1351 to 1388 A.D. He worked major
Bhagavat sect is nine which are also called Navadha welfare policies, He established a charity department
Bhakti. These are shravana, kirtana, smarana, pada- looked into the welfare of widows, orphans and needy
sevana, archana vandana, dasya, sakhya and atma- children. He set up an employment bureau to provide
nivedana. The world bhakti comes from the root bhaj, job opportunities to unemployed people. He built
which means to worship. It is turning towards the various canals, tanks and dug wells. He died in 1388
supreme with a loving heart. and the Tughlaq dynasty could not continue much after
2. Lord Buddha came to Kaushambi in whose his death.
ruling period? 6. Who was the highly educated ruler among the
(a) Shatanik (b) Udayana following Medieval Rulers?
(c) Bodhi (d) Nikakshu (a) Balban (b) Allauddin Khilji
Ans. (b) : Mahatama Buddha visited the Kaushambi (c) Ibrahim Lodi (d) Shershah
during the reign of king Udayana. Kaushambi was ninth Ans. (d) : Shershah Suri born as Farid Khan was the
resting place, It was the capital of the Vatsa kingdom, founder of Sur empire in India with it capital in Sasaram
one of the sixteen mahajanpadas. Udayana was an in modern day Bihar. He was highly educated among
Upasaka (lay follower) of Buddha. the medieval rulers. Sher Shah Suri did special study on
Arabic, Persian, History while staying in Jaunpur.
3. Which religion had got protection from
Jaunpur was the center of famous education at the time,
Rashtrakutas?
which was famous in India as 'Siraj' of the East.
(a) Budhist Religion (b) Jain Religion
7. 'Heliodorus' mentioned in the 'Besnagar
(c) Shaivili Religion (d) Shakta Religion
inscription' was a resident of?
Ans. (b) : Dantidurga or Dantivarman was the founder (a) Pushlavati (b) Takshila
of the Rashtrakutas dyanasty. The dynasty ruled large (c) Sakal (d) Mathura
parts of Indian subcontinent between the sixth and tenth
centuries. The Rashtrakutas were patrons of Jainism and Ans. (b) : Heliodorus of Besnagar inscription was a
liberally patronized the Digambara sect of Jains. resident of Takshila. In the 14th year of reign of the
Amoghavarsha-I, Indra IV, Krishna-II and Indra III ninth ruler of the Sunga dynasty, Bhagbhadra,
were supporters of Jainism. In the court of Heliodorus, the ambassador of the Yavana ruler
Amoghavarsha-I, famous Jain scholar and author of 'Antialkita' of Takshila, erected a Garuna pillar in
'Adipuran', Jinsen used to reside. honour of Vasudeva near Vidisha. The Garuna pillar of
Heliodorus is the first stone movement related to
4. Who was known as Shaka conqueror? Hinduism. The pillar is also known as Garudadhwaja or
(a) Chandragupta I (b) Samudragupta Besnagar pillar inscription. Hence, option (b) is correct
(c) Chandragupta II (d) Kumargupta answer.
Ans. (c) : Chandragupta-II was known as Shaka- 8. The Mughal rulers of the Medieval India were
conqueror. He was also known as Vikramaditya. His actually?
greatest achievement was to destroy the power of the (a) The Persians (Iranian)
Shaka from Malwa, Gujarat and Saurashtra permanently. (b) The Afghans
Chandragupta-II also defeated the northern rulers like (c) The Chagtai Turks
Hunas, Kambojas, Kiratas etc. His others names (as (d) None of these
mentioned in coins) include Vikram, Devagupta, Devraj, Ans. (c) : The Mughal rulers of medieval India
Simhavikram, Vikramaditya Sakari etc Oahein, a buddhist belonged to the Chagatai dynasty because Babar was
from China came to India during his reign. the fifth descendant of Timur from the paternal side and
5. Who of the following sultans of Delhi had the 14th descendant of Genghis Khan from the maternal
established an employment exchange to help side, thus mixing the blood of Turks and Mongols. The
the unemployed people? new dynasty that Babur laid the foundation was the
(a) Balban (b) Allauddin Khilji 'Chagatai' Turk dynasty of turk breed (turki race). This
(c) Mohd Bin Tughlaq (d) Firoz Shah Tughlaq dynasty was named the second son of Genghis Khan.
UP Ro/ARO (Pre) Exam 2010 217 YCT
CLICK HERE FOR FREE MATERIAL

9. In context of the Mughal Mansabdari system 13. Who among the following women
consider the following statements. revolutionaries had fired upon the English
(1) There used to be granted posts of 'Jat & Viceroy (Governor) while receiving her degree
Sawars' in a convocation ceremony?
(2) Mansabdars were hereditary officers (a) Shanti Ghosh (b) Suniti Chaudhari
(3) There were three categories of Mansabdars (c) Bina Das (d) Kalpana Dutt (Joshi)
(4) They were disbursed salaries from the Ans. (c) : Bina Das was the female revolutionary who
Deewan fired at the English Governor (Chancellor) while receiving
Choose the correct answer from the codes her degree at the convocation. Bina Das was an Indian
given below:- revolutionary and nationalist from West Bengal.
Codes: 14. Among the following revolutionaries of 1857
mutiny whose actual name was 'Ramchandra
(a) 1, 2, 3 and 4 (b) 1, 2 and 3
Panduranga'?
(c) 1 and 3 (d) None of these
(a) Kunwar Singh (b) Tatya Tope
Ans. (c) : Mansabdari system was the bureaucratic (c) Nana Saheb (d) Mangal Pandey
administration system of Mugal Rulers in India. Akbar
Ans. (b) : Tatya Tope was a member of the Indian
introduced the mansabdari system of administration. Rebellion of 1857. Tantia Tope, also spelled Tatya Tope
Mansabdari was both civil and military. The mansabdar or Tantia Topi, original name Ramchandra Panduranga,
held two designations Zat (personal rank and pay status) after the fall of Gwalior he went to Nepal in April,
and Sawar (number of horseman of maintain). The 1859, where he was caught after the betrayal of a
position of mansabdari was not hereditary. It ended landlord friend Mansingh and hanged on April 18,
automatically after the death or removal of the 1859. The arrest of Tatya Tope was the final event of
mansabdar. The mansabdars received their salary in the Revolt of 1857 in central India.
form of cash or jagir, their office of salary was under 15. The president of the Annual Session of Indian
'Mirbakshi'. Hence, option (c) are correct answer. National Congress held at Surat in 1907 was-
10. Which Mughal Emperor abolished the 'Sijda' (a) Dadabhai Naoroji
court custom introduced by emperor Balban? (b) Bal Gangadhar Tilak
(a) Akbar (b) Jahangir (c) Gopal Krishna Gokhale
(c) Shahjahan (d) Aurangzeb (d) R.B. Ghosh
Ans. (c) : Shah Jahan was the Mughal emperor who Ans. (d) : Rash Behari Ghose was the president of the
abolished the Persian (1636-37) court custom of Sijda annual session of the Indian National Congress in 1907
which was introduced by Balban. Sijda (Prostration) at Surat. This convention was being held in Nagpur
and Paibos was a regular form of the kings salutation. earlier. Congress on the interpretation of the word
To receive the Sultan, people had to kneel in sijada and 'Swaraj', the 1907 congress session resulted in the Surat
touch the ground with their heads. split which resulted in formation of two groups, the
11. Which of the following statements regarding moderates and extremists.
Mohd. Ali Jinnah is not correct? 16. Who had advocated in favour of the Indian
(a) He was supporter of two nations theory. National Army (Azad Hind Fauj) officers in
their trial being conducted at the Red Fort?
(b) He presided over the annual session of
(a) C.R. Das (b) Motilal Nehru
Muslim league at Lahore in 1940.
(c) M.A. Jinnah (d) Sir T.B. Sapru
(c) He did not participate in the Non-Cooperation
Movement. Ans. (d) : Three Indian Nation Army soldiers
(d) He always used to keep a Diary. Shahnawaz Khan, Gurbaksh Singh Dhillon and Prem
Kumar Sehgal were accused of treason against the country
Ans. (d) : He always used to keep a diary, is false during the World War-II. Bhulabhai Desai was part of the
statements related to Jinnah. Muhammad Ali Jinnah was 17 member defense team of three INA soldiers. The trial
responsible for the partition of country on the basis of began in October, 1945 at Red Fort in Delhi. Bhulabhai
Hindus and Muslims. Desai headed the group of advocates Jawaharlal Nehru,
12. The slogan of 'Inquilab Zindabad' is associated Tej Bahadur Sapru to argue the case on behalf of the
with? Indian National Army in 1946 in Red Fort trials.
(a) Chandrasekhar Azad (b) Ram Prasad Bismil 17. On which of the following dates was christened
(c) Bhagat Singh (d) Lala Hardayal as "Direct Action Day" by the Muslim league?
(a) 15th Aug, 1946 (b) 16th Aug, 1946
Ans. (c) : The famous slogan 'Inquilab Zindabad' was th
given by Maulana Hasrat Mahani in 1921. The slogan (c) 17 Aug, 1946 (d) 18th Aug, 1946
th
emerged as one of the famous slogans during the Ans. (b) : On 16 August 1946, the League started
freedom struggle in India. It was popularized by Bhagat "Direct Action Day". The purpose of this action of the
Singh during the late 1920 through his speeches and League was to spread communal riots and create an
writings. It was used as the official slogan for the atmosphere of terror to prove that Hindu-Muslims could
Hindustan Socialist Republican Association. not live together.
UP Ro/ARO (Pre) Exam 2010 218 YCT
CLICK HERE FOR FREE MATERIAL

18. In which of the following sessions of Indian Ans. (d) : Foehn is a local wind of Switzerland. Foehn
National Congress, Subhas Chandra Bose is a type of dry, relatively warm, down slope wind that
defeated 'Pattabhi Sitarmayya' in the election occurs in the lee ward side (downward side) of a
to the post congress president? mountain range. Foehn blows through Switzerland,
(a) Haripura Session, 1938 Southern Germany and Western Austria and crosses the
(b) Tripuri Session, 1939 Alps mountains and reaches Northern Italy.
(c) Lahore Session, 1929 22. Which one among he following is not correctly
(d) Madras Session, 1929 matched?
Ans. (b) : In the annual session of Tripuri Congress in (a) Nathula - Arunachal Pradesh
1939, Subhash Chandra Bose became the candidate for (b) Lipulekh - Uttarakhand
the post of president for the second time, he was elected (c) Rohtang - Himanchal Pradesh
president for second time by defeating Pattabhi (d) Palghat - Kerala
Sitaramyya, the candidate of Gandhiji choice. Gandhiji Ans. (a) : The correct match as follows :-
described the defeat of Pattabhi Sitaramyya as his (a) Nathula Pass - Sikkim
defeat. After the election, by the resolution passed in the (b) Lipulekh Pass - Uttarakhand
session, the chairman was appointed as a member of the (c) Rohtang Pass - Himanchal Pradesh
working committee as desired by Gandhiji. But Gandhiji (d) Palghat Gap - Kerala
refused to suggest any name. Finally, in April, 1939 23. Statement (A) The rivers of Western Ghats do
Subhas Chandra Bose resigned from the post of Congress not form Deltas.
president. Dr. Rajendra Prasad became the new congress Reason (R) : They flow swiftly over the hard
president in place of Subhash Chandra Bose. rocks though narrow routes.
19. Which of the following pairs is correctly (a) Both (A) and (R) are true and (R) is the
matched? correct explanation of (A)
(a) Lala Lajpat Rai : Poverty and Unbritish Rule (b) Both (A) & (R) are true but (R) is not the
in India correct explanation of (A)
(b) Dadabhai Naoroji : Unhappy India (c) (A) is true but (R) is false
(c) Rafiq Zakaria : The man who divided India (d) (A) is false but (R) is true
(d) Subhash Chandra Bose : Guilty man of India's Ans. (a) : The major rivers of peninsular India originate
Partition from the Western Ghats. The rivers flowing towards the
Ans. (c) : The correct match is as follows :- east follow a longer course and form a delta. Whereas
(a) Lala Lajpat Rai - Unhappy India the rivers of the Western Ghat flow through hard rocks
and follow a shorter course. These rivers would not
(b) Dadabhai Naoroji - Poverty and un-British have been able to form deltas in this way. Hence, option
rule in India (a) both assertion and reason are correct and reason is
(c) Rafiq Zakaria - The Man who divided the correct explanation for assertion.
India 24. Which is the youngest Mountain Ranges among
(d) Ram Manohar - Guilty Men of India's the following?
Lohia Partition (a) Vindhya (b) Aravali
20. Who presided over the Lahore Session of the (c) Shivalik (d) Annamalai
Indian National Congress in 1932? Ans. (c) : Shivalik range is the newest mountain range
(a) Motilal Nehru (b) C.R. Das of Himalayas, which starts from Dhauladhar in the
(c) Jawaharlal Nehru (d) S.C. Bose south and extends to Kangra in the north. It is average
Ans. (c) : Jawaharlal Nehru presided over the Lahore altitude ranges from 1500 to 2000 m. It is originated in
session of the Indian National Congress in 1929, passed the Jurassic period. It is the highest mountain among the
the historic 'Purna Swaraj' - (total independence) newly folded mountains. The Shivalik Hills are also
resolution. Jawaharlal Nehru was selected as President called the Sub-Himalayan Range, the Shivalik Range or
due to following major decisions :- the outer Himalayas.
Aravali is the oldest mountain range in India, running
• The report of Nehru Committee was declared across the states of Gujarat, Rajasthan and Haryana
cancelled. ending in Delhi.
• The Round Table Conference was to be boycotted. 25. Epiphytes are mainly found in?
• Complete independence was declared as the aim of (a) Coniferous Forests (b) Monsoonal Forests
the Congress. (c) Savana Forests (d) Equatorial Forests
• Civil Disobedience Movement for complete Ans. (d) : Epiphytes are plants that grow on another
independence to be launched. living plant. They make their own food because they
contain chlorophyll. They depend only on the host
• Congress decided to observe 26th January as the total plants for mechanical support. They are also called 'air
independence or Purna Sawaraj Day. plant' because their roots are not attached to the soil.
21. 'Foehn' is a local wind of? They are mainly found in equatorial forests. Some of
(a) China (b) Korea the better-known epiphytes include ferns, lichens,
(c) Japan (d) Switzerland mosses, cati, bromeliads and archids.
UP Ro/ARO (Pre) Exam 2010 219 YCT
CLICK HERE FOR FREE MATERIAL

26. Match List-I & List-II and choose the correct Ans. (b) : The correct match is as follows :-
answer from the codes given below- List-I List-II
List-I List-II (Substances found in Nature) (Elements)
A. Jute 1. Assam A. Diamond - Carbon
B. Tea 2. Kerala B. Marble - Calcium
C. Sand - Silicon
C. Rubber 3. West Bengal
D. Ruby - Aluminum
D. Sugarcane 4. Uttar Pradesh 30. Statement (A) : The inter state transportation
Codes: of coal is an important factor of the rail
A B C D transport of railways.
(a) 3 1 2 4 Reason (R) : The mines of Bengal-Jharkhand
(b) 4 3 1 2 are a major source of coal supplies to North
(c) 2 1 3 4 Western States.
(d) 1 2 3 4 Choose the correct answer from the codes
Ans. (a) : The correct match is as follow :- given below:
List-I - List-II (a) Both (A) & (R) are true and (R) is the correct
A. Jute - West Bengal explanation of (A)
B. Tea - Assam (b) Both (A) & (R) are true but (R) is not correct
explanation of (A)
C. Rubber - Kerala
(c) (A) is true but (R) is false
D. Sugarcane - Uttar Pradesh
(d) (A) is false but (R) is true
27. The 'One Crop' or 'Single Crop' Agricultural
practice is a characteristic of? Ans. (a) : The inter-state transport of coal was done by
railways. It caters about 85% of the outgoing transport.
(a) Commercial Crop Farming
The major coals mines of the country are the main
(b) Shifting Agriculture source of coal supply to the north western states.
(c) Subsistence Farming Major production of electricity is achieved through
(d) Bio-Agriculture thermal power plants which is around 75% of the total
Ans. (a) : Commercial crop farming in which a single power generation. Major coal producing states are
crop is grown on a large area. Cultivation of rubber, coffee, Chhattisgarh, Jharkhand, Orissa, West Bengal. Both
tea or cocoa on large estates or hill stations is done. assertion and reason are correct and reason is the correct
28. In context of Total Geographical Area, the explanation for assertion.
highest percentage of forest cover is found in? 31. In India the correct sequence of share of energy
(a) Arunachal Pradesh (b) Nagaland sources in different energy segments are-
(c) Tripura (d) Mizoram (a) Thermal > Hydro > Atomic > Wind
Ans. (d) : According to Forest Survey Report 2021, In (b) Thermal > Atomic > Hydro > Wind
terms of forest cover as percentage of total geographical (c) Hydro > Atomic > Thermal > Wind
area, the top five states are Mizoram (84.53%), (d) Atomic > Hydro > Wind > Thermal
Arunachal Pradesh (79.33%), Meghalaya (76.00%), Ans. (*) : According to Central Electricity Agency
Manipur (74.34%) and Nagaland (73.90%). (CEA) as on 30.06.2022, India total installed energy
Note :- When question out of was asked out of Total capacity are 4,03,760 MW of coal contributes (50.7%),
geographical area, the highest percentage of forest cover Hydro (11.6%), Wind, Solar & other RE (28.3%), Wind
was found in Mizoram (85.41%). (10.1%), Nuclear (1.7%). At present the correct order is
29. Match List-II (Substances found in Nature) nearest are - Thermal > Hydro > Wind > Nuclear.
and List-II (Elements) and choose the correct 32. In India two leading (foremost) states in terms
answer from the codes given below:- of coal production are-
List-I List-II (a) Jharkhand and Odissa
A. Diamond 1. Calcium (b) Jharkhand & Chattisgarh
B. Marble 2. Silicon (c) Chattisgarh and Madhya Pradesh
(d) Madhya Pradesh and Andhra Pradesh
C. Sand 3. Aluminum
Ans. (*) : The Coal resources of India are available in
D. Ruby 4. Carbon older Gondwana formations of peninsular India and
Codes: younger tertiary formations of north-eastern region. IN FY
A B C D 2020-21, Chhattisgarh registered highest coal production
(a) 1 2 3 4 of 158.409 MT, followed by Odisha 154.150 MT. Madhya
(b) 4 1 2 3 Pradesh 132.531 MT, and Jharkhand 119.296 MT,
(c) 3 1 2 4 Whereas the first three states in terms of coal reserves are
(d) 4 2 1 3 Jharkhand, Odisha, Chhattisgarh respectively.
UP Ro/ARO (Pre) Exam 2010 220 YCT
CLICK HERE FOR FREE MATERIAL

33. Match List-I with List-II and choose the 37. Which of the following countries are most
correct answer from the codes given below- affected by Acid Rain? Choose the correct
List-I List-II answer from the codes given below-
(Coal producing (Coal Mines) (1) Canada (2) Norway
Region) (3) Sweden (4) United Kingdom
A. Damodar Valley 1. Talcher Code:
(a) 1 and 2 (b) 1 and 3
B. Son Valley 2. Barakar
(c) 1, 2 and 3 (d) 2, 3 and 4
C. Godavari Valley 3. Umaria Ans. (d) : The rain in which the chemical elements
D. Mahanadi Valley 4. Singareni contained in the atmosphere or the pollutants are mixed
and which fall to the earth as a mild acidic
Codes:
concentration. Canada, Sweden and Norway are three
A B C D such countries in the world which are most affected by
(a) 2 3 4 1 acid rain. Apart from this, countries like Finland,
(b) 3 2 1 4 Netherlands, Germany, Italy are also affected by this
(c) 1 3 4 2 rain. In Asia, India and China are mostly affected
(d) 4 1 2 3 mainly because of the large number of factories.
Ans. (a) : The correct match as follow are :- 38. In Europe, what is the place of France among
List-I List-II iron ore producing nations?
(Coal producing Region) (Coal Mines) (a) First (b) Second
A. Damodar Valley 1. Barakar (c) Third (d) Fourth
Ans. (*) : According to World Steel Organization 2019,
B. Son Valley 2. Umaria the order of iron producing countries is (in million
C. Godavari Valley 3. Singareni tonnes)- Russia (71.7), Germany (42.4), Italy (24.5),
D. Mahanadi Valley 4. Talcher Ukraine (21.1), France (15.4), Spain (14.3).
Note:- The world's top five largest iron producing
34. Consider the following statements and choose countries in 2020 - Australia (900 M.T), Brazil (400 M.T),
the correct answer from the codes given below: China (340 M.T), India (230 M.T) and Russia (95 M.T).
(1) The most abundant reserves of iron ores is 39. Match List-I & List-II and choose the correct
found in Karnataka answer from the codes given below-
(2) India is the second largest producer of iron
ores in the world. List-I (Centre) List-II (Industry)
(3) In India, Odissa is the largest producer of Iron A. Nagoya 1. Iron and Steel
ores B. Salem 2. Aircraft
(a) (a) only 1 (b) Only 2 & 3
C. Los Angeles 3. Textiles
(c) 1 and 3 (d) All are true
Ans. (c) : India is the fourth largest producer of iron D. Abadan 4. Oil Refinery
ores in the world. The world's top five largest iron ore Codes:
producing countries are Australia, Brazil, China, India A B C D
and Russia respectively. The richest deposits of iron (a) 2 3 4 1
ores were found in Karnataka and Odisha the largest (b) 4 2 3 1
producer of iron ore in India. (c) 1 4 2 3
35. In India, which state has the longest length of (d) 3 1 2 4
Highways? Ans. (d) : The correct match are:-
(a) Uttar Pradesh (b) Bihar
(c) Madhya Pradesh (d) Maharashtra List-I List-II
(Centre) (Industry)
Ans. (d) : Maharashtra has the largest Highway
Network with 32000 kms. Which is nearly 18% of to A. Nagoya - Textiles
total state highway network in the country. B. Salem - Iron and Steel
36. The highest percentage of forest cover in the C. Los Angeles - Aircraft
world is related/belong to- D. Abadan - Oil Refinery
(a) Temperate zone coniferous forests
(b) Temperate zone deciduous forests 40. Match List-I & List-II and choose the correct
answer from the codes given below-
(c) Tropical zone monsoonal forest
(d) Tropical zone rainy forests List-I (Crop) List-II (Region)
Ans. (d) : The highest percentage of the worlds forest A. Rubber 1. Mauritius
cover is tropical rain forest. Tropical rain forests are B. Coffee 2. Italy
considered as one of the most complex biomes
(structure and species diversity) on Earth. It is the oldest C. Olive 3. Columbia
living ecosystem on the earth. D. Sugarcane 4. Malaysia
UP Ro/ARO (Pre) Exam 2010 221 YCT
CLICK HERE FOR FREE MATERIAL

Codes: 44. Statement (A) : There is found variation in the


A B C D population increase pattern in India's Large
(a) 1 2 3 4 Metropolitan Cities.
(b) 4 3 2 1 Reason (R) : According to Census 2001, there
are four Metropolitan Cities in India.
(c) 4 3 1 2
Choose the correct answer using the codes
(d) 3 4 2 1 given below:
Ans. (b) : The correct match are:- Codes:
List-I (Crop) List-II (Region) (a) Both (A) and (R) are true and (R) is the
A. Rubber - Malaysia correct explanation of (A)
(b) Both (A) and (R) are true but (R) is not the
B. Coffee - Columbia correct explanation of (A)
C. Olive - Italy (c) (A) is true but (R) is false
D. Sugarcane - Mauritius (d) (A) is false but (R) is true
41. According to census 2001, the ratio of Ans. (c) : Variation is found in the population growth of
population/number of female to male in Uttar major cities in the country. As of census of India 2001, the
Pradesh was: number of major cities was seven. Among the standard
population of mega cities in 2011 there are three urban
(a) 878 females per thousand males
agglomerations in India based on the standard population
(b) 918 females per thousand males of mega cities 11 crore (10 million).
(c) 898 females per thousand males Under Metropolitan Cities are :-
(d) 925 females per thousand males 1. Greater Mumbai (18.41 million), Growth Rate (12.05).
Ans. (c) : According to Census 2001, the sex ratio of 2. Delhi (16.31 million), Growth Rate (26.09).
Uttar Pradesh was 898, but it increased to 912 in 2011 3. Kolkata (14.11 million), Growth Rate (26.09)
census. Accordingly, there is a difference in the growth rate
42. Which of the following statements is not among the three. Thus Assertion (A) is correct and
correct in terms of Uttar Pradesh on the basis Reason (R) is incorrect.
of census 2001? 45. There was a prescribed aim under the National
(a) The percentage of Rural population out of Population Policy, 2000, that a stability will be
total state population is 79.21% achieved in population by the year 2045 Now
(b) The male literacy rate is 68.8% the target year for the same has been kept as?
(c) In terms of population density the state's (a) 2055 (b) 2060
position in India is twelfth. (c) 2065 (d) 2070
(d) The lowest populated district of U.P. is Ans. (d) : The National Population Policy 2000 was
Mahoba announced during the Prime Ministership of Atal
Bihari. The goal of this policy was to achieve
Ans. (c) : According to Census 2001, 20.78% of the
population stability by the year 2045, which was
state's population lives in cities and 79.2% in villages. increased to 2070 in 2010. The first population policy in
The male literacy rate in the state was 68.8% and the country was announced in 1976.
female literacy rate was 42.2%. The state 9th ranks in
terms of population density. The district with minimum 46. As per the Census 2001, what is the position of
population in the state is Mahoba. According to the Uttar Pradesh among the States/Union
current census 2011, 77.73% of the states population is Territories in literacy rate are?
in villages and 22.27% of the population lives in cities. (a) 29 (b) 30
The total literacy rate of Uttar Pradesh is 67.72%, the (c) 31 (d) 32
male literacy rate of the state is 77.28% and the female Ans. (c) : According to Census 2001, the position of
literacy rate is 57.18%. Uttar Pradesh are 31st in the State/Union Territories at
43. In India progressive population- Resource literacy rate. According to the current census 2011
regions following areas are included: statistics, Uttar Pradesh has a total literacy of 67.7%,
male literacy 77.3% and female literacy are 57.2%. The
(1) Ganga-Yamuna Doab
total literacy rate of Uttar Pradesh is 5.3% less than the
(2) Coastal Odissa national average. During the 2011-2011, the state
(3) Rajasthan literacy increased by (11.34%). Uttar Pradesh is literacy
(4) Tamil Nadu (in all states). Districts of U.P with maximum and
Choose the correct answers from the codes: minimum district with literacy are Gautam Buddha
(a) 1 and 2 (b) 1 and 3 Nagar and Shravasti respectively.
(c) 2 & 3 (d) 1 and 4 47. In South Asia, the country with highest
Ans. (d) : The Ganga Yamuna Doab and the Nilgiri percentage of Elderly/Old population is?
region of Tamil Nadu are included in the Progressive (a) Bhutan (b) India
Population Resource Area of India. (c) Nepal (d) Sri Lanka
UP Ro/ARO (Pre) Exam 2010 222 YCT
CLICK HERE FOR FREE MATERIAL

Ans. (d) : Asia and Europe are home to some of the Ans. (c) : The lowest gender-ratio in Daman & diu
world's oldest populations, those aged 65 and above at the Union Territory Sex Ratio
top is Japan at 28%. In South Asia, Sri Lanka with the (as per 2011 Census)
highest 7.8% of total population of above 65 year. About Chandigarh - 818
6.1% of India's population is 65 year of age or above. Dadra and Nagar Haveli - 774
48. Nearly 80% population of the world is not Daman and diu - 618
protected by- 52. Which of the following statements are true
(a) Economic Security (b) Food Security 1. Tamil Nadu registered maximum increase in
(c) Infant Security (d) Social Security the urbanization ratio during the 1961-2001
period.
Ans. (d) : Nearly 80% population of the world is not
protected by social security. 2. In India, the average size of an urbanized
centre is 45000.
49. Which is the correct descending order of 3. The largest number of urbanized centers come
urbanization percentage of following nations of in the fourth category.
South Asia? 4. In the fifth and sixth category cities, more
(1) Bangladesh than 5% of the total urban population of
(2) India country resides.
(3) Pakistan Choose the correct answer from the codes
(4) Sri Lanka given below:
Choose the correct answer from the codes (a) 1 and 2 (b) 2, 3 and 4
given below: (c) 1 and 3 (d) 1, 2 & 3
(a) 2, 3, 1 and 4 (b) 3, 2, 1 and 4 Ans. (a) : On the basis of urban population data in India,
(c) 2, 1, 4 and 3 (d) 4, 2, 3 & 1 Tamil Nadu recorded the highest growth in the
urbanization ratio in the period 1961-2001. Average size of
Ans. (b) : According to the United Nations World
an urbanization in India, minimum population is 5000.
Urbanization Prospects Report 2018, Pakistan's urban
population are 36.67%, Bangladesh's urban population Urban Class Population size
are 36.63%, India's urban population are 34.03% and I - Greater than 1 lakh
Sri Lanka's is 18.48%. Therefore, according to the II - 50,000 to 99,999
condition of urban population, the descending order of III - 20,000 to 49,999
the above countries - Pakistan, Bangladesh, India, Sri IV - 10,000 to 19,999
Lanka. Option (b) is the correct answer during question V - 5000 to 9,999
was asked. VI - less than 5000
50. Statement (A): In India the level of 53. Which of the following statements is/are true?
urbanization is much lower than China 1. Generally an unbalanced gender ratio is found
Reason (R) : Indian Cities are less planned. in India's northern cities.
Chose the correct answer from the codes given 2. In contrast to centralization of western cities.
below: 3. In South, due to low number of female
(a) Both (A) and (R) are true and (R) is the Christians in the labour force and high
correct explanation of (A) literacy rate wider participation of women.
(b) Both (A) & (R) are true but (R) is not the 4. In the western cities of India, there is lower
correct explanation of (A) migration from rural regions/areas.
(c) (A) is true but (R) is false Choose the correct answer from the codes
given below:
(d) (A) is false but (R) is true
(a) 1, 2 & 3 (b) 2, 3 & 4
Ans. (b) : According to the data of the United Nations (c) 1, 2 & 4 (d) 1, 3& 4
'World Urbanization Prospects Report'-2018, the
Urbanization in China are 59.15%, while the Ans. (c) : Cities in the northern states of India
(Haryana, Punjab, Himachal Pradesh) generally have a
Urbanization of India are 34.03%, so the percentage of
imbalanced sex-ratio, while the western cities (Mumbai,
Urbanization in India are much lower than China.
Ahmedabad, Gandhinagar, Pune) are concentrated and the
Indian cities are less planned. Hence, both the
cities of eastern India are scattered. In India there are less
statements (A) and (R) are true but (R) is not the correct migration from rural areas in western cities as compared to
explanation of (A). eastern cities, whereas statement (3) is not correct.
51. In India, the lowest Gender Ratio is found in 54. Which of the following proposed the idea of
the Union Territory of- constituting a constituent Assembly for India?
(a) Chandigarh (a) Simon Commission
(b) Dadra and Nagar Haveli (b) Rajaji Formula
(c) Daman & Div (c) Cabinet Mission Plan
(d) None of these (d) Wavell Plan
UP Ro/ARO (Pre) Exam 2010 223 YCT
CLICK HERE FOR FREE MATERIAL

Ans. (c) : In January 1946, Clement Attlee, the leader 59. Which of the following subjects has not been
of Britain's Labour Party, decided to send a included in the provisions of Money bill?
parliamentary party (Cabinet Mission) to India to hold (a) Provisions regarding/relating to taxes
informal talks with Indian leaders. Announcing the (b) Provisions relating to credit sanction
Cabinet Mission in India, Lord Penthick Lawrence said provisions
that its objective was to quickly prepare a methodology for (c) The consolidated funds and custodianship
the preparation of the constitution for India and to make
necessary arrangements for the Interim Government. (d) Provisions related to penalty & fine
55. Which of the following Acts provided a system Ans. (d) : The provision relating to penalty or fine has
of federal-rule in India? not been included in the provision of money bill. Article
(a) The Government of India Act, 1909 110 of the Indian Constitution lays the definition of
(b) The Government of India Act, 1919 money bill. According to it, the bills pertaining to
(c) The Government of India Act, 1935 following matters are deemed to be money bills.
(d) The Government of India Act, 1947 • Related to imposition, abolition or remission of taxes.
Ans. (c) : The first federal government was established • Borrowing by the government.
in India by the Government of India Act, 1935. By the
Act of 1935, the diarchy in the provinces was abolished • Matters pertaining to custody, withdrawal/
and the diarchy was implemented at the center. The appropriation from, payments to the consolidated or
executive power of the central government was vested contingency fund of India.
in the Governor-General. • Money Bill can be introduced only in Lok Sabha and
56. The idea/concept of Fundamental Rights in not in Rajya Sabha and that too on the
Indian constitution has been borrowed from- recommendation of the President.
(a) British Constitution 60. Indian Vice-President-
(b) Irish Constitution 1. Is India's second highest prestigious office
(c) Canadian Constitution bearer.
(d) American Constitution 2. Does not have any informal work
Ans. (d) : Fundamental Rights, the system of the Supreme (responsibility) related to the post.
Court, the office of the Vice President and the functions 3. Transacts all the functions of president in his
assigned to him has been included in the Indian absence.
Constitution from the U.S. Constitution. The independence 4. Works in the capacity of President in lieu of
of the judiciary, the power of judicial review, the removal
of jades, etc. have the effect of the U.S. Constitution. resignation of president, dismissal or death.
Choose the correct answer from the codes
57. MNREGA programme has been introduced to
given below:
implement which Article of Indian
Constitution? (a) 1 and 2 (b) 1, 2 & 3
(a) Article 43 (b) Article 45 (c) 1, 3 & 4 (d) All of them
(c) Article 47 (d) Article 50 Ans. (d) : The Vice-President is the highest office after
Ans. (a) : MNREGA program has been implemented the president. The general function of the Vice-
under Article 43. The provision has been made in President is to be the ex-officio chairman of the Rajya
Article 43 of the Directive Principles of State Policy, Sabha. According to Article-64, the Vice-President shall
that the state by suitable legislation or in any other manner be the ex-officio Chairman of the Rajya Sabha and shall
provide for work, subsistence wages, a decent standard of not hold any other office of profit. Provided that during
living and leisure to all workers or agriculture of industry any period during which the Vice-President acts as
or of any other nature. Working conditions and social President or discharges the functions of the President
ensuring total consumption and shall endeavour to provide under article 65, he shall not perform the duties of the
cultural opportunities and in particular. Efforts will be office of the chairman of the Rajya Sabha under article
made to increase the cottage industries in the villages on 97 and shall not be entitled to the salary or allowances
individual or cooperatives basis. payable to the chairman.
58. Which one out of the following is not a 61. Which one among the following is not included
directive principle of state policy?
in the context of Republican Concept?
(a) Prohibition
(a) A state in which people are Supreme
(b) Cow-protection
(c) Environmental protection (b) The highest authority is vested in the
(d) Free education to children below 14 yrs. age sovereign elect
Ans. (d) : The Right to Free Education for children (c) The highest authority is vested in a single
from the age of 6-14 years has been included in the person just as a monarch
Fundamental Right under Article 21(A) by the 86th (d) Such a government which comprises of
Constitutional Amendment Act, 2002. Whereas Alcohol elected representatives
Prohibition (Article-47), Cow-Protection (Article-48), Ans. (c) : Under the Republican concept of India, the
Environment Protection (Article-48A) was included supreme power of the state is vested in the people and
under the Directive Principles of State Policy. the supreme power is delegated to the elected head, so
UP Ro/ARO (Pre) Exam 2010 224 YCT
CLICK HERE FOR FREE MATERIAL

the republican government of India is formed by the 66. In India the largest average size of operand
majority of the representatives elected by the people. land holding is-
Therefore, the supreme power of the state is not vested (a) In Punjab (b) In Gujarat
in one person like a king, but arrangements have been (c) In Uttar Pradesh (d) In Rajasthan
made for the separation of powers, in which the Ans. (c) : Uttar Pradesh has the largest size of operand
legislature, executive and judiciary of jurisdiction land holding in India followed by Bihar, Maharashtra,
properly exercise the power of their area as prescribed Madhya Pradesh, Karnataka, Andhra Pradesh, Tamil
by the constitution. Nadu, Rajasthan.
62. In the parliamentary dignitaries is included 67. Assertion (A) : From October 2009 to February
1. Speaker of Lok Sabha 2010, the Inflation Rate in India became
2. Deputy Speaker Lok Sabha negative.
3. General Secretary Lok Sabha Reason (R) In the global market the price of
4. Chairman, Rajya Sabha petrol slumped to 30 dollars a barrel from 140
Choose the correct answer using the codes dollars per barrel.
given below: Choose the correct answer from the codes
(a) 1 & 2 (b) 1, 2 & 3 given below:
(c) 1, 3 & 4 (d) All four (a) Both (A) & (R) are true and (R) is the correct
Ans. (d) : The dignitaries of Parliament include all of explanation of (A)
the above i.e. Speaker of Lok Sabha, Secretary General (b) Both (A) and (R) are true but (R) is not the
of Lok Sabha and Speaker of Rajya Sabha. correct explanation of (A)
63. In India an Act prohibiting bonded labour was (c) (A) is correct but (R) is false
passed by the Parliament in the year? (d) (A) is false but (R) is true
(a) 1974 (b) 1976 Ans. (a) : Inflation rate in India turned negative from
(c) 1979 (d) 1980 October 2009 to February 2010 because the price of
Ans. (b) : The Bonded Labour System (Abolition) Act petrol in the global market was reduced from $140 per
was passed by the Parliament of India in the 1976. The barrel to $30 per barrel. Therefore, Both (A) & (R) are
Bonded Labour System (Abolition) Act, 1976 has been true and (R) is the correct explanation of (A).
passed to provide for the abolition of bonded labour 68. In the year 2007-08, which of the following
system with a view to preventing the economic and nations had the largest share in Indian
physical exploitation of the weaker sections of the people imports?
and for matters connected therewith or incidental thereto.
(a) U.S.A. (b) Saudi Arabia
64. Which one among the following is included in (c) China (d) Switzerland
the 73rd Constitution Amendment of the Indian
Constitution? Ans. (c) : India exported about $422 billion
1. District Panchayat merchandise in the financial year 2021-2022. India
2. Kshetra Panchayat largest trade partners with their total trade (sum of
imports and exports) in billion of US dollars for the
3. Village Panchayat financial year 2021-22 were as follows :-
4. Nagar Panchayat
Choose the correct answer from the codes Rank Country Exports Imports Total
given below: Trade
(a) 1 and 2 (b) 1, 2 and 3 1. United States 76.11 43.31 119.42
(c) 1, 3 and 4 (d) All four 2. China 21.25 94.16 115.41
Ans. (b) : District Panchayat, Kshetra Panchayat and 3. U.A.E. 28.10 44.80 72.90
Village Panchayat are included in the 73rd Constitution 4. Saudi Arabia 8.00 34.00 42.00
Amendment Act, 1992 of Indian Constitution. The 73rd 5. Switzerland 1.21 16.90 18.11
amendment 1992 added a new Part-IX to the constitution
69. In which of the following sectors India had
titled "The Panchayats" covering provision from Article
largest area of imports business in 2009-10?
243 to 243 (O); and a new Eleventh Schedule covering 29
subjects within the function of the Panchayats. (a) Asia and ASEAN
(b) Europe
65. The Swadhar Scheme is-
(a) Meant for establishing unique identity (c) North America and Canada
(b) Self employment for males (d) South America and Africa
(c) A scheme for women in difficult circumstances Ans. (a) : According to the Data, Asia and ASEAN
(d) A provision of shared homes for senior citizens were the largest regions of India's imports in the year
Ans. (c) : The Swadhar scheme was launched by the 2018. Participation in overall imports Asia and Asean
Union Ministry of Women and Child Development in (60.3%), Europe (15.8%), Africa (8.2%) and North
2002 for rehabilitation of women in difficult America (8.1%).
circumstances. The scheme provides shelter, food, 70. 'Light Year' is________
clothing and care to the marginalized women/girls who (a) An year, in which February month has 29
are in need. days
UP Ro/ARO (Pre) Exam 2010 225 YCT
CLICK HERE FOR FREE MATERIAL

(b) The distance which light covers in one year Ans. (a) : Many times farmers harvest fruits and
(c) The time, which the sun rays take to reach the vegetables much before they get ripe and traders then
earth ripen them artificially at the destination using certain
(d) The time, which a space shuttle takes to travel chemicals. The chemical name of ethylene is ethephon.
from Earth to Moon It penetrates into the fruit and decomposes ethylene. It
Ans. (b) : Light year is the distance that light travels in is used for ripening of fruits.
one year. Light year is used to measure very long 77. Biogas is mainly Composition of?
distances i.e. light year is the unit of distance. One light (a) CO2 and hydrogen
year equal to 9.46×1015 m. (b) Hydrogen and Methane
71. From the invisible part of Sunlight, the process (c) CO2 and Methane
of photosynthesis is performed by- (d) Hydrogen and Oxygen
(a) Some Trees (b) Some Fungi Ans. (c) : Biogas is produced after organic materials
(c) Molds (d) Bacteria (plant and animal products) are broken down by
Ans. (d) : Some bacteria are also called cyanobacteria. bacteria in an oxygen - free environment, a process
They perform the process of photosynthesis with the called anaerobic digestion. Biogas systems use
invisible part of sunlight. anaerobic digestion to recycle these organic materials,
turning them into biogas, which contains both energy
72. Which unit of measurement when multiplied (gas) and valuable soil products (liquids and solids).
by 0.39 gives one inch? Biogas contains roughly 50-70% methane, 30-40%
(a) Millimetre (b) Centimetre carbon dioxide and trace amounts of other gases.
(c) Metre (d) Decimetre
78. Acid Rain is caused due to air pollution by-
Ans. (b) : Multiplying 0.3937 in centimeters gives 'inch' (a) Carbon dioxide (CO2)
and one meter = 39.37 inches. (b) Carbon Monoxide (CO)
73. An optical fibre works on the principle/theory (c) Methane (CH4)
of? (d) NO and SO2
(a) Total Internal Reflection (b) Refraction
Ans. (d) : Acid rain is caused mainly due to the
(c) Scattering (d) Interference emission of sulfur dioxide (SO2) and nitrogen oxides
Ans. (a) : Optical fibre is cable by which a light signal can (NO) which combine with water to produce sulfuric
be communicated from one place to the other place without acid and nitric acid respectively.
any loss of energy or intensity. The working principle of an 79. Match List-I with List-II and choose the
optical fibre is based on total internal reflection. correct answer using the codes given below-
74. As soon as a train starts, moving, the head of a List-I List-II
person sitting in that transcends backwards. (Harmone) (Secretion Gland)
The cause of it is-
(a) Inertia of stability A. Progesterone 1. Thyroid
(b) Inertia of motion B. Testosterone 2. Pancreas
(c) Moment of Inertia C. Thyroxin 3. Uterus (women)
(d) Conservation of mass D. Insulin 4. Testicles (men)
Ans. (a) : According to the law of inertia of stability, if
Codes:
an object is at rest or moves with uniform velocity in a
straight line, then its state of rest or uniform motion A B C D
changes only when an external force is applied on it. (a) 3 4 1 2
Hence, when the train start moving, the man is still at (b) 4 3 1 2
rest and hence appears to move backwards with respect (c) 3 4 2 1
to the train. This is called as Inertia of rest. (d) 1 2 3 4
75. In the Remote control of Television Receiver, Ans. (a) : The correct match as follows are :-
what type of electromagnetic radiation is List-I List-II
employed? (Harmone) (Secretion Gland)
(a) Visible (b) Ultra Violet A. Progesterone Uterus (Women)
(c) Infrared (d) None of them B. Testosterone Testicles (Men)
Ans. (c) : TV remote controls using a type of light called C. Thyroxin Thyroid
infrared (or IR for short). The remote control has an LED D. Insulin Pancreas
light in it which flashes really quickly to emit a messages 80. Consider the following statements:
which is then picked up by the TV. The remote is called Statement (A) : In India both the Rich and the
the transmitter and the TV is called the receiver. Poor are malnourished
76. Which of he following chemicals is used for the Reason (R) : The Rich eats wrong food where
repening of fruits? as the Poor eat dry food.
(a) Ethephon (b) Milathion Choose the right answer from the codes given
(c) Isoprotane (d) Atrazine below-
UP Ro/ARO (Pre) Exam 2010 226 YCT
CLICK HERE FOR FREE MATERIAL

(a) Both (A) & (R) are true and (R) is the correct Codes:
explanation of (A) A B C D
(b) Both (A) & (R) are true but (R) is not the (a) 3 2 4 1
right explanation (A) (b) 1 2 3 4
(c) (A) is true but (R) is false (c) 2 4 1 3
(d) (A) is false but (R) is true
(d) 3 1 2 4
Ans. (a) : In India, both rich and poor are malnourished
because rich take wrong food for example- fast food, Ans. (b) : The correct match are:-
junk food etc., due to which they suffer from many List-I List-II
diseases like high blood pressure, diabetes etc. and poor (Agencies) (Location)
eat dry food due to which they are malnourished A. ISRO - Bengaluru
because they do not get proper amount of vitamins,
proteins, carbohydrates, fats etc. Hence both the B. IUCAA - Pune
statements (A) and (R) are correct and (R) is the correct C. IUAC - New-Delhi
explanation of (A).
D. VSSC - Thiruvananthapuram
81. Match List-I with List-II and choose the
correct answer from the codes given below- 84. The most permanent ecosystem among the
following is-
List-I List-II
(Disease) (Cause) (a) Desert (b) Mountaineous
(c) Ocean (d) Forest
A. Night Blindness 1. Vitamin-D
Ans. (c) : The main source of energy for the ecosystem
B. Rickets 2. Vitamin-C is the sun and the solar radiation from it. Solar energy is
C. Scurvy 3. Vitamin-B also directly and indirectly helpful in the transmission,
D. Beri-Beri 4. Vitamin-A recycling of substances in the biosphere ecosystem. The
most stable ecosystem is the ocean region.
Codes:
A B C D 85. India's Antarctic and Ocean Research Centre
(a) 1 2 3 4 is located at?
(b) 3 4 1 2 (a) Dakshin (b) Gangotri
(c) 4 1 2 3 (c) Goa (d) Maitri
(d) 2 3 4 1 Ans. (c) : The National Centre for Polar and Ocean
Ans. (c) : The correct match are:- Research (NCPOR) formally known as the National
Centre for Antarctic and Ocean Research (NCAOR) is
List-I List-II an Indian research and development institution situated
(Disease) (Cause) in Vascoda Gama, Goa.
A. Night Blindness - Vitamin-A
86. "Green House" Effect mainly occurs due to-
B. Rickets - Vitamin-D (a) Release of CO2 in Atmosphere
C. Scurvy - Vitamin-C (b) Infrared Rays
D. Beri-Beri - Vitamin-B (c) Humidity in Atmosphere
(d) Ozone layer
82. Seed examination is basically done to
determine/measure- Ans. (a) : The greenhouse effect is the way in which
(a) The purity heat is trapped close to Earth's surface by "greenhouse
(b) Germination power gases". These heat-trapping gases can be thought of as a
blanket wrapped around earth, keeping the planet
(c) The quality of moisture
warmer than it would be without them. The greenhouse
(d) All the above effect is caused due to the interaction of the Sun's
Ans. (d) : Seed testing is done to determine the energy with greenhouses gases such as methane, carbon
standards of seed lot viz., physical purity, moisture, dioxide, nitrous oxide and fluorinated gases in the
germination and ODV and thereby enabling the farming Earth's atmosphere.
community to get quality seeds.
87. A periodical and sequential change in the
83. Match List-I & List-II and choose the right ecological mix of a region is known as-
answer using the codes given below: (a) Biome (b) Succession
List-I List-II (c) Trophic level (d) Extreme
(Agencies) (Location) Ans. (c) : Due to the change in the environment new
A. ISRO 1. Bengaluru organisms come in the place of the earlier organisms
B. IUCAA 2. Pune and a new community is established, it is called
ecological succession. Succession is a long-term
C. IUAC 3. New-Delhi process in the final stage of which a permanent or
D. VSSC 4. Thiruvananthapuram extreme community is established.
UP Ro/ARO (Pre) Exam 2010 227 YCT
CLICK HERE FOR FREE MATERIAL

88. Among the following districts of U.P., which Ans. (d) : The correct match as follows are :-
district has the largest Area?
(a) Lakhimpur Kheri (b) Sonbhadra List-I List-II
(c) Sitapur (d) Hardoi (Industry) (Centre)
Ans. (a) : Lakhimpur Kheri is the largest district in A. Wooden crafts/toys - Varanasi
Uttar Pradesh. The area of Lakhimpur Kheri is 7,680 B. Sports goods - Meerut
km2. While Sonbhadra has an area of 6788 km2. Whose
place is second in Uttar Pradesh. Sitapur area is 5743 C. Brass statues - Mathura
km2 and Hardoi area is 5989 is km2. D. Match Industry - Bareilly
89. Uttar Pradesh is not the largest producer of 93. Dr. Ambedkar Institute of Technology for
which of the following produce among the Handicap is located at-
Indian states? (a) Agra (b) Allahabad
(a) Flowers (b) Wheat (c) Kanpur (d) Lucknow
(c) Potato (d) Sugarcane
Ans. (c) : Dr. Ambedkar Institute of Technology for
Ans. (a) : Uttar Pradesh is a prosperous state in terms of
Handicapped (A.I.T.H) was established in 1997 at
agriculture and animal husbandry due to favorable
geographical structures. Uttar Pradesh is the leading Kanpur, Uttar Pradesh, India by Government of Uttar
state in Wheat, Potato and Sugarcane production in Pradesh under World Bank assisted project through
India among the states. Floriculture is now Technical Education Program.
commercially practical in several states with highest 94. The oldest University in Uttar Pradesh is-
area in states such as Kerala (16.56%), Tamil Nadu (a) B.H.U. Varanasi
(13.3%), Karnataka (11.4%), Madhya Pradesh (11.1%) (b) Allahabad University, Allahabad
and Uttar Pradesh (7%).
(c) Chaudhari Charan Singh University, Meerut
90. In Uttar Pradesh, Nuclear Power Project is (d) Lucknow University, Lucknow
located at which of the following place?
(a) Obra (b) Panki Ans. (b) : The University of Allahabad is one of the
(c) Narora (d) Harduaganj oldest University of Uttar Pradesh. It was established on
Ans. (c) : Narora Atomic Power Station (NAPS) is a 23rd September 1887. It is the fourth oldest University
nuclear power plant located in Narora, Dabai Tahsil, of India after Calcutta, Bombay and Madras University.
Bulandshahar District in Uttar Pradesh. The plant 95. In which state of India, the population of
houses two reactors each a Pressurized Heavy-Water scheduled castes is maximum?
Reactor (PHWR) capable of producing 220 MW of (a) Bihar (b) Tamil Nadu
electricity. Commercial operations of NAPS-1 began on (c) Uttar Pradesh (d) West Bengal
January 1991, NAPS-2 on 1st July 1992.
Ans. (c) : According to the Census of 2011, the
91. Which of the following is not correctly
population of Scheduled Caste in the Uttar Pradesh is
matched?
4,13,57,608, which is 20.7% of the total population of
(a) Deva-Barabanki (b) Alha - Mahoba
the state and Scheduled Caste total population of the
(c) Karma - Mathura (d) Kajri - Mirzapur
whole country is 16.6%. Uttar Pradesh is the largest
Ans. (c) : The correct match as follow are - Scheduled Castes state in the country in terms of
Karma folk dance - Sonbhadra population. Whereas in other states the population of
Deva Mela - Barabanki scheduled castes are as follows :-
Alha folk song - Mahoba Bihar (1,65,67,325), Tamil Nadu (1,44,38,445), West
Kajri folk song - Mirzapur Bengal (2,14,63,270).
92. Match List-I & List-II and choose the correct
96. Rampura, which became the first village to
answer using the codes given below:
establish its own solar plant in India is located
List-I List-II
in?
(Industry) (Centre)
(a) Madhya Pradesh (b) Himachal Pradesh
A. Wooden 1. Meerut
(c) Andhra Pradesh (d) Uttar Pradesh
crafts/toys
B. Sports goods 2. Bareilly Ans. (d) : Rampura, which is the first village in India to
have its own solar power plant, is located in the
C. Brass statues 3. Varanasi Bundelkhand region of Uttar Pradesh. Its geographical
D. Match Industry 4. Mathura position is such that here the rays of the sun fall
Codes: vertically throughout the year.
A B C D 97. Which number will come at the end of the
(a) 1 4 3 2 number series?
(b) 3 2 1 4 85, 77, 70, 64, 59, 55_____?
(c) 2 1 4 3 (a) 53 (b) 51
(d) 3 1 4 2 (c) 50 (d) 52
UP Ro/ARO (Pre) Exam 2010 228 YCT
CLICK HERE FOR FREE MATERIAL

Ans. (d) :
(a) (b)

Hence, ? = 52
98. If KAMAL word is substituted by 21413 in a (c) (d)
code language, then how will the word
MAHAL be written in the same code language?
(a) 48113 (b) 43811 Ans. (d) : As the shape is related to shape
(c) 41813 (d) 38141
Ans. (c) : Just as,

Similarly
= 21413
same as, Shape is related

to

= 41813 102. In the series IKJ, NPO, SUT_______what will


come in place of the blank space________?
99. BL DG EI MO QS
(a) WXY (b) VXW
12 9 12 26 X (c) XYZ (d) XZY
What is the value of X in the above table Ans. (d) :
(a) 32 (b) 34
(c) 36 (d) 38
Ans. (b) : In the given series the digits of the letters are
reduced to two by adding them together,
XZY come in the place of blank space.
BL = 2+12 = 14–2 = 12
DG = 4+7 = 11–2 = 9 103. Which of the following qualities is different
(inconsistent) from the rest?
EI = 5+9 = 14–2 = 12
(a) Bravery (b) Loyalty
MO = 13+15 = 28–2 = 26
(c) Honesty (d) Cleverness
QS = 17+19 = 36–2 = 34
Ans. (d) : In the given options Loyalty, Bravery,
100. 82 94 44 Honesty is expressing a positive emotion while
16 36 16 cleverness is expressing a negative emotion.
4 6 4 Hence, 'd' is correct option.
10 X 8 104. A student travels on foot upto a distance of 1
(one) km. straight in East direction from his
What is the value of X in the given figure- schools entrance gate and after turning at a 90º
(a) 19 (b) 18 angle covers another one (1) km distance after
(c) 15 (d) 13 turning to left and after turning left again,
Ans. (d) : In given figure, 13 will come in the place of x stops at a further one km distance and stands
it order will be as follows- over there. He is now standing in which
82 = 8+2 = 10 direction from his starting (departure) point?
(a) West-South (b) North
94 = 9+4 = 13
(c) North East (d) South North
44 = 4+4 = 8
The sum at the digit at the first column is written the Ans. (b) : According to the question
last column.

101. Just as the figure is related to figure

likewise figure is related to

which of the following figure? Now the student is facing north from his starting point.

UP Ro/ARO (Pre) Exam 2010 229 YCT


CLICK HERE FOR FREE MATERIAL

105. In the following number series which number Ans. (d) : Let the number is x
will come in the end? Then-
13, 23, 34, 46, 59, 73_______? 3x 2x
(a) 79 (b) 92 + = 370
8 7
(c) 98 (d) 88
21x + 16x
Ans. (d) : = 370
56
37x
= 370
? = 88 56
x = 56×10
106. A is 5 years younger to B, B is 3 times bigger
(Elder) to C. If C was 10 years old, four years x = 560
ago then what will be the age of A after 6 110. The irrational number in the following is-
years? 1
(a) 9 (b)
(a) 35 (b) 38 9
(c) 43 (d) 45 (c) 0.9 (d) .09
Ans. (c) :
A B C Ans. (c) : 0.9 cannot be written in p/q
↓ ↓ ↓ format so it is irrational number all other options are
rational number.
37 42 10 year (4 year before)
So present age of C = 14 years 111. The total number of Prime Factors of
12 15
Then after 6 year the age of A will be = 37+6 1 3
 6  × 8 ×  4  will be?
25
= 43 years    
107. If the surface area of a cube is 216 sq. meters. (a) 33 (b) 37
Then what will be its volume? (c) 52 (d) None of them
(a) 512 cu.mtr. (b) 480 cu.mtr. Ans. (d) :
(c) 216 cu.mtr. (d) 100 cu.mtr. 12 15
Ans. (c) : Surface area of cube (6a2) = 216 1  3
=   × 825 ×  
a = 6 cm 6 4
3
Then the volume of cube = a ( 3)
15
1
= (6) 3 = × ( )
2 3 25
×
(3 × 2)12 ( 22 )
15
= 216
108. A sum of Rs. 2000 was invested at a rate of 1 315
10% interest. The compound interest was = × 2 75
×
( 3) × ( 2 )
12 12
230
calculated at an interval of every six months.
What will be the net Amount after the end of = 275–30–12 × 315–12
one year? = (2)33 × (3)3
(a) Rs. 2200 (b) Rs. 2220 So, the numbers of prime factor = 33+3 = 36
(c) Rs. 2210 (d) Rs. 2205 112. Two numbers are in the ratio of 5 : 6 and their
Ans. (d) : Given- L.C.M. is 660
R = 10% (p.a.) (a) 22 (b) 165
So rate will be 5% half yearly (c) 132 (d) 110
P = `2000 Ans. (d) : Let the small number is 5x
T And the bigger number = 6x
 R 
A = P 1 +  LCM of both = 30x
 100  Then, 30x = 660
2
 5  660
= 2000  1 +  x=
 100  30
105 105 x = 22
= 2000 × × Small number = 5×2
100 100
= 110
= 2×10.5×105
113. By performing which of the following operation
= 21×105 on a set of 4 numbers we shall get the numbers
= `2205 6, 14, 54, 149?
109. If the sum of 3/8 and 2/7 of any number is 370, (a) By adding 1 on the square of their sum
then what will be that number? (b) Subtracting one (1) from its cube
(a) 520 (b) 540 (c) Multiplying by 2 after adding 2.
(c) 500 (d) 560 (d) By adding 5 in its square
UP Ro/ARO (Pre) Exam 2010 230 YCT
CLICK HERE FOR FREE MATERIAL

Ans. (d) : Set of 4 number = (6,14,54,149) 118. The winner of Gyanpeeth Award for the year
each given number is obtained by adding 5 to the square 2008 was/were_______
of the number (a) Akhlaq Khan Shaharyar
6 = 12+5 (b) Kunwar Narayan
14 = 32+5 (c) O.N.V. Kurup
54 = 72+5 (d) Rahman Rahi
149 = 122 +5
Ans. (a) : The Jnanpith Award for the year 2008 was
Hence, 'd' is correct.
given to Akhlaq Mohammad Khan 'Shahryar'. He was a
114. The concept which the Indian Tourism
famous writer of Urdu language. The 57th Jnanpith
Ministry has popularized to promote tourism
in India is? Award for 2022 was given to Eminent Konkani
(a) Country of God (b) Pearl of the East Damodar Mauzo.
(c) Beautiful India (d) Incredible India 119. In the list of "25 most powerful women of last
Ans. (d) : The concept which the Indian Ministry of century" issued by Times Magazine in
Tourism has used to popularize tourism in India, is November 2010, what was the position,
Incredible India. Incredible India concept was assigned to Smt. Indira Gandhi.
established in the year 2002 by the Ministry of Tourism. (a) 8th (b) 9th
115. The head of the expert group established, (c) 10th (d) 11th
formed in context of "Fair and sustainable Ans. (b) : Indira Gandhi was ranked 9th among the
pricing of petroleum products was_____ world's 25 most powerful women of last century
(a) Sitaram Yechuri published by Times Magazine in November 2010.
(b) Bharat Jhunjhun Wala Mother Teressa has been ranked 22nd in this list.
(c) Yudhvir Reddy
(d) Kirit Parikh 120. During his Visit to New Delhi in November
2010, President Barrack Obama of USA went
Ans. (d) : A committee was constituted under the
chairmanship of Kirit Parikh, a former member of the to see?
Planning Commission, regarding the fair and (a) Tomb of Humayun (b) Jama Masjid
sustainable value of petroleum in products, which (c) Kutub Minar (d) Red Fort
submitted its report to the Ministry of Petroleum Ans. (a) : During his visited to Delhi in November,
February 2010. 2010. U.S.A. President Barack Obama went to see
116. The 'Stuxnet' bug detected in the Iranian Humayun's Tomb.
computers is-
121. The winner of 2010, Bookers Prize was?
(a) A hardware to provide fillip (boost) to
internet operations (a) Howard Jacobson (b) J.M. Coetzee
(b) A malicious programme designed to destroy (c) Peter Carey (d) Tom McCarthy
Nuclear Centrifuges Ans. (a) : In 2010, the Booker Prize Winner was
(c) A software to make hacking ineffectual Howard Jacobson. Hindi novelist Geetanjali Shree, won
(d) A process/method to harness BPO work. the Booker Prize 2022 for her novel 'Tomb of Sand'.
Ans. (b) : The 'Stuxnet' worm first caught in Iran's Her's is the first novel written in an Indian language,
computers was a malicious program to destroy nuclear and translated into English, which has won the
centrifuges. Stuxnet specifically targets Progammable prestigious award.
Logic Controllers (PLCs) that allow the automation of 122. Match List-I with List-II and choose the
electromechanical processes. It caused considerable
damage to Iran's nuclear program. correct answer from the codes given below-
117. National Green Tribunal Act 2010 is related List-I List-II
to? (Winner of 2010 (Region)
(a) The use of green fuels Nobel Prize)
(b) Encourage the use of alternative sources of A. Robert Edwards 1. Peace
energy B. Richard F. Heck 2. Economics
(c) Environment conservation
(d) Construction/formation of green courts C. Liu Xia Obo 3. Chemistry
Ans. (d) : The National Green Tribunal Act, 2010 deals D. Peter A. Diamond 4. Medical Science
with the creation of Green Courts. For the effective Codes:
implementation of environmental laws and protection of A B C D
environmental rights, the Central Government
established the National Green Tribunal on 18th (a) 2 3 4 1
October 2010 in Delhi. The branches of the NGT was (b) 3 4 1 2
set up in four place of the country were Bhopal, Pune, (c) 4 1 2 3
Kolkata and Chennai. The headquarter of NGT is Delhi. (d) 4 3 1 2
UP Ro/ARO (Pre) Exam 2010 231 YCT
CLICK HERE FOR FREE MATERIAL

Ans. (d) : The correct match are:- Ans. (a) : South African player A.B. De Villiers has
List-I List-II won the ICC One day player for year 2010 and was
(Winner of 2010 Nobel (Area) Awarded ODI Player of the Year in Bangalore.
Prize) 129. Among the following which three performing
A. Robert Edwards - Physiology or Art forms were included in the Intangible
Medicine Cultural Heritage of Humanity?
B. Richard F. Heck - Chemistry (1) Mudiyettu of Kerala
C. Liu Xia Obo - Peace Prize (2) Chhau dance form of Eastern India
(3) Kalbeliya dance form of Rajasthan
D. Peter A. Diamond - Economic Sciences
(4) Kuchipudi of Andhra Pradesh
123. In 2010, the Indian-American Nicky Halley was Choose the correct answers from the codes
elected as governor of? given below:
(a) Illinois (b) North Carolina (a) 1, 2 and 4 (b) 2, 3 & 4
(c) South Carolina (d) Virginia (c) 1, 3 & 4 (d) 1, 2 & 3
Ans. (c) : India-origin politician Nicky Halley has been Ans. (*) : UNESCO's concept of Intangible Cultural
elected governor of South Carolina. She was the second
Heritage emerged in the 1990's, focusing mainly on the
Indian after Bobby Jindal to be elected to the post of
intangible aspects of culture. Presently a total of 14
governor in the U.S.A.
Intangible Cultural Heritage Sites of India are included
124. The Dada Saheb Phalke award winner for year in the UNESCO list, which include Mudiyettu of
2009 was- Kerala, Chhau dance of Eastern India, Kalbeliya folk
(a) D. Ramanayadu (b) Mannadey songs of Rajasthan, Kuchipudi of Andhra Pradesh etc.
(c) Tapan Sinha (d) V.K. Murthy
130. The International Food Policy Research
Ans. (a) : Veteran film maker D. Ramanaidu on 9th Institute of United States of America had
September 2010 was selected for the Dadasaheb Phalke announced the "Global Hunger Index', 2010"
Award 2009. The Dadasaheb Phalke Award for the year
on 10th October. The position assigned to India
2020 was given to actress Asha parekh.
as per the index is-
125. Rajiv Gandhi Khel Ratna Prize for 2010 was (a) 65th (b) 66th
given to- th
(c) 67 (d) 68th
(a) Mary Kom (b) Saina Nehwal
(c) Sushil Kumar (d) Vijender Singh Ans. (c) : India was ranked 67 in Global Hunger Index
(GHI)-2010.
Ans. (b) : The country's female badminton player Saina
Nehwal was honored with the country's highest award, Note:- India's rank has dropped to the 101st position
the Rajiv Gandhi Khel Ratna Award 2010, for her among 116 countries in the Global Hunger Index (GHI)
outstanding international achievements. Decision has 2021. In 2020, India was placed at 94th delete, out of
been taken in 2022 by Indian Government to rename 107 countries.
Rajiv Gandhi Khel Ratna Award to Major Dhyan Chand 131. The 17th ASIAN Summit Conference concluded
Khel Ratna Award. in the last week of October 2010 at?
126. How many country players participated in the (a) Bangkok (b) Hanoi
16th Asian Games? (c) Jakarta (d) Singapore
(a) 43 (b) 44 Ans. (b) : The 17 ASEAN Summit, 5th Asia Summit,
th

(c) 45 (d) 46 were held from 26 to 30 October 2010 in Hanoi,


Ans. (c) : In the 16th Asian Games held in 2010, players Vietnam. The Association of Southeast Asian Nations
from 45 countries participated in the competition. The or ASEAN, was established on 8th August, 1967 in
2022 Asian Games is officially known as the 19th Asian Bangkok, Thailand.
Games and also known as Hangzhou 2022. 132. Who won the maximum gold medal in the 2010
127. The Indian Player who won maximum number commonwealth games?
of gold medals in the 19th commonwealth (a) Christine Gerard (b) Marilou Prevost
games?
(c) Alicia Jen Coutts (d) Pron Pears
(a) Gagan Narang (b) Gurpreet Singh th
(c) Omkar Singh (d) Vijay Kumar Ans. (c) : The 19 Commonwealth Games were held in
Delhi from 3-14 October 2010. Australian swimmer
Ans. (a) : The 19th Commonwealth Games were held in Alicia Coutts won the most five gold medals. At
Delhi from 3-14 October 2010. Indian shooter Gagan
Commonwealth Games 2022, Indian athletes won 61
Narang won the most four gold medals for India in
medals, 22 gold, 16 silvers and 23 bronze.
these Games and became the only male athlete to do.
128. The "I.C.C. one day player of the year" for the 133. World's largest solar power station has been
year 2010 was? started in?
(a) A.B. Devilliers (b) Reyan Harris (a) Australia (b) England
(c) Sachin Tendulkar (d) Shane Watson (c) France (d) Germany
UP Ro/ARO (Pre) Exam 2010 232 YCT
CLICK HERE FOR FREE MATERIAL

Ans. (c) : World largest solar power station was started 138. The largest producer of wine in the world is?
in France. Total energies has launched its largest (a) France (b) Italy
photovoltaic solar power plant in France, with a (c) Span (d) U.S.A.
capacity of 55 MW. Ans. (b) : Italy is the biggest wine producing country
Note :- In 2022, The Bhadla Solar Park, which is the with 44.5 Mhl. It is followed by Spain (35.0 Mhl),
largest solar power plant in the world based in Bhadla France (34.2 Mhl), United States (24 Mhl) and Australia
village, in India's Rajasthan Jodhpur district with (14.2 Mhl).
capacity of 2.25GW.
139. Match List-I with List-II and choose the
134. In the list of Annual Forbes India capitalist correct answer from the codes given below-
(November 2010), who was the richest Indian
Women? List-I List-II
(a) Km. Mrilani Sarabhai (Region) (Artist)
(b) Km. Namrata Bhartia A. Khayal 1. Suraj Khan
(c) Smt. Savitri Jindal B. Pakhawaj 2. Sadiq Ali Khan
(d) Km. Rayshri Pathi. C. Veena Vadan 3. Pandit Ayodhya
Ans. (c) : The last time Savitri Jindal was in the top 10 Prasad
of the Forbes India Richest list, the year was 2012. She D. Tabla 4. Veeru Mishra
was on rank seven and her fortune was $8.2 billion. In
2021, She's back in the elite club of India's richest, Codes:
again at rank seven and her net worth has more than A B C D
doubled to $18 billion. (a) 1 2 3 4
135. Village Tembhli was in the news in September, (b) 2 1 4 3
2010 for. (c) 1 3 2 4
(a) Mass suicides of farmers (d) 3 4 1 2
(b) Jugnu Project Ans. (c) : The correct match of follow are :-
(c) Aadhar Project List-I List-II
(d) None of these (Region) (Artist)
Ans. (c) : Tembhli is a village in Nandurbar district of A. Khayal - Suraj Khan
Maharashtra state in India, from where the ambitious
Aadhaar Scheme was launched on 29th September 2010. B. Pakhawaj - Pandit Ayodhya Prasad
Ranjana Sonawane became the first person to be given C. Veena Vadan - Sadiq Ali Khan
an Aadhaar identity number. D. Tabla - Veeru Mishra
136. The Indian Pulses Research Institute is located 140. Match List-I and List-II and choose the correct
in-
answer from the codes given below-
(a) Lucknow (b) Jabalpur
List-I List-II
(c) Kanpur (d) New Delhi
(Sphere/Domain) (Person)
Ans. (c) : Indian Institute of Pulses Research is a
government institute in Kanpur, Uttar Pradesh. It was A. Hindi Literature 1. Raskhan
established in the year 1983 by the Indian Council of B. Urdu 2. Gyan Chandra Jain
Agricultural Research to carry out basic strategic and C. Music and Dance 3. Savita Devi
applied research on major pulses crops.
D. Painting 4. Satish Chandra
137. In the whole world 17th November is celebrated
as "Global C.O.P.D. Day" the full form of Codes:
C.O.P.D.? A B C D
(a) Chronic Obligatory Pulmonary Disease (a) 1 2 3 4
(b) Communicable obstructive Pulmonary (b) 2 1 4 3
Disease (c) 1 3 2 4
(c) Chronic Obstructive Pharyngeal Disease (d) 3 4 1 2
(d) Chronic Obstructive Pulmonary Disease Ans. (a) : The correct match as follow are :-
Ans. (d) : Chronic Obstructive Pulmonary Disease List-I List-II
(COPD) is a debilitating condition that obstructs airflow
from the lungs, thus hampering one's ability to carry out (Sphere/Domain) (Person)
day-to-day activities with ease. Every year, November A. Hindi Literature 1. Raskhan
18 is observed as World COPD Day to raise awareness B. Urdu 2. Gyan Chandra Jain
about the chronic inflammatory lung condition. This
year's (2022) theme, "Living Well With COPD- C. Music and Dance 3. Savita Devi
Everybody-Everywhere". D. Painting 4. Satish Chandra

UP Ro/ARO (Pre) Exam 2010 233 YCT


CLICK HERE FOR FREE MATERIAL

Gòej ØeosMe meceer#ee DeefOekeâejer/meneÙekeâ meceer#ee DeefOekeâejer (Øeerefuecme) hejer#ee, 2010


meeceevÙe efnvoer
nue ØeMve-he$e
1. efvecveefueefKele ceW mes lelmece Meyo nw – 7. `eEmeieej' Meyo keâe lelmece nw –
(a) iejce (b) vejkeâ (a) ße=bieej (b) ßebieej
(c) vejce (d) leerLe& (c) ëe=bieej (d) eEMeieej
Gòej – (d) Gòej – (c)
JÙeeKÙee – efnvoer ceW ØeÙegkeäle DeefOekeâebMe MeyoeW keâer peveveer mebmke=âle Yee<ee JÙeeKÙee – `eEmeieej’ Meyo keâe lelmece ¤he ‘ëe=bieej’ neslee nw~ DevÙe
ner nw~ FveceW keâefleheÙe Meyo leodveg¤he Deheveer ieefjcee efnvoer Yee<ee ceW efJekeâuhe $egefšhetCe& nw~
DeÅeleve yeveeÙes ngS nQ~ pees lelmece kesâ veece mes peeves peeles nQ~ Gkeäle efoÙes 8. `nuoer' Meyo keâe lelmece nw –
ieÙes efJekeâuheeW ceW leerveeW keâes ÚesÌ[keâj efJekeâuhe (d) `leerLe&’ mebmke=âle keâe (a) njoer (b) nefjõe
Meyo nw pees lelmece Meyo nw~ (c) nequokeâe (d) njefõkeâe
2. `DeBieer"er' keâe lelmece Meyo nw – Gòej – (b)
(a) Deeqivekeâe (b) Debefve‰keâe JÙeeKÙee – `nuoer’ Meyo keâe lelmece Meyo `nefjõe’ neslee nw peyeefkeâ
(c) Deeqiveef‰keâe (d) Deeqiveef‰keâer DevÙe leerveeW efJekeâuhe $egefšhetCe& nQ~
Gòej – (c) 9. `iesntB' Meyo keâe lelmece nw –
JÙeeKÙee – `DeBieer"er’ keâe lelmece Meyo `Deeqiveef‰keâe’ nw~ peyeefkeâ (a) ieesOetce (b) ieesntB
efJekeâuhe ceW efoÙes ieÙes DevÙe leerveeW Meyo $egefšhetCe& nQ~ (c) ieesngce (d) ieesOegce
3. efvecveefueefKele ceW mes keâewve leodYeJe Meyo nw?
Gòej – (a)
(a) efovekeâj (b) efoJeekeâj JÙeeKÙee – `iesntB’ Meyo keâe lelmece Meyo `ieesOetce’ neslee nw~ DevÙe leerveeW
(c) ØeYeekeâj (d) metjpe efJekeâuhe DemelÙe nQ~
Gòej – (d) 10. `KeBÌ[]nj' keâe lelmece Meyo nw –
JÙeeKÙee – Ssmes Meyo pees mebmke=âle mes GlheVe SJeb efJekeâefmele ngS nQ, (a) KeC[nj (b) Keb[Iej
leodYeJe Meyo keânueeles nQ~ leled + YeJe, efpemekeâe DeeMeÙe nw `Gmemes (c) KeC[ie=n (d) KeÌ[nj
GlheVe'~ Ssmes MeyoeW keâes osKekeâj menpe ner Devegceeve nes peelee nw efkeâ Ùes Gòej – (c)
mebmke=âle kesâ efkeâme Meyo mes ØemHegâefšle ngS nQ~ pewmes– efoÙes ieÙes efJekeâuhe ceW JÙeeKÙee – `KeB[nj’ Meyo keâe lelmece Meyo `KeC[ie=n’ neslee nw~ Mes<e
`metjpe’ Meyo keâes osKekeâj mhe„ Yeeve nes peelee nw efkeâ Ùen `metÙe&’ Meyo efJekeâuheebs kesâ Meyo $egefšhetCe& nQ~
mes yevee ngDee nw~ 11. `Jen ßes‰ Gheemekeâ nw' ceW efJeMes<Ùe nw –
4. efvecveefueefKele ceW mes keâewve mee Meyo `lelmece' veneR nw? (a) Jen (b) ßes‰ (c) Gheemekeâ (d) nw
(a) DeeBKe (b) veÙeve Gòej – (c)
(c) ves$e (d) Âie JÙeeKÙee – pees Meyo meb%ee Ùee meJe&veece keâer efJeMes<elee yeleueelee nw, Gmes
Gòej – (a) efJeMes<eCe keânles nQ leLee efJeMes<eCe efpemekeâer efJeMes<elee yeleueelee nw Gmes
JÙeeKÙee – efoÙes ieÙes efJekeâuheeW ceW ves$e, veÙeve Deewj Âie lelmece Meyo nQ efJeMes<Ùe keânles nQ~ `Gheemekeâ’ efJeMes<Ùe Meyo nw~
peyeefkeâ `DeeBKe’ leodYeJe Meyo nw~ DeeBKe keâe lelmece Meyo `Deef#e’ neslee nw~ 12. `Yeues Deewj ceneve ueesie GefÛele Deewj mebÙeefcele JÙeJenej keâjles nQ~'
5. efvecveefueefKele ceW `lelmece' Meyo nw – GheÙeg&keäle JeekeäÙe ceW efkeâleves efJeMes<eCe efJeMes<Ùe nQ?
(a) keâeve (b) peerYe (a) oes efJeMes<eCe Deewj oes efJeMes<Ùe
(c) cegKe (d) oeBle (b) leerve efJeMes<eCe Deewj oes efJeMes<Ùe
Gòej – (c) (c) Ûeej efJeMes<eCe Deewj leerve efJeMes<Ùe
(d) Ûeej efJeMes<eCe Deewj oes efJeMes<Ùe
JÙeeKÙee – efoÙes ieÙes efJekeâuhe ceW `cegKe’ lelmece Meyo nw peyeefkeâ Fmekeâe Gòej – (d)
leodYeJe Meyo `cegBn’ neslee nw~ DevÙe efJekeâuhe keâeve, peerYe Deewj oeBle Ùes
leodYeJe Meyo nQ efpevekeâe lelmece ¤he ›eâceMe: nesiee keâCe&, efpe£e Deewj oble~ JÙeeKÙee – efoÙes ieÙes efJekeâuhe ceW ‘Ûeej efJeMes<eCe Deewj oes efJeMes<Ùe’ nQ~
`Yeues’ Deewj `ceneve’, GefÛele Deewj mebÙeefcele efJeMes<eCe nQ~ ueesie SJeb
6. efvecveefueefKele ceW mes keâewve Meyo leodYeJe nw – JÙeJenej efJeMes<Ùe nQ~
(a) ceOeghe (b) ceOegkeâj
13. `Ùen ieeÙe DeefOekeâ otOe osleer nw' Gkeäle JeekeäÙe ceW `DeefOekeâ'
(c) YeÇcej (d) YeBJeje
Gòej – (d) efJeMes<eCe efkeâmekeâer efJeMes<elee yelee jne nw?
(a) ieeÙe keâer (b) otOe keâer
JÙeeKÙee – efvee|o„ efJekeâuheeW ceW ceOeghe, ceOegkeâj Deewj YeÇcej Ùes lelmece Meyo nQ, (c) osves keâer (d) efkeâmeer keâer veneR
peyeefkeâ `YeBJeje’ leodYeJe Meyo nw~ `YeBJeje’ keâe lelmece ¤he `YeÇcej’ neslee nw~ Gòej – (b)
UP UDA/LDA (Pre) General Hindi 2010 234 YCT
CLICK HERE FOR FREE MATERIAL

JÙeeKÙee–efoÙes ieÙes efJekeâuhe ceW `DeefOekeâ’ Meyo `otOe’ keâer efJeMes<elee yeleuee jne JÙeeKÙee – `peerYe’ keâe heÙee&ÙeJeeÛeer Meyo `jmevee’ neslee nw peyeefkeâ
nw ve efkeâ ieeÙe keâer~ DeefOekeâ SJeb keâce heefjceeCeJeeÛekeâ efJeMes<eCe Meyo nQ~ JeÛeve, OJeefve Deewj peerJe keâe peerYe mes keâesF& DeeMeÙe veneR nw~
14. `efJeÉeved JÙeeqkeäle hetpÙe nesles nQ' ceW ØeÙegkeäle efJeMes<eCe nw – 22. keâewve mee Meyo `Deefn' keâe heÙee&ÙeJeeÛeer veneR nw –
(a) meeJe&veeefcekeâ efJeMes<eCe (b) iegCeJeeÛekeâ efJeMes<eCe
(a) Gjie (b) mejerme=he
(c) mebKÙeeJeeÛekeâ efJeMes<eCe (d) heefjCeeceyeesOekeâ efJeMes<eCe
(c) heJeveeMe (d) eEmeOegj
Gòej – (b)
Gòej – (d)
JÙeeKÙee – `efJeÉeve JÙeeqkeäle hetpÙe nesles nQ’ ceW iegCeJeeÛekeâ efJeMes<eCe nw~
efpeme efJeMes<eCe mes meb%ee Ùee meJe&veece kesâ iegCe, ¤he, jbie, Deekeâej, DeJemLee, JÙeeKÙee – efoÙes ieÙes efJekeâuheeW ceW `Deefn’ keâe heÙee&ÙeJeeÛeer Meyo `eEmeOegj’
mJeYeeJe, oMee, mJeeo, mheMe&, iebOe, efoMee, mLeeve, meceÙe, Yeej, leeheceeve veneR nw~ efmebOegj keâe DeeMeÙe neLeer mes nw, peyeefkeâ Deefn, Gjie, mejerme=he,
Deeefo keâe yeesOe neslee nw, iegCeJeeÛekeâ efJeMes<eCe keânueeles nQ~ heJeveeMe mehe& keâe heÙee&ÙeJeeÛeer Meyo nw~
15. efvecveefueefKele ceW mes keâewve Meyo efJeMes<eCe veneR nw – 23. `vewmee|iekeâ' keâe heÙee&ÙeJeeÛeer nw –
(a) Glke=â„ (b) efveke=â„ (a) melke=âle (b) Ûecelke=âle
(c) Oe=„ (d) efJe<eeo (c) Øeeke=âeflekeâ (d) Ûelege|okeâd
Gòej – (d) Gòej – (c)
JÙeeKÙee – efoÙes ieÙes efJekeâuheeW ceW Glke=â„, efveke=â„ Deewj Oe=„ Meyo JÙeeKÙee – `vewmee|iekeâ’ keâe heÙee&ÙeJeeÛeer Meyo `Øeeke=âeflekeâ’ neslee nw peyeefkeâ
efJeMes<eCe nQ~ peyeefkeâ `efJe<eeo’ Meyo efJeMes<eCe veneR nw~ Ùen Skeâ mebÙegkeäle melke=âle, Ûecelke=âle Deewj Ûelege|okeâ keâe vewmee|iekeâ Meyo mes keâesF& uesvee-osvee
ef›eâÙee nw~ peneB ef›eâÙee ceW oes Ùee oes mes DeefOekeâ Oeeleg efceues neW Jen
mebÙegkeäle ef›eâÙee keânueeleer nw~ veneR nw~
16. efvecveefueefKele MeyoeW ceW mes pees Meyo efJeMes<eCe veneR nw, Gmekeâe 24. efvecveefueefKele ceW keâewve mee Meyo `neLe' keâe heÙee&ÙeJeeÛeer veneR nw?
GuuesKe keâerefpeS – (a) keâefš (b) nmle (c) heeefCe (d) keâj
(a) meppeve (b) ogpe&ve Gòej – (a)
(c) megkegâceej (d) ceeveme JÙeeKÙee – `neLe’ keâe heÙee&ÙeJeeÛeer Meyo `keâefš’ veneR neslee peyeefkeâ nmle,
Gòej – (d) heeefCe Deewj keâj neLe kesâ heÙee&ÙeJeeÛeer Meyo nQ~ `keâefš’ Meyo `keâcej
JÙeeKÙee – efoÙes ieÙes efJekeâuhe ceW meppeve, ogpe&ve Deewj megkegâceej efJeMes<eCe ØeosMe’ kesâ efueS ØeÙegkeäle neslee nw~
Meyo nQ peyeefkeâ `ceeveme’ efJeMes<eCe veneR nw Deefheleg Ùen Meyo keâce& keâe
Åeeslekeâ nw~ 25. efvecveefueefKele ceW keâewve mee Meyo `metÙe&' keâe heÙee&ÙeJeeÛeer veneR nw?
17. efvecveefueefKele ceW keâewve mee Meyo `efJeMes<eCe' veneR nw? (a) ØeYeekeâj (b) efJeYeekeâj
(a) Deeieeceer (b) Meevle (c) keâeuee (d) legce (c) efovekeâj (d) efovesMe
Gòej – (d) Gòej – (b)
JÙeeKÙee – Deeieece, keâeuee Deewj Meevle efJeMes<eCe Meyo nw peyeefkeâ `legce’ JÙeeKÙee – `metÙe&’ keâe heÙee&ÙeJeeÛeer Meyo ØeYeekeâj, efovekeâj Deewj efovesMe
Meyo heg®<eJeeÛekeâ meJe&veece nw~ Ssmes meJe&veece pees efkeâmeer heg®<e Ùee m$eer kesâ neslee nw peyeefkeâ `efJeYeekeâj’ `metÙe&’ keâe veneR yeefukeâ Ûevõcee keâe heÙee&Ùe nw~
efueS ØeÙegkeäle nesles nQ, heg®<eJeeÛekeâ meJe&veece keânueeles nQ~ pewmes - ceQ, nce, 26. `ÛeeBoveer' keâe heÙee&ÙeJeeÛeer Meyo veneR nw –
let, legce, Deehe, Jen, Jes, Ùen Deewj Ùes~
(a) Ûevõelehe (b) keâewcegoer
18. `efJeMes<Ùe' Meyo nw –
(c) pÙeeslmvee (d) ceÙebkeâ
(a) ueefuelee (b) megvoj
Gòej – (d)
(c) uecyee (d) ueIeg
Gòej – (a) JÙeeKÙee – `ceÙebkeâ ÛeeBoveer’ keâe heÙee&ÙeJeeÛeer Meyo veneR nw~ ceÙebkeâ keâe
JÙeeKÙee – `ueefuelee’ efJeMes<Ùe Meyo nw, peyeefkeâ megvoj, uecyee, ueIeg DeeMeÙe `Ûevõcee’ mes nw peyeefkeâ Ûebõelehe, keâewcegoer Deewj pÙeeslmvee ÛeeBoveer
efJeMes<eCe Meyo nw~ keâe heÙee&ÙeJeeÛeer Meyo nw~
19. efvecveefueefKele ceW efJeMes<Ùe heo nw – 27. efvecveefueefKele ceW keâewve mee Meyo `mejmJeleer' keâe heÙee&ÙeJeeÛeer
(a) Gcee (b) keâeefuecee (c) ceOegefjcee (d) ceefncee veneR nw?
Gòej – (a) (a) JeerCeeheeefCe (b) ceneMJeslee
JÙeeKÙee – keâeefuecee, ceefncee Deewj ceOegefjcee efJeMes<eCe Meyo nQ~ peyeefkeâ (c) heodcee (d) Yeejleer
Gcee efJeMes<Ùe heo nw~ Gòej – (c)
20. efvecveefueefKele ceW efJeMes<eCe heo nw – JÙeeKÙee–efoÙes ieÙes efJekeâuhe ceW `heodcee’ Meyo mejmJeleer keâe heÙee&ÙeJeeÛeer
(a) Gheemevee (b) Godieej veneR nw~ heodcee keâe DeLe& `ue#ceer’ mes nw peyeefkeâ JeerCeeheeefCe, ceneMJeslee
(c) DeeÙeleueesÛevee (d) Jevovee Deewj Yeejleer, mejmJeleer keâe heÙee&ÙeJeeÛeer Meyo nw~
Gòej – (c)
28. `peveeo&ve' efkeâmekeâe heÙee&ÙeJeeÛeer nw?
JÙeeKÙee – Gòeâ ØeMve efJekeâuheeW ceW DeeÙeleueesÛevee efJeMes<eCe heo nw~
(a) jece (b) ke=â<Ce (c) efJe<Ceg (d) yeÇÿee
21. `peerYe' keâe heÙee&Ùe nw –
Gòej – (c)
(a) JeÛeve (b) jmevee
(c) OJeefve (d) peerJe JÙeeKÙee – `peveeo&ve’ efJe<Ceg keâe heÙee&ÙeJeeÛeer Meyo nw~ peveeo&ve kesâ DevÙe
Gòej– (b) heÙee&ÙeJeeÛeer nQ – nefj, ue#ceerkeâevle Deeefo~
UP UDA/LDA (Pre) General Hindi 2010 235 YCT
CLICK HERE FOR FREE MATERIAL

29. efvecveefueefKele ceW keâewve mee Meyo `mecegõ' keâe heÙee&ÙeJeeÛeer veneR nw? 37. `efJe«en' keâe efJeueesce –
(a) heÙeesefOe (b) peueo (a) meeqvOe (b) DeefJe«en
(c) peueefOe (d) JeeefjefOe (c) Dee«en (d) «enCe
Gòej – (b) Gòej – (a)
JÙeeKÙee – efoÙes ieÙes efJekeâuhe ceW `mecegõ’ keâe heÙee&ÙeJeeÛeer heÙeesefOe, JÙeeKÙee – `efJe«en’ keâe efJeueesce `meeqvOe’ neslee nw ve efkeâ Dee«en, «enCe
peueefOe, JeeefjefOe neslee nw~ peyeefkeâ `peueo’ Meyo keâe DeeMeÙe yeeoue mes Deewj DeefJe«en~
nw~ mecegõ keâe DevÙe heÙee&ÙeJeeÛeer Meyo jlveekeâj, veerjefveefOe, heÙeesefveefOe,
GoefOe, JeejerMe Deeefo nw~ 38. efvecveefueefKele ceW keâewve mee efJeueesce Ùegice $egefšhetCe& nw?
(a) Dehes#ee - Ghes#ee (b) De«epe - Devegpe
30. efvecveefueefKele ceW `efMeJe' keâe heÙee&ÙeJeeÛeer Meyo nw –
(c) GVele - DeJeiele (d) Deeoeve - Øeoeve
(a) efMeJeeueÙe (b) ®õ Gòej – (c)
(c) ®õe#e (d) nefj
Gòej – (b) JÙeeKÙee – efoÙes ieÙes efJeueesce Ùegice ceW Dehes#ee - Ghes#ee, De«epe -
Devegpe Deewj Deeoeve - Øeoeve mener efJeueesce Ùegice nw~ peyeefkeâ `GVele’ keâe
JÙeeKÙee – efoÙes ieÙes efJekeâuhe ceW `efMeJe’ keâe heÙee&ÙeJeeÛeer Meyo `®õ’ neslee efJeueesce `DeJevele’ neslee nw ve efkeâ DeJeiele~
nw ve efkeâ efMeJeeueÙe, ®õe#e Deewj nefj~ efMeJe keâe DevÙe heÙee&ÙeJeeÛeer Meyo nw
– MecYeg, ef$ehegjeefj, ceneosJe, osJeeefOe osJe, kewâueeMeheefle, Mebkeâj Deeefo~ 39. `ÙeLeeLe&' keâe efJeueesce nw –
31. `DeLe' keâe efJeueesce nw – (a) ke=âef$ece (b) DeeoMe& (c) GefÛele (d) DevegefÛele
(a) hetCe& (b) meceehle
Gòej – (b)
(c) Fefle (d) Kelce JÙeeKÙee – ÙeLeeLe& keâe efJeueesce `DeeoMe&’ neslee nw JeneR `ke=âef$ece’ keâe
Gòej – (c) Øeeke=âle, DevegefÛele keâe GefÛele efJeueesce Meyo neslee nw~
JÙeeKÙee – `DeLe’ keâe efJeueesce `Fefle’ neslee nw peyeefkeâ hetCe& keâe ‘DehetCe&&’, 40. `meeOeg' keâe efJeueesce Meyo nw –
meceehle keâe ‘DeejcYe’ Deewj Kelce keâe ‘Meg¤’ efJeueesce Meyo neslee nw~ (a) meeOegveer (b) mevÙeeefmeveer
32. `DeefYe%e' keâe efJeueesce nw – (c) meeOJeer (d) DemeeOeg
(a) De%e (b) le%e (c) Øe%e (d) Ûelegj Gòej – (d)
Gòej – (a) JÙeeKÙee – `meeOeg’ keâe efJeueesce `DemeeOeg’ neslee nw ve efkeâ meeOJeer,
JÙeeKÙee – `DeefYe%e’ keâe efJeueesce DeveefYe%e neslee nw peyeefkeâ `De%e’ keâe mebvÙeeefmeveer Deewj meeOegveer~
`efJe%e’ neslee nw~ ÛetBefkeâ efJekeâuhe ceW DeveefYe%e veneR efoÙee ieÙee nw Dele: 41. efkeâme Meyo keâer Jele&veer DeMegæ nw?
DeefYe%e keâe efJeueesce `De%e’ ner nesiee~
(a) uebieÌ[ (b) yegPekeäkeâÌ[ (c) keâeWkeâCe (d) YegäKKeÌ[
33. `ke=âle%e' keâe efJeueesce nw – Gòej – (d)
(a) Deke=âle%e (b) mebJesovenerve
JÙeeKÙee – efoÙes ieÙes efJekeâuheeW ceW uebieÌ[, yegPekeäkeâÌ[ Deewj keâeWkeâCe Meyo
(c) ke=âleIve (d) peÌ[
Gòej – (c) Megæ nQ~ efJekeâuhe (d) ceW efoÙee ieÙee Meyo `YegKKeÌ[‘ DeMegæ nw~ Fmekeâe
Megæ ¤he nesiee `YegkeäKeÌ[’~
JÙeeKÙee – `ke=âle%e’ keâe efJeueesce Meyo `ke=âleIve’ neslee nw peyeefkeâ peÌ[ keâe
Ûesleve, mebJesovenerve keâe mebJesoveMeerue neslee nw~ 42. efkeâme Meyo keâer Jele&veer DeMegæ nw?
(a) efvejvegveeefmekeâ (b) efÚõevJesMeer
34. `mebkeâerCe&' keâe efJeueesce nw –
(c) ielÙeLe& (d) DevleMÛeslevee
(a) meb#eshe (b) efJemleej
Gòej – (b)
(c) efJekeâerCe& (d) efJemleerCe&
Gòej – (d) JÙeeKÙee – efvejvegveeefmekeâ, ielÙeLe& Deewj DevleMÛeslevee leerveeW Megæ nQ
JÙeeKÙee – `mebkeâerCe&’ keâe efJeueesce `efJemleerCe&’ neslee nw ve efkeâ efJemleej, peyeefkeâ efÚõevJesMeer DeMegæ nw~ Fmekeâe Megæ ¤he nesiee efÚõevJes<eer~
efJekeâerCe& Deewj meb#eshe~ peyeefkeâ meb#eshe keâe efJeueesce efJemleej neslee nw~ 43. keâewve mee JeekeäÙe Megæ nw?
35. `Devegjeqkeäle' keâe efJeueesce nw – (a) Deehekeâe Meemeve mecyevOeer keâeÙe& DeefOekeâ efJeKÙeelehetCe& nw
(a) efJejeie (b) efJejeqkeäle (b) Deehekeâes Yetefce-YeJeve-Jeenve keâe DeefYeveJe megKe keâer Øeeefhle nesieer
(c) eflejesYeeJe (d) mebmeeqkeäle (c) Ùen meceeÛeej hetjs osMe Yej ceW legjvle Hewâue ieÙee
Gòej – (b) (d) legcnW keâue kegâÚ nes peeÙes lees ceQ keâneR keâe veneR jntBiee
Gòej – (d)
JÙeeKÙee – `Devegjeqkeäle’ keâe efJeueesce `efJejeqkeäle’ neslee nw peyeefkeâ efJejeie
keâe Devegjeie leLee eflejesYeeJe keâe DeeefJe&YeeJe neslee nw~ JÙeeKÙee – efoÙes ieÙes efJekeâuheeW ceW efJekeâuhe (d) legcnW keâue kegâÚ nes
peeÙes lees ceQ keâneR keâe veneR jntBiee~ JeekeäÙe Megæ nw~
36. `meehes#e' keâe efJeueesce Meyo nw –
44. keâewve mee JeekeäÙe Megæ nw?
(a) Demeehes#e (b) efve<he#e
(a) jesieer Deheveer keâcepeesefjÙeeW kesâ keâejCe G" lekeâ veneR hee jne Lee
(c) efvejhes#e (d) Deehes#e
(b) jepeheLe keâer meÌ[keâ mes PeeBefkeâÙeeb Jeeheme ueewš ieFË
Gòej – (c)
(c) efMekeâejer Gme hej ieesueer ÛeueeF& hej Jen Mesj yeÛe efvekeâuee
JÙeeKÙee – `meehes#e’ keâe efJeueesce `efvejhes#e’ neslee nw peyeefkeâ Demeehes#e, (d) Gme meceÙe ÛejKee Ûeueevee Yeer Skeâ DevegMeemeve Lee
efve<he#e Deewj Deehes#e Meyo keâe DeeMeÙe ØeMve mes efyeukegâue Demecyeæ nw~ Gòej – (d)
UP UDA/LDA (Pre) General Hindi 2010 236 YCT
CLICK HERE FOR FREE MATERIAL

JÙeeKÙee – efoÙes ieÙes efJekeâuheeW ceW `Gme meceÙe ÛejKee Ûeueevee Yeer Skeâ 53. efpemekesâ heeme kegâÚ ve nes, Gmekesâ efueS GheÙegkeäle Meyo nw –
DevegMeemeve Lee’~ JeekeäÙe Megæ nw~ (a) DeYeeJe«emle (b) DeeEkeâÛeve
(c) oervenerve (d) ceneoerve
45. efvecveefueefKele ceW Megæ Jele&veer Jeeuee Meyo nw – Gòej – (b)
(a) keâJeefÙe$eer (b) keâefJeefÙe$eer JÙeeKÙee – `efpemekesâ heeme kegâÚ ve nes’ Gmekesâ efueS GheÙegkeäle Meyo nw -
(c) keâefJeÙe$eer (d) keâefJe$eer
‘DeeEkeâÛeve’ ve efkeâ oerve-nerve, ceneoerve SJeb DeYeeJe«emle~
Gòej – (a)
54. `efpemes yegueeÙee ve ieÙee nes' JeekeäÙe kesâ efueS ØeÙegkeäle nesves Jeeuee
JÙeeKÙee – efoÙes ieÙes efJekeâuheeW ceW Megæ Jele&veer Jeeuee Meyo `keâJeefÙe$eer' nw
ve efkeâ DevÙe leerveeW efJekeâuhe~ Meyo nw –
(a) DeefleefLe (b) DeYÙeeiele
46. efvecveefueefKele ceW Megæ Jele&veer Jeeuee Meyo nw – (c) Deveentle (d) efjMlesoej
(a) Megßet<ee (b) megßet<ee (c) megßeg<ee (d) ßegMet<ee Gòej – (c)
Gòej – (a) JÙeeKÙee – `efpemes yegueeÙee ve ieÙee nes’ JeekeäÙe kesâ efueS GheÙegkeäle Meyo nw -
JÙeeKÙee – efoÙes ieÙes efJekeâuheeW ceW Megæ Jele&veer Jeeuee Meyo `Megßet<ee’ nw Deveentle~ peyeefkeâ DeefleefLe, efpemekesâ Deeves keâer keâesF& efleefLe %eele ve nes~ Fmeer
ve efkeâ DevÙe leerveeW Meyo~ Øekeâej DeYÙeeiele Je efjMlesoej keâe ØeMve mes keâesF& DeeMeÙe veneR nw~
47. Megæ Jele&veer Jeeuee Meyo nw – 55. `cees#e keâer FÛÚe jKeves Jeeuee' JeekeäÙeebMe kesâ efueS GheÙegkeäle
(a) F&Mee& (b) F&<ee& (c) F<ee& (d) F&<Ùee& Meyo nw –
Gòej – (d) (a) Ùeesieer (b) cegeqkeälekeâeceer
JÙeeKÙee – efoÙes ieÙes efJekeâuheeW ceW Megæ Jele&veer Jeeuee Meyo ‘F&<Ùee&’ nw~ (c) cegceg#eg (d) lehemJeer
Mes<e efJekeâuheeW kesâ Meyo $egefšhetCe& nQ~ Gòej – (c)
JÙeeKÙee – cees#e keâer FÛÚe jKeves Jeeuee `cegceg#eg' keâne peelee nw ve efkeâ
48. efvecveefueefKele ceW mes DeMegæ Jele&veer Jeeuee Meyo nw – Ùeesieer, cegeqkeälekeâeceer SJeb lehemJeer~
(a) GuuebIeve (b) ceveesjLe 56. jbiecebÛe kesâ heox kesâ heerÚs keâe mLeeve keâne peelee nw –
(c) GpJeue (d) Jelmeue (a) he=‰Yetefce (b) vesheLÙe
Gòej – (c) (c) cebÛehe=‰ (d) ieg¢ecebÛe
JÙeeKÙee – efoÙes ieÙes efJekeâuheeW ceW GuuebIeve, ceveesjLe Deewj Jelmeue mener Gòej – (b)
Meyo nw peyeefkeâ efJekeâuhe (c) ceW efoÙee ieÙee Meyo GpJeue Meyo DeMegæ JÙeeKÙee – jbiecebÛe kesâ hejos kesâ heerÚs keâe mLeeve keâne peelee nw -
nw~ Fmekeâe Megæ Jele&veer ¤he nesiee - `GppJeue’~ `vesheLÙe’, ve efkeâ he=‰Yetefce, cebÛehe=‰ SJeb ieg¢ecebÛe~
49. Megæ Meyo ¤he nw – 57. `nJeve ceW peueeves Jeeueer uekeâÌ[er' kesâ efueS Megæ Meyo nw –
(a) ogjefveJeej (b) ogveeaJeej (a) nJevemeece«eer (b) Jevekeâe‰
(c) og:efveJeej (d) oge|veJeej (c) Meg<keâkeâe‰ (d) meefceOee
Gòej – (d) Gòej – (d)
JÙeeKÙee – efoÙes ieÙes efJekeâuheeW ceW Megæ Jele&veer Jeeuee Meyo ‘oge|veJeej’ JÙeeKÙee – nJeve ceW peueeves Jeeueer uekeâÌ[er kesâ efueS Megæ Meyo nw -
nw~ Mes<e efJekeâuheeW kesâ Meyo $egefšhetCe& nQ~ `meefceOee’, ve efkeâ nJeve meece«eer, Jevekeâe‰ SJeb Meg<keâ keâe‰~
50. `TBšeW keâe ...... pee jne nw' JeekeäÙe ceW efjkeäle mLeeve keâer hete|le 58. `efoMeeSb ner efpevekeâe Jem$e nw' GvnW keâne peelee nw –
GheÙegkeäle (Megæ) Meyo mes keâerefpeS~ (a) efJeMJecYej (b) efokedâheeue
(a) keâejJeeB (b) pelLee (c) keâeefHeâuee (d) yesÌ[e (c) efoiecyej (d) hewiecyej
Gòej – (c) Gòej – (c)
JÙeeKÙee – efoÙes ieÙes efJekeâuhe ceW `keâeefHeâuee' Meyo Megæ Øeleerle neslee nw JÙeeKÙee – `efoMeeÙeW ner efpevekeâe Jem$e nQ GvnW efoiecyej keâne peelee nw’,
ve efkeâ keâejJeeb, pelLee Deewj yesÌ[e~ ve efkeâ efJeMJecYej, efokedâheeue SJeb hewiecyej~
51. KeeÅe meece«eer pees Ùee$ee kesâ meceÙe jemles ceW GheYeesie kesâ efueS oer 59. efpemekeâe pevce keâvÙee kesâ ieYe& mes ngDee nes –
peeleer nw – (a) keâvÙeeheg$e (b) keâeveerve
(a) mJeuheenej (b) heLÙe (c) heeLesÙe (d) Gheenej (c) DeJewOe heg$e (d) kegâceejermegle
Gòej – (c) Gòej – (b)
JÙeeKÙee – KeeÅe meece«eer pees Ùee$ee kesâ meceÙe jemles ceW GheYeesie kesâ efueS JÙeeKÙee – `efpemekeâe pevce keâvÙee kesâ ieYe& mes ngDee nes’ Gmes `keâeveerve’
Øeoeve keâer peeleer nw, Gmes `heeLesÙe' keâne peelee nw~ keânles nQ ve efkeâ DeJewOe heg$e, keâvÙeeheg$e Je kegâceejermegle~
52. DeeOeer jele keâe meceÙe – 60. `hesš keâer Deeqive' kesâ efueS Megæ Meyo nw –
(a) MeJe&jer (b) efJeYeeJejer (a) oeJeeeqive (b) yeÌ[Jeeeqive
(c) efveMee (d) efveMeerLe (c) pe"jeeqive (d) cevoeeqive
Gòej – (d) Gòej – (c)
JÙeeKÙee – DeeOeer jele keâe meceÙe kesâ efueS ØeÙegkeäle Meyo `efveMeerLe' neslee nw JÙeeKÙee – `hesš keâer Deeqive’ kesâ efueS Megæ Meyo nw `pe"jeeqive’, ve efkeâ
ve efkeâ MeJe&jer, efJeYeeJejer Deewj efveMee~ oeJeeeqive, yeÌ[Jeeeqive Je ceboeeqive~
UP UDA/LDA (Pre) General Hindi 2010 237 YCT
CLICK HERE FOR FREE MATERIAL

UPPSC RO-ARO Special (Mains) Exam-2010


GENERAL STUDIES
Solved Paper
1. Which carnation of Lord Vishnu has been Ans. (d) : Firoz Shah Tughlaq had established an
depicted as saving Earth from the sea? employment bureau for helping jobless people. Firoz
(a) Kashap (b) Matsya Shah Tughlaq took a number of significant steps during
(c) Varaha (d) Narsingha his reign. He set up employment bureau, marriage
Ans. (c) : Varaha, third carnation of Lord Vishnu has bureau, built public hospitals. He also set up a
been depicted as saving Earth from the sea. According department of public works which repaired old
to Hindu mythology, when a demon named monuments and build new ones. Many new cities like
Hiranyaksha dragged the earth to the bottom of the sea, Firozpur, Hissar-i-Firoza among others were built
Vishnu took the form of a boar in order to rescue it. during his reign. He also established a charity
department called Diwan-i-Khairat. A water clock and
2. The inscriptional evidence mentioning about a sun clock were also constructed during his reign.
King Nanda is:
(a) Hathi Gumpha inscription of Kharvela 7. Who among the following was influenced by
Islam Religion?
(b) Rummin dei Pillar edict
(a) Chaitanya (b) Meerabai
(c) Junagarh Edict of Rudradaman (c) Namdev (d) Vallabhacharya
(d) Ayoydhya edict of Dhandev
Ans. (c): Saint Namdev was influenced by Islam
Ans. (a) : The inscriptional evidence mentions about religion. He made bhakti movement very famous. He
the existence of King Nanda is Hathi Gumpha didn't believe in caste system and Muslims were also his
inscription of Kharvela. According to the Hathigumpha disciples. He was against idol worship and ritualism.
inscription of Kharvela 'Nand Raja' (King Nanda) had
excavated a canal in Kalinga and also had taken a Jain 8. Consider the following statements about
idol from Kalinga. Mughal Mansabdari System and select the
correct answer from the codes given below:
3. The sculpture of the Buddha with an earth- 1. Jat and Sawar posts were granted.
touching gesture dates back to:
2. Mansabdar were ancestral officers.
(a) Kushan period (b) Gupta period
3. There were three categories of Mansabdars.
(c) Vardhan period (d) Rajput period
4. They were given salaries by the Diwan office.
Ans. (b) : The sculpture of the Buddha with an earth- (a) All four statements are correct.
touching gesture dates back to Gupta period. Five (b) All four statements are wrong.
Mudras are found in Gupta period Buddhist Sculpture
(c) Only (1),(2) and (3) are correct.
namely - Dhyaba (meditation), Bhumisparsha (Earth
touching), Abhaya (gesture of fearlessness), Varada (d) Only (1) and (3) are correct.
(Blessing) and Religions posture. Ans. (d) : Akbar introduced 'Mansabdari System' in
4. The first Gupta ruler to issue silver coins was: 11th year of his reign. Based on decimal system,
Mansab system was a grading system used by the
(a) Chandragupta-I (b) Samudragupta Mughal rulers to fix the rank and salary of a Mansabdar,
(c) Chandragupta-II (d) Kumargupta who were basically royal officers. The post of
Ans. (c) : The first Gupta ruler to issue silver coins was Mansabdars were not hereditary. Akbar started the posts
Chandragupta II. The silver coins of Guptas begin after the of 'Jaat' and 'sawar' in last years of his reign. A
Western Satraps were overthrown by Chandragupta II. mansabdar were provided 5000 Zat and 2000 Sawar as
5. Kailasha Temple of Ellora was constructed by: well as Akbar had divided each category of Mansabdar
(a) Rashtrakutas (b) Chalukyas of Vatapi in three sub-divisions by which there had various
(c) Ganga Rulers (d) None of them mansabdar comes under 5000 category. Thus,
statements (1) and (3) are correct.
Ans. (a): Kailasha Temple of Ellora was constructed by
Rashtrakuta rulers. The Kailash Temple at Ellora was 9. Shivaji signed the 'Treaty of Purandar'(1665
built by the Rashtrakuta King Krishna I in 756-773 CE. AD) with which Mughal Army Commandar'?
He was a great patron of art and architecture. (a) Jaswant Singh (b) Jai Singh
(c) Shaista Khan (d) Diler Khan
6. Who among the following established an
employment bureau for helping jobless people? Ans. (b) : Shivaji signed the 'Treaty of Purandar' with
(a) Balban Mughal Army Commandar Jai Singh. As per treaty,
Shivaji handed over twenty-three forts to the Mughals
(b) Allauddin Khilji which had an annual income of 40 lakh Huns. Shivaji
(c) Mohammad Bin Tughlaq had promised Mughals to provide military help against
(d) Firozshah Tughlaq Bijapur province.
UP RO/ARO Special (Mains) Exam 2010 238
CLICK HERE FOR FREE MATERIAL

10. Which of the following pairs is not matched 14. The Ancient Monument Conservation Act, was
correctly? passed during which Governor General
(a) Battle of Buxar -1764 tenure?
(b) Subsidiary Alliance-1863 (a) Lord Minto (b) Lord Harding
(c) Monopoly of East India Company over (c) Lord Curzon (d) Lord Linlithgo
India - 1833 Ans. (c) : Lord Curzon had a passion for preserving the
(d) Opening of trade market for British citizens ancient monuments of historical importance in India.
and companies - 1813 He passed a law called the Ancient Monuments Act,
Ans. (b) : Subsidiary alliance was introduced in 1798 1904 which made it obligatory on the part of the
by Lord Wellesley, not in 1863. He was also the government and local authorities to preserve the
governor general of India from 1798-1805. The first to monuments of archaeological importance.
accept the Subsidiary Alliance in 1798 was the Nizam 15. The secret organization established by V.D.
of Hyderabad. The Subsidiary Alliance was a treaty Savarkar was:
between the British East India Company and the (a) Yugantar Samiti
princely Indian states, which forced the Indian (b) Anusheelan Samiti
kingdoms to surrender their authority to the English. (c) Hindustan Republican Association
11. Which one of the following was the first (d) Abhinav Bharat
newspaper published in India? Ans. (d) : Abhinav Bharat Society was a secret society
(a) The Bengal Gazette founded by Vinayak Damodar Savarkar and his brother
(b) The Kolkata Gazette (Calcutta) Ganesh Damodar Savarkar in 1904.
(c) Madras Courier 16. Who among the following had recommended to
(d) Bombay Herald conduct the Public Services Exam in India and
Ans. (a) : The Bengal Gazette also known as Calcutta England together?
General Advertiser was India's first newspaper published (a) Etchison Commission
on 29 January,1780 by James Augustus Hicky. (b) Hobhouse Commission
12. Brahma Samaj was established by: (c) Montague Chelmsford Report
(a) Rajaram Mohan Roy (d) Lord Cornawallis
(b) Devendranath Tagore Ans. (c) : Montague Chelmsford Report had
(c) Ishwar Chandra Vidyasagar recommended to conduct the Public Services Exam in
(d) Keshavchandra Sen India and England together. Civil Servants for the East
Ans. (a): Brahma Samaj was started at Calcutta on 20 India Company used to be nominated by the Directors
August 1828 by Raja Ram Mohan Roy. He established of the Company and thereafter trained at Haileybury
Brahmo Samaj as the reform movement influenced by College in London and then sent to India. Following
Modern and western ideas. He believed in western Lord Macaulay’s Report of the Select Committee of
ideas, human dignity principles, and social equality. British Parliament, the concept of a merit based modern
13. Match List-I with List-II and choose the Civil Service in India was introduced in 1854.
correct answer using the codes given below: 17. Match List-I with List-II and select the correct
List-I List-II answer using the codes given below:
(A) Indian Council Act, 1. Beginning of List-I List-II
1909 Diarchy (Movement) (Persons
system known for)
(B) Government of 2. 1916 (A) HomeRule 1. M.K. Gandhi
India Act, 1935 Movement
(C) Lucknow Pact 3. Lord Minto (B) Bardoli Satyagrah 2. C.R. Das
(D) Government of 4. Provincial (C) Non-Cooperation 3. Vallabh Bhai
India Act, 1919 Autonomy Movement Patel
Code : (D) Formation of 4. Annie Besant
A B C D Swaraj Party
(a) 1 2 3 4 Code :
(b) 2 4 1 3 A B C D
(c) 2 4 1 3 (a) 1 2 3 4
(d) 3 4 2 1 (b) 4 3 1 2
Ans. (d) : The correct match is as follows: (c) 4 3 2 1
List–I List-II (d) 2 1 4 3
(a) Indian Council Act,1909 – Lord Minto Ans. (b) : The correct match is as follows:
(b) Government of India Act, 1935 – Provincial List- I List- II
autonomy (a) Home rule Movement - Annie Besant
(c) Lucknow Pact – 1916 (b) Bardoli Satyagarh - Vallabh Bhai Patel
(d) Government of India Act, 1919 – Beginning of (c) Non-Cooperation Movement - M.K. Gandhi
Diarchy system (d) Formation of Swaraj Party - C.R. Das
UP RO/ARO Special (Mains) Exam 2010 239
CLICK HERE FOR FREE MATERIAL

18. At the time of First World War where was 22. Which one of the following was not included
Provisional government was formed under the the in resolutions passed at the Lahore session
presidentship of King Mahendra Pratap? of Indian National Congress?
(a) Germany (b) Singapore (a) Announcement of Indian foreign policy.
(c) Afghanistan (d) Turkey (b) Announcement of aim of complete
Ans. (c) : On 1 December 1915, Mahendra Pratap independence.
established the first Provisional Government of India at (c) Preparation to start the civil disobedience
Kabul in Afghanistan as a government-in-exile of Free movement.
Hindustan, with himself as President, Maulavi (d) Abolition of untouchability.
Barkatullah as Prime Minister, and Maulana Ubaidullah Ans. (d): Abolition of untouchability was not included
Sindhi as Home Minister, declaring jihad on the British. in the resolutions passed at the Lahore session of Indian
19. The Slogan - "Inquilab Zindabad" is associated National Congress. Rests are included in the resolutions
with: passed at the Lahore session of Indian National
(a) Chandrasekhar Azad (b) Ramprasad Bismil Congress in 1929.
(c) Sardar Bhagat Singh (d) Lala Hardayal 23. Which of the following pair of islands is
Ans. (c) : The slogan – ‘Inquilab Zindabad’ is separated by 10 degree channel?
associated with Sardar Bhagat Singh. Inquilab (a) Lakshadweep and Minicoy
Zindabad, can be translated as “Long Live Revolution” (b) South Andaman and Little Andaman
was coined by Moulana Hasrat Mohani in the year (c) Andaman and Nicobar
1921. It was one of the most famous slogans during the (d) Pamban and Mannar
Indian freedom struggle. It was used by Shahid-e-Azam Ans. (c) : The Ten Degree(100) Channel is a channel
Bhagat Singh through his speeches and writings in the that separates the Little Andaman and Car Nicobar in
early 1920s. the Bay of Bengal.
20. After the announcement of ‘Communal 24. Glaciations of peninsular India occurred
Award’, which one of the following had been during which of the following ice age?
declared? (a) Huronian Ice age
(a) Lucknow Pact (b) Karachi Pact (b) Carboniferrous Ice age
(c) Lahore Pact (d) Poona Pact (c) Pleistocene Ice age
Ans. (d) : In 1932, B.R. Ambedkar negotiated the (d) Little Ice age
Poona Pact with Mahatma Gandhi. The background to Ans. (c) : Glaciations of peninsular India occurred
the Poona Pact was the Communal Award of August during Pleistocene Ice age. The Pleistocene is the
1932 which provided a separate electorate for depressed geological epoch that lasted from about 2,580,000 to
classes. Mahatma Gandhi was bitterly opposed to the 11,700 years ago, spanning the Earth's most recent
communal award, as the award in continuum of period of repeated glaciations.
constant efforts of the British imperialists to divide
25. Which of the following pairs is correctly
Indian people into a number of special-interest groups
matched?
and to weaken the national movement. Initially, Ambedkar
was in favour of the award, as according to him political Lake Location
solutions like separate electorate would work for (a) Lonar - Madhya Pradesh
upliftment of depressed classes. However, after the series (b) Nakki - Gujarat
of negotiations both Gandhiji and Ambedkar agreed to a (c) Koleru - Andhra Pradesh
solution called Poona Pact and thereby withdrawing (d) Pulicat - Kerala
separate electorate for depressed classes. Ans. (c): Kolleru Lake is one of the largest freshwater
21. Who among the following advocated on behalf lakes in India located in state of Andhra Pradesh and
of the officers of the Azad Hind Fauj in their forms the largest shallow freshwater lake in Asia while
ongoing trial at the Red Fort? Pulicat lake is saltwater lagoon on the Coromandel
Coast of Andhra Pradesh. Nakki Lake is a very ancient
(a) C.R.Das (b) Motilal Nehru sacred lake located in Mount Abu, Rajasthan while
(c) M.A. Jinnah (d) T.B Sapru Lonar Lake, also known as Lonar crater, is a notified
Ans. (d) : Eminent lawyer Tez Bahadur Sapru had National Geo-heritage Monument, saline, soda lake,
advocated on behalf of the officers of the Azad Hind located at Lonar in Buldhana district, Maharashtra,
Fauj in their ongoing trial at the Red Fort. Bhulabhai India. Lonar Lake is an astrobleme created by a
Desai, Tej Bahadur Sapru, and Jawahar Lal Nehru were meteorite impact during the Pleistocene Epoch. Thus,
the advocates appointed by the Indian National option (c) is correctly matched.
Congress. The British government charged the officials 26. Which one of the following has been the largest
of the Indian National Army with injustice. The
coffee producing state of India?
procedures of their preliminary trial started before the
military council held in Red Fort of Delhi. Pandit (a) Maharashtra (b) Karnataka
Jawaharlal Nehru, Bhulabhai Desai, Tej Bahadur Sapru, (c) Tamil Nadu (d) Kerala
and other recognized legal scholars defended them. The Ans. (b) : Largest coffee growing state in India is
military court held them blameworthy and they were Karnataka. Karnataka accounts for nearly 71% of total
condemned to life detainment. coffee production in India.
UP RO/ARO Special (Mains) Exam 2010 240
CLICK HERE FOR FREE MATERIAL

27. Match List-I with List-II and select the correct Reason (R) : In Equatorial region, the average
answer from the codes given below:- annual temperature variation is very nominal.
List-I List-II Codes:
(Coal Region) (State) (a) Both (A) and (R) are true and (R) is the
correct explanation of (A).
(A) Karanpura 1. Chattisgarh
(b) Both (A) and (R) are true but (R) is not the
(B) Singareni 2. Tamil Nadu correct explanation of (A).
(C) Neyveli 3. Jharkhand (c) (A) is true but (R) is false.
(D) Korba 4. Andhra (d) (A) is false but (R) is true.
Pradesh Ans. (b) : Equatorial region experiences high
Code : temperatures all year round. The average monthly
A B C D temperatures are about 26 – 28 degrees Celsius.
(a) 4 1 3 2 Humidity is usually very high. In Equatorial region, the
average annual temperature variation is very nominal.
(b) 1 2 4 3
These regions usually experience 200 mm of rainfall or
(c) 3 4 2 1 more in a year. Rainfall is high for most of the year. In
(d) 2 3 1 4 equatorial regions, the year round high temperatures and
Ans. (c) : The correct match is as follows: abundant rainfall support plant growth all year round.
List – I List- II These forests have very dense vegetation. However for
(Coal Region) (State) humans these climatic condition are not very favorable.
Thus, both (A) and (R) are true but (R) is not the correct
(a) Karanpura Jharkhand
explanation of (A).
(b) Singareni Andhra Pradesh(Now in Telangana)
31. Grand Bank is located on the -
(c) Neyveli Tamil Nadu
(a) Western coast of North America.
(d) Korba Chattisgarh (b) West European Coast
28. Match List-I with List-II and select the correct (c) Eastern Coast of North America.
answer using the codes given below: (d) East Coast of Africa.
List-I List-II Ans. (c) : Grand Bank is located on the eastern coast of
(Oil Refinery) (State) North America. The Grand Banks are one of the richest
(A) Numaligarh 1. Bihar fishing grounds in the world. It is situated where cold
(B) Tatipaka 2. Gujarat current Labrador meets warm water of Gulf stream. The
mixing of these waters and the shape of the ocean
(C) Koyali 3. Andhra Pradesh bottom lifts nutrients to the surface. These conditions
(D) Barauni 4. Asam created one of the richest fishing grounds in the world.
Code: 32. Rukwa Lake region (Tanzania) is famous for
A B C D the production of:
(a) 1 4 3 2 (a) Mica (b) Coal
(b) 2 1 4 3 (c) Iron Ore (d) Gold
(c) 3 2 1 4 Ans. (b) : Rukwa Lake region (Tanzania) is famous for
(d) 4 3 2 1 the production of coal. Mining is a leading industrial
Ans. (d) : The correct match is as follows: sector in Tanzania with the value of mineral exports
constantly increasing for the past several years. The
List – I List – II sector is comprised of both small- and large-scale
(Oil Refinery) (State) operations. Mining in Tanzania incudes metals (gold,
(a) Numaligarh Assam iron ore, nickel, copper, cobalt, silver), industrial
(b) Tatipaka Andhra Pradesh minerals (diamonds, tanzanite, ruby, garnet, limestone,
(c) Koyali Gujarat soda ash, gypsum, salt, phosphate, gravel, sand, dimension
(d) Barauni Bihar stones and graphite), and fuel minerals (coal, uranium).
Tanzania is also home to many rare earth and critical
29. Ozone layer is located in: minerals that are currently in the exploration stage.
(a) Troposhere (b) Stratosphere 33. Match List-I with List-II and select the correct
(c) Merosphere (d) Ionosphere answer using the codes given below :
Ans. (b) : About 90% of the ozone in the Earth's List-I List-II
atmosphere is found in the stratosphere. Ozone layer (Iron Ore Region) (Producing
absorbs UV rays, reflects the infrared radiation and does Country)
not reflect back radio waves.
(A) Loraine 1. Sweden
30. Consider the above statements and select the
(B) Midland 2. Kazakhstan
correct answer using the codes given below:
Statement (A) : The environment of Equatorial (C) Kiruna 3. U.K
region is compatible to plants, but not for human. (d) Kostane 4. France
UP RO/ARO Special (Mains) Exam 2010 241
CLICK HERE FOR FREE MATERIAL

Code : 38. As per the Census -2011, which of the following


A B C D Indian state has highest percentage of urban
(a) 4 2 3 1 population?
(b) 3 4 2 1 (a) Maharashtra (b) Goa
(c) 4 3 1 2 (c) Tamil nadu (d) Mizoram
(d) 1 2 3 4 Ans. (b) : As per the Census -2011, Goa has highest
Ans. (c) : The correct match is as follows: percentage of urban population. The urban population
List–I List-II of Goa is 62.20%. As per the Census -2011, the urban
(Iron-ore Region) (Producing Country) population of Maharashtra, Mizoram and Tamil Nadu is
(a) Loraine France 45.2%, 52.11% and 48.45 % respectively. The urban
(b) Midland United Kingdom population of Delhi is 97.5%.
(c) Kiruna Sweden 39. Which one of the following year decadal
(d) Kostane Kazakhstan population growth rate has been negative?
34. Which of the following statements is not true? (a) 1901 (b) 1911
(a) Osaka is known as the Manchester of the (c) 1921 (d) None of these
East. Ans.(c) : Decadal population growth rate has been
(b) All the Iron-steel industries of Japan are negative in the year 1921. In the census year 1911-
located in the southern coastal region. 1921 registered a negative growth rate of-0.31 per cent
(c) The North Honshu region of Japan is famous which happened only once throughout the demographic
for Cotton Textile Industry. history of India. The year 1921 is often referred to as
(d) In East Asia, the Japanese ship manufacturing the “Year of the Great Divide,” the last period in which
industry is ahead. India’s population declined, due to the effects of
famine. The growth rate of population substantially
Ans. (b): The iron and steel industry of Japan is mainly increased after independence from British rule going up
concentrated in the Chukyo region, Osaka - Kobe,
Fukuoka-Yamaguchi, Oka-Yamaha and Hokkaido to 2.2% during 1961-1981.
region contributes about 20 per cent of the Japanese 40. As per the United Nations estimates, the
steel production. Major cities in where steel industries tentative population of the world by 2050 will be:
based are Kobe, Osaka and Kitakyushu. The regions are (a) 9.0 billion (b) 9.25 billion
located different directions in all over Japan, so, this (c) 8.92 billion (d) 10 billion
statement is wrong that all the Iron-steel industries of Ans. (a) : When this question was asked, the United
Japan are located in the southern coastal region. Rest Nations Estimates had projected tentative population of
statements are correct. the world 9.08 billion in 2050. The World Population
35. Which of the following state is famous for Prospects, published by the United Nations Department
Lapps Tribal community? of Economic and Social Affairs, provides a
(a) Alaska Scandinavia (b) Scandinavia comprehensive review of global demographic trends
(c) Greenland (d) Iceland and prospects for the future. The information is
Ans. (b) : Scandinavia region is famous for Lapps Tribal essential to guide policies aimed at achieving the new
community. The Lapps is the only indigenous people Sustainable Development Goals. The current world
living in the northern parts of Finland, Norway, Sweden population of 7.6 billion is expected to reach 8.6 billion
and Russia. Eskimos tribe is found majorly in Alaska (of in 2030, 9.8 billion in 2050 and 11.2 billion in 2100,
the United States), sub-arctic region of Greenland, Canada, according to a new United Nations report.
eastern Siberia (Russia) and the Arctic. 41. According to the Census 2011, the correct
36. As per Census, 2011, the percentage of urban ascending order of the population density of
population in India was - following states is -
(a) 27.80 (b) 31.16 (a) Assam, Haryana, Gujarat, Karnataka
(c) 32.00 (d) 32.16 (b) Assam, Gujarat, Karnataka, Haryana
Ans. (b): As per Census, 2011, the percentage of urban (c) Haryana, Assam, Gujarat, Karnataka
population in India is 31.16 and the percentage of rural (d) Gujarat, Karnataka, Assam, Haryana
population in India is 68.84. Ans. (d): According to the Census 2011, the correct
37. The aim of Nation Population Policy 2000, is to ascending order of the population density of above
stabilize the national population by the year: states is Gujarat (308), Karnataka (319), Assam (398)
(a) 2045 (b) 2040 and Haryana (573).
(c) 2050 (d) 2035 42. According to the Census-2011, which of the
Ans. (a) : The aim of Nation Population Policy 2000, is following state has maximum population of
to stabilize the national population by the year 2045. Scheduled Tribes?
The National Population Policy 2000 , released on (a) Madhya Pradesh (b) Maharashtra
February 15 that aims to bring the total fertility rate (c) Odissa (d) Gujarat
(TFR) to replacement level by 2010 and to achieve a Ans. (a): Madhya Pradesh has the largest Tribal
stable population by 2045, at a level consistent with population. It has more than 1.6 crore Scheduled Tribe
sustainable economic growth, social development, and population who is 21% of the state population according
environmental protection. to the census-2011.
UP RO/ARO Special (Mains) Exam 2010 242
CLICK HERE FOR FREE MATERIAL

43. Provisions related to Citizenship are provided 48. Under which article of constitution the
in which part of constitution? examination/investigation of parliamentary
(a) Part-1 (b) Part-3 proceedings by courts has been restricted?
(c) Part-2 (d) Part-4 (a) Article-127 (b) Article-122
Ans. (c) : Part II of the Constitution of India (Articles 5- (c) Article-26 (d) Article-139
11) deals with the Citizenship of India. Article 5 speaks Ans. (b) : Under article 122 article of constitution, the
about the citizenship of India at the commencement of examination/investigation of parliamentary proceedings
the Constitution (Nov 26, 1949). Article 11 gave powers by courts has been restricted. Article 122 deals with the
to the Parliament of India to regulate the right of restrictions on courts not to inquire into proceedings of
citizenship by law. Parliament.
44. "No person will be prosecuted and sentenced 49. By which of the following acts, the
for a single crime more than once". In terms of Fundamental right to property was abolished?
proving guilty for crimes, this above protection (a) 40th Constitutional Amendment Act
has been provided under which article? (b) 42th Constitutional Amendment Act
(a) Article-19 (b) Article-20 (c) 44th Constitutional Amendment Act
(c) Article-21 (d) Article-22 (d) 46th Constitutional Amendment Act
Ans. (b) : ‘No person will be prosecuted and sentenced Ans. (c) : In 1978, the 44th amendment to the
for a single crime more than once’. In terms of proving Constitution removed the right to property from the list
guilty for crimes, this above protection has been of fundamental rights and converted it into a simple
provided under article 20. Article 20 is designed to legal right under article 300 A.
protect the people against the excess of the legislature, 50. Which article of constitution provides
the judiciary and the executive respectively. These guarantee of one-third representation of
protections are available to both citizens and foreigners women in village Panchayats?
for criminal cases only. (a) Article-243-C (b) Article-243-D
45. Which of the following article has been made (c) Article-243-H (d) Article-243-I
provisions for the representation of Anglo- Ans. (b) : Article-243-D of constitution provides
Indian community in the Lok- Sabha? guarantee of one-third representation of women in
(a) Article-331 (b) Article-221 village Panchayats. Clause (3) of Article 243D of the
Constitution ensures participation of women in
(c) Article-21 (d) Article-139
Panchayati Raj Institutions by mandating not less than
Ans. (a) : Article -331 has been made provisions for the one- third reservation for women out of total number of
representation of Anglo- Indian community in the Lok- seats to be filled by direct election and number of
Sabha. Article 331 states that the President can offices of chairpersons of Panchayats.
nominate two members of the Anglo-Indian community 51. Match List-I with List-II and select the correct
to the Lok Sabha, if the community is not adequately answer using the codes given below :
represented. 104th Constitutional Amendment removed
List-I List-II
the reserved seats for the Anglo-Indian community in
the Lok Sabha and state assemblies. (A) Economic growth 1. Gross Domestic
Product
46. Which article of Indian Constitution has
special provisions about finance bills? (B) Economic rise 2. Environment
(a) Article-117 (b) Article-119 (C) Sustainable 3. Health
Development
(c) Article-121 (d) Article-123
(D) Quality of Life 4. Structural
Ans. (a) : Article-117 of Indian Constitution has special change
provisions about finance bills. Article 119 of the
constitution states about the regulation by law of Code :
procedure in Parliament in relation to financial A B C D
business. Article 123 explains about power of President (a) 1 2 3 4
to promulgate ordinances during recess of Parliament. (b) 4 2 3 1
Article 121 of the Indian Constitution mentions the (c) 3 4 1 2
restriction on the discussion in Parliament. (d) 4 1 2 3
47. Which article of Constitution has been made Ans. (d) : The correct match is as follows:
provisions for constituting National List–I List–II
Commission for Scheduled Tribes? A . Economic growth – Structural change
(a) 338-A (b) 337 B . Economic rise – Gross domestic Product
(c) 338 (d) 339 C . Sustainable development – Environment
Ans. (a) : The National Commission for Scheduled D . Quality of Life – Health
Tribes (NCST) was established by amending Article 52. The basis of the poverty line determination in
338 and inserting a new Article 338A in the India is :
Constitution through the Constitution (89th (a) Income statistics (b) Saving data
Amendment) Act, 2003. (c) Consumption data (d) Investment data
UP RO/ARO Special (Mains) Exam 2010 243
CLICK HERE FOR FREE MATERIAL

Ans. (c) : Poverty line is measured in terms of per Code:


capita consumption expenditure over a month in India. A B C D
The poverty ratio in India has been measured from an (a) 1 3 2 4
exogenously determined poverty line quantified in (b) 2 4 3 1
terms of per capita consumption expenditure over a (c) 3 2 1 4
month and the class distribution of persons obtained (d) 4 1 2 3
from the large sample survey of consumer expenditure Ans. (d) : The correct match is as follows:
data of the National Sample Survey Office (NSSO).
List- I List – II
53. The aim of Rajiv Awas Yojana is: a. ICICI 1955
(a) Arrangement/provision of homes for the rural b. IDBI 1964
poor. c. IFCI 1948
(b) Slum free India. d. SIDBI 1990
(c) Provision of hostels for poor students.
56. Which of the following is covered by the Kisan
(d) Provision for night shelters in villages. Credit Card Scheme?
Ans.(b) : Rajiv Awas Yojana (RAY) envisaged a “Slum (a) Consumption credit and Investment credit
Free India” with inclusive and equitable cities in which (b) Only Consumption credit.
every citizen would have access to basic civic (c) Only Investment credit.
infrastructure, social amenities and decent shelter. (d) None of the above.
Government of India announced the Rajiv Awas Yojana
in 2009. It focused on giving property rights to the slum Ans. (a) : The Kisan Credit Card (KCC) scheme was
dwellers and urban poor. introduced in 1998 for issue of Kisan Credit Cards to
farmers on the basis of their holdings for uniform
54. Which among the following controls the credit adoption by the banks so that farmers may use them to
creation by commercial banks in India? readily purchase agriculture inputs such as seeds,
(a) Finance Ministry fertilizers, pesticides etc. and draw cash for their
(b) Reserve Bank of India production needs. The scheme was further extended for
(c) Government of India the investment credit requirement of farmers viz. allied
(d) State Bank of India and non-farm activities in the year 2004. KCC covers
Ans. (b) : Reserve Bank of India controls the credit post-harvest expenses, produce marketing loan,
creation by commercial banks in India. Being a central consumption requirements of farmer household,
working capital for maintenance of farm assets and
bank of India, RBI serves a critical role in regulating the
activities allied to agriculture, investment credit
financial transactions in the country. Some of the
requirement for agriculture and allied activities.
important functions of RBI are listed below:
57. The name of Arthur Dunkel is associated with-
• Issue of Bank Notes
(a) IMF (b) GATT
• Banker to the Government
(c) O.C.G.C (d) EXIM Bank
• Custodian of the Cash Reserves of Commercial
Banks Ans. (b) : The name of Arthur Dunkel is associated
with GATT. The General Agreement on Tariffs and
• Custodian of country’s forex reserves Trade (GATT)—signed on Oct. 30, 1947, by 23
• Lender of last resort countries—was a legal agreement minimizing barriers
• Controller of credit to international trade by eliminating or reducing quotas,
55. Match List-I with List-II and choose the tariffs, and subsidies while preserving significant
correct answer using the codes given below : regulations.
List-I List-II 58. Which one among the following is not included
(Institution) (Year of among the main 3 pillars of 'Agreement on
Establishment) Agriculture' under the World Trade
Organization?
(A) Industrial Credit 1. 1964 (a) Market Access
and Investment
(b) Internal Support
Corporation of
India (ICICI) (c) Export Competition
(d) Anti Dumping and Compensatory octroi
(B) Industrial 2. 1948
Development Bank Ans. (d) : Anti-dumping and compensatory octroi is
of India. (IDBI) not included among the main 3 pillars of 'Agreement on
Agriculture' under the World Trade Organization. The
(C) Industrial Finance 3. 1990 Agreement establishes a number of general rules and
Corporation of commitments, mainly in three areas sometimes called
India (IFCI) the “three pillars”. These are: market access, domestic
(D) Small Industries 4. 1955 support and export competition (which covers export
Development Bank subsidies and export-related measures with equivalent
of India (SIDBI) effect).
UP RO/ARO Special (Mains) Exam 2010 244
CLICK HERE FOR FREE MATERIAL

59. Alternating current is converted into direct 64. Which of the following chemical is used for the
current by: preservation of food substances?
(a) Dyanamo (b) Motor (a) Sodium Chloride (b) Sodium Benzoate
(c) Transformer (d) Rectifier (c) Sodium Tartarate (d) Sodium Acetate
Ans. (d) : Alternating current is converted into direct Ans. (b) : Sodium Benzoate is used for the preservation
current by rectifier. A rectifier is an electronic device of food substances. Sodium benzoate is used to prevent
used for converting alternating current to direct current spoilage from harmful bacteria, yeasts, and molds. It
with the help of one or more p-n junction diodes. It acts also helps maintain freshness in food by helping to slow
as a one-way valve allowing the current to flow in one or prevent changes in color, flavor, pH, and texture.
direction only. This process by which a diode acts as a 65. Which chemical compound is present in
rectifier is called rectification. Bleaching powder?
60. The minimum height of plane mirror for seeing (a) Calcium Hydroxide
the full image of a person is: (b) Calcium Oxychloride
(a) Equal to person's height (c) Calcium Carbonate
(b) Half of a Person's height (d) Ammonium Chloride
th
1 Ans. (b) : Calcium Oxy-chloride is the chemical
(c) of a persons's height
4 compound present in bleaching powder. The chemical
(d) None of them formula of bleaching powder is CaOCl2. Materials used
for the preparation of bleaching powder are calcium
Ans. (b) : In order to see full image of a person, the
hydroxide and chlorine. It is used for disinfecting
minimum size of the mirror should be half of the person's
drinking water supply.
height. This is so because, in reflection, the angle of
incidence is equal to angle of reflection. So, to see the 66. Which of the following essential oil is found in
image of a 6 ft tall person, a 3 ft long mirror is required. clove?
61. Inside the earth, the gravitation water remains (a) Menthol (b) Eugenol
at what pressure - (c) Methanol (d) Benzildihyde
(a) 1/3 Atmosphere (b) 1.25 Atmosphere Ans. (b): Eugenol is the essential oil found in clove.
(c) 5 Atmosphere (d) 15 Atmosphere Cloves are the dried flowers of the clove tree. Roughly,
89% of the clove essential oil is eugenol and 5% to 15%
Ans. (a) : Inside the earth, the gravitation water remains
at 1/3 atmospheric pressure. is eugenol acetate and β-cariofileno.
62. If Va, Vw and Vs are the velocity of sound in air, 67. Azolla is a ____
water and steel respectively then - (a) Aquatic Fern (b) Aquatic Algae
(a) Va<Vw<Vs (b) Vs<Vw<Va (c) Aquatic Bacteria (d) None of them
(c) Vw<Vs<Va (d) Vs<Va<Vw Ans. (a) : Azolla is an aquatic fern. It is used as a bio-
Ans. (a) : If Va, Vw and Vs are the velocity of sound fertilizer in a paddy field due to its ability to fix the
in air, water and steel respectively. Then it would be atmospheric nitrogen. The aquatic fern Azolla is used as
Va<Vw<Vs. The speed of the sound depends on the a natural plant-factory for ammonia removal from fish-
density and the elasticity of the medium through which breeding fresh wastewater.
it travels. In general, sound travels faster in liquids than 68. Vitamin 'C' helps in the absorption of:
in gases and quicker in solids than in liquids. The (a) Iron (b) Calcium
greater the elasticity and the lower the density, the faster (c) Iodine (d) Sodium
sound travels in a medium. Ans. (a) : Vitamin 'C' helps in the absorption of Iron.
63. Which of the following is not an allotrope of Ascorbic acid facilitates iron absorption by forming a
Carbon? chelate with ferric iron at acid pH that remains soluble at
(a) Diamond (b) Graphite the alkaline pH of the duodenum. Vitamin C is a powerful
(c) Oxocarbons (d) Fullerenes aid in absorption of iron from non-heme sources.
Ans. (c) : Oxocarbons is not an allotrope of Carbon, but 69. The hormone, controls the excretion of urine
diamond, graphite and fullerences are allotropes of from Kidneys, is:
Carbon. Diamond is extremely hard, transparent crystal, (a) TSH (b) ACTH
with the carbon atoms arranged in a tetrahedral lattice. (c) FSH (d) ADH
This allotrope of carbon is a poor electrical conductor Ans. (d) : The hormone that controls the excretion of
and an excellent thermal conductor. Graphite is a soft, urine from kidneys, is Anti-Diuretic Hormone (ADH).
black, flaky solid, a moderate electrical conductor. The Anti-diuretic hormone (ADH) is a chemical produced in
carbon atoms are bonded in flat hexagonal lattices the brain that causes the kidneys to release less water,
(graphene), which are then layered in sheets. Fullerenes, decreasing the amount of urine produced. A high ADH
including Buckminsterfullerene, are also known as level causes the body to produce less urine. A low level
“buckyballs”, such as C60. results in greater urine production.
UP RO/ARO Special (Mains) Exam 2010 245
CLICK HERE FOR FREE MATERIAL

70. Which one of the following is associated with 75. The intensity of Maize-potato-sugarcane-
Blue revolution? paddy-wheat is:
(a) Animal (b) Bees (a) 133% (b) 166%
(c) Fish (d) Hen (c) 200% (d) 250%
Ans. (c) : Fish is associated with Blue revolution. India Ans. (b) : The soil intensity of Maize-Potato-
is the second largest fish producer country in the world. Sugarcane-Paddy-Wheat is 166%.
It is notable that milk is associated with White 76. The most suitable variety of Piegon Pea
revolution, oilseeds to Yellow revolution and meat to
(Arhar) for double cropping system with wheat
Pink Revolution & tomato to Red revolution.
is -
71. Which one among the following is not one of the (a) N.A-I (b) Bahar
objectives of ‘National Horticulture Mission’?
(c) U.P.A.S 120 (d) None of these
(a) To provide financial help.
Ans. (c) : The most suitable variety of Piegon Pea
(b) To achieve high growth in horticulture field.
(Arhar) for double cropping system with wheat is
(c) Post Harvest Management.
UPAS-120.
(d) Human Resource Development.
77. Nix and Hydra are the satellite of:
Ans. (a) : ‘To provide financial help’ is not one of the
objectives of ‘National Horticulture Mission’. After the (a) Pluto (b) Venus
launch of National Horticulture Mission (NHM) in (c) Jupiter (d) Mercury
2005-06, significant progress has been made in area Ans. (a) : On 11 July 2012, a pair of small moons that
expansion under horticulture crops resulting in higher NASA's Hubble Space Telescope discovered orbiting
production. The objectives of National Horticulture Pluto now have official names: Nix and Hydra.
Mission are to increase the growth of the horticulture Photographed by Hubble in 2005, Nix and Hydra are
sector by conducting research, technology promotion, roughly 5,000 times fainter than Pluto and are about two
post-harvest management, processing and marketing to three times farther from Pluto than its large moon,
etc., to improve horticulture production, enhance Charon, which was discovered in 1978.
nutritional security and double the income of farmers. 78. Which of the following is not correctly
72. Life Saving or Safe Irrigation indicates - matched?
(a) Continuous water flow. (a) Space Application Centre - Ahmedabad
(b) Irrigation at critical growth stage. (b) National Remote Sensing Centre - Hyderabad
(c) P.W.P Irrigation. (c) Indian Space Research Organisation Satellite
(d) First hydrate then dehydrate. Centre - Sriharikota
Ans. (b) : Life saving or safe irrigation indicates (d) Vikram Sarabhai Space Centre -
irrigation at critical growth stage. The stage at which Thiruvananthapuram
the water stress causes severe yield reduction is also Ans. (c) : The Secretariat of DOS and ISRO
known as critical stage of water requirement. It is also Headquarters are located at Antariksh Bhavan in
known as moisture sensitive period. Bangalore while Satish Dhawan Space Centre (SDSC)
73. Sugarcane seeds are produced at: is located in Sriharikota. It is a satellite launching
(a) IIMR, Lucknow station of the Indian Space Research Organization. It
(b) IARI, New Delhi was renamed the Satish Dhawan Space Centre in the
(c) Seed Research Directorate, Man year 2002. Rests are correctly matched.
(d) S.B.I Coimbatore 79. In context of Bio-diversity, which of the
Ans. (d) : Sugarcane seeds are produced at (Sugarcane following region is considered as ‘Hotspot’?
Breeding Institute) S.B.I, Coimbatore. (a) Gangetic plains (b) Eastern Himalayas
74. How long does it take for Adsali sugarcane (c) Gujarat (d) Central India
crops to ripen? Ans.(b) : In context of Bio-diversity, eastern Himalayas
(a) 6 months (b) 12 months region is considered as ‘Hotspot’. Coined by Norman
(c) 18 months (d) 24 months Myers in the year 1988, the term “Biodiversity
Ans. (c) : Sugarcane take generally one year to mature hotspots” can be defined as the regions which are
in sub tropical and tropical states while when it matures known for their high species richness and endemism.
in 18 months , called 'Adsali'. It can grow in any type 80. The biggest source of air pollution in the
of soil as long as the soil can retain moisture. The soil metropolitan cities of India is -
should be rich in nitrogen, calcium and phosphorous.It (a) Thermal power segment
requires temperature between 20ºC and 24ºC. For (b) Industrial segment
ripening and harvesting, dry winter is ideal. It needs (c) Urban solid waste
rainfall of 100-155 cm distributed throughout the year. (d) Transport segment
UP RO/ARO Special (Mains) Exam 2010 246
CLICK HERE FOR FREE MATERIAL

Ans. (d) : The biggest source of air pollution in the 84. In Uttar Pradesh, red soil is mainly found in -
metropolitan cities of India is transport segment. Air (a) Sitapur-Barabanki (b) Etah-Mainpuri
pollution is contamination of the indoor or outdoor (c) Mirzapur-Jhansi (d) Agra-Mathura
environment by any chemical, physical or biological Ans. (c) : The red soil is mainly found in Mirzapur,
agent that modifies the natural characteristics of the Southern part of Allahabad, Sonbhadra, Jhansi, Banda,
atmosphere. In Indian cities, a major part of air Hamirpur and Chandauli. It is formed by weathering of
pollution is sourced, all year around, from burning of red sandstones rocks. Due to the presence of iron oxide,
diesel, petrol, gas, coal, biomass, and waste and re- the colour of soil is red. The parent materials of the red
suspended dust. soil are crystalline and metamorphic rocks like acid
81. Which one among the following is not included granites, gneisses and quartzite.
in the eight mission of Climate Action Plan
85. How much percentage of India's arable land is
envisaged by Government of India ?
located in Uttar Pradesh?
(a) Solar Power
(a) 20 (b) 15
(b) Atomic Energy
(c) 17 (d) 19
(c) Energy transformation from waste
(d) Forestation Ans. (d) : 19 percent of India's arable land is located in
Uttar Pradesh. Agriculture is the main occupation in
Ans. (b) : Atomic Energy is not included in the eight Uttar Pradesh. According to the Situation Assessment of
mission of Climate Action Plan envisaged by
Agricultural Households (2012–13), UP had 18 million
Government of India. The Government of India
agricultural households, which accounted for 20% of
launched National Action Plan on Climate Change
the total agricultural households in rural India. There is
(NAPCC) on 30thJune, 2008 outlining eight National
nine (9) agro-climatic zones in Uttar Pradesh and
Missions on climate change. These include: National
Solar Mission, National Mission for Enhanced Energy approx 41.1% of total working population depends on
Efficiency, National Mission on Sustainable Habitat, agriculture.
National Water Mission, National Mission for 86. The main commercial crop of Uttar Pradesh is:
Sustaining the Himalayan Eco-system, National (a) Cotton (b) Jute
Mission for a Green India, National Mission for (c) Oil seeds (d) Sugarcane
Sustainable Agriculture and National Mission on Ans. (d) : The main commercial crop of Uttar Pradesh is
Strategic Knowledge for Climate Change. Sugarcane. It can grow in any type of soil as long as the
82. In the January month of year 2012, what was soil can retain moisture. The soil should be rich in
the seasonal factor responsible for abnormal nitrogen, calcium and phosphorous. It requires temperature
cold wave in North India? between 20ºC and 24oC. For ripening and harvesting, dry
(a) Deforestation (b) Atmospheric winter is ideal. It needs rainfall of 100-155 cm distributed
(c) La Nina (d) EL-Nino throughout the year. It is notable that Uttar Pradesh ranks
Ans. (c) : La Nina was the seasonal factor responsible first in sugarcane, potato and milk production.
for abnormal cold wave in North India. The cold wave 87. The only open University of Uttar Pradesh is
ordinarily shows up from the west, through the Western located in:
Disturbance wind framework. This framework is (a) Jhansi (b) Gorakhpur
additionally answerable for causing downpours in (c) Allahabad (d) Agra
northern and northwestern parts, in the wake of having
gotten moisture on its way from the Mediterranean Ans. (c) : The only open University of Uttar Pradesh is
Sea.The effect of the cold weather additionally is located in Prayagraj ,i.e, Uttar Pradesh Rajarshi Tandon
caused by the measure of snowfall that occurs in Jammu Open University.
and Kashmir, Ladakh, Himachal Pradesh, and close by 88. Match the List-I with List-II and choose the
regions. The cold winds from the Tasmanian and correct answer using the codes given below :
Caspian Sea blow over the Northern part of India which List-I List-II
affects the coldness severely. (Industry) (Centre)
83. Which among the following is the most ancient (A) Wooden Toys 1. Meerut
university? (B) Sports goods 2. Bareilly
(a) Banaras Hindu University
(C) Brass Idols 3. Varanasi
(b) Allahabad University
(D) Match Industry 4. Mathura
(c) Chaudhary Charan Singh University
(d) Lucknow University Code :
A B C D
Ans. (b) : Among the given universities, Allahabad
University is the oldest university, established in 1887. The (a) 1 4 3 2
foundation of Banaras Hindu University was laid in 1916 (b) 3 2 1 4
while Lucknow University was established in 1921 and (c) 2 1 4 3
Chaudhary Charan Singh University was set up in 1965. (d) 3 1 4 2
UP RO/ARO Special (Mains) Exam 2010 247
CLICK HERE FOR FREE MATERIAL

Ans. (d) : The correct match is as follows: Ans. (a) : The correct match is as follows:
List-I List- II List-I List-II
(Industry) (Centre) A. Apparel Park Kanpur
a. Wooden toys Varanasi B. Biotechnology Park Lucknow
b. Sports goods Meerut C. Agro Processing Zone Saharanpur
c. Brass idols Mathura D. Leather Technology Park Unnao
d. Match Industry Bareilly 93. Dr. Ambedkar Institute of Technology for
89. According to Herbert Risley (1901), the Uttar Handicaped is located at -
Pradesh is native place of : (a) Agra (b) Allahabad
(a) Indo-Aryan Race (c) Kanpur (d) Lucknow
(b) Arya-Dravidian Race Ans. (c) : Dr. Ambedkar Institute of Technology for
(c) Mangoloid Race Handicaped is located at Kanpur.
(d) Scytho-Dravidian Race 94. Match List-I with List-II and select the correct
Ans. (b) : According to Herbert Risley (1901), the Uttar answer using the codes given below :
Pradesh is native place of Arya-Dravidian race. Risley List-I List-II
is perhaps most well known for his role as the architect (A) Sonbhadra 1. Non plastic fire clay
of the 1901 Census of India, and his subsequent (B) Allahabad 2. Copper
publication The People of India (1908), based on the
survey information gathered for the Census. He (C) Lalitpur 3. Silica sand
proposed a theory of the caste system as a racial (D) Mirzapur 4. Lime stone
hierarchy of classification, which was highly influential Code :
in colonial administrative policy. Risley argued that A B C D
caste was a system of social precedence deriving from a (a) 1 2 3 4
race-based hierarchy of social life. (b) 4 3 2 1
90. 'Karma' is a folk dance of: (c) 4 3 1 2
(a) Sonbhadra (b) Brijbhumi (d) 2 1 4 3
(c) Bundelkhand (d) None of these
Ans. (b) : The correct match is as follows:
Ans. (a) : ‘Karma’ is a folk dance of Sonbhadra district
List- I List- II
of Uttar Pradesh.
A. Sonbhadra Lime stone
91. Which of the following pairs is not correctly
B. Allahabad Silica sand
matched?
(a) Noida-Software and Information Technology C. Lalitpur Copper
(b) Sitapur - Plywood Industry D. Mirzapur Non-plastic fire clay
(c) Gorakhpur-Sportsgoods Manufacture Industry 95. Just as is related to likewise is
(d) Varanasi - Silk Industry associated to which of the following.
Ans. (c) : Gorakhpur of Uttar Pradesh is famous for
terracotta and pottery work. The terracotta work of (a) (b)
Gorakhpur is a traditional art form that transcends
centuries and potters make various animal figures like
horse, elephant, camel, goat, ox, etc. with hand-applied (c) (d)
ornamentation. So, option (c) is not correctly matched.
Merrut is famous for sports goods manufacturing Ans. (d) : Just as'
industry. Rests are correctly matched.
is related to
92. Match List-I with List-II and select the correct Same as,
answer using the codes given below :
List-I List-II
is related to
(A) Apparel Park 1. Unnao
So, option (d) is correct
(B) Biotechnology Park 2. Saharanpur
96. What is the substitute letter for the questions
(C) Agro Processing Zone 3. Kanpur
mark (?)
(D) Leather Technology 4. Lucknow
Park
Code :
A B C D
(a) 3 4 2 1
(b) 4 3 1 2
(c) 2 1 3 4 (a) R (b) N
(d) 3 2 4 1 (c) M (d) L
UP RO/ARO Special (Mains) Exam 2010 248
CLICK HERE FOR FREE MATERIAL

Ans. (c) : The letter given in the quadrilateral are the Ans. (a) : According to the question –
sum of the number in front of them. x +x +x
1 2 3 = 14 ... (i )
3 + 2 = 5 (E) 9 + 8 = 17 = Q
3
7 + 6 = 13 (M) 5 + 4 = 9 = (I) and (x1 + x2)×2 = 30
30
x1 + x2 = = 15
97. If is realted to then will 2
15 + x 3
be related to which of the following figure. then = 14
3
x3 = 42 – 15
(a) (b) x3 = 27
102. If two numbers are in the ratio of 5:6 and its
L.C.M is 660, then what is the smaller number
out of these two numbers ?
(c) (d) (a) 22 (b) 165
(c) 132 (d) 110
Ans. (a) : option (a) is correct. Ans. (d) : Let the number are
5x and 6x
98. If C P I is D M K then B J P H.C.F. of number = x
will be - first number × IInd number = HCF × LCM
5x × 6x = x × 660
(a) C G S (b) C R G
30x2 = 660x
(c) C G R (d) C S G 30x = 660
x = 22
Ans. (c) : Just as So
First number = 5x
= 5 × 22
= 110
Same as, and second number = 6x
= 6 × 22
= 132
103. In the series 462, 420, 380, x, 306 the value of x
is -
So BJP = CGR (a) 342 (b) 332
99. A Statement whose meaning is not (c) 322 (d ) 312
implicit/clear, is known as - Ans. (a) : The sequence of number is as follows –
(a) Misleading (b) Dubious 462 – 42 = 420
(c) Erroneous (d) Unclear or illusive 420 – 40 = 380
Ans. (d) : A statement whose meaning is not clear is 380 – 38 = 342
known as – unclear or illusive 342 – 36 = 306
100. In the series comprising of following paired So, option (a) is correct.
numbers, which one paired number is wrong ? 104. If the sum of two numbers is 10 and the sum of
6-1, 7-2, 8-6, 9-24, 10-120, 11-620 their squares is 52, then what are these two
numbers ?
(a) 8-6 (b) 9-24
(a) 3 and 7 (b) 4 and 6
(c) 11-620 (d) 10-120 (c) 2 and 8 (d) 9 and 1
Ans. (c) : The first order of series is sequential while Ans. (b) : Let the numbers are
the second order is in cryptic order. a, and b
6, 7, 8, 9, 10, 11 a + b = 10
1, 2, 6, 24, 120, 720 a2 + b2 = 52
Such that the wrong pair in the series of these number is 2ab = (a + b)2 – (a2 + b2 )
11 – 620. = 102 – 52
101. If average of x1, x2 & x3 is 14 and the two times = 100 – 52
of the sum of x1 and x2 is 30 then what will be = 48
the value of x3 ? (a – b)2 = (a2 + b2) – 2ab
(a) 27 (b) 16 = 52 – 48 = 4
(c) 12 (d) 20 (a – b) = 2
UP RO/ARO Special (Mains) Exam 2010 249
CLICK HERE FOR FREE MATERIAL

So, 108. The India Resident Mission of Asian


a + b = 10 ...(i) Development Bank is located in:
a–b=2 ...(ii) (a) Mumbai (b) New Delhi
by solving eq (i) and (ii) (c) Kolkata (d) Bengaluru
a = 6, b = 4 Ans. (b) : The Asian Development Bank is a regional
105. If in the coded language the word KAMAL is development bank established on 19 December 1966
written as 21413 then how will the word whose main objective is to promote social and
MAHAL written in the same code language ? economic development in Asia. They have opened its
(a) 48113 (b) 43811 residential office in New Delhi to promote Indian
(c) 41823 (d) 38141 Resident Mission (INRM).
Ans. (c) : Just as,
109. Who among the following has been selected for
the Indira Gandhi Peace, Disarmament and
Development Award for the year 2012?
(a) Lula De Silva
(b) Ellen Johnson Sirleaf
= 21413 (c) Sheakh Hasina
Same as, (d) Mohammed Al Baraki
Ans. (b) : Ellen Johnson Sirleaf, Liberian President and
Nobel laureate, was awarded Indira Gandhi Prize for
Peace, Disarmament and Development in the year 2012
for being an ‘inspiration’ to women and ensuring return
= 41813 of peace and democracy in her country,
106. If the coded substitute for word ABSENT is 110. As per the data released by the American
ZYHVMG then what will be the code for word Agricultural department, which of the
'PRESENT' ? following nation has been the largest exporter
(a) KIHVHGM (b) KIVHVMG by Rice in year 2012 ?
(c) KITMHMG (d) GKITYTL (a) India (b) Pakistan
Ans. (b) : Just as, (c) Thailand (d) Vietnam
Opposite letter
ABSENT  → ZYHVMG Ans. (a) : As per the data released by the American
Same as, Agricultural department, Thailand has been the largest
PRESENT  Opposite letter
→ KIVHVMG exporter by rice in year 2012. Now, India is largest
Note :- The opposite letter of each letter is given to find exporter of rice and Thailand ranks second in this list.
the opposite of each letter its alphabetical order is 111. Which among the following dates has been
subtracted from 27. declared as Malala Day in the honour of Child
107. Which of the following have not been accorded Activist, Malala Yousafjai recently by the
the status of Maharatna? United Nations?
(a) Steel Authority of India Limited. (a) 10th Oct (b) 10th Nov
st
(b) Bharat Heavy Electricals Ltd. (c) 1 Dec (d) 10th Dec
(c) Oil and Natural Gas Corporation Ltd. Ans. (b) : The United Nations has declared November
(d) Oil India Limited. 10 as 'Malala Day' in honour of Pakistani teenage rights
Ans. (d) : Oil India Limited have not been accorded the activist Malala Yousafzai, who was shot in the head by
status of Maharatna. As of January 2022, India has 10 the Taliban in the month of October, 2012 for
Maharatna companies, 14 Navratna companies in India, campaigning for girls' education.
and 73 Miniratna companies divided into Category 1
112. From 1 Jan 2013, which among the following
and Category 2. There is list of Maharatna companies as
follows: has been selected as the Indian Premier League
1. Bharat Heavy Electricals Limited title sponsor?
2. Bharat Petroleum Corporation Limited (a) Airtel (b) D.L.F
3. Coal India Limited (c) Reliance (d) Pepsico
4. GAIL (India) Limited Ans. (d) : Global beverage giant PepsiCo has bagged
5. Hindustan Petroleum Corporation Limited the IPL title sponsorship rights for the next five years
6. Indian Oil Corporation Limited (2013-17) for Rs 39 6.8 crore ($71.77 million approx).
7. NTPC Limited 113. Who among the following became the Japanese
8. Oil & Natural Gas Corporation Limited Prime Minister in Dec 2012?
9. Power Grid Corporation of India Limited (a) Naoto Kan (b) Shinjo Abe
10. Steel Authority of India Limited (c) Taro Aso (d) Yasuo Fukuda
UP RO/ARO Special (Mains) Exam 2010 250
CLICK HERE FOR FREE MATERIAL

Ans. (b) : The Diet (Parliament) selected Shinzo Abe Ans. (c) : Eminent American magazine Newsweek
to again lead the country after his Liberal Democratic discontinued its print edition in December 2012 after 80
Party scored a solid comeback in elections in years, amid falling advertising revenues as readers
December, 2012. That ends a three-year stay in power moved towards online publications.
for the Democratic Party of Japan. He assumed office 117. Who is the newly appointed General Secretary
on 26th December, 2012. of Chinese Communist Party in 2012?
114. The Government of India has announced which (a) Deng Xioping (b) Shi Mingzne
of the following change in the Indira Awas (c) Shi Jongeinsu (d) Xi Jinping
Yojana in January, 2013 ? Ans. (d) : Xi Jinping become newly appointed General
(1) The house construction credit in Plain Secretary of Chinese Communist Party in 2012. Xi
regions has been enhanced to Rs. 70,000/-. Jinping is Chinese politician and government official
(2) In the hilly regions the constuction has who served as vice president of the People’s Republic
been increased to Rs. 75000/-. of China (2008–13), general secretary of the Chinese
(3) People can get a capital amount of Rs. Communist Party (CCP; 2012– till now ), and president
20,000/- to purchase landless residential of China (2013– till now ).
land. 118. Who got an honour to become Major General
Codes : in Indian Army first after receiving the Ashok
(a) Only (1) and (2) are correct. Chakra?
(b) Only (3) is correct. (a) Major General Madan Mohan Singh Bakshi
(c) only (1) is correct. (b) Major General Cyrus Addie Pithawalla
(d) All (1), (2) & (3) are true. (c) Major General Mohd Amin Nayak
Ans. (d) : Indira Awaas Yojna is 100 percent (d) Major General K. Sudhakar
subsidized sponsored scheme with the resources Ans. (b) : On 1 August 2008, Cyrus Addie Pithawalla
being shared on 75:25 percent basis between Centre became the first recipient of either the Param Vir
and State respectively. The cost norms under Indira Chakra or the Ashoka Chakra to attain the one-star rank
Awaas Yojana (IAY) Scheme have been changed of Brigadier. Upon promotion to the rank of Major
from time to time. Initially the unit cost was fixed at General on 20th January 2013, he became the first
Rs. 10,000 which was enhanced to Rs. 12,700 in plain Ashoka Chakra recipient to attain the rank of a general
areas and 14,500 in hilly difficult areas in 1990. Later officer in a Two-star rank.
on, it was increased to Rs. 25,000 and 27,500 in
119. Which of the following films has won the
2004, Rs. 35,000 and 38,500 in 2008, Rs. 45,000 and
Golden Globe Award for the best film in the
48,500 in 2010 and at present cabinet has approved
Drama Category announced in Jan, 2013?
enhancement of the unit assistance from 45,000 to Rs.
70,000 in plain areas and from Rs. 48, 500 to Rs. 75,000 (a) Argo (b) Life of Pie
in hilly areas with effect from 1st August 2013. People (c) Lincoln (d) Skyfall
can get a capital amount of Rs. 20,000/- to purchase Ans. (a) : Iranian hostage drama ‘Argo’ has won the
landless residential land. All statements are true Golden Globe Award for the best film in the Drama
regarding changes in the Indira Awas Yojana in Category announced in January, 2013. ‘Argo’ combines
January, 2013. the true story of the rescue of U.S. diplomats from
115. What is the name of that Sri lankan Chief Tehran after the Islamic Revolution in 1979 under the
Justice who had been dismissed after his guise of making a movie, with a satire on Hollywood.
impeachment in January 2013? Argo had received prize in two categories as best film
and best director.
(a) Chandrika Bhandar
(b) Shirani Bandaranayake 120. Who had consecutively won for the fourth time
(c) Sunetra Bandaranayake the best football player of the year award -
Ballon d'or for the year 2012?
(d) Luisa Dias Bhandarnaike
(a) Christiano Ronaldo
Ans. (b) : Shirani Bandaranayake, the 43rd Chief Justice
of Sri Lanka, was impeached by Parliament and then (b) Diego Maradona
removed from office by President Mahinda Rajapaksa (c) Lionel Messi
in January 2013.Shirani Bandaranayake was first (d) Sergio Ramos
woman to head Sri Lanka's judiciary. Ans. (c.) : Lionel Messi had consecutively won for the
116. Which of the following Magazine has closed the fourth time the best football player of the year award -
publication of its printed version of its Ballon d'or for the year 2012. Argentinian Lionel Messi
Magazine after December, 2012? won the Ballon d'Or four years running between 2009
(a) Forbes (b) Fortune and 2012 and claimed the trophy twice more in 2015
(c) Newsweek (d) Times and 2019.
UP RO/ARO Special (Mains) Exam 2010 251
CLICK HERE FOR FREE MATERIAL

Gòej ØeosMe meceer#ee DeefOekeâejer/meneÙekeâ meceer#ee DeefOekeâejer efJeMes<e (cegKÙe) hejer#ee, 2010
meeceevÙe efnvoer
nue ØeMve-he$e
9. ‘yeeIe Deewj yekeâjer Skeâ Ieeš heeveer heerleer nQ’ JeekeäÙe keâe Megæ
meeceevÙe Meyo-%eeve SJeb JÙeekeâjCe ™he nw
1. ‘efÛejbleve’ Meyo keâe efJeueesce efueefKeS~ (a) yeeIe-yekeâjer Skeâ Ieeš hej heeveer heerleer nQ~
(a) efÛevlee keâjves Jeeuee (b) efÛevlee veneR keâjves Jeeuee (b) yeeIe Deewj yekeâjer Skeâ Ieeš hej heeveer heerles nQ~
(c) veMJej (d) efÛelee (c) yeeIe Deewj yekeâjer Skeâ ner Ieeš hej heeveer heerleer nQ~
Gòej–(c) (d) yeeIe Deewj yekeâjer heeveer heerleer nQ~
JÙeeKÙee- efÛejbleve keâe DeeMeÙe MeeMJele neslee nw Deewj Fmekeâe efJeueesce Gòej–(b)
veMJej neslee nw, peyeefkeâ efÛevlee keâjves JeeueeW kesâ efueS efÛevleveerÙe Meyo JÙeeKÙee- Ùeefo JeekeäÙe ceW oesveeW efuebie kesâ SkeâJeÛeve kesâ efJeYeefòeâ jefnle
keâe ØeÙeesie efkeâÙee peelee nw Deewj efÛevlee ve keâjves Jeeues kesâ efueS Deveskeâ keâlee& Ùee Fmeer DeLe& ceW JÙeJeùle efkeâmeer DevÙe DeJÙeÙe mes mebÙegòeâ
DeefÛevleveerÙe~ nes, lees ef›eâÙee ØeeÙe: yengJeÛeve Deewj hegefuebie nesieer~
2. ‘Deveg«en’ Meyo keâe efJeueesce efueefKeS~ 10. veerÛes efoÙes JeekeäÙeeW ceW keâewve mee JeekeäÙe $egefšnerve nw?
(a) «enCe (b) ie=nerle (c) Dee«en (d) efJe«en (a) cesjs Iej kesâ heeme Skeâ heeve keâer otkeâeve nw~
Gòej–(d) (b) cesjs Iej kesâ heeme Skeâ heeve keâer otkeâeve efmLele nw~
(c) cesjs Iej kesâ heeme Skeâ heeveeW keâer otkeâeve nw~
JÙeeKÙee – Deveg«en Meyo keâe efJeueesce efJe«en neslee nw, peyeefkeâ «enCe (d) cesjs Iej kesâ heeme heeve keâer Skeâ ogkeâeve nw~
keâe lÙeeie, Dee«en keâe Devee«en Deewj ie=nerle keâe Deefhe&le efJeueesce Meyo Gòej–(d)
neslee nw~ JÙeeKÙee- cesjs Iej kesâ heeme Skeâ heeve keâer otkeâeve nw~ GmeceW ›eâce keâe
3. ‘DeveefYe%e’ keâe efJeueesce nw oes<e nw~ De›eâcelJe oes<e kesâ keâejCe Yeer ØeeÙe: JeekeäÙe DeMegæ nes peeles nQ~
(a) De%e (b) Øe%e (c) DeefYe%e (d) DeefJe%e Ùeneb hej Skeâ heeve "erkeâ ve neskeâj Skeâ ogkeâeve meeLe&keâ DeeMeÙe oslee nw~
Gòej–(c) efJekeâuhe (d) cesjs Iej kesâ heeme heeve keâer Skeâ ogkeâeve nw~ mener JeekeäÙe nw~
JÙeeKÙee – De%e keâe efJeueesce efJe%e neslee nw~ DeveefYe%e keâe efJeueesce 11. ‘heefle-helveer kesâ PeieÌ[s keâe nsleg keäÙee nes mekeâlee nw’ JeekeäÙe keâe
DeefYe%e nesiee~ Megæ ™he efueefKeS~
4. ‘meewcÙe’ Meyo keâe efJeueesce nw (a) heefle-helveer kesâ PeieÌ[s keâe keäÙee nsleg nes mekeâlee nw?
(a) meewYeeiÙe (b) G«e (c) Me$eg (d) ogjeMeÙe (b) heefle-helveer kesâ PeieÌ[s keâe nsleg keäÙee nw?
Gòej–(b) (c) heefle-helveer kesâ PeieÌ[s keâe keâejCe keäÙee nes mekeâlee nw?
(d) heefle Deewj helveer kesâ PeieÌ[s keâe nsleg keäÙee nes mekeâlee nw
JÙeeKÙee- meewcÙe Meyo keâe efJeueesce G«e nesiee~ Fmeer Øekeâej meewYeeiÙe keâe
Gòej–(c)
ogYee&iÙe, Me$eg keâe efce$e Deewj ogjeMeÙe keâe meoeMeÙe efJeueesce Meyo nesiee~
JÙeeKÙee-Heefle-helveer kesâ PeieÌ[s keâe nsleg keäÙee nes mekeâlee nw~ ÙeneB hej
5. ‘Üeme’ Meyo keâe efJeueesce nw
Megæ Meyo nsleg keâer peien keâejCe nesiee~ DeleSJe Megæ JeekeäÙe nesiee –
(a) nemÙe (b) Je=efæ (c) nBmeer (d) nmle heefle-helveer kesâ PeieÌ[s keâe keâejCe keäÙee nes mekeâlee nw~
Gòej–(b) 12. Jele&veer keâer Âef° mes Meyo keâe Megæ ™he keâewve nw?
JÙeeKÙee– Üeme Meyo keâe efJehejerleeLe&keâ Meyo Je=efæ nesiee peyeefkeâ nemÙe (a) yevovee (b) Jebovee (c) yeveovee (d) Jevovee
keâe ®ove Deewj nmle keâe heeo nesiee~ nBmeer (ØemeVe) keâe og:Keer nesiee~ Gòej–(b)
6. ‘vÙetve’ Meyo keâe efJeueesce nw JÙeeKÙee- Jele&veer keâer Âef° mes Megæ Meyo ‘Jebovee’ nesiee ve efkeâ yevovee,
(a) DeefOekeâ (b) veJeerve (c) veJeveerle (d) veiej Jevovee Ùee yeveovee~
Gòej–(a) 13. ‘pees efkeâÙes ieÙes GhekeâejeW keâes ceevelee nw’ kesâ efueS Skeâ Meyo
JÙeeKÙee – vÙetve Meyo keâe efJeueesce DeefOekeâ nesiee peyeefkeâ veJeerve keâe efueefKeS~
ØeeÛeerve, veiej keâe ieeBJe efJeueesce Meyo nesiee~ (a) ke=âleIve (b) ke=âle%e (c) ke=âlekeâeÙe& (d) De%e
7. ‘heg°’ Meyo keâe efJeueesce nw Gòej–(b)
(a) #eerCe (b) og° (c) hegjmkeâej (d) Øeke=âefle JÙeeKÙee- ‘‘pees efkeâÙes ieÙes GhekeâejeW keâes ceevelee nw~’’ Gmekesâ efueS
Gòej–(a) ‘ke=âle%e’ Meyo keâe GheÙeesie keâjles nQ~ pees efkeâÙes ngS GhekeâejeW keâes veneR
JÙeeKÙee- heg° Meyo keâe efJeueesce ‘#eerCe’ nesiee, peyeefkeâ Øeke=âefle keâe ceevelee~ Gmekesâ efueS ke=âleIve meeLe&keâ Meyo nw~ ke=âle keâeÙe& keâe DeeMeÙe
efkeâÙee ngDee keâeÙe& Deewj De%e keâe DeeMeÙe pees veneR peevelee nw~
heg®<e, hegjmkeâej keâe oC[ SJeb og° keâe meppeve efJeueesce Meyo nesiee~
14. ‘hesš keâer Deefive’ keâes keânles nQ
8. ‘iegCe’ Meyo keâe efJeueesce nw (a) oeJeeefive (b) Je[Jeeefive (c) pe"jeefive (d) cevoeefive
(a) oes<e (b) iegÌ[ (c) iegCee (d) ie=nmLe Gòej–(c)
Gòej–(a) JÙeeKÙee- hesš keâer Deefive kesâ efueS meeLe&keâ Meyo pe"jeefive neslee nw~
JÙeeKÙee-iegCe Meyo keâe efJeueesce oes<e nesiee peyeefkeâ ie=nmLe keâe mebvÙeeme, peyeefkeâ pebieue keâer Deeie keâer keâes ‘oeJeeefive’ leLee mecegõ keâer Deeie keâes
iegCee keâe Yeeie nesiee~ ‘yeÌ[Jeeefive’ keânles nQ~
UP UDA/LDA Special (Main) General Hindi 252 YCT
CLICK HERE FOR FREE MATERIAL

15. ‘yeÛÛeeW keâes megueeves kesâ efueS ieeÙee peeves Jeeuee ieerle’ nw 24. FveceW mes leodYeJe nw :
(a) ØeYeeleer (b) efJeneie (c) ueesjer (d) meesnj (a) Jeevej (b) yevoj
Gòej–(c) (c) heJeve (d) heÙeËkeâ
JÙeeKÙee- yeÛÛeeW keâes megueeves kesâ efueS ieeÙee peeves Jeeuee ieerle ueesjer nw~ Gòej–(b)
peyeefkeâ meesnj yeÛÛes kesâ pevce kesâ meceÙe ieeÙee peelee nw~ JÙeeKÙee- efveefo&° efJekeâuheeW ceW Jeevej, heJeve Deewj heÙeËkeâ lelmece Meyo
16. ‘pees veYe ceW Ûeuelee nw’ kesâ efueS Meyo nw nw~ yevoj Meyo leodYeJe nw~
(a) KesÛej (b) KeÛÛej (c) veYeeslhevve (d) veYeÛeeueer
25. ‘keâeuee IeesÌ[e lespe oewÌ[lee nw’ ceW ef›eâÙee efJeMes<eCe nw
Gòej–(a)
(a) IeesÌ[e (b) keâeuee
JÙeeKÙee- ‘‘pees veYe ceW Ûeuelee nw’’ kesâ efueS Skeâ Meyo nw-‘KesÛej’~ (c) lespe (d) oewÌ[lee nw
‘Kes’ keâe DeeMeÙe ‘DeekeâeMe’ neslee nw leLee ‘Ûej’ keâe DeeMeÙe ‘Ûeuevee’~ Gòej–(c)
17. ‘JÙeekeâjCe kesâ %eelee’ kesâ efueS Meyo nw
JÙeeKÙee- keâeuee IeesÌ[e lespe oewÌ[lee nw~ ef›eâÙee kesâ hetJe& ueieves Jeeuee
(a) JÙeekeâjCeer (b) JÙeekeâlee& (c) JewÙeekeâjCe (d) JÙeekeâjCe%e
Gòej–(c) efJeMes<eCe ef›eâÙee efJeMes<eCe neslee nw~ Dele: ÙeneB hej lespe ef›eâÙee
JÙeeKÙee- JÙeekeâjCe kesâ %eelee keâes JewÙeekeâjCe keânles nQ~ DevÙe leerveeW efJeMes<eCe nw~
efJekeâuhe DeMegæ nw~ 26. ‘ieeruee’ nw
18. ‘yeÌ{e ÛeÌ{e keâj keânvee’ kesâ efueS Skeâ Meyo nw (a) meeJe&veeefcekeâ efJeMes<eCe (b) iegCeJeeÛekeâ efJeMes<eCe
(a) DeefleJeeoer (b) DeefleMeÙe (c) DelÙevle (d) DeefleMeÙeesefòeâ (c) mebKÙeeJeeÛekeâ efJeMes<eCe (d) FveceW mes keâesF& veneR
Gòej–(d) Gòej–(b)
JÙeeKÙee- ‘‘yeÌ{e-ÛeÌ{ekeâj keânvee’’ kesâ efueS Skeâ Meyo nw– JÙeeKÙee – ‘ieeruee’ ceW iegCeJeeÛekeâ efJeMes<eCe nw~ efpeme efJeMes<eCe ceW
DeefleMeÙeesefòeâ~ Mes<e efJekeâuhe Demebiele nQ~ ™he, jbie, Deekeâej, DeJemLee, mJeYeeJe, oMee, mJeeo, mheMe&, iebOe,
19. efvecveefueefKele lelmece-leodYeJe MeyoeW kesâ Ùegice ceW mes keâewve Ùegice efoMee, mLeeve, meceÙe, Yeej, leeheceeve Deeefo keâe yeesOe neslee nw, Jeneb
$egefšhetCe& nw? iegCeJeeÛekeâ efJeMes<eCe neslee nw~
(a) Ie=le - Ieer (b) Gš^ - TBš 27. ‘oesveeW’ Meyo nw
(c) lJeefjle - legjle (d) eflekeäle - leerlee (a) mecegoeÙe JeeÛekeâ efJeMes<eCe (b) DeeJe=efòeJeeÛekeâ efJeMes<eCe
Gòej–(b) (c) ieCeveeJeeÛekeâ efJeMes<eCe (d) ›eâceJeeÛekeâ efJeMes<eCe
JÙeeKÙee- efveefo&° Meyo Ùegice ceW mener peesÌ[s nQ – Ie=le-Ieer, lJeefjle- Gòej–(a)
legjle, eflekeäle-leerlee~ Gš^ - TBš DeMegæ Meyo iegice nw, peyeefkeâ Fmekeâe JÙeeKÙee- oesveeW Meyo mecegoeÙeJeeÛekeâ efJeMes<eCe nw~ pees efkeâ efveefMÛele
Megæ ™he nesiee – G°^ - TBš~ mebKÙeeJeeÛekeâ keâe Yeso nw~ pees efvecve Øekeâej nQ~
20. efvecveefueefKele lelmece-leodYeJe MeyoeW kesâ Ùegice ceW mes $egefšhetCe& nw-
ieCeveeJeeÛekeâ- Skeâ, oes, leerve
(a) ieesceÙe - ieesyej (b) #eerj - Keerj
›eâceJeeÛekeâ- henuee, otmeje, leermeje
(c) heÙeËkeâ - hešjer (d) mehelveer - meewle
Gòej–(c) DeeJe=efòeJeeÛekeâ-otvee, efleiegvee, Ûeewiegvee
mecegoeÙeJeeÛekeâ- oesveeW, leerveeW, ÛeejeW
JÙeeKÙee- ieesceÙe keâe ieesyej, #eerj keâe Keerj Deewj mehelveer keâe meewle mener
28. efvecveefueefKele MeyoeW ceW mes efJeMes<Ùe keâewve nw?
Meyo Ùegice nw~ heÙeËkeâ keâe hešjer Ùegice ieuele nw~ Fmekeâe mener Ùegice nesiee
– heÙeËkeâ-heuebie~ (a) DeekeâeMeerÙe (b) DeekeâeMe
21. ‘heCe&’ keâe leodYeJe Meyo nw (c) DeejeOÙe (d) Deeefßele
(a) he$e (b) heCe (c) hevvee (d) he$ee
Gòej–(b)
Gòej–(a) JÙeeKÙee- DeekeâeMe efJeMes<Ùe Meyo nw~ Fmekeâe efJeMes<eCe DeekeâeMeerÙe neslee
JÙeeKÙee-HeCe& keâe leodYeJe Meyo he$e nesiee ve efkeâ heCe, he$ee Deewj hevvee~ nw~ Dele: Gòej (b) DeekeâeMe nesiee~
22. ‘Debieer"er’ keâe lelmece nw 29. efvecveefueefKele MeyoeW ceW mes efJeMes<eCe keâewve nw?
(a) Deefivekeâe (b) Debefveef‰keâe (a) DepeÙe (b) Deefpele
(c) Deefiveef‰keâe (d) Deefiveef‰keâer (c) Dekeâce& (d) DevegMebmee
Gòej–(c) Gòej–(b)
JÙeeKÙee- Debieer"er keâe lelmece ™he nesiee Deefiveef‰keâe ve efkeâ JÙeeKÙee- Deefpele, efJeMes<eCe Meyo nw peyeefkeâ DepeÙe, Dekeâce& Deewj
Deefivekeâe, Deefiveef‰keâer SJeb Debefveef‰keâe~ DevegMebmee efJeMes<Ùe Meyo nw~
23. efvecveefueefKele ceW mes keâewve leodYeJe Meyo nw? 30. efvecveefueefKele MeyoeW ceW mes efJeMes<eCe keâewve veneR nw?
(a) efovekeâj (b) efoJeekeâj
(a) Deebleefjkeâ (b) Deblej
(c) ØeYeekeâj (d) metjpe
(c) DeeivesÙe (d) DeefOekeâejer
Gòej–(d)
Gòej–(b)
JÙeeKÙee- Gòeâ efoÙes ieÙes efJekeâuheeW ceW lelmece Meyo efovekeâj, efoJeekeâj
Deewj ØeYeekeâj nw, peyeefkeâ metjpe Meyo leodYeJe nw~ metjpe keâe lelmece Meyo JÙeeKÙee- Deebleefjkeâ, DeeivesÙe Deewj DeefOekeâejer Meyo efJeMes<eCe nw~
metÙe& nesiee~ ‘Deblej’ Meyo efJeMes<eCe veneR nw~ Ùen efJeMes<Ùe Meyo nw~
UP UDA/LDA Special (Main) General Hindi 253 YCT
CLICK HERE FOR FREE MATERIAL

UPPSC RO-ARO Special (Pre) Exam-2010


GENERAL STUDIES
Solved Paper
1. In Rigveda, the word Aghanya has been used 6. The Kailash Temple of Ellora was constructed
for - by -
(a) Goats (b) Cow (a) the Rastrakutas
(c) Elephant (d) Horse (b) Chalukyas of Vatapi
Ans. (b) : "Aghanya" refers to cows in Rig Veda. (c) Ganga Dynasty rulers
Rigveda is one of the four Vedas. It is the oldest of the (d) None of the above
four Sacred Vedas. It Contains 10 mandala and 10,552 Ans. (a) : The Rashtrakuta Kings were patron of art.
verses. Their great Contribution to Indian art can be seen in the
2. Which incarnation of Lord Vishnu has been Ellora and Elephanta Caves temples. The kailasnath
depicted by Salvaging Earth from the Ocean ? temple at Ellora built by Krishna I (756-773 C.E) is a
(a) Kurma (b) Matsya monolithic wonder.
(c) Varaha (d) Narsingha 7. Alberuni came to India in -
Ans. (c) : Varaha Avatar, the third incarnation among (a) the 9th century A.D
the 10 Dashavatara of Lord Vishnu, is in the form of a (b) the 10th century A.D
Boar. In his third incarnation, Lord Vishnu as Varaha (c) the 11th century A.D
saved the earth from demon Hiranyaksha, thus saving (d) the 12th century A.D.
the mankind. Ans. (c) : Alberuni visited India during the 11th
3. Which emperor of Magadha is known by the Century. He was born in 973 A.D in khiza region. He
name - 'Aproparshuram' ? was a renowned philosopher, a Mathematician and a
(a) Bindusar (b) Ajatashatru historian. He came to India with Mahmud and lived
(c) Kalashok (d) Mahapadmanand here for many years.
Ans. (d) : Mahapadmananda is known as 8. Which of the following Sultans is known for
"Aproparashuram". According to the puranas, Lifting/abolishing tax on grain?
Mahapadmananda, the founder of the Nanda dynasty. (a) Allauddin Khilji
was a shudra. He was known as Sarvakshatrantak (b) Gayasuddin Tughlaq
(destroyer of Kshatriya) and Bhargava (incarnation of (c) Firozshah Tughlaq
the second Parshuram). He uprooted the kshatriya (d) Sikandar Lodhi
dynasty in North India and assumed the tittle Ekarat.
Ans. (d) : Sikandar Lodhi was the Sultan of Delhi
4. Who was the author of 'Matt Vilas Prahasan' ? between 1489 to 1517 A.D. He was a laborious,
(a) Gautami Putra Satakarni generous and just ruler. Owing to a transient shortage of
(b) Mahasatrap Rudradaman maize, he abolished the zakat on grain and it was not
(c) Mahendra Verman renewed by any subsequent Sultan the prices of all
(d) Pulkeshin-II necessary articles remained low during his period.
Ans. (c) : 'Matta Vilas Prahasan' was composed by 9. Which of the following Saints belonging to
Mahendra Varman I in the Seventh Century. Bhakti movement was influenced by Islam ?
Mahendravarman was a Pallava ruler who was first a (a) Chaitanya (b) Meerabai
follower of Jainism and later adopted shaivism after (c) Namdev (d) Vallabhacharya
coming in contact with a saint.
Ans. (c) : Among the Saints of Bhakti Movement,
5. Bhumi Sparsha (earth witness) mudra of Namdev was highly influenced by Islam. He was tailor
Buddha in Sarnath is related to which of the by profession. He emphasized on cultural unity of the
following period? Marathas by introducing a tradition called Mahapurusha
(a) Kushan Period (b) Gupta Period Sampradaya. some of his devotional songs are found in
(c) Vardhan Period (d) Rajput Period the Guru Granth Sahib.
Ans. (b) : Bhumisparsha Mudra, represents the moment 10. Rana Sanga had fought which of the following
of Buddha's attaining enlightenment. It is more battles against Babar?
commonly known as the 'earth witness' mudra. This (a) Battle of Panipat (b) Battle of Khanwa
mudra is related to Gupta period. (c) Battle of Chanderi (d) Battle of Ghaghra
UP RO/ARO Special (Pre) Exam 2010 254 YCT
CLICK HERE FOR FREE MATERIAL

Ans. (b) : The Battle of Khanwa was fought between Ans. (d) : Abhinav Bharat Society a secret society
Babur and the Rajput ruler Rana Sanga on 17 March founded by Vinayak Damodar Savarkar and his brother
1527. The Battle of Khanwa was a decisive battle that Ganesh Damodar Savarkar in 1904 by several
helps the establishment of Mughal rule in India. revolutionary and political activists with initially founded
Khanwa is located 60 Km west of Agra. at Nashik as Mitra Mela, the society was associated with
11. Shivaji signed the "Treaty of Purandhar" branches in various parts of India and London.
(1665) with which Mughal Army Commander? 16. Who was related to the foundation of Deccan
(a) Jaswant Singh (b) Jai Singh Educational Society ?
(c) Shaista Khan (d) Diler Khan (a) Justice Ranade (b) Firozshah Mehta
Ans. (b) : The treaty of Purandhar was signed on June (c) B.J Tilak (d) Dayanand Saraswati
11, 1665 AD between Shivaji Maharaj and Raja Jai Ans. (a) : The Deccan education society was setup in
Singh-I of Amber, who was deputed by Mughal 1884. The society was founded by honorable Justice
emperor Aurangzeb. Mahadev Govind Ranade, V.S Apte, M.S. Gole etc. The
seeds for the society were sown by Bal Gangadhar Tilak,
12. Match List-I with List-II and choose the right Vishnushastri Chiplunkar and Gopal Ganesh Agarkar by
answer using the codes given below : establishing New English School in Pune in 1880.
List-I List-II 17. Who founded the Indian Brahma Samaj ?
A) Babur 1. Jami Mosque [Sambhal] (a) Raja Ram Mohan Roy
B) Humayun 2. Deen Panah (b) Debendranath Tagore
C) Akbar 3. Jahangiri Mahal (c) Ishwar Chandra Vidyasagar
D) Jahangir 4. Aitmad-ud-daula's (d) Keshav Chandra Sen
Tomb Ans. (d) : Indian Brahma Samaj was founded by
Codes : Keshav Chandra Sen in the year 1866 after a split in
A B C D Brahmo samaj. The Brahmo samaj was founded by Raja
(a) 1 2 3 4 Ram Mohan Roy in 1828. The another faction after split
(b) 4 3 2 1 was named Adi Brahmo Samaj in 1866 and led by
(c) 3 4 1 2 Devendra Nath Tagore.
(d) 2 1 4 3 18. Aruna Asaf Ali was associated with which of
the following movements ?
Ans. (a) : The Correct match are :
(a) Non-cooperation movement
Babur - Jami Mosque (Sambhal) (b) Civil disobedience movement
Humayun - Deen Panah (c) Individual Satyagrah
Akbar - Jahangiri Mahal (d) Quit India movement
Jahangir - Aitmad-ud-daulas' Tomb Ans. (d) : Aruna Asaf Ali was an Indian educator
13. Who among the following had called the 'Govt. political activist and publisher. An active participant in
of India Act, 1935' as 'a charter of slavery' ? the Indian Independence movement, She was connected
(a) J.L Nehru with Quit India movements and started consolidating
underground networks. The role of the underground
(b) M.A Jinnah movement was seen as one keeping up popular morale
(c) Dr. Rajendra Prasad by providing a line of command and guidance to
(d) Maulana Abul Kalam Azad activists across the country.
Ans. (a) : "It was a new charter of slavery", was said by 19. Statement (A) : Congress had boycotted the
Jawaharlal Nehru about the Government of India Act, Simon Commission.
1935. Reason (R): There was no Indian in the Simon
14. Who was the General Secretary of Indian Commission.
National Congress in 1885 ? (a) If both (A) and (R) are correct and (R) is the
(a) A.O. Hume (b) Dadabhai Naoroji correct explanation of (A).
(c) W.C. Bonnerji (d) Firoz Shah Mehta (b) If both (A) and (R) are correct but (R) is not
Ans. (a) : On 28 December 1885, the Indian National the correct explanation of (A).
Congress was founded at Gokuldas Tejpal Sanskrit College (c) If (A) is correct, but (R) is incorrect.
in Bombay. With 72 delegates in attendance. Hume (d) If (A) is incorrect, but (R) is correct.
assumed office as the General Secretary, and Womesh Ans. (a) : In 1928, Simon Commission came to India to
Chandra Bonnerjee of Calcutta was elected president. look after the working of the Government of India Act,
15. What was the name of the Secret organization 1919 and to suggest changes. But it consisted only of
established by V.D. Savarkar ? Englishmen and did not have a single Indian
(a) Yugantar Samiti representative.
(b) Anusheelan Samiti 20. Who presided over the 1946, Meerut session of
(c) Hindustan Republic Association Indian National Congress ?
(d) Abhinav Bharat (a) J.B. Kripalani
UP RO/ARO Special (Pre) Exam 2010 255 YCT
CLICK HERE FOR FREE MATERIAL

(b) Maulana Abul Kalam Azad 25. Match List-I with List-II and choose the right
(c) Dr. Rajendra Prasad answer using the codes given below :
(d) B. Pattabhi Sitaramayya List-I [Centre] List-II [Industry]
Ans. (a) : J.B Kripalani presided over the 1946, Meerut A) Kankinara 1. Carpet
session of Indian National Congress. B) Virudhnagar 2. Jute
21. Which of the following is not a tributary river C) Channa Patna 3. Cotton Textiles
of Krishna river ? D) Bhadohi 4. Silk
(a) Bheema (b) Don Codes :
(c) Tel (d) Tungabhadra A B C D
(a) 1 2 3 4
Ans. (*) : The Krishna river is a river in the Deccan
(b) 4 3 2 1
plateau and is the third longest river in India. It
(c) 4 3 2 1
originates in the western ghats near mahabaleshwar and
it ends at Bay of Bengal in Krishna district (A.P). Its (d) 3 2 1 4
tributaries are Warna, Koyna, panchganga , Ans. (b) : The correct pairs are.
Dudhaganga, Ghataprabha, Malprabha, Tungabhadra, List-I List-II
Bheema, Musi, etc. The River Tel is a tributary of (Centre) (Industry)
Mahanadi river and Don River flows in Russia. a. Kanikinara - Jute
22. Among the following places where does one b. Virudhnagar - Cotton Textile
experiences a humid climate ? c. Channa Patna - Silk
1. Ahmedabad 2. Kochi 3. Ludhiana d. Bhadohi - Carpet.
4. Tejpur 26. At which of the following places, there is no oil
Choose the correct answer from the codes refinery ?
given below- (a) Koyali (b) Noonmati
(a) only 1 & 2 (b) only 1 & 3 (c) Hatia (d) Barauni
(c) only 2 & 3 (d) only 2 & 4 Ans. (c) : There are total 23 refinery in the country, 18
Ans. (d) : Among the given options, Kochi and Tejpur in public sector, 2 in joint venture and 3 in the private
experiences humid climate because, this two places sector, well spread geographically and connected with
receives more rainfall. cross country pipelines. In the given options, Hatia is
place near Ranchi, with no refinery.
23. The headquarters of 'Indian Bureau of Mines'
27. In India two top producer states of Bauxite are?
is located at ?
(a) Jharkhand and Gujarat
(a) Ranchi (b) Ahmedabad
(b) Gujarat and Odisha
(c) Nagpur (d) Mysore
(c) Odisha and Jharkhand
Ans. (c) : Indian Bureau of Mines headquarter is (d) Odisha and Jharkhand
located in Nagpur.
Ans. (b) : Gujarat and Odisha are the largest Bauxite
24. Match List-I with List-II and choose the right producing state in the country. Odisha is top producer
answer using the codes given below : followed by Gujarat, Jharkhand etc. Bauxite is principal
List-I List-II are of aluminium production and very important non-
A) Ukai 1. Jharkhand ferrous metal.
B) Patratu 2. Gujarat 28. The North-South and East-West Corridor
C) Pench 3. Maharashtra being constructed under the National Highway
D) Dabhol 4. Madhya Pradesh Development Project meet each other at ?
(a) Kanpur (b) Lucknow
Codes :
(c) Jhansi (d) Varanasi
A B C D
Ans. (c) : Jhansi is the junction of North-South and
(a) 4 2 3 1
East-West corridor and four stretches are common
(b) 2 1 4 3 between the Golden quadrilateral the NS-EW corridors.
(c) 3 2 1 4 The North-South corridor connects Srinagar to
(d) 1 3 4 2 Kanyakumari and the East-West corridor connects
Ans. (b) : The correct pairs are Silchar to Porbandar.
List-I List-II 29. If in England a Cricket commentary is
a. Ukai - Gujarat broadcast at 10.00 AM GMT then it can be
heard in India as per the Indian standard time
b. Patratu - Jharkhand on ?
c. Pench - Madhya Pradesh (a) 10 = 10 AM (b) 4 = 30 AM
d. Dabhol - Maharashtra (c) 3:30 PM (d) 2 = 30 P.M.
UP RO/ARO Special (Pre) Exam 2010 256 YCT
CLICK HERE FOR FREE MATERIAL

Ans. (c) : As per the Indian Standard Time (IST) the 1. Nagaland 2. Himanchal Pradesh
commentary will be heard in India at 3.30 P.M, if it is 3. Sikkim 4. Tripura
broadest at 10.00 A.M in England. Codes :
30. Which of the following country receives almost (a) 2, 4, 3, 1 (b) 4, 2, 1, 3
two third of its total export money from rice (c) 4, 2, 3, 1 (d) 2, 1, 3, 4
trade.
Ans. (c) : Literacy Rate of given states in descending
(a) Japan (b) Thailand
order as per census 2011 is - Tripura (87.22%),
(c) Indonesia (d) Myanmar Himachal Pradesh (82.80%), Sikkim (81.42%) and
Ans. (d) : Myanmar accounts for about two - thirds of Nagaland (79.55%) respectively
earning of its total export from the rice trade. Rice Therefore, the correct answer is option (c).
production in Myanmar account for approximately 43%
of total agricultural production. 37. As per the latest figures of Census 2011, which
of the following states has a population density
31. In Africa the largest copper producing nation
is ? less than 100 ?
(a) South Africa (b) Zambia (a) Nagaland (b) Meghalaya
(c) Kenya (d) Tanzania (c) Mizoram (d) Manipur
Ans. (b) : Zambia is home to some of the world's Ans. (c) : According census 2011 population density of
richest mines and deposits. As per recent data given states is - Mizoram (52), Meghalaya (132),
Democratic Republic of Congo is the top producer in Nagaland (119) and Manipur (115) respectively,
Africa of copper and Zambia is the second largest Therefore, the correct answer is Mizoram, option (c).
copper producer in Africa. 38. Every year population day is celebration ?
32. Which of the following is not correctly (a) 5th June (b) 11th June
matched. th
(c) 5 July (d) 11th July
(a) Bolvia - Tin (b) Brazil - Iron Ore
Ans. (d) : The world population Day is celebrated on
(c) Mexico - Silver (d) Peru - Saltpetre
11th July every year. Theme for 2022 is " A world of 8
Ans. (d) : Peru is an important producer of silver, lead, billion" which expects a strong future for all
and copper. guaranteeing freedom and decision for all.
33. In which country have market based. Iron and 39. As per the last Census figures of year 2011
steel industries ?
what is the percentage of women in India's
(a) China (b) India total population ?
(c) Japan (d) U.K.
(a) 49.80 (b) 48.46
Ans. (c) : There are the market based iron and steel (c) 47.98 (d) 47.24
factories in Japan. Japan's first iron and steel factory
was opened in yawata. Due to the lack of production of Ans. (b) : The population of the country as per the
cast iron according to the demand of the country, Japan provisional figures of census 2011 is 1210.19 million of
has to import it. which 623.72 million (51.54%) are males and 586.46
34. Which of the following country has highest million (48.46%) are females.
percentage of forest cover of its total 40. As per the latest figures of Census 2011, during
geographical area ? the 2001-11 decade, the decadal growth rate of
(a) China (b) India Uttar Pradesh total population was :-
(c) Indonesia (d) Japan (a) 25.07% (b) 21%
Ans. (d) : Japan has the highest forest cover of 67% of (c) 20.30% (d) 20.09%
its geographical area, while 24.62% of India's land area Ans. (b) : According to the provisional data of Uttar
is covered by forests and trees, which is only 2.4% of Pradesh was 20.09%. According to the final census
the total land area of the world. 2011 data released on 30 April 2013, the decadal
35. As per the latest figure of 2011 Census, what growth rate of Uttar Pradesh was 20.2%
percentage of Indians population lives in Uttar 41. The states in which as per the latest Census
Pradesh ? figures of year 2011, the women literacy rate
(a) Approx 19% (b) Approx 18% was maximum are respectively :
(c) Approx 17% (d) Approx 16% (a) Kerala and Bihar
Ans. (*) : According to the census of 2011, 16.50%. of (b) Kerala & Jharkhand
India's population lives in Uttar Pradesh which is 199,
(c) Kerala and Rajasthan
812, 341.
(d) Kerala and Uttar Pradesh
36. As per the last/latest figures of census 2011,
arrange the following states in a decreasing Ans. (a) : As per census 2011 the highest female
order of their literacy percentage and choose literacy rate state is Kerala with 65.46% while Bihar has
the correct answer from the codes given below: the lowest female literacy rate of 51.50%.
UP RO/ARO Special (Pre) Exam 2010 257 YCT
CLICK HERE FOR FREE MATERIAL

42. As per the latest figure if Census 2011, which of 47. Which of the following is considered as the first
the following statements which statement is not Law Officer of the Government of India ?
correct ? (a) Chief Justice of India
(a) Less than 25% of the U.P population lives in (b) Solicitor General of India
urban cities (c) Attorrney General of India
(b) Uttar Pradesh is largest state in terms of (d) Advocate General
having the highest urban population
Ans. (c) : The first law officer of the Government of
(c) Allahabad is largest populated district in Uttar India is the Attorney General of India. The Attorney
Pradesh General of India in the Indian government's chief legal
(d) "KAVAL CITIES" come under the category advisor and is the primary lawyer in the supreme court
of those cities which has a population of more of India.
than 10 lacs.
48. The Union Public Service Commission submits
Ans. (b) : According to the provisional data of census its Annual Report to ?
2011, 22.3% percent of the population of Uttar Pradesh
(a) Prime Minister (b) President
lives in cities. The state with the largest urban
population in India is Maharashtra (45.2%). (c) Speaker of lok sabha (d) Home Minister
Kawal Nagai comes in the category of cities with Ans. (b) : The Union Public Service Commission
population more than one million. (UPSC) submits its annual report on its work to the
President of India. The president places this report
43. Which of the following Article under the
Indian constitution prohibits employment of before both the Houses of Parliament. Therefore, the
children in factories ? correct answer is option (c).
(a) Article 19 (b) Article 17 49. Below are given two statements the first is
(c) Article 23 (d) Article 24 called as statement (A) and the second as
statement (B).
Ans. (d) : Article 24 under the constitution of India says
that prohibition of employment of children in factories, Statement (A) : Any person will be eligible to
etc. No child below the age of fourteen years shall be be elected for the post of Vice-President. When
employed to work in any factory or mine or engaged in he is eligible to become member of Rajyasabha.
any other hazardous employment. Reason (R) : Vice-President is the ex-officio
chairman of Rajya sabha.
44. Which of the following Articles had been
termed as Heart and Soul of Constitution ? In the prespective of the above statements
which of the following answers is/are true ?
(a) Article 14 (b) Article 25
(a) Both (A) and (R) are true and (R) is the
(c) Article 29 (d) Article 32
correct explanation of (A)
Ans. (d) : The Supreme Court is made the protector and (b) Both (A) and (R) are correct but (R) is not the
guarantor of fundamental rights. It is provided with correct explanation of (A)
power and controls to provide remedies if these rights
(c) (A) is true but (R) is false.
are infringed upon by Article 32 of the constitution. Dr.
Ambedkar rightly heralded it as the 'Heart and soul' of (d) (A) is false but (R) is true.
the constitution. Ans. (a) : According to Article 66 of the Constitution,
45. Right to Information is- no person shall be eligible for election as vice-president
(a) a fundamental right (b) a legal right unless he is a citizen of India, and has completed the
age of thirty five years, is qualified for election as a
(c) Both (a) & (b) (d) Neither (a) nor (b)
member of council of states (Rajya Sabha). A person
Ans. (a) : Right to Information Act in a fundamental shall not be eligible if he holds any office of profit
right and is an aspect of Article 19(1)(a) of the Indian under any local or other authority subject to the control
Constitution, Right to Information replaced the freedom of any of the said Government. The vice-president is the
of Information Act, 2002. ex-officio chairman of Rajya sabha.
All constitutional authorities, agencies, owned and
50. Consider the following statements :-
controlled, also those organisation which are
substantially financed by the government comes under (1) Right to live and personal freedom,
the preview of the act. fundamental right cannot be suspended in
the emergency period.
46. Which of the following right of second
(2) Vice-President of India can be removed
generation ?
from his post by moving a resolution
(a) Right to Employment (b) Right to Education passed by the Rajya Sabha and by
(c) Right to Freedom (d) Right of Equality approval of the same by Lok Sabha.
Ans. (d) : The second generation of human rights (3) At present the leader of opposition in
encompasses socio economic rights. They are based on Rajya Sabha is Shri Arun Jaitley.
the ideas of equality and guaranteed access to essential (4) The President/chairman of Planning
social and economic goods, services and opportunities. commission is Montek Singh Ahluwalia.
UP RO/ARO Special (Pre) Exam 2010 258 YCT
CLICK HERE FOR FREE MATERIAL

Among the above, the statements which are Ans. (a) : The High Courts at Bombay, Madras and
true are Culcutta were established in the year 1861 under the
(a) only 1 & 2 are true Indian High Courts Act, 1861. It was inaugurated on
(b) only 2 & 3 are true 14th August, 1862.
(c) only 3 & 4 are true 55. Which of the following programme does not
(d) only 1 2 and 3 are true come under the Rural Infrastructure
Ans. (*) : The right to life and personal liberty has been Development Fund (RIDF) ?
given in Article 21 of the constitution. It cannot be (a) Rural water supply (b) Rural Roads
suspended in emergency. The vice-president of India (c) Rural Electrification (d) Rural Industries
can be removed from his office by a resolution passed Ans. (d) : Government of India created the Rural
by the Rajya Sabha and agreed by the Lok Sabha. Infrastructure Development Fund (RIDF) in NABARD
NITI Aayog has been established in place of planning in 1995-96, with an initial corpus of Rs 2,000 crore.
commission with effect from 1 January, 2015, whose With the allocation of Rs 40,000 crore for 2021-22
current chairman is Narendra Modi. under RIDF XXVII, the cumulative allocation has
51. Which one of the following statements related reached Rs 4, 18, 402. 73 crore, including 18, 500 crore
to a states Cheif Minister is not true? under Bharat Nirman.
(a) Chief Minister are appointed by Governors. At present, there are 39 eligible activities under RIDF as
approved by GoI. The eligible activities are classified
(b) Generally Cheif Minister preside over the
under three broad categories, i.e,
cabinet meetings.
(c) Governors acts on the advice of chief ministers. → Agriculture and related sector.
(d) Ministers are appointed by Governors on the → Social sector
advice of Cheif Minister. → Rural connectivity.
Ans. (c) : The Chief Minister is appointed by the Therefore, Rural industries is not a part of RIDF.
Governor. Under Article 164 of the Constitution, who is 56. The J.N.N.U.R.M. project is related to
the leader of the majority party in the Legislative bringing/effectively improvement in ?
Assembly. The Chief Minister presides over the (a) Rural house construction
meetings of the Council of Ministers. The Governor (b) Urban & Rural Marketting Infrastructure
appoints other members of the Council of Ministers. (c) Employment of Educated Persons
52. The Article 370 of Indian Constitution is (d) Urban Infrastructure
related to ? Ans. (d) : Jawaharlal Nehru National Urban Renewal
(a) Jammu & Kashmir state (b) Sikkim state Mission (JNNURM) was launched on 3rd December
(c) Nagaland state (d) Manipur state 2005 and closed in 2014. For urban infrastructure. This
Ans. (a) : Article 370 was a constitutional provision scheme is now succeeded by Atal AMRUT Mission.
that gave Jammu and Kashmir its special status. On 5th 57. For Fixation of prices of Public commodities.
August 2019, President of India in the exercise of the Who postulated the idea/concept of "Shadow
powers conferred by clause (1) of Article 370 of the Prices" ?
Constitution has issued the Constitution (Application to (a) J.Tinbergin (b) A.K. Sen
Jammu Kashmir) Order, 2019. Through this, (c) P.C. Mahalnobis (d) R.Nerx
Government of India has made modification in Article Ans. (a) : A shadow price is an estimated price for
370 itself (not revoked it.) with this Government of something that is not normally priced or sold in the
India has dramatically altered the relationship between market. Shadow pricing was coined by J. Tinbergen.
the state of Jammu and Kashmir and the Indian union. 58. Which is the correct base year to measure
53. The dispute related to sharing of Kaveri river National Income in India ?
water is concerned with which of the following (a) 2004-05 (b) 2055-06
states ? (c) 2011-12 (d) 2010-11
(a) Tamilnadu and Karnataka Ans. (c) : National income data is released by the
(b) Tamilnadu, Karnataka and Kerala Central Statistical Office (CSO) at prevailing prices and
(c) Tamilnadu, Karnataka, Kerala & Gujarat constant base year values. The base year for estimating
(d) Tamilnadu, Karnataka, Kerala and Pondicherry GDP and national income data at constant price was
Ans. (d) : The Kaveri river dispute involves 3 states and charged from 2004-05 to 2011-12, but at present the
one union territory namely Tamil Nadu, Kerala, ministry of statistics has charged the base year for GDP
Karnataka and Puducherry. The root of this dispute lie and IIP to 2017-18 and for CP I is 2018 is proposed.
in two agreements between the erstwhile Madras 59. Who is responsible to prepare the Annual
Presidency and the Mysore state in 1892 and 1924. Economic Survey ?
54. When was the Bombay Madras & Calcutta (a) Planning Ministry
High Courts established ? (b) Finance Ministry
(a) 1861 (b) 1862 (c) Reserve Bank of India
(c) 1863 (d) 1864 (d) Central Statistical Organisation
UP RO/ARO Special (Pre) Exam 2010 259 YCT
CLICK HERE FOR FREE MATERIAL

Ans. (b) : The Economic Survey of India is an annual Ans. (d) : The Tarapore Committee was established on
document released by the Finance Ministry, Capital Account Convertability (CAC), it refers to the
Government of India and reviews the developments in ability to convert local financial assets into international
the Indian economy over the past twelve months. financial assets or vice-versa at market - determined
60. In the Co-operative sector the largest fertilizer exchange rates. While other options are correctly matched.
factory is located at - 65. Who is authorized to Circulate coins in India?
(a) Phulpur (U.P) (b) Hazira (Gujarat) (a) RBI (b) Finance Ministry
(c) Haldia (West Bengal) (d) Siwan (Bihar) (c) SBI (d) National Stock Exchange
Ans. (a) : In the co-operative sector the largest fertilizes Ans. (b) : Reserve Bank of India has the sole right to issue
factory is located at Phulpur (U.P). i.e, Indian Farmers currency notes of various denominations except one rupee
Fertilizer Cooperative Limited (IFFCO). This society is
notes under section 22 of Reserve bank of India Act. The
engaged in the business of manufacturing and
marketing of fertilizers. one rupee note and coins are issued by ministry of finance
and it bears the signature of Finance secretary.
61. In 2010-11, What was the contribution of th
agriculture and allied sectors in India Gross 66. The provisions of recommendation of 13
domestic product (GDP). Finance Commission is associated to which
(a) approx 20% (b) approx 15% period ?
(c) approx 10% (d) approx 8% (a) 2005-10 (b) 2009-14
Ans. (b) : According to the Economic Survey 2011-12, (c) 2010-15 (d) 2014-19
th
the share of agriculture and allied sector in GDP at cost Ans. (c) : The 13 Finance Commission was constituted
of production is 14.1%, the share of industry is 30.2% by on 13 November 2007 to make recommendations for
and the share of service sector is 55.7% the period 2010-15. Dr. Vijay Kelkar was appointed the
62. The main objective of Vishesh Krishi Gram chairman of the commission.
Udyog Yojana 67. In which year the full convertibility of rupee in
(a) Promotion of Agriculture exports current account was announced ?
(b) To increase productivity of food crop (a) 1994 (b) 1996
(c) Promoting the cultivation of oil seeds (c) 1998 (d) 2001
(d) Promotion of Cultivation of Pulses Ans. (a) : RBI on 19 August, 1994 in respect of
Ans. (a) : The main objective of Vishesh Krishi and different schemes related to foreign currency non-
gram udyog yojana (VKGUY) are - resident accounts, the rupee was made fully convertible
(1) Agricultural produce and their value - added products. on the current account of the balance of payments.
(2) Minor forest produce and their value - added variants. 68. The commercial bank which was merged with
(3) Gram Udyog products PNB was ?
(4) Forest based products (a) Bank of India
(5) Other such products. (b) New Bank of India
63. Which of the following Financial arrangement (c) Indian Overseas Bank
is correct for Rashtriya Krishi Vikas Yajana ? (d) Oriental Bank of Commerce
State Centre
Ans. (d) : Oriental Bank of Commerce and United Bank
(a) 0% 100% were merged with Punjab National Bank. The government
(b) 25% 75% made this announcement on 30 August, 2019.
(c) 50% 50%
69. Which of the following nation is not a member
(d) 75% 25%
of south Asian Association for Regional
Ans. (a) : Rashtriya Krishi Vikas Yojana (RKVY) was Cooperation?
initiated in 2007 as an umbrella scheme for ensuring (a) India (b) Pakistan
holistic development of agriculture and allied sectors. It
(c) Cambodia (d) Nepal
was converted into a centrally sponsored scheme in
2014-15 also with 100% central assistance. Since, 2015- Ans. (c) : The South Asian Association for Regional
16, the funding pattern of the scheme has been altered Cooperation (SAARC) is an economic and political
in the ratio of 60:40 between centre and states (90:10 organization of eight countries in South Asia. It was
for North Eastern states and Himalayan States). established in 198 when the Heads of State of
Bangladesh Bhutan, India, Maldives, Nepal, Pakistan
64. Which of the following pairs is not correctly and Sri Lanka formally adopted the charter.
matched ?
(a) Goswami committee-Problem of Industry sickness 70. Which among the following has not been
admitted/included to provide protection under
(b) Janaki Raman Committee - Investigation of
Share scandal the TRIPS agreement?
(c) Malhotra Committee - Upgradation of (a) Trade mark
Insurance sector (b) External outline of shape of Integrated Circuit
(d) Taropore Committee - Customer Service in (c) Geographic Indicator
Banks (d) Sapling production
UP RO/ARO Special (Pre) Exam 2010 260 YCT
CLICK HERE FOR FREE MATERIAL

Ans. (d) : Sapling production has not been admitted / conditions, such as Grave's disease, as well liver and
included to provide protection under the TRIPS brain Tumors. Asenic - 74 is used for the detection of
Agreement. While trade mark, external outline of shape of small tumourr in the body. Cobalt - 60, boron - 10 and
integrated circuit and geographic indicator are included to iodine - 131 are used for the treatment of brain tumour.
provide production under the TRIPS Agreement. 76. Which of the following vitamins is helpful in
71. The name of Dunkel is associated with remembering any dream for a sufficient long
(a) W.T.O (b) GATT time period ?
(c) O.C.G.C (d) EXIM Bank (a) Vitamin-A (b) Vitamin-D
Ans. (b) : Arthur Dunkel was the chairman of GATT. (c) Vitamin B6 (d) Vitamin-C
In 1989, he passed a resolution in opposition to the Ans. (c) : Vitamin B6 could help people to recall their
developing countries in the Uruguay Round. Arthur dreams. It enhance the ability to remember dreams. It
Dunkel was a Swiss (born in Portugal) administrator. occurs naturally in a number of foods, including fish,
They signed General Agreements on Tariffs and Trade poultry, organ meats, starchy vegetables such as
between 1980 and 1993. Under his leadership the potatoes and fruits. Vitamin B6 is also available in the
'Dunkel Draft was accepted and the foundation of the form of a dietary supplement.
world Trade organization was laid. 77. Jonas Salk is known for the discovery of ?
72. Which of the following can be used to measure (a) Small Pox (b) Cholera
temperature above 1500ºC? (c) Polio Vaccine (d) None of the above
(a) Clinical Thermometer Ans. (c) : Jonas Salk was an American virologist and
(b) Thermocouple Thermometer medical researcher who developed the first successful
(c) Platinum resistance thermometer polio vaccines. Therefore, the correct answer is option (c).
(d) Pyrometer 78. Which of the following is synthesized by the
Ans. (d) : A pyrometer is a device for measuring very intestinal bacterias?
high temperatures and uses the principle that all (a) Vit-B12 (b) Vit-C
substances emit radiant energy when not, the rate of (c) Vit-K (d) Vit-A
emission depending on their temperature. A Ans. (a) : Vitamin B-12 is synthesized and utilized by
thermoelectric pyrometer instrument can be used for the bacteria in the human gut microbiome and is required
measurement of a temperature above 1500ºC. for over a dozen enzymes in bacteria, compared to only
73. In water, an airbubble functions as - 2 in humans. However, the impact of vitamin B-12 on
(a) a convex mirror (b) a concave lens the gut microbiome has not been established. Hence, the
(c) a concave mirror (d) a convex lens correct answer option (a).
Ans. (b) : The air bubble inside water behaves like a 79. The smallest bone of the human body is ?
concave lens. This is because the normal of any point of (a) Vomer (b) Stapes
a spherical bubble will pass through the center of the (c) Mallens (d) Incus
bubble. It is clearly visible that the air bubble act as a Ans. (b) : There are three bones in the middle ear -
diverging lens. malleus, Stapes and incur. The stapes is the smallest of
74. In Television telecast, the technique employed the three bones and the smallest in the human body. The
for transmission of sound/audio signals is size of the stapes is 3mm × 2.5mm. The femur or thigh
(a) Amplitude Modulation bone is the largest and heaviest bone in the human body.
(b) Pulse Signal Modulation 80. As per Indian Council of Agricultural Research
(c) Frequency Modulation Report what is total number of Agro-ecological
(d) Time Extension Multiplication zones in India?
Ans. (c) : Frequency modulation techniques are used to (a) 15 (b) 17
transmit audio signals in television are television (c) 18 (d) 20
broadcasts. Transmission of matter/video signals from Ans. (d): As par Indian Council of Agricultural
one place to all directions as in one direction is called Research Report there are 20 number of agro -
transmission. At remote locations, these signals are ecological zones (AEZ) in India. AEZ is a uniform as
captured in a suitable manner and after necessary possible in terms of physiographic, climate, length of
changes (amplification de-modulation etc) an audio as growing period and soil type of macro level, land - use
video is obtained. planning and effective transfer of technology.
75. The Radio-isotope employed/used to indicate 81. Zero tillage seeds and Fertiliser drill was
Tumour is - development in ?
(a) As- 74 (Arsenic-74) (b) Co-60 (Cobalt-60) (a) P.A.U., ludhiana
(c) Sodium-24 (d) Carbon-14 (b) G.B. Pant Agricultural and Technological
Ans. (c) : The radioactive elements are widely used in University, Pantnagar
medical field for tumour detection and treatment. (c) I.I.S.R. Lucknow
Soldium 24 is used to monitor goiter and treat thyroid (d) I.A.R.I. New Delhi
UP RO/ARO Special (Pre) Exam 2010 261 YCT
CLICK HERE FOR FREE MATERIAL

Ans. (b) : Zero tillage seeds and fertilizer drill was 89. The largest Telescope of the world is
developed in G.B. Pant Agricultural and Technological (a) Galileo (b) Icecube
University, Pantnagar. (c) Discovery (d) Challenger
82. Central Arid Zone Institute is located at ? Ans. (b) : When the question was asked, the world's
(a) Hyderabad (b) Jodhpur largest telescope was Icecube. Currently, the world's
(c) Ahmadabad (d) Bangalore largest telescope is China's Aperture Spherical Radio
Ans. (b) : Central Arid Zone Institute is located in Telescope, nicknamed Tianyan.
Jodhpur, Rajasthan. There are five Regional Research
Stations located in different agro - climatic zones to 90. Consider the following statements :-
work on location - specific problems. 1. American Agency NASA has launched its
83. Which one among the following is not found in dream machine - Mars Science laboratory
Tea ? (Curiosity Rover) which will examine the
(a) Theine (b) Caffeine prospect of life on planet Mars.
(c) Tannin (d) Morphine 2. Curiosity Rover is a solar powered vehicle.
Ans. (d) : Morphine is not found in Tea. While Theine, 3. In the Rover there is a provision of a
Caffeine and Tannin are mainly found in tea. Hence, the special chemical Science Key and
correct answer option (d). Camera which can detect micro level
84. Which of the following is generally used as an elements in the substances present on
adulterator in Mustard seeds ? mars surface.
(a) Seeds of Argemone (b) Papaya seeds Which of the above statements are true ?
(c) Cumin seeds (d) Coriander seeds (a) only 1 (b) only 1 & 2
Ans. (a) : The argemone seeds look like mustard seeds. (c) only 2 & 3 (d) All of them are true
They nature with mustard crop and may be harvested Ans. (d) : In order to search for life on Mars, the US
along with mustard. Argemone seeds (or oil) can be space agency NASA launched the Curosity Rover by
mixed with mustard (oil) to adulterate it. The Atlas V rocket on 26 November, 2011 from Cape
contamination may sometimes be accidental. Canaveral Air Force Station in Florida under the Mars
85. The state known as 'Spice Garden' is ? Science Laboratory Mission. The rover is powered by
(a) Kerala (b) Kamataka electricity from radioisotope generators on Mars, using
(c) Tamilnadu (d) Gujarat plutonium 238 as fuel.
Ans. (a) : Kerala is known as the 'spice garden' of India 91. The Submarine Launched Ballistic Missile
as it is known for producing great varities of spices. The
spices are exported from Kerala to all parts of the (SLBM)
world, the state has become the 'Spice Trade Hub'. (1) It has been developed by India's D.R.D.O.
86. The Blue green Algae is used as a Biofertiliser (2) It will deployed on Indian Submarine - INS
for increasing production of which of the Arihant.
following crop ? (3) Its strike range is 700 kms Which of the
(a) Wheat (b) Paddy above statements are true ?
(c) Gram (d) Mustard (a) 1 only (b) 1 and 2
Ans. (b) : Cynobactaria are one of the major (c) 2 and 3 (d) 1, 2 and 3
components of the nitrogen fixing biomass in paddy Ans. (d) : The K family of missiles are primarily
fields. The agricultural importance of cynobactaria in submarine launched Ballistic Missile (SLBMS). They
rice cultivation is directly related with their ability to fix have been indigenously developed by the Defence
nitrogen and other positive effects for plants and soil.
Research and Development organization (DRDO).
87. Which of the following is a source of Green Shaurya is a land variant of short range SLBM K-15
fuel?
Sagarika, which has a range of at least 750 Kilometers.
(a) Pine (b) Karanji
India has also developed and successfully tested
(c) Fern (d) None of them
multiple times the K-4 missiles from the family which
Ans. (b) : Karanji in a source of green fuel. Apart from has a range of 3500 Km.
this medicinal properties are found in it. Karanji is They are launched from Arihant class of nuclear
beneficial in diseases like baldness, eyes, teeth diseases,
submarines. Hence, all the statements are correct.
leprosy, stomach disorders, piles, etc.
92. To establish the paternity of any child. Which
88. Which is the 20th Nuclear Power Station of
India ? of the following techniques can be employed?
(a) Tarapore (b) Rawatbhata (a) Protein Analysis
(c) Kaiga (d) Narora (b) D.N.A. finger printing
Ans. (c) : Kaiga Generating Station is a nuclear power (c) Chromosomal Counting
generating station situated at Kaiga, district of Karnataka. (d) Quantitative analysis of D.N.A.
The plant has been in operation since March 2000 and is Ans. (b) : To establish the paternity of any child DNA
operated by the Nuclear Power Corporation of India. finger printing technique can be employed.
UP RO/ARO Special (Pre) Exam 2010 262 YCT
CLICK HERE FOR FREE MATERIAL

93. The Information Technology terminology Ans. (a) : Uttar Pradesh is place where Kathak, one of
which is used to ascertain the e-mail Id of the the six classical dances of India, flourished. The word
user is ? Katha, from which the name is derived, means story. In
(a) Login (b) Password originated in northern India, during the 7th century AD.
(c) Finger (d) Archie Kathak has very intricate and complex movements of
Ans. (c) : Finger is a program that tells you the name hands and feets along with facial expressions.
associated with an E-mail address. It may also tell you 99. Which is the famous Pilgrimmage Centre of
whether they are currently logged in at their system or both Jain & Buddist religion ?
their most recent login session and possibly other (a) Sarnath (b) Kaushambi
information, depending on the data that is maintained (c) Kushinagar (d) None of them
about users on that computer.
Ans. (c) : Kushinagar belong to the Great Lord Buddha,
94. Which of the following acts as a bio-indicator founder of Buddhism, who delivered his last sermon,
of air pollution? attained "Mahaparinirvana" and was cremated at
(a) Lichen (b) Fern Ramabhar (Kushinagar).
(c) Money Plant (d) Amarbel It is also associated with Lord Mahavir, 24th tirthankar of
Ans. (a) : Lichen are sensitive to atmospheric pollution jain sect, founder of Jainism. It is believed that Lord
such as nitrogen (N) because they receive all their Mahavir passed away or attained Parinirvana at Pawapuri.
nutrients and water from wet and dry atmospheric 100. Dhola folk song is famous in ?
deposition (fall out). Nitrogen deposition can increase
(a) Agra-Meerut
the load of nutrients. Too much can harm and kill the
algae's chlorophyll which is used to produce sugars (b) Allahabad- Kanpur
feeding it and the fungi. (c) Gorakhpur - Jaunpur
95. The Mobilephones & Automobiles has brought (d) Lucknow - Saharanpur
a revolution in the social lives of Indians Ans. (a) : In Uttar Pradesh, the practice if Dhola and
especially Rural area people in terms of ? Ragini is in the western region. Therefore, the correct
(1) In terms of movement/travelling of people. answer would be Agra - Meerut.
(2) In terms of Peoples connectivity. 101. Which of the following is not correctly matched?
(3) In terms of sensitivity of people. (a) Oldest University of U.P.-Allahabad University
Choose the correct answer from the codes (b) Nauchandi fair - Meerut
given below : (c) Bharat Kala Bhavan - Varanasi
(a) only 1 & 2 (b) only 1 & 3 (d) Chaurasia Dance - Gorakhpur
(c) only 2 & 3 (d) 1, 2 & 3 Ans. (d) : The correct match is -
Ans. (d) : Mobile phones and automobiles have made Oldest University of U.P - Allahabad University
the following impact on the social life of Indians, Nauchandi fair - Meerut
especially in rural areas - Bharat Kala Bhavan - Varanasi
• In terms of movement of people; Chaurasia Dance - Kahars in Jaunpur
• In terms of people connectivity; Therefore, the incorrect answer is option (d).
• In terms of sensitivity of people. 102. Which of the following is not matched ?
Therefore, the correct answer is option (d). (a) Scooter India Ltd. - Lucknow
96. Which of the following fuel produces (b) Modern Bakeries - Kanpur
lowest/minimum environmental pollution? (c) Cement factory - Bareily
(a) Diesel (b) Coal (d) Fertiliser plant - Aonla
(c) Hydrogen (d) Kerosene
Ans. (c) : The correct match is -
Ans. (c) : Hydrogen fuel produces lowest / minimum Scooter India Ltd. - Lucknow
environmental pollution.
Modern Bakeries - Varanasi
97. In U.P Potato Export Zone is located at - Cement Factory - Churk and Dalla
(a) Agra (b) Allahabad Fertilizer Plant - Aonla
(c) Fatehpur (d) Kanpur Therefore, the correct answer is option (d).
Ans. (a) : The Government of Uttar Pradesh has
103. In U.P. Export oriented software park is
established on export zone for the export of potatoes and
mangoes, in which the Agra zone has been established for established in ?
potato exports. Agra, Farrukhabad, Kannauj, Hathras, (a) Noida and Kanpur
Meerut etc. districts come under the Agra zone. (b) Kanpur and Lucknow
98. Which is the distinct folk dance of Uttar (c) Noida and Agra
Pradesh ? (d) Meerut and Ghaziabad
(a) Kathak (b) Bharat Natyam Ans. (c) : In U.P Export oriented software park is
(c) Odissi (d) Kuchipudi established in Nodia and Agra.
UP RO/ARO Special (Pre) Exam 2010 263 YCT
CLICK HERE FOR FREE MATERIAL

104. Match List-I and List-II and choose the correct 107. If ABSENT is coded as ZYHVMG then what
answer from the codes given below– will be the coded word of PRESENT -
List-I (Park/Zone) List-II (District) (a) KIHVHGM (b) KITMHMG
A) Apparel Park 1. Unnao (c) KIVHVMG (d) GKITYTL
B) Biotechnology Park 2. Saharanpur Ans. (c) : Just as-
C) Agro-Processing 3. Kanpur A 
opposite word
→Z
Zone
B 
→Y
D) Leather Technology 4. Lucknow
Park S 
→H
Code : E 
→V
A B C D N 
→M
(a) 3 4 2 1
(b) 4 3 1 2 T 
→G
(c) 2 1 3 4 Same as-
(d) 3 2 4 1 P 
opposite word
→K
Ans. (a) : The correct match is - R 
→I
Apparel Park - Kanpur
Biotechnology - Kushinagar E 
→V
Agro-Processing Zone - Saharanpur S 
→H
Leather-Technology Park - Unnao E 
→V
105. Match List-I and List-II and choose the correct
N 
→M
answer from the codes given below -
List-I (Historical List-II (Location) T 
→G
buildings in U.P.) 108. In the series 1, 3, 7, x, 31, y
A) Chaurasi Gumbad 1. Chitrakoot the value of x & y are respectively -
B) Jama Masjid 2. Kushinagar (a) 14 & 60 (b) 13 & 63
C) Budhist Stupas 3. Moradabad (c) 15 & 60 (d) 15 & 63
D) Kamadgiri Mountain 4. Jalaun Ans. (d) :
Code :
A B C D
(a) 1 2 3 4
(b) 4 2 3 1 2, 4, 8, 16 and 32 are added to the series respectively.
(c) 4 3 2 1
109. In a group of 500 persons, 300 persons can
(d) 3 4 1 2 speak Hindi only and 120 can speak only
Ans. (c) : The correct match is - English. Then what is the number of persons
Chaurasi Gumbad - Jalaun who can speak both Hindi & English -
Jama Masjid - Muradabad (a) 20 (b) 80
Buddhist Stupa - Kushingar (c) 60 (d) 100
Kamadgiri Mountain - Chitrakoot
Ans. (b) : We have,
106. Which of the following pairs is not correctly 500 = 300 + 120 + x
matched?
x = 500 – 300 – 120
(a) Lal Bahadur Shastri Ganna Vikas Sansthan -
Meerut x = 80
(b) PICUP - Lucknow Thus 80 persons speak both English and Hindi.
(c) State Leather Development & Marketing 110. A is younger to B by 5 years and age of B is
Corp. Ltd. - Agra three times that of C. If age of C was 10 years
(d) U.P.F.C. (Uttar Pradesh Finance Corporation) four years ago, then what will be the age of A
- Kanpur after 6 years.
Ans. (a) : The correct match is - (a) 35 years (b) 43 years
Lal Bahadur Shastri Garma Vikas - Lucknow (c) 40 years (d) 30 years
Sansthan Ans. (b) : Present Age
PICUP - Lucknow
State Leather Development - Agra
Marketing Corp Ltd.
UPFC (Uttar Pradesh Finance - Kanpur 4 Year before, (3x – 5 – 4), (3x – 4), (x – 4)
Corporation) x – 4 = 10 {Given}
Hence, the correct answer option (a). x = 14 Years
UP RO/ARO Special (Pre) Exam 2010 264 YCT
CLICK HERE FOR FREE MATERIAL

Age of A after 6 Years n −1


= 3x – 5 + 6 Ans. (b) : Rate = × 100
t
= 3x + 1
2 −1
= 14 × 3 + 1 = ×100
= 42 + 1 16
= 43 Years 25 1
= = 6 %
111. What is the number in the vacant segment (?) 4 4
116. If 4 is subtracted from any number then it
becomes 21 times inverse of the original
number. What is that number?
(a) 10 (b) 9
(c) 8 (d) 7
Ans. (d) : Let number be x.
(a) 29 (b) 39 According to question,
(c) 37 (d) 49
( x − 4 ) = 21 
1
Ans. (b) : 3×2 -1 = 5 , 5×2-2 = 8
8×2-3 = 13, 13×2-4 = 23 x
and 22×2-5 = 39 ⇒ x − 4x − 21 = 0
2

112. Which of the following is different from all ⇒ x − 7x + 3x − 21 = 0


2

other words ? ⇒ x ( x − 7 ) + 3( x − 7 ) = 0
(a) Telegram (b) E-mail
(c) FAX (d) Messenger ⇒ ( x − 7 )( x + 3) = 0
Ans. (d) : In the given option, messenger is odd one Hence, x = 7, –3
because if works with facebook app, while other option 117. If 40 students out of 100 students fail in
work individual. English, 20 fail in Hindi and 12 students fail in
113. If the 7th of any month falls two days before Hindi and English both, then what is the
Sunday, then what will be the next day after number of students who have passed in both
the 27th of the month? the subjects?
(a) Friday (b) Saturday (a) 48 (b) 52
(c) Thursday (d) Monday (c) 28 (d) 40
th
Ans. (b) : 7 day of the month = (Sunday -2) Ans. (b) : Total no. of students = 100
= Friday Students fail in English = 40
Next day after the 27th day of the month = (27+1)th day Students fail in Hindi = 20
= 28th day Students fail in English and Hindi both = 12
= (7×4)th day ∵ Total fail students = 60-12 = 48
= Friday Hence, the no. of students passed in both subject = 100-
114. If the sum of 3/8 part and 2/7 part of any 48 = 52
number is 370, then what is the number 118. The speed of a boat in calm water is 6 km/hr
(a) 520 (b) 540 and it covers a distance of 1 km in 15 minute
(c) 500 (d) 560 crossing against the flow of river current. Then
Ans. (d) : Let the number is x. what is the speed of River water current ?
3 x×2 (a) 2 km/hr (b) 3 km/hr
∴ x× + = 370 (c) 4 km/hr (d) None of them
8 7
Ans. (a) : Let the speed at flow of river water = x Km/h
3x 2x
+ = 370 The speed at boat in calm water = 6 Km/h
8 7
The speed at boat against the flow of river current = (6-
21x + 16x
= 370 x) Km/h
56 Distance
37x = 370 × 56 ∵ Speed = ,
Time
x = 560
1
115. At what rate of simple interest will an amount ∴ ( 6 − x ) =
become two times after 16 years? (15 60 )
1 1 ⇒ 6−x = 4
(a) 6 % Annual (b) 6 % Annual
2 4 ⇒ x = 6−4
3 ⇒ x = 2 Km / h
(c) 6% Annual (d) 6 % Annual
4
UP RO/ARO Special (Pre) Exam 2010 265 YCT
CLICK HERE FOR FREE MATERIAL

2
– Ans. (d) : South Sudan is the world's latest
 1  3
internationally recognized country. It was declared
119. The value of  –  is
 343  independent from Sudan on July 9,2011.
(a) 49 (b) –49 126. Hurricane Irene caused extreme damage to
1 1 which of the following country?
(c) (d) –
49 49 (a) Japan (b) Philippines
Ans. (a) : (c) Spain (d) United States of America
−23 Ans. (d) : Hurricane Irene was a destructive tropical
 1  cyclone which affected Caribbean and East Coast of the
− 
 343  United States of America during late August 2011. It
−2 −2×
3 caused huge damage to New York and New England
 1 3  1 3 region of USA.
= − 3  = − 
 7   7 127. Which among the following won silver medal in
= (–7)2 London Olympics?
= 49 (1) Saina Nehwal
120. Who is the Chairperson of Press Council of (2) Gagan Narang
India (3) Sushil Kumar
(a) P.L. Punia (b) Markandey Katju (4) Vijay Kumar
(c) Kuldeep Nayyar (d) Pooja Bhatt Choose the correct answer from the codes
Ans. (b) : The chairperson of press council of India given below-
from 2011 to 2014 was Justice Markandey Katju. At (a) 1 and 4 (b) 2 and 3
present, the chairperson is Justice C.K. Prasad. (c) 3 and 4 (d) 1 and 3
121. Who is the chairman of Competition Ans. (c) : The London Olympics was held from 27 July
Commission of India ? 2012 to 12 August 2012 in London, UK. In this multi
(a) Ashok Chawla (b) Mohan Dharia sports event India secured 6 medals including 2 silver
(c) Pankaj Oswal (d) None of them and 4 bronze and 56th rank. India's medal winners
were- Vijay Kumar (silver), Sushil Kumar (silver),
Ans. (a) : Ashok Chawla was chairperson of the
Gagan Narang (Bronze), Saina Nehwal (Bronze) and
Competition Commission of India (CCI) from 2011 to
2016. In 2018, Ashok Kumar Gupta was appointed as Yogeshwar Dutt (Bronze).
the chairperson of CCI. 128. Which of the following pair is NOT correctly
122. Which of the following countries was the top matched?
counting for India's export products in the year (2011 Nobel prize winners) (field)
2010-11? (a) Adam G. Riess Physics
(a) UAE (b) USA (b) Bruce A. Beutler Chemistry
(c) China (d) Singapore (c) Brain P. Schmidt Physics
Ans. (a) : According to economic survey 2010-11 top (d) Dan Shechtman Chemistry
country for India's export products in the year 2010-11 Ans. (b) : Bruce A. Beutler won Nobel Prize in
was UAE (United Arab Emirates) whereas for the year Physiology or medicine in 2011 and not in chemistry.
2021-22 it is United States of America (USA). Reats are correctly matched.
123. in the year 2010-11, export of goods and 129. Who among the following have won Asian
services from India, was what percent of its Billiards championship in April 2012?
GDP? (a) Anju Thakur (b) Ashok Shandilya
(a) 26% (b) 24% (c) Pankaj Advani (d) Tarundeep Rai
(c) 22% (d) 20% Ans. (c) : Pankj Advani won the ONGC Asian Billiards
Ans. (c) : in the year 2010-11, the share of export of championship in April 2012 by defeating Thawat
goods and services in GDP was 22% whereas in 2021- Sujaritthurakarn of Thailand in the final held in Panaji.
22 it is approximately 21.4% 130. In June 2012, a massive fire damaged the holy
124. Who is the author of the book "India from shrine of Dastgir Sahib in Srinagar.Who among
Curzon to Nehru and After"? the following was known as Dastgir Sahib?
(a) Durga Das (b) Webb Miller (a) Bahauddin Zakaria
(c) Frank Moraes (d) Louise Fischer (b) Syed Abdul Jilani
Ans. (a) : Durga Das is author of the book, ''India from (c) Khwaja Baqi Billah
Curzon to Nehru & After". (d) Qutubuddin Bakhtiyar Kaki
125. Which of the following is the newest nation of Ans. (b) : Syed Abdul Qadir Jilani was known as
the world? Dastgir Sahib. The Shrine of Dastgir Sahib was 200
(a) Eritrea (b) Ethiopia year old wooden architecture which was damage by fire
(c) Congo (d) South Sudan in June-2012.

UP RO/ARO Special (Pre) Exam 2010 266 YCT


CLICK HERE FOR FREE MATERIAL

131. Where was the SARC Summit held in 136. Consider the following statements
November 2011? (1) Agni II missile that was launched recently
(a) Maldives (b) Myanmar has surface to surface range of 2000km
(c) Nepal (d) Sri Lanka (2) Helle Thorning Schmidt is the first female
Ans. (a) : The 17th SAARC Summit was held during Prime Minister of Denmark.
10-11 November 2011 at Addu city of Maldives. (3) June 3 is observed as International Day for
older persons.
132. Which of the following country is the host of
(4) 'The Untold story ' was written by B.M.
2018 winter Olympics? Kaul
(a) France (b) Germany Which among the above statements are correct
(c) Japan (d) South Korea (a) Only 1 and 2 (b) Only 2 and 3
Ans. (d) : Pyeongchang city of South Korea was the (c) Only 2,3 and 4 (d) Only 1,2 and 4
host city of the 2018 Winter Olympics, whereas the Ans. (d) : United Nations General Assembly designated
2022 Winter Olympics was held in Beijing of china. October 1 as the International Day for Older Persons
133. Which one of the following pair is NOT Hence statement (3) is incorrect. Rest of the given
correctly matched with reference to the 4th statements are correct.
BRICS Summit held in March 2012? 137. Adam's Bridge starts from-
(Country) (Representative) (a) Dhanushkodi (b) Mandapam
(a) Brazil Dilma Rousseff (c) Pamban (d) Rameshwaram
(b) China Hu Jintao Ans. (a) : Adam's bridge starts as a chain of shoals from
(c) Russia Vladimir Putin the Dhanushkodi of Pamban Island. it ends at Sri
(d) South Africa Jacob Zuma Lanka's Mannar Island.
Ans. (c) : The 4th BRICS summit was hosted by India 138. Which of the following years was declared by
at New Delhi in 2012 which was attended by Brazilian the United Nations General Assembly (UNGA)
President Dilma Rousseff, Chainese President Hu as the International Year of Rice?
Jintao, Russian Presiden Dmitry Medvede and South (a) 2004 (b) 2005
African President Jacob Zuma. Therefore option (c) is (c) 2008 (d) 2009
not correctly matched. Ans. (a) : The year 2004, was declared as the
134. Which among the following statements is not International Year of Rice by the United Nations
correct? General Assembly (UNGA).
(a) As per census 2011 (Provisional data), 139. Who was the prime minister of India when
population of Uttar Pradesh is more than that India had to keep gold in foreign Banks?
of Brazil. (a) P.V. Narsimha Rao (b) V.P. Singh
(b) Mary Kom won Bronze medal in boxing in (c) Rajiv Gandhi (d) Chandrashekhar
London Olympics 2012. Ans. (d) : Shri Chandrashekhar was the prime minister
(c) World Breastfeeding week was observed of India who pledged gold in foreign banks.
from August 1 to 7 in 2012 140. Match List I with List II and select the correct
(d) In London Olympics 2012, Vijay Kumar was answer using the codes given below the list
flag bearer of Indian team, in the opening List I List II
ceremony. (Historic Place ) (States)
Ans. (d) : In the opening day ceremony of London A. Bhimbetka 1. Assam
Olympics 2012 Sushil Kumar was Flag bearer for B. Shore Temple 2. Madhya Pradesh
Indian team while in closing ceremony Mary Kom was C. Hampi 3. Tamil Nadu
the flag bearer . D. Manas 4. Karnataka
135. Which one of the following statement is not Cade
correct? A B C D
(a) The famous 'God particles' is Higgs boson. (a) 2 1 3 4
(b) Daman and Diu is the smallest union territory (b) 2 3 4 1
in terms of area (c) 1 3 4 2
(c) The latest Ramon Magsaysay Award winner (d) 4 2 3 1
from India is Kulandei Francis. Ans. (b) : The correct match is an follows-
(d) India won 4 Bronze medals is the 2012 (Historic Palce) (States)
London Olympics. Bhimbetka Madhya Pradesh
Ans. (b) : In India smallest Union territory is Shore Temple Tamil Nadu
Lakshadweep with the area of 32km2. Hence statement Hampi Karantaka
(b) is in correct Rests are correct statement. Manas Assam
UP RO/ARO Special (Pre) Exam 2010 267 YCT
CLICK HERE FOR FREE MATERIAL

Gòej ØeosMe meceer#ee DeefOekeâejer/meneÙekeâ meceer#ee DeefOekeâejer efJeMes<e (Øeer.) hejer#ee, 2010
meeceevÙe efnvoer
nue ØeMve-he$e
1. FveceW Skeâ ‘uenj’ keâe heÙee&ÙeJeeÛeer Meyo veneR nw : JÙeeKÙee – jmee keâe heÙee&ÙeJeeÛeer Meyo leefceœee veneR nw~ DevÙe efJekeâuhe
(a) JeerefÛe (b) ogketâue jlveieYee&, DeÛeuee Deewj Oeefj$eer, jmee kesâ heÙee&ÙeJeeÛeer Meyo nw~peyeefkeâ
(c) lejbie (d) efnueesj ‘leefceœee’ jeef$e keâe heÙee&ÙeJeeÛeer Meyo nw~
Gòej─(b) 8. ‘ce=iesvõ’ keâe heÙee&ÙeJeeÛeer Meyo nw
JÙeeKÙee – uenj keâe heÙee&ÙeJeeÛeer Meyo nw – JeerefÛe, lejbie, efnueesj nw, (a) kegâjbie (b) kesâmejer
peyeefkeâ ogketâue keâe heÙee&ÙeJeeÛeer JeŒe neslee nw~ (c) Yegpebie (d) legjbie
2. FveceW mes Skeâ Meyo ‘osJelee’ keâe heÙee&Ùe nw : Gòej─(b)
(a) DeuekesâMe (b) efJeyegOe JÙeeKÙee – ce=iesvõ keâe heÙee&ÙeJeeÛeer Meyo kesâmejer nw~ Fmekesâ DevÙe
(c) Deveerkeâ (d) pÙeesefle<keâ heÙee&ÙeJeeÛeer Meyo JÙeeIeÇ, ce=ieeefj, ce=iejepe, hegC[jerkeâ, Deeefo nw peyeefkeâ
Gòej─(b) Yegpebie, mehe& keâe heÙee&Ùe nw legjbie, IeesÌ[s keâe heÙee&Ùe nw~
JÙeeKÙee – efoÙes ieÙes efJekeâuheeW ceW osJelee keâe heÙee&Ùe efJeyegOe nw~ osJelee kesâ 9. ‘<ešdheo’ keâe heÙee&ÙeJeeÛeer Meyo nw
DevÙe heÙee&ÙeJeeÛeer Meyo nw – megj, Decej, osJe, DeeefolÙe, ieerJee&Ce (a) efleleueer (b) Yeücej
Deeefo~ (c) cekeâÌ[er (d) kesâkeâÌ[e
3. ‘yeieerÛee’ keâe heÙee&ÙeJeeÛeer Meyo nw Gòej─(b)
(a) efvepe&ve (b) JÙepeve JÙeeKÙee – <ešdheo keâe heÙee&ÙeJeeÛeer Meyo Yeücej nw~ Fmekesâ DevÙe heÙee&Ùe nQ
(c) Deejece (d) keâuheMeeue – YeBJeje, Deefue, efÉjsHeâ, ceOeghe Deeefo~
Gòej─(c) 10. FveceW mes Skeâ Meyo ‘ceÚueer’ keâe heÙee&ÙeJeeÛeer veneR nw :
JÙeeKÙee – yeieerÛee keâe heÙee&ÙeJeeÛeer Meyo Deejece nw~ efvepe&ve, JÙepeve Ùee (a) celmÙe (b) ceerve
keâuheMeeue keâe yeieerÛee mes keâesF& DeeMeÙe veneR~ (c) MeHeâjer (d) peueesojer
4. FveceW mes ‘legjbie’ Meyo keâe heÙee&Ùe nw : Gòej─(d)
(a) Jeeefpe (b) DecyegOej JÙeeKÙee – ceÚueer keâe heÙee&ÙeJeeÛeer Meyo ceerve, celmÙe, MeHeâjer nw,
(c) efJepeve (d) efkebâkeâj peyeefkeâ ‘peueesojer’ ceÚueer mes mecyevOe veneR jKelee~
Gòej─(a) 11. ‘F&efhmele’ Meyo keâe efJeueesce nw
JÙeeKÙee – legjbie keâe heÙee&ÙeJeeÛeer Meyo nw – Jeeefpe~ Fmekesâ DevÙe heÙee&Ùe (a) Deveerefhmele (b) DeYeerefhmele
Jeeues Meyo nw – Ieesškeâ, IeesÌ[e, mewvOeJe, nÙe, jefJe-heg$e, Deyee&~ (c) DeOeerefhmele (d) kegâeflmele
5. ‘efJe[ewpee’ heÙee&ÙeJeeÛeer Meyo nw Gòej─(a)
(a) ‘ve#e$e’ keâe (b) ‘Dehmeje’ keâe JÙeeKÙee – Fefhmele Meyo keâe efJeueesce Deveerefhmele nw, DeYeerefhmele,
(c) ‘Fvõ’ keâe (d) ‘helLej’ keâe DeOeerefhmele SJeb kegâeflmele Meyo Fmemes he=Lekeâ Meyo nw~
Gòej─(c) 12. ‘mJepeeefle’ keâe efJeueesce nw
JÙeeKÙee – efJe[ewpee heÙee&ÙeJeeÛeer Meyo Fvõ keâe nw~ Fvõ kesâ DevÙe (a) Depeeefle (b) kegâpeeefle
heÙee&ÙeJeeÛeer Meyo nw – megjheefle, MeÛeerheefle, ceOeJee, Me›eâ, hegjvoj, (c) megpeeefle (d) efJepeeefle
osJejeÙe, Decejheefle, Je»eOej Deeefo~ Gòej─(d)
6. ‘ceej’ Meyo heÙee&ÙeJeeÛeer nw JÙeeKÙee – mJepeeefle keâe efJeueesce efJepeeefle neslee nw~ peyeefkeâ kegâpeeefle,
(a) ‘peeot’ keâe (b) ‘mJeCe&’ keâe megpeeefle SJeb Depeeefle keâe mebyebOe mJepeeefle mes veneR nww~
(c) ‘DeOece’ keâe (d) ‘Devebie’ keâe 13. ‘Gvcetueve’ keâe efJeueesce nw
Gòej─(d) (a) Deecegueve (b) efveceerueve
JÙeeKÙee – ceej Meyo keâe heÙee&ÙeJeeÛeer Meyo Devebie nw~ Devebie keâe (c) mecetueve (d) jesheCe
DeeMeÙe keâeceosJe mes nw~ Fmekesâ DevÙe heÙee&ÙeJeeÛeer Meyo nw – cevceLe, Gòej─(d)
ceove, jefleheefle, kegâmegcesMe, ceveefmepe, cekeâjOJepe, keâvohe&, ceervekesâleg JÙeeKÙee – Gvcetueve keâe efJeueesce jesheCe neslee nw~ efveceerueve, mecetueve,
Deeefo~ Deecegueve keâe Fmemes keâesF& mecyevOe veneR nw~
7. FveceW mes Skeâ ‘jmee’ keâe heÙee&ÙeJeeÛeer Meyo veneR nw : 14. ‘Deefcele’ Meyo keâe efJeueesce nw
(a) jlveieYee& (b) DeÛeuee (a) megefcele (b) kegâefcele
(c) Oeefj$eer (d) leefceœee (c) heefjefcele (d) ogefce&le
Gòej─(d) Gòej─(c)
UP UDA/LDA Special (Pre) General Hindi 2010 268 YCT
CLICK HERE FOR FREE MATERIAL

JÙeeKÙee – Deefcele keâe efJeueesce heefjefcele neslee nw~ kegâefcele, ogefce&le SJeb JÙeeKÙee – efoÙes ieÙes efJekeâuheeW ceW Megæ JeekeäÙe nw- nceW keâYeer nleeslmeeefnle
megefcele keâe Fme Meyo mes keâesF& DeeMeÙe veneR nw~ veneR nesvee ÛeeefnS~ Mes<e efJekeâuheeW kesâ JeekeäÙe DeMegæ nQ~
15. ‘ßeerieCesMe’ Meyo keâe efJeueesce nw 23. efvecveefueefKele ceW Megæ Jele&veer keâe Meyo nw :
(a) Fefle (b) Fefleßeer (a) JeeB*dceÙe (b) JeebieceÙe
(c) DeLe (d) FlÙeeueced (c) Jee*dceÙe (d) yeeBieceÙe
Gòej─(b) Gòej─(c)
JÙeeKÙee – ßeerieCesMe keâe Meyo efJeueesce ‘Fefleßeer’ nw~ peyeefkeâ DeLe keâe JÙeeKÙee – ‘Jee*dceÙe’ Megæ Jele&veer Jeeuee Meyo nw~ Mes<e efJekeâuheeW kesâ
efJeueesce ‘Fefle’ neslee nw~ Meyo $egefšhetCe& nQ~
24. FveceW mes keâewve mee JeekeäÙe Megæ nw?
16. ‘Oeveer’ keâe efJeueesce Meyo nw
(a) cesje ieghle jnmÙe keâesF& veneR peevelee~
(a) Oevenerve (b) DeOeerve
(b) MÙeece meppeve Deeoceer nw~
(c) DeOeveer (d) efveOe&ve (c) Gòej keâe DeefOekeâebMe Yeeie heneÌ[er nw~
Gòej─(d) (d) FveceW mes Skeâ Yeer JeekeäÙe Megæ veneR nw~
JÙeeKÙee – Oeveer keâe efJeueesce ‘efveOe&ve’ neslee nw~ DeOeveer, DeOeerve SJeb Gòej─(d)
Oevenerve, Oeveer kesâ efJeueesce Meyo veneR nw~ JÙeeKÙee – efoÙes ieÙes efJekeâuheeW ceW efJekeâuhe (a) ceW ieghle jnmÙe Meyo keâe
17. ‘Gvceerueve’ keâe efJeueesce Meyo nw ØeÙeesie Skeâ ner YeeJe mhe° keâjlee nw pees hegvejeJe=efòe keâe Åeeslekeâ nw~ Dele:
(a) DeJeceerueve (b) megcesueve jnmÙe ner heÙee&hle DeeMeÙe jKelee nw~ Fmeer Øekeâej MÙeece meppeve Deeoceer
(c) Devegceerueve (d) efveceerueve nw~ ÙeneB hej MÙeece meppeve nw mes YeeJe keâer hetefle& nes pee jner nw~ efJekeâuhe
Gòej─(d) (c) ceW DeefOekeâebMe Yeeie ceW DeefOekeâ + DebMe hegve: Deeies ØeÙegòeâ Yeeie DebMe
Meyo keâe hegvejeJe=efòe nw~ Dele: Ùen JeekeäÙe Yeer oes<ehetCe& nw~ Dele: efJekeâuhe
JÙeeKÙee – Gvceerueve keâe efJeueesce Meyo ‘efveceerueve’ neslee nw~ Mes<e
(d) mener Gòej nw~
efJekeâuheeW kesâ Meyo Demebiele nQ~
25. FveceW mes Jele&veer keâer Âef° mes Megæ nw :
18. ‘DeefOeke=âle’ Meyo keâe efJeueesce nw
(a) DevleOee&ve (b) <e‰dce
(a) DeveeefOeke=âle (b) DeveefOeke=âle (c) menŒe (d) Deveg<ebefiekeâ
(c) DeveeefOekeâeefjkeâ (d) ØeeefOeke=âle Gòej─(a)
Gòej─(b)
JÙeeKÙee – Jele&veer keâer Âef° mes efJekeâuhe (a) DevleOee&ve Megæ nw~ DevÙe
JÙeeKÙee – DeefOeke=âle keâe efJeueesce ‘DeveefOeke=âle’ neslee nw peyeefkeâ ØeeefOeke=âle, efJekeâuhe <e‰dce, menŒe SJeb Deveg<ebefiekeâ Jele&veer keâer Âef° mes $egefšhetCe& nw~
DeveeefOeke=âle Deewj DeveeefOekeâeefjkeâ keâe DeefOeke=âle mes keâesF& mecyevOe veneR nw~ Fmekeâe Megæ Meyo nesiee - <e‰, menœe SJeb Deeveg<ebefiekeâ~
19. ‘Devegueesce’ Meyo keâe efJeueesce nw 26. Fve veeceeW ceW mes Skeâ keâer Jele&veer DeMegæ nw :
(a) megueesce (b) Øeefleueesce (a) keâewmeuÙee (b) cewefLeueerMejCe ieghle
(c) Deveueesce (d) DeJeueesce (c) cesIeveeo (d) jengue meebmke=âlÙeeÙeve
Gòej─(b) Gòej─(d)
JÙeeKÙee – Devegueesce Meyo keâe efJeueesce ‘Øeefleueesce’ neslee nw~ Mes<e JÙeeKÙee – Jele&veer keâer Âef° mes DeMegæ Meyo efJekeâuhe (d) nw~ Fmekeâe
efJekeâuheeW kesâ Meyo Demebiele nQ~ Megæ Jele&veer ¤he nesiee – jengue meebke=âlÙeeÙeve~ Fmekesâ Deefleefjòeâ DevÙe
20. ‘heefjßece’ keâe efJeueesce nw efJekeâuhe Jele&veer keâer Âef° mes Megæ nQ~
(a) Deeßece (b) efJeßece 27. FveceW mes Skeâ Meyo keâer Jele&veer DeMegæ nw :
(c) efJeßeece (d) efyeßeeble (a) DeOeerve (b) YeeieerjLeer
Gòej─(c) (c) peeie=le (d) Devegie=nerle
JÙeeKÙee – heefjßece keâe efJehejerleeLe&keâ Meyo efJeßeece nesiee, peyeefkeâ efoÙes Gòej─(c)
ieÙes DevÙe efJekeâuheeW ceW ØeÙegòeâ Meyo Deeßece, efJeßece Deewj efJeßeevle keâe JÙeeKÙee – efoÙes ieÙes efJekeâuheeW ceW efJekeâuhe (c) $egefšhetCe& nw~ peeie=le keâe
Devegueesce mes keâesF& DeeMeÙe veneR nw~ Megæ ™he pee«ele nesiee~ DevÙe efJekeâuhe DeOeerve, YeeieerjLeer Deewj Devegie=nerle
21. efvecveefueefKele ceW mes Megæ JeekeäÙe nw :
Megæ Meyo nw~
28. FveceW mes Megæ Jele&veer keâe Meyo nw :
(a) Ùen ™ceeue DeÛÚer nw~ (b) Gmekeâer oner Keóer nw~
(c) keâF& neefLeÙeeb pee jner nQ~ (d) Gmekeâe cekeâeve yengle DeÛÚe nw~ (a) pÙeeslmevee (b) pÙeeslemevee
Gòej─(d) (c) pÙeeslmvee (d) pÙeewlmvee
Gòej─(c)
JÙeeKÙee – efoÙes ieÙes efJekeâuheeW ceW Megæ JeekeäÙe nw- Gmekeâe cekeâeve yengle
JÙeeKÙee – Gòeâ efoÙes ieÙes efJekeâuheeW ceW pÙeeslmvee Megæ Jele&veer keâe Meyo
DeÛÚe nw~ Mes<e efJekeâuheeW kesâ JeekeäÙe DeMegæ nQ~
nw~ Fmekesâ Deefleefjòeâ DevÙe efJekeâuhe DeMegæ nw~
22. FveceW mes Megæ JeekeäÙe nw :
29. FveceW mes Megæ Jele&veer keâe Meyo nw :
(a) [e@. ieghlee nceejs he=Yeejer nQ~
(a) efnjCÙekeâefMeheg (b) efnjCÙekeâMÙeheg
(b) Deepe ceQ Fkeâefleme Je<e& keâe nes ieÙee ntB~
(c) efnjCÙekeâMÙehe (d) efnjCÙekeâmÙehe
(c) Fme Je<e& heneÌ[eW hej pecekeâj leg<eejheele ngDee nw~ Gòej─(a)
(d) nceW keâYeer nleeslmeeefnle veneR nesvee ÛeeefnS~
Gòej─(d) JÙeeKÙee – efJekeâuhe (a) ‘efnjCÙekeâefMeheg’ Megæ Jele&veer keâe Meyo nw~
UP UDA/LDA Special (Pre) General Hindi 2010 269 YCT
CLICK HERE FOR FREE MATERIAL

30. FveceW mes DeMegæ Jele&veer keâe Meyo nw : JÙeeKÙee – `Deheveer FÛÚe mes otmejeW keâer mesJee keâjves Jeeuee' kesâ efueS
(a) cenòJe (b) Jeeuceerefkeâ GheÙegòeâ Meyo mJeÙebmesJekeâ nw~ peyeefkeâ `efyevee Jesleve kesâ keâece keâjves Jeeues
(c) hewle=keâ (d) mevÙeemeer keâes' DeJewleefvekeâ keâne peelee nw~
Gòej─(d)
38. ‘ieg® kesâ meceerhe jn keâj efMe#ee «enCe keâjves Jeeuee’ kesâ efueS
JÙeeKÙee – mevÙeemeer DeMegæ Jele&veer keâe Meyo nw~ Fmekeâe Megæ Jele&veer Skeâ Meyo nw
mebvÙeemeer nesiee~ peyeefkeâ cenòJe, hewle=keâ,Jeeuceerefkeâ keâer Jele&veer Megæ nw~ (a) ieg®kegâueJeemeer (b) Úe$eeJeemeer
31. ‘neLeer keâer heer" hej jKes peeves Jeeues Deemeve’ kesâ efueS Megæ (c) DevlesJeemeer (d) DeeßeceJeemeer
Meyo nw Gòej─(c)
(a) peerve (b) newoe
JÙeeKÙee – `ieg® kesâ meceerhe jn keâj efMe#ee «enCe keâjves Jeeuee' kesâ efueS
(c) keâe"er (d) yeKlej
Gòej─(b) Skeâ Meyo nw `DevlesJeemeer'~
39. ‘Dehevee GösMÙe hetCe& nesves hej mebleg°’ Ssmes JÙeefòeâ kesâ efueS Skeâ
JÙeeKÙee – neLeer keâer heer" hej jKes peeves Jeeues Deemeve keâes ‘newoe’ kesâ
Meyo nw
veece mes peevee peelee nw~ peyeefkeâ keâe"er keâe GheÙeesie TBš kesâ efueS neslee
(a) efÛejØemeVe (b) ke=âle%e
nw~ ieeefÌ[ÙeeW hej megj#ee kesâ efueS yeKlej keâe ØeÙeesie efkeâÙee peelee nw~
(c) DeeYeejer (d) ke=âleeLe&
32. ‘efpemes ØeceeCe Éeje efmeæ ve efkeâÙee pee mekesâ’ Gmekesâ efueS mener Gòej─(d)
Meyo nw
(a) DeØeceeefCele (b) DevegØecesÙe
JÙeeKÙee – Dehevee GösMÙe hetCe& nesves hej mebleg° JÙeefòeâ keâes ke=âleeLe& keâne
(c) DeØecesÙe (d) DeØeeceeefCekeâ peelee nw~ peyeefkeâ pees ncesMee ØemeVe jnves Jeeuee nw, Gmekesâ efueS efÛej
Gòej─(c) ØemeVe Meyo GheÙegòeâ nesiee Deewj pees efkeâÙes ngS Ghekeâej keâes ceevelee nw Gmes
ke=âle%e keânles nw~
JÙeeKÙee – efpemes ØeceeCe Éeje efmeæ ve efkeâÙee pee mekesâ Gmekesâ efueS
40. ‘efveÙeceefJe®æ, Demeeceeefpekeâ keâeÙe& keâjves JeeueeW keâer metÛeer’ kesâ
GheÙegòeâ Meyo DeØecesÙe neslee nw~
efueS Skeâ Meyo nw
33. ‘GheefveJesMe mes mecyevOe nes efpemekeâe’ Gmekesâ efueS Skeâ Meyo nw
(a) DehejeOemetÛeer (b) keâeueermetÛeer
(a) GheefveJesefMekeâ (b) DeewheefveJesefMekeâ
(c) DeJewOemetÛeer (d) MJeslemetÛeer
(c) DeewhevÙeeefmekeâ (d) GheefveJesMeJeeo
Gòej─(b) Gòej─(b)
JÙeeKÙee – `GheefveJesMe mes mecyevOe nes efpemekeâe' Gmekesâ efueS Skeâ Meyo JÙeeKÙee – ‘efveÙeceefJe®æ, Demeeceeefpekeâ keâeÙe& keâjves JeeueeW keâer metÛeer’
‘DeewheefveJesefMekeâ’ nw~ kesâ efueÙes Skeâ Meyo ‘keâeueermetÛeer’ nw~
34. FveceW mes Skeâ kesâ efueS ØeÙeespÙe Meyo nw – ‘DemetÙe&cheMÙee’ : 41. ‘osJej’ keâe lelmece ™he nw
(a) Jen mLeeve peneb metÙe& efoKeeÙeer ve os~ (a) efÉJej (b) efÉleerÙeJej (c) otYej (d) ogye&n
(b) Jen mLeeve peneb metÙe& keâe ØeKej ØekeâeMe yeÌ[e keâ°keâejer neslee nw~ Gòej─(a)
(c) Jes ØeeCeer pees metÙe& keâe oMe&ve ve keâj heeÙeW~ JÙeeKÙee – osJej keâe lelmece Meyo ‘efÉJej’ nw~ Mes<e efJekeâuheeW kesâ Meyo
(d) jefveJeeme ceW keâÌ[s heox ceW jnves Jeeueer Œeer~ Demebiele nQ~
Gòej─(d) 42. FveceW mes keâewve mee Meyo lelmece veneR nw?
JÙeeKÙee – DemetÙe&cheMÙee keâe DeeMeÙe nw–‘jefveJeeme ceW keâÌ[s heox ceW jnves (a) Deefive (b) Ieesškeâ (c) Ieš (d) ceesleer
Jeeueer Œeer~’ Gòej─(d)
35. ‘GòejeefOekeâej ceW Øeehle mecheefòe’ kesâ efueS Skeâ Meyo nw JÙeeKÙee – efoÙes ieÙes ÛeejeW efJekeâuheeW ceW ceesleer Meyo lelmece veneR nw~
(a) efjkeäLe (b) Oejesnj Fmekeâe lelmece ¤he ceewefòeâkeâ nesiee~ DevÙe Meyo Deefive, Ieesškeâ Deewj Ieš
(c) JemeerÙele (d) meboeÙe lelmece Meyo nw~ Fvekeâe leodYeJe Meyo ›eâceMe: nesiee – Deeie, IeesÌ[e Deewj
Gòej─(a) IeÌ[e~
JÙeeKÙee – `GòejeefOekeâej ceW Øeehle mecheefòe' kesâ efueS Skeâ Meyo nw 43. ‘heueBie’ Meyo keâe lelmece ™he nw
`efjkeäLe'~ (a) heueBiee (b) hueJebie
36. ‘efieje ngDee’ kesâ efueS Skeâ Meyo nw (c) heÙeËkeâ (d) hešue
(a) heeflele (b) uegbef"le Gòej─(c)
(c) OejeMeeÙeer (d) heelekeâer JÙeeKÙee – heueBie Meyo keâe lelmece ¤he nw – ‘heÙeËkeâ’~ Mes<e efJekeâuheeW kesâ
Gòej─(a) Meyo Demebiele nQ~
JÙeeKÙee – `efieje ngDee' kesâ efueS Skeâ Meyo nw heeflele~ 44. FveceW mes Skeâ Meyo leodYeJe nw :
37. efvecveefueefKele JeekeäÙe KeC[eW ceW mes Skeâ kesâ efueS ØeÙegòeâ nesves (a) efove (b) DebOekeâej
Jeeuee Meyo nw ‘mJeÙebmesJekeâ’ : (c) mkeâvOe (d) keâheeme
(a) meyekeâer mesJee keâjves Jeeuee~ Gòej─(d)
(b) mJeÙeb keâer mesJee keâjves Jeeuee~
(c) Deheveer FÛÚe mes otmejeW keâer mesJee keâjves Jeeuee~
JÙeeKÙee – efoÙes ieÙes efJekeâuheeW ceW keâheeme Meyo leodYeJe nw~ Fmekeâe lelmece
(d) efyevee Jesleve kesâ keâece keâjves Jeeuee mesJekeâ~
™he keâhee&me nesiee~ peyeefkeâ mkeâvOe keâe leodYeJe keâvOee, DevOekeâej keâe
Gòej─(c) DeBOesje leLee efove keâe efoJeme leodYeJe neslee nw~
UP UDA/LDA Special (Pre) General Hindi 2010 270 YCT
CLICK HERE FOR FREE MATERIAL

45. ‘DeKejesš’ Meyo keâe lelmece ™he nw 54. ‘efpeme efJekeâejer Meyo mes meb%ee keâer JÙeeefhle ceÙee&efole nesleer nw’
(a) De#eJeeš (b) De#eesš Gmes keânles nQ
(c) De#ejesš (d) De<ejesš (a) efJeMes<Ùe (b) efJeMes<eCe
Gòej─(b) (c) efJeMes<eCe SJeb efJeMes<Ùe (d) efJeefMe°
JÙeeKÙee – DeKejesš Meyo keâe lelmece ¤he nw – De#eesš~ Mes<e efJekeâuheeW Gòej─(b)
kesâ Meyo Demebiele nQ~ JÙeeKÙee – efpeme efJekeâejer Meyo mes meb%ee keâer JÙeeefhle ceÙee&efole nesleer nw
46. ‘DekeâeÙe&’ Meyo keâe leodYeJe ™he nw Gmes ‘efJeMes<eCe’ keânles nQ~
(a) Dekeâece (b) Dekeâepe 55. efvecveefueefKele MeyoeW ceW mes efJeMes<eCe Meyo nw
(c) DekeâejLe (d) FveceW mes keâesF& veneR (a) heeBÛeJeeB (b) ØehebÛe (c) mejhebÛe (d) hengBÛe
Gòej─(b) Gòej─(a)
JÙeeKÙee – DekeâeÙe& Meyo keâe leodYeJe ¤he nw – Dekeâepe~ Mes<e efJekeâuheeW JÙeeKÙee – efoÙes ieÙes efJekeâuheeW ceW efJekeâuhe (a) `heeBÛeJeeB' Meyo
kesâ Meyo Demebiele nQ~ mebKÙeeJeeÛekeâ efJeMes<eCe nw~ efpeme efJeMes<eCe mes meb%ee Ùee meJe&veece keâer
47. ‘škeâmeeue’ keâe lelmece ™he nw mebKÙee keâe yeesOe nes Gmes mebKÙeelcekeâ efJeMes<eCe keânles nQ~
(a) šbkeâMeeuee (b) šbkeâMeeue 56. efvecveefueefKele ceW mes ef›eâÙee-efJeMes<eCe nw :
(c) škeâMeeue (d) škeâMeeuee (a) DeBOesje (b) Oeerjs-Oeerjs
Gòej─(a) (c) Ûeeue-Ûeueve (d) megvoj
JÙeeKÙee – škeâmeeue keâe lelmece ¤he nw – šbkeâMeeuee~ Mes<e efJekeâuheeW kesâ Gòej─(b)
Meyo Demebiele nQ~ JÙeeKÙee – ‘Oeerjs-Oeerjs’ ef›eâÙee efJeMes<eCe Meyo nw~ pees Meyo ef›eâÙee keâer
48. FveceW mes lelmece Meyo nw : efJeMes<elee yeleeles nQ Gmes ef›eâÙee efJeMes<eCe Yeer keânles nQ~
(a) YeQme (b) yeejn (c) meÛe (d) heo
57. ‘oesveeW’ Meyo efkeâme Øekeâej keâe mebKÙeeJeeÛekeâ efJeMes<eCe nw?
Gòej─(d)
(a) mecegoeÙeyeesOekeâ (b) hegve®efkeäleyeesOekeâ
JÙeeKÙee – efoÙes ieÙes efJekeâuheeW ceW lelmece Meyo ‘heo’ nw~ Mes<e (c) DeeJe=efòeyeesOekeâ (d) ›eâceyeesOekeâ
efJekeâuheeW kesâ Meyo Demebiele nQ~ Gòej─(a)
49. ‘ieesyej’ keâe lelmece ™he nw JÙeeKÙee – `oesveeW' Meyo mecegoeÙe yeesOekeâ mebKÙeeJeeÛekeâ efJeMes<eCe nw~
(a) iegye&j (b) ieesceÙe (c) iendJej (d) iegyyej keäÙeeWefkeâ Fmemes Skeâ mecegoeÙe keâe mhe°erkeâjCe neslee nw~
Gòej─(b) mebKÙeeJeeÛekeâ efJeMes<eCe kesâ efvecve Yeso nw-
JÙeeKÙee – ieesyej keâe lelmece ¤he nw – ieesceÙe/ieesefJe„/ieesefJeš~ Mes<e (De) efveefMÛele mebKÙeeJeeÛekeâ (ieCeveeJeeÛekeâ, ›eâceJeeÛekeâ,
efJekeâuheeW kesâ Meyo Demebiele nQ~ DeeJe=efòeJeeÛekeâ, mecegoeÙeJeeÛekeâ Deewj ØelÙeskeâyeesOekeâ)
50. ‘ceeB’ keâe lelmece ™he nw (ye) DeefveefMÛele mebKÙeeJeeÛekeâ
(a) ceele= (b) ceele=keâe (c) ceeleeßeer (d) ceelegßeer (me) heefjceeCe yeesOekeâ
Gòej─(a) 58. ‘Øeefleefove’ efkeâme Øekeâej keâe ef›eâÙee-efJeMes<eCe nw?
JÙeeKÙee – ‘ceeB’ keâe lelmece ™he ceele= neslee nw~ Mes<e efJekeâuheeW kesâ Meyo (a) jerefleJeeÛekeâ (b) heefjceeCeJeeÛekeâ
Demebiele nQ~ (c) mLeeveJeeÛekeâ (d) keâeueJeeÛekeâ
Gòej─(d)
51. FveceW mes keâewve mee Meyo efJeMes<eCe veneR nw?
(a) YeÙeYeerle (b) efveYeeakeâ JÙeeKÙee – efoÙes ieÙes efJekeâuheeW ceW ‘Øeefleefove’ keâeueJeeÛekeâ ef›eâÙee
(c) Yeer® (d) YeJe efJeMes<eCe nw~
Gòej─(d) 59. FveceW mes efkeâme JeekeäÙe ceW ieuele efJeMes<eCe ØeÙegòeâ ngDee nw?
JÙeeKÙee – GheÙeg&òeâ efJekeâuheeW ceW YeJe Meyo efJeMes<eCe veneR nw~ peyeefkeâ (a) keâefJelee heefjßeceer ÙegJeleer nw~
efveYeeakeâ, Yeer® Deewj YeÙeYeerle Meyo efJeMes<eCe nw~ (b) ØeyegæpeveeW mes nceejer Dehes#ee nw~
52. efvecveefueefKele ceW mes keâewve mee Meyo efJeMes<eCe nw? (c) Ùener mejkeâejer ceefnueeDeeW keâe Demheleeue nw~
(a) megvojlee (b) keâefJe (d) Jen DeÛÚe Deeoceer Lee, uesefkeâve keâece ve DeeÙee~
(c) efJeÉeve (d) YeueeF& Gòej─(c)
Gòej─(c) JÙeeKÙee – efoÙes ieÙes efJekeâuhe ceW efJekeâuhe (c) $egefšhetCe& JeekeäÙe nw~ `Ùener
JÙeeKÙee – efveefo&° efJekeâuheeW ceW ‘efJeÉeve’ Meyo efJeMes<eCe nw~ peyeefkeâ mejkeâejer ceefnueeDeeW keâe Demheleeue nw~' FmeceW Meyo ›eâce oes<e nw~ mener
keâefJe, YeueeF& Deewj megvojlee meb%ee Meyo nwb~ JeekeäÙe nesiee – `Ùen ceefnueeDeeW keâe mejkeâejer Demheleeue nw~'
53. ‘Jen ke=âMekeâeÙe JÙeefkeäle oew[ Ì ves ueiee’ – Fme JeekeäÙe ceW efJeMes<Ùe nw 60. ‘iegCeJeeÛekeâ efJeMes<eCe’ kesâ efkeâleves Yeso nQ?
(a) JÙeefòeâ (b) ke=âMekeâeÙe (a) Ûeej (b) heeBÛe (c) Ún (d) meele
(c) Jen (d) oewÌ[ves ueiee Gòej─(c)
Gòej─(a) JÙeeKÙee – efpeme Meyo mes meb%ee keâe iegCe, oMee, mJeYeeJe Deeefo ueef#ele
JÙeeKÙee – `Jen ke=âMekeâeÙe JÙeefòeâ oewÌ[ves ueiee' Fme JeekeäÙe ceW efJeMes<Ùe nes Gmes iegCeJeeÛekeâ efJeMes<eCe keânles nQ~ Fmekesâ Yeso efvecve nw – keâeue,
Meyo ‘JÙeefòeâ’ nw~ peyeefkeâ ke=âMekeâeÙe efJeMes<eCe Meyo nw~ mLeeve, Deekeâej, jbie, oMee, iegCe~
UP UDA/LDA Special (Pre) General Hindi 2010 271 YCT
CLICK HERE FOR FREE MATERIAL

UPPSC UDA/LDA (Pre) Exam-2006


GENERAL STUDIES
Solved Paper
1. Which of the following pairs related to 6. Ptolemy Philadephus, with whom Emperor
Harappan Culture sites and their location is Ashoka had diplomatic relations was the ruler
not correctly matched ? of ?
(a) Alamgirpur - Uttar Pradesh (a) Cyrene (b) Egypt
(b) Banawali - Haryana (c) Macedonia (d) Syria
(c) Daimabad - Maharashtra Ans. (b) : Ptolemy Philadephus was the king of Egypt
(d) Rakhigarhi - Rajasthan (285-246 B.C.). He was the second king of the
Ans. (d) : Rakhigarhi is found in the state of Haryana. It Ptolemaic dynasty, who extended his power by skilful
is the largest Harappan site discovered by Shri diplomacy, developed agriculture and commerce. Ptolemy
Amarendra Nath to ASI. sent an ambassadar named Dionysius to the Mauryan
2. During the Gautam Buddha's time period, the Court at Patliputra in India to Emperor Ashoka.
famous physician - Jeevak was related to whose 7. From the inscription of which dynasty supports
court ? this traditional belief that Lumbini was Shakya
(a) Bimbisar (b) Chand Pradyot mumi, Budha's birth place ?
(c) Prasenjit (d) Udyan (a) Maurya (b) Shunga
Ans. (a) : The famous physician-Jeevak was related to (c) Satvahan (d) Kushan
Bimbisara. Jeevaka became the physician to king Ans. (a) : The Maurya dynasty inscription of
Bimbisara who was a follower of Gautama Buddha. Rummindei also known as Lumbini Pillar Edict
3. Who was Nayanar ? supports the traditional belief that Lumbini was Shakya
(a) A Shaivite (b) A Shakta Muni, Buddha's birth place.
(c) A Vishanavite (d) A Sun worshipper 8. Among the following Kings, who had a strong
inclination toward Jainism ?
Ans. (a) : Nayanar were Shaivite. The Nayanars were a
group of 63 saints devoted to Lord Shiva who lived (a) Dasrath (b) Brihdrath
during the 6th to 8th centuries CE. (c) Kharvela (d) Huvishka
4. How many years difference is there between Ans. (c) : Kharavela was a strong king who ruled over
Vikram and Saka era ? the Kalinga kingdom of present day Odisha.
(a) 57 yrs (b) 78 yrs Jainism spread to Kalinga in the fourth century B.C. He
(c) 135 yrs (d) 320 yrs enjoyed the patronage of the Kalinga king Kharavela.
Ans. (c) : Saka Era (Samvat) started from 78 A.D. 9. Who established/founded the Jaunpur City ?
Whilst Vikram Era (Samvat) started much before in (a) Muhammad bin Tuglaq
57 B.C. (b) Firoz Shah Tughlaq
So, the difference between the Saka Era and the Vikram (c) Ibrahim Shah Sharki
Era is 135 years approximately. (d) Sikandar Lodi
5. Which of the following rulers paid attention Ans. (b) : Firoz Shah Tughlaq founded the city of
towards water resources system in the Girnar Jaunpur in 1359. He founded this city in the memory of
Region during a few centuries before the birth Muhammad bin Tughlaq (nickname - Jauna).
of Christ time period (B.C.) ? 10. Who established the Public works department
1) Mahapadma Nand first ?
2) Chandragupta Maurya (a) Allauddin Khilji (b) Balbam
3) Ashoka (c) Firoz Shah Tughlaq (d) Iltutmish
4) Rudradaman Ans. (c) : Firoz Shah Tughlaq established the Public
Choose the correct answer from the codes Works Department. He built many schools, hospitals,
given below :- river canals, reservoirs, rest houses among other things.
(a) 1, 2 (b) 2, 3 He constructed canals from Yamuna to the city of Hisar,
(c) 3, 4 (d) 2, 3 & 4 Sutlej to the Ghaggar etc.
Ans. (b) : Rulers like Chandragupta Maurya, Ashoka 11. Famous Vijaynagar ruler Krishnadev Raya
paid attention towards water resources system in the ruling period was the golden period of which
Girnar Region during a few centuries before the birth of literature ?
Christ time period (B.C.). Rudradaman also paid (a) Konkani (b) Malyalam
attention to resource in Girnar Region. (c) Tamil (d) Telugu
UP UDA/LDA (Pre) Exam 2006 272 YCT
CLICK HERE FOR FREE MATERIAL

Ans. (d) : Famous Vijaynagar ruler Krishna Deva Raya 17. As a symbol of expressing protest against
ruling period was the golden period of Telugu Literature. which of the following did Ravindra Nath
Amuktamalyada, which is written in Telugu and describes Tagore renounced his Kinghthood title ?
the wedding of Vishnu and Goda Devi aka Andal. (a) Partition of Bengal
12. During the rule of Shershah Suri what was the (b) Press Act of 910
exchange rate between Dam currency of (c) Jallianwala Bagh Massacre
Copper and Silver rupee ? (d) Salt Act
(a) 16:1 (b) 32:1 Ans. (c) : Rabindra Nath Tagore renounced his
(c) 48:1 (d) 64:1 kinghthood title as a symbol of expressing protest
against the Jallianwala Bagh massacre in the year 1919.
Ans. (d) : During the rule of Sher Shah Suri, the
exchange rate between the Dam currency of Copper and It was an incident in which British troops fired on a
Silver rupee was 64 : 1. large crowd of unarmed Indians in an open space
known as the Jallianwala Bagh in Amritsar in Punjab on
13. Arrange the following events of Akbar's ruling 13th April, 1919.
period in a chronological order :-
18. The transfer of Capital of India from Calcutta
1) End of Jazia to New Delhi was effected during the period of
2) Construction of Ibadatkhana which British viceroy ?
3) Signature on Mahjar (a) Lord Minto (b) Lord Hardinge
4) Din-e-Elahi establishment (c) Lord Chelmsford (d) Lord Reading
Choose the correct answer from the codes Ans. (b) : During the Delhi durbar on 12th December
given :- 1911, George V. then Emperor of India, along with the
(a) 1, 2, 3, 4 (b) 2, 3, 4, 1 Queen and during the period of Lord Hardinge as
(c) 1, 3, 2, 4 (d) 3, 4, 1, 2 Viceroy, made of the announcement to shift capital
Ans. (a) : The correct chronology of the events during from Calcutta to Delhi.
Akbar's period is - 19. Who was the revolutionary connected with the
End of Jazia - 1564 Kakori incident who was not retended to
Construction of Ibadat Khana - 1575 death?
Signature of Mahjar - 1579 (a) Sachindranath Bakshi
Din-e-Elahi Establishment - 1582 (b) Mukundi Lal
14. Which of the following ruler granted Diwani (c) Chandra Shekhar Azad
rights to the East India Company. ? (d) Vallabh Bhai Patel
(a) Farukhsiyar (b) Shah Alam-I Ans. (c) : Kakori incident was an armed robbery which
(c) Shah Alam-II (d) Shujauddaulah took place on August 9, 1925 on a train in central Uttar
Pradesh.
Ans. (c) : The Mughal emperor Shah Alam-II granted
The robbery was planned by Ram Prasad Bismil and
the Diwani rights to the East India Company after the
Ashfaqullah Khan. It was executed by Bismil Khan,
defeat in the Battle of Buxar (1764). Chandra Shekhar Azad, Rajendra Lahiri, Sachindra
15. By whose concerted efforts the first Women Bakshi etc.
University at Bombay was established ? Chandra Shekhar Azad shot himself after being
(a) Dayaram Gidumal grievously enquired in a shootout with the police on 27
(b) D.K Karvey February 1931 at Chandrashekhar Park (then known as
(c) Ramabai Alfred Park).
(d) Mahadev Gound Ranade 20. Who was the only Indian Governor General ?
Ans. (b) : With the concerted efforts of D.K. Karve the (a) C. Rajgopalachari
first women university in Bombay was established in the (b) Pattabhi Sitaramaiyyah
year of 1916 for a noble cause of women's education. (c) Rajendra Prasad
16. What was the name of that Indian who (d) Sardar Vallabh bhai Patel
participated in all the three Round Table Ans. (a) : C. Rajagopalachari was the only Indian
Conferences ? Governor General. He was a founder of the Swatantra
(a) B.R Ambedkar (Independent) Party in 1959.
(b) Mahatma Gandhi 21. Which of the following Plan became a
(c) Muhammad Ali Jinnah foundation for Indian Freedom/Indepedance ?
(d) Tej Bahadur Sapru (a) Cripps Mission Plan (b) Wavell Plan
Ans. (a) : B.R. Ambedkar was an Indian who (c) Mountbatten Plan (d) None of these
participated in all the three Round Table Conferences of Ans. (c) : Mountbatten Plan of 3 June 1947 became a
(1930-32) which was a series of peace conferences foundation for Indian Freedom/Independence.
organized by the British Government and Indian The June 3 plan included the principles of partition,
political personalities to discuss constitutional reforms autonomy, sovereignty to both nations, right to make
in India. their own constitution.
UP UDA/LDA (Pre) Exam 2006 273 YCT
CLICK HERE FOR FREE MATERIAL

22. During the Salt Satyagraha after the arrest of 29. On which Indian river, worlds highest bridge is
'Gandhiji', who took over his seat as a leader of being constructed ?
the revolution/movement ? (a) Chenab (b) Sutlej
(a) Abbas Tayabji (b) Abul Kalam Azad (c) Jhelum (d) Beas
(c) Jawaharlal Nehru (d) Vallabh Bhai Patel Ans. (a) : The world's highest bridge is being
Ans. (a) : During the Salt Satyagraha after the arrest of constructed on the Indian river of Chenab also known as
'Gandhiji', Abbas Tayabji, took over his seat as a leader Chenab Rail Bridge.
of the revolutionary movement Abbas Tayabji was an 30. Laterite soil is found in ?
Indian freedom fighter from Gujarat and associate of (a) Uttar Pradesh (b) Himanchal Pradesh
Mahatma Gandhi. (c) Punjab (d) Maharashtra
23. Who change the name of the organisation from Ans. (d) : Laterite soil is rich in iron-oxide and derived
'Hindustan Republic Association' to from a wide variety of rocks weathering under strongly
'Hindustan Socialist Republic Association' ? oxidizing and leaching conditions. These soils are
(a) Ras Behari Bose mainly found in Maharashta, Karnataka, Kerala, T.N.,
(b) Batukeshwar Dutt M.P., A.P. and hilly areas of Odisha and Assam.
(c) Sardar Bhagat Singh 31. Which of the following state is the largest
(d) Chandra Sekhar Azad producer of coffee in India ?
(a) Kerala (b) Karnataka
Ans. (d) : Chandrashekhar Azad change the name of the (c) Tamil Nadu (d) West Bengal
organization from 'Hindustan Republic Association to
'Hindustan Socialist Republic Association'. Ans. (b) : The largest producer of coffee in India is
Karnataka. 71% India's coffee is produced in Karnataka.
HRA was founded by Sachindra Nath Sanyal in Kanpur The largest coffee producing region of Karnataka is
in the year 1924. Later it was reconstructed and named Kodagu (Coorg) district.
as HSRA in 1928 at Ferozshah Kotla in Delhi on the
insistence of Bhagat Singh. 32. Consider the following statements :-
Statement (A) : An Encouraging source of
24. Which revolutionary won sentenced to death energy supply to meet the future energy
by hanging at Gorakhpur jail ? demand of India is Atomic Energy.
(a) Ramprasad Bismil (b) Rajendra Lahiri Reason (R) : In India Atomic mineral is
(c) Roshan Singh (d) Ashfaqullah Khan available everywhere.
Ans. (a) : On 19th December 1927, Ram Prasad Bismil, Choose the correct answer from the codes
a revolutionary, was hanged at Gorakhpur Jail for his given below :-
role in the Kakori Conspiracy. (a) Both (A) and (R) are true and (R) is the
25. Which of the following states has the longest correct explanation of (A).
coast line ? (b) Both (A) & (R) are true but (R) is not the
(a) Gujarat (b) Maharashtra correct explanation of (A).
(c) (A) is true but (R) is false.
(c) Kerala (d) Tamilnadu
(d) (A) is false but (R) is true.
Ans. (a) : Gujarat has the longest Coastline in India
Ans. (c) : Atomic minerals are the most important
along its Kathiawar region.
among non fossil energy resources. Uranium and
The length of the Gujarat coast line is about 1600 km Thorium are the main atomic minerals. Other atomic
surrounded by the Arabian Sea. minerals are feryllium, lithium and zirconium. Uranium
26. Chilika lake is situated in ? is found in the Singhbhum and Hazaribagh districts of
(a) West Bengal (b) Andhra Pradesh Jharkhand, and Gaya Districts of Bihar. Thorium is
(c) Odisha (d) Tamil Nadu produced in Kerala, Jharkhand, Bihar, Tamilnadu and
Rajasthan Hence, statement- In India Atomic minerals
Ans. (c) : Chilika lake is situated in Odisha. It is a is available every where was incorrect. Therfore option
brackish water lagoon. (c) is corrct answer.
27. Damodar is the tributary of which river ? 33. Out of the following Rock system, which is a
(a) Ganga (b) Hooghly major source of coal deposits in India ?
(c) Padma (d) Subernarekha (a) Dharwad System (b) Gondwana System
Ans. (a) : Damodar is the triburary of Ganga river. It (c) Cuddapah System (d) Vindhya System
flows across the Indian States of Jharkhand and West Ans. (b) : Gondwana rock system is a major source of coal
Bengal. deposits in India. It contains 95% of coal deposits in India.
28. Durand line determines the Indian border with 34. Which of the following technique is used to
which of the following country ? determine the age of earth ?
(a) Afghanistan (b) Burma (a) Carbon Dating (b) Germanium Dating
(c) Nepal (d) Tibet (c) Uranium Dating (d) All the above
Ans. (a) : Durand Line separates the boundary of Ans. (c) : The age of earth is calculated by using
Pakistan and Afghanistan with a length of 2640 uranium dating technique whereas carbon dating is used
kilometer. for determining the age of fossils.
UP UDA/LDA (Pre) Exam 2006 274 YCT
CLICK HERE FOR FREE MATERIAL

While Germanium is a silvery white metal and is an 41. In which country the life expectancy at the time
important component in semiconductors and fibre of birth is highest ?
optics. (a) Japan (b) Denmark
35. Titan is the largest moon/satellite of ? (c) U.S.A (d) Switzerland
(a) Mars (b) Venus Ans. (a) : The life expectancy at the time of Birth is
(c) Jupiter (d) Saturn maximum for Japan. In Japan, the life expectancy at the
Ans. (d) : Titan is the Saturn largest moon. Titan is an time of birth in 2022 is-
icy world whose nature is completely obscured by a for men - 87.7 years.
golden hazy atmosphere. It is the seconds-largest moon for women - 81.5 years.
is our solar system. While Hong Kong has the highest like expectancy in the
36. Every Solar Eclipse occurs on - world.
(a) Only on full moon day 42. The correct chronological descending order of
(b) Only on new moon day population densities (2001) of India's major
(c) Both (a) & (b) states is -
(a) Uttar Pradesh, West Bengal, Bihar, Kerala
(d) Neither on (a) & (b)
(b) West Bengal, Kerala, Bihar, Uttar Pradesh
Ans. (b) : Solar Eclipse occurse only when the moon is (c) Uttar Pradesh, Kerala, West Bengal, Bihar
in between the earth and the sun, so it can happen only
(d) West Bengal, Bihar, Uttar Pradesh, Kerala
on a new moon day.
Ans. (d) : According to the 2001 census. The
37. It was whose contention (opinion) that the descending order of population densities of India's
earth revolves around the Sun? major states are -
(a) Aryabhata (b) Brahmaguptra Bihar > W.B. > Kerala > U.P.
(c) Varahmihir (d) None of them (1106) (1028) (860) (829)
Ans. (d) : In 1543, Nicolaus Copernicus given his Population Density of India - 324 persons/km2.
opinion that the earth revolves around the Sun. 43. As per the Census, 2001, which is the most
38. Which of the following is not matched urbanised state of India ?
correctly? (a) Gujarat (b) West Bengal
(a) Kimberley - Diamond (c) Maharashtra (d) Kerala
(b) Havana - Meat Packaging Ans. (c) : As per 2001 census most urbanised state -
(c) Milan - Silk Maharashtra.
(d) Sheffield - Cutlery As per 2011 census, 4 most urbanised states are
Ans. (b) : The correct match is as follows :- (population wise).
Kimberley - Diamond 1. Maharashtra 2. U.P. 3. T.N. 4. W.B.
(South Africa) 44. As per the Census 2001, the correct ascending
Havana - Cigar order of states of lowest population density in
(Cuba's Capital) India is -
Milan - Silk (a) Arunachal Pradesh, Mizoram, Nagaland and
(Italy) Sikkim
Sheffield - Cutlery (b) Arunachal Pradesh, Nagaland, Mizoram and
Kansas - Meat Packaging Sikkim.
(U.S.) (c) Arunachal Pradesh, Mizoram, Sikkim and
Nagaland.
39. Bauxite is an Ore of ?
(d) Mizoram, Arunachal Pradesh, Sikkim and
(a) Aluminium (b) Boron
Nagaland.
(c) Lead (d) Silver
Ans. (c) : As per the census 2001, the correct ascending
Ans. (a) : Bauxite is an Ore of Aluminium. order of states of lowest population density in India is -
40. Which of the following is not correctly Arunachal Pradesh, Mizoram, Sikkim and Nagaland. As
matched? per 2011 census, the top 4 states with minimum
(a) Kyrgyz - Central Asia population density are as follows -
(b) Masai - West Africa Aruncahal Pradesh, Mizoram, Sikkim, Nagaland
(c) Red Indians - North America 45. The Concept of Fundamental Rights vested in
(d) Eskimo - Greenland the Indian Constitution has been acquired from
Ans. (b) : Masai tribes are associated with eastern which country's constitution ?
Africa and NOT with Western Africa. Rest are correctly (a) France (b) Britain
matched. Therefore, the correct match is as follows - (c) U.S.A (d) Russia
Kyrgyz - Central Asia Ans. (c) : The concept of 'Fundamental Rights' have
Masai - Eastern Africa been adopted from the United States of America. The
Red Indians - North America part III of the Indian Constitution comprises
Eskimo - Greenland Fundamental Rights [Articles (12-35)].
UP UDA/LDA (Pre) Exam 2006 275 YCT
CLICK HERE FOR FREE MATERIAL

46. The Indian constitution was approved by the Ans. (d) : A Money Bill can only be introduced in the
Constituent Assembly on ? Lok Sabha by a minister. Money Bill is defined in
(a) 15th Aug, 1947 (b) 30 June, 1948 Article 110 of the Indian Constitution.
(c) 26th Nov, 1949 (d) 26 Jan, 1950 Money Bills are concerned with financial matters like
Ans. (c) : The Indian Constitution was approved by the taxation, public expenditure etc.
Constituent Assembly on 26th November 1949 and the 52. The desired minimum age for the election of
Constitution came into force on 26th January 1950. Rajya sabha member is ?
47. Which part of the Indian Constitution is (a) 25 yrs (b) 30 yrs
termed as the Soul of Constitution? (c) 32 yrs (d) 35 yrs
(a) Fundamental Rights Ans. (b) : The desired minimum age for the election of
(b) Directive Principles of State Policy Rajya Sabha member is 30 years.
(c) Preamble Rajya Sabha is the second chamber of the parliament
(d) Right to Constitutional Remedy and represents the states and union territories of the
nation.
Ans. (c) : The 'Preamble' is called the soul of the
Constitution because it is a reflection of the core Maximum strength - 250 members
constitutional values that embody the Constitution. 238 members - states
While according to B.R. Ambedkar, Article 32 i.e. 12 members - nominated by the President.
Right to Constitutional Remedies is the "Heart and Fourth schedule of Indian Constitution deals with
Soul" of our Constitution. allocation of seats in Rajya Sabha.
48. The President can nomonate - 53. Which state legislative assembly has maximum
(a) 10 members to Rajya Sabha members ?
(a) Andhra Pradesh (b) West Bengal
(b) 2 members to Rajya Sabha
(c) Maharashtra (d) Uttar Pradesh
(c) 15 members to Rajya Sabha
(d) 12 members to Rajya Sabha Ans. (d) : Uttar Pradesh is the largest legislature in
India.
Ans. (d) : The President can nominate 12 members to The Uttar Pradesh Assembly consists of 403 elected
the Rajya Sabha from people who have special members.
knowledge or practical experiences in art, literature,
Before, the (104th amendment) Act, 2019 which came
science and social service. into force on 25th January 2020, it has 403+1 (for Anglo
49. The maximum numbers of seats which has Indian) seats. But after the discontinuation of Anglo
been prescribed to Lok Sabha is - Indian Seats. U.P. has 403 seats for state legislative
(a) 540 (b) 545 assembly.
(c) 550 (d) 555 The Legislative Assembly Seats for -
Ans. (*) : As per the Constitution, the maximum seats Maharashtra - 288
prescribed to Lok Sabha is 552. Andhra Pradesh - 175
530 - for States West Bengal - 294
20 - for Union Territories 54. Those subjects on which both Centre (Union)
2 - For Anglo Indians and the states can frame laws have been
While the 104th Constitutional Amendment Act have mentioned in the ?
removed the reserved seats for the Anglo-Indian (a) Union list (b) State list
community in the Lok Sabha and State assemblies. (c) Concurrent list (d) Residual list
50. In Indian Parliament the appointment of Ans. (c) : The 7th schedule of the Indian Constitution
President of Public Accounts Committee is deals with the division of powers between the Union
done by government and state governments.
(a) Indian President It is divided into 3 lists -
(b) Prime Minister of India 1. Union List- List-I - Only centre can make laws.
(c) Lok Sabha Speaker 2. State List- List-II - Only state can make laws.
(d) Chairman of Rajya Sabha 3. Concurrent List- List-III-Both the centre and state
Ans. (c) : In Indian Parliament the appointment of can make laws. But in case of dispute, the centre's laws
President of Public Accounts Committee is done by Lok prevails.
Sabha Speaker. The terms of office of the members is 1 55. Economic Planning is included in which of the
year. The current PAC is headed by Adhir Ranjan Chow following list of Indian Constitution ?
dhury since July, 2019. (a) Union list (b) State list
51. Which of the following statements is/are true (c) Concurrent list (d) Special list
regarding introduction of money bill ? Ans. (c) : Economic Planning is included in the
(a) Money is introduced in Rajya Sabha. concurrent list. i.e., both the centre and the state can
(b) It can be introduced in any of the house of frame laws.
Parliament. 56. 52nd Constitutional Amendment is related to ?
(c) It can not be introduced into the Lok Sabha. (a) Defection (b) Reservation
(d) It can be introduced into the Lok Sabha. (c) Election (d) Protection of Minorities
UP UDA/LDA (Pre) Exam 2006 276 YCT
CLICK HERE FOR FREE MATERIAL

Ans. (a) : 52nd Constitutional Amendment, 1985 i.e., 62. What is meant by Mixed Economy ?
the 10th scheduled is related to Defection. It provides for (a) Existence of both small & large industry.
disqualification of members of parliament and state (b) Existence of both public sector & private sector.
legislatures on the ground of defection. (c) Existence of both primary & secondary sectors.
57. Which of the following made a (d) None of the above.
recommendation of Panchayati Rajya ? Ans. (b) : Mixed economy is the existence of both public
(a) Govt. of India Act, 1935 sector and private sector. It is an economic system that
(b) Cripps Mission, 1942 combines aspects of both capitalism and socialism.
(c) Indian Indepedence Act, 1947 For ex - (INDIA, US, FRANCE).
(d) Balwant Rai Mehta Committee Report, 1957 63. Under the 11th five year plan, how many IITs
Ans. (d) : Balwant Rai Mehta Committee Report, 1957 (Indian Institute of Technology) are proposed
made a recommendation for Panchayati Raj, B.R. to be set up ?
Mehta is called the father of PRIs. (a) 6 (b) 7
It was the first committee to suggest three-tier PRIs. (c) 8 (d) 9
• Gram Panchayat at the village level. Ans. (c) : Under the 11th Five year plan 8 IITs were
• Panchayat Samiti at the block level. proposed to set up.
• Zila Parishad at the district level. 64. Which of the following bank is the commercial
58. Sikkim became a new state by - bank of the public sector ?
(a) By 30th Amendment (a) I.C.I.C.I Bank (b) H.D.F.C. Bank
(b) By 34th Amendment (c) Indian Overseas Bank (d) U.T.I Bank
(c) By 35th Amendment Ans. (c) : A commercial bank is a financial institution
(d) By 36th Amendment which accepts deposits from the public and gives loans
Ans. (d) : Sikkim became the 22nd state of India via for the purposes of consumption and investment to
Constitution (36th Amendment Act, 1975). make profit.
59. The United Nations has set a target of Indian Overseas Bank is the commercial bank of the
'Education For All' by public sector.
(a) end of year 2012 (b) end of your 2013 65. In India the privatisation of commercial banks
(c) end of year 2014 (d) end of year 2015 took place in -
Ans. (d) : The United Nations has set a target of (a) 1947 (b) 1951
'Education For All' by the end of year 2015. (c) 1969 (d) 1992
60. The Sarva Shiksha Abhiyan (drive) is meant Ans. (c) : In India the privatisation of commercial banks
for the children of what age group ? took place in 1969. This would reduce the financial
(a) All the children belonging to 3-10 yrs age group. burden on the government while also ensuring that
(b) All the children belonging to 4-8 yrs age group. PSBs become more efficient and profit making entities
under private ownership.
(c) All the children belonging to 5-15 yrs age group.
(d) All the children belonging to 6-14 yrs age group. 66. Out of the following taxes which tax is imposed
by the Union govt. but is shared by both the
Ans. (d) : The Sarva Shiksha Abhiyan (SSA) drives is
Union and the States ?
meant for all the children belonging to 6-14 yrs age
(a) Corporation Tax
group. SSA is an Indian Govt. programme aimed at the
universalisation of elementary education in a time bound (b) Tax imposed on extra income earned from
manner, the 86th Amendment to the Constitution of India agriculture
making free and compulsory education to children (c) Tax on Railway fare & goods freight
between the ages of 6 to 14 a fundamental right. (d) Custom Duty
61. Who is the Chairman of the Planning Ans. (b) : Tax imposed on extra income earned from
Commission ? agriculture is imposed by the Union govt. but is
(a) President shared by both the Union and States.
(b) Prime Minister Corporation Tax - levied, collected and
(c) Finance Minister appropriated by the Union.
(d) Governor of Reserve Bank Custom Duty - imposed by the Union.
Ans. (b) : Chairman of the Planning Commission is the 67. The division of Revenue between the Union and
Prime Minister on 1 January 2015, a cabinet resolution States is carried by the recommendation of ?
was passed to replace the Planning Commission with (a) Finance Ministry
the newly formed NITI Aayog (National Institute for (b) Finance Commission
Transforming India). (c) Reserve Bank of India
The chairman of NITI Aayog is also the Prime Minister. (d) NABARD
UP UDA/LDA (Pre) Exam 2006 277 YCT
CLICK HERE FOR FREE MATERIAL

Ans. (b) : The division of Revenue between the Union 74. The most toxic metal emanating from
and State's is carried by the recommendation of finance Automobiles is ?
commission. (a) Copper (b) Lead
The Finance Commission is constituted by the President (c) Cadmium (d) Mercury
under Article 280 of the Constitution. Ans. (b) : Lead (Pb) is the most toxic metal emanating
68. National Dairy Development Board is located from Automobiles.
at ? 75. The vitamin which helps in healing of wounds
(a) Anand (b) Gandhinagar is ?
(c) Vadodra (d) Valsad (a) Vitamin-B (b) Vitamin-C
Ans. (a) : National Dairy Development Board is located (c) Vitamin-A (d) Vitamin-D
at Anand, Gujarat. It was created to boost, finance and
support producer-owned and controlled organisations in Ans. (b) : Vitamin-C helps in healing of wounds. Its
the dairy industry. chemical name is Ascorbic acid. It is a water-soluble
vitamin found in citrus and other fruits and vegetable.
69. The name of which of the following
headquarters is correctly matched ? Vitamin C are found in high proportion in gooseberries
(a) U.N.O. - London (b) W.T.O - Geneva and oranges.
(c) ILO - New York (d) F.A.O - Chicago 76. Which of the following disease is caused by
Ans. (b) : Virus ?
Institute - Headquarter (a) Dephtheria (b) Malaria
UNO - New York, USA (c) Cholera (d) Hepatitis
W.T.O. - Geneva, Switzerland Ans. (d) : Disease caused by virus is Hepatitis.
ILO - Geneva, Switzerland Diphtheria - bacteria (Cornynebacterium)
F.A.O. - Rome, Italy Malaria - Parasite
70. The theory of Atomic bomb is based upon ? Cholera - Vibrio cholerae bacteria
(a) Nuclear Fission (b) Nuclear Fusion 77. White bud in Maize indicates acute deficiency
(c) Both (d) None of these of ?
Ans. (a) : The theory of Atomic bomb is based upon (a) Nitrogen (b) Zinc
Nuclear Fission. Atomic bomb is a weapon with great (c) Copper (d) Manganese
explosive power that results from the sudden release of Ans. (b) : White bud in maize indicates acute
energy upon the splitting or fission of the nuclei of a deficiency of zinc.
heavy element such as plutonium or uranium.
78. Asia's first Human DNA bank has been
71. The Electron-volt is a unit of ? established in ?
(a) Energy (b) Charge of electron (a) New Delhi (b) Hyderabad
(c) Potential difference (d) Power (c) Lucknow (d) Mumbai
Ans. (c) : The electron-volt is a unit of potential
Ans. (c) : Asia's first Human DNA bank has been
difference.
established in Lucknow.
Unit of energy - Joule
Unit of charge of electron - Coulombs 79. Word 'Saimese Twins' is used for -
Unit of Power - Watt (a) Such twins in which there is one male and
one female.
72. In which of the following fruits, Iron is found in
(b) Such twins in which both are females.
enormous quantity ?
(a) Black berry (b) Gran berry (c) Such twins which are afflicted by the same
siam disease.
(c) Loquat (d) Guava
(d) Such twins which are joined with each other.
Ans. (b) : In Gran berry, Iron is found is enormous
quantity. Ans. (d) : Word 'Saimese Twins' is used for such twins
which are joined with each other.
73. Under normal condition, the gas that pollutes
the atmosphere is ____ ? 80. Indian Institute of Remote Sensing is located at?
(a) Carbon monoxide (CO) (a) Ahmedabad (b) Dehradun
(b) Carbon dioxide (CO2) (c) Sriharikota (d) None of them
(c) Nitrogen (N2) Ans. (b) : Indian Institute of Remote Sensing is located
(d) Oxygen (O2) at Dehradun, Uttarakhand.
Ans. (b) : Under normal condition the gas that pollutes 81. In which district of Uttar Pradesh the first
the atmosphere is carbon dioxide. Police Museum has been established ?
While carbon monoxide is a highly pollutant gas, which (a) Allahabad (b) Agra
causes air pollution. (c) Ghaziabad (d) Lucknow
Nitrogen and Oxygen are the composition gases of Ans. (c) : In Ghaziabad district of Uttar Pradesh the
atmosphere Nitrogen-78%, Oxygen - 21%. first police museum has been established.
UP UDA/LDA (Pre) Exam 2006 278 YCT
CLICK HERE FOR FREE MATERIAL

82. In which of the following district of Uttar List-I List-II


Pradesh, the Indian Carpet Technology (a) Best Feature Film 1. Care of
Institute has been established ? Footpath
(a) Agra (b) Mirzapur (b) Best Entertainment 2. Khosla Ka
(c) Moradabad (d) Sant Kabir Nagar Movie Ghosla
Ans. (b) : Indian Carpet Technology Institute has been (c) Best Children 3. Pulijamam
established in Mirzapur district of Uttar Pradesh. Movie
83. The Swavlamban Yojana (Self - Help Scheme) (d) Best Hindi Film 4. Lage Raho
fulfills which of the following objective ? Munna Bhai
(a) Employment to Rural women. Code :
(b) Employment to Urban women. A B C D
(c) Employment to handicapped persons. (a) 1 3 2 4
(d) To provide training & skill to women. (b) 3 4 1 2
Ans. (d) : Swavalamban scheme launched in the year (c) 3 2 4 1
2010 fulfils the objective of providing training and skill (d) 4 2 1 3
to women. Ans. (b) :
84. Dudhwa National Park is located in which List-I List-II
district ? (a) Best Feature Film (3) Pulijamam
(a) Lalitpur (b) Pilibhit
(b) Best Entertainment (4) Lage Raho
(c) Lakhimpur kheri (d) Sonbhadra Movie Munna Bhai
Ans. (c) : Dudhwa National Park is located in the (c) Best Children Movie (1) Care of Footpath
Lakhimpur Kheri district of U.P.
(d) Best Hindi Film (2) Khosla Ka
85. If the circumference of a circle is increased by Ghosla
50%, then its area will increase by what
percentage? 88. Which of the following bank opened its branch
in China first ?
(a) 50% (b) 100%
(a) Bank of Baroda (b) Punjab National Bank
(c) 125% (d) 225%
(c) State Bank of India (d) UCO Bank
Ans. (c) : Let circumference (2πR) = 100 Ans. (c) : State Bank of India has opened its branch in
Then, π R = 50 China first.
50 89. The objective of 'Sangam Yojana' is ______
R=
π (a) to provide pure drinking water.
2 (b) make provision for cleaning of rivers.
 50  2500
∴ Area · π R2 = π ×   = (c) to promote communal harmony.
 π π
(d) provide help to handicaps.
New circumference · 150 Ans. (d) : The objective of 'Sangam Yojana' is to
then, 2 π R = 150 ⇒ R =
75 provide help to handicaps. It was launched on 15th
π August 1996.
 75 
2
5625 90. The name of the train that was started from
∴ New area · πR2 = π ×   = 14th April 2008 between Kolkata and Dhaka
 π  π
(Bangladesh) is ?
 3125 π  (a) Sadbhavana Express (b) Shanti Express
Increase in area ·  × ×100  = 125% (c) Maitri Express (d) Aman Express
 π 2500 
Ans. (c) : The name of the train that was started from
86. A number exceeds by 20 from its 20% value.
14th April 2008 between Kolkata and Dhaka
What is the number?
(Bangladesh) is Maitri Express.
(a) 20 (b) 25
91. In the Indian Open Grand Prix Badminton
(c) 50 (d) 80
Championship organised at Hyderabad in
Ans. (b) : Let number is x. April 2008, the mens single title was won by
20 x Thailands Ponsana Boon Sak who defeated ?
According to question, x × = x – 20, or = x – 20
100 5 (a) Anand Pawar (b) Arvind Bhatt
or, x = 5x – 100, or x – 5x = – 100 (c) Chetan Anand (d) Zendong Guo
or, - 4x = - 100, or x = 25 Ans. (c) : In the Indian Open Grand Prix Badminton
87. Match List-I (Class) with List-II (Winners of Championship organised at Hyderabad is April 2008,
54th National Film Awards) and choose the the Men's Single title was won by Thailands Ponsana
correct answer from the codes given below :- Boon Sak. He defeated Chetan Anand.
UP UDA/LDA (Pre) Exam 2006 279 YCT
CLICK HERE FOR FREE MATERIAL

92. 62nd Santosh Trophy which was organised in 97. The winner of French Open, 2008 Women's
June 2008 was won by ? Singles title was ?
(a) Karnataka (b) West Bengal (a) Dinara Safina (b) Ana Ivanovie
(c) Services (d) Punjab (c) Jelena Jankovic (d) Svetlana Kuznestsova
Ans. (d) : 62nd Santosh Trophy was organised in June Ans. (b) : The winner of French Open 2008 women
2008 was won by Punjab. single was Ana Ivanovie. While French Open 2022,
The 75th Santosh Trophy Winner, 2022 is Kerala. women single winner is Iga Swiatek from Poland.
93. The Cheif guest at the Republic day, 2008 was? 98. Match List-I with List-II and choose the
(a) British Prime Minister, Gordon Brown. correct answer from the codes given below :-
(b) France President - Nicolas Sarkozy List-I List-II
(c) Denmark Prime Minister-Anders Fogh Rasmussen [Place] [Industry]
(d) Russian President - Vladimir Putin (a) Vishakhapatnam 1. Ship Building
Ans. (b) : France President-Nicolas Sarkozy was the (b) Muri 2. Aluminium
chief guest at the Republic day, 2008.
(c) Gurgaon 3. Motor Vehicles
At 2022, Republic Day, No chief guests were invited
due to the 3rd wave of COVID-19. (d) Panki 4. Fertilisers
94. In April 2008, The Convention of Nuclear Code :
Safety (Atomic Safety Conference) was A B C D
organised in - (a) 2 3 4 1
(a) Geneva (b) London (b) 2 4 1 3
(c) Paris (d) Rome (c) 1 2 3 4
Ans. (a) : In April 2008, The Convention of Nuclear Safety (d) 2 4 3 1
(Atomic Safety Conference) was organised in Geneva. Ans. (b) : The correct match is as follows -
95. Who among the following received the Padma List-I List-II
Vibhusan Award in 2008 ? (Place) (Industry)
(a) Madhuri Dixit (b) Ratan Tata (a) Vishakhapatnam - Alumunium
(c) Sachin Tendulkar (d) Vishwanathan Anand (b) Muri - Fertilizers
Ans. (d) : Vishwanathan Anand an Indian chess (c) Gurgaon - Ship building
player received the Padma Vibhushan Award in 2008. (d) Panki - Motor Vehicle
While in 2022, Padma Vibhushan awardees are - 99. Match List-I with List-II and choose the
Prabha Atre, Bipin Rawat, Kalyan Singh and Radhey correct answer from the codes given below :-
Shyam Khemka.
List-I List-II
Prabha Atre - Indian classical vocalist
[Mineral Production] [States]
from the Kirana Gharana.
(a) Mineral oil 1. Odisha
Gen. Bipin Rawat - First Chief of Defence
Staff of the Indian Armed (b) Gypsum 2. Karnataka
forces. (c) Gold 3. Gujarat
Kalyan Singh - Indian politician and a (d) Bauxite 4. Rajasthan
member of BJP and former Code :
CM of U.P. A B C D
Radheyshyam - Journalist. (a) 1 2 3 4
Khemka (b) 2 1 4 3
96. After the signature of President on the (c) 4 3 1 2
recommendations of the Delimitation (d) 3 4 2 1
Commission, there demarcation of boundaries of
Parliamentary and legislative assembly regions Ans. (d) :
will be carried with some exceptions. Which of List-I List-II
the following are the exceptions to this ? [Mineral Production] [States]
(1) Arunachal Pradesh & Manipur (a) Mineral oil 3. Gujarat
(2) Assam (b) Gypsum 4. Rajasthan
(3) Nagaland (c) Gold 2. Karnataka
(4) Jharkhand (d) Bauxite 1. Odisha
Choose the correct answer from the codes 100. 24th March (2008) is celebrated as the -
given below :
(a) World AIDS day.
(a) only 1 (b) 2 and 3
(c) 3 and 4 (d) 1, 2, 3, 4 (b) World Handicap day
(c) World Environment day
Ans. (d) : The states which are exceptions for the
demarcation of boundaries of parliamentary and (d) World Tuberculosis Day (T.B.)
legislative assembly regions are Arunachal Pradesh and Ans. (d) : 24th March (2008) is celebrated as the World
Manipur, Assam, Nagaland and Jharkhand. Tuberculosis Day (T.B.).
UP UDA/LDA (Pre) Exam 2006 280 YCT
CLICK HERE FOR FREE MATERIAL

UPPSC UDA/LDA (Pre) Exam-2001


GENERAL STUDIES
Solved Paper
1. Match the List-I with List-II and choose the 4. In which of the following texts Remarriage of
correct answer from the codes given below :- widow is prohibited -
List-I List-II (a) Jataka (b) Manusmriti
A. Rigveda 1. Musical Hymns (c) Yagyavalakya smriti (d) Arthashastra
B. Yajurveda 2. Hymns and Rituals Ans. (b) : While Manusmiriti allows men to remarry, it
C. Samveda 3. Occult and Hypnotism is strictly prohibited for the woman. It is stated that
even when she withers due to illness or depression, she
D. Atharvaveda 4. Hymns and Prayers "must never mention even the name of another man."
Code : 5. In the earlier centuries information regarding
A B C D fast/brisk commercial relations between India
(a) 4 2 1 3 and Rome has been obtained from which
(b) 3 2 4 1 archaeological site excavations ?
(c) 4 1 2 3 (a) Madurai (b) tamralipt
(d) 2 3 1 4 (c) Tondi (d) Arikamedu
Ans. (a) : Ans. (d): Arikamedu, located in the Union Territory of
List-I List-II Pondicherry, is a famous archaeological site in India.
The remains found from this place give information
A) Rigveda Hymns and Rituals
about ancient trade relations between India and Rome.
B) Yajurveda Hymns and Prayers Tamralipti, located in West Bengal, was a well-known
C) Samveda Musical Hymns port of eastern India. It was also a famous education
D) Atharvaveda Occult and Hypnotism centre. Hiuen Tsang, Fahien, Itsing had studied while
2. In which Upanishad, the dialogue between staying here. The present Madurai of Tamil Nadu
province was the city of Madura in ancient times. It
Nachiketa & Yama on the subject of spritual
served as the capital of the Pandya kingdom. The
knowledge is mentioned ? Meenakshi temple of Madurai is famous. Tondi was a
(a) Brihadaranyak Upanishad trading port of the Cholas.
(b) Chandogya Upanishad
6. Match the list-I and List-II and choose the
(c) Kena Upanishad correct answer from the codes given below :
(d) Kathopanishad List-I List-II
Ans. (d) : The story dealing with dialogue between (Court Poets) (King)
Nachiketa and Yama, is mentioned in Kathopanishad. A. Amir Khusro 1. Chandragupta-II
3. In India, the success of Alexander was B. Kalidas 2. Samudragupta
attributed to following reasons :- C. Harisena 3. Harshavardhan
1. At the time there was no central authority
D. Banabhatt 4. Allauddin Khilji
in India.
2. He had a superior army. Codes :
A B C D
3. He got assistance from traitor rulers.
(a) 1 2 3 4
4. He was a good administrator.
(b) 4 1 2 3
Choose the correct answer from the codes
(c) 4 3 2 1
given below :-
(d) 2 4 1 3
(a) 1 and 2 (b) 1, 2 and 3
(c) 2, 3 & 4 (d) All of them Ans. (b) :
List-I List-II
Ans. (d) : During the time of the invasion by Alexander
the north-western region of India was divided into a (Court Poets) (King)
number of small independent states. There was no A. Amir Khusro Allauddin Khilji
central authority in India during that time. His military B. Kalidas Chandragupta-II
and administrative capability was for more superior C. Harisena Samudragupta
than those of Indian rulers of those time. Above all he D. Banabhatt Harshavardhan
also got support from traitorous ruler like Ambhi. The Political and Social Condition between 800 AD
Therefore all the given statements are correct. to 1200 AD
UP UDA/LDA (Pre) Exam 2001 281 YCT
CLICK HERE FOR FREE MATERIAL

7. The Gurjar-Pratihara dynasty was founded by- 13. Among the following Sufism Cults (Silsilas)
(a) Nagabhat (b) Vatsray which was against playing music ?
(c) Harshavardhan (d) Mihir Bhoja (a) Chistiya (b) Suharvardi
Ans. (a): The period from C 750-1000 CE was marked (c) Kadiriya (d) Nakshabandi
by the growth of three important political powers, Ans. (d) : The Naqshbandi Silsila was founded by
namely, the Gurjara-Prathiharas (western India); the Khwaja Bahauddin Naqshband in the 14th century. This
Palas (eastern India), and the Rashtrakutas (Deccan). sect was founded in India by Khwaja Baqi Billa around the
* The Gurjara-Pratiharas dynasty was founded by 16th century. His main disciple was Sheikh Ahmed
Nagabhatta-I Sirhindi. This silsila avoided music. The Mughal emperor
* The Palas dynasty was founded by Gopala. Aurangzeb was a supporter of this Sufi order.
* The Rashtrakutas dynasty was founded by Dantidurga. 14. The title of Persian Translation of Epic
8. Amir Khusro was born at ? Mahabharat is -
(a) Agra (b) Barabanki (a) Anware Suheli (b) Razmnama
(c) Etah (d) Etawah (c) Hast Bahishat (d) Ayar Danish
Ans. (c) : Amir Khusrow was a Sufi mystic and a spiritual Ans. (b) : The Razmnama (Book of War), the
disciple of Nizamuddin Auliya. He was born in 1253 at first-ever Persian translation of the
Patiyali in Kasganj district in modern-day Uttar Pradesh.
He lived in the courts of as many as ten different rulers of Mahabharata was commissioned by Mughal
the Delhi Sultanate. Alauddin Khalji gave Amir Khusrau emperor Akbar in 1582.
the title of Tuti-i-Hind which means 'Parrot of India'. When 15. Moti Masjid in Delhi's Red Fort was
the question was asked it was in Etah district. constructed by -
9. Jaunpur was etablished in the memory of ? (a) Akbar (b) Jahangir
(a) Giyasuddin Tughlak (c) Shahjahan (d) Aurangzeb
(b) Mohammad-bin-Tughlak Ans. (d) : The moti masjid was built by the Mughal
(c) Firozshah Tughlak emperor Aurangzeb at the Red Fort complex in Delhi
(d) Akbar from 1659-1660 for his 2nd wife Nawab Bai (Rahmat-
Ans. (b) : In the memory of Mohammad-bin Tughlaq, un-Nissa).
Firoz Shah Tughlaq founded the city of Jaunpur (Jauna 16. Choose the correct chronological order of the
Khan). Ghiyas-ud-din Tughluq or Ghazi Malik was the following Muslim rulers from the codes listed
founder of the Tughluq dynasty. below :-
10. Who among the following did not contribute in 1. Ahmadshah Abdali, 2. Muhammad Shah
the construction of Qutub-Minar 3. Jahangir, 4. Bahadur Shah
(a) Qutubuddin Aibek (b) Iltutmish (a) 1, 2, 3, 4 (b) 4, 3, 2, 2
(c) Allauddin Khilji (d) Firozshah Tughlaq (c) 3, 2, 1, 4 (d) 2, 1, 3, 4
Ans. (c) : The construction of the Qutub minar was started Ans.(c): Correct chronological orders of the given
in 1200 AD by Qutubu'd-din Aibak .His successor, Muslim rulers are :
Iltutmush, added three more storeys, and in 1368, Firoz (1) Jahangir - 1605-1627 AD
Shah Tughlaq constructed the fifth and the last storey.
(2) Muhammad Shah - 1719 - 1748 AD
11. Shuddha Advaitavad was enunciated by (3) Ahmadshah Abdali - 1747 - 1772 AD
(a) Madhavacharya (b) Vallabhacharya (4) Bahadur Shah II - 1837-1862 AD
(c) Srikantacharya (d) Ramanujan
17. Which of the following Sikh Gurus were
Ans. (b) : Indian philosophy sentenced to death by their Contemporary
1.Ramanujacharya -Vishishtadvaita Rulers.
2. Madhavacharya - Dvaita (1) Guru Angad (2) Guru Arjun Dev
3. Nimbarka- Dvaitadvaita (3) Guru Hargovind (4) Guru Teg Bahadur
4. Vallabhacharya- Shuddha Advaita Choose the correct answer from the following
12. Which of the following Sufi Saints, equated codes :-
Lord Krishna as an Muslim Pir (Saint) ? (a) 2 and 4 (b) 2 and 3
(a) Shah Mohammed Gaus (c) 1 and 3 (d) 1 and 2
(b) Shah Abdul Aziz Ans. (a) : Guru Arjan Dev tried to strengthen the Sikh
(c) Shah Waliullah community. The guru had given his blessings to the
(d) Khwaja Mir Dard rebellious prince Khusrau because of which Jahangir
Ans. (a) : Shattari Silsila was founded by Sheikh hanged the Guru in 1606. Teg Bahadur, the youngest
Sirajuddin Abdullah Shattari and gained popularity son of the sixth Guru Har Gobind, opposed Aurangzeb's
because of Shaikh Mohammad Ghaus of Gwalior. Sheikh policies, due to which he was imprisoned in Delhi and
Mohammed Ghaus's most famous work is 'Jawahir-e- murdered in December 1665 for not accepting Islam.
Khamsa', in which he expressed his spiritual quest. Sheikh Guru Arjan's son Hargobind revolted against Shah Jahan
Mohammed Ghaus was particularly interested in and defeated his army in 1628. He established the Akal
Hinduism, he considered Krishna as Auliya. Takht. Guru Angad developed Gurmukhi script.
UP UDA/LDA (Pre) Exam 2001 282 YCT
CLICK HERE FOR FREE MATERIAL

18. The most decisive battle, which established Ans. (b) : By the Regulating Act of 1773, a Supreme
British supremacy in India was - Court was established in Calcutta. Pitt's India Act
(a) Battle of Buxar (1784) established the dual system of control by the
(b) Battle of Plassey British Government and the East India Company. By
(c) Battle of Wandiwash the Charter Act of 1813, Christian missionaries were
(d) Third battle of Panipat allowed to work and preach in India. By the Charter
Ans. (a) : Battle of Buxar was the most decisive battle Act 1833, the Governor-General's Council was
that established British superemacy in India. The Battle empowered to make laws for the whole of India. A law
of Buxar (22 October 1764) was a conflict at Buxar in member was appointed in this council by the Charter
northeastern India between the forces of the British East Act of 1833.
India Company, commanded by Major Hector Munro, 22. The word Mahatma was added before the
and the combined army of an alliance of Indian states Gandhi name during -
including Bengal, Awadh, and the Mughal Empire . (a) Champaran Satyagrah
19. Who was All India Trade Union Congress first (b) Satyagrah started against Rowlatt Act
President ? (c) The Amritsar session of Indian National
(a) Dewan Chamanlal Congress 1919
(b) Acharya N.G. Ranga (d) The beginning of Khilafat Movement
(c) Swami Sahajanand Ans. (a) : The Champaran Movement (1917–18) led by
(d) Lala Lajpat Rai Gandhi was the first mass movement in the national
Ans. (d): All India Trade Union Congress (AITUC) was freedom struggle, in which Gandhiji used 'truth and
established in 1920. It was an organization related to the non-violence' as his weapons. Seeing the successful
labour movement under the Congress. Lala Lajpat Rai operation of the movement and the leadership ability of
was its founding president. All the persons given in the Gandhiji, Rabindranath Tagore addressed him with the
question i.e. Dewan Chaman Lal (First General Secretary title of 'Mahatma'.
of AITUC) Acharya N.G. Ranga and Swami Sahajanand,
were eminent leaders of the labour movement. 23. Who among the following is known as mother
of India Revolution ?
20. Under which of the following Acts, the Central (a) Annie Basant (b) Sarojini Naidu
Legislative Council obtained power to
undertake debate on budget - (c) Ramabai (d) Bhikaji Rustam Cama
(a) Indian Council Act, 1861 Ans. (d) : Madame Bhikaji Rustam Cama is known as
(b) Indian Council Act, 1892 the 'Mother of Indian Revolution' because she raised the
(c) Indian Council Act, 1909 first tricolour flag of India is an International
(d) The Government of India Act, 1919 Conference.
Ans. (b) : Under the Indian Councils Act 1892, the 24. Who said -
Central Legislative Assembly got the power to debate "Daro Deewar pe hasrat ki Nazar Karte hain,
the federal budget for the first time but it did not have Khush Raho Ahle-Watan hum to Safar Karte
the right to vote on the budget. hain"-
21. Match List-I with List-II and choose the (a) Ashfaqullah (b) Bahadur Shah Zafar
correct answer from the codes given below :- (c) Ram Prasad Bismil (d) Wajid Ali Shah
List-I List-II Ans. (d) : The above statement is of Wajid Ali Shah,
(a) Establishment of 1. Regulating the last Nawab of Awadh. This poem was used by him
Dual system of Act, 1773 during his imprisonment.
control. 25. Statement (A) : Salt Agitation by Mahatma
(b) Establishment of 2. Pitt's India Gandhi was conducted in 1930.
Supreme Court. Act Reason (R) : Gandhijis objective was to make
(c) Permussion granted 3. Charter Act, salt available to poor people free of cost.
to English 1913 With reference to above statements which of
Missionaries to the followings are true.
work in India. (a) Both (A) & (R) are true and (R) is the right
(d) Appointment of 4. Charter Act, explanation of (A).
Legal member in 1833 (b) Both (A) and (R) are true but (R) is not the
the Governor- right explanation of (A).
General Council.
(c) (A) is true but (R) is false.
Codes :
(d) (A) is false but (R) is true.
A B C D
(a) 1 2 3 4 Ans. (c) : The Salt march lasted from 12th March, 1930
(b) 2 1 3 4 (Sabarmati) to 6th April, 1930( Dandi) as a direct action
(c) 1 2 4 3 campaign of tax resistance and nonviolent civil
(d) 2 4 1 3 disobedience against the British Rule.
UP UDA/LDA (Pre) Exam 2001 283 YCT
CLICK HERE FOR FREE MATERIAL

26. Who accused Indian National Congress of Code :


playing the politics of - Pray, Plea & Protest ? A B C D
(a) Lala Hardayal (a) 1 3 2 4
(b) Bal Gangadhar Tilak (b) 3 1 4 2
(c) Subhash Chandra Bose
(c) 3 1 2 4
(d) Sardar Bhagat Singh
(d) 1 3 4 2
Ans. (b) : Bal Gangadhar Tilak accused Indian
National Congress of playing the politics of three P’s – Ans. (c) :
Prayer, petition and protest as political mendicancy. List-I List-II
27. During the non-co-operation movement, who (Tourist Places) (State)
wrote to Gandhiji that it is just "a ruthless (a) Chakrata 1. Uttaranchal
destruction on the issue of burning foreign clothes?
(b) Haflong 2. Assam
(a) Madan Mohan Malviya
(b) Ravindra Nath Tagore (c) Kalimpong 3. West Bengal
(c) Shaukat Ali (d) Kufri 4. Himanchal Pradesh
(d) Moti Lal Nehru 32. Out of the following states of India which state
Ans. (b) : Rabindranath Tagore said about the burning has recently started Green House Farming ?
of foreign clothes during the non-cooperation (a) Haryana (b) Punjab
movement, it is 'a ruthless destruction'. (c) Maharashtra (d) Uttar Pradesh
28. The tinkathia system prevalent in the
Ans. (a) : Green house farming in India is done in an
Champaran district meant ?
area of about 3000 hectares. There is a lot of potential
(a) Cultivation of Indigo jon 3/20 portion of land
by peasants. for protected agriculture in the states of North India.
(b) Cultivation of Indigo on 3/21 part of land by When the question was asked, green house agriculture
farmers. was started by Haryana.
(c) Cultivation of Indigo on 3/19 part of land by 33. Match the following :-
farmers. List-I List-II
(d) None of the above. (Dams) (States)
Ans. (a): The 'Tinkathia system' was the medium of (a) Farakka 1. West Bengal
exploitation of the farmers in Champaran (Bihar). Under
this system farmers were faced to cultivate Indigo in 3 (b) Ghatprabha 2. Karnataka
parts of the land out of every 20 parts of the land. (c) Hirakud 3. Orrisa
29. A list of traditional dresses of women and their (d) Kakrapar 4. Gujarat
respective states is provided below. Which of Code :
them is not correctly matched ?
A B C D
(a) Bakhu - Sikkim (b) Mekhala - Asam
(c) Mundu - Chattisgarh (d) Pheran - Kashmir (a) 1 4 2 3
(b) 2 4 3 1
Ans. (c) : Mundu is a type of white dhoti/lungi which is
very popular in South India. Pheran is a long, loose (c) 4 3 2 1
woolen garment of Kashmir while Mekhla garment is (d) 1 2 3 4
related to Assam and Bakhu is a loose cloak like Ans. (a) : Correctly match the following -
garment worm by people of Sikkim. List-I List-II
30. Which of the following is a major tribe of (Dams) (States)
Dhauladhar Range ?
A. Farakka West Bengal
(a) Abor (b) Gaddi
(c) Lepcha (d) Tharu B. Ghatprabha Karnataka
Ans. (b): The Dhauladhar range begins from a place C. Hirakud Odisha
near Dalhousie of Himachal Pradesh and merges with D. Kakrapar Gujarat
Pir Panjal near Manali of Himanchal Pradesh. Gaddis 34. Which is the leading state of India in terms of
are major tribe of the Dhauladhar Range. production of Soyabean ?
31. Match List-I with List-II and choose the (a) Maharashtra
correct answer from the codes given below :-
(b) Madhya Pradesh
List-I List-II
(Tourist Places) (State) (c) Punjab
(a) Chakrata 1. Asam (d) Kerala
(b) Halflong 2. West Bengal Ans. (b) : Madhya Pradesh is called 'Soybean State',
(c) Kalimpong 3. Uttaranchal because of being leading state in production of
(d) Kufri 4. Himanchal Soyabean. The second place belongs to Maharashtra for
Pradesh the production of Soyabean.
UP UDA/LDA (Pre) Exam 2001 284 YCT
CLICK HERE FOR FREE MATERIAL

35. Match the List-I with List-II and choose the 38. In India an important Nuclear Fuel available in
correct answer from the codes given below. sufficient quantity is -
List-I List-II (a) Uranium (b) Thorium
(Multipurpose projects) (Rivers) (c) Iridium (d) Plutonium
(a) Idukki 1. Betwa Ans. (b): Thorium is the most abundant nuclear fuel
available in India. It is found in large quantities in
(b) Matatila 2. Godavari
Kerala and Jharkhand.
(c) Nagarjuna Sagar 3. Krishna
39. Match List-I with List-II
(d) Pochampad 4. Periyar
List-I List-II
Code : (Place) (Industry)
A B C D (a) Vishakhapatnam 1. Motor-cars
(a) 4 2 3 1
(b) Muri 2. Ship-building
(b) 2 1 3 4
(c) Gurgaon 3. Fertiliser
(c) 4 1 3 2
(d) Panki 4. Aluminium
(d) 1 3 4 2
Choose the correct answer from the codes
Ans. (c) : Correct match the following are : given below-
List-I List-II Code :
(Multipurpose project) (Rivers) A B C D
A. Idukki Periyar (a) 2 3 4 1
B. Matatila Betwa (b) 1 2 3 4
C. Nagarjuna Sagar Krishna (c) 2 4 3 1
D. Pochmpad Godavari (d) 2 4 1 3
36. In the summer season we feel humidity when Ans. (d) : Correct match of List-I with List -II
the weather is - List-I List-II
(a) Turbulent (b) Hot (Place) (Industry)
(c) Scorching (d) Humid A. Vishakhapatnam Ship-building
Ans. (d): During the rainy season when the sky is B. Muri Aluminium
covered with clouds humidity increases because the C. Gurgaon Motor-Car
heat of the earth is not radiated back. Because of it the D. Panki Fertiliser
person experiences humidity. 40. Match the List-I and List-II and choose the
37. Statement (A) : In comparison to the western correct answer from the codes given below :
coast, eastern coast is more affected by List-I (State) List-II (Atomic
cyclones. power station)
Reason (R) : India Eastern Coast falls in the (a) Gujarat 1. Narora
North-East trade winds belt. (b) Karnataka 2. Kakrapar
With reference to above statements, which of (c) Rajasthan 3. Rawat bhata
the following is true - (d) Uttar Pradesh 4. Kaiga
(a) Both (A) and (R) are true and (R) is the Code:
correct explanation of (A).
A B C D
(b) Both (A) and (R) are true but (R) is not the
(a) 1 2 3 4
correct explanation of (A).
(b) 4 3 2 1
(c) (A) is true but (R) is false.
(c) 2 4 3 1
(d) (A) is false but (R) is true.
(d) 4 2 1 3
Ans. (b): Tropical cyclones originate and intensity over
Ans. (c) : Correct match the following are :
worm tropical oceans. The Conditions favorable for the
formation and intensification of tropical cyclones are : List-I List-II
1. Large Sea Surface with temperature higher than (State) (Atomic Power Station)
27ºC. A. Gujarat Kakrapar
2. Presence of the Conolis force. B. Karnataka Kaiga
3. Small variations in the Vertical Wind Speed. C. Rajasthan Rawatbhata
4. A pre-existing weak-low-pressure area or low-level D. Uttar Pradesh Narora
cyclonic circulation. 41. Statement (A) : Lignite is a low grade coal in
5. Upper divergence above the Sea level system. which percentage of carbon is 35-40%.
The above reasons make eastern coast more prone for Reason (R) : Jharkhand in India is the major
the formation of Tropical Cyclone or Cyclones. producer of Lignite.
UP UDA/LDA (Pre) Exam 2001 285 YCT
CLICK HERE FOR FREE MATERIAL

In context of above statements which of the (a) Both (A) and (R) are true and (R) is the
following is true - correct explanation of (A).
(a) Both (A) & (R) are true and (R) is the correct (b) Both (A) and (R) are true but (R) is not the
explanation of (A). correct explanation of (A).
(b) Both (A) & (R) are true but (R) is not the (c) (A) is true but (R) is false.
correct explanation of (A). (d) (A) is false but (R) is true.
(c) (A) is true but (R) is false. Ans. (a) : The North Atlantic Shipping Route is the
(d) (A) is false but (R) is true. busiest sea route in the world. It connects the two most
Ans. (c) : Classification of Coal can be done on the developed industrial regions of North America and
basis of carbon content and time period into four ranks: Europe. Hence both assertion and reason are true and
anthracite, bituminous, subbituminous, and lignite. reason is also the correct explanation of the assertion.
Anthracite: It is the best quality of coal with highest 45. Bermuda Triangle is located in -
calorific value and carries 80 to 95% carbon content. (a) the Western-Northern Atlantic ocean.
Bituminous: It has a low level of moisture content with (b) the South-Eastern Atlantic ocean.
60 to 80% of carbon content.
(c) North Pacific ocean
Lignite carries 40 to 55% carbon content.
(d) South Indian ocean
Peat coal is of low calorific value and have less than
40% carbon content. Ans. (a) : The Bermuda Triangle (Devil's Triangle) is
Billion Tonne (1.4.2019). The major deposits of Lignite located in the northwestern Atlantic Ocean. This
are located in the State of Tamil Nadu, followed by triangle extends between Florida, Puerto Rico and
Rajasthan, Gujarat, Kerala. Bermuda. It is also known as the Satanic Triangle.
42. Out of the following pair of cities which of the 46. Most densely populated SAARC nation is ____
pair cities recently have been linked by a 6 lane (a) India (b) Pakistan
express way - (c) Srilanka (d) Bangladesh
(a) Ahmedabad and Vadodra Ans. (d) : Out of the given options Bangladesh (1,265
(b) Dhaka and Chitagoan people per square km.) has highest population density
(c) Mumbai & Pune among SAARC nations. However as per World Bank
(d) Islamabad & Lahore data, in 2020 Maldives has the highest population
Ans. (c) : When question was asked Mumbai and Pune density (1802 person per square km.) among SAARC
had been connected by 6 lane Express way .. countries.
43. Match List-I with List-II and choose the 47. Kamaiya system is -
correct answer from the codes given below - (a) an establishment of Canals in Nepal which
List-I List-II covers/irrigates the non-irrigated land.
(Climate Type) (Region) (b) a traditional system of bonded labour
(a) Equatorial 1. California practiced from one-generation-to other
generation.
(b) Mediterranean 2. Bangladesh
(c) A contractual system for labourers working in
(c) Monsoonal 3. Sudan Assam's Tea gardens.
(d) Savannah 4. Congo (d) A system of labourers for loading &
Code : unloading of goods on ports.
A B C D Ans. (b) : Kamaiya system is a system of bonded labour
(a) 4 1 2 3 in the Terai region of Nepal which is passed on from
(b) 2 3 1 4 generation to generation. The Deuba government in
(c) 1 2 3 4 Nepal had decided to end it in 2000. It was earlier
(d) 4 3 2 1 prevalent in Eastern India especially in Bihar.
Ans. (a): Correct match the following are . 48. The most urbanised region among the
List-I List-II following regions is -
(Climate type) (Region) (a) East Asia (b) South Asia
A. Equatorial Congo (c) Europe (d) Oceania
B. Mediterranean California Ans. (c) : According to the question Europe is the most
C. Monsoonal Bangladesh Urbanized continent (74.5%)
D. Savannah Sudan Urbanization in the year 2021 is as following
44. Statement (A) : In the world the North-Atlantic Country/continent 2021 (estimated)
Navigational route is the busiest sea route. Oceania 68%
Reason (R) : It connects two major Industrial Europe 75%
regions with reference to the above statements Asia 66.18%
which of the following conclusions is true - Whole World 56.61%
UP UDA/LDA (Pre) Exam 2001 286 YCT
CLICK HERE FOR FREE MATERIAL

49. Although Indian population is increasing at an Ans. (c) : In the year 1999-2000, the states in
astonishing rate, but the birds population is descending order were West Bengal, Assam, Nagaland
dwindling fast because - and Meghalaya according to the percentage of men
1. There has been an outstanding increasing employed per 1000 in the urban area.
in the number of poachers. 53. Statement (A) : The urban population of India
2. Biological insecticides and organic is more than total population of Britain.
fertilisess are being used on a large scale. Reason (R) : The urban standard of Britain is
3. There has been a sharp decrease of natural higher than India's urbanisation standard.
habitats of birds on a large scale.
Choose the correct answer from the codes
4. Insecticides, Fertilisers and Mosquito given below :-
repellent chemicals are being used on a big
(a) Both (A) and (R) are true (R) is the correct
scale.
explanation of (A).
Choose the correct answer from the codes
(b) Both (A) and (R) are true but (R) is not the
given below :-
true explanation of (A).
(a) 1 & 2 (b) 2 and 3
(c) (A) is true but (R) is false.
(c) 3 and 4 (d) 1 & 4
(d) (A) is false but (R) is true.
Ans. (c) : India's population is increasing at an amazing
rate. But the number of birds is decreasing rapidly Ans. (b) : India's urban population is 377.11 million,
because the habitat of birds is being reduced on a large while the total population of Britain is 66.87 million.
scale. Roads, towns, tourist places, canals and power Therefore, the urban population of India is more than
stations have been built that reduces natural habitat for the total population of Britain. Britain's urbanization
birds. Another reason for the rapid decline in the (83.4%) is higher than India's (34.3%) due to better
number of birds is the large-scale use of insecticides, infrastructure and other facilities.
fertilizers and mosquito repellents. 54. Under the Cabinet Mission Plan, in order to
50. The Urban population Annual Exponential prescribe the total number of members alloted
growth rate in India was highest in the - to each state every representative member
(a) 1951-61 decade (b) 1961-71 decade represented what ratio of population -
(c) 1971-1 decade (d) 1981-91 decade (a) Eight lac persons
Ans. (b) : The annual exponential growth rate of India's (b) 10 lac persons
urban population was the highest in the decade 1961-71. (c) 12 lac persons
51. In the context of 1991 census, which of the (d) 15 lac persons
following statements is not true - Ans. (b) : Under the Cabinet Mission Plan each
(a) The largest concentration of Indias urban province and princely state (or group of states in case of
population was found in Maharashtra. small states) were to be allotted seats in proportion to
(b) The most urbanised state in the country was their respective population. Roughly, one seat was to be
Goa. allotted for every one million (10 Lakh) population.
(c) In the fourth Category cities, the number of 55. Meaning of "Extra Constitutional Rights" is -
urban centres was highest. (a) that power which encroaches upon the
(d) In the first class cities, maximum concentration constitutional
of urban population of India was found. (b) that person which possesses extraordinary
Ans. (c) : According to the population of the year 1991, powers under the constitution
the highest concentration of urban population of India (c) the power which is non-conforming to
was found in Maharashtra. Goa was the most urbanized constitution
state in India. Also, the maximum concentration of (d) that officer in which all the powers of state
urban population was in the first class cities. are vested in the event of declaration of
The total urban population in the country as per Census emergency
2011 is more than 377 million forming 31.16% of the Ans. (c) : Extra constitutional rights are those inherent
total population. rights that are not confirmed by the constitution.
52. In the context of year 1999-2000, arrange the 56. In the preamble of Indian Constitution by
following states in an descending order in reading the inflicted (included) word Socialism
terms/accordance with the percentage of men with which Article of constitution does
employed per 1000 in the urban area - Supreme Court been enabled to define the
A) Meghalaya B) Nagaland fundamental right of 'Equal pay for equal
C) Assam D) West Bengal work' -
Choose the correct answer from the codes (a) Article 14
given below :- (b) Article 14 & 15
(a) D, C, A, B (b) A, B, C, D (c) Article 14, 15 and 16
(c) D, C, B, A (d) D, A, C, B (d) Article 14 & 16
UP UDA/LDA (Pre) Exam 2001 287 YCT
CLICK HERE FOR FREE MATERIAL

Ans. (c) : The provisions of the articles given in the Ans. (a) :
question are as follows: List-I List-II
Article 14 - Right to equality (Articles of the (Subject)
Article 15 - Resistance to discrimination on grounds of constitution)
religion, caste, sex, place of birth etc. Article 40 1. Organisation of village
Article 16 - Equality of opportunity in relation to public Panchayats
employment. Article 41 2. Right to work, education and
Hence, Equal pay for equal work is in accordance with public assistance in certain case
the Article 14, 15 & 16 of the constitution of India. Article 44 3. Uniform civil code for citizens
57. Which of the following rights is available under Article 48 4. Organisation of agriculture
Article 19(1)d read with 'Article 21'? and animal husbandry
(a) Right to foreign travel 60. In the constitutional draft, the original plan of
(b) Right to take Asylum (Shelter) providing elected governors was shelved,
(c) Right to privacy because-
(d) Right to obtain information 1) That meant to go for a second (two round)
Ans. (a) : Article 19 (1) (d) is related to 'right to election.
freedom of travel' and Article 21 is related to 'right to 2) Election would have been fought on main
life and personal liberty'. Therefore right to travel political issues.
abroad is available under Article 19(1) (d) read with 3) Governor elected would have considered
himself superior to Cheif Minister.
Article 21.
4) Governor has to act under the
58. Which of the following statements related to Parliamentary system.
Fundamental Duties is not correct ? Which are/is the correct statement :-
(a) They can be made effective through writ of (a) 1 & 2 (b) 2 and 3
mandamus. (c) 1, 3 & 4 (d) 2, 3 & 4
(b) They can be promoted only by constitutional Ans. (c) : The original plan of provision for elected
process. governors was abandoned in the draft constitution
(c) They can be employed for definining because it would have led to differences between the
vague/unclear laws. chief minister and the governor over their official
(d) In order to follow any specific duty comes position, as well as it would be a wastage of resource if
under the ambit of constitutional law which is direct elections of Governor is held.
determined by court. 61. A list of some states, and the number of members
Ans. (a) : Fundamental duties cannot be made effective allotted to them has been given below. Which of
by Mandamus. Mandamus is a type of writ which is them is not correctly matched?
issued by the Supreme Court or High Court. (a) Madhya Pradesh-16 (b) Maharashtra - 19
Fundamental duties are non enforceable in nature. (c) Tamil Nadu - 18 (d) West Bengal - 16
59. Match List-I with List-II and choose the Ans. (a) : After the formation of the state of
correct answer from the codes given below :- Chhattisgarh, there are 11 Rajya Sabha seats in Madhya
Pradesh while have 5 seats in Chhattisgarh.
List-I List-II
(Articles of the (Subject) 62. For the first time an M.P./M.L.A was
constitution) disqualified as a member of the house that has
absented for sixty continuous days in the house
(a) Article 40 1. Formation of without permission.
Gram Pancayats That member was of -
(b) Article 41 2. Right to work (a) Rajya Sabha
(c) Article 44 3. Common Civil (b) Lok Sabha
Code (c) Punjab Legislative
(d) Article 48 4. Provision of (d) Rajasthan Legislative Assembly
Agriculture and Ans. (a) : For the first time an MP was disqualified as the
animal husbandry member of Rajya Sabha that has absented for sixty continuous
Code : days. That member was Barjinder Singh Hamdard an
A B C D independent member of Rajya Sabha Assembly.
The Chairman or Speaker can declare the seat vacated if
(a) 1 2 3 4
a member has remained absent from all its meetings for
(b) 2 3 1 4 a period of 60 days without permission. While
(c) 1 3 4 2 calculating the 60 days, the period for which the house
(d) 3 2 4 1 is prorogued or adjourned is not counted.
UP UDA/LDA (Pre) Exam 2001 288 YCT
CLICK HERE FOR FREE MATERIAL

63. Which of the following Article of Indian 67. Prime Minister's "Gram Sadak Yojana"
Constitution is related to Appellate jurisdiction (a) Increasing access to village through roads and
of Supreme Court in case of a Constitutional to provide food grains to the poorest of the
dispute ? poor on subsidised rates.
(a) Article 131 (b) To arrange patrolling & keeping vigil on
(b) Article 132 misuse of electricity by unauthorised persons.
(c) To read article 133 in combination of Article (c) To facilitate/expedite speedy arrival of police
134 A personnels at the incident site to curb increase
(d) To read article 132 in combination with in crimes.
article 134A (d) To construct roads in those villages which are
Ans. (d) : The appellate jurisdiction of the Supreme not properly connected to main roads in order
Court can be invoked by a certificate granted by the to augment community development work.
High Court concerned under Article 132(1), 133(1) or Ans. (d) : The Pradhan Mantri Gram Sadak Yojana was
134 of the Constitution in respect of any judgement, launched in December 2000. It aims to provide road
decree or final order of a High Court in both civil and connectivity to the initial 1.6 lakh unconnected rural
criminal cases, involving substantial questions of law as
habitations with a population of 500 persons and above
to the interpretation of the Constitution.
by the end of the Tenth Plan period.
64. Statement (A) : A distinct change in their
Activities has been discovered due to 68. Which of the following programmes has not
reservation of seats for women belonging to been included in the Swarna Jayanti Swarojgar
Scheduled Caste, Scheduled tribe and Yojana ?
backward classes on the posts of village (a) I.R.D.P. (b) TRYSEM
Panchayat Cheifs, Village heads (Mukhiyas) (c) DWAKRA (d) J.R.Y
and Gram Panchayat President. Ans. (d) : On April 1, 1999, the Government of India
Reason (R) : The rural sector women have been started 'Swarna Jayanti Gram Swarozgar Yojana' for the
demanding this reservation for a long time. development of the poor in rural areas. In this scheme,
With reference to the above statements, which all the earlier programs run by the government like-
of the following is true - Women and Child Development Program (DWACRA),
(a) Both (A) and (R) are true and (R) truly I.R.D.P, TRYSEM etc. were included. Jawahar Rozgar
defines (A) Yojana was not included in this Scheme.
(b) Both (A) and (R) are true but (R) is not the 69. As per the 2001-02 budget, maximum central
correct explanation of (A) allocation (Appropriation) ?
(c) (A) is true but (R) is false (a) For energy sector
(d) (A) is false but (R) is true (b) For social development sector
Ans. (c) : It is true that there have been remarkable (c) For transport sector
change in the activities of women panchayat heads as a (d) For rural development
result of reservation, these project bodies are Ans. (*) : According to the 2021-22 budget central
continuously moving towards their objective but, the
sector allocation are as following:
demand for such reservation is the result of the long-
standing demand of the developed social class and not Education – 93224 cr
the demand of rural women. Defence – 478196 cr
65. As per the 1999-2000 report of planning Trandport – 118101 cr
commission, the maximum percentage of Major Subsidy – 369899 cr
people living below the poverty line is in - 70. The National Development Council (NDC) is
(a) Bihar (b) Madhya Pradesh mainly concerned/connected with -
(c) Odissa (d) Uttar Pradesh (a) Approval of Panchayat Plans
Ans. (c) : According to the Planning Commission 1999- (b) Implementation of village development plans
2000, Orissa (47.15%) was the first in India while Bihar (c) Construction of development projects
(42.60%) ranks second in terms of people below the (d) Financial relations of centre & states
poverty line. Ans. (d) : National Development Council was set up on
66. "Golden Hand Shake" term is associated/ August 6, 1952, to strengthen and mobilize the effort
referred to - and resources of the nation in support of the Plan, to
(a) To honour very-very important (VVIP) promote common economic policies in all vital spheres,
persons and to ensure the balanced and rapid development of all
(b) To Voluntary Retirement Scheme parts of the country.
(c) A wish to have a happy sea voyage 71. Statement (A) : All traders earn profit by cost
(d) Welcoming distinguished guests escalation of commodities.
Ans. (b) : A Golden Handshake Scheme is associated Reason (R) : Due to rise in costs, customers
with voluntary retirement. have to cut their wants.
UP UDA/LDA (Pre) Exam 2001 289 YCT
CLICK HERE FOR FREE MATERIAL

In context of the above two statements, which (c) to determine/define the financial resources of
one of the following is true - central government.
(a) Both (A) & (R) are true and (R) correctly (d) to determine the share of states in central
explains (A) grants and revenue collection of the Union
(b) Both (A) & (R) are time but (R) does not government.
properly explains (R) Ans. (d) : Under Article 280 of the Constitution, the
(c) (A) is true but (R) is false Finance Commission is constituted to determine the
(d) (A) is false but (R) is true financial relations between the Center and the States. At
Ans. (b) : Increase in price is a good source of profit for present, the 15th Finance Commission is functioning,
the traders. The increased value is the additional income whose term is 2020 till 2025. Its chairman is NK Singh.
of the traders. On the other hand, the increased price The chairman of the first Finance Commission of India
increases the expenditure of the consumer. To balance was KC Niyogi.
this, the consumer cuts down on his needs. 76. In India, which are included in Tertiary
72. Agmark is - Sector?
(a) A cooperative committee meant for egg I) Trade and Transportation
production II) Finance and Real Estate
(b) A cooperative committee for farmers III) Forestry and Fishery
(c) A regulated market of eggs Choose the correct answer from the given
(d) A guarantee of quality below:-
Ans. (d) : Agmark is a certification mark that assures (a) only I (b) I and II
conformity to set of standards approved by Government (c) II and III (d) only III
agency Directorate of Marketing and Inspection. Ans. (b) : Trade, transport, finance, real estate etc are
It is legally enforced by Agricultural Produce (Grading included under tertiary sector or service sector. Whereas
and Marking) Act of 1937 (amended in 1986). forestry, fisheries and agriculture are included under the
73. Which of the following is not a cause of low primary sector.
productivity of Indian Agriculture? 77. Statement (A) : When a ship enters a sea from
(a) High population pressure. a river, then the ship rises above little
(b) Disguesed unemployment. Reason (R) : The density of sea water is more
(c) Cooperative farming. than density of river water.
(d) Small size of Tilled/Cultivable land. Which of the following conclusions is/are true
Ans. (c) : Co-operative farming is a method of doing in context of above two statements.
agriculture by combining holdings to make agriculture (a) Both (A) and (R) are true and (R) is correct
profitable, whereas population pressure, disguised explanation of (A).
unemployment and small size of land holdings are the (b) Both (A) and (R) are true and (R) is not
reasons for the low productivity of Indian agriculture. correct explanation of (A).
74. Statement (A) : In India self reliance in food (c) (A) is true but (R) is false.
grain (cereal) production has been achieved. (d) (A) is false but (R) is true.
Reason (R) : Now India does not resort to bulk Ans. (a) : The end of the drainage system of the river
import of food grains. takes place after its merger with the ocean. During the
With reference to above statements, which of journey of the ocean from the point of origin, many
the following conclusions is true ? waste materials get mixed in the water of the river,
(a) Both (A) and (R) are true and (R) is the true which after mixing increases the density of the ocean
explanation of (A). water. Due to this reason the ship rises on entering the
(b) Both (A) and (R) are but (R) is not the correct water of the ocean.
explanation of (A). 78. Statement (A) : A black hole is such a celestial
(c) (A) is true but (R) is false. entity which can not be seen through.
(d) (A) is false but (R) is true. Reason (R) : The gravity force on black hole is
Ans. (b) : Presently, India has achieved self reliance in such a strong that it does not allow even light
food grain production. Therefore there is no need to rays to escape.
import food grains is bulk. However the given reason With reference to above statements which of
for the statement is not correct. Here the statement (A) the following is true ?
is a reason for the (R). (a) Both (A) and (R) are true and (R) correctly
75. Generally, Finance Commission in India is set explains (A)
up after every five years - (b) Both (A) and (R) are true and (R) does not
(a) to determine the financial condition of states. explains (A) correctly
(b) to determine the financial condition of central (c) (A) is true but (R) is false.
government. (d) (A) is false but (R) is true.
UP UDA/LDA (Pre) Exam 2001 290 YCT
CLICK HERE FOR FREE MATERIAL

Ans. (a) : After the explosion, due to excessive gravity, 83. Consider the following statements :-
the big stars become very compressed and dense. Their (1) Teflon and Dacron are polymers.
density is very high. Due to its high density, no matter, (2) Neoprene is an Artificial (Synthesised)
even sunlight, can escape from its gravity. This is the Rubber
reason why it is called Black Hole. (3) Polythene and Polyethylene are polymers
79. Two persons standing on Moon's Surface (4) Natural rubber is chloroprene
cannot listen each others voice because - Which of the above statements is/are true ?
(a) Their ears stop working on Moon. (a) 1, 2 and 3 (b) 1, 2 and 4
(b) There is no atmosphere on Moon. (c) 2, 3 and 4 (d) 1, 3 and 4
(c) On moon they wear a specially type of Ans. (a) : Polymerization is a reaction in which more
internal suit. than one molecule of the same type joins together to
(d) On moon, sound moves at a very mild speed. form a larger molecule of higher molecular weight.
Ans. (b) : A medium is required for the propagation of Teflon and Dacron are polymers. Natural rubber is
sound waves. The atmosphere serves as a medium for made from latex derived from rubber trees. Synthetic
propagation of sound waves. There is no atmosphere on rubber is a polymer of isoprene. Neoprene is a synthetic
the surface of the Moon. For this reason two persons on rubber. Polythene is obtained as a result of
the Moon cannot hear each other. polymerization of ethylene at high temperature and high
80. Which one among the following is not an pressure. It is used in making toys, bottles, buckets,
explosive ? foils etc. It is a polymer of polyethylene.
(a) Trinitrotoluene (TNT) 84. Which of the following are used in the form of
(b) Trinitroglycerine Bio-fertilisers -
(c) Cyclo Trimethylene Trinitramine 1) Azolla 2) Blue-green Algae
(d) Nitrochloroform 3) Alfa-Alfa 4) Nitrolim
Ans. (d) : Nitrochloroform is a chemical compound used Choose the correct answer from the following :
as a broad spectrum antimicrobial, fungicide herbicide etc. (a) 2 and 4 (b) 1, 2 and 4
It is also used as tear gas. It is not an explosive. (c) 2, 3 & 4 (d) 1, 2 & 3
81. Which of the following pairs is not matched Ans. (d) : Biofertilizer refers to such micro-living
correctly ? bacteria, which provide nutrients for the use of plants.
(a) Fullerenes - A carbonic compound containing In fact, the most important function of biofertilizers is in
fluorine. terms of nitrogen availability, because it absorbs free
(b) Dry Ice - Solid Carbon dioxide. nitrogen from the atmosphere to form the nutrient
(c) Keratin - A protein which is found on the material that plants use for their growth. The
epidermis of human skin. components of organic fertilizers are Rhizobium,
(d) Mustard Gas - A poisonous liquid used in Azolla, Azospirillum, Lipopherum Mycorrhiza, etc. The
chemical war. two major biofertilizers used in India are Rhizobium
and blue green algae (including Azolla). It is also used
Ans. (a) : Fullerenes, graphite, diamond, etc. are
as a biofertilizer Nitrolim is a chemical fertilizer.
allotropes of carbon, so option (a) is not correctly
Alfalfa is also known as Lucerne.
matched. All other options are correctly matched.
85. Consider the following effects of Genetic
82. Which of the following pairs is not matched
Engineering :-
correctly.
(a) Discovery of Mesons- Hideki Yukawa 1) Resistance to Diseases
(b) Discovery of Positrons - C.D. Anderson and 2) Swelling/growth escalation
U.F. Haines 3) Animal Cloning
(c) Theory of Energy production in sun & stars - 4) Human Cloning
H.A. Bethe Which of the above have been tested
(d) Synthesis of trans uranium elements - Enrico successfully ?
Fermi (a) 1, 3 and 4 (b) 2, 3 & 4
Ans. (d) : The basic particles called pi-mesons were (c) 1, 2 and 4 (d) 1, 2 & 3
discovered by Hideki Yukawa in 1935. Positron is a Ans. (d) : Except for Human Cloning, the remaining
positively charged fundamental particle, whose mass three have been successfully tested.
and charge are equal to that of the electron, hence it is 86. What is Arthoplasty technique used in surgery?
also called the antiparticle of the electron. It was (a) Open Heart Surgery
discovered by Anderson in 1932. The source of energy
(b) Kidney transplantation
of the Sun and other stars in the universe is the nuclear
fusion that takes place there. In 1938, American (c) Hip joint Replacement
scientists 'Bethe' told that there are two types of fusion (d) Blood Transfusion
reaction on the Sun, the proton cycle and the carbon- Ans. (c) : Arthoplasty is a surgical procedure to restore
nitrogen cycle. Transuranium element, for the first time, the function of a joint such as Hip joint replacement and
synthesised by E.M. Mcmillan & P.H. Abelson. Knee joint replacement.
UP UDA/LDA (Pre) Exam 2001 291 YCT
CLICK HERE FOR FREE MATERIAL

87. Match List-I with List-II and choose the (a) Both (A) & (R) are correct and (R) is the true
correct answer from the codes given below :- explanation of (A).
List-I List-II (b) Both (A) & (R) are correct and (R) is not the
(Radio-isotopes) (Diagnostic Usage) true explanation of (A).
(a) Arsenic-74 1. Hyper thyroidism (c) (A) is correct but (R) is false.
(b) Cobalt-60 2. Blood Abnormality (d) (A) is false but (R) is true.
(c) Iodine-131 3. Tumour Ans. (a) : When a spacecraft enters the Earth's
atmosphere, the surface temperature of the vehicle
(d) Sodium-24 4. Cancer increases, resulting in a cover of ionized air around it.
Code : As a result the shuttle loses radio contact with the
A B C D ground station for some time.
(a) 1 2 3 4 91. Match List-I with List-II and choose the
(b) 4 3 1 2 correct answer from the codes given below :-
(c) 3 4 1 2 List-I List-II
(d) 4 3 2 1 (A) Trishul 1. A antitank missile
Ans. (c) : (B) Prithvi 2. A medium range missile
List-I List-II (C) Agni 3. A short range surface to
(Radio-isotopes) (Diagnostic Usage) air missile
(a) Arsenic-74 1. Tumour (D) Naag 4. A surface to surface
(b) Cobalt-60 2. Cancer missile
(c) Iodine-131 3. Hyper thyroidism Code :
(d) Sodium-24 4. Blood Abnormality A B C D
(a) 1 2 3 4
88. Which of the following harmone is known as
(b) 4 3 2 1
fight or flight harmone ?
(a) Insulin (b) Adrenaline (c) 3 4 2 1
(d) 2 1 4 3
(c) Estrogen (d) Oxytocin
Ans. (c) : Correct answer of the following
Ans. (b) : The adrenaline hormone is secreted from the
medulla. This hormone helps the body to react more List-I List-II
quickly in times of crisis and special circumstances. (A) Trishul 1. A short range surface to air
This hormone is called 'fight or flight' hormone due to missile
its excessive secretion in situations like anger, fear, (B) Prithvi 2. A surface to surface missile
stress, anxiety etc. (C) Agni 3. A medium range missile
89. Kalaripayattu is a - (D) Naag 4. A antitank missile
(1) A martial arts system for increasing
92. Consider the following statements :-
physical capacity.
1. Fibre optics is based upon the principle of
(2) A process for curing Arthritis and bone
total internal reflection.
related disorders.
2. In optical fibre based communication system,
(3) A form of painting famous during Ancient
the energy consumption is very low.
India.
3. Optical fibre communication is free from
(4) Name of a tribal dance.
radio frequency obstruction.
Choose the correct answer from the codes
4. In India Reliance Industry group is
given below :- associated/engaged in the production of
(a) 1 and 2 (b) 1 & 3 optical fibres.
(c) 2 & 3 (d) 1 & 4 Which of the above statement is/are true.
Ans. (a) : Kalaripayattu is a martial art which is a (a) 1, 2 and 3 (b) 1, 2 and 4
martial art system to increase physical stamina. It also (c) 1, 3 and 4 (d) 2, 3 and 4
helps in curing rheumatic bone diseases like Arthritis.
Ans. (a) : 'Optical Fiber Communication System'. It has
90. Statement (A) : A space shuttle while landing some major features. (i) The speed of communication is
on earth loses radio contact with the earth very fast in this medium, (ii) their capacity is very high
station for some moments. (iii) very little energy is required for their operation. (iv)
Reason (R) : When a space shuttle enters Optical fiber communication is free from radio
earth's atmosphere then the temperature of frequency interference. In India various companies are
shuttle external layer increases and the involved in the production of optical fibers such as
surrounding Air is ionised. Sterlite Technology Ltd., Birla cabes, finolex cables etc.
With reference to the above two statements However Reliance Industries is not associated with
which is correct ? optical fiber production yet.
UP UDA/LDA (Pre) Exam 2001 292 YCT
CLICK HERE FOR FREE MATERIAL

93. For which achievement the Global-500 award Ans. (d) : Banthra Plant Gene Bank will protect the
presented ? endangered plant species and prevent piracy of
(a) Population control biological diversity. It will also identify economically
(b) Drive against terrorism important plants. At the same time, those species,
(c) Environment Protection whose existence is threatened and which are also
(d) A Campaign against Narcotic Substances commercially important, will also be protected.
Ans. (c) : The Global 500 Award is given by the United 97. The highest trophic level in the ecosystem has
Nations Environment Program for its outstanding been gained by -
contribution in the field of environmental protection and (a) Herbivores (b) Carnivores
improvement. (c) Omnivores (d) Decomposers
94. Consider the following statements related to Ans. (c) : Omnivores constitute the highest trophic level
Ecosystem - in an ecosystem. Omnivores are associated with various
(1) The word ecosystem, was first used by A.G. tropic level in the ecosystem which eat Herbivores and
Tansley. Omnivores.
(2) Those organisms/animals which produce their 98. With relation to bionomics which of the
own food are known as Antotrophs. following statements is true :-
(3) Consumers consume their food by the photo- 1) Its literal meaning is "management of life".
synthesis process.
2) Its is synonymous to Ecosystem.
(4) Decomposers change inorganic substances
3) It places emphasis on the value of Natural
into organic substances.
System which impact human systems.
Which of the above statements are correct -
(a) only 1 (b) 1 and 2
(a) 1 and 2 (b) 1 and 3
(c) 2 and 3 (d) 1, 2 and 3
(c) 2 and 4 (d) 3 and 4
Ans. (c) : Bionomics is the study of an organism and its
Ans. (a) : The word ecosystem was first used in print by
relation with its environment. The word Bionomics is
A.G. Tansely (1935) in his known paper on vegitational
made up of words bio and (nomos = law). It is
concept and terms. Hence statement 1 is correct.
synonymous with ecology. At the same time, it also
An Autotroph is an organism that can produce its own
emphasizes the values of natural systems, which affect
food using light, CO2 and other chemicals. Hence
human systems, so statements 2 and 3 are true.
statement 2 is also correct.
Consumers consume their food by eating and digesting and 99. Highest Biodiversity is found in ?
not by photosynthesis. Hence 3rd statement is incorrect. (a) Silent Valley (b) Kashmir Valley
Decomposers convert organic substances into inorganic (c) Valley of flowers (d) Surma Valley
substances. Hence 4th statement is also incorrect. Ans. (a) : The region with the highest biodiversity, the
95. As a consequent of Global warming - Silent Valley, is located in Kerala. It comes under the
1) Communication system have improved in Palakkad district of Kerala. It is situated at the heart of
world. the Nilgiri hills. It is one among the world's ten hottest
2) Glaciers started Melting. Bio-diversity hot spots.
3) Mango Trees started blossoming earlier. 100. Which of the following gas is responsible for
4) Health has been affected adversely. generation of Acid rain from the atmosphere ?
Which of the following statements is/are true ? (a) CFC (b) CH4
(a) 1, 2 and 3 (b) 2, 3 and 4 (c) O3 (d) SO2
(c) 1, 2 and 4 (d) 1, 3 and 4 Ans. (d) : The major factor for acid rain is Sulfur
Ans. (b) : Global warming is the long term heating of Dioxide. (SO2) and Nitrogen dioxide. These gases are
Earth's surface, primarily due to human activities. There result of the burning of oils and coal etc. Sulfur dioxide
are many consequences of global warming such as - (SO2) reacts with the humidity of the atmosphere to
• Melting of glacial ice. form sulfuric acid (H2SO4) and nitrogen dioxide (NO2),
• Rise in the sea level. nitric acid (HNO3). The famous Taj Mahal of Agra is
turning black due to acid rain. The countries most
• Various health issues such as skin diseases, stress etc.
affected by acid rain are Sweden, Norway and America
• Early blossoming of plants such as mango. respectively.
• Severe loss to marine life etc. 101. Below the List-I contains names of persons who
96. The plant field gene bank at Banthara - have been awarded the Academy Ratna Prize
(a) will preserve the critically endangered plant by the Uttar Pradesh Natya Academy whereas
species categories. List-II mentions the fields in which they have
(b) check piracy of bio-diversity. attained excellence. Match both the lists and
(c) economically important wild plants. choose the correct answer from the codes given
(d) will execute all these functions. below.
UP UDA/LDA (Pre) Exam 2001 293 YCT
CLICK HERE FOR FREE MATERIAL

List-I List-II Ans. (b) : Given that –


1. Ustad Vilayat Khan A) Sarod
2. Master Fida B) Kathak
Hussain Narsi
3. Sitara Devi C) Hindi Theatre
The radius of semicircle = r
4. Jatin Bhattacharya D) Sitar
1
Code : The Area of ∆ABC = × OB × AC
(a) 1B, 2A, 3C, 4D (b) 1D, 2A, 3C, 4B 2
(c) 1C, 2B, 3D, 4A (d) 1D, 2C, 3B, 4A 1
= × r × 2r
Ans. (d) : The correct match is as follows - 2
List-I List-II = r2 cm2
(Artist) (field) 105. Which of the following options is more than
Ustad Vilayat Khan Sitar 1/4?
Master Fida Hussain Narsi Hindi Theatre 1
Sitara Devi Kathak (a) (0.25)2 (b)
4
Jatin Bhattacharya Sarod
1
102. 'Taleem' is a plan prepared/devised by U.P. (c) 0.04 (d)
25
Government which has been designed to -
1) provide education in Rural areas via 1
Ans. (b) : ∵ = 0.25
Doordarshan. 4
2) provide computer education to the students 1
of universities & degree colleges. (a) (0.25)2 = 0.0625 <
4
3) to impart training to selected candidates in
sports. 1 1 1
(b) = >
4) to acuainst all primary level students with 4 2 4
the Urdu language. 4 1 1
(a) 1 and 4 (b) 2 and 3 (c) 0.04 = = <
100 25 4
(c) 3 and 4 (d) 2 and 4 1 1
Ans. (a) : The Taleem scheme was started with the joint(d) = 0.04 <
25 4
effort of Uttar Pradesh government and Doordarshan. Hence, option (b) is correct answer.
Under this scheme, educational activities are organized
in rural areas to introduce Urdu language to elementary106. Amar, Akbar and Antony are friends who are
level children. look after by Matron Farah. The weight at
Amar is 50% more than that of Akbar and
103. At some definite time two sportsmen/players
Antony's weight, is 25% less than Amar.
are standing in a playground. The Cartesian rd
coordinates of their locations are (20, 60) and 1
Weight of Farah is of the total weight of
(–40, –20). What is the distance between them - 3
(a) 60 units (b) 80 units these three boys. The total weight of these 4
(c) 100 units (d) 140 units persons is 232 kg.
Ans. (c) : Given that What is the correct ascending order of their
The Cartesian Co-ordinates of location of two players weights ?
are (20, 60) and (–40, –20). (a) Antony, Akbar, Farah, Amar
⇒ (x1, y1) = (20, 60) (b) Antony, Akbar, Amar, Farah
(x2, y2) = (–40, –20) (c) Akbar, Antony, Farah, Amar
(d) Akbar, Antony, Amar, Farah
∴ Distance = ( x 2 − x1 ) + (y 2 − y1 ) 2
2
Ans. (c) : Let the weight of Akbar = x kg.
= ( −40 − 20 ) + ( −20 − 60 )
2 2 x × 50 3x
Weight of Amar = x + =
100 2
= 3600 + 6400
3x 3x 25
= 10000 = 100 units Weight of Antony = − ×
2 2 100
104. The area of the largest triangle constructed in a 3x 3x 12x − 3x 9x
semicircle of radius r is ? = − = =
2 2 2 2 2 8 8 8
(a) 2r cm (b) r cm
3x 9x 8x + 12x + 9x
r2 Total weight of all three = x + + =
(c) 4r2 cm2 (d) cm 2 3 8 8
2
UP UDA/LDA (Pre) Exam 2001 294 YCT
CLICK HERE FOR FREE MATERIAL

29x Ans. (b) : Just as,


= kg
8
8 29x
Thus weight of Farah = = kg
3 24
Combined weight of all the fair
29x 29x 87 + 29 116x Same as,
= + = x=
8 24 24 24
116x
According to question - = 232kg
24
24 1392
So, x = 232 × = = 48 kg 111. In the line diagram shown above, rectangle
116 29
Hence, figure denotes women, triangle denotes Police
sub-inspectors and circle denotes the
Akbar's weight = 48 kg Graduates. Which numbered area/region
3x 48 × 3 denotes those women sub inspectors which are
Amar's weight = = = 72 kg
2 2 not Graduates -
9x 29 × 48
Antony's weight = = = 54kg
24 24
29x 29 × 48
Farah's weight = = = 58kg
24 24
(a) 5 (b) 9
Hence, required answer – 48, 54, 58, 72
(c) 8 (d) 3
= Akbar, Antony, Farah, Amar
Ans. (b) :
107. Suchitra is taller than Pushpa but Shorter than
Malti and Geeta is shorter than Viji. Viji is not
as tall as Pushpa. If all of them are standing in
▭ → Women
order of their heights then which woman is
standing in the middle ?
∆ → Police sub Inspectors

(a) Malti (b) Suchita ○


→ Graduate
(c) Pushpa After studying the picture it is clear that the field of
(d) Viji
digit '9' represents those female sub inspectors who are
Ans. (c) : The standing order of the girls given in the
not graduates.
question according to their height is as
follows – 112. Consider the following statements related to
National Commission for the review of
Malti > Suchitra > Puspa > Viji > Geeta
constitution.
108. What is the value of x in the number sequence -1) The nature of the report of this commission
5, 8, 13, x, 34, 55, 89 is recommendatory.
(a) 20 (b) 21 2) It has been appointed by the approval of
(c) 23 (d) 29 Parliament.
3) It is being presided by Justice M.N
Ans. (b): Each next number in the given number sequence
is the sum of the previous two consecutive number. Venkatachaliah.
109. What is the next number in the number series - 4) It will focus on the socio-economic needs of
11, 121, 1331 ..... the country.
(a) 1441 (b) 14411 Which of the above statements is/are true ?
(c) 14441 (d) 14641 (a) 1 & 2 (b) 1 & 3
(c) 1, 2 & 3 (d) 2, 3 & 4
Ans. (d) : The next number in the number series is
given below. Ans. (b) : The National Commission to review the
working of the constitution (NCRWC) was established
11 121 1331 14641 in the year 2000. Its report was recommendatory in
↓ ↓ ↓ ↓ nature. The commission was headed by Justice M.N.
Venkatachaliah.
11 11× 11 11× 11×11 (11×11×11× 11) 113. As per the economic survey of year 2000-01,
110. If word 'VICTORY' is coded/indicated by the economic growth of India is -
'YLFWRUB' then. How will the word (a) 5.1% (b) 5.5%
FAILURE be coded (c) 6.0% (d) 6.5%
(a) JELOXUH (b) IDLOXUH Ans. (c) : As per the economic survey of year 2000-01
(c) JDLKWUH (d) IDOLKUH the economic growth of India was 6%.
UP UDA/LDA (Pre) Exam 2001 295 YCT
CLICK HERE FOR FREE MATERIAL

114. As per the 2001-02 Annual Union Budget 2) For their outstanding achievement in
India's current fiscal deficit is - sports.
(a) 4.7% (b) 5.1% 3) For exemplary courage & bravery of
(c) 5.5% (d) 6.1% women.
Ans. (a) : As per the 2001-02 annual union Budget- 4) For their contribution towards Nation &
India's fiscal deficit was 4.7%. People.
Choose the correct answer from the codes
115. Which of the following persons were conferred. given below :-
Jnanpith Award 1999. Choose the correct
Codes:-
answer from the codes given below :
(a) 1 & 2 (b) 2 & 3
1) Nirmal Verma 2) Arundhati Roy
(c) 3 & 4 (d) 1 & 4
3) Gurdial Singh 4) Sardar Jafri
Ans. (c) : Stree Shakti Puraskar (Women Power Award)
Codes :
was established in 1999. It is awarded by the Ministry
(a) 1 and 2 (b) 2 and 3 of Women and Child Development (Government of
(c) 1 and 3 (d) 3 and 4 India) to those women who have set an example of
Ans. (c) : The Jnanpith Award 1999 was awarded to courage in their personal or professional life. It is
Shri Gurdial Singh and Shri Nirmal Verma. awarded on 8th March every year, in which one lakh
116. In U.S. Hopman Cup-2000, who was the winner rupees in cash and a citation are given.
of Men's Singles Title - 120. Who won the Legend Cup - 2000 ?
(a) Mahesh Bhupati (b) Leander Paes (a) Byorn Borg (b) Pat Cash
(c) Roger Federer (d) Michael Gambit (c) Henri Leconte (d) Vijay Amritraj
Ans. (c) : U.S. Hopman Cup-2000 was won by Rogar Ans. (b) : The winner of the Legend Cup 2000 was Pat
Federer. Cash. Currently (2019) the winner of this cup is J.P.
Morgan.
117. Two Indian Business Houses, who are included
in the 50 richest persons of the world as per the 121. In the following Table the number of women
'Forbes billionaires list' are ? per 1000 males of World & India are given by
1) Ratan Tata the end of year 2000. Which of the following
pair is correctly matched ?
2) Azim Premji
India World
3) Kumar Mangalam (Birla)
(a) 897 960
4) Dhirubhai Ambani (b) 907 870
Choose the correct answer from the codes (c) 917 980
given below :-
(d) 927 990
(a) 1 & 2 (b) 2 & 3
Ans. (d) : At the end of the year 2000, the number of
(c) 3 & 4 (d) 2 & 4
women per 1000 men in India and the World were 927
Ans. (d) : irrelevant question. and 990 respectively, whereas at present (2021-22) the
118. Statement (A) The Union Power Minister said number of women per thousand men in India and the
that for the failure of Electricity grid 'Greed' is world were 943 and 982 respectively.
as much responsible as Technical reasons. 122. According to the report by CAG published in
Reason (R) : The power minister's pointed the year 2000 regarding P.D.S, the increase in
presumably towards the exaggerated demands the grant provided on food grains from year
put forth by the states. 1992-93 to 1998-99 is about -
Which of the following is true ? (a) more than two times
(a) Both (A) & (R) are true and (R) is the correct (b) more than three times
explanation of (A). (c) more than four times
(b) Both (A) & (R) are true but (R) is not the (d) more than five times
correct explanation of (A). Ans. (b) : Public Distribution System (PDS) is an
(c) (A) is true but (R) is false. Indian food security system, which was implemented in
(d) (A) is false but (R) is true. 1997 by the Government of India through the Ministry
Ans. (a) : The Union Power Minister said that technical of Consumer Affairs, Food and Public Distribution. It
reasons are responsible for the failure of the power grid, aims to distribute subsidized food and non-food items to
no doubt, but greed is equally responsible. Indirectly, the poor in the country jointly with the state
the minister said that the states are exaggerating their governments. PDS According to the report of the
real demands. Controller of Accounts and Examination (CAG)
119. Women are granted 'Stri Shakti Award' for the published in the year 2000, there has been an increase
following :- of more than three times in the grant given on food
1) For their excellence in Atheletics. grains in the year 1998-99 from 1992-93.
UP UDA/LDA (Pre) Exam 2001 296 YCT
CLICK HERE FOR FREE MATERIAL

123. "Bahujan Hitay, Bahujan Sukhai" is an ideal Ans. (c) : According to the World Development Report
of - 2000, the country with the highest per capita income
(a) All India Radio was Luxembourg. The country with the current (2020-
(b) Doordarshan 21) highest per capita income is Qatar.
(c) Life Insurance Corporation of India. 128. Recently which two countries signed on the
(d) General Insurance Corporation of India. Mekong-Ganga Project meant to augment
Ans. (a) : All India Radio was established in the year cultural and economic cooperation ?
1930. Its motto is 'Bahujan Hitay, Bahujan Sukhay'. It is (a) India - Cambodia
working under the Ministry of Information and (b) India - Vietnam
Broadcasting, Government of India. Its headquarter is (c) Bangladesh - Vietnam
located in New Delhi.
(d) India - Myammar
124. Vishwanathan Anand became world chess
Ans. (b) : Mekong-Ganga Project built for mutual
champion after defeating whom ?
cooperation of 6 countries. It was established on 10
(a) Anatoli Karpov November 2000 in Vietnam. This organization includes
(b) Alexei Shirov India, Myanmar, Thailand, Cambodia, Laos and
(c) Gary Kasparov Vietnam. The purpose of establishing this organization
(d) Evgeny IIgizovich Bareev is to cooperate with each other for tourism, culture,
Ans. (b) : Viswanathan Anand became the world education and transportation. When the question was
champion in 2000 by defeating Alexei Shirov. Magnus asked, India and Vietnam (2nd MGC ministerial
Carlsen became the Chess World Champion in 2013 meeting) signed Mekong-Ganga project for cultural &
after defeating Viswanathan Anand. After this, Carlsen economic cooperation.
won the championship in 2014, 2016 and 2018 129. 'Stephen Hawkings' who gave the Albert
consecutively. Magnus Carlsen is currently the world
Einstein lecture in Delhi on February 2001-
champion of chess.
1) Travel on a motorised wheel chair.
125. A voluntary organisation called "Tarun Bharat
2) Use a walk synthesiser.
Sangh" is working in which of the following
states ? 3) Keep a portable computer with him.
(a) Gujarat (b) Karnataka 4) is a Renowned Australian cosmologist.
(c) Madhya Pradesh (d) Rajasthan Which of the above statements is/are true ?
Ans. (d) : Tarun Bharat Sangh is a non-governmental (a) 1 & 2 (b) 2 & 3
organization. It was established on 30 May 1975. This (c) 1, 2 & 3 (d) 2, 3 & 4
organization has been working on the issues of Ans. (c) : Stephen Hawking is a famous British
conservation of forest and water for more than four astrophysicist who is acknowledged as the greatest
decades. This association was founded by Dr. Rajendra physicist after Einstein. He uses a motorised wheelchair
Singh who is currently its president. Presently the office and a speech synthesizer and carry a portable computer
of this organization is located in Alwar district of with him.
Rajasthan. The aim of this association is to work for the
130. Which among the following country has
holistic development of men, women and children
choosen the Gross National Happiness as an
irrespective of any economic status, caste and religion.
indicator of progress ?
126. In Dec, 2000 a news was published that Delhi (a) Bhutan (b) Japan
High Court has allotted non-judicial task to
Nine judges of Delhi because - (c) Sweden (d) Switzerland
(a) a lot of non-judicial work was accumulated in Ans. (a) : Gross National Happiness is a philosophy
Delhi. adopted by the Government of Bhutan, which was
(b) The above referred judges were found very implemented by the Government of Bhutan in 1970.
efficient in executive work. 131. Which of the following nations is not a member
(c) They were expert specically in public relations of Shanghai-5 group ?
(d) They were indicted with allegations of misuse (a) China (b) Kazakhstan
of their rights. (c) Russia (d) Vietnam
Ans. (a) : Due to the accumulation of a lot of extra- Ans. (d) : SCO is comprised of Eight Member
judicial work in Delhi, the Delhi High Court appointed countries. Namely-
its nine judges to deal with these works. As a result, the
Kazakhstan, China, Kyrgyzstan, Russia, Tajikistan,
burden of extra-judicial work in Delhi could be reduced
to a great extent. Uzbekistan, India, Pakistan
127. According to the World Development Report 132. Which of the following Nation is not a member
2000 the highest percapita income is of ? of OPEC ?
(a) Switzerland (b) Norway (a) Algeria (b) China
(c) Luxembourg (d) Japan (c) Indonesia (d) U.A.E
UP UDA/LDA (Pre) Exam 2001 297 YCT
CLICK HERE FOR FREE MATERIAL

Ans. (b) : The Organization of the Petroleum Exporting Ans. (c) :


Countries (OPEC) was founded in Baghdad, Iraq, with Person Prize
the signing of an agreement in September 1960 by five Bhupen Hazarika Lata Mangeshkar Award
countries namely Islamic Republic of Iran, Iraq, Sitakant Mohapatra All India Kabir Samman
Kuwait, Saudi Arabia and Venezuela. They were to Bhishma Sahani Maithilisharan Gupt Samman
become the Founder Members of the Organization. Daler Mehandi Sikh Millennium Award
These countries were later joined by Qatar (1961), 136. Who was the American Journalist who was all
Indonesia (1962), Libya (1962), the United Arab alongwith Gandhi during Quit India
Emirates (1967), Algeria (1969), Nigeria (1971), Movement ?
Ecuador (1973), Gabon (1975), Angola (2007), (a) William L. Shirer (b) Louis Fischer
Equatorial Guinea (2017) and Congo (2018). (c) Web Miller (d) Negley Farson
133. Statement (A) : The NDA government does not Ans. (b) : American journalist Louis Fischer was with
like any dispute regarding Rule 184 of Lok Gandhiji during the Quit India Movement. Webb Miller
Sabha. was present in India at the time of Civil Disobedience
Reason (R) : In this Rule, there is a provision of and H. Nevinson was present during Swadeshi
voting along with Debate. Movement.
(a) Both (A) & (R) are true and (R) is the correct 137. The Sea-Bird Project of Indian navy is related
explanation of (A). to -
(a) A ship mounted missile development which
(b) Both (A) & (R) are true and (R) is the correct
can strike any target in the open Sea's.
explanation of (A).
(b) Construction of aircraft carrier ships.
(c) (A) is true but (R) is false.
(c) the development of ultra fast Torpedo cruiser
(d) (A) is false but (R) is true. boats.
Ans. (a) : Under rule 184 of Parliament Procedure Rule, (d) Shifting of a large chunk of Mumbais Naval
Lok Sabha there is a provision for division of votes after fleet to Karwar naval base.
Parliamentary debates. The NDA government does not Ans. (d) : It is a naval infrastructure project which
like any dispute regarding Rule 184 of Lok Sabha. entails the creation of a naval base at Karwar
Hence both (A) & (B) are true and (R) is the correct (Karnataka) on the west coast of India.
explanation of (A). 138. The winner of A.T.F Asia Tennis Cup 2000 was
134. Zhores Alferov, H. Kroemer & Jack Kilby (a) Leander (b) Daine Udanchok
were awarded Nobel Prize for the year 2000 in (c) Hyung-Taik (d) Seung Itun Lee
which discipline ? Ans. (b) : Dane Udan Chok is the winner of Asia
(a) Medicine (b) Physics Tennis Cup 2000. Currently (2019) winner of this cup is
(c) Chemistry (d) Botany Taro Daniels(Japan)
Ans. (b) : Zhors Alferov, H. Kroement & Jack Kilby 139. U.S. Hopman Cup-2000 women's singles tittle
were awarded Noble Prize for the year 2000 in Physics. was won by -
Nobel Prize awarded in Physics for the year 2022 - (a) Monica Seles (b) Martina Hingis
(a) Alain Aspect (c) Anna Karnikov (d) Jelena Djokic
(2) John F Clauser Ans. (b) : U.S. Hopman Cup-2000 women's singles title
was won by Martina Hingis.
(3) Anton Zeilinger
140. Dialogue between Kabir & Dharmdas has been
135. The awarded mentioned in column 2 were compiled under the tittle of
given to persons whose names are listed in (a) Sabad (b) Amarmool
Column 1. Match these pair with the correct (c) Sakhi (d) Rameiny
code given below
Ans. (b) : A compilation of philosophical dialogues and
Column-I Column-II arguments between Kabir and Dharamdas has been
1. Bhupen - Sikh Millennium compiled under the title 'Amarmool'. The remaining
Hazarika Award three 'Sakhi', 'Sabad' and Ramani' are the compositions
2. Sitakant - Lata Mangeskar of Kabir.
Mahapatra Award 141. The world's first floating Nuclear Power
station is under construction and is stepulated
3. Bhisham Sahni - All India Kabir
to be ready for operation by year 2001. This
Samman project has been initiated by which nation ?
4. Daler Mehndi - Mathilisharan (a) U.S.A (b) United Kingdom
Gupt Samman (c) Russia (d) France
(a) 1A, 2B, 3C, 4D (b) 1D, 2C, 3B, 4A Ans. (c) : The world's first floating nuclear power plant
(c) 1B, 2C, 3D,4A (d) 1C, 2D, 3A, 4C was installed by Russia in the year 2001.
UP UDA/LDA (Pre) Exam 2001 298 YCT
CLICK HERE FOR FREE MATERIAL

142. Arti Saha was the first Indian woman to swim 146. 'Losoong' is festival which is celebrated in ?
across the English Channel. Who was the first (a) Tibet
woman in the world who crossed the English (b) Arunachal Pradesh
Channel inspite of being paralysed below the (c) Sikkim
waist ? (d) Kerala
(a) Geeta Chand (b) Bhanu Athayya
Ans. (c) : Lossong festival is celebrated by Bhotia
(c) Tarjani Vakil (d) C.N Janaki
community of Sikkim.
Ans. (d) : C.N. Janaki was the first women in the world
147. Out of the following which nation celebrates
who crossed the English Channel inspite of being
the last Saturday of every month as National
paralysed below the waist.
Sanitation day ?
143. Match the following :- (a) France (b) Italy
List-I List-II (c) Sierre Leone (d) Singapore
(Books name) (Author)
Ans. (a) : National Sanitation Day is celebrated by the
(a) Price of Partition 1. Abul Kalam government of France on the last Saturday of every month.
(b) Anand Math 2. S.S Gill 148. Which of the following statements is not true ?
(c) India-2020 3. Rafiq Zakaria (a) Neptal is the only Hindu state in the world.
(d) Pathology of 4. Bankim Chandra (b) Cow slaughter is prohibited in Nepal.
Corruption Chatterjee (c) In Nepal, Indian Rupee is not a legal tender.
Choose the correct answer from the codes (d) Nepal king is among the three very important
given below : persons (VIPs) who are permitted to enter the
Code : Sanctom Sanctorum of Rameshwaram temple.
A B C D Ans. (c) : Nepal is the only Hindu Kingdom of the
(a) 3 4 2 1 world with the constitutional monarchy. Cow slaughter
(b) 4 2 3 1 is prohibited in Nepal. The King of Nepal is one of the
(c) 3 4 1 2 three special persons who are allowed to enter the
(d) 4 3 2 1 sanctum sanctorum of the Rameshwaram temple. Indian
Ans. (c) : Correct match the following . currency Rupee is the legal currency in Nepal. Its
List-I List-II capital is Kathmandu, where SAARC is headquartered.
(Books) (Author) 149. Which is the oldest Musical instrument ?
Price of Partition Rafiq Zakaria (a) Sitar (b) Veena
Anand Math Bankim Chandra Chatterjee (c) Sarod (d) Tabla
India-2020 Abul Kalam Ans. (b) : In ancient Vedic literature, Vagdevi
Pathology of Corruption S.S. Gill Saraswati, Devarshi Narad, Nagavanshi ruler Udayana
144. In whose memory 28 February is celebrated as in the historical era, Gupta ruler Samudragupta as a
National Science Day every year ? veena player have been mentioned in various historical
(a) Raman-Effect Day evidences. On this basis the Veena can be called the
(b) Launch of 1st Artificial Satellite Arya bhatt oldest musical instrument.
(c) Successful Launch of A.S.L.V. 150. Which of the following Academy is responsible
(d) Birthday of Homi J. Bhabha for encouraging the growth of Dance, drama
Ans. (a) : 28 February was the birth anniversary of Sir and music in India ?
Chandrasekhar Venkataraman, the inventor of the (a) Sangeet Academy
Raman-effect, which is celebrated every year as (b) Lalit Kala Academy
National Science Day. (c) Sahitya Academy
145. The story of the book "Proudest Day" is (d) National School of Drama
related with - Ans. (a) : Sangeet Natak Akademi is the national
(a) Unification of Indian states institution of music, dance and drama. It was
(b) India's freedom established on 1953. Its objective is to promote the
(c) Pokharan Nuclear Explosion performing arts in collaboration with states and
(d) Formation of NDA government at centre voluntary organizations. It supports the publication of
Ans. (b) : The Proudest Day is authored by Anthony books in Indian languages and English on music, dance
Reed and David Fischer. The story of this book is and acting, and awards and fellowships to eminent
related to the freedom struggle of India. artists and scholars.
UP UDA/LDA (Pre) Exam 2001 299 YCT
CLICK HERE FOR FREE MATERIAL

UTTARAKHAND UDA/LDA (Pre) Exam-2003


GENERAL STUDIES
Solved Paper
1. Indus Valley Civilization is known for– 3. The reforms brought by Shershah Suri
1. Its town planning included–
2. The Mohanjodaro and Harappa sites 1. Revenue Reforms
3. Its Agricultural practices 2. Administrative Reforms
4. Its Industries 3. Military Reforms
Choose the correct answer from the codes given 4. Reforms in the currency system
below– Choose the correct answer using the codes given
Codes: below–
(a) 1 and 2 (b) 1, 2 and 3 (a) 1 and 2 (b) 1, 2 and 3
(c) 2, 3 and 4 (d) All of them (c) 2, 3 and 4 (d) All of them
Ans. (d): Indus valley civilization is known for its town Ans. (d) : Sher Shah Suri eventually challenged and
planning and water drainage system. Mohenjodaro, defeated the Mughal emperor Humayun. Sher Shah
Harappa and Lothal are major cities of Indus Valley captured Delhi and established his own dynasty. Sher
Civilization. Generally, houses were either one or two Shah Suri introduced an administration that borrowed
storeys high, with rooms built around a courtyard. Most elements from Alauddin Khalji and made them more
houses had a separate bathing area and some had wells efficient. During his 15 years regime (1540-1555), Sher
to supply water. Many of these cities had covered Shah Suri introduced effective monetary system like
drains. There were men and women, crafts persons, silver coin, which was called Rupiya. Sher Shah was the
making all kinds of things, either in their own homes or first Muslim ruler who got the whole of the land
in special workshops. People were travelling to distant measured and fixed the land-tax on it on just and fair
lands or returning with raw materials. Most of the things principles. He divided his empire into separate units
that have been found by archaeologists are made of called Sarkars (Districts). He appointed Shiqdar
stone, shell and metal, including copper, bronze, gold (Military officer). Sher Shah Suri introduced first postal
and silver. Copper and bronze were used to make tools, system in India. Sher Shah’s administration became the
weapons, ornaments and vessels. Gold and silver were model followed by the great emperor Akbar, when he
used to make ornaments and vessels. The Harappans consolidated the Mughal Empire.
also made seals out of stone. These are generally 4. Akbar's administrative rule was known–
rectangular and usually have an animal carved on them.
While many people lived in the cities, others living in 1. For conquering various regions
the countryside grew crops and reared animals. These 2. For its administrative system
farmers and herders supplied food to crafts persons, 3. For its judicial administration
scribes and rulers in the cities. Harappans grew wheat, 4. For its religious fanaticism
barley, pulses, peas, rice, sesame, linseed and mustard. Choose the correct answer from the codes given
A new tool, the plough, was used to dig the earth for below–
turning the soil and planting seeds. Shell working was Codes:
another flourishing industry. (a) 1 and 2 (b) 1, 2 and 3
2. Babar's Empire included– (c) 2, 3 and 4 (d) All of them
1. Kabul Region Ans. (b) : Akbar was 13 years old when he became
2. Region of Punjab emperor. He captured Malwa in 1561, Gondwana in
3. Modern Uttar Pradesh 1564, Mewar in 1567, Ranthambor in 1569, Gujarat in
4. Modern Rajasthan Region 1573. Akbar divided his kingdom into provinces called
Choose the correct answer from the codes given Suba's governed by Subedars. Akbar commanded large
below– army and had access to large amount of revenue due to
Codes: large territory. Akbar introduced many judicial reforms.
(a) 1 and 2 (b) 2 and 3 Akbar was married to many Hindu princesses and his
(c) 1, 2 and 3 (d) 2, 3 and 4 ministry included many Hindus.
Ans. (c) : Babur, the first Mughal emperor (1526- 5. Which one of the following person was never
1530), seized Kabul in 1504. In 1526, he defeated the associated with Indian National Congress?
Sultan of Delhi, Ibrahim Lodi at Panipat and captured (a) Firozshah Mehta
Delhi and Agra. In 1527, Babur defeated Rana Sanga, (b) Hakim Ajmal Khan
Rajput ruler and allies at Khanwa. In 1528, Babur (c) Khan Abdul Gaffar Khan
defeated the Rajputs at Chanderi. (d) Sir Sayed Ahmad Khan
UK UDA/LDA (Pre) Exam 2003 300 YCT
CLICK HERE FOR FREE MATERIAL

Ans. (d): Among the given options, Sir Sayyed Ahmad Ans. (b) : Mumbai's Gowalia Tank Maidan also known
Khan was a social reformer and was never associated as August Kranti Maidan is the place where Quit India
with Indian National Congress. He was one of those Movement was launched by Mahatma Gandhi on
early pioneers who recognized the critical role of August 8 and 9, 1942.
education in the empowerment of the poor and Quit India Movement succeeded in awakeming people
backward Muslim community. and to infuse courage in people.
6. Diarchy in Provinces was introduced under The slogan 'Do or Die' put a lasting impact on people's
which Act? mind.
(a) 1892 (b) 1909 Both (A) and (R) are true and (R) is the right
(c) 1919 (d) 1935 explanation of (A).
Ans. (c) : Diarchy in Provinces was introduced under 9. Match List-I and List-II and choose the correct
Government of India Act, 1919. Montagu, the secretary answer from the codes given below–
of state and Lord Chelmsford, the viceroy, produced List-I
their scheme of constitutional reforms, which led to A. Madan Mohan Malviya
enactment of the Government of India Act of 1919. B. Motilal Nehru
C. Smt. Annie Beasant
7. Match List-I with List-II and choose the correct D. Gopal Krishna Gokhale
answer from the codes given below– List-II
List-I 1. Home Rule League founder
A. The Regulating Act, 1773 2. Founder of Servants of India Society
B. Indian Council Act, 1909 3. Founder of Hindi University
C. Government of India Act, 1919 4. Constituted the Swaraj Party along with
D. Government of India Act, 1935 other people
List-II A B C D A B C D
1. Provisions for States Autonomy (a) 3 4 1 2 (b) 4 3 2 1
2. Initiation of Diarchy Rule (c) 1 2 3 4 (d) 2 1 4 3
3. Beginning of Communal Electoral College Ans. (a) : Correct match is-
4. Establishment of Supreme Court Madan Mohan Malviya - Founder of Hindi University
Codes: Motilal Nehru - Constituted the Swaraj Party
A B C D along with other people
(a) 1 2 3 4 Smt. Annie Beasant - Home Rule League founder
(b) 4 3 2 1 Gopal Krishna Gokhale - Founder of Servants of India
(c) 2 1 4 3 Society
(d) 3 4 1 2 10. In Indian National Congress, Surat Split
occurred in the year–
Ans. (b) : Correct match is-
(a) 1905 (b) 1906
The Regulating Act, 1773 - Establishment of
(c) 1907 (d) 1908
Supreme Court
Indian Council Act, 1909 - Beginning of Ans. (c) : In Indian National Congress, Surat Split
Communal Electoral occurred in the year 1907 during Surat session. Bal
College Gangadhar Tilak led the Surat split in 1907, as he was
the leader of extremist.
Government of India Act, 1919 - Initiation of Diarchy
Rule 11. 'Quit India Movement' was an outcome of–
Government of India Act, 1935 - Provisions for States 1. Discontentment arising in Indian's on
Autonomy account of Cripps proposal
8. Below two statements are given– 2. Threat of Japanese attack on India
Statement (A) : Quit India Movement succeeded 3. Instigating the people of India to adopt
in awakening people and to infuse courage in violent means in Gandhi's Articles
people. 4. Passage of a resolution in 1942 by A.I.C.C.
Reason (R) : The slogan 'Do or Die' put a lasting Choose the correct answer using the codes given
impact on people's mind. below–
In context of the above which of the following Codes:
options is/are correct? (a) 1 and 2 (b) 1, 2 and 4
(a) Both (A) and (R) are true and (R) is the right (c) 2, 3 and 4 (d) All of them
explanation of (A). Ans. (b) : After the failure of the Cripps Mission,
(b) Both (A) and (R) are true but (R) is not the Mahatma Gandhi decided to launch his third major
correct explanation of (A). movement against British rule. This was the “Quit
India” campaign, which began in August, 1942. By the
(c) (A) is true but (R) is false. middle of 1942, Japanese troops were approaching the
(d) (A) is false but (R) is true. borders of India. Pressure was mounting on Britain to
UK UDA/LDA (Pre) Exam 2003 301 YCT
CLICK HERE FOR FREE MATERIAL

solve the issue of the future status of India before Ans. (b) : Loam sandy soil has the lowest water
the end of Second World War. From 29 April to 1 May retaining capacity.
1942, the All India Congress Committee assembled in Water holding capacity order-
Allahabad to discuss the resolution of the Working Clay soil > Silt soil > Loamy soil > Sandy soil.
Committee. The most significant of them being the
Sandy soil mostly consists of sand and clay. The water
commitment to non-violence. On 7 to 8 August 1942,
the All India Congress Committee met in Bombay and holding capacity of this soil is very low.
ratified the 'Quit India' resolution. Gandhi called for 'Do 18. Maximum area under forest is found in which of
or Die'. the following states of our country?
12. Partition of Bengal Province was effected during (a) Kerala (b) Uttar Pradesh
the Viceroy rule of– (c) Madhya Pradesh (d) Rajasthan
(a) Lord Lansdowne (b) Lord Elgin Ans. (c) : As per Forest Survey Report, 2021, Area-
(c) Lord Curzon (d) Lord Morley wise, Madhya Pradesh has the largest forest cover in
Ans. (c) : In 1905, Viceroy Curzon partitioned Bengal. India followed by Arunachal Pradesh, Chhattisgarh,
At that time Bengal was the biggest province of British Odisha and Maharashtra.
India and included Bihar and parts of Orissa. The 19. The largest producer State of Soyabeen in India
British argued for dividing Bengal for reasons of is?
administrative convenience. (a) Andhra Pradesh (b) Karnataka
13. Kalagarh Dam is built upon the river? (c) Madhya Pradesh (d) Uttar Pradesh
(a) Yamuna (b) Sharda Ans. (c) : Madhya Pradesh is the largest Soyabeen
(c) Ganga (d) Ram Ganga producing State in India followed by Maharashtra and
Ans. (d) : Kalagarh Dam is built upon the river Ram Rajasthan.
Ganga. The Kalagarh Dam also referred as The 20. Which of the following is an oilseed crop?
Ramganga Dam, is an earth and rock-fill embankment (a) Masoor
dam built on Ramganga River about 3 Km. upstream in (b) Lobia
Pauri Garhwal of Uttarakhand. This Dam is located (c) Sunflower
within Jim Carbett National Park. (d) Clover (Trifolium Alexandrinum)
14. Which of the following is not matched correctly? Ans. (c) : Seeds of sunflower plant contain oil.
(a) Sunderbans – West Bengal Sunflower seeds contain nutrients and plant compounds
(b) Bhitar Kanika – Odissa that help reduce risk of inflammation, heart disease and
(c) Pitchavaram – Tamilnadu type 2 diabetes.
(d) Vembanad – Karnataka 21. At present the largest textile producing centre of
Ans. (d) : Vembanad lake is in Kerala. In the India is?
Vembanad Lake, there is a small beautiful island called (a) Ahmedabad (b) Kanpur
Pathiramanal or the Midnight sands. Rest are correctly (c) Mumbai (d) Surat
matched. Ans. (c) : Mumbai is the largest textile producing center
15. Which of the following is a westward flowing followed by Ahmedabad.
river of India? 22. Which of the following is not a large circle on a
(a) Ramganga (b) Narmada globe?
(c) Godavari (d) Chambal (a) Equator line
Ans. (b) : Narmada, Tapi, Mahi and Periyar rivers (b) Chief Longitude line
discharge their waters in the Arabian Sea i.e they flow (c) 600 East Longitude
towards west. Narmada River is also known as the (d) 600 North Latitude
Rewa River. The River originates from Maikala range
near Amarkantak, Tributaries of Narmada river are Ans. (d) : 600 North Latitude does not form circle on
Kolar river, Shakkar river, T globe. Rest forms circle on globe.
16. In the Temperate Zone of Western Himalayas 23. In which of the following city, Mediterranean
which tree variety is found predominantly? type of climate is not found?
(a) Pine (b) Deodar (a) Los Angles (b) Rome
(c) Silver Fir (d) Blue Pine (c) Capetown (d) New York
Ans. (b) : The deodar is a symbol of prosperity and is Ans. (d) : Mediterranean climate occurs around
venerated as the ‘tree of God.’ In the Temperate Zone Mediterranean sea, along the west coast of continents in
of Western Himalayas, Deodar trees are found at a subtropical latitudes between 30° - 40° latitudes e.g.
height of 1000 m to 3000 m. A full-grown Deodar tree Central California, Central Chile, along the coast in
has height of 15 to 20 feet. south eastern and south western Australia.
17. The water retaining/carrying capacity of which 24. Which of the following is not a national capital
type of soil is lowest? city?
(a) Sandy loam soil (b) Loam sandy soil (a) Bonn (b) Canberra
(c) Loamy clay soil (d) Loam Soil (c) New York (d) Beijing
UK UDA/LDA (Pre) Exam 2003 302 YCT
CLICK HERE FOR FREE MATERIAL

Ans. (c) : New York is not a capital city. Bonn is capital 31. Consider the following statements–
of Germany, Canberra is capital of Australia and Statement (A): The urban population of India
Beijing is capital of China. (2001) is more than the total population of U.S.A.
25. Which of the following salt is found in abundant Reason (R) : India is more urbanized than U.S.A.
quantity in seas? With reference to the above, which of the
(a) Calcium Carbonate (b) Sodium Chloride following statement is true?
(c) Potassium Chloride (d) Magnesium Sulphate (a) Both (A) and (R) are true and (R) is the correct
Ans. (b) : Sodium Chloride is found in abundant explanation of (A).
quantity in seas. Rocks on land are the major source of (b) Both (A) and (R) are true but (R) is not the
salts dissolved in seawater. Another source of salts in correct explanation of (A).
the ocean is hydrothermal fluids, which come (c) (A) is true but (R) is false.
from vents in the seafloor. (d) (A) is false but (R) is true.
26. Which country holds the first place in Milk Ans. (c): India has second largest population in the
production? world followed by China. The urban population of India
(a) China (b) India (2001) is more than the total population of U.S.A.
(c) Russia (d) U.S.A. U.S.A is most developed nation of the world. Hence
reason is not correct.
Ans. (b) : India is the world's largest milk producer,
with 22 percent of global production, followed by the 32. Indian Constitution explicitly does not provide
United States of America, China, Pakistan and Brazil. for 'Freedom of Press' but this freedom is
implicitly defined in the Article–
27. Which of the following country is the world's
(a) 19 (1) A of Constitution
largest producer of Wheat and Rice both?
(a) China (b) India (b) 19 (1) B of Constitution
(c) 19 (1) C of Constitution
(c) Russia (d) U.S.A.
(d) 19 (1) D of Constitution
Ans. (a) : China is the largest producer of both Wheat
and Rice in world. Ans. (a) : Article 19 (1) A is regarding freedom of
speech and expression. Freedom of Press comes under
28. According to the 2001 census, the state which this Article. Article 19(1) says that all citizens shall
has the lowest population density in India is? have the right to freedom of speech and expression. But
(a) Arunachal Pradesh (b) Sikkim the right is subject to reasonable restrictions imposed on
(c) Uttaranchal (d) Himachal Pradesh the expression of this right for certain purposes under
Ans. (a) : As per 2001 Census, Arunachal Pradesh had Article 19(2).
lowest population density in India. Population density 33. Who among the following can summon the Joint
of India according to census 2011 is 382 persons per Session of Indian Parliament?
square km. Bihar has 1,102 persons per square km and (a) President (b) Vice-President
Arunachal Pradesh has 17 persons per square km. (c) Prime Minister (d) Lok Sabha Speaker
29. According to 2001 census, the percentage of Ans. (a): As per Article 108 (3) of the Constitution,
urban population in contrast to total population President can summon the Joint Session of Indian
of India is– Parliament. It is presided over by the Deputy Speaker of
(a) 23.34% (b) 25.72% the Lok Sabha, or in their absence, the Deputy
(c) 27.78% (d) 35.30% Chairman of the Rajya Sabha.
Ans. (c) : According to 2001 census, the percentage of 34. The speaker of Lok Sabha in India is–
urban population in contrast to total population of India (a) Nominated (b) Choosen
is 27.78%. Level of urbanisation at the country as a (c) Elected (d) Appointed
whole increased from 27.7 percent in 2001 to 31.1
percent in 2011. Ans. (b) : According to Article 93 of the Constitution,
The House of the People (Lok Sabha) shall, as soon as
30. The main cause of urbanization is– may be, choose two members of the House to be
1. Rural-Urban imbalance respectively Speaker and Deputy Speaker.
2. Decline in employment opportunities in the 35. Who among the following was appointed as
rural areas Prime Minister more than one time?
3. Non expansive nature of farming land 1. Pandit Jawahar Lal Nehru
4. Magnetic quality of cities 2. Smt. Indira Gandhi
Choose the correct answer using the codes given below: 3. Gulzari Lal Nanda
(a) 1 and 2 (b) 1, 2 and3 4. Atal Bihari Vajpayee
(c) 2, 3 and 4 (d) All of them Choose the correct answer from the codes given
Ans. (d): Urbanization in India is due to several factors. below–
There is lot of employment opportunities in city.
(a) 1 and 2 (b) 1 and 3
Standard of living in cities is far better than villages as
there is lot of facilities in cities. Farming in India mostly (c) 1, 2 and 4 (d) All of these
depends upon nature and does not produce good Ans. (d) : All the persons given in the options were
income. Hence, all the given options are correct. elected Prime Minister of India more than once.
UK UDA/LDA (Pre) Exam 2003 303 YCT
CLICK HERE FOR FREE MATERIAL

Pandit Jawahar Lal Nehru (August 15, 1947 - May 27, 41. The State Finance Commission is Constituted
1964), Gulzari Lal Nanda (January 11, 1966 - under the–
January24, 1966 May 27, 1964 - June 9, 1964), Smt. (a) Article 24 (h) of Constitution
Indira Gandhi (January 14, 1980 - October 31, 1984 (b) Article 243 (I) of Constitution
January 24, 1966 - March 24, 1977) and Atal Bihari (c) Article 243 (J) of Constitution
Vajpayee (March 19, 1998 - May 22, 2004 (d) Article 243 (K) of Constitution
May 16, 1996 - June 1, 1996) . Ans. (b) : Under Article 243-I of the Constitution, the
36. The planning commission of India is constituted Governor of a State is required to constitute a Finance
under/by– Commission every five years. A State Finance
(a) The constitution of India Commission has functions similar to that of the Central
(b) Under the Act of Parliament Finance Commission. It allocates resources of a State to
(c) A proposal by Cabinet its Panchayati Raj institutions at all three levels in terms
(d) A proposal initiated by a lok sabha proposal of taxes, duties and levies to be collected by the State
Ans. (c) : The planning commission of India is and the local bodies.
constituted under/by a proposal by Cabinet. Planning 42. The most formidable challenge before India is–
Commission of India was formed in 15 March, 1950. Its (a) to mobilize greater public funds for
objectives was to formulate India's Five-year plans. augmenting infrastructure facilities
Note- Niti Aayog replaced Planning Commission of (b) To maintain the bio-diversity of nation
India on 1st January, 2015 with the Prime Minister as (c) To ensure better and more balanced
its Ex-offcio Chairman. development
37. The head of 13th Finance Commission is/was– (d) Cleansing of the main Rivers of India
(a) K. C. Pant (b) Vimal Jalan Ans. (c) : The most formidable challenge before India is
(c) C. Rangarajan (d) Jaswant Singh to ensure better and more balanced development for
Ans. (*) : Vimal Jalan was the head of 13th Finance every citizen.
Commission. The 13th Finance Commission was 43. Which of the following is the largest head of
constituted by the President under Article 280 of the non-planned expenditure of Central
constitution on 13 November 2007 to make Government in India?
recommendations for the period 2010-15. Dr. Vijay (a) Grants (b) Administration
Kelkar was appointed the Chairman of the Commission. (c) Payment of Interest (d) Social Expenditure
No option given is correct. Ans. (c) : Payment of Interest is the largest head of non-
38. The Preamble of Indian constitution was planned expenditure of Central Government in India.
Amended by– Non-plan expenditure is the amount spent by the
(a) Seventeenth Constitution Amendment government on the non-productive areas, such as
(b) Twenty Fourth Constitution Amendment salaries, subsidies, loans and interest, pensions,
(c) 42nd Constitution Amendment statutory transfers to states and UTs governments.
(d) 44th Constitution Amendment 44. In India the largest Commercial Bank is–
Ans. (c) : The Preamble of Indian Constitution was (a) State Bank of India
Amended by 42nd Constitution Amendment, 1976. The (b) I. C.I. C. I.
Preamble has been amended only once for, in 1976. The (c) Reserve Bank of India
amendment added three new words :- (d) I. D. B. I.
1. Socialist 2. Secular 3. Integrity Ans. (a) : State Bank of India is the largest commercial
39. The Area of a Local Panchayat is determined bank of India. State Bank of India (SBI) is an Indian
by– public sector bank and financial services body. Its
(a) State Election Commission headquarter is in Mumbai. It was founded on 1 July,
(b) State Government 1955.
(c) Divisional Commissioner 45. In India Value Added Tax (VAT) will replace–
(a) Sales and Purchase Tax
(d) District Magistrate of district
(b) Entry Tax
Ans. (b) : According to Article 243 of the Constitution,
the Area of a Local Panchayat is determined by State (c) Turnover Tax
Government. (d) All of them
40. In Uttaranchal a Gram Sabha is Ans. (a): As a taxation concept, VAT replaced Sales
established/formed by– Tax. Value Added Tax (VAT) is an indirect value added
(a) Divisional Commissioner tax which was introduced into Indian taxation system on
(b) A Magistrate of the district 1st April 2005. A Value Added Tax (VAT) is a
consumption tax levied on a commodity whenever it adds
(c) Jila Panchayat
value at any point in the supply chain, from production to
(d) State Government sale. The amount of VAT that the consumer pays is based
Ans. (d) : In Uttaranchal a Gram Sabha is on the cost of the product, minus any previously taxable
established/formed by State Government. costs of products used in the product.
UK UDA/LDA (Pre) Exam 2003 304 YCT
CLICK HERE FOR FREE MATERIAL

The Goods and Services Tax (GST), which has replaced 52. Which of the following three conditions are most
the Central and State indirect taxes such as VAT, excise significant for the germination of a seed?
duty and service tax was implemented from 1 July, (a) Soil, Water, Oxygen
2017. (b) Water, Ambient temperature, Oxygen
46. Consider the following statements– (c) Ambient temperature, Oxygen, CO2
1. The sky appears blue due to scattering of (d) Temperature, Oxygen, Sunlight
light Ans. (b) : Water, Ambient temperature, Oxygen are
2. Seven colours of Rainbow is produced due three conditions that are most significant for the
to scattering of light germination of a seed. The beginning of the growth of a
3. The sun appears/looks Red during sunrise seed into a seedling is known as germination. All seeds
on account of scattering of light need water, oxygen and the right temperature to
Out of the above statements– germinate. Dormancy is a state of suspended animation
(a) Only 1 is true (b) 1 and 3 are correct in which seeds delay germination until conditions are
(c) Only 2 is correct (d) 2 and 3 are correct right for survival and growth.
Ans. (b) : Seven colours of Rainbow is produced due to 53. Which of the following group of animals are
the dispersion of light. Hence, second statement is not usually nocturnal in nature?
correct. Rest are correct. When light passes from one (a) House fly, Bedbugs, Parrot
medium to another then a part of light is absorbed by (b) Mosquito, Bats, Owls
particles of the medium, preceded by its subsequent (c) Mosquito, Sparrow, Deer
radiation in a particular direction. (d) Owl, Bats, Dogs
47. Those atoms in which the number of Protons is Ans. (b) : Owl, Bats and Mosquito is the group of
same but the number of Neutrons are different animals which are usually nocturnal in nature.
are known as– Nocturnality is an animal behavior in which the animal
(a) Isotopes (b) Isobars remains active during the night and sleeps during the
(c) Isomers (d) Isotones day. Creatures that sleep during the day and are active
Ans. (a) : Isotopes have same number of protons and at night are known as nocturnal, and they take the form
different number of Neutrons. Isobars are atoms of of both predators and prey.
different chemical elements that have the same number 54. Sleeping sickness disease is caused by–
of nucleons and Differ in atomic number but have same (a) Deficiency of Vitamin-A
mass number. Isotones are two or more species of atoms (b) Deficiency of Calcium in body
or nuclei that have the same number of neutrons.
Isomers are molecules or poly atomic ions with (c) Increase in blood pressure
identical molecular formulae. (d) An unicellular organism called Trypanosoma
48. Who among the following invented 'X-Rays'? Ans. (d) : Sleeping sickness disease is caused by an
(a) Rutherford (b) Roentgen unicellular organism called Trypanosoma. Human
(c) Maxwell (d) Torricelli African trypanosomiasis, also known as sleeping
sickness, is a vector-borne parasitic disease. It is caused
Ans. (b) : Roentgen invented X-Rays in 1895. by infection with protozoan parasites belonging to the
49. The lead of pencil is made up of– genus Trypanosoma.
(a) Lead (b) Antimony 55. Name the scientist who discover that Malaria is
(c) Graphite (d) None of them caused by a protozoa called – Plasmodium?
Ans. (c) : The lead of pencil is made up of graphite. (a) G. J. Mendel
Carbon has three allotropes : diamond, graphite and (b) E. Haeckel (Zoologist)
fullerene. The core of the lead pencil is made up of a (c) Sir Ronald Ross
mixture of graphite, powder and clay. (d) Darwin
50. The colourless gas emitted from motor cars and Ans. (*) : Alphonse Laveran discovered that malaria is
cigarettes due to incomplete combustion is– caused by protozoan parasite (Plasmodium) in 1880. He
(a) CO2 (b) NO discovered Plasmodium and got nobel prize in 1907. Sir
(c) CO (d) CH4 (Methane) Ronald Ross in 1897, a doctor in Indian Army,
Ans. (c) : Carbon monoxide is a colourless gas which is established that malaria parasite is transmitted by the
emitted from motor cars and cigarettes due to bite of a female Anopheles mosquito and in 1902, he
incomplete combustion. CO consists of one carbon got Nobel prize for this discovery.
atom and one oxygen atom connected by a triple bond. 56. The scientific study of Insects is known as–
51. Which of the following is lighter than water? (a) Ichthyology (b) Entomology
(a) Aluminium (b) Sodium (c) Parasitology (d) Malacology
(c) Magnesium (d) Manganese Ans. (b) : Entomology is a branch of zoology dealing
Ans. (b) : Among the given options, Sodium is lighter with the scientific study of insects. Basically,
than water. Sodium is a chemical element with the Entomology is the study of insects and their relationship
symbol Na and atomic number 11. It is soft, silvery- to humans, the environment, and other organisms. An
white, highly reactive metal. entomologist is a scientist who studies insects.
UK UDA/LDA (Pre) Exam 2003 305 YCT
CLICK HERE FOR FREE MATERIAL

57. The cause of spoiling of milk is? Ans. (d): The maximum weight of human body is on
(a) Lactobacillus (b) Aspergillus account of water. The amount of water in the human
(c) Pseudomonas (d) Staphylococcus body ranges from 45-75%. The average adult human
Ans. (c) : The cause of spoiling of milk is body is 50-65% water, averaging around 57-60%.
Pseudomonas. Spoilage of milk and milk products Water is of major importance to all living things; in
results from growth of fermentative bacteria when some organisms, up to 90% of their body weight comes
storage temperatures are sufficiently high for from water. Up to 60% of the human adult body is
psychrotrophs. Heat-resistant proteinases of water.
psychrotrophic bacteria cause spoilage in processed 63. A marriage is solemnized between colour blind
milk because of enzyme-retaining activity after the heat male with a normal female whose parents eye-
treatment. About 65–70% of psychrotrophic sight was also normal. Then what percentage of
microorganisms in raw milk is Pseudomonas species. children born will have the probability of being
58. Seal belongs to the family of? colour blind?
(a) Fish (b) Birds (a) 25% (b) 50%
(c) Reptiles (d) Mammals (c) 100% (d) 0%
Ans. (d) : Seals are semi-aquatic mammals that are in a Ans. (b) : Colour blindness is specifically linked to
group called pinnipeds, meaning ‘fin-footed’. Seals defects within the X chromosome. For colour blindness
have fins and flippers instead of legs and are beautifully to be present all ‘X’ chromosomes within an individual
adapted for life in the water. They live chiefly in cold have to be affected, and since men only have one X
seas and whose body shape is round at the middle and chromosome and women have 2, the chances of men
tapered at the ends, is adapted to swift and graceful being impacted are exponentially higher.
swimming. However, in this case, it would seem that due to the
59. The hardest part of human body is? genetic inheritance of the mother the chances of their
child being colour blind would be approximately 50%
(a) Bone (b) Enamel
because each parent is a carrier. However, the chances
(c) Dentin (d) Cementum of inheriting a colour blind X chromosome would be
Ans. (b) : Enamel is the hardest substance in the human twice as likely for a female child. Below is how this
body, but it does not grow back once it’s lost. The tooth scenario broke down, where the lower case x represents
enamel is made of calcium phosphate. All teeth have the affected X chromosome.
three layers: enamel, dentine, and pulp. The enamel is the M: xY F: xX
outermost layer and is primarily made of calcium xx—-Female child (colour blind)
phosphate minerals. xX—Female Child (not colour blind but a carrier)
60. The number of chromosome pairs found in cells Yx—Male Child (Colour Blind)
of human body is– YX—Male Child (not colour blind and not a carrier)
(a) 21 (b) 22 So, the correct answer is '50%'.
(c) 23 (d) 24
64. The Red Colour of R. B. Cs is due to–
Ans. (c) : The total number of chromosomes in a human (a) Cutin (b) Chlorophyll
cell is 23 pairs. Humans have 23 pairs of chromosomes,
(c) Haemocyanin (d) Haemoglobin
for a total of 46 chromosomes in which 22 pairs of
chromosomes are same in male and female and Ans. (d) : Haemoglobin (Hb or Hgb ) is an iron-
homologus to each other in pair. They are collectively containing protein present in erythrocytes or Red blood
known as autosomes. The 23rd pair, the sex cells (RBC) of almost all vertebrates. Each haemoglobin
chromosomes, differ between males and females. These is made up of 4 subunits, 2 alpha subunits and 2 beta
chromosomes are known as heterosomes. subunits. Each subunit surrounds the central heme
group that contains iron and binds to one oxygen
61. The enzyme which helps in digestion of milk molecule. The blood cells are red due to the interaction
protein is? between iron and oxygen. Hence, the red colour of the
(a) Pepsin (b) Trypsin red blood cells is due to the presence of haemoglobin.
(c) Renin (d) Erypsin
65. Yeast and Mushrooms are–
Ans. (c) : Rennin, known also as chymosin, is a (a) Algae (b) Gymnosperm
proteolytic enzyme related to pepsin that is synthesized (c) Fungi (d) Tuberous Roots
by chief cells in the stomach of some animals. Its role in
Ans. (c) : Yeast and Mushrooms are fungi. Fungi are
digestion is to curdle or coagulate milk in the stomach,
multicellular, eukaryotic, heterotrophs, that acquire
a process of considerable importance in the very young
their nutrients by absorption. They feed on dead or
animal.
decaying organic matter or depend on host organisms.
62. The maximum weight of human body is on
account of? 66. Insulin is produced by the–
(a) Bones (a) Islets of Langerhans
(b) Various body organs (b) Pituitary glands
(c) Skin, Muscles and Organs (c) Thyroid glands
(d) Water (d) Adrenal glands
UK UDA/LDA (Pre) Exam 2003 306 YCT
CLICK HERE FOR FREE MATERIAL

Ans. (a):Insulin is produced by the islets of Langerhans. 71. The name ascribed to the first prototype of Civil
The islets of Langerhans contain four cell types that Passenger Aircraft developed in our country by
each secrete a different peptide: alpha cells secrete the Public Sector Undertaking is–
glucagon, beta cells secrete insulin, delta cells secrete (a) Cheetal (b) Chetak
somatostatin, and PP or F cells secrete pancreatic (c) Mayur (d) Saras
polypeptide. Ans. (d) : National Aeronautics Limited (NAL)
67. In Computers, the types of memory is– developed first prototype of Civil Passenger Aircraft
1. Semiconductors 2. Magnetic "Saras" in 2004. It is the first Indian multi-purpose
3. Server 4. Optical civilian aircraft in the light transport aircraft category as
Choose the correct answer using the codes given designed by the NAL.
below– 72. The most important group of crops cultivated in
Codes: Uttaranchal is–
(a) 1 and 2 (b) 2 and 3 (a) Paddy, Sugarcane, Jhingora and Arhar (Pigeon
(c) 1, 2 and 4 (d) All the four Pea)
Ans. (c): Semiconductor, Magnetic and Optical are the (b) Paddy, Wheat, Madua and Potato
types of memory in a computer. In computing, a server (c) Wheat, Mandua, Jhingora and Potato
is a piece of computer hardware of software that (d) Paddy, Wheat, Arhar (Pigeon Pea) and Masur
provides functionality for other programs or devices. In (Red lentils)
computing, a server is a piece of computer hardware or Ans. (b) : Paddy, Wheat, Madua and Potato are the
software that provides functionality for other programs most important group of crops grown in Uttarakhand.
or devices. Major crops grown in Uttaranchal are Rice, Wheat,
68. The word Monitor is related to– Sugarcane, Maize, Soyabean, Pulses, Oil seeds.
1. An ordinary operating system Uttaranchal has four agro-climatic zones so there is
2. To give permission to the user to enter the potential to grow a wide variety of crops within the
programme state.
3. To give permission to user to run the 73. Transhumance is popular in–
programme (a) Bhotia Tribes (b) Jaunsari Tribes
4. To allow the user to make changes in (c) Raji Tribes (d) Tharu Tribes
programme Ans. (a) : Transhumance (mobility) is popular among
Choose the correct answer using the codes given Bhotia tribes in Uttarakhand. Bhotiya is one of the most
below– popular scheduled tribe of Uttarakhand. People of this
Codes: community living in upper Bhagirathi basin are known as
(a) 1 and 2 (b) 1, 2 and 3 Jaad. The most of the families of Jaad community move
(c) 1, 3 and 4 (d) All the four Dunda to Bagori in summer, but one or two family
Ans. (*) : Monitor is a display device of a computer. members move from with their herds of sheep and goats
Computer monitor is an output device that displays in between the shivaliks touching Dehradun, Rishikesh
formation in pictorial or text form. Modern computer region and the high altitude Bugyals.
monitors are easily interchangeable with conventional 74. Askot wild life sanctuary is located in the
television sets and vice-versa. district of?
69. Brahmos is a– (a) Almora (b) Chamoli
(a) Aeroplane (b) Computer virus (c) Uttarkashi (d) Pithoragarh
(c) Missile (d) Submarine Ans. (d) : Askot wild life sanctuary is located in
Ans. (c):Brahmos is a supersonic missile jointly developed Pithoragarh district of Uttarakhand. It was established in
by Russia and India. Brahmos is a medium-range stealth 1986. It was created in order to protect the endangered
ramjet supersonic cruise missile that can be launched from Musk Deer. It is a haven for snow leopards, Himalayan
submarine, ships, air craft or land. The name Brahmos is black bears, musk deers, snow cocks, tahra, bharals,
taken from the names of two rivers, the Brahmaputra of chirs, koklas, pheasants and chukors.
India and the Moskva of Russia. 75. The commission established by the State
70. The meaning of E-Commerce is– Government for selecting a place for a
(a) Export trade permanent capital in the Uttaranchal State is?
(b) Trade with European Nations (a) Pant Commission
(c) Trade on Internet (b) Liberhan Commission
(d) None of these (c) Dikshit Commission
Ans. (c) : E-Commerce covers a firm’s interactions with (d) Banerji Commission
its customers and suppliers over the internet. E- Ans. (c) : Dikshit Commission was appointed by the
commerce (electronic commerce) is the buying and Uttarakhand government for selecting a place for a
selling of goods and services. E-commerce typically uses permanent capital in the Uttarakhand State. Ever since
the web for at least a part of a transaction life cycle Uttarakhand came into existence or November 9, 2000,
although it may also use other technologies such as e- it has been a demand that Garsain in Chamoli, be made
mail. capital.
UK UDA/LDA (Pre) Exam 2003 307 YCT
CLICK HERE FOR FREE MATERIAL

76. Which one among the following is the Fodder 82. In Uttaranchal, the Mayor of any particular
tree which is a source of popular fodder in Municipal Corporation is–
Uttaranchal's hilly region? (a) Appointed by the State Government
(a) Pine (b) Banj (b) Nominated by the Governor
(c) Deodar (d) Teak (Sagaun) (c) Elected by the local people
Ans. (b) : Banj (Oak) fodder tree is a source of popular (d) Elected by the Corporators
fodder in Uttaranchal's hilly region. Banj oak (Quercus Ans. (c) : In Uttranchal, the Mayor of any Particular
leucotrichophora) is an evergreen oak tree of Asia, and Municipal corporation is elected by Local People. The
particularly of the Central Himalayas. mayor is elected directly through a first-past-the post
voting system and the deputy mayor is elected by the
77. The most populous district of Uttaranchal State is? corporators from among their members.
(a) Almora (b) Dehradun
83. The river Bhagirathi and Alaknanda together
(c) Haridwar (d) Udham Singh Nagar combine to be named Ganges River from–
Ans. (c) : As per 2011 Census, Haridwar (1,890,422) is (a) Haridwar (b) Rishikesh
most populous district of Uttarakhand. Haridwar is the (c) Dev Prayag (d) Rudra Prayag
most populous out of the 13 Districts of Uttarakhand. It Ans.(c): Devprayag is the confluence of two
has a population density of 801 people per square km, holy rivers Alaknanda and Bhagirathi to form Ganga.
over 4 times the state average (189 persons per sq. km) River Bhagirathi originates from Gangotri and
sex ratio of state is 963 females per one thousand males. Alaknanda originates from Badrinath. In Sanskrit,
78. Uttaranchal State was constituted on– Prayag means 'Confluence' and Devprayag stands for
(a) 9th April, 2000 (b) 1st November, 2000 'Holy confluence'.
(c) 9 November, 2000 (d) 15th November, 2000
th
84. Badrinath Temple is located in–
Ans. (c) : Uttaranchal State was constituted on 9th (a) Chamoli district
November, 2000. It is often referred to as the (b) Rudra Prayag district
Devbhumi (Land of the Gods) due to its religious (c) Tehri district
significance and numerous Hindu temples and (d) Uttar-Kashi district
pilgrimage centres. In 2000, Government of India Ans.(a): The Badrinath Temple, located in
created three new states, Chhattisgarh (1 November), Uttarakhand's Badrinath town, Chamoli district is one
Uttaranchal (9 November) and Jharkhand (15 of the four Char Dhams (four important pilgrimages) in
November). the State. There are four pilgrim-destinations namely
79. Which among the following is not a district of Yamunotri, Gangotri, Kedarnath, and Badrinath,
collectively known as Char Dham.
Uttaranchal?
(a) Roorkie (b) Rudra Prayag 85. The first National Park established in India is
located in Uttaranchal. It is known as–
(c) Udham Singh Nagar (d) Bageshwar
(a) Rajaji National Park
Ans. (a) : There are 13 districts in Uttarakhand namely (b) Flowers Valley
Almora, Bageshwar, Chamoli, Champawat, Dehradun, (c) Jim Corbett National Park
Haridwar, Nainital, Pauri Garhwal, Pithoragarh, (d) Gangotri National Park
Rudraprayag, Tehri Garhwal, Udham Singh Nagar and
Ans.(c): Jim Corbett National Park (Nainital,
Uttarkashi.
Uttarakhand) was established in the year 1936 as Hailey
80. Chipko Movement was started originally from National Park. Jim Corbett National Park has the glory
which district of Uttaranchal? of being India's oldest and most prestigious National
(a) Uttarkashi (b) Tehri Garhwal Park. It is also being honored as the place where Project
(c) Chamoli (d) Pauri Garhwal Tiger was first launched in 1973. Jim Corbett National
Ans. (c) : The Chipko movement started in 1973 in Park is located in the Nainital district and Pauri
Chamoli district. The Chipko movement was a non- Garhwal district of Uttarakhand.
violent agitation that was aimed at protection and 86. The State flower of Uttaranchal is–
conservation of trees. It is best remembered for the (a) Buransh (b) Lotus
collective mobilization of women for the cause of (c) Brahma Kamal (d) None of these
preserving forests, which also brought about a change in Ans. (c) : Brahma Kamal (Botanical name- Saussurea
attitude regarding their own status in society. The name Obvallata) is the State flower of Uttarakhand. This tree
of the movement ‘chipko’ comes from the word is native to the Himalayas in Bhutan, Nepal, Pakistan
’embrace’, as the villagers hugged the trees and and china. Flowers on these trees bloom in monsoons.
encircled them to prevent being hacked. 87. Rajaji National Park is a natural habitat of–
81. The total number of cities in Uttaranchal, which (a) Mahseer Fish (b) Cheetal
has a municipal corporation– (c) Musk Deer (d) Asian Elephant
(a) 1 (b) 2 Ans. (d) : Rajaji National Park is best known for Asian
(c) 3 (d) 4 wild elephants. Located at the Himalyan foothills, near
Ans. (*): As on date, Uttarakhand has 9 municipal Haridwar, Rishikesh and Dehradun, with Ganga and song
corporations. Srinagar, Uttarakhand became the ninth rivers, flowing through it other wild animals found in
Municipal Corporation of State on 1st January, 2022. park include:- Indian leopard, Jungle cat, sloth bear etc.

UK UDA/LDA (Pre) Exam 2003 308 YCT


CLICK HERE FOR FREE MATERIAL

88. To promote Organic or Bio-farming which state 92. If a Pump A fills a water tank in 6 hours and
has declared itself as an Organic or Bio-State? another Pump B fills the same water tank in 12
(a) Uttaranchal (b) Chhattisgarh hours then in how many hours the tank will get
(c) Arunachal Pradesh (d) Rajasthan filled if both the Pumps are run together
Ans. (a) : To promote Organic or Bio-farming, (simultaneously)?
Uttarakhand has declared itself as an Organic or Bio- (a) 2 (b) 3 (c) 4 (d) 5
State. 70 % of the population of Uttarakhand is Ans. (c): Time taken to fill the tank by both pump
dependent on farming. The Governor of Uttarakhand together
has approved the Organic Agriculture Act (2019). 6 × 12
= = 4 hrs
89. Below two statements are given– ( 6 + 12 )
Statement (A) : The legislature in Uttaranchal
comprises of 70 members. 93. The missing number in the following number
Reason (R) : State has a bicameral legislative. series is –
Which of the following is correct in context of 82, 97, 114, 133, ?
above to statements? (a) 152 (b) 153 (c) 154 (d) 155
(a) Both (A) and (R) are true and (R) is the correct Ans. (c): 97- 82 = 15, 114 -97 = 17, 133-114 = 19
explanation of (A). so, 133+21= 154
(b) Both (A) and (R) are true, but (R) is not the 94. A person is standing with his face in North-West
correct explanation of (A). direction. He turns 900 to right hand side and
(c) (A) is true but (R) is false. again turns towards his left. He is now facing
(d) (A) is false but (R) is true. which direction?
Ans. (c): Uttarakhand Assembly has 70 seats. Therefore (a) North (b) South (c) East (d) West
assertion is correct but Uttarakhand do not have Ans. (a) :
bicameral legislature i.e Uttarakhand has unicameral
governing and law making body.
90. Consider the following statements–
Statement (A): In Uttaranchal state there is lot of
potentiality for growth and development.
Reason (R): For Growth there is a need for 95. 100 mm is equivalent to–
Scientific Planning and Effective Implementation (a) 0.001 metre (b) 0.01 metre
Machinery. (c) 0.1 metre (d) 1.0 metre
In context of above, which of the following is Ans. (c): 100 mm = 0.1 metre
correct?
(a) Both (A) and (R) are true and (R) is the correct 96. The present age of Rajan is 15 years more than
explanation of (A). Mohan's age. After five years Rajan's age will
(b) Both (A) and (R) are true but (R) is not the become two times the age of Mohan, What is the
correct explanation of (A). present age of Rajan?
(c) (A) is true but (R) is false. (a) 10 years (b) 15 years
(d) (A) is false but (R) is true. (c) 20 years (d) 25 years
Ans. (b) : In Uttaranchal state there is lot of potentiality Ans. (d) : Let present age of Mohan = x years
for growth and development specially in the field of Present age of Rajan = (x+15) years
tourism but in order to implement that, effective planning According to question,
and proper implementation of projects are required. (x+20) = 2 (x+5)
91. Consider the following statements– x + 20 = 2x + 10
Statement (A):The main occupation of Uttaranchal x = 10
people is agriculture.
Hence, present age of Rajan = 25 years
Reason (R): Nearly 90% of its population is
dependent on Agriculture. 97. The highest growth rate in the 10th Five Year
Plan, has been targeted in?
In context of above which of the following
(a) Agriculture (b) Manufacturing
statements is true?
(c) Communication (d) Trade
(a) Both (A) and (R) are true and (R) is the correct
explanation of (A). Ans. (c) : The highest growth rate in the 10th Five Year
(b) Both (A) and (R) are true but (R) is not the Plan (2002-2007) was targeted in the field of
correct explanation of (A). communication. The plan's salient features were to
(c) (A) is true but (R) is false. promote inclusive Growth and equitable development.
(d) (A) is false but (R) is true. 98. The Chief Supervisor or Chief Inspector who
Ans. (a) : Due to lack of industries and other options in led the Iraqi disarmament campaign was?
Uttarakhand, about 90% of the population is dependent (a) Mohammed Al-Baradei (b) Hans Blix
on agriculture for their livelihood. (c) Collin Powell (d) Carolbela
UK UDA/LDA (Pre) Exam 2003 309 YCT
CLICK HERE FOR FREE MATERIAL

Ans. (b): Hans Blix was the Chief Supervisor or Chief Ans. (c): EPA stands for Environment Protection
Inspector who led the Iraqi disarmament campaign. The Agency. It is an independent federal agency under the
Iraq disarmament crisis was claimed as one of primary United States federal government which is Tasked with
issues that led to the multinational invasion of Iraq on environmental protection matters.
20 March 2003. 106. Uttaranchal State's Forest cover percentage is–
99. The aim of National Health Policy, 2002 is (a) 33 (b) 53 (c) 56 (d) 63
elimination of Polio by the year– Ans. (d):At the time when question was asked,
(a) 2005 (b) 2007 (c) 2010 (d) 2015 percentage of forest cover in Uttarakhand was 63.
Ans. (a) : The aim of National Health Policy, 2002 was Note- As per India State of Forest Report, 2021,
elimination of Polio by the year 2005. Uttarakhand has a total forest cover of 24,465 km2,
Note- Within two decades, India received 'Polio-free which is 45.74% of the geographical area of the State
certification' from World Health Organization on 27 and 3.61% of the forest cover of the country.
March, 2014. 107. Which of the following player has been declared
100. The targeted growth rate of Tenth Five Year CEAT International Cricket Player of the
Plan is Century?
(a) Sachin Tendulkar (b) Sunil Gavaskar
(a) 5.5% (b) 6.5%
(c) Vivian Richards (d) Kapil Dev
(c) 8.0% (d) 8.5%
Ans. (d): Kapil Dev was declared CEAT International
Ans. (c) : The targeted growth rate of Tenth Five Year Cricket Player of the Century.
Plan (2002-2007) was 8%. Reduction of poverty rate
upto 5% by 2007. 20-point program was introduced. 108. World Cricket Cup, 2007 will be organized by–
Providing gainfull and high-quality employment at least (a) Australia (b) England
in addition to the labour force. (c) Sri Lanka (d) West Indies
101. The Drishti 2025 is related to– Ans. (d): World Cricket Cup, 2007 was organized by
(a) Stabilization of population West Indies. In 2023, India is the host nation of cricket
world cup.
(b) Industrial growth
(c) Increase in food grain production 109. Football World Cup, 2006 will be held in–
(a) Brazil (b) China
(d) Global Warming
(c) Germany (d) Kenya
Ans. (b) : Drishti 2025 is related to Industrial growth by
making percentage of infrastructure sector to 25% of the Ans. (c): Football World Cup, 2006 was held in
Germany. In 2022, Qatar will be host nation.
GDP by the year 2025.
110. In Cricket World Cup 2003, which bowler took
102. 'Asian Brown Cloud' 2002 was mostly spread in– maximum number of wickets?
(a) East Asia (b) South-East Asia (a) B. Lee (b) C. Vass
(c) South Asia (d) West Asia (c) G. McGrath (d) Zahir Khan
Ans. (c) : 'Asian Brown Cloud' 2002 was mostly spread Ans. (b): Chanminda Vaas from Sri Lanka took
in South Asia. The Asian Brown Cloud is a layer of air maximum number of wickets in World Cup Cricket,
pollution that covers the entire atmosphere over a region. 2003. He took 23 wickets.
103. The Earth Summit 2002 was organized in– 111. Which player was declared 'Player of the
(a) Cairo (b) Rio-de-Janeiro Tournament' in Cricket World Cup 2003?
(c) Johannesburg (d) Kyoto (a) A. Gilchrist (b) R. Ponting
Ans. (c): The Earth Summit 2002 was organized in (c) S. Ganguly (d) Sachin Tendulkar
Johannesburg, South Africa under the title "Sustainable Ans. (d) : Sachin Tendulkar was declared 'Player of the
Development". The goal of the Summit was to Tournament' in Cricket World Cup 2003. In final, India
regenerate, at the highest political level, a global was beaten by Australia.
commitment to sustainable development and to 112. In World Cup Hockey, 2002, India stood in
accelerate action to implement Agenda 21. which position?
104. United Nations has declared year 2003 as an (a) Fifth (b) Sixth (c) Ninth (d) Tenth
International year of– Ans. (d) : In World Cup Hockey, 2002, India stood in
(a) Child (b) Disabled Tenth position. Host country was Malaysia. Germany
(c) Fresh Water (d) Ocean won the world cup hockey defeating Australia.
Ans. (c): United Nations has declared year 2003 as an 113. Who won the World Cup Football, 2002?
International year of Fresh Water. In the year 2023, a (a) China (b) Korea
proposal for an international year of millets (2023) was (c) Brazil (d) Japan
brought forward by the Government of India. Ans. (c) : Brazil won the World Cup Football, 2002 by
105. The full form of EPA is– defeating Germany, making them the first country and
(a) Environment Pollution Agency only one to have won the world cup five times.
(b) Environment Prohibition Agency 114. Which of the following player was stripped of
(c) Environment Protection Agency his/her medals because of Doping charges in the
(d) None of the above Busan Asian Games?
UK UDA/LDA (Pre) Exam 2003 310 YCT
CLICK HERE FOR FREE MATERIAL

(a) Anju A. George (b) Madhuri Singh Ans. (b): The Gandhi Peace Prize for the year 2002 has
(c) Neelam J. Singh (d) Sunita Rani been conferred on Bharti Vidya Bhawan. Sheikh Majibur
Ans. (d) : Sunita Rani was stripped of her medals Rahman has been given 2020 Gandhi peace prize.
because of Doping charges in the Busan Asian Games. 122. Hariyali is a new plan–
Indian long-distance runner after failing to pass doping (a) To herald Second Green Revolution
test was stripped of her medals. (b) For the development of Waste Land
115. In the Cricket World Cup, 2003 lowest score (c) Afforestation of Desert Land
was scored by which country? (d) Development of Pasture Land (Gochars)
(a) Canada (b) Netherland Ans. (b): A new initiative 'Hariyali' was launched in
(c) Namibia (d) India January 2003 for watershed programs being
Ans. (a): Canada scored lowest runs (36) during Cricket implemented through Panchayati Raj Institutions.
World Cup, 2003 against Srilanka at Paarl. 123. In India, Silicon Valley is located at?
116. The 13th Non-Aligned Movement Summit (a) Bangalore (b) Chennai
Conference was held in– (c) Kolkata (d) Hyderabad
(a) Bangkok (b) Djakarta Ans. (a) : In India, Bengaluru, Karnataka is called as
(c) Kualalumpur (d) Singapore Silicon Valley of India as major software industries are
located in Bengaluru.
Ans.(c): 13th Non-Aligned Movement Summit Conference
124. Who among the following has been
was held in Kualalumpur, Malaysia. from 20-25 February
decorated/embellished by the Padma Vibhusan
2003. NAM is a forum of 120 countries that are not
Award, 2003?
formally aligned with or against any major power bloc.
(a) Amir Khan (b) Danny Dengzongpa
19th summit will be held in Uganda in 2023.
(c) Jagjit Singh (d) Sonal Man Singh
117. The Dada Saheb Phalke Award for the year Ans. (d): Sonal Man Singh had been decorated/
2001 was given to– embellished by the Padma Vibhusan Award, 2003.
(a) Amir Khan (b) Devanand Others who won Padma Vibhusan are Brihas Pati Dev
(c) Gulzar (d) Yash Chopra Triguna, Kazi Lhendup Dorjee and Bal Ram Nanda.
Ans. (d) : The Dada Saheb Phalke Award for the year 125. Which Indian Super Cop has been selected as a
2001 was given to Yash Chopra. Dadasaheb Phalke Police Advisor to supervise Peace Keeping
Award 2019 was given to actor Rajnikanth. operations in United Nations?
118. The new President of National Human Rights (a) K. P. S. Gill (b) Joginder Singh
Commission is– (c) Kiran Bedi (d) J. F. Ribeiro
(a) A. S. Anand (b) Jaichandra Reddy Ans. (c) : Kiran Bedi, First female IPS officer of India
(c) Madan Mohan Punchi had been selected as a Police Advisor to supervise
(d) M. N. Venkatchalliah Peace Keeping operations in United Nations.
Ans. (a) : At the time when question was asked, Justice 126. Which of the following was not a part of the
A. S. Anand was the President of National Human declaration jointly signed by Prime Minister
Rights Commission (NHRC). At present, Justice Arun Atal Bihari Vajpayee and Russian President
Kumar Mishra is Chairperson of Commission. Vladimir Putin?
119. Columbia is /was– (a) International Terrorism
(a) An advance War Ship of America (b) Rupee-Rouble Agreement
(b) A National Aeroplane of U.S.A. (c) Iraq and Afghanistan
(c) An advance Multinational Company of (d) India's claim for permanent membership in UN
America Security Council in wake of demand for
(d) An American Spacecraft Security Council Expansion
Ans. (d): Columbia was an American spacecraft which Ans. (b) : Rupee-Rouble Agreement was not a part of
crashed while coming back to earth from space. Six the declaration jointly signed by Prime Minister Atal
astronauts died in this crash including Kalpana Chawla, Bihari Vajpayee and Russian President Vladimir Putin.
an astronaut of Indian origin. 127. What have been provided in the Women's
120. Mercy Death has been accorded legal Empowerment Programmes?
recognition by– 1. Improvement in Literacy and Education
(a) U. S. A. (b) China 2. Imparting Skills to enhance Earning
(c) Japan (d) Netherland potential
Ans. (d): Mercy Death has been accorded legal 3. Participation in Decision Making process at
recognition by Netherland. local levels
121. The Gandhi Peace Prize for the year 2002 has 4. Opportunities in Employment.
been conferred on? 5. Health Care and Family Welfare.
(a) Baba Amte Choose the correct answer using the codes given
(b) Bhartiya Vidya Bhawan below–
(c) Dr. Gerhard Fischer (a) 1 and 2 (b) 2, 3 and 4
(d) Ram Krishna Mission (c) 1, 2, 3 and 4 (d) 1, 2, 3 and 5
UK UDA/LDA (Pre) Exam 2003 311 YCT
CLICK HERE FOR FREE MATERIAL

Ans. (d): Women's empowerment programs are Ans. (c): Peacock is national bird of India. Indian
designed and implemented to increase literacy, skill Peacock, Pavo cristatus, is a colourful, swan-sized bird,
development for earning, family welfare and health with a fan-shaped crest of feathers, a white patch under
care. Reservation is provided to women candidates in the eye and a long, slender neck.
Panchayat elections. In recent past, some State 135. Silk bug/worm Rearing is called–
governments are providing reservation to women
(a) Apiculture (b) Horticulture
candidates for government jobs.
(c) Sericulture (d) Floriculture
128. The objective of constructing Golden
Quadrilateral under the National State Highway Ans. (c) : Silk bug/worm Rearing is called Sericulture.
Development Project is– Horticulture is the study of flowers and vegetables.
(a) To connect Delhi, Kolkata, Chennai and Apiculture or beekeeping is the care and management of
Mumbai honey bees for the production of honey and wax.
(b) To connect Srinagar, Mumbai, Chennai and 136. Famous player, Pele is associated with–
Kolkata (a) Belgium (b) Brazil
(c) To connect Kandla, Mumbai, Chennai and (c) Portugal (d) Senegal
Kolkata Ans. (b): Famous football player, Pele is associated
(d) To connect Srinagar, Kolkata, Bangalore and with Brazil.
Ahmadabad 137. Match List-I and List-II and choose the correct
Ans. (a) : The objective of constructing Golden answer from the codes given below:
Quadrilateral under the National State Highway List-I List-II
Development Project is to connect Delhi, Kolkata, A. Rangaswami Cup 1. Cricket
Chennai and Mumbai. B. Reliance Cup 2. Boat Race
129. ITC Music Awards, 2003 was conferred on– C. Rovers Cup 3. Football
(a) Ali Akbar Khan (b) Girija Devi D. Wellington Cup 4. Hockey
(c) Kishori Amonkar (d) Ravi Shankar 5. Golf
Ans. (c) : ITC Music Awards, 2003 was conferred on Codes:
Kishori Amonkar. A B C D
130. Who among the following has been elected as (a) 5 1 3 4
the Head of Sahitya Academy? (b) 2 3 1 5
(a) Gopichand Narang (b) Jagannath Azad (c) 4 1 3 2
(c) Mahasweta Devi (d) Prof. K. P. Singh (d) 5 3 4 2
Ans. (a) : At the time when question was asked, Ans. (c) : Correct match is-
Gopichand Narang was the head of Sahitya Academy. Rangaswami Cup Hockey
The current president of the Sahitya Akademi is Reliance Cup Cricket
Chandra Shekhara Kambara. Rovers Cup Football
131. Baglihar Hydro Electric Project, which was Wellington Cup Boat Race
recently been in limelight, is located in the State of? 138. The meaning of W.L.L. is–
(a) Jammu and Kashmir (b) Maharashtra (a) Without Liver Line
(c) Odissa (d) West Bengal (b) Within Local Line
Ans. (a) : Baglihar Hydro Electric Project built on the (c) Wireless in Local Loop
Chenab river is located in Jammu and Kashmir. It is a (d) Wireless in Long Line
run-of-the-river Power Project.
Ans. (c) : W.L.L stands for wireless in local loop. It is a
132. The National tree of India is– communication technology, which was first used by
(a) Neem (b) Mango (c) Peepal (d) Deodar Germans during Second World War. Later it was used
Ans. (*) : Indian fig tree, Ficus bengalensis (Banyan) for mobile communication in India.
whose branches look like root themselves like new trees 139. The full meaning of SMS is–
over a large area is the national tree of India. This tree (a) Swift Mail System
is considered immortal and is an integral part of the
(b) Short Messaging Service
myths and legends of India.
(c) Short Hand Manual Script
133. Indian Institute of Remote Sensing is located in–
(a) Delhi (b) Dehradun (d) Speed Mail Service
(c) Bhopal (d) Kolkata Ans. (b): SMS stands for Short Messaging Service used
Ans. (b) : Indian Institute of Remote Sensing is located by mobile phones to send text messages.
in Dehradun, Uttarakhand. The Indian Institute of 140. In Kaveri water dispute states involved are–
Remote Sensing is a premier institute for research, (a) Karnataka–Maharashtra–Goa–Andhra Pradesh
higher education and training in field of remote sensing. (b) Karnataka–Tamil Nadu – Maharashtra – Kerala
134. The National bird of India is– (b) Karnataka–Tamil Nadu–Kerala–Andhra Pradesh
(a) Koel (b) Goose (d) Karnataka – Tamil Nadu – Pondicherry –
(c) Peacock (d) Pigeon Kerala
UK UDA/LDA (Pre) Exam 2003 312 YCT
CLICK HERE FOR FREE MATERIAL

Ans. (d): Karnataka, Tamil Nadu, Pondicherry and Codes:


Kerala are the Indian States which have dispute A B C D
regarding distribution of water of Kaveri river. The (a) 2 3 4 1
dispute is 150 years old and dates back to the two (b) 3 4 1 2
agreements of arbitration in 1992 and 1924 between (c) 1 3 2 4
then Madras presidency and Mysore. It entailed the (d) 4 1 3 2
principle that the upper riparian state must obtain Ans. (b) : Correct match is-
consent of lower riparian state for any construction State Capital
activity. Assam - Dispur
141. The largest city of Asia is– Mizoram - Aizawl
(a) Shanghai (b) Mumbai Gujarat - Gandhinagar
(c) Osaka (d) Tokyo Manipur - Imphal
Ans. (d): Tokya, Japan is the largest city of Asia 147. Match List-I with List-II and choose the correct
followed by Delhi. answer using the codes given below–
List-I List-II
142. Which of the following States is known as
'Kohinoor of India'? (Old Name) (New Name)
(a) Andhra Pradesh (b) Kerala A. Siyam 1. Taiwan
(c) Madhya Pradesh (d) Rajasthan B. Formosa 2. Myanmar
C. Mesopotamia 3. Thailand
Ans. (a) : Andhra Pradesh is known as Kohinoor of
D. Burma 4. Iraq
India. Kohinoor, the largest diamond of the world, was
mined in the very heart of city of Golconda in Andhra Codes:
Pradesh that may be the reason behind its nickname. A B C D A B C D
(a) 2 1 3 4 (b) 4 2 1 3
143. The Southern Most Point of India is–
(c) 1 3 2 4 (d) 3 1 4 2
(a) Kanyakumari
(b) Rameshvaram Ans. (d) : Correct match is-
(c) Indira Point Old Name New Name
(d) Point Collimar Siyam Thailand
Formosa Taiwan
Ans. (c) : Indira Point is the Southern Most Point of
Mesopotamia Iraq
India. It is a village in the Nicobar district at Great
Nicobar Island. The village lost many of its residents in Burma Myanmar
the 2004 tsunami. 148. The percentage of people living below the
144. Among the following SAARC group Nations, poverty line in India is–
which is the most densely populated country? (a) Approx 26% (b) Approx 30%
(a) Bangladesh (b) India (c) Approx 35% (d) Approx 40%
(c) Pakistan (d) Sri Lanka Ans. (a) : At the time when question was asked, approx
Ans. (a): Bangladesh is most densely populated country 26% population of India was living below poverty line.
among SAARC nations. The South Asian Association Note- Statistics has been changed in recent past due to
for Regional Cooperation (SAARC) was established in Covid Pandemic.
Dhaka on 8 December, 1985. SAARC comprises of 149. Who among the following has been connected
eight member states: Afghanistan, Bangladesh, Bhutan, with Green Revolution in India?
India, Maldives, Nepal, Pakistan and Sri Lanka. The (a) M. S. Swaminathan (b) D. R. Gadgil
Secretariat of the Association is in Kathmandu, Nepal. (c) C. D. Deshmukh (d) C. N. Vakil
145. Which of the following city is not a well planned Ans. (a) : M. S. Swaminathan has been connected with
capital city? Green Revolution in India. He is also called as Father of
(a) Chandigarh Green Revolution in India, for his contributions to the
(b) Bhuvaneshwar development of high-yielding varieties of wheat. He
(c) Bangalore founded the MS Swaminathan Research Foundation, the
(d) Gandhinagar pioneer organization behind the "evergreen revolution".
Ans. (b) : Among the given options, Bhuvneshwar, 150. National Rural Development Institute is located in–
capital of Odisha is not a well planned capital city. (a) New Delhi (b) Mumbai
(c) Kanpur (d) Hyderabad
146. Match List-I with List-II and choose the correct
answer using the codes given below– Ans. (d) : National Rural Development Institute is
located in Rajendra Nagar, Hyderabad, The National
List-I (State) List-II (Capital) Institute of Rural Development and Panchayati Raj
A. Assam 1. Gandhinagar (NIRD & PR), an autonomous organisation under the
B. Mizoram 2. Imphal union ministry of Rural Development, is a premier
C. Gujarat 3. Dispur national centre of excellence in rural development and
D. Manipur 4. Aizawl Panchayati Raj.
UK UDA/LDA (Pre) Exam 2003 313 YCT
CLICK HERE FOR FREE MATERIAL

UTTARAKHAND RO/ARO (Pre) Exam-2006


GENERAL STUDIES
Solved Paper
1. 'Economic survey of India' is published every 5. From which classical dance system Indrani
year by Rahman is related?
(a) Ministry of commerce, Government of India. (a) Kathak (b) Bharatnatyam
(b) Ministry of Finance, Government of India. (c) Odissi (d) Kuchipudi
(c) Ministry of Industry. Government of India. Ans. (b) : Indrani Rahman was a famous classical
(d) Reserve Bank of India. dancer of Bharatnatyam and she popularized it in
Ans. (b) : The Economic Survey is prepared by the western countries.
Economics Division of the Department of Economic 6. In India, Dearness Allowances of employees is
Affairs in the Finance Ministry under the overall provided on the basis of
guidance of the Chief Economic Adviser and is (a) National Income
released after it is approved by the finance minister. The (b) Consumer Price Index
Economic Survey is the flagship annual document of (c) Living Standard
the Finance Ministry. It reviews the economic (d) Per Capita Income
development in India over the past financial year by Ans. (b) : In India, Dearness Allowances of employees
giving detailed statistical data of all the sectors- is provided on the basis of Consumer Price Index. The
industrial, agricultural, manufacturing among others. central government compensates its employees for the
Besides, it analyses the whole macroeconomics of the increased cost of living by providing dearness
country in the past year and provides an outline for the allowance.
next financial year. The first Economic Survey in India 7. Which is called 'Plastic Money'?
was presented in the year 1950-51. (a) Paper Currency (b) Credit Card
2. Mixed economy means. (c) Discount Coupon (d) Shares
(a) existence of both small and large industries Ans. (b) : Plastic money is a term that is used
(b) existence of both private and public sectors predominantly in reference to the hard plastic cards
(c) existence of both primary and secondary used everyday in place of actual bank notes. Debit card,
sectors Credit card are examples of plastic money.
(d) none of these 8. In India 'Teacher's Day' is celebrated on
Ans. (b) : In a mixed economy, the private sector will (a) 8th March (b) 1st May
provide whatever goods and services it can produce th
(c) 14 July (d) 5th September
well and the government will provide essential goods
Ans. (d) : Teachers Day in India is celebrated every
and services which the private sector fails to do. Hence
year on 5th September to commemorate the birth
in a mixed economy, both private and public sector
anniversary of Dr. Sarvepalli Radhakrishnan, first Vice
works together.
President of India. World Teachers’ Day is celebrated
3. The headquarters of World Trade annually on 5 October to celebrate all teachers around
Organization is in the globe.
(a) Montreal (b) Seatle 9. 10th December is celebrated as
(c) Geneva (d) The Hague (a) Human Rights Day (b) World Health Day
Ans. (c) : The headquarters of World Trade (c) U.N. Day (d) Labour Day
Organization (WTO) is in Geneva, Switzerland. The Ans. (a) : 10th December every year is observed
World Trade Organization (WTO) is the only global as International Human Rights Day. On 10th December,
international organization dealing with the rules of trade 1948, United Nations General Assembly
between nations. adopted Universal Declaration of Human
4. The 'Raga' which is sung early in the morning Rights (UDHR).
is 10. The first Indian to win Nobel Prize was
(a) Todi (b) Darbari (a) C.V. Raman
(c) Bhopalia (d) Bhimpalasia (b) Rabindranath Tagore
Ans. (a) : Raga Todi, invented by Main Tansen is sung (c) Hargovind Khurana
early in the morning. (d) Amartya Sen
UK RO-ARO (Pre) Exam-2006 (Ex. Dt. 24.04.2022) 314 YCT
CLICK HERE FOR FREE MATERIAL

Ans. (b): Rabindranath Tagore was the first Indian to Ans. (c) : 'Rath Yatra' at Puri is celebrated in honour of
get a Nobel Prize in 1913 for his work in literature. Lord Jagannath. It is a Hindu festival that is celebrated
11. Meenakshi Temple is situated at at the famous Jagannatha temple every year in Puri,
(a) Chennai (b) Kolkata Odisha.
(c) Madurai (d) Mahabalipuram 19. The 'Bhoodan Movement' made its beginning
Ans. (c): Arulmigu Meenakshi Sundareshwarar Temple in the state of
is a historic Hindu temple located on the southern bank (a) Uttar Pradesh (b) Madhya Pradesh
of the Vaigai River in the temple city of Madurai, Tamil (c) Andhra Pradesh (d) Himachal Pradesh
Nadu. This temple is dedicated to Parvati. Ans. (c) : 'Bhoodan Movement' made its beginning in
12. In which year, the first 'Bharat Ratna' was Andhara Pradesh by Mahatma Gandhi's disciple Vinoba
awarded? Bhave in April 1951. Bhoodan Movement saw
(a) 1951 (b) 1953 landowners gifting land to the landless.
(c) 1954 (d) 1956 20. Pandit Bhimsen Joshi is related to
Ans. (c) : ‘Bharat Ratna’, the highest civilian Award of (a) Astrology (b) Politics
the India was instituted in the year 1954 in 1954 it was (c) Environment (d) Music
given to Servrpalli Radhakrishanan, Chakravardi
Ans. (d): Pandit Bhimsen Joshi was an Indian classical
Rajgopalachari and C.V, Raman.
music singer from kirana gharana.
13. "Devidhura Mela" is held in the District of
21. The author of the book 'LAJJA' is
(a) Pithoragarh (b) Champawat
(a) Sheikh Mujibur Rehman
(c) Haridwer (d) Udhamsingh Nager
(b) Tasleema Nasreen
Ans. (b) : "Devidhura Mela" is held in the District of
(c) Kiran Bedi
Champawat. Stone pelting ritual is performed during
(d) Arundhati Roy
Devidhura mela.
14. Who has been called the 'Martin Luther of Ans. (b) : Taslima Nasrin is the author of the book
India'? Lajja which created lot of controversy.
(a) Raja Rammohan Roy 22. Which of the following is also known as 'Cape
(b) Swami Dayanand Saraswati Comorin'?
(c) Swami Vivekanand (a) Mizoram (b) Kashmir
(d) Swami Shraddhanand (c) Kanyakumari (d) Gujarat
Ans. (b) : Swami Dayanand Saraswati is known as Ans. (c) : Kanyakumari, Tamil Nadu is known as Cape
'Martin Luther of India'. Swami Dayanand Saraswati Comorin. It is a coasted town on India's southern tip.
founder of Arya Samaj, was a renowned scholar who 23. Which State of India has highest coal reserve
promoted the Vedic philosophy and principles of karma with it?
and reincarnation it is work composed in the 12th (a) Chhatisgarh (b) Orissa
century. (c) Bihar (d) Jharkhand
15. Who is the author of 'Geet Govind'? Ans. (d) : In India, Jharkhand has highest coal reserve
(a) Vidyapati (b) Surdas followed by Odisha and Chhattisgarh.
(c) Jayadev (d) Mirabai
24. The famous book, "DAS CAPITAL" was
Ans. (c) : Jayadev is the author of sanskrit poem 'Geet written by
Govind'. (a) Amartya San (b) Karl Marx
16. Who is the author of 'Bharat Bharati'? (c) Gandhiji (d) Jawaharlal Nehru
(a) Rabindranath Tagore Ans. (b) : Karl Marx wrote famous book Das Capital.
(b) Mulkraj Anand
(c) Maithili Sharan Gupta 25. 'Valentines Day' is celebrated each year on
(d) Bankim Chandra (a) 13th February (b) 14th February
th
(c) 15 February (d) 10th February
Ans. (c) : Maithili Sharan Gupta is the author of 'Bharat
Bharati'. Ans. (b) : 'Valentines Day' is celebrated each year on
14th February.
17. Which of the following cities in India is called
the 'pink City'? 26. 'Operation Vijay' was conducted against which
(a) Mysore (b) Jaipur nation?
(c) Chandigarh (d) Srinager (a) China (b) Bangladesh
Ans. (b) : Jaipur is called as 'Pink City' of India as most (c) Nepal (d) Pakistan
of the buildings in the city are coloured in pink. Ans. (d) : India launched 'Operation Vijay' to clear the
18. 'Rath Yatra' at Puri is celebrated in honour of Kargil sector of infiltration by Pakistani soldiers and
(a) Lord Rama (b) Lord Vishnu Kashmiri militants on the Indian side of the Line of
(c) Lord Jagannath (d) Lord Shiva Control.
UK RO-ARO (Pre) Exam-2006 (Ex. Dt. 24.04.2022) 315 YCT
CLICK HERE FOR FREE MATERIAL

27. Who is the first Indian actress to receive the (a) London House
'Padma Shree' Award ? (b) Buckingham Palace
(a) Smita Patil (b) Nargis Dutt (c) 10 Downing Street
(c) Meena Kumari (d) Madhubala (d) None of the above
Ans. (b) : Nargis Dutt was the first Indian actress to Ans. (c) : 10 Downing Street is the official residence
receive the 'Padma Shree' Award in 1958. and the office of the British Prime Minister.
28. The first Prime Minister of Republic of Nepal 35. Aryabhatta was
was (a) Indian Politician
(a) Shri Prachanda (b) Shri koirala (b) Indian Mathematician and Astronomer
(c) Shri Bhatrai (d) Shri Birendra Singh
(c) Indian Sanskrit Scholar and Poet
Ans. (b) : First General Elections were held in 1959 in (d) None of these
Nepal and B.P. Koirala became the first Prime Minister
of Republic of Nepal. Ans. (b) : Aryabhatta (476–550 CE) was the first of
the major mathematician/astronomers from the classical
29. Chinese Parliament is known as
age of Indian mathematics and Indian astronomy.
(a) National Assembly
(b) National Peoples Congress 36. Bhopal Gas, Tragedy (leakage of methyl
(c) The National Parliament of china isocyanate 'MIC') accident happened on
(d) The House of Democracy of China (a) December 2, 1982 (b) December 3, 1985
Ans. (b) : Chinese Parliament is known as National (c) December 3, 1984 (d) December 4, 1986
People's Congress. Ans. (c) : Bhopal Gas Tragedy happened on 3rd
30. Which of the following is the capital of December, 1984 due to the leakage of Poisonous methyl
Jharkhand State? isocyanate chemical from Union Carbide pesticide
(a) Jamshedpur (b) Patna factory, Bhopal, killing more than 15000 people and
(c) Ranchi (d) Dhanbad affected over 600000 workers. Bhopal gas tragedy is
Ans. (c) : Ranchi is the capital of Jharkhand State. known as world's worst industrial disaster.
31. Which of the following is not a member of 37. The first and last Indian Governor-General of
'South Asian Association of Regional Co- India was
operation' (SAARC)? (a) R.M. Gopala
(a) Pakistan (b) Sri Lanka (b) Dr. S. Radhakrishnan
(c) Nepal (d) Thailand (c) C. Rajagopalachari
Ans. (d) : The South Asian Association for Regional (d) Ramanuja Acharya
Cooperation (SAARC) was established on 8 December Ans. (c) : C. Rajagopalachari was the first and last
1985. SAARC comprises of eight Member States Indian Governor-General of independent India.
namely Afghanistan, Bangladesh, Bhutan, India, 38. Which Continent has the highest population
Maldives, Nepal, Pakistan and Sri Lanka. The density?
Secretariat of SAARC is in Kathmandu (Nepal). (a) Asia (b) Europe
32. The Indian Economist, who is the Nobel Prize (c) Africa (d) North America
Winner, is
Ans. (a) : Population of Asia is equivalent to 59.76%
(a) Dr. Manmohan Singh
of the world population. The population density of Asia
(b) Amartya Sen
is 150 people per square kilometer.
(c) Malthus
(d) None of the above 39. First President of United States of America was
(a) George Washington (b) John Adams
Ans. (b) : Indian Economist, Amartya Sen received
Nobel Prize for his contribution to welfare economics. (c) Abraham Lincoln (d) J.F. Kennedy
33. Find out the odd one out from the following: Ans. (a) : On 30 April, 1789, George Washington took
(a) Physiology (b) Psychology oath of the office as the first President of the United
(c) Pathology (d) Bacteriology States of America.
Ans.(b) Physiology is science of life. Pathology is a 40. Who of the following was the U.S President to
branch of medical science primarily concerning the resign Presidency?
cause, origin and nature of disease while Bacteriology is (a) Bill Clinton
a branch of microbiology that is concerned with the (b) Richard Nixon
study of bacteria. Psychology is the scientific study of (c) Geotge W. Bush (Senior)
the mind and behavior. (d) None of the above
34. Among the following which is named the Ans. (b) : Richard Nixon is the only U.S President to
official residence of the British prime Minister? ever resign Presidency as a result of watergate scandal.
UK RO-ARO (Pre) Exam-2006 (Ex. Dt. 24.04.2022) 316 YCT
CLICK HERE FOR FREE MATERIAL

41. Alipore Central Jail is situated at Ans. (c): Tiger, Panthera tigris is national animal of
(a) Mumbai (b) Kolkata India.
(c) Chennai (d) Delhi 49. 'Bihu' is the folk dance of which of the
Ans. (b) : Alipore Central Jail is situated in Kolkata. following States?
42. 'Swaraj' party was formed by (a) Uttar Pradesh (b) Assam
(a) B.G. Tilak and Mahatma Gandhi (c) West Bengal (d) Maharashtra
(b) Bipin Chandra Pal and Lala Lajpat Rai Ans. (b): Bihu is popular folk dance of Assam
(c) C.R. Das and Pt. Motilal Nehru performed during the famous Bihu festival celebrated
(d) Sardar Patel and Rajendra prasad around April.
Ans. (c) : C. R. Das and Motilal Nehru formed the 50. 'World Population Day' is
Swaraj Party within the Congress to advoate for a (a) October 16 (b) July 11
return to council politics. (c) October 3 (d) June 5
43. Who was the first Indian Speaker in the Ans. (b) : World Population Day is celebrated annually
Central Legislative Assembly? on 11 July to focus attention on the urgency and
(a) Hari Singh Gaur importance of population issues. The Day was
(b) Vithal Bhai j. Patel established by the then Governing Council of the UN
(c) Vallabh Bhai J. Patel Development Program (UNDP) in 1989, an outgrowth
(d) Purushottam Das Tandon of the interest generated by the Day of Five Billion,
Ans. (b) : Vithal Bhai j. Patel was the first elected which was observed on 11 July 1987. Theme for World
President of the Central Legislative Assembly in 1925 Population Day 2022 is, "A world of 8 billion"..
and was re-elected in 1927. 51. Barrack Hussein Obama was elected as _____
44. 'The Indian Independence Act' was passed by President of the United States of America.
the British Parliament in (a) 40th (b) 42nd
(a) January, 1947 (b) July, 1947 (c) 44 th
(d) None of these
(c) August, 1947 (d) August, 1946 Ans. (c) : Barack Obama served as the 44th President of
Ans. (b) : The Indian Independence Act received royal the United States of America.
assent on 18 July, 1947. The Indian Independence Act
52. Which one of the following was awarded the
created two new independent Dominions: India and
Rajiv Gandhi 'Khel Ratna' Puraskar for the
Pakistan.
year 2007?
45. Who is the author of the book, "Landmarks in (a) Kapil Dev (b) Anil Kumble
Indian Constitutional and National (c) Abhinav Bindra (d) M.S Dhoni
Development"?
(a) Bipin Chandra Ans. (d) : M.S Dhoni was awarded Rajiv Gandhi
(b) Gurumukh Nihal Singh 'Khel Ratna' Puraskar (award) for the year 2007.
(c) B. R. Nanda 53. The inventor of Yoga was
(d) Ram Gopal (a) Aryabhatta (b) Charak
Ans. (b) : Gurumukh Nihal Singh authored the book (c) Patanjali (d) Ramdeva
"Landmarks in Indian Constitutional and National Ans. (c) : Maharshi Patanjali systematized and
Development". codified the then existing practices of Yoga, its meaning
46. The Editor of "Congress Presidential and its related knowledge through his Yoga Sutras.
Addresses" was 54. Which state is known as "Silicon state" of
(a) G.A. Natesan India?
(b) B.P. Sitaramayya (a) Goa (b) Andhra Pradesh
(c) Ramanand Chatterjee (c) Karnataka (d) Kerala
(d) H.N. Mitra Ans. (c) : Karnataka is known as "Silicon state" of
Ans. (a) : G.A. Natesan was the Editor of "Congress India. Bengaluru, capital of Karnataka is one of the
Presidential Addresses" while B.P. Sitaramayya wrote major software centers of the world. All the leading
the history of the Congress. software companies have offices here.
47. Which is the National Bird of India? 55. Deficiency of Iron causes
(a) Peacock (b) Duck (a) Phylaria (b) Malaria
(c) Parrot (d) Pigeon (c) Anemia (d) Fluorosis
Ans. (a) : The Indian peacock, Pavo cristatus is the Ans. (c) : Deficiency of Iron causes Anemia. Common
national bird of India. causes of iron deficiency include not getting enough
48. India's National Animal is iron in diet. Chronic blood loss, pregnancy or blood loss
(a) Deer (b) Elephant due to mensuration and internal bleeding, inability to
(c) Tiger (d) Lion absorb iron. endometriosis and genetics.
UK RO-ARO (Pre) Exam-2006 (Ex. Dt. 24.04.2022) 317 YCT
CLICK HERE FOR FREE MATERIAL

56. If the lens in eye becomes opaque, the disease is (c) Dr. K.S Krishna
called - (d) None of the above
(a) Myopia (b) Astigmatism Ans. (b) : Homi J. Bhabha is known as the Father of
(c) Glaucoma (d) Cataract Atomic Energy in India. Bhabha set up the Atomic
Ans. (d) : Cataracts are a degenerative form of eye Energy Commission in 1948, with him as its first
disease in which the lens gradually becomes opaque and director. He was awarded the Padma Bhushan in 1954
vision mists over. This causes partial or complete loss for his outstanding contributions to nuclear science.
of vision. 64. Who invented Steam Engine?
57. The universal law of gravitation was (a) Robert Watson (b) James Watt
propounded by (c) William Harway (d) None of the above
(a) Kepler (b) Galileo Ans. (b) : James Watt invented steam engine. James
(c) Newton (d) Copernicus Watt patented steam engine in 1769.
Ans. (c) : The universal law of gravitation was 65. The Indian Rice Research Institute is located in
propounded by Newton. It states that every object in the (a) Cuttack (b) Kolkata
universe attracts every other object with a force, which (c) Trivandrum (d) Mumbai
is proportional to the product of their masses and Ans. (c) : The Central Rice Research Institute is
inversely proportional to the square of the distance situated Near vidyadharpur Village on the Cuttack-
between them. Paradeep road, Odisha. It is one of the premier national
58. 'World Environment Day' is celebrated on research institutes under the Indian Council of
which of the following day? Agricultural Research.
(a) 5th October (b) 5th August 66. Which of the following converts milk into
th
(c) 5 July (d) 5th June curd?
Ans. (d) : In 1972, the UN General Assembly (a) Bacteria (b) vitamin
designated 5 June as World Environment Day. It is the (c) Enzyme (d) None of the above
United Nations flagship day for promoting worldwide Ans. (a) : Lactobacillus bacteria converts milk to curd
awareness and action for the environment. Theme for by producing lactic acid and making the taste of the
World Environment Day, 2022 is 'Only One Earth'. curd sour.
59. Which is the densely populated city of India? 67. Cause of 'AIDS' is
(a) Kolkata (b) Chennai (a) Bacteria (b) Protozoa
(c) Mumbai (d) Delhi (c) Virus (d) Fungus
Ans. (c) : Among the given options, Mumbai is the Ans. (c) : Acquired Immuno Deficiency Syndrome
most densely populated city of India. (AIDS) is a chronic, potentially life threatening
60. Among the following which fruit is a rich condition caused by the Human Immuno Deficiency
source of iron? Virus (HIV). By damaging immune system, HIV
(a) Jamun (b) Karonda interferes with body's ability to fight infection and
(c) Loquat (d) Guava disease.
Ans. (b) : Among the given options, Karonda is a rich 68. Night blindness is caused due to the deficiency
source of iron. It also contains a fair amount of of
Vitamin C. (a) Vitamin B12 (b) Vitamin A
61. Quinine, used to treat Malaria is obtained from (c) Vitamin C (d) Vitamin E
which part of the plant? Ans. (b) : Vitamin A deficiency can lead to night
(a) Leaf (b) Root blindness. Vitamin A, also called retinol, plays a role in
(c) Bark (d) Flower transforming nerve impulses into images in the retina.
Ans. (c) : Quinine, as a component of the bark of 69. The gas used to inflate the tyres of an aircraft is
the cinchona tree is used to treat malaria. (a) Hydrogen (b) Nitrogen
62. The richest source of Vitamin 'C' is (c) Helium (d) Neon
(a) Potato (b) Sugarcane Ans. (b) : Nitrogen gas used to inflate the tyres of an
(c) Orange (d) Sugar beet aircraft as nitrogen does not support combustion like air
Ans. (c): Orange is richest source of Vitamin C. Oranges which has oxygen and nitrogen does not corrode the
contain 53 mg of vitamin C per 100 grams. One medium wheels of aircraft.
orange delivers 70 mg of vitamin C. 70. 'SILICOSIS' is a
63. Who is called the father of Atomic Energy in (a) kidney disease
India? (b) liver disease
(a) Prof. Satish Dhawan (c) lung disease
(b) Homi J. Bhabha (d) neurological disorder
UK RO-ARO (Pre) Exam-2006 (Ex. Dt. 24.04.2022) 318 YCT
CLICK HERE FOR FREE MATERIAL

Ans. (c): Silicosis is a long term lung disease caused by Ans. (b) : The Nobel Prize in Physics 1930 was
inhaling large amounts of crystalline silica dust, usually awarded to Sir Chandrasekhara Venkata Raman for his
over many years. Silica is a substance naturally found in work on the scattering of light and for the discovery of
certain types of stone, rock, sand and clay. the effect named Raman effect.
71. 'POLIO' is caused by 78. Who is considered to be the 'Father of
(a) Bacteria (b) Virus Computers'?
(c) Insects or Flies (d) Fungi (a) Bill Gates (b) Charles Babbage
Ans. (b) : Polio is caused by polio virus. The virus (c) Narayan Murthy (d) Tim Berners Lee
spreads from person to person and can infect a person's Ans. (b) : Charles Babbage is considered to be the
spinal cord, causing paralysis. 'Father of Computer'.
72. The main source of Vitamin 'C' is 79. Who among the following was never a
(a) Raw and fresh fruits President of the 'Indian National Congress'?
(b) Milk (a) B.G Tilak
(c) Ghee (b) Badruddin Tyabji
(d) Pulses (c) G.K Gokhale
Ans. (a) : Citrus foods such as orange such are the (d) Subhash Chandra Bose
major source of Vitamin C. Ans. (a) : Among the given options, Bal Gangadhar
73. 'World Computer Literacy Day' is celebrated Tilak was never a President of the Indian National
on Congress. Badruddin Tyabji was elected
the President in Madras session (1887) of Indian
(a) 2nd December (b) 5th July
th National Congress. G.K Gokhale became Congress
(c) 14 November (d) 3rd November
President at its Banaras session in 1905. Subhash
Ans. (a) : Every year 2nd December is observed as Chandra Bose became the President of Indian National
World Computer Literacy Day to create awareness Congress in 1938 and 1939.
about the growing use and importance of
80. 'Harijan' was started by
computers. The renowned Indian computer company,
(a) Tilak (b) Gokhale
National Institute of Information Technology (NIIT),
(c) Gandhiji (d) Naoroji
first celebrated this day in 2001 on the 20th anniversary
of the NIIT's establishment. Theme of World Computer Ans. (c) : Gandhi coined the term 'Harijan' to be used
Literacy Day, 2021 is “Literacy for human-centered for the untouchables. Gandhi edited a weekly
recovery: Narrowing the digital divide." newspaper named Young India. In 1933, he renamed
Young India as Harijan because he believed that the
74. On which principle digital computer works?
campaign to abolish untouchability was as vital as
(a) Calculation (b) Measurement winning political freedom.
(c) Electric (d) Logical
81. The leader of 'Lal Kurti' movement was
Ans. (a) : Digital computer works on the principle of (a) Maulana Azad
calculation by using binary digits 0 and 1. (b) Khan Abdul Gaffar Khan
75. 'MOUSE' is (c) Mohammad Ali Jinnah
(a) Memory (b) C.P.U. (d) Iqbal
(c) Input Device (d) Output Device Ans. (b) : The leader of 'Lal Kurti' movement was Khan
Ans. (c) : A mouse is a small hardware input device Abdul Gaffar Khan also known as Frontier Gandhi.
used by hand. It controls the movement of the cursor on 82. 'Nehru Report' was prepared by
the computer screen and allows users to move and (a) M.L. Nehru (b) J. Nehru
select folders, text, files and icons on a computer.
(c) R. K Nehru (d) B. L. Nehru
76. In computer terminology U.S.B. stands for
Ans. (a) : M.L. Nehru prepared Nehru Report. It was
(a) Universal Serial Bus
first ever attempt by the Indians to draft its own
(b) Universal set bus Constitution. In December 1927, at its Madras session,
(c) Unified serial bus the Indian National Congress decided to set up an All
(d) Undefined serial Bus Parties Conference to draft a Constitution for India.
Ans. (a) : U.S.B. stands for Universal Serial Bus. USB Some of the notable members of this Committee were:
port is used to connect various peripheral devices such Motilal Nehru (Chairman), Sir Ali Imam, Tej Bahadur
as speaker, camera, scanner, pen drive etc. Sapru and Subash Chandra Bose. M.R. Jayakar and
77. An eminent Indian physicist Dr. C. V. Raman Annie Besant joined the Committee later. Jawaharlal
was given Nobel prize for his work in the Nehru was appointed the secretary to the Committee.
(a) 1925 (b) 1930 83. Who was the first Muslim President of the
(c) 1935 (d) None of these Indian National Congress ?
UK RO-ARO (Pre) Exam-2006 (Ex. Dt. 24.04.2022) 319 YCT
CLICK HERE FOR FREE MATERIAL

(a) Badruddin Tyabji (b) Abul Kalam Azad 90. Dr. Ambedkar and Gandhiji had a pact called
(c) Rafi Ahmed Kidwai (d) M. A. Ansari (a) Calcutta Pact (b) London Pact
Ans. (a) : First Muslim president of the Indian National (c) Poona Pact (d) Lahore Pact
Congress party was Badruddin Tyaji. He was Ans. (c) : Dr. B.R.Ambedkar demanded seperate
unanimously elected President of third congress session electorates for dalits during second round table
in Madras in 1887. confrerence in London. When the British government
84. In 1906 Muslim League was founded at conceded Ambedkar’s demand, Gandhiji began a fast
(a) Lahore (b) Delhi unto death. He believed that separate electorates for
(c) Calcutta (d) Dhaka dalits would slow down the process of their integration
Ans. (d) : In 1906 Muslim League was founded at into society. Ambedkar ultimately accepted Gandhiji’s
Dhaka (Now Bangladesh) by Aga Khan to protect the position and the result was the Poona Pact of September
interest of Muslims in India. 1932. It gave the Depressed Classes reserved seats in
85. Who was the speaker of the first Lok sabha? provincial and central legislative council, but they were
(a) R. Venkataraman to be voted in by the general electorate.
(b) Y. B. Chavan 91. Delhi became the capital of India in
(c) Hukam Singh (a) 1910 (b) 1911
(d) G. V. Mavalankar (c) 1916 (d) 1923
Ans. (d) : G. V. Mavalankar was the first Speaker of the Ans. (b) : On 12th December, 1911, at the historic
Lok Sabha (1952 to 1956). Delhi Durbar, George V, then Emperor of the British
86. 'Harijan Sewak Sangh' was organized by Empire proclaimed the shifting of the capital of India
(a) Raja Rammohan Roy from Calcutta to Delhi.
(b) Dr. B.R. Ambedkar 92. 'Satyarth Prakash' is a sacred book of
(c) Indian Socialist (a) Arya Samaj
(d) Comrade (b) Brahmo samaj
Ans. (*) : 'Harijan Sewak Sangh' was organized by (c) Theosophical society
Gandhi to counter Communal Award made by British (d) Prarthana samaj
governement to provide seperate electorate for the Ans. (a) : 'Satyarth Prakash' (Light of Truth) is a sacred
depressed class. book of Arya Samaj written by its founder Swami
87. Which one of the following Journals was Dayanand Saraswati.
brought by Maulana Abul Kalam Azad? 93. Which of the following started during the
(a) Al-Hilal (b) Zamindar Rajput period?
(c) Indian Socialist (d) Comrade (a) Sati (b) Child-marriage
Ans. (a) : Al-Hilal was the news paper published by (c) Juhar (d) None of these
Maulana Abul Kalam Azad to critisize British rule in
Ans. (c) : Johar custom started during the Rajput
India.
period. Jauhar is generally viewed as the Hindu custom
88. Who was the founder of British Empire in of mass self-immolation by women in parts of the India
India? to avoid capture, enslavement and rape by any foreign
(a) Warren Hastings invaders, when facing certain defeat in a war.
(b) Lord Amherst
94. Arrange the following in a chronological order:
(c) Lord Robert Clive
1. Tulsidas
(d) Lord William Bentinck
2. Raja Rammohan Roy
Ans. (c) : Lord Robert Clive is considered to be the
3. Swami vivekanand
founder of British Empire in India. Some of his major
achievents are defence of the Carnatic’s capital of Arcot 4. Dayanand Saraswati
in October 1751 against the French. He commanded the (a) 1, 2, 3, 4
expedition sent to recover Calcutta from Nawab Siraj- (b) 1, 2, 4, 3
Ud-Daulah in December 1756. Clive’s greatest victory (c) 2, 1, 3, 4
was at Plassey on 23 June 1757. (d) 2, 3, 4, 1
89. To whom was the title of "Punjab kesari" Ans. (b) : Tulsidas- 1532 AD
conferred? Raja Rammohan Roy- 1772 AD
(a) Bhagat Singh (b) Ranjeet Singh Dayanand Saraswati- 1824 AD
(c) Lala Lajpat Rai (d) Lala Hardayal Swami vivekanand- 1863 AD
Ans. (c) : Lala Lajpat Rai was conferred with the title 95. Which is the oldest Veda?
of Punjab Kesari for his contribution to the freedom (a) Atharva veda (b) Rig veda
struggle. (c) Sama veda (d) Yajur vada
UK RO-ARO (Pre) Exam-2006 (Ex. Dt. 24.04.2022) 320 YCT
CLICK HERE FOR FREE MATERIAL

Ans. (b): The Rig Veda is the earliest of the four Vedas Ans. (d) : Under the Indian Constitution, Supreme
and one of the most important texts of the Hindu Court is the may Guardian of the Fundamental Rights.
tradition. It is a large collection of hymns in praise of Article 32 deals with the ‘Right to Constitutional
the Gods, which are chanted in various rituals. Remedies’, or affirms the right to move the Supreme
96. Given below are the names of freedom fighters Court by appropriate proceedings for the enforcement
and the newspapers started by them. Among of the rights conferred in Part III of the Constitution
these which combination is incorrect ? which includes Fundamental Rights.
(a) Maulana Azad – Al-Hilal 103. In which year the Indian States were
(b) Lokmanya Tilak – Kesari reorganized on the recommendation of states'
(c) Jawaharlal Nehru – National Herald Reorganization Commission?
(a) 1947 (b) 1951
(d) Mahatma Gandhi – The Pioneer
(c) 1956 (d) 1966
Ans. (d) : Mahatma Gandhi started four publications
Ans. (c) : The States Reorganisation Act, 1956, was
namely Indian Opinion, Young India, Navjivan and
enacted by the Parliament under Article 4 of the
Harijan. Rest are correctly matched.
Constitution of India. The new States formed because of
97. Sharda Act relates to the reorganisation of States in 1956 are Andhra Pradesh,
(a) Prevention of Child marriage Bombay Kerala, Madhya Pradesh, Madras, Mysore,
(b) Prevention of Inter-caste marriage Punjab and Rajasthan.
(c) Prevention of widow re-marriage
104. In which language was the 'Shriman Bhagavad
(d) Prevention of Tribal marriage
Gita' originally written?
Ans. (a) : The Child Marriage Restraint Act, 1929 also (a) Sanskrit (b) Urdu
known as Sharda Act was related to prohibiting marriage (c) Pali (d) Hindi
for girls under the age of 14 and boys under 18.
Ans. (a) : 'Shriman Bhagavad Gita' was originally
98. In which city the first atomic bomb was written in Sanskrit language. It consists of 700 verses in
exploded?
18 chapter in total.
(a) Hiroshima (Japan) (b) Kabul
(c) Lisbon (d) Tome 105. Who among the following was the
Constitutional Advisor to the Constituent
Ans. (a) : On August 6, 1945, the United States
Assembly?
dropped an atomic bomb on the city of Hiroshima
(a) Dr. B.R. Ambedkar
Japan). The bomb was known as "Little Boy", a
uranium gun-type bomb that exploded with about (b) K.M. Munshi
thirteen kilo tons of force. (c) B.N. Rau
(d) T.T. Krishnamachari
99. The Battle of Plassey was fought in the year
(a) 1761 (b) 1757 Ans. (c) : B.N. Rau was appointed as Constitutional
(c) 1760 (d) 1764 Advisor to the Constituent Assembly headed by Dr.
B.R. Ambedkar.
Ans. (b) : The Battle of Plassey was fought on June 23,
1757 between British East India Company and Nawab 106. The Article of Indian Constitution related to
of Bengal Siraj Ud Daulah. It is considered to be first abolition of untouchability is
major victory of the East India Company in India. (a) Article 18 (b) Article 17
100. When was the first amendment to the Indian (c) Article 16 (d) Article 15
Constitution carried out? Ans. (b) : Article 17 of the Constitution is related to
(a) 1950 (b) 1951 abolition of untouchability.
(c) 1955 (d) 1958 107. The ex-officio-Chairman of planning
Ans. (b) : The Constitution of India was first amended Commission is
in 1951 for the welfare of Schedule Caste, Schedule (a) President (b) Finance Minister
Tribes and Backward classes. (c) Vice-President (d) Prime Minister
101. What is the minimum age requirement to Ans. (d) : The ex-officio-Chairman of planning
become the prime Minister of India? Commission was Prime Minister. National Institution
(a) 21 years (b) 25 years for Transforming India (NITI Aayog), formed on 1
(c) 30 years (d) 35 years January 2015, replaced Planning Commission.
Ans. (b) : Minimum age requirement to become the 108. The term of a State Legislative Assembly can
Prime Minister of India is 25 years. be extended during Emergency by
102. Under the Indian Constitution who is the (a) The President
Guardian of the Fundamental Rights? (b) The Parliament
(a) Parliament (b) President (c) The Governor of State
(c) Cabinet (d) Supreme Court (d) The State Legislature
UK RO-ARO (Pre) Exam-2006 (Ex. Dt. 24.04.2022) 321 YCT
CLICK HERE FOR FREE MATERIAL

Ans. (b):According to Article 172(1) of the Constituion, Ans. (b) : Tourism industry makes the highest
while a Proclamation of Emergency is in operation, contribution to the income of Uttarakhand.
duration of a state legislative Assembly be extended by 117. which president of India was called the 'Missile
Parliament by law for a period not exceeding one year Man'?
at a time and not extending in any case beyond a period (a) Dr. Radhakrishnan
of six months after the Proclamation has ceased to (b) Dr. A.P.J. Abdul Kalam
operate.
(c) Dr. Shankar Dayal Sharma
109. There were originally how many articles in the (d) None of the above
Constitution of India?
Ans. (b) : Dr. A.P.J. Abdul Kalam, 11th President of
(a) 420 (b) 380
India (2002-2007) was called the Missile Man for his
(c) 395 (d) 270
contribution towards developing missile and nuclear
Ans. (c) : The original text of the Constitution weapons.
contained 395 articles in 22 parts and eight Schedules.
118. The total number of Districts in Uttarakhand is
110. The term 'Socialist' was added in the preamble (a) 12 (b) 13
by the
(c) 14 (d) 15
(a) 42nd Amendment
(b) 44th Amendment Ans. (b) : There are 13 districts in Uttarakhand which
(c) 52nd Amendment are grouped into two divisions namely Kumaun and
(d) None of the above Garhwal.
Ans. (a) : The term 'Socialist' was added to the 119. Name the only personality from Uttarakhand
Preamble by the 42nd Amendment, 1976. who was conferred with the 'Bharat Ratna'?
(a) Bhimsen Joshi
111. Under which Article of the Indian Constitution
(b) Govind Ballabh pant
President's Rule can be imposed in a state?
(a) Article 370 (b) Article 368 (c) General B.C. Joshi
(c) Article 356 (d) Article 352 (d) H.N. Bahuguna
Ans. (c) : The provision of President Rule in a State is Ans. (b) : Govind Ballabh pant is the only personality
being provided in Article 356 of the Constitution. from Uttarakhand till date, who was conferred with the
'Bharat Ratna' in the year 1957.
112. Who presides over the joint session of both
Houses of Parliament in India? 120. Name the only player originally from
(a) President Uttarakhand to have played test cricket.
(b) Prime Minister (a) Mahendra singh Dhoni
(c) Speaker of the Lok Sabha (b) Sunil Joshi
(d) Vice-President (c) Raghuram Bhatt
Ans. (c) : Speaker of the Lok Sabha presides over the (d) Amit Bhandari
joint session of both Houses of Parliament in India. Ans. (a) : Mahendra singh Dhoni, who origanally
113. Where the High Court of Uttarakhand is belongs to Uttarakhand, then shifted to Ranchi, is the
situated? player who has played test cricket.
(a) Dehradun (b) Almora 121. Which one of the following temples is one of the
(c) Nauital (d) srinagar 'Twelve Jyotirlingas'?
Ans. (c) : The High Court of Uttarakhand is situated at (a) Bagnath (b) Tungnath
Nainital. (c) Kalpeshwar (d) Kedarnath
114. In which district of Uttarkhand is the temple of Ans. (d) : Kedarnath is one of the 'Twelve
Kedarnath situated? Jyotirlingas'.
(a) Chamoli (b) Rudraprayag 122. Who was the first chief Minister of
(c) Uttarkashi (d) tehri Uttarakhand?
Ans. (b) : Kedarnath temple is situated at Rudraprayag (a) Narain Dutt Tiwari
district of Uttarakhand. (b) Nityanand Swami
115. Where is the forest research Institute located? (c) Bhagat Singh Khoshiari
(a) Delhi (b) Bhopal (d) None of the above
(c) Dehradun (d) Lucknow Ans. (b) : Nityanand Swami (2000-2001) was the first
Ans. (c) : forest research Institute is situated in chief Minister of Uttarakhand.
Dehradun. It was established in 1906. 123. The total area of Uttarakhand is
116. Which industry makes the highest contribution (a) 60, 480 square km.
to the income of Uttarakhand? (b) 53, 483 square km.
(a) Electricity (b) Tourism (c) 55, 483 square km.
(c) Forest (d) Transport (d) 65, 480 square km.
UK RO-ARO (Pre) Exam-2006 (Ex. Dt. 24.04.2022) 322 YCT
CLICK HERE FOR FREE MATERIAL

Ans. (b): The geographical area of Uttarakhand is 130. Which State University of Uttarakhand has
53483 sq. km. Uttarakhand State comprises of 13 been granted the status of Central University ?
districts, 78 Tehsils and 95 community development (a) Doon University
blocks. (b) Kumaun University
124. Name the only person from Uttarakhand to (c) H.N.B. Grahwal University
have received the 'Gyanpeeth Award' for (d) G.B. Pant Agriculture University
literature. Ans. (c) : Hemvati Nandan Bahuguna (H.N.B) Garhwal
(a) Leela bhar Jaguri University was established as a State University on 23
(b) Ila Chandra Joshi November, 1973. The university was converted to a
(c) Viren Dangwal Central University on 15th January, 2009 by an Act of
(d) Sumitra Nandan Pant Parliament.
Ans. (d) : Sumitra Nandan Pant is the only person from 131. How many states are there in India?
Uttarakhand to have received the 'Gyanpeeth Award' for (a) 25 (b) 27
literature. (c) 28 (d) 29
125. When was the first General Election held in Ans. (d) : When question was asked there were 29
India for Lok sabha? states but at present there are 28 States and 8 Union
(a) 1947 (b) 1948 Territories in India.
(c) 1949 (d) 1952 132. The Supreme Court consists of a chief justice
Ans. (d) : The first general elections of India started on and
25th October 1951 and continued till 21st February (a) Seven Judges (b) Nine judges
1952. (c) Eleven judges (d) Twenty five judges
126. 'Bonded Labour Abolition Act' was passed in Ans. (d) : At time when question was asked judges
(a) 1972 (b) 1976 number was 25 but present, Supreme Court consist of
(c) 1982 (d) 1948 34 Judges including Chief Justice.
Ans. (b) : 'Bonded Labour Abolition Act' was passed in 133. By which amendment the fundamental duties
1976. The Act together with the judgments of the of citizens were incorporated in the Indian
constitution?
Supreme Court has laid the framework for eliminating
and eradicating the Bonded Labour System in India. (a) 38th (b) 41st
nd
(c) 42 (d) 45th
127. How many seats have been reserved for the
Union Territories in the Lok Sabha? Ans. (c) : The Fundamental Duties of citizens were
added to the Constitution by the 42nd Amendment in
(a) 20 (b) 25
1976, upon the recommendations of the Swaran Singh
(c) 30 (d) None of the above
Committee. Article 51-A deals with Fundamental
Ans. (a) : 20 seats are reserved for Union Territories in Duties, which contains 10 Fundamental Duties for
the Lok Sabha. Maximum seats in Lok Sabha is 552. citizens.
Currently The Lok Sabha comprises of a total of 545
134. Who was called the 'Grand Old Man' of India?
seats. Out of these, elections is conducted by the
(a) Dada Bhai Naoroji
Election Commission to fill 543 seats. The remaining
(b) Gopal krishna Gokhale
two seats are filled by nomination of representatives of
(c) Ramesh Chandra Bannerji
the Anglo-Indian Community if the President feels that
(d) Sir Syed Ahmad Khan
this community has not been represented adequately.
Ans. (a) : Dada Bhai Naoroji was called the 'Grand
128. The Chief Election Commissioner of India is Old Man' of India. He was the first Indian to be elected
appointed by to British Parliament.
(a) Lok Sabha (b) Prime Minister
135. What is the main source of political power in
(c) President (d) Chief Justice India?
Ans. (c) : According to Article 324(2) of the (a) The people (b) The Constitution
Constituion, the Chief Election Commissioner of India (c) The Parliament (d) The President
is appointed by the President. Ans. (a) : The people of India is the main source of
129. In which Article the Fundamental right to political power in India.
Freedom of speech and expression is provided 136. Who was the Chairman of 'Constituent
in the constitution of India? Assembly of India?
(a) Article 14 (b) Article 19 (a) Dr. Rajendra Prasad
(c) Article 21 (d) Article 22 (b) Dr. B.R. Ambedkar
Ans. (b) : Article 19 of the Constitution is regarding (c) Mr. Ayyar
freedom of speech and expression. (d) Pandit Jawaharlal Nahru
UK RO-ARO (Pre) Exam-2006 (Ex. Dt. 24.04.2022) 323 YCT
CLICK HERE FOR FREE MATERIAL

Ans. (a): The Constitution was framed by the 144. The President of India can nominate two
Constituent Assembly of India, established by the members in the Lok Sabha to give
members of the provincial assemblies elected by the representation
people of India. Dr Sachchidanand Sinha was the first (a) Indian Christians (b) Anglo-Indians
President of the Constituent Assembly. Later, Dr (c) Buddhists (d) Parsis
Rajendra Prasad was elected its President. Ans. (b) : The President of India can nominate two
137. The elections to Panchayats are to be held in members in the Lok Sabha to give representation to the
(a) every four years Anglo-Indian community if in his opinion that the
(b) every five years community is not adequately represented in the House.
(c) every six years
145. In which state 'Nanda Devi Mahotsava' is
(d) at the Government's discretion
celebrated?
Ans. (b) : Panchayat elections to be conducted every (a) Uttarakhand (b) Manipur
five years under the overall superintendence, direction
(c) Rajasthan (d) Bihar
and control of the State Election Commission.
Ans. (a) : 7 day 'Nanda Devi Mahotsava' is celebrated
138. Which one of the follwing prime ministers of
in Uttarakhand.
India, never attended the parliament during his
tenure? 146. 'Pongal' is the festival of which state?
(a) Atal Bihari Vajpayee (a) Andhra Pradesh (b) Tamil Nadu
(b) V.P. Singh (c) Maharashtra (d) Kerala
(c) Chandra Shekhar Ans. (b) : Pongal is a harvest festival celebrated by the
(d) Chaudhari Charan Singh Tamil community. It is a celebration to thank the Sun,
Ans. (d) : Chaudhari Charan Singh, Prime Minister of Mother Nature and the various farm animals that help to
India from 1979 to 1980 never attended the parliament contribute to a bountiful harvest. Celebrated over four
during his tenure. days, Pongal also marks the beginning of the Tamil
139. The World Bank was established in month called Thai, which is considered an auspicious
(a) 1920 (b) 1945 month.
(c) 1950 (d) 1935 147. Mimansa was initiated by
Ans. (*) : The World Bank previously known as (a) Kansd (b) Vasistha
International Bank for Reconstruction and (c) Vishwamitra (d) Jaimini
Developement was established in 1944. Ans. (d) : Mimansa, sanskrit word meaning 'revered
140. Nationalisation of Banks in India took place in thought' was initiated by Jaimini. Mimarsa is one of the
(a) 1967 (b) 1969 six orthodox schools of Hindu philosophy.
(c) 1970 (d) 1971
148. In which state maximum production of
Ans. (b) : 14 Commercial Banks were nationalized on Soyabean takes place?
19th July, 1969. (a) Uttar Pradesh (b) Bihar
141. From which year sixth pay commission report (c) Madhya Pradesh (d) Rajasthan
has been implemented?
Ans. (c) : Madhya Pradesh is the largest producer of
(a) 2006 (b) 2005
Soyabean in India followed by Maharashtra. When the
(c) 2007 (d) 2008
question was asked. But recently Mahrashtra became
Ans. (d):Sixth pay commission report was implemented
largest Producer followed by madhya Pradesh.
in 2008.
149. The book, "Ramcharit Manas" was written by
142. "Brahmo Samaj" was established by
(a) Tulsidas (b) Valmiki
(a) Raja Rammohan Roy
(b) Jawaharlal Nehru (c) Surdas (d) Ved Vyas
(c) Sankaracharya Ans. (a) : "Ramcharit Manas" was written by Tulsidas.
(d) Chandra Dev It is one of the most important epic of Hindu religion.
Ans. (a) : "Brahmo Samaj" was established by Raja 150. According to Hindu Mythology, which serpent
Rammohan Roy in Calcutta in 1828. The Brahmo offered himself as a rope for churning the
Samaj was a monotheistic sect of Hinduism. ocean?
143. Which state enjoys the distinction of first being (a) Kalia (b) Vasuki
created on linguistic basis in India? (c) Pushkar (d) Sheshnag
(a) West Bengal (b) Punjab Ans. (b) : According to Hindu Mythology, Vasuki
(c) Punjab (d) Andhra Pradesh offered himself as a rope for churning the Ocean.
Ans. (d) : Andhra Pradesh was the first State being Vasuki the serpent king that Loils around the neck of
created on linguistic basis in India on 1st October, 1953. Lord Shiva is worshipped on Naga Panchami.
UK RO-ARO (Pre) Exam-2006 (Ex. Dt. 24.04.2022) 324 YCT
CLICK HERE FOR FREE MATERIAL

UTTARAKHAND RO/ARO (Mains) Exam-2006


GENERAL STUDIES
Solved Paper
1. Which country won the first World-Cup of (c) Shah Hussain Sharqi
Women's Twenty-20 Cricket? (d) Akbar
(a) New Zealand (b) England Ans. (d) : Akbar abolished the tax on pilgrimage in
(c) India (d) Sri Lanka called as Jazia in 1564 as two were based on religious
Ans. (b) : The first edition of Women's Twenty-20 discrimination. Jazia was the tax, which was levied on
Cricket World Cup held in 2009, was won by the non-muslims.
hosts England. They defeated New Zealand by six 7. In which Puran do we find the longest list of
wickets in the final. the Andhra-Satavahana Kings?
2. Who is the author of "The Audacity of Hope"? (a) Vayu Puran (b) Vishnu Puran
(a) Al Gore (b) Barack Obama (c) Matsya Puran (d) None of the above
(c) Hillary Clinton (d) Bill Clinton Ans. (c) : The longest list of the Andhra-Satavahana
Ans. (b) : The author of the book "The Audacity of Kings is found in Matsya Puran. The Matsya Purana is
Hope" is US former President Barack Obama. The book one of the eighteen Mahapuranas and among the oldest
divided in to nine chapter, outline Obama political and and better preserved in Sanskrit literature.
spiritual beliefs, as well as his opinions on different 8. The empire of which of the following dynasties
aspects of American culture. extended beyond India?
3. Which is the first Indian State whose all (a) Gupta Dynasty (b) Maurya Dynasty
districts are connected with the State capital (c) Kushana Dynasty (d) None of the above
with fibre-optical network? Ans. (c) : Empire of Kushana dynasty extended beyond
(a) Uttar Pradesh (b) Bihar India. Kushana controlled Silk Route and ruled central
(c) Andhra Pradesh (d) Uttarakhand Asia and north-west India. During their rule, a branch of
Ans. (c) : Andhra Pradesh is the first Indian State the Silk Route extended from Central Asia down to the
whose all districts are connected with the State capital seaports at the mouth of the river Indus, from where silk
with fibre-optical network for better governance. was shipped westwards to the Roman Empire.
4. Where was the first summit of BRIC countries 9. The Third Battle of Panipat was fought in the
held in June 2009? year
(a) India (a) 1526 A.D. (b) 1761 A.D.
(b) Brazil (c) 1556A.D. (d) 1857A.D.
(c) Yekaterinberg (Russia) Ans. (b) : The Third Battle of Panipat took place on 14
(d) China January 1761, at Panipat between a northern
Ans.(c):First BRIC summit was held in Yekaterinburg, expeditionary force of the Maratha Empire and a
Russia on 16 June, 2009. BRIC group was renamed as coalition of the King of Afghanistan, Ahmad Shah
BRICS (Brazil, Russia, India, China, South Africa) after Abdali with two Indian Muslim allies namely the
South Africa was accepted as a full member in Rohilla Afghans of the Doab and Shuja-ud-Daula, the
September 2010. BRICKS headquarters is in Shanghai, Nawab of Awadh.
China. 10. To which Buddhist School did Nagarjuna
5. To which Sikh Guru, Akbar donated 500 belong.
Bighas of land? (a) Sautrantika (b) Vaibhashika
(a) Arjun Dev (b) Ramdas (c) Madhyamika (d) Yogachara
(c) Har Rai (d) Teg Bahadur Ans. (c) : Nagarjuna, Buddhist philosopher is founder
Ans. (b) : Akbar donated 500 Bighas of land to Fourth of the Madhyamika (Middle Path) school, an important
Sikh Guru Ramdas (1534-1581). Ramdas founded the tradition of Mahayana sect of Buddhism. He articulated
city of Amritsar upon the land gifted by Akbar. the doctrineof emption (shunyata).
6. Which of the following rulers was the first to 11. Which of the following sites is famous for Pre-
abolish Jazia-Tax? historic paintings?
(a) Zain-UI-Abidin (a) Ajanta (b) Bhimbetka
(b) Mohammad-Bin-Tughlaq (c) Bagh (d) Amravati
UK RO-ARO (Mains) Exam-2006 325 YCT
CLICK HERE FOR FREE MATERIAL

Ans. (b): Bhimbetka, Madhya Pradesh is famous for 17. Who is called the father of 'Indian
Pre-historic paintings. Bhimbetka has around 243 rock Renaissance'?
shelters and have earned the honour of UNESCO World (a) Raja Ram Mohan Rai
Heritage Site in 2003. (b) Dayanand Saraswati
12. The teacher who taught 'Ved-Vedanga' for (c) Swami Vivekanand
livehood, was called (d) Ramkrishmna Paramhansa
(a) Acharya (b) Adhvaryu Ans. (a) : Raja Ram Mohan Rai is called as the father
(c) Upadhyaya (d) Purohita of 'Indian Renaissance'. Raja Ram Mohan Rai was the
founder of Brahmo Sabha in 1828, an influential social-
Ans. (c) : Upadhyaya was the one who adopted religious reform movement. The Brahmo Samaj played
teaching as a profession to earn his livelihood and a major role in reforming and modernizing the Indian
taught only a portion of the Veda or Vedangas. society. Raja Ram Mohan Rai fought to abolish the
Acharya was a type of teacher who taught his pupil practice of Sati. He established a number of schools to
Vedas without charging fee from the pupils. Adhvaryu popularize a modern education system in India. He
is in charge of the execution of the Yiojna. Purohita is campaigned for equal rights for women, including the
the family priest within the Veidc Priesthood. right to remarry and the right to hold property.
13. Which of the following propounded the 'Drain 18. Which of the following countries is credited for
Theory'? establishing 'Uranium City'?
(a) Dada Bhai Naoroji (a) Australia
(b) Gopal Krishna Gokhale (b) Canada
(c) Lokmanya Tilak (c) Russia
(d) Madam Mohan Malviya (d) United States of America
Ans. (a) : DadaBhai Naoroji propounded the 'Drain Ans. (b) : Canada is credited for establishing 'Uranium
Theory'. Through this theory, Naoroji explained how City'. Canada has the world's largest deposits of high-
grade uranium and is second largest producer of
the British rule in India is draining India by taking its
Uranium.
wealth to their country through exploitative economic
politices. 19. Which of the following States has started the
programme 'One kg rice for one rupee'?
14. Who constructed the Ratha Temples at (a) West Bengal (b) Orissa
Mahabalipuram? (c) Kerala (d) Tamil Nadu
(a) Narasinh Varman-I (b) Samudragupta
Ans. (d): Chief Minister of Tamil Nadu, M.
(c) Harsha (d) Pulakesin-II Karunanidhi started the program 'One kg rice for one
Ans. (a) : Ratha Temples at Mahabalipuram was rupee'.
constructed under the patronage of Narasimhavarman 20. At which of the following has the Head-Office
I (630-668 CE). There 'Ratha' temples of of NABARD?
Mahabalipuram are commonly called the Pancha (a) New Delhi (b) Chennai
Pandava Ratha. These temples are pioneer of the (c) Mumbai (d) Jaipur
Dravidian Monolith era. There are actually eight Raths Ans. (c) : Head-Office of NABARD (National Bank for
at Mahabalipuram act of which five are named after five Agriculture and Rural Development) is in Mumbai.
Pandavas of Mahabalipuram and are ofter Draupadi. NABARD came into existence on 12 July 1982. Its
15. The 'Great Bath' was found at the mission is to promote sustainable and equitable agriculture
archaeological site of and rural development through participative financial and
(a) Ropar (b) Harappa non-financial interventions, innovations, technology and
(c) Mohenjodaro (d) Kali Banga institutional development for securing prosperity.
Ans. (c) : Mohenjodaro is a well known city of Indus 21. The committee on whose recommendation, the
Valley Civilization. The 'Great Bath' was found at the 'Panchayati Raj' was introduced in the country
archaeological site of Mohenjodaro. The great public was headed by
bath was surrounded by a courtyard with steps leading (a) Jivraj Mehta (b) Balwant Rai Mehta
down into it. It is also called the "earlist public water (c) Shriman Narayan (d) Jagjivan Ram
tank of the ancient world". Ans. (b) : In 1957, Balwant Rai Mehta Committee
recommended three tier Panchayats Gram Panchayat at
16. 'Indica' was originally written by
village level, Panchayat Samiti at block level and
(a) Nearchus (b) Megasthenes thereafter Zilla Parishad at district level. The
(c) Plutarch (d) Diodorus Constitution 73rd Amendment, 1992 added a new Part
Ans.(b): 'Indica' was originally written by Megasthenes. IX to the constitution titled “The Panchayats” covering
Megasthenes was an ambassador who was sent to the provisions from Article 243 to 243(O) and a new
court of Chandragupta by the Greek ruler Seleucus Eleventh Schedule covering 29 subjects within the
Nicator. functions of the Panchayats.
UK RO-ARO (Mains) Exam-2006 326 YCT
CLICK HERE FOR FREE MATERIAL

22. With which sport is, Abhinav Bindra (a) Jaunsari (b) Bhutia
associated? (c) Boxa (d) Tharu
(a) Polo (b) Golf Ans. (d) : There are five tribes in Uttarakhand. Tharu
(c) Shooting (d) Badminton (33.4%) is the largest tribe of Uttarakhand followed by
Ans. (c) : Abhinav Bindra is associated with shooting. Jaunsari (32.5%), Buksa (18.3%) and Bhotia (14.2%).
At the 2008 Beijing Olympics, shooter Raji is small in number.
Abhinav Bindra made history by winning a gold medal 29. Out of the following, who was awarded
in the 10m Air Rifle event. 'Victoria Cross'?
23. The launching of Community Development (a) Gabar Singh Negi
Programme (on 2nd October, 1952) set the stage (b) Subedar Ram Singh
for (c) General B.C. Joshi
(a) Organisation of economic planning (d) Major Dhan Singh Thapa
(b) Organisation of Panchayati Raj Ans. (a) : Gabar Singh Negi was awarded 'Victoria
(c) Development of SC and ST Cross' from King George V. He was a Rifleman with
(d) Protection of girl child the 2nd Battalion of the 39th Garhwal Rifles during the
Ans. (a) : Community Development Program was First World War.
launched on October 2, 1952 with objective for 30. In which district of Uttarakhand is the 'Valley
economic development, social justice and democratic of flowers' situated?
growth. (a) Uttarkashi (b) Almora
24. "Dulhasti Power Station" is situated on the (c) Tehri (d) Chamoli
river Ans. (d) : 'Valley of flowers' is situated at Chamoli
(a) Beas (b) Chenab district of Uttarakhand. The Valley of Flowers National
(c) Ravi (d) Satluj Park is a UNESCO World Heritage Site.
Ans. (b) : Dulhasti Power Station is situated on the 31. Who in Uttarakhand, was awarded "Shaurya
river Chenab in Kishtwar district of Union Territory of Chakra" in 2009?
Jammu and Kashmir. (a) Subedar Ram Singh
25. What is approximately the population of (b) Major Sunder Singh Bisht
Uttarakhand State? (c) Captain Manoj
(a) 85 lac (b) 70 lac (d) None of these
(c) 50 lac (d) 40 lac Ans. (b) : Major Sunder Singh Bisht , an officer of the
Ans. (a) : As per Census 2011, Uttarakhand has Intelligence Corps in the Indian Army was awarded
population of 1.01 Cr. The population of Uttarakhand "Shaurya Chakra" in 2009.
forms 0.83 percent of India in 2011. When the question 32. Who was the leader of the famous 'Peshawar
was asked the population of Uttarakhand was approx. Kand', against the British Government, in the
85 Lac according to 2001 census. year 1930?
26. Which district of Uttarakhand has the (a) General B.C. Joshi
maximum population? (b) Major Dhan Singh Thapa
(a) Dehradun (b) Pauri (c) Veer Chandra Singh Garhwali
(c) Haridwar (d) Nainital (d) Prem Singh Negi
Ans. (c) : Haridwar (1,890,422) has maximum Ans. (c) : Veer Chandra Singh Garhwali was the
population in Uttarakhand followed by Dehradun leader of the famous 'Peshawar Kand', against the
(1,696,694) and Udham Singh Nagar (1,648,902). British Government, in the year 1930. His refusal to fire
27. Which district of Uttarakhand touches the on unarmed, peaceful civilians averted a massacre and
border of Nepal? sparked off the Peshawar revolt of 1930.
(a) Pauri-Garhwal (b) Uttarkashi 33. Where in Uttarakhand, was a copy of Emperor
(c) Chamoli (d) Pithoragarh Ashok's Rock Edicts discovered?
Ans.(d) : Uttarakhand shares a 350 km border with (a) Nainital
China and a 275 km boundary with Nepal. In the east, (b) Pauri
the districts of Pithoragarh, Champawat and Udham (c) Tehri
Singh Nagar share International boundary with Nepal. (d) Kalsi (Dehradun District)
In the North-West, Uttarkashi, Chamoli and Ans. (d) : Rock edicts of Kalsi erected by King Ashoka
Pithoragarh districts of Uttarakhand share International is located at Kalsi, Dehradun. It was discovered by
boundary with China. Alexander Cunningham.
28. Which tribe has maximum population in 34. Which person from Uttarakhand was awarded
Uttarakhand? 'Padma Vibhusan' in the year 2009?
UK RO-ARO (Mains) Exam-2006 327 YCT
CLICK HERE FOR FREE MATERIAL

(a) Sri Sunder Lal Bahuguna Ans. (b): Mola Ram is associated with Garhwal-
(b) Dr. Ramesh Pokhariyal 'Nishank' Painting. Mola Ram developed a style of painting
(c) Bachendri Pal equaled in romantic charm only by few other styles of
(d) None of these painting.
Ans. (a) : Sri Sunder Lal Bahuguna was awarded 41. 'WADA' is associated with
'Padma Vibhusan' in the year 2009 for his contribution (a) Aviation (b) Dope testing
towards Chipko Movement in 1970. (c) Insurance (d) Stock Exchange
35. The river Yamuna originates from Ans. (b) : The World Anti Doping Agency (WADA)
(a) Chaukhamba (b) Bandarpoonch was established in 1999 as an international independent
(c) Nanda Devi (d) Neelkanth agency composed and funded by the sports movement
Ans. (b) : The Yamuna River originates from the and governments of the world.
Yamunotri Glacier near Bandarpoonch peaks in the 42. The winner of Wimbledon-2009 title is
Mussoorie range of the lower Himalayas at an elevation (a) Roger Federer (b) Rafael Nadal
of about 6,387 meters above the mean sea level in (c) Andy Murray (d) Andy Roddick
district Uttarkashi (Uttarakhand). Ans. (a) : Roger Federer won Wimbledon-2009 title.
36. The founder of Srinagar-Panwar dynasty was Novak Djokovic beat Nick Kyrgies in 2022 Wimbledon
(a) Vijay Pal (b) Prithwi Pati Shah Championship.
(c) Ajay Pal (d) Man Shah 43. Biological Oxygen Demand (BOD) is an
Ans. (c) : The credit for uniting various warlords of indication of pollution in
Garhwal into one entity goes to King Ajay Pal, the ruler (a) Aquatic environment (b) Soil
from the Panwar dynasty who won more than 52 Garhs (c) Air (d) All the above
(forts) and brought them under the rule of the kingdom Ans. (a) : The amount of oxygen required by bacteria to
of Garhwal without disturbing the traditional, social, break down the organic matter present in a certain
political and administrative system. volume of a sample of water is called Biochemical
37. At which of the following places is a famous Oxygen Demand or Biological Oxygen Demand
Sikh Gurudwara located? (BOD). Clean water could have BOD value of less than
(a) Roop Kund (b) Hem Kund 5 ppm whereas highly polluted water could have a BOD
(c) Tara Kund (d) Brahm Kund value of 17 ppm or more.
Ans. (b) : Hem Kund Sahib, among the highest 44. In atmosphere, the highest percentage is of
Gurudwaras in the world, is situated at an altitude of (a) Oxygen (b) Carbon Dioxide
around 4,329 m, on the bank of the pristine Hemkund (c) Nitrogen (d) Hydrogen
lake at the base of the Saptshrungi glacier, Chamoli Ans. (c) : Earth's atmosphere is composed of about 78
district, Uttarakhand. It is recorded in the holy Granth percent nitrogen, 21 percent oxygen, 0.9 percent argon
Sahib that the tenth guru of Sikhs, Guru Govind Singh, and 0.1 percent other gases. Trace amounts of carbon
meditated on the serene banks of Hemkund lake in one dioxide, methane, water vapor and neon are some of the
of his earlier births. It is said Lakshmana, younger other gases that make up the remaining 0.1 percent.
brother of Lord Rama from the epic Ramayana, 45. Which of the following materials has the
regained his health by meditating on the banks of highest electrical conductivity?
Hemkund after the severe injuries he received in the (a) Diamond (b) Silver
war. The Lakshmana temple is said to have been built at (c) Graphite (d) Wood
the place where Lakshmana meditated.
Ans. (b) : Silver has the highest electrical conductivity
38. The winter abode of Kedarnath is of 6.30×107 σ(s/m) at 20ºC. Graphite has 2 to 3×105 and
(a) Rudra Prayag (b) Gupta-kashi Diamond has 10–13 conductivity.
(c) Gopeshwar (d) Ukhi-math
46. Red Blood Corpuscles are formed mainly in
Ans. (d) : The winter abode of Kedarnath is Ukhimath's (a) Liver (b) Kidney
Omkareshwar temple. (c) Heart (d) Bone Marrow
39. The Pauranik name of Kumayun was Ans. (d) : RBCs (Red Blood Corpuscles) are formed in
(a) Kedar Khand (b) Reva Khand the red bone marrow in the adults. A healthy adult man
(c) Hemwat Khand (d) Manas Khand has on an average, 4.7 million to 6.1 million of RBCs
Ans. (d) : As per Skanda Purana, the Pauranik name of mm–3 of blood.
Kumayun was Manas Khand. 47. Source of Energy from the Sun is
40. A famous name associated with Garhwal- (a) Nuclear fission
Painting is (b) Nuclear fusion
(a) Mangat Ram (b) Mola Ram (c) Photo electric effect
(c) Balak Ram (d) Tulsi Ram (d) cherenkov effect
UK RO-ARO (Mains) Exam-2006 328 YCT
CLICK HERE FOR FREE MATERIAL

Ans. (b) : Source of Energy from the Sun is Nuclear (a) Printer (b) Compiler
fusion. Fusion means joining lighter nuclei to make a (c) Mouse (d) Key board
heavier nucleus. It releases a tremendous amount of Ans. (b) : Among the given options, compiler is not a
energy. computer hardware. Compiler is a program, which
48. Which of the following is known as "Dakshin- converts source code into machine code.
Gangotri"? 55. A computer derives its strength from
(a) The place of origin of the river Kaveri (a) Its speed (b) Accuracy
(b) The place where Periyar flows Northwards (c) Memory (d) All of the above
(c) India's first Antarctic Research Station
Ans. (d) : A computer derives its strength from its
(d) The rocket launching centre in Kerala
speed, accuracy and memory. Modern computers have
Ans. (c) : India's research station at Antarctica is named high speed, great accuracy and large memory space.
as "Dakshin-Gangotri" established in 1984. "Dakshin-
Gangotri research station in Antarctica is now non- 56. Written Programmes, due to which computers
operational due to melting of ice. Two other Indian function in the desired way, are called
research stations at Antarctica are Maitri in 1989 and (a) Codes (b) Instructions
Bharati in 2012. (c) Software (d) None of these
49. Chemically dry ice is Ans. (c) : A set of instructions, called software program
(a) Solid sulphur di-oxide directs the computer to complete desired task. Software
(b) Ice formed from distilled water is a set of computer programs and associated
(c) Mixture of ice and common salt documentation and data.
(d) Solid Carbon Di-oxide 57. ROM is a Memory
Ans. (d) : Solid carbon dioxide is known as dry ice. (a) To read only (b) To write only
Solid carbon dioxide gets converted directly to gaseous (c) To read and write (d) None of these
state on decrease of pressure without coming into liquid Ans. (a) : ROM stands for Read Only Memory. ROM is
state. non-volatile memory, means its contents are not lost
50. Aluminum metal is obtained from even when the power is turned off. It is used as a small
(a) Pitch blende (b) Graphite but faster permanent storage for the contents, which are
(c) Bauxite (d) Argentite rarely changed.
Ans. (c) : Aluminum metal is obtained from Bauxite. 58. Modem is a hardware device that interfaces
Bauxite first artificially creates aluminum oxide then it (a) CPU and CRT
is refined by electrolysis to extract aluminum. (b) Printer and Main memory
51. According to weight, the percentage of (c) Telephone line and Computer
hydrogen in water (H2O) is (d) Input and Output devices
(a) 44.45% (b) 5.55%
Ans. (c) : Modem (Modulator/ Demodulator) is a
(c) 88.89% (d) 11.11%
hardware device that interfaces telephone line and
Ans. (d) : According to weight, the percentage of computer. This device converts the digital signal from
Hydrogen in water (H2O) is 11.11% and percentage of the computer into an analogue signal that can travel
oxygen in water is 88.89%. through the telephone line. On the other end, this
52. The most active light for photosynthesis is analogue signal is again converted back to digital form
(a) Violet light (b) Red light by the modem at the destination end.
(c) Blue light (d) Green light
59. LAN means
Ans. (b) : The most effective wavelength for (a) Large Area Network
photosynthesis is red light. Green light is least effective. (b) Local Area Network
Green plants carry out photosynthesis, a physico-
(c) Logical Area Network
chemical process by which they use light energy to
drive the synthesis of organic compounds. (d) None of these
53. Which spice is commonly known as "Black Ans. (b) : LAN stands for Local Area Network. It is a
Gold" of India? network that connects computers, mobile phones, tablet,
(a) Pepper (b) Cardamom mouse, printer, etc., placed at a limited distance.
(c) Clove (d) Saffron 60. Windows operating system was developed by
Ans. (a) : Black Pepper is known as Black Gold of (a) Microsoft (b) I.B.M.
India. In ancient India, Romans bought black Pepper (c) A.T. & T (d) H.P.
from Southern India. They paid for it in gold, which is Ans. (a) : Windows operating system to run personal
why the spice was also called Black Gold. computers was developed by Microsoft Corporation.
54. Which of the following is not a computer Microsoft's Windows operating system was first
hardware? introduced in 1985.
UK RO-ARO (Mains) Exam-2006 329 YCT
CLICK HERE FOR FREE MATERIAL

61. I.R.C. stands for Ans. (b) : Jharkhand is the largest coal producing State
(a) Internet Real time Communication of India followed by Odisha and Chhattisgarh.
(b) Internet Relay Chat In 2020-21, Chhattisgarh registered highest coal
(c) Internet Real time Chat production of 158.409 MT, followed by Odisha 154.150
(d) None of the above MT, Madhya Pradesh 132.531 MT and Jharkhand
Ans. (b) : IRC (Internet Relay Chat), created in 1988, is 119.296 MT.
a protocol for real time text messaging between 69. Which of the following is known as the "Coffee
computers connected through internet. Port" in the world?
62. 1 MB memory is (a) Costa Rica (b) Santos
(a) 1024 Kilo-Bytes (b) 210 Byte (c) Rio-de-Janeiro (d) Buenos Aires
(c) 1024 Kilo-Bits (d) 1000 Kilo-Bytes Ans. (b) : Santos, Brazil is known as the "Coffee Port"
Ans. (a) : 1 MB memory is equal to 1024 Kilo-Bytes. in the world. Brazil is largest coffee producer in the
63. A world-wide-web (www) is a hyper media world.
system because 70. Which of the following is an Island-Continent?
(a) It is hyper fast (a) Africa (b) New Zealand
(b) It links to other computer resources (c) Greenland (d) Australia
(c) It can be used only to bring in video material Ans. (d) : Australia is an island continent as it is also a
(d) None of these country which is surrounded by sea from all four sides.
Ans. (b) : A world-wide-web (www) is a hyper media 71. Which of the following States does not border
system because it links to other computer resources. with Bangladesh?
Web servers and clients that may be located at any part (a) Mizoram (b) Tripura
of the world are connected to each other by
(c) Manipur (d) Meghalaya
telecommunication links through World Wide Web.
Ans. (c) : India shares 4,096.7 Km of its largest land
64. Which of the following State is the leading
border with Bangladesh. West Bengal, Assam,
producer of Soyabean?
Meghalaya, Tripura and Mizoram are the States, which
(a) Madhya Pradesh (b) Maharashtra
share the border with Bangladesh.
(c) Punjab (d) Tamil Nadu
72. In which State is the Lunej petrol producing
Ans. (a) : According to Economic Survey 2021-22
area located?
Maharstra is the leading producer followed by Madhya
Pradesh is the leading producer of Soyabean in India (a) Assam
followed by Maharashtra, Rajasthan and Karnataka. (b) Mumbai-High
(c) Arunachal Pradesh
65. Rajasthan is the chief producer of
(d) Gujarat
(a) Gram (b) Mustard
(c) Cotton (d) Wheat Ans. (d) : ONGC (Oil and Natural Gas Corporation)
discovered its first oil well in Lunej village of Anand
Ans. (b) : Rajasthan is the leading producer of mustard
district of Gujarat.
in India. It contributes 46.06% of total mustard
production in India. 73. With which mineral is the Bailadila-mine
associated?
66. Medagascar is the largest island in
(a) Iron ore (b) Coal
(a) Indian Ocean (b) Pacific Ocean
(c) Manganese ore (d) Mica
(c) Bay of Bengal (d) Red Sea
Ans. (a) : Bailadila mine, Chhattisgarh is associated
Ans. (a) : The Indian Ocean island of Madagascar is the
with iron ore.
leading global producer of vanilla and blessed with a
bountiful biodiversity. It is fourth largest island of the 74. Where is Great-Salt-Lake?
world. It is separated from the African continent by the (a) Iran
Mozambique Channel. (b) U.S.A. (United States of America)
67. Where is Raja Sansi International Airport? (c) Turkey
(a) Hyderabad (b) Nagpur (d) India
(c) Amritsar (d) Chennai Ans. (b) : Great salt lake, west of Mississippi, is in
United States of America. It is the largest salt water lake
Ans. (c) : Raja Sansi International Airport is in
in Western Hemisphere.
Amritsar, Punjab. It is also called Sri Guru Ram Das Jee
airport. 75. Harike Barrage (the source of water for Indira
Gandhi Canal) is at the confluence of the rivers
68. Which of the following is the largest coal
(a) Ravi and Beas
producing State of India?
(b) Jhelum and Chenab
(a) Madhya Pradesh (b) Jharkhand
(c) Chenab and Satluj
(c) Orissa (d) Bihar
(d) Beas and Satluj
UK RO-ARO (Mains) Exam-2006 330 YCT
CLICK HERE FOR FREE MATERIAL

Ans. (d): Harike Barrage came into existence in 1952 82. Name the scheme launched by Central
after the construction of barrage near the confluence of Government for the Girl-child.
rivers Satluj and Beas. (a) Raj-Lakshmi (b) Rani-Bitia
76. Which among the following was the composer (c) Dhan-Lakshmi (d) Raj-Rani
of the song 'Bande Matram'? Ans. (c) : Dhan-Lakshmi scheme was launched by
(a) Bankim Chandra Chatterji Central Government in 2008 to prevent female
(b) Tara Shankar Bandopadhyaya foeticide as well as to boost the level of girl child
(c) Sarat Chandra education.
(d) Rabindra Narth Tagore 83. Which one amongst the following is the oldest
Ans. (a) : Bankim Chandra Chatterji was a Bengali refinery in India?
writer, poet and a journalist who composed India's (a) Haldia (b) Digboi
National Song 'Bande Mataram' during the India's (c) Koyali (d) Mathura
freedom struggle. Ans. (b) : Digboi Refinery, commissioned on 11th
77. Who was the first person to call Mahatma December, 1901, is India's oldest operating refinery and
Gandhi, 'Father of Nation'? one of the oldest operating refineries in the world.
(a) Abul Kalam Azad 84. In the States of India, the State Finance
(b) Jawaharlal Nehru Corporations have given assistance mainly to
(c) Chittaranjan Das develop
(d) Subhash Chandra Bose (a) Agriculture-farms
Ans. (d) : Subhas Chandra Bose called Mahatma (b) Cottage Industry
Gandhi, 'Father of Nation', in a radio address from (c) Large Scale Industry
Singapore in 1944. (d) Medium and Small Scale Industry
78. Which of the following organisations was Ans. (d) : State Financial Corporation Act, 1951
established by Mahatma Jyotiba-Phule? established State Financial Corporations. State Finance
(a) Gopal Mandali Corporations have given assistance mainly to develop
(b) Sri Narayan Sabha Medium and Small Scale Industry (MSME).
(c) Satya Shodhak Samaj 85. The author of the book 'Planning and the
(d) Mahajan Sabha Poor', is
Ans. (c) : Satyashodhak Samaj was a revolutionary (a) D.R. Gadgil (b) B.S. Minhas
organization founded by Jyotiba Phule at Pune in 1873. (c) Charan Singh (d) Rudra Dutt
It was also called as Truth Seekers' Society Ans. (b) : 'Planning and the Poor', on Indian economic
79. Who had expressed the view that 'British policy is authored by B.S. Minhas.
Economic' policy in India is loathsome? 86. Who had advocated the policy of providing
(a) B.G. Tilak (b) Dada Bhai Naoroji 'Urban Amenities' in rural areas?
(c) Karl Marx (d) Adam Smith (a) Dr. A.P.J. Abdul Kalam
Ans. (c) : Karl Marx expressed the view that 'British (b) Dr. Manmohan Singh
Economic' policy in India is loathsome and it will lead (c) Dr. Karan Singh
to revolution. (d) Dr. Montek Singh Ahluwalia
80. Bhagat Singh was executed along with two of Ans. (a) : Former President and Missile Man of India,
his comrades, on 23rd March of 1931. Who Dr. A.P.J. Abdul Kalam advocated the policy of
were those two? providing 'Urban Amenities' in rural areas.
(a) Rajguru and Sukhdev 87. Who was the Chairman of the Twelfth Finance
(b) Sukhdev and B.K. Dutt Commission?
(c) Rajguru and B.K. Dutt (a) C.Rangarajan (b) Rajah Chelliah
(d) Chandra Shekhar Azad and Sukhdev (c) K.C. Pant (d) Y.B. Chavan
Ans. (a) : On March 23, 1931, Bhagat Singh, Rajguru Ans. (a) : The Twelfth Finance Commission was
and Sukhdev were hanged for the killing of British appointed by President on 1st November, 2002 under
officer JP Saunders. the Chairmanship of Dr. C. Rangarajan.
81. The first session of Indian National Congress 88. Allocation of budget for MNREGA in the
was held at financial year 2009-10, is
(a) Bombay (b) Calcutta (a) Rs. 3,900 crores (b) Rs. 39,000 crores
(c) Nagpur (d) Delhi (c) Rs. 59,200 crores (d) Rs. 89,300 crores
Ans. (a) : The first session of Indian National Congress Ans. (b) : Allocation of budget for MNREGA in the
was held at Bombay from December 28 to 30 1885. financial year 2009-10, was Rs. 39,000 crores. In 2022
Womesh Chandra Bannerjee was President for this Budget, Centre allocated Rs 73,000 crore to
session. MGNREGA.
UK RO-ARO (Mains) Exam-2006 331 YCT
CLICK HERE FOR FREE MATERIAL

89. India became the member of WTO (World (a) January 26, 1950
Trade Organisation) in the year (b) August 15, 1950
(a) 1995 (b) 1996 (c) October 2, 1952
(c) 1997 (d) 1998 (d) June 01, 1975
Ans. (a) : India became the member of WTO (World Ans. (c) : 'Community Development Programme' was
Trade Organization) on 1st January, 1995. The overall launched on 2nd October, 1952. It is a multi project
objective of the WTO is to help its members use trade program with the aim of an overall development of rural
as a means to raise living standards, create jobs and people. This program consisted of agriculture, animal
improve people’s lives. The WTO operates the global husbandry, irrigation, cooperation, public wealth,
system of trade rules and helps developing countries education, social education, communication, village
build their trade capacity. industries etc.
90. C.A.G (Comptroller and Auditor General) of 96. The Judges of 'Supreme Court' of India are
India acts as appointed by the President
(a) The guardian of people's liberties. (a) After being approved by Rajya Sabha
(b) The guardian of Public Finance. (b) On the advice of Lok Sabha
(c) The Chief Legal Advisor of the Government. (c) On the advice of Prime Minister
(d) The guardian of all these. (d) In consultation with the Chief Justice of the
Ans. (b) : Article 148, 149, 150, 151 and 279 deals with Supreme Court
Comptroller and Auditor General of India and his Ans. (d) : The Chief Justice of India and the Judges of
duties. Comptroller and Auditor General of India is the
the Supreme Court are appointed by the President under
guardian of Public Finance.
clause (2) of Article 124 of the Constitution. The
91. Narasimham Committee is related to President in consultation with the Chief Justice of the
(a) Land reforms Supreme Court appoints the Judges of the Supreme
(b) Banking-sector reforms Court of India.
(c) Labour reforms
97. Which district in Uttarakhand has the lowest
(d) Agricultural reforms population?
Ans. (b) : Narasimham Committee is related to (a) Uttarkashi (b) Champawat
Banking-sector reforms. Narasimham was the 13th (c) Rudraprayag (d) Bageshwar
Governor of the Reserve Bank of India.
Ans. (c) : Rudraprayag (242,285) has the lowest
92. Which of the sectors of Indian economy has the population in Uttarakhand. Haridwar (1,890,422) has
highest contribution to G.D.P. (Gross Domestic highest population in Uttarakhand.
Product)?
(a) Agricultural sector (b) Industrial sector 98. What is the main occupation of Uttarakhand?
(c) Service sector (d) None of these (a) Tourism (b) Agriculture
(c) Industry (d) All the above
Ans. (c) : Service sector has the highest contribution to
G.D.P. (Gross Domestic Product). As per the Economic Ans. (d) : Agriculture is the main occupation of
Survey 2021-22, service sector had contributed over Uttarakhand. About 14% of the land in Uttarakhand is
50% to India's GDP. used for cultivation.
93. Which of the following election is not 99. Which Dalit-leader led the 'Dola-Palki'
conducted by the Election-Commission? movement in Uttarakhand?
(a) Lok Sabha's (b) Rajya Sabha's (a) Jayanand Bharti (b) Hari Prasad Tamta
(c) President's election (d) Local bodies (c) K.C. Kansal (d) Puran Lal
Ans. (d) : Elections of local bodies is conducted by Ans. (a) : Jayanand Bharti led the 'Dola-Palki'
State Election Commission. For rest, of the institutions movement in Garhwal, Uttarakhand. During the
Election Commission of India conducts election. marriage of Doms the bridegrooms and brides were not
94. In which year were the States reorganized on allowed by the higher castes to use dola and palki (palki
linguistic basis? was used to carry the bridegroom and dola the bride).
(a) 1947 (b) 1951 100. Which was the first newspaper published in
(c) 1956 (d) 1966 Uttarakhand?
Ans. (c) : In 1956, States namely Andhra Pradesh, (a) Garhwali (b) Almora Akhbar
Bombay, Kerala, Madhya Pradesh, Madras, Mysore, (c) Sakti (d) Garhwal Samachar
Punjab and Rajasthan were reorganized by the State Ans. (b) : Almora Akhbar was the first newspaper
Reorganization Act,1956. published in Uttarakhand in 1871. It was the first and
95. When was the 'Community Development the only letter of kumaon to be published continuously
Programme' launched? from 1971 to 1918.

UK RO-ARO (Mains) Exam-2006 332 YCT


CLICK HERE FOR FREE MATERIAL

UTTARAKHAND RO/ARO (Pre) Exam-2016


GENERAL STUDIES
Solved Paper
1. Which of the following develops immune 6. By which of the following can petrol fire be
system in the body? extinguished?
(a) Antigen (b) Antibody (a) Sand (b) Water
(c) Enzymes (d) Hormones (c) Carbon dioxide (d) Saw dust
Ans. (a) : The immune system has a vital role. It Ans. (c) : The CO2 fire extinguisher can be used to
protects body from harmful substances, germs and cell extinguish electrical fires and fires involving flammable
changes that can make ill. It is made up of various liquids such as petrol.
organs, cells and proteins. The immune system can be 7. Fruit ripening hormone is
activated by many different things that the body doesn’t (a) Ethylene (b) Auin
recognize as its own. These are called antigens. (c) Kinetin (d) All of these
Examples of antigens include the proteins on the
Ans. (a) : Ethylene is a gaseous plant hormone which
surfaces of bacteria, fungi and viruses.
regulates a wide range of biological processes in plants.
2. Which of the following causes Swine flu? It is associated with the ripening processes in a number
(a) Virus (b) Bacteria of fruits such as apple and pear.
(c) Fungi (d) Tape worm 8. Seeds of sunflower plant contain
Ans. (a) : Swine flu is an infection caused by a virus.In (a) Alkaloids (b) Oils
2009, a strain of swine flu called H1N1 infected many (c) Dyes (d) Crystals
people around the world.
Ans. (b): Seeds of sunflower plant contain oil.
3. The main element of protein is Sunflower seeds contain nutrients and plant compounds
(a) Hydrogen (b) Nitrogen that help reduce risk of inflammation, heart disease and
(c) Oxygen (d) Carbon type 2 diabetes.
Ans. (b) : Protein are made up of molecules called 9. The 'parsec' is the unit of
Amino acid which mainly contains four elements (a) Distance (b) Time
namely nitrogen, oxygen, hydrogen and carbon. (c) Energy (d) Temperature
Nitrogen is the main element of protein.
Ans. (a) : A parsec is a convenient unit of distance at
4. The 'first' law of thermodynamics is related to astronomical scale.
which theory of conservation?
10. The first mechanical computer designed by
(a) Charge (b) Momentum Charles Babbage was called
(c) Energy (d) Matter (a) Abacus (b) Analytical Engine
Ans. (c) : The First Law of Thermodynamics is simply (c) Calculator (d) Processor
the general law of conservation of energy applied to any
Ans. (b) : The first mechanical computer designed by
system in which the energy transfer from or to the
Charles Babbage was called Analytical Engine.
surroundings is taken into account.
11. Which of the following is an example of non-
5. In view of environmental problems, the use of
volatile memory?
which insecticide has been reduced?
(a) Cache memory (b) RAM
(a) Gammexane (b) D.D.T
(c) ROM (d) None of the above
(c) B.H.C (d) D.M.T
Ans. (c) : Read Only Memory (ROM) is a non volatile
Ans. (b) : DDT (Dichloro-Diphenyl-Trichloroethane)
memory (Information is permanently stored).
was developed as the first of the modern synthetic
insecticides in the early 19405. It was initially used with 12. A device that converts digital signals to analog
great effect to combat malaria. DDT and its related signals is
chemicals persist for a long time in the environment and (a) A Modem (b) A Packet
in animal tissues. Therefore, use of DDT has been (c) A keyboard (d) None of these
reduced in some countries and strictly banned by some Ans. (a) : Modem (Modulator/ Demodulator) is a
countries. device that converts digital signal to analog signal.

UK RO-ARO (Pre) Exam-2016 333 YCT


CLICK HERE FOR FREE MATERIAL

13. Which of the following is the most powerful Ans. (b): Second generation computers used transistors.
type of computer? First generation computers used vacuum tubes while
(a) Super-micro (b) Super Computer third generation computers used integrated circuits (ICs)
(c) Micro Computer (d) Mini computer and fourth generation computers are using VLSI (Very
Large Scale Integration) circuits.
Ans. (b): Super computer is the most powerful
computer which performs task at a very high speed. 20. Which one of the following rivers crosses the
Equator twice?
PARAM 8000 is India's first super computer. PARAM
(a) Zaire (b) Amazon
Siddhi-AI in the fastest supercomputer in India built
(c) Nile (d) Niger
under NSM
Ans. (a) : Zaire river of Africa crosses the equator
14. Which of the following is used to manage Data twice. This river is also known as Congo river.
Base?
21. Which of the following pairs (country-capital)
(a) Operating System (b) Compiler
is not correctly matched?
(c) DBMS (d) None of the above (a) Brazil-Brasilia
Ans. (c) : A database management system (DBMS) is a (b) Tunisia-Tunis
software that can be used to create and manage (c) Algeria-Algiers
databases. DBMS let users to create a database, store, (d) Morocco-Marrakech
manage, update/modify and retrieve data from that Ans. (d) : Rabat is the capital of Morocco. Rest are
database by users or application programs. Some correctly matched.
examples of open source and commercial database
22. What is the correct north-south sequence of the
management system include MySQL, Oracle, states of India along the Myanmar border?
PostgreSQL, SQL Server, Microsoft Access, (a) Arunachal Pradesh, Nagaland, Manipur,
MongoDB. Mizoram
15. The term 'Pentium' is related to (b) Arunachal Pradesh, Assam, Nagaland,
(a) DVD (b) Hard Disk Manipur
(c) Microprocessor (d) Mouse (c) Arunachal Pradesh, Manipur, Nagaland,
Ans. (c): The term Pentium is related to Mizoram
microprocessor. Processor is the electronic circuit of a (d) Arunachal Pradesh, Manipur, Nagaland,
Mizoram.
computer that carries out the actual processing and is
usually referred to as the brain of the computer. Ans. (a) : India and Myanmar share a 1,643
km unfenced border from north to south along
16. What type of process creates a smaller file that Arunachal Pradesh, Nagaland, Manipur and Mizoram.
is faster to transfer over the internet?
23. Which of the following countries is called
(a) Compression (b) Fragmentation 'Land of Thousand lakes'?
(c) Encapsulation (d) None of the above (a) Sweden (b) Canada
Ans. (a) : Compression process creates a smaller file (c) Poland (d) Finland
that is faster to transfer over the internet. Ans. (d) : With more than 180,000 lakes, Finland is
17. 'ALU' stands for called as land of thousand lakes.
(a) Arithmetic Long Unit 24. Which of the following countries is famous for
(b) All Longer Units livestock ranching in the world?
(c) Around Logical Units (a) Brazil (b) Colombia
(d) Arithmetic and Logical Unit (c) Argentina (d) Venezuela
Ans. (d) : ALU stands for Arithmetic Logic Unit. ALU Ans. (c) : Argentina is famous for livestock ranching in
performs all the arithmetic and logic operations that the world. Ranching is the practice of raising herds of
need to be done as per the instruction in a program. animals on large tracts of land. Ranchers commonly
raise grazing animals such as cattle and sheep.
18. Which of the following is not an input device?
(a) Mouse (b) Light pen 25. With which of the following countries does
India have its longest International boundary?
(c) Keyboard (d) VDU
(a) China (b) Pakistan
Ans. (d) : Among the given options, VDU(Visual (c) Bangladesh (d) Nepal
Display Unit) is an output device.
Ans. (c) : India shares its longest international
19. In which generation of computers, transistors boundary with Bangladesh (4,096 km) followed by
were used? China (3,488 km), Pakistan (3,323 km), Nepal (1,751
(a) First (b) Second km), Myanmar (1,643 km), Bhutan (699 km) and
(c) Third (d) Fourth Afghanistan (106 km).
UK RO-ARO (Pre) Exam-2016 334 YCT
CLICK HERE FOR FREE MATERIAL

26. As per 2011 census, which of the following 32. Which of the following States is the largest
states had the highest literacy rate? producer of pulses in India?
(a) Mizoram (b) Goa (a) Madhya Pradesh (b) Uttar Pradesh
(c) Himachal Pradesh (d) Tripura (c) Maharashtra (d) Rajasthan
Ans. (a) : As per 2011 Census, Kerala 94% has highest Ans. (a) : Madhya Pradesh is the largest producer of
literacy rate followed by Mizoram (91.3%), Tripura pulses in India followed by Rajasthan.
(87.2%) and Goa (88.7%). 33. Which of the following country has the largest
27. Which of the following mountain peak is not reserve of bauxite of the world?
located in Uttarakhand? (a) India (b) Russia
(a) Kamet (b) Banderpunch (c) Suriname (d) Australia
(c) Doonagiri (d) Nanga Parbat Ans. (d) : Guinea has the largest reserve of bauxite of
Ans. (d) : Among the given options, Nanga Parbat is 7,400 MMT 879 while Australic in at third position
not located in Uttarakhand. Nanga Parbat also, known with reserve of 5,300 MMT. Bauxite is an ore from
as 'Killer Mountain' is located in Kashmir. which aluminum is produced.
28. As per United Nations' 2015 estimate, how 34. Which of the following State has the largest
much will be the world population by 2030? forest area in India?
(a) 8.2 Billion (b) 8.5 Billion (a) Chhattisgarh (b) Maharashtra
(c) 8.8 Billion (d) 9.1 Billion (c) Madhya Pradesh (d) Andhra Pradesh
Ans. (b) : As per United Nation's 2015 estimate, the Ans. (c) : Area-wise, Madhya Pradesh has the largest
world's population is projected to increase to 8.5 forest cover in India followed by Arunachal Pradesh,
billion by 2030. Chhattisgarh, Odisha and Maharashtra.
29. At which of the following coasts of India is the 35. In which of the following year the 'National
average sea level measured? Population Policy' (NPP) was announced in
(a) Mumbai (b) Chennai India?
(c) Cochin (Kochi) (d) Vishakhapatnam (a) 1999 (b) 2000
Ans. (a) : In India, average sea level is measured at (c) 2001 (d) 2002
Mumbai. Ans.(b) :The National Population Policy, 2000 (NPP-
30. Which of the following statement is incorrect? 2000) affirms the commitment of government towards
(a) The Godavari is the largest river of South voluntary and informed choice and consent of citizens
India while availing of reproductive health care services and
(b) The Kosi River is called the 'Sorrow of Bihar' continuation of the target free approach in administering
family planning services.
(c) The Brahmaputra is an antecedental river
(d) The Ganges originates from Gangotri. 36. The 'World Population day' is observed on
(a) 11th April (b) 11th May
Ans. (*) : All the four given options are correct. In th
(c) 11 July (d) 12th August
terms of length, catchment area and discharge, Godavari
is the largest river in South India. It is also known as Ans. (c) : World Population Day is celebrated annually
Dakshin Ganga. The Kosi River is known as the Sorrow on 11 July to focus attention on the urgency and
of Bihar as the annual floods affect life and livelihood importance of population issues. The Day was
of the people. Both Brahmaputra and Indus rivers are established by the then Governing Council of the UN
antecedent rivers. Bhagirathi and Alaknanda meets at Development Program (UNDP) in 1989.
Devprayag. Hereafter combined stream is known as 37. Who propounded the 'Mobility Transition
Ganga. Model of Migration'?
31. Which of the following pairs (State-date of (a) Lee (b) Charke
formation) is correctly matched? (c) Ravenstein (d) Zelinsky
(a) Haryana-01 November, 1966 Ans. (d) : Wilbur Zelinsky propounded the 'Mobility
(b) Mizoram - 25 June, 1986 Transition Model of Migration'. Zelinsky identified and
(c) Telangana - 15 August, 2014 translated regularities from the demographic transition
model into a set of postulates about mobility and
(d) Chhattisgarh - 20 November, 2000
migration.
Ans. (a):
38. Which of the following pairs is not correctly
State Formation matched?
Haryana – 1 November, 1996 (a) Tharu - Uttar Pradesh
Mizoram – 21 January, 1972 (b) Gaddi - Himachal Pradesh
Telangana – 2 June, 2014 (c) Konyak - Kerala
Chhattisgarh – 1November, 2020 (d) Toda - Tamil Nadu
UK RO-ARO (Pre) Exam-2016 335 YCT
CLICK HERE FOR FREE MATERIAL

Ans. (c): Konyak tribe is known to be one of the 45. The main reason of Ozone depletion is
fiercest warrior tribes in Nagaland. Rest are correctly (a) Carbon Monoxide
matched. (b) Carbon Dioxide
39. As per 2011 census, which of the following states (c) Hydro Carbon
of India had the highest sex-ratio? (d) Chloro-fluro Carbon
(a) Karnataka (b) Kerala Ans. (d): Ozone depletion occurs when
(c) Uttar Pradesh (d) West Bengal chlorofluorocarbons (CFC5) and halon gases formerly
Ans. (b) : As per 2011 Census, Kerala represents the found in aerosol spary and refrigerants are halons cause
highest sex ratio with 1084 females per 1000 males chemical reactions that break sown ozone molecules,
while Haryana features the lowest sex ratio in India reducing ozone's ultraroilet radiation absorbing
with just 879 women per 1000 males. capacity.
40. Which of the following is called "Heart of the 46. Which of the following canal is responsible for
City"? land-degradation in Rajasthan?
(a) Ganga Canal
(a) Central Business District
(b) Sirhind Canal
(b) Zone of Better Residence
(c) Indira Gandhi Canal
(c) Zone of Better Social Amenities
(d) Agra Canal
(d) Commuters' Zone
Ans. (c) : Indira Gandhi Canal is responsible for land-
Ans. (a) : Central Business District is called heart of the
degradation in Rajasthan due to alkalinity and salinity.
city.
47. 'Bio-Diversity' is
41. Who made the Functional Classification of
(a) Totality of all species, all genes and all
Indian Cities?
ecosystems.
(a) R.L. Singh (b) G.S. Gosal
(b) Variety of plants
(c) Ashok Mitra (d) A.B. Mukherjee
(c) Variety of animals
Ans. (c) : Ashok Mitra made the Functional (d) Totality of cultural environment
Classification of Indian cities. Ashok Mitra’s
Ans. (a) : Biodiversity is the term popularized by the
classification is based on categories of workers
sociobiologist Edward Wilson to describe the combined
classification available in the census of 1961 and 1971.
diversity at all the levels of biological organization. The
42. The National Award for Child Welfare was term biodiversity refers to the variety of life on earth at
instituted in the year all its levels, from genes to ecosystems and can
(a) 1996 AD (b) 1979 AD encompass the evolutionary, ecological and cultural
(c) 2008 AD (d) 2014 AD processes that sustain life.
Ans. (b) : The National Child Welfare Awards were 48. Which of the following is responsible for acid
instituted in 1979 to give recognition to those who have rain?
made outstanding contributions in the fields of Child (a) Nitrogen oxide and Carbon dioxide
Development, Child Protection and Child Welfare. (b) Nitrogen oxide and Carbon monoxide
43. The term 'Ecosystem' was first used in 1935 by (c) Sulphur dioxide and Nitrogen oxide
(a) Taylor (b) Clark (d) None of the above
(c) Tansley (d) Lindman Ans. (c) : Acid rain is a byproduct of a variety of
Ans. (c) : The term Ecosystem was coined by Sir human activities such as burning of fossil fuels that emit
Arthur G. Tansley in 1935. An ecosystem can be the oxides of sulphur and nitrogen in the atmosphere.
visualized as a functional unit of nature, where living 49. Match the following with the given code:
organisms interact among themselves and also with the Plant Climatic Region
surrounding physical environment.
A. Megatherm 1. Tundra Region
44. Which of the following states incorporated
B. Mesotherm 2. Equatorial Region
'Environmental Education' as a subject in
school curriculum for the first time? C. Microtherm 3. Tropical Region
(a) Uttarakhand D. Hekistotherm 4. Temperate Region
(b) Maharashtra Codes
(c) Kerala A B C D
(d) Rajasthan (a) 1 2 3 4
Ans. (b) : For the first time Maharashtra incorporated (b) 1 3 2 4
'Environmental Education' as a subject in school (c) 3 4 2 1
curriculum. (d) 2 3 4 1
UK RO-ARO (Pre) Exam-2016 336 YCT
CLICK HERE FOR FREE MATERIAL

Ans. (d): Correct match is- 55. The year 2016-17 will be observed by SAARC
Megatherm Equatorial Region nations as
Mesotherm Tropical Region (a) Year of Democracy and Development
Microtherm Temperate Region (b) Year of Fight Against Terrorism
Hekistotherm Tundra Region (c) Year of Cultural Heritage
50. With which of the following is the Central (d) Year of 'Save the Earth'
Government's 'Stand-up India' scheme is Ans. (c) : The year 2016-17 was observed by SAARC
related to promote entrepreneurship? nations as Year of Cultural Heritage. The South Asian
(a) Women and Minorities Association for Regional Cooperation (SAARC) was
(b) SC/STs and Women established in, 1985. SAARC comprises of eight
(c) SC/STs and Minorities member States: Afghanistan, Bangladesh, Bhutan,
(d) All of the above India, Maldives, Nepal, Pakistan and Sri Lanka. The
Secretariat of the Association is in Kathmandu.
Ans. (b) : Stand-up India scheme seeks to leverage the
institutional credit structure to reach out to the people 56. Aung San Suu Kyi was barred from swearing
such as Schedule Caste, Schedule Tribes and woman in as the President of Myanmar because
entrepreneurs so as to enable them to participate in the (a) She has a criminal record.
economic growth of the nation. It was launched in April (b) Her party did not win absolute majority in
2016. elections.
51. On which of the following routes will India's (c) The military installed its own President.
first Bullet Train run (upto 2024)? (d) Her sons are British Citizens.
(a) New Delhi - Ahmedaba Route Ans. (d) : Aung San Suu Kyi was barred from swearing
(b) New Delhi - Mumbai Route in as the President of Myanmar because her two sons
(c) Ahmedabad - Mumbai Route are British Citizens.
(d) Pune - Ahmedabad Route 57. Due to the outbreak of which virus did the
Ans. (c) : India's first Bullet Train will run on WHO declare a Public Health Emergency of
Ahmedabad - Mumbai Route. International Concern in February, 2016?
52. Which of the following State has been declared (a) Zika (b) Ebola
as India's first State to have adopted organic (c) Swine (d) Chikungunya
farming? Ans. (a) : WHO declare a Public Health Emergency of
(a) Tripura (b) Sikkim International Concern in February, 2016 due to Zika
(c) Arunachal Pradesh (d) Nagaland virus. Zika virus is a mosquito borne flavivirus that was
first identified in Uganda in 1947 in monkeys. It was
Ans.(b): Sikkim became the first state in India to
later identified in humans in 1952 in Uganda and the
officially announce adoption of organic farming in the
year 2003 to ensure long term sustenance of soil United Republic of Tanzania. Outbreaks of Zika virus
fertility, Protection of environment and ecology, healthy disease have been recorded in Africa, America, Asia
living and decreasing risk of health ailments. and the Pacific.
53. Who created a new world record in scoring the 58. Which of the two European Cities were the
fastest century in Test Cricket history in victims of IS terrorist attacks in November,
February, 2016 ? 2015 and March, 2016 respectively?
(a) Steve Smith (b) Chris Gayle (a) London and Brussels
(c) Brendon McCullum (d) Virat Kohli (b) Paris and Brussels
(c) Frankfurt and Brussels
Ans. (c) : New Zealands cricket player Brendon Mc
Cullum smashed the faster ever test century in February (d) Copenhagen and Brussels
2016. He made Century in 54 balls. Ans. (b) : Paris and Brussels were the victims of IS
54. The multi-billion dollar project TAPI on which terrorist attacks in November, 2015 and March, 2016
work started in December, 2015 is related to respectively.
(a) Natural Gas (b) Solar Energy 59. Which of the following cities are hosts for 2016
(c) Road Construction (d) Railway Line and 2020 Olympic Games, respectively?
(a) Rio de Janeiro and Tokyo
Ans. (a) : TAPI (Turkmenistan-Afghanistan-Pakistan-
India) pipeline is a 1,814 km trans-country natural gas (b) London and Rio de Janeiro
pipeline running across four countries. TAPI pipeline (c) Tokyo and Rio de Janeiro
will begin in Turkmenistan and transverse Afghanistan (d) London and Tokyo
to enter Pakistan and India. Political turmoil in Ans. (a) : Rio de Janeiro and Tokyo were host for
Afghanistan has halted the completion of project. 2016 and 2020 Olympic Games, respectively.
UK RO-ARO (Pre) Exam-2016 337 YCT
CLICK HERE FOR FREE MATERIAL

60. Which state had the highest per capita income (c) On all stages between production and sale
during 2014-15? (d) On final stage of production.
(a) Kerala (b) Haryana Ans. (c) : VAT (Value Added Tax) was imposed on 1 st
(c) Punjab (d) Uttarakhand April, 2005. Value added tax (VAT) is a type of indirect
Ans. (b) : Among the states Haryana recorded highest tax levied on goods for value added at every point of
per capita income in country at Rs 1, 47, 076 during production or distribution cycle, starting from raw
2014-15. Presently State of Goa has highest per capita materials and going all the way to the final retail
income of Rs 4,35,959(2019-20) as per economic purchase. Goods and Services Tax (GST) was
survey 2021-22. introduced in July 2017, which replaced VAT. Though
61. Who is the Vice Chairman of the NITI Aayog GST replaced VAT on most goods, some goods are still
of India? not covered under GST. VAT continues to be the tax
(a) Raghuram Rajan (b) Arun Jately levied on such goods.
(c) Arvind Panagariya (d) Rahul Gandhi 67. What is 'Super-301'?
Ans. (c) : National Institution for Transforming India (a) Modern Computer
(NITI Aayog) is the premier policy think tank of the (b) A variety of rice
Government of India, providing directional and policy (c) An insecticide
inputs. Apart from designing strategic and long term (d) American Trade Law
policies and programs for the Government of India, Ans. (d) : One of the amendments of section 301 of the
NITI Aayog also provides relevant technical advice to United States Trade Act of 1974, is called Super 301.
the Centre, States and Union Territories. At the time This Act was passed to solve problems with the U.S.
when question was asked, Arvind Panagariya was the bilateral trade deficits, which were considered a
Vice Chairman of the NITI Ayog. At present, Suman consequence of trade barriers to U.S. exports in foreign
Bery is Vice Chairman of the NITI Aayog. countries.
62. Manodaya Kavya was composed by 68. Who is called the Father of White Revolution
(a) Bharat Kavi (b) Maularam in India?
(c) Bhushan (d) Gumani Kavi (a) Dr. Norman Borlaug
Ans. (a) : Manodaya Kavya was composed by Bharat (b) Dr. M.S. Swaminathan
Kavi. (c) Dr. Verghese Kurien
63. The rank of India in HDI according to Human (d) Dr. William Gande
Development Report-2015 of UNDP was Ans. (c) : Dr. Verghese Kurien is called the Father of
(a) 128 (b) 134 White Revolution in India. Dr Kurien was instrumental
(c) 130 (d) 132 in developing the ‘Amul’ brand for marketing of the
Ans. (c) : According to Human Development Report- milk and milk products.
2015 of UNDP, rank of India was 130. In 2021-22, 69. Which of the following taxes in India is
India's rank was 132 out of 191 countries. collected by Panchayats?
64. The 'Closed Economy' is the economy in which (a) Sales Tax (b) Custom Tax
(a) Only export takes place (c) Land Revenue (d) Tax on local fairs
(b) Budget deficit is less Ans. (d) : Panchayats collect tax on local fairs Central
(c) Only import takes place and state government collects sales tax. Land revenue is
(d) There is no foreign trade. collected by state government.
Ans. (d) : In closed economy is one which does not 70. Arrange the following movements of Mahatma
trade with outside economies i.e there is no foreign Gandhi in the right order from the beginning
trade. (a) Champaran, Ahmedabad, Kheda, Non-
65. The National Institute for Rural Development Cooperation.
(NIRD) is situated in (b) Ahamedabad, Champaran, Kheda, Non-
(a) Shimla (b) Hyderabad cooperation.
(c) Dehradun (d) New Delhi (c) Kheda, Champaran, Ahamedabad, Non-
Cooperation.
Ans. (b) : The National Institute for Rural Development
and Panchayati Raj is situated in Hyderabad. In addition (d) Non-cooperation, Champaran, Kheda,
to the main campus at Hyderabad, this Institute has Ahamedabad.
North-Eastern Regional Centre at Guwahati, Assam to Ans. (a) : Correct sequence is-
meet the North- Eastern regional needs. Champaran (1917)
66. 'VAT' is imposed Ahmedabad ( Feb-March, 1918)
(a) Directly on consumer Kheda (22nd March, 1918)
(b) On first stage of production. Non-Cooperation (September, 1920 - February 1922).
UK RO-ARO (Pre) Exam-2016 338 YCT
CLICK HERE FOR FREE MATERIAL

71. What was 'Sangam' in ancient India? Ans. (d): On February 20, 1947, Clement Attlee, then
(a) Sangha or Mandal of Tamil poets Prime Minister of the United Kingdom, announced the
(b) Tamil King's court poets granting of full self-government to India by June 30,
(c) Tamil settlement 1948.
(d) Mesolithic graves 77. The first Sultan who started the work of
Ans. (a) : Some of the earliest works in Tamil, known translation of Hindu Religious texts into
as Sangam literature, were composed around 2300 years Persian Language, was
ago. These texts were called Sangam because they were (a) Iltutmish
supposed to have been composed and compiled in (b) Alauddin Khilji
assemblies (known as Sangams) of Tamil poets that (c) Mohammad-Bin-Tughlaq
were held in the city of Madurai.
(d) Firoz-Shah-Tughlaq
72. Who built the Khajuraho temples?
Ans. (d) : Firoz Shah Tughlaq was the first Sultan who
(a) Vijainagar Kings (b) Bahmani Sultans
started the work of translation of Hindu religious texts
(c) Chandel Kings (d) Gupta Kings
into Persian language.
Ans. (c) : Khajuraho temple, situated in Chhatarpur 78. Which was the highest gold coin in the reign of
district of Madhya Pradesh were built around late ninth
Akbar?
century to twelfth century. Khajuraho temple were
(a) Ilahi (b) Jalali
patronized by Chandel kings. All the temples made at
Khajuraho are made of sandstone. (c) Shahanshah (d) Asharfi
73. The real name of Sabarmati Ashram? Ans. (c) : Shahanshah, weighted around 102 tolas was
(a) Gandhi Ashram (b) Satyagraha Ashram the highest gold coin in the reign of Akbar.
(c) Finiks Farm (d) Dandi Ashram 79. Who started the 'Silk-route' (Marg) for
Ans. (b) : The Sabarmati Ashram, originally called the Indians?
Satyagraha Ashram (also known as Harijan Ashram) (a) Kanishka (b) Harshwardhan
was home to Mohandas Gandhi from 1917 until 1930 (c) Ashok (d) Fahien
and served as one of the main centers of the Indian Ans. (a) : Kanishka of Kushanas empire started the
freedom struggle. 'Silk-route' (Marg) for Indians. Silk route linked China
74. From 1906 to 1920, the role of Md. Ali Jinnah With the west.
during freedom struggle of India was 80. 'Privy Purse' was related to whom in the Post-
(a) Separatist Independence period?
(b) Fundamentalist (a) Landlords (b) Former kings
(c) Nationalist (c) Industrialists (d) Indigo Planters
(d) Nationalist and Secular
Ans. (b) : Privy Purse was a special privilege given to
Ans. (d) : Md. Ali Jinnah was a nationalist and secular former kings. At the time of independence, more than
regarded as an ambassador of Hindu Muslim unity until 555 small or large princely States existed in India. The
1920. Jinnah played an important role in the making of Privy Purse was a specific amount of money that was to
the Lucknow Pact. He reorganized the Muslim League be paid annually by the Indian government to the rulers
and after 1934 he became the major spokesperson for
of princely states and their successors who had acceded
the demand for Pakistan.
to India. The privy purses were guaranteed to these
75. Dr. B.R. Ambedkar published three periodicals rulers under Article 291 of the Indian Constitution.
for the safeguard & Dalit rights. Which of the These payments were tax-free and were to be paid from
following is not one of them. the Consolidated Fund of India. The 26th Constitutional
(a) Mook Nayak (b) Bahishkrit Bharat Amendment abolished it.
(c) Bahishkrit Samaj (d) Equality Janta
81. Who nominates the 12 members for Rajya
Ans. (c): Dr. B.R. Ambedkar started his first Sabha in India?
newspaper, Mooknayak, on January 31, 1920. It ran for
(a) Prime Minister (b) Chief Justice
three years before being closed. Later, he went on to
(c) President (d) Vice President
found three more newspapers namely Bahishkrut Bharat
(1927-1929), Janata (1930-56) and Prabuddha Bharat Ans. (c) : Under Article 80 of the Constitution, the
(1956). Council of States (Rajya Sabha) is composed of not
76. By which date did the British Government more than 250 members, of whom 12 are nominated by
declare to grant India full self Government? the President of India from amongst persons who have
(a) January 26, 1946 (b) August 15, 1947 special knowledge or practical experience in respect of
(c) December 31, 1947 (d) June 30, 1948 such matters as literature, science, art and social service.

UK RO-ARO (Pre) Exam-2016 339 YCT


CLICK HERE FOR FREE MATERIAL

82. In the Constitution of India, the Residuary with the election of the President or Vice President shall
powers are entrusted to be inquired into and decided by the Supreme Court
(a) State (b) Centre whose decision shall be final.
(c) Both (a) and (b) (d) None of the above 88. Who propounded the theory of 'Economic
Ans. (b) : According to Article 248 of the Constitution, Drain'?
Parliament has exclusive power to make any law with (a) M.K. Gandhi (b) Jawahar Lal Nehru
respect to any matter not enumerated in the Concurrent (c) Dadabhai Naoroji (d) R.C. Dutt
List or State List. Ans. (c) : Dadabhai Naoroji propounded the Theory of
83. By which Constitutional amendment, the Economic Drain in which he explained how the colonial
subject of 'Education' was transferred from power drained wealth from India.
State list to concurrent list? 89. The first 'Lokpal Bill' was introduced in the
(a) 5th (b) 9th Parliament of India in
th
(c) 42 (d) 44th (a) 1971 (b) 1967
Ans. (c) : The Constitution 42nd Amendment Act, 1976 (c) 1972 (d) 1968
transferred the subject of education from State list to Ans. (d) : The first 'Lokpal Bill' was introduced in the
Concurrent list. Parliament of India in 1968. It was finally passed by the
84. Which of the following state does not have Parliament in 2013.
'Legislative Council? 90. The first book written by M.K. Gandhi was
(a) Uttar Pradesh (b) Maharashtra (a) My experiment with Truth
(c) Karnataka (d) Uttarakhand (b) Hind Swaraj
Ans. (d) : Legislative Council is upper house of the (c) India of My Dreams
State. Its institution is mentioned in Article 169 of the (d) Key to the Health
Constitution of India. As of January 2022, six out of 28 Ans. (b) : Hind Swaraj, a severe criticism of modern
states have a legislative council. The states with civilization was written at stretch during Gandhi's
bicameral legislature include Andhra Pradesh, Bihar, voyage from England to South Africa in 1909.
Karnataka, Maharashtra, Telangana and Uttar Pradesh.
These states have both the Legislative Council and 91. The main object of a 'Budget' is
Legislative Assembly. (a) To ensure Accountability
(b) To serve as a tool of Management
85. The concept of Public Interest Litigation (PIL)
originated in (c) To facilitate Economic Analysis
(a) United Kingdom (d) All of the above
(b) Australia Ans.(d):The most important objectives of a government
(c) United States of America budget are re-allocating the resources across the nation,
(d) India bringing down the inequalities in terms of earning and
wealth, paving way for economic stability, managing
Ans. (c) : The term Public Interest Litigation (PIL) public enterprises, contributing to economic growth and
originated in the United States in the mid 1980s. In addressing the regional disparity.
India it was introduced in the early 1980's Justice V R
Krishna Iyer and Justice P.N. Bhagwati were the 92. 'Parliamentary form of Government' functions
poineers of concept of PIL. on the principle of
(a) Separation of powers
86. The Constitution of India declares India as
(b) Checks and Balances
(a) A voluntary Federation
(c) Close relationship between Legislature and
(b) A Confederation
Executive
(c) A Union of States
(d) Control of Judiciary on the executive
(d) A Federation
Ans.(c): Parliamentary form of Government functions
Ans. (c) : According to Article 1 of the Constitution, on the principle of close relationship between
India shall be Union of States. Legislature and Executive. Cabinet is formed by the
87. Who settles the dispute regarding the election Parliament and cabinet is responsible before the
of the President of India? Legislature.
(a) Supreme Court 93. The 'Zero Hour' in the Parliament of India
(b) Election Commission starts at
(c) Parliament (a) First hour of the sitting
(d) None of the above (b) Last hour of the sitting
Ans. (a): According to Article 71(1) of the Constitution, (c) At 12.00 Noon
all doubts and disputes arising out of or in connection (d) At any time
UK RO-ARO (Pre) Exam-2016 340 YCT
CLICK HERE FOR FREE MATERIAL

Ans. (c): Zero hour is an informal device available to 2x = 52


members of parliament to raise matters without 10 days x = 26
advance notice. The 'Zero Hour' in the Parliament of Hence, The man has 26 hens.
India starts at 12 noon following Question hour. 99. If 20% of a = b, then b% of 20 is the same as
94. Which part of the Constitution of India is (a) 4% of a (b) 5% of a
related to the 'Citizenship'?
(c) 20 % of a (d) 25% of a
(a) II (b) III
(c) IV (d) V a
Ans. (a) : b =
Ans. (a) : Part II of the Constitution of India (Articles 5
5-11) deals with the Citizenship of India. a
( 2.39 ) – (1.61) is ⇒ 20 × 5
2 2

95. The value of 100


2.39 – 1.61
a 4
(a) 2 (b) 4 ⇒ 20 × =a×
(c) 6 (d) 8 5 × 100 100
or 4% of a
(2.39) − (1.61)
2 2
Ans. (b) : 100. Mahesh sells 18 eggs at the price for which he
(2.39 − 1.61)
bought 20 eggs. Find his profit or loss
(2.39 + 1.61)(2.39 − 1.61) percentage.
(2.39 − 1.61) 1 1
(a) 11 % loss (b) 11 % profit
(4.00)(0.78) 3 3
= 4 Ans.
(0.78) 1 1
(c) 11 %loss (d) 11 % profit
96. If 0.75 : X :: 5 : 8, then value of X is 9 9
(a) 1.12 (b) 1.20 Ans. (d) : Let Mahesh bought 20 eggs for ` x.
(c) 1.25 (d) 1.30 x
∴ Cost price of 1 egg · `
Ans. (b) : 0.75 : x :: 5:8 20
0.75 × 8 x
x= ∴ Selling price of 1 egg · `
5 18
x = 1.20 x x
97. The average age of a class of 22 students is 21 −
18 20
years. The average age increases by 1 year Required ³ · x
×100
when the teacher's age is also included. What is 20
the age of the teacher?
10 x − 9 x 20
(a) 48 years (b) 45 years · × × 100
(c) 43 years (d) 44 years 180 x
Ans. (d) : The total age of 22 students of the class x 20 1
· × × 100 · 11 % Profit
= 22×21 180 x 9
= 462 years 101. Count the number of squares in the figure
Total age including teacher's given below:
age · 23×22 · 506 years
Teacher's Age · 506 - 462 · 44 years
98. A man has some hens and cows. If the number
of their heads be 48 and the number of feet
equal to 140, then the number of hens is (a) 8 (b) 12
(a) 24 (b) 25 (c) 15 (d) 18
(c) 26 (d) 27
Ans. (c) : Total no. of square in figure = 15
Ans. (c) : Let the no. of hens · x
102. It 'light' is called ' morning'; 'morning' is called
and the no. of cow · (48-x) 'dark'; 'dark' is called 'night; ' night' is called
According to question, 'sunshine'; then normally when do we sleep?
2x + 4(48-x) = 140 (a) Sunshine (b) Night
2x + 192 - 4x = 140 (c) Dark (d) Morning
UK RO-ARO (Pre) Exam-2016 341 YCT
CLICK HERE FOR FREE MATERIAL

Ans. (a): Normally we sleep in the night and in the Answer Figure :
question 'night' is called 'sunshine'. Hence 'sunshine' is
correct answer.
103. A family has a man, his wife, their four sons
and their wives. The family of every son also
has 3 sons and one daughter. Find out the total (a) 4 (b) 3
number of male members in the whole family. (c) 2 (d) 1
(a) 5 (b) 10 Ans. (b) :
(c) 16 (d) 17
Ans. (d) : Total number of male = 1 + 4 + 12 = 17 +4 +4 +4
H  → L  → P  →T
104. K is the brother of N and X. Y is the mother of +4 +4 +4
F  → J  → N  →R
N, and Z is the father of K. Which of the +4 +4 +4
C → G → K → O
following statements is not true?
107. Raman borrows ` 12,000 for 3 years at 10.5%
(a) K is son of Z.
per annum simple interest; while Rahul
(b) Y is wife of Z borrows the same amount for the same period
(c) N is always the brother of X. at 10% compounded annually. Who pays more
(d) Z is the father of X. and by how much?
Ans. (c) : (a) Raman, ` 132 (b) Raman, ` 145
(c) Rahaul, ` 158 (d) Rahul, ` 192
PRT
Ans. (d) : The interest paid by Raman ·
100
12000 × 10.5 × 3
= = 3780
100
Thus, Total amount paid by Raman · 12000+3780
= 15,780 `
n

· P  1 + 
r
Amount paid by Rahul 
 100 
3
 10 
Hence, the statement "N is always brother of X" is not = 12000  1 + 
 100 
correct. N can be either brother or sister of X.
11 11 11
105. In a row of 30 girls, when Shivani was shifted = 12000 × × ×
10 10 10
to her left by 6 places, her number from the left
= 12 × 1331 = 15,972 `
end of the row becomes 8th. What will be the
∴ 15,972 - 15,780 = `192
number of Urvashi from the right end of the
Hence, Rahul paid ` 192 more.
row, if Urvashi was 4 places to right of
108. The mean of the ages of father and his son is 27
Shivani's original position?
years. After 18 years father will be twice as old
(a) 12th (b) 13th as his son. Their respective present ages are
th
(c) 14 (d) 16th (a) 42, 12 (b) 40, 14
Ans. (b) : Original position of Shivani from left = 8 + 6 (c) 30, 24 (d) 36, 18
= 14th Ans. (a) : Total age of father and his son
Original position of Urvashi from left = 14 + 4 = 18th = 27× 2 = 54 years
∴ Position of Urvashi from the right = 30 – 18 + 1 = Let the present age of son be x years.
13th And present age of father = (54 – x) years
According to question:- 54-x +18 = (x+18)×2
106. Select the missing figure from answer figures.
72-x = 2x+36
Problem figures : 3x = 36
x = 12
Present age of son (x) = 12 years
Present age of father = (54 – 12) = 42 years
UK RO-ARO (Pre) Exam-2016 342 YCT
CLICK HERE FOR FREE MATERIAL

109. The next term in the series is : (a) 31 days (b) 25 days
B2E, D5H, F12K, H27N, ? (c) 27 days (d) 29 days
(a) Q62J (b) J56Q Ans. (c) : Let 'B' completes a work in x days.
(c) I62Q (d) J58Q A completes the same work = x/2 days.
Ans. (d) : According to question-
1 2 1
+ =
x x 18
1+ 2 1
=
x 18
Hence, ? = J 58 Q x = 54 days
110. Pointing towards Bhuvan, Madhu said, "He is Hence, A will complete the work alone
the son of only son of my father". How is the = x/2
mother of Bhuvan related to Madhu? = 54/2
(a) Sister (b) Sister-in-law = 27 days
(c) Daughter (d) Aunt 114. The ratio between the speeds of two trains is
Ans. (b) : 7:8. If the second train runs 400 kms in 4
hours, then the speed of the first train is :
(a) 70km/hr. (b) 75km/hr
(c) 84km/hr (d) 87.5 km
Ans. (d) : ∵ The speed of second train
400
= = 100km / hr
4
∴ The speed of first train
100
= × 7 = 87.5km / hr
8
Hence, Bhuvan's mother will be Madhu's sister-in-law. 115. Anil after travelling 6 km towards East from
111. Find the missing number in the third triangle: his house realized that he has travelled in
wrong direction. He turned back and travelled
12km towards West, then turned right and
travelled 8 km to reach his office. The straight
distance of his office from his house is
(a) 18 (b) 46 (a) 12 km (b) 14 km
(c) 80 (d) 70 (c) 10 km (d) 20 km
Ans. (d) : L.C.M. of the paints located on all the three Ans. (c) : Direct distance from home to office
vertices of the triangles are marked in the middle of
= 82 + 62
them.
Just as = 10 km
The L.C.M. of 6, 8, 12 = 24
L.C.M. of 3, 4, 6 = 12
Similarly, L.C.M. of 2, 5, and 7 = 70
112. Fill in the blank:
4, 6, 12, 14, 28, 30, ?
(a) 32 (b) 64
(c) 62 (d) 60
116. Find the missing number.
Ans. (d) :
4 6 12 14 28 30 60
7 5 6
3 2 1
+2 ×2 +2 ×2 +2 ×2
9 6 ?
113. 'A' is twice as good a workman as 'B' and
together they finish a work in 18 days. In how (a) 5 (b) 6
many days 'A' alone will finish the work? (c) 7 (d) 8
UK RO-ARO (Pre) Exam-2016 343 YCT
CLICK HERE FOR FREE MATERIAL

Ans. (b): Just as, (a) West (b) West-South


7+3 ⇒ 10 - 1 = 9 (c) South (d) North
5+2 ⇒ 7 - 1 = 6 Ans. (c) :
Same as,
6+1⇒7-1=6
117. Free note books were distributed equally
among children of a class. The number of
th
1
notebooks each child got was of the number
8
of children. Had the number of children been
half, each child would have got 16 notebooks.
In total how many notebooks were distributed?
(a) 256 (b) 432
120. In a mixture of 60 litres, the ratio of milk and
(c) 512 (d) 640 water is 2 : 1. To make this ratio 1 : 2, what
Ans. (c) : Let the total no. of children = x quantity of water (in litres) is to be added to the
1 mixture?
∵ The no. of note books received by 1 child = × x (a) 30 (b) 60
8
(c) 90 (d) 100
x x2
∴ Total no. of notebooks = x × = Ans. (b) : The amount of milk in the mixture
8 8 60
= × 2 = 40
According to question - 3
x x Amount of water in the mixture
x × = ×16
8 2 60
= × 1 = 20
⇒ x = 64 3
64 Let x liter of water be required–
∴ Total no. of notebooks = 64 × = 512 According to question –
8
40 1
118. In a certain code BREAKTHROUGH is =
written as EAOUHRBRGHKT. How is 20 + x 2
DISTRIBUTION written in that code? 20 + x = 80
(a) STDIBURIONTI x = 60 litre
(b) STTIBUDIONRI 121. If one-third of one-fourth of a number is 15,
then three-tenth of that number is
(c) TISTBUONDIRI
(a) 34 (b) 46
(d) STTIBUODRIDI
(c) 49 (d) 54
Ans. (B) : Just as Same as Ans. (d) : Let number is x.
1 1
⇒ x × × = 15
4 3
x = 180
3
⇒ 180 × = 54
10
122. A clock is started at 12 o'clock noon. By 10
minutes past 5, the hour hand has turned
through
(a) 145º (b) 150º
(c) 155º (d) 160º
Ans. (c) : The angle made by the hour hand in 12 hour
= 360º
10
119. A man is facing towards West, he turns ∴ The angle made by the hour hand in 5 hours
through 45º clockwise, again 180º clockwise 60
and then turns through 315º anti-clockwise. In 360 31
= × = 155º
which direction is he facing now? 12 6
UK RO-ARO (Pre) Exam-2016 344 YCT
CLICK HERE FOR FREE MATERIAL

123. 39 persons can repair a road in 12 days, Ans. (a) : The sir La Pass connects Bidang in the Drma
working 5 hours a day. In how many days will valley to the Byans valley. It is located in the
30 persons, of same efficiency working 6 hours Pithoragarh district of the Uttarakhand state in eastern
a day, complete the work? Kuaon.
(a) 10 (b) 13 129. The objective of forming the "Praja Mandal"
(c) 14 (d) 15 was a
Ans. (b) : Let x days are required to complete the work. (a) to get freedom form British Rule
According to question– (b) to get freedom from Gorkha Rule
M1D1H1 = M 2 D 2 H 2 (c) to organise people for social reform
(d) to free the people from misrule of Tehri State
39 × 12 × 5 = 30 × D 2 × 6
Ans. (d) : Praja Mandal was formed to free the people
39 × 12 × 5 from the misrule of the Tehri State. Sridev Suman was
D2 = = 13 days
30 × 6 the main leader of the Tehri Praja Mandal.
124. The difference between a two digit number and 130. The Khatling glacier is the origin-place of
the number obtained by interchanging the (a) Jalkur river (b) Bhilangana river
positions of its digits is 36 what is the difference (c) Pilang Ganga river (d) Balganga river
between the two digits of that number? Ans. (b) : Khatling rivers is located in district Tehari
(a) 4 (b) 9 Garhwal of Uttarakhand, the state which is known for
(c) 12 (d) 16 his high calm and beautiful hills. Khatling glacier is
Ans. (a) : Let the digit no. = 10x + y very important glacier of Garhwal Himalayas due to the
Required difference 10x + y – (10y+x) = 36 source of Bhilanganga rivers.
10x + y – 10y – x = 36 131. Which of the following is not a foot hill town?
9x – 9y = 36 (a) Tanakpur (b) Kotdwar
x–y=4 (c) Rudrapur (d) Haridwar
125. Major Somnath Sharma who was honoured Ans. (c) : Rudrapur is not a foot hill town.
with Paramveer Chakra posthumously in 1947 132. The famous 'Kailash-Manasarovar Yatra'
belonged to passes through
(a) Gorkha Rifle (a) Niti pass (b) Lipulekh pass
(b) Garhwal Rifle (c) Lampiya Dhura pass (d) Barahoti pass
(c) Dogra Regiment Ans. () :
(d) Kumaon Regiment 133. Which one of the following is not a railway
Ans. (d) : Major Somnath Sharma was first recipient of terminus point in Uttarakhand?
the Param Vir Chakra which he was awarded (a) Kathgodam (b) Dehradun
posthumously. He belongs to Kumaon Regiment. (c) Kotdwar (d) Laksar
126. The Lakhu cave having ancient rock paintings Ans. (d) : Laksar is not a railway terminus point in
is located in Uttarakhand.
(a) Chamoli district (b) Nainital district 134. In which district does the "Nandhaur Dun"
(c) Haridwar district (d) Almora district lie?
Ans. (d) : The Lakudiyar caves are known for (a) Udham Singh Nagar (b) Nainital
prehistoric wall paintings depicting crimal humans also (c) Garhwal (d) Haridwar
tectiforms. The cave is located at the banks of the Ans. (b) : Nandnaur doon is located in Nainital district
Suyal river in Almora district of Uttrakhand. of Uttarakhand.
127. The court language of Katyuris was 135. In the Himalaya, the "Main Central Thrust"
(a) Kumaoni (b) Garhwali (MCT) separates
(c) Sanskrit (d) Prakrit (a) Lesser Himalaya and Outer Himalaya
Ans. (c) : The Katyuris kings were the medieval ruling (b) 'Dun' valleys and the Shivaliks
clan of present Uttarkhand that ruled over the region (c) Greater Himalaya and Trans-Himalaya
known as Kumaon from 800 to 1100 AD . Their court (d) Greater Himalaya and Lesser Himalaya
language was Sanskrit. Ans. (d) : Main central Thrust (MCT) is major
128. The pass connecting Darma and Byans Valley geological fault line between greater and lesser
is Himalayas and grater Himalayas. It runs along 2200 km
(a) Sinla (b) Unta boundary of Himalayas in Northwest-Southeast
(c) Jyatia (d) Ramal direction It is sloping towards North.
UK RO-ARO (Pre) Exam-2016 345 YCT
CLICK HERE FOR FREE MATERIAL

136. The historical name of Uttar Kashi is Ans. (c) : The 38th National Games were planned to
(a) Saumyakashi (b) Shrikshetra organized in 2018 in Uttarakhand. However due to
(c) Gangotri (d) Barahat various reasons it got postponed. Now it is planned to
Ans. (d) : Uttarkashi's ancient name was Barahat. be held 2024.
137. During which period did the Colonial conflict 145. Which city of Uttarakhand was recently
start in Uttarakhand? ranked 61st in the list of 'Clean Cities' of India?
(a) Mughal period (b) Gorkha period (a) Dehradun (b) Roorkee
(c) Dogra period (d) British period (c) Haldwani (d) Haridwar
Ans. (b) : During Gorkha period the colonial conflict Ans. (a) : The Capital of Uttarakhand, Dehradun was
started in Uttarakhand. ranked 61st in the list of clean cities of India in the year
138. Which of the following was known as 2016.
'Kubjamrak' in ancient time?
146. The President's Rule in Uttarakhand was
(a) Rishikesh (b) Devprayag
enforced on
(c) Haridwar (d) Srinagar
(a) 20th March, 2016
Ans. (a) : Present Rishikesh in Uttarakhand, was
(b) 25th March, 2016
known as Kubjamrak in ancient times.
(c) 27th March, 2016
139. In which language are the 'Pandukeshwar
copper plates' written? (d) 29th March, 2016
(a) Pali (b) Prakrit Ans. (c) : On 27th March 2016 President rule was
(c) Sanskrit (d) Armike imposed in Uttarakhand.
Ans. (): 147. Which of the following is dominant in
140. In which of the following dances, the song is not heredity?
sung? (a) Albinism (b) Rh-factor
(a) Chhopti (b) Johra (c) Color blindness (d) Haemophilia
(c) Chhapeli (d) Chholia Ans. (b) : Rh-factor is dominant in heredity. It is a type
Ans. (d) : Chholia dance, The song is not sung . of protein found on the outside of red blood cells. The
Kumaon is a place in Uttarkhand, Which has a unique Protein is genetically inherited.
way of celebrating weddings . A special and unique 148. The drug produced through biotechnology for
dance is performed at wedding called the Choliya
cancer treatment is
dance.
(a) Interferon (b) Insulin
141. The 'Dhawaj Temple' of 9th century in
Dwarahat is also known as (c) H.G.H. (d) T.S.H.
(a) Maha Mrityunjay Temple Ans. (a) : Interferon is a natural substance that helps the
(b) Katarmal Temple body's immune fight infection and other diseases, such
(c) Gujardev Temple as cancer. Interferon are made in the body by white
(d) Kachheri Temple blood cells and other cells, but they can also be made in
the laboratory to use as drug for treatment of different
Ans. (d) : Dhawaj Temple of 9th Century in Dwarahat diseases.
is also known as Kachheri temple. It is situated in
Almora district. 149. How many lobes are found in human lungs?
(a) 2 (b) 3
142. Which of the following tribes worship the deity
'Bhumsen'? (c) 4 (d) 5
(a) Tharu (b) Boxa Ans. (d) : Each lung is divided into lobes separated
(c) Jaad (d) Bhotia from one another by a tissue fissure. The right lung is
Ans. (a) : Tharu Tribe inhabiting in terai region of composed of three lobes. The left lung has only two
Uttarakhand, Uttar Pradesh worship deity Bhumsen. lobes separated by an oblique fissure.
143. 'Brass City' in Uttarkhandis being set-up at 150. The bird that can move its upper jaw is
(a) Bajpur (b) Rudrapur (a) Pigeon (b) Parrot
(c) Pithoragarh (d) Almora (c) Vulture (d) Sparrow
Ans. (d) : Brass city in Uttarkhand is being set up at Ans. (b) : Birds can move both their upper and lower
Marchula in Almora. jaws, but fruit eating birds like parrots and Macaws
144. In which year will the 38th National Games be can also their upper jaws sideways, exerting enough
organized in Uttarakhand? force and pressure to crack open tough seeds or fruit
(a) 2016 (b) 2017 stones, and to hang onto branches as they make their
(c) 2018 (d) 2020 way to be next food source.
UK RO-ARO (Pre) Exam-2016 346 YCT
CLICK HERE FOR FREE MATERIAL

UTTARAKHAND RO/ARO (Mains) Exam-2016


GENERAL STUDIES
Solved Paper
1. December 1929-Lahore Congress Convention Ans. (d): The New Industrial Policy of India, 1991 was
put emphasis on based on the principles of Liberalization, Privatization
(a) Home-Rule and Globalization.
(b) Purna Swaraj 7. Human Development Index (HDI) in India
(c) Dominion status includes
(d) Proper representation of Indians in the
(a) Life expectancy
administration
(b) Educational attainment
Ans. (b) : During 1929 Congress session held in
(c) Per Capita Income
Lahore, Congress took the resolution of Purna Swaraj
(Complete Independence). During Lahore session, (d) All of the above
Indian tricolour flag was hoisted by Pandit Jawahar Lal Ans. (d) : The Human Development Index (HDI) is a
Nehru on the bank of the Ravi river. statistical tool used to measure a country's overall
2. Who led the Bardoli Satyagraha of 1928? achievement in its social and economic dimensions.
(a) Followers of Gandhi Calculation of the Human Development Index combines
(b) Congress Socialist Party four major indicators: life expectancy for health,
(c) Leftist expected years of schooling, mean of years of schooling
(d) Kisan Sabha for education and Gross National Income per capita for
Ans. (a) : Bardoli Satyagraha of 1928 was led by standard of living.
Followers of Gandhi in Bardoli, Gujarat against 8. 'TRYSEM' is a Programme of
enhancement of land revenue. It was led by Sardar (a) Rural Development
Vallabh Bhai Patel (b) Industrial Development
3. Who was the founder of Ghadar Party? (c) Urban Development
(a) Barkatullah (b) Lala Hardayal (d) Deference Development
(c) Bhagat Singh (d) Lala Lajpat Rai Ans. (a) : The scheme TRYSEM (Training of Rural
Ans. (b) : Lala Hardayal was the founder of Ghadar Youth for Self-Employment), launched in 1979, aimed
Party. The Ghadar Movement was formed in 1913 by at providing basic technical and entrepreneurial skill to
expatriate punjabis in the United States with shared the rural poor in the age group of 18-35 years and
leadership from Sikhs, Hindus and Muslims. enable them take up income generating activities.
4. Who is considered as the "Liberator of Indian 9. In which year was the Life Insurance
press" Corporation (LIC) of India founded?
(a) Augustus Hicky (b) Charles Metcalfe (a) 1953 (b) 1956
(c) MacGrath (d) Disaraeli (c) 1954 (d) 1958
Ans. (b) : Charles Metcalfe (1835-36), the then Ans. (b) : Life Insurance Corporation (LIC) of India
Governor General of India, lifted the restriction was created on 1st September, 1956, with the objective
imposed on Indian press by the press Act 1835. Thus he of spreading life insurance much more widely and in
is considered as the liberator of Indian press. particular to the rural areas with a view to reach all
5. When did Jawaharlal Nehru became President insurable persons in the country, providing them
of Indian national Congress for the first time? adequate financial cover at a reasonable cost.
(a) 1929 (b) 1927 10. When did first Industrial Policy of India come
(c) 1921 (d) 1932 into existence?
Ans. (a) : In 1929, Jawaharlal Nehru was elected as (a) March, 1942 (b) May, 1946
the President of the Lahore Session of the Indian (c) April, 1948 (d) June, 1952
National Congress, where Complete Independence for Ans. (c) : first Industrial Policy of India came into
the country was adopted as the goal. existence in April 1948, which is also known as
6. New Industrial Policy of India, 1991 is based on Industrial Policy Resolution of 1948.
(a) Liberalization (b) Privatization 11. Fourteenth Finance Commission (2015-20)
(c) Globalization (d) All of the above submitted its report to the president of India on
UK RO-ARO (Pre) Exam-2016 347 YCT
CLICK HERE FOR FREE MATERIAL

(a) December, 2014 (b) January, 2015 17. The concept of 'Vicious Circle' of poverty is
(c) October, 2016 (d) May, 2017 related to
Ans. (a) : The Fourteenth Finance Commission, headed (a) Karl Marx (b) R. Nurkse
by Dr. Y. V. Reddy was constituted by the President (c) Adam Smith (d) None of the above
under Article 280 of the Constitution on 2 January, Ans. (b) : The concept of 'Vicious Circle' of poverty is
2013 to make recommendations for the period 2015- 20. given by Professor Ragnar Nurkse.
It submitted its report on 15th December, 2014. The 18. Which of the following does not contain
Finance Commission is required to recommend the carbon?
distribution of the net proceeds of taxes of the Union (a) Diamond (b) Graphite
between the Union and the States and the allocation (c) Coal (d) Sandstone
between the States of the respective shares of such
proceeds. Ans. (d) : Among the given options, sandstone does not
contain carbon. Sandstone is made from grains of sand.
12. The Rural Infrastructure Development Fund These rocks may also contain fossils of plants, animals
(RIDF) has been created under and other microorganisms that once lived on them.
(a) RBI
19. Heavy water is a
(b) NABARD
(a) Coolant (b) Moderator
(c) Ministry of Agriculture
(c) Kind of medicine (d) Fuel
(d) Ministry of Rural Development
Ans. (b) : Heavy water also known as deuterium oxide
Ans. (b) : In 1995-96, Government of India created the (D2O), is used as a moderator in some reactors because
Rural Infrastructure Development Fund (RIDF) under it slows down neutrons effectively and also has a low
NABARD (National Bank for Agriculture and Rural probability of absorption of neutrons.
Development).
20. The chemical name of Vitamin C is
13. From which sector in India, is the "Maximum (a) Folic Acid (b) Citric Acid
Gross Domestic Savings" obtained?
(c) Ascorbic acid (d) Lactic acid
(a) Public sector (b) Household sector
Ans. (c) : The chemical name of Vitamin C is ascorbic
(c) Private sector (d) None of the above
acid.
Ans. (b) : Maximum gross domestic saving in India is
21. Which of the following metals, can be cut by a
obtained from Household sector.
knife?
14. In which year did NITI AAYOG came into (a) Silver (b) Aluminum
existence?
(c) Zinc (d) Sodium
(a) January 01, 2014 (b) January 26, 2014
Ans. (d) : Generally Alkali metals such as lithium,
(c) January 26, 2015 (d) January 01, 2015
sodium and potassium are so soft that they can be cut
Ans. (d) : National Institute for Transforming India with a knife. Among the given options, sodium can be
(NITI Aayog) was formed via a resolution of the Union cut by a knife. Rest of the metals are very hard and it is
Cabinet on 1 January 2015. NITI Aayog is the premier not possible to cut them by knife.
policy think tank of the Government of India, providing
22. "Chipko Movement' is related to
directional and policy inputs.
(a) Project Tiger (b) Plant breeding
15. Which of the following was the first Five Year (c) Forest conservation (d) Tissue culture
Plan period?
Ans. (c) : The Chipko movement was a non violent
(a) 1946-50 (b) 1951-56
agitation in 1973 that was aimed at protection and
(c) 1952-57 (d) 1953-58 conservation of forest. The name of the movement
Ans. (b) : India's first Five Year Plan covered ‘chipko’ comes from the word ‘embrace’, as the
the period from April 1951 to March 1956. It focused villagers hugged the trees and encircled them to prevent
primarily on the development of the primary sector, being hacked.
specifically agriculture and irrigation. 23. Which of the following is a bio-technological
16. When was National Statistical Commission process?
(NSC) constituted? (a) Curding of milk in the stomach
(a) 1 June, 2000 (b) 10 June, 2002 (b) Preparation of yoghurt or curd
(c) 24 May, 2004 (d) 12 July, 2006 (c) Making of pickle
Ans. (d) : The National Statistical Commission (NSC) (d) None of the above
was constituted on 12th July 2006 with a mandate to Ans. (b) : Use of microbes to produce and process
evolve policies, priorities and standards in statistical material is called biotechnology. Preparation of yoghurt
matters. The NSC has four members besides a or curd is a bio-technological process. Curd is made
Chairperson, each having specialization and experience by mixing a yoghurt starter with warm milk and
in specified statistical fields. allowed to ferment for some hours.

UK RO-ARO (Pre) Exam-2016 348 YCT


CLICK HERE FOR FREE MATERIAL

24. Anemia is caused due to deficiency of which of 31. The tenure of RBI Governor is
the following vitamins? (a) 2 years (b) 3 years
(a) A1 and B2 (b) B6 and B12 (c) 4 years (d) 5 years
(c) A and D (d) E and K Ans. (b) : The tenure of RBI Governor is 3 years. The
Ans. (b) : Anemia is caused due to deficiency of Reserve Bank of India was established on April 1, 1935
vitamin B6 and B12 and folate (B9). Anemia is a in accordance with the provisions of the Reserve Bank
condition in which body does not have enough healthy of India Act, 1934.
red blood cells. 32. Chabahar Port will be jointly built by
25. Elaioplasts store (a) Iran, Afghanistan, India
(a) Starch (b) Iraq, Afghanistan, India
(b) Proteins (c) India, Pakistan, Iran
(c) Fats (d) Pakistan, Iraq, India
(d) Essential amino acids Ans. (a) : Chabahar Port is a joint effort of India, Iran
Ans. (c) : An elaioplast is a leucoplast that is primarily and Afghanistan to support landlocked Afghanistan by
involved in storing fats or lipids inside fat droplets in giving access to the open seas.
plants. 33. Which organization was fined by the National
26. Doors made of wood, swell up during rainy Green Tribunal for damaging Biodiversity of
season due to Yamuna at Delhi during the World Cultural
(a) Transpiration (b) Imbibition Festival?
(c) Exosmosis (d) Guttation (a) Green Peace
Ans. (b) : Doors made of wood, swell up during rainy (b) Art of Living
season due to Imbibition. Imbibition is a special type of (c) World Culture Society
diffusion when water is absorbed by solids, causing (d) None of the above
them to enormously increase in volume. Ans. (b) : Art of Living foundation was fined by the
27. Which of the following is referred to as the National Green Tribunal for damaging Biodiversity of
"Graveyard of RBC' s" ? Yamuna at Delhi during the World Cultural Festival.
(a) Liver (b) Spleen 34. Which Team won the IPL 2016 Championship?
(c) Bone marrow (d) Intestine (a) Kings XI - Punjab
Ans. (b) : Spleen is referred to as the Graveyard of (b) Sun Risers-Hyderabad
RBC's. After completion of its lifespan, RBCs are (c) Mumbai Indians
destroyed in spleen where they are ingested by free (d) Rising Pune supergiant
macrophages.
Ans. (b) : Sun Risers-Hyderabad won the IPL 2016
28. When ice melts, its volume Championship. The 2022 IPL Championship was won
(a) Decreases by Gujarat Titans.
(b) Increases
35. How many satellites were put in orbit, by
(c) Decreases and then Increases ISRO, through PSL V-C-37?
(d) Remains the same (a) 2 (b) 9
Ans. (a) : When ice melts, it becomes denser (0ºC to (c) 104 (d) 11
4Cº) due to which its volume decreases.
Ans. (c) : PSLV-C37 was 39th flight of PSLV. It
29. Plants add water to the atmosphere in a launched 104 Satellites.
process called
36. Which is the 29th State of India?
(a) Condensation (b) Precipitation
(a) Jharkhand (b) Uttarakhand
(c) Runoff (d) Evapotranspiration
(c) Andhra Pradesh (d) Telangana
Ans. (d) : Evapotranspiration is a combined process by
Ans. (d) : On 2nd June, 2014, Telangana was formed as
which water moves from the Earth's surface into the
atmosphere. It covers evaporation and transpiration. 29th State of India.
Note- Currently India has 28 States and 8 Union
30. Bajaj Auto has launched a motorcycle built Territories.
from metal of an Indian Naval ship. The name
of that ship is; 37. Jeev Milkha Singh is associated with which
(a) INS Vikramaditya (b) INS Vikrant sport?
(c) INS Chakra (d) INS Virat (a) Chess (b) boxing
(c) Golf (d) Wrestling
Ans. (b) : Bajaj Auto has launched a motorcycle built
from metal of an Indian Naval ship INS Vikrant. Indian Ans. (c) : Jeev Milkha Singh is an Indian Professional
naval ship (INS) Vikrant was the first aircraft carrier, Golfer. In 2007, the Government of India honoured him
which played a pivotal role in the 1971 Indo-Pak war. with the Padma Shri.

UK RO-ARO (Pre) Exam-2016 349 YCT


CLICK HERE FOR FREE MATERIAL

38. Which of the following won the Women 44. Right to Information Act in India was passed in
Single's Title of Wimbledon Tennis (a) 2004 (b) 1999
Tournament 2017? (c) 2005 (d) 2009
(a) Venus Williams (b) Garbine Muguruza Ans. (c) : The Right to Information Act, 2005 has come
(c) Serena Williams (d) Johanna Konta into force from 12th October, 2005. The Right to
Ans. (b) : Garbine Muguruza won the Women's Singles Information Act empowers citizens to seek any
Title of Wimbledon Tennis Tournament, 2017. accessible information from a public authority in order
39. Panama Papers Scandal related to: to promote transparency and accountability in the
working of every public authority.
(a) International Film Piracy
(b) Scams by big shots 45. Total number of Union Territories in India is
(c) Copyright (a) 5 (b) 6
(d) Match fixing (c) 7 (d) 8
Ans. (d) : After the creation of Jammu and Kashmir and
Ans. (b) : The Panama papers Scandal involved a leak
Ladakh as Union Territory and merger of Daman Diu
of confidential documents from Panamanian Law firm
& Dadra and Nagar Haveli in August 2019, India has
Mossack Fonseca. Panama Papers Scandal related to
28 States and 8 Union Territories.
scams by big shots. Through Panama Papers, authorities
came to know about the tax evasion by the some high 46. The Election Commission of India is
profile names and their alleged illegal entities. (a) Legal Institution
40. Which one of the following states has bi- (b) Administrative Institution
cameral legislature? (c) Statutory Institution
(a) Tamil Nadu (b) Tripura (d) Constitutional Institution
(c) West Bengal (d) Bihar Ans. (d) : The Election Commission of India is a
Constitutional Body. Article 324 of the constitution
Ans.(d): 6 Indian States have bi-cameral legislature
provides for Election commission of India.
namely Andhra Pradesh, Maharashtra, Karnataka,
Telangana, Bihar and Uttar Pradesh. 47. How many members of Anglo-India
community are nominated to Lok Sabha by the
41. Indian Federation closely resembles the
President of India?
federation of
(a) 12 (b) 8
(a) USA (b) Australia
(c) 4 (d) 2
(c) Switzerland (d) Canada
Ans. (d) : Two members of Anglo-Indian community
Ans. (d) : India is a federation with a unitary bias and were been nominated to Lok Sabha by the President of
thus referred by some scholars as a quasi-federal State India However 104th Constitutional Amendment Act,
because of strong central government. It is similar to 2019 removed the nomination -based representation of
that of Canada. the Anglo-Indian Community in Lok Sabha and
42. 93rd Amendment of the Constitution deals with Legislative Assemblies.
(a) Panchayati Raj Institutions 48. Pro-tem Speaker of Lok Sabha is appointed by
(b) Fundamental Duties (a) President of India
(c) North-Eastern States of India (b) Vice-President of India
(d) Reservation of OBC's in educational (c) Prime Minister of India
Institutions (d) Chief Justice of India
Ans. (d) : The Constitution 93rd Amendment Act, Ans. (a) : Pro-tem Speaker of Lok Sabha is appointed
2005, enables the State to make special provisions for by President of India. Pro-tem speaker is the temporary
members of the Schedule Caste, Schedule Tribes and speaker who presides over the first meeting of lower
socially and educationally backward classes, for House of Parliament (Lok Sabha) after the general
admission in all educational institutions, including elections. Pro-tem Speaker presides over the sitting in
private unaided institutions. which the Speaker is elected for the newly elected
43. Political parties in India are recognized by House.
(a) Central Government 49. Under Article 70 of the constitution of India,
(b) State Government who will officiate as the President of India
(c) Supreme Court of India during the absence of President and the Vice-
(d) Election Commission of India President?
Ans. (d) : Political parties in India are recognized by (a) Speaker of the Lok Sabha
Election Commission of India. The registration of all (b) Prime Minster of India
political parties is governed by the provisions of Section (c) Chief Election Commissioner of India
29A of the Representation of the People Act, 1951. (d) Chief Justice of India
UK RO-ARO (Pre) Exam-2016 350 YCT
CLICK HERE FOR FREE MATERIAL

Ans. (d): Chief Justice of India will officiate as the (c) Vishnu Ganga-Dhauli (East)
President of India during the absence of President and (d) Bhagirathi- Jadganga
the Vice-President. Ans. (c) : Vishnu Ganga is tributary of Alaknanda river.
50. The official language of the Supreme Court of Sarju, Panar and Dhauli (East) rivers are tributary of
India is Kali river.
(a) Hindi-Devnagri Jodh Ganga is tributary of Bhagirathi.
(b) Urdu Persian Thus pair (c) is incorrectly matched.
(c) English 56. Which of the following climatic conditions are
(d) 18 Languages listed in the constitution of found in the Bhabar belt of Uttarakhand?
India (a) Alpine (b) Sub-Tropical
Ans. (c) : According to Article 348 of the Constitution, (c) Tropical
the official language of the Supreme Court of India is (d) Temperate
English.
Ans. (b) : Sub-Tropical climate conditions are found in
51. Under whose Chairmanship, was the Second Bhabar belt of Uttarakhand. Bhabar belt of Uttarakhand
Administrative Reforms Commission comprises tropical moist forest. The Bhabar belt of
Constituted in 2005? Uttarakhand lies mainly in Garhwal and Nainital-
(a) Y.V. Reddy (b) V. Moily Pilibhit region.
(c) Manmohan Singh (d) V. Subramanyam 57. Which one of the following mountain ranges
Ans. (b) : The Second Administrative Reforms falls between the main boundary fault and the
Commission was setup by the Government of India on main central thrust in Uttarakhand?
31 August, 2005 under the Chairmanship of Shri M. (a) Shivalik Range (b) Lesser Himalaya
Veerappa Moily with the mandate to suggest measures (c) Great Himalaya (d) Tethys Himalaya
to achieve a proactive, responsive, accountable,
Ans. (b) : Lesser Himalaya falls between the main
sustainable and efficient administration for the country
boundary fault and the main central thrust in
at all levels of the government.
Uttarakhand. The Main Central Thrust, which is known
52. 'Manodaya kavya' was composed by by different names in different regions, divides the
(a) Mansah Lesser Himalaya from the Great Himalayas.
(b) Shambhu Prasad Bahuguna 58. What was "Atta-Batta" in Uttarakhand?
(c) Medhekar (a) Social belief (b) Marriage System
(d) Bharat kavi (c) Ritual (d) Religious Festival
Ans. (d) : 'Manodaya Kavya' was composed by Bharat Ans. (b) : Atta is marriage system prevalent in Gaddi
kavi. Bharat kavi was poet in the court of Garhwali
tribe of Uttarakhand.
king-Manshah.
59. In which fair of Uttarakhand does the tradition
53. Purushottam Pant was rewarded by which
of killing fish exist?
king?
(a) Maun fair (b) Chariti fair
(a) Rudrachandra (b) Kalyanchandra
(c) Gendi fair (d) Syaldey Bikhoti fair
(c) Lakshmi Chandra (d) Uddhyotchandra
Ans. (a) : Maun or Maund is a fishing festival
Ans. (a) : Purushottam Pant was rewarded by
celebrated in Aglar Yamuna Valley of Tehri Garhwal.
Rudrachandra, the king of Chandra dynasty who ruled
Kumaun region of Uttarakhand. 60. What is 'Chaufula?
(a) Tehsil (b) Folk instrument
54. The Credit for establishing Bacteriological
Laboratory at Mukteshwar, is given to (c) Dance (d) Holy place
(a) Dr. .T. Edward Ans. (c) : Chaufula is folk dance in Garhwal region of
(b) Dr. Alfred Lingard Uttarakhand which is performed during the festival of
(c) A.W. Siltatan Baishakhi.
(d) Prof. Robert Cock 61. According to 2011 census, in Uttarakhand
Ans. (b) : Imperial Bacteriological Laboratory was State which district had lowest literacy rate?
established in 1881 at Pune under leadership of Dr. (a) Uttarkashi
Alfred Lingard. In 1893 it was shifted to Mukteshwar in (b) Tehri-Garhwal
Kumaon. (c) Haridwar
55. Which of the following river-pairs do not (d) Udham Singh Nagar
match? Ans. (d) : Udham Singh Nagar (73.10%) has the lowest
(a) Saryu-Panar literacy rate in Uttarakhand. Dehradun (85.24%) has the
(b) kali-Gauri Ganga highest literacy rate followed by Nainital (83.9%).
UK RO-ARO (Pre) Exam-2016 351 YCT
CLICK HERE FOR FREE MATERIAL

62. According to Census 2011, the sex-ratio of Ans. (a): Arithmetic density is ratio between total land
Uttarakhand state is to total population. It is expressed as number of person
(a) 932 (b) 951 per unit area.
(c) 956 (d) 963 69. From the following, which is the Southernmost
Ans. (d) : The average sex ratio is the number of point of India?
females per 1000 males. As per the Census 2011, (a) Indira point (b) Shastri point
the Average Sex Ratio of Uttarakhand is 963 which is (c) Modi point (d) Ram point
above than national average of 943 females per 1000 Ans. (a) : Indira point, situated on Great Nicobar
males. islands is the southernmost point of India.
63. Ancient name of 'Uttarkashi' was 70. Height of Bhakra Nangal Dam is
(a) Sudarshan khestra (b) Barahat (a) 406 metres (b) 316 metres
(c) Laksheshwar (d) Varnavarta (c) 226 metres (d) 186 metres
Ans. (b) : Ancient name of 'Uttarkashi' was Barahat. Ans. (c) : Height of Bhakra Nangal Dam, situated
64. Where is the Headquarters of Garhwal across Sutlej river, is 740 ft (226 meters). The Bhakra-
Regiment' situated? Nangal Dam is the second tallest dam in Asia and
(a) Chakrata (b) Ranikhet located on the border of Punjab and Himachal Pradesh.
(c) Dehradun (d) Lansdowne 71. According to the Census of 2011, Which state
Ans. (d) : Garhwal Rifles as a Regiment came into has the highest population?
being on 05 May, 1887. The headquarter of Garhwal (a) Uttar Pradesh (b) Bihar
Regiment was established at Lansdowne on 1 October, (c) West-Bengal (d) Kerala
1921. Ans. (a) : According to the Census of 2011, Uttar
65. The founder of Kartikeyapura dynasty was Pradesh has the highest population. Sikkim is the least
(a) Lalitsurdeva (b) Bhudeva populous state in India.
(c) Basantan (d) Kalyanraj dev 72. Sunderban Delta is famous for which tree?
Ans. (c) : The founder of Kartikeyapura dynasty was (a) Rosewood (b) Sal
Basantan. Its capital was Joshimath (Chamoli district). (c) Sundari (d) Shisham
66. Established in 1969 A.D. the first Ans. (c) : The sunderbans mangrove Forest, one of the
Superintendent of Kumaon Government largest mangroves forest in the world lies on the delta of
Garden was Ganges, Brahmputra and Meghna rivers on the Bay of
(a) Craw (b) Normen Gill Bengal. The Delta is Famous for Sundari Tree.
(c) R.D. fordam (d) J .G. Burns 73. Which of the following volcanoes occur in the
Ans. (a) : In 1869, first government orchard of Kumaon middle of a crustal plate?
was established at Chaubattiya. Mr. Craw who was a (a) Mount Fujiyama
trained gardner, served as the first superintendent of this (b) Mount St. Helens
Garden. (c) Karkotoa
67. What is the main source of livelihood of rural (d) Mount Kilimanjaro
population in the mountainous part of Ans. (d) : Mount Kilimanjaro, snow capped volcano,
Uttarakhand? located in Tanzania, Africa is situated in the middle of a
(a) Horticulture crustal plate.
(b) Horticulture and Tourism 74. Most of Earth Continuous Permafrost is found
(c) Agriculture and Animal Husbandry in
(d) Agriculture and Trade (a) High altitude mountainous environment
Ans. (c) : Agriculture and Animal Husbandry is the (b) Antarctica
main source of livelihood of rural population in the (c) North America and Siberia
mountainous part of Uttarakhand due to the (d) Greenland
geographical conditions in the region. Ans. (c) : Permafrost is a permanently frozen layer
68. The arithmetic density of population is below Earth's surface. It consists of soil, gravels and
(a) Ratio between total land and total population. sand, usually bound together by ice.
(b) Ratio between total rural land and total Permafrost is often found in Arctic regions such as
population. Greenland, the US state of Alaska, Russia, China and
(c) Ratio between total agricultural land and total Eastern Europe.
rural population. 75. The biome with the shortest growing season is
(d) Ratio between total land and total rural (a) Savana (b) Taiga
population. (c) Chapparal (d) Tundra
UK RO-ARO (Pre) Exam-2016 352 YCT
CLICK HERE FOR FREE MATERIAL

Ans. (d): Tundra biome has shortest growing period. Ans. (b): The Boreal Forest also known as Taiga
Tundra regions have long, cold winters with high winds occupies large regions of both North America and Asia
and average temperatures below freezing point for six to between temperate forest and Tundra. The boreal forests
ten months of year. On average, only six to ten weeks of hold the largest pool of living biomass of the
year have sufficiently warm temperatures for plant coniferous trees.
growth. 81. Which of the following is an example of
76. Xerophytic plants mostly occur in Isostasy?
(a) Tundra and Taiga (a) Formation of Andaman Islands
(b) Midlatitude and Mediterranean forests (b) Formation of Rocky mountains
(c) Grasslands and deserts (c) Sinking of the Mississippi Delta
(d) Savanna and tropical rain forests (d) The San Andreas Fault
Ans. (c) : Xerophytic plants are mostly found in Ans. (c) : Sinking of the Mississippi Delta is is an
Grasslands and deserts. Xerophytic plants are adapted example of Isostasy. The surface of earth is irregular.
to live in a dry habitat. Hence, a sort of dynamic equilibrium is always in
77. Which of the following biomes is most likely to operation which is termed as isostasy.
be subject to desertification?
82. Which of the following is not a programming
(a) Tropical grasslands language?
(b) Midlatitude forests (a) VB (b) C
(c) Mountains
(c) Oracle (d) Java
(d) Boreal forests
Ans. (c) : Oracle is relational data base management
Ans. (a) : Tropical grasslands are most likely to be system. Rest are programming language.
subject to desertification. Tropical grasslands occur on
either side of the equator and extend till the tropics. 83. Which of the following is responsible for data
This type of vegetation grows in the areas of moderate transfer between Memory and ALU?
to low amount of rainfall. The grass can grow very tall, (a) ROM (b) Internet
about three to four meters in height. Savannah (c) Control unit (d) RAM
grasslands of Africa are of this type. Ans. (c) : Control unit is responsible for data transfer
78. During the Summer Monsoon in South-East between Memory and ALU (Arithmetic and logical
Asia unit).
(a) Weather is usually clear and dry 84. Which of the following is known as Brain of
(b) Winds blow Southward from Central Asia Computer?
(c) Air flows into Asia from the Indian Ocean (a) Monitor (b) Mouse
(d) Air flows low to high pressure (c) Key Board (d) C.P.U.
Ans. (c) : The summer monsoon is associated with Ans. (d) : CPU (Central Processing Unit) is known as
heavy rainfall. It usually happens between April and brain of the computer. It performs all the arithmetic and
September. As winter ends, warm, moist air from the logical operations.
southwest Indian Ocean blows toward countries like
India, Sri Lanka, Bangladesh and Myanmar. The 85. TCP stands for
summer monsoon brings a humid climate and torrential (a) Text control Protocol
rainfall to these areas. (b) Transmission Control Protocol
79. Man's Impact upon permafrost includes (c) Transmission Communication Protocol
(a) Increase in depth of permafrost (d) Transmission Calibrated Protocol
(b) Gradual pole ward shift of Permafrost zone Ans. (b) : TCP stands for Transmission Control
(c) Protecting permafrost from animals Protocol. Transmission Control Protocol is a transport
(d) Inducing thermal erosion layer protocol that facilitates the transmission of packets
(data) from source to destination.
Ans. (d) : Human impact such as construction activities
in permafrost results in stream flows, soil shrinkage, 86. .html is an example of
land slumping and salinisation, resulting in significant (a) Domain (b) Database
permanent environmental damage. Out of the given (c) Protocol (d) File extension
options inducing thermal erosion would be correct Ans. (d) : Html is an example of file extension. Web
option. pages use a language called the Hyper Text Markup
80. The most important plant resource of the Language (HTML). The browser applications
boreal forest environment is (Microsoft's Internet Explorer, Mozilla's Firefox, etc.)
(a) Wheat (b) Forest products are designed to interpret HTML to display text, images
(c) Citrus fruits (d) Natural rubber and using hyperlinks, navigate the web.

UK RO-ARO (Pre) Exam-2016 353 YCT


CLICK HERE FOR FREE MATERIAL

87. Generally www pages are developed by Ans. (a) : 'Buland Darwaza' at Fatehpur Sikri was built by
(a) URL (b) HTML Akbar to commemorate the Mughals victory over Gujarat.
(c) IRS (d) FTP
95. Which one of the following ruler had
Ans. (b) : WWW(World Wide Web), which is also introduced lkta first in India
known as a Web, is a collection of websites or web (a) Iltutimish (b) Balban
pages stored in web servers and connected to local
(c) Allauddin Khilji (d) Razia
computers through the internet. The building blocks of
the Web are web pages which are formatted in HTML Ans. (a) : Ikta system was introduced in India during
and connected by links called hypertext or hyperlinks the regime of Iltutimish. In the Iqta system, the lands of
and accessed by HTTP. the Delhi Sultanate were divided into several large and
88. One NIBBLE is equal to––––bits. small tracts called Iqta and these Iqtas were assigned to
(a) 2 (b) 4 the Sultan’s soldiers, officers and nobles.
(c) 6 (d) 8 96. Who introduced Mansabdari System in India?
Ans. (b) : One NIBBLE is equal to 4 bits. (a) Shahjahan (b) Jahangir
(c) Akbar (d) Aurangzeb
89. What type of device is a ROUTER?
(a) Input (b) Output Ans. (c) : Mansabdari System was introduced by Akbar
(c) Networking (d) Telenet in India. Mansabdari was a Mughal administrative
system which has military as well as bureaucratic
Ans. (c) : A router is a networking device that can apparatus which was responsible for looking after the
receive the data, analyses it and transmit it to other
civil and military affairs of the state.
networks. A router connects a local area network (LAN)
to the internet. 97. Kabir was brought up by a Muslim weaver
named:
90. Computer Virus affects
(a) Software (b) Hardware (a) Rahim (b) Ramanand
(c) Computer language (d) None of the above (c) Neeru (d) Chaitanya
Ans. (a) : Computer Virus affects software. A virus is a Ans. (c) : Kabir was brought up by a Muslim weaver
piece of software code created to perform malicious named Neeru and his wife Nima. Kabir Das was an
activities and hamper resources of a computer system like Indian poet, saint followed by almost all the religions.
CPU memory, personal files or sensitive information. 98. Who was the leading exponent of the
91. Which Satvahana king designated himself title 'Chaukhambha Theory?
of Ekbrahman? (a) M.G. Ranade
(a) Yagnashree Satakarni (b) Mahatma Gandhi
(b) Satakarni (c) Ram Manohar Lohia
(c) Gautamiputra Satakarni (d) Opposition to social reforms
(d) Vashisthiputra Satakarni Ans. (c) : Chaukhambha Theory was propounded by
Ans. (c) : Satavahana king Gautamiputra Satakarni Ram Manohar Lohia to remove poverty.
designated himself the title of Ekbrahman. 99. What was the main objective of Wahabi
92. What is meant by 'BALI' in the Vedic period? movement?
(a) Sacrifice (a) Good relations with the British
(b) Bull (b) Purification of Islam
(c) Hereditary (c) Support of English education
(d) Gift to ruler by the subjects (d) Opposition to social reforms
Ans. (d) : 'Bali' is referred in Rigveda as a voluntary Ans. (b) : The leader of the Wahabi movement (1830-
offering to the king by his subjects.
1860) was Syed Ahmed of Rae Bareilly who was
93. Which Buddhist text refers to 'Solah Maha greatly influenced by the teachings of Abdul Wahab of
janpadas? Arabia and Shah Waliullah, a Delhi saint. The
(a) Anguttar Nikaya (b) Mahavansh movement was primarily religious in its origin and was
(c) Deegh Nikaya (d) Mahavagga meant for the purification of Islam.
Ans. (a) : The Janpadas were the major kingdoms of 100. Which one of the following socio-religious
ancient India. With the development of iron in parts of movement raised voice in favour of the
UP and Bihar, the Janpadas became more powerful
depressed class?
and turned into Mahajanapadas. Buddhist text
Anguttar Nikaya mentions about existence of (a) Brahmo Samaj (b) Prarthana Samaj
Mahajanapadas. (c) Arya Samaj (d) Satya Shodhak Samaj
94. By whom was the 'Buland Darwaza' built? Ans. (d) : Mahamana Jyotiba Phule laid the foundation
(a) Akbar (b) Jahangir of 'Satya Shodhak Samaj' in 1873 to take forward his
(c) Shahjahan (d) Babur movement in favour of the depressed class.
UK RO-ARO (Pre) Exam-2016 354 YCT
CLICK HERE FOR FREE MATERIAL

UTTARAKHAND RO/ARO (Pre) Exam-2021


GENERAL STUDIES
Solved Paper
1. In which language Sangam literature is 6. During whose reign the third Buddhist council
written? was organized?
(a) Tamil (b) Sanskrit (a) Ashok (b) Kalashok
(c) Marathi (d) None of these (c) Kanishka (d) Ajat Shatru
Ans. (a) : Sangam literature is written in Tamil Ans. (a) : Third Buddhist council was held at Patliputra
language. Some of the earliest works in Tamil, known (250 BC) under the patronage of Emperor Ashoka of
as Sangam literature, were composed around 2300 years Maurya dynasty. The council was presided over by
ago or the period between the 3rd century BC and 3rd Mggaliputta Tissa.
century A.D. in South India. These texts were called 7. According to Mortimer wheeler and Stuart
Sangam because they were supposed to have been Piggott, the date of Indus Civilization was
composed and compiled in assemblies (known as (a) 2350 B.C.-1700 B.C.
Sangams) of poets that were held in the city of Madurai.
(b) 2500 B.C.-1500 B.C.
The Sangam literature includes Tolkappiyam,
Silappathikaram, Manimekhalai etc. (c) 2500 B.C.-1750 B.C.
(d) 2800 B.C.-2500 B.C.
2. At which place Muslim League was formed in
1906. Ans. (b) : According to famous archaeologist,
(a) Multan (b) Dhaka Mortimer Wheeler and Stuart Piggott, the date of Indus
(c) Deoband (d) Peshawar Civilization was 2500 B.C.-1500 B.C.
Ans. (b) : On 30th December, 1906, Muslim League 8. The author of 'Prabandh Chintamani' was
was formed at Dhaka (now Bangladesh). First President (a) Sandhyakar Nandi (b) Nayachandra Suri
of Muslim League was Aga Khan. (c) Bhavbhuti (d) Merutung
3. Great scholar Ashwaghosh was the Ans. (d) : The author of 'Prabandh Chintamani' was
contemporary of which ruler? Merutung. Prabandh Chintamani is a Sanskrit language
(a) Chandra Gupta-I (b) Skand Gupta collections of prabandhas (historic biographic
(c) Kanishka (d) Harshvardhan narrative).
Ans. (c) : Great scholar Ashwaghosh was the 9. In which language 'Periplus of the Erythraen
contemporary to Kanishka. The most famous Kushana Sea' is written?
ruler was Kanishka, who ruled around 127 AD-150AD. (a) Greek (b) French
He organised 4th Buddhist council, in kashmir in 72AD. (c) German (d) English
Where scholars met and discussed important matters. Ans. (a) : The 'Periplus of the Erythraean Sea' is an
Ashwaghosh, a poet who composed a biography of the eyewitness account of ancient travel
Buddha, the Buddhacharita, lived in his court. to Africa and India via the Red Sea written in Greek
4. Who had constructed the Sudarshan Lake? language.
(a) Tushaspha (b) Parndutta 10. Who was the Viceroy of India at the time of
(c) Pushyagupta Vaishya (d) Chakrapalit foundation of Indian National Congress?
Ans. (c) : Sudarshan Lake was constructed by (a) Lord Lytton (b) Lord Curzon
Pushyagupta Vaishya, the governor of Saurashtra under (c) Lord Dufferin (d) Lord Rippon
Chandragupta Maurya. Ans. (c) : On 28 December 1885, 72 social reformers,
5. Which animal picture is not inscribed in journalists and lawyers congregated for the first session
Ashoka's Sarnath Pillar? of the Indian National Congress at Gokuldas Tejpal
(a) Horse (b) Elephant Sanskrit College, Bombay. Lord Dufferin (1884-1888)
(c) Lion (d) Rhinoceros was the Viceroy of India at that time.
Ans. (d): While noticing Ashoka's Sarnath Pillar, at 11. In which year Gadar party was formed?
first glance four majestic lions are visible, roaring and
(a) 1911 (b) 1914
facing the four cardinal directions. They represent
power, courage, pride, confidence. The lions sit atop a (c) 1913 (d) 1916
cylindrical abacus, which is adorned with Ans. (c) : Gadar Party was formed in 1913 by the
representations of a horse, a bull, a lion and an elephant, migrant Punjabis in America under the leadership of
made in high relief. Lala Hardayal.
UK RO-ARO (Pre) Exam-2016 355 YCT
CLICK HERE FOR FREE MATERIAL

12. In which year Vernacular Press Act was (c) Sir Saiyyad Ahmad Khan
passed? (d) Khan Abdul Gaffar Khan
(a) 1878 (b) 1875 Ans. (b): Mohammad Ali Jinnah used to say "I want to
(c) 1879 (d) 1881 be Gokhale of Muslims".
Ans. (a) : The Vernacular Press Act, 1878 was enacted 18. Which Sultan of Delhi wrote his
in British India to limit the freedom of the Indian- autobiography?
language (non-English) press. Lord Lytton, then viceroy (a) Sikandar Lodi
of India (1876-80), proposed the Act to prevent the
(b) Muhammad bin Tughlaq
Vernacular Press from criticising British policies,
particularly the opposition that had grown since the start (c) Iltutmish
of the Second Anglo-Afghan War (1878-80). (d) Firoz Tughlaq
13. Gandhi-Irwin Pact was signed in the year : Ans. (d) : Firoz Shah Tughlaq was the ruler of Delhi
(a) 1930 (b) 1931 Sultanate from 1351 to 1388. Firoz Shah Tughlaq wrote
(c) 1932 (d) 1935 his autobiography which is entitled Futuhat-e-
Firozshahi.
Ans. (b) : Gandhi-Irwin (then Viceroy of India) Pact
was signed in 1931, by the terms of which Civil 19. When did Shyam Ji Krishna Verma establish
Disobedience was called off, all prisoners were released 'India House' in London?
and salt manufacture allowed along the coast. (a) 1901 (b) 1903
14. Poona Pact was signed on (c) 1905 (d) 1907
(a) 23rd October, 1930 Ans. (c) : In February, 1905, Shyam Ji Krishna Verma
(b) 24th September, 1931 founded the Indian Home Rule Society. He then
(c) 24th September, 1932 established India House in Highgate (at 65 Cromwell
(d) 24th November, 1933 Avenue) in July, 1905, as a hostel for Indian students,
which became a meeting place for Indian
Ans. (c) : Poona Pact was signed on 24th September, revolutionaries in London.
1932. It gave the Depressed Classes reserved seats in
Provincial and Central Legislative Councils, but they 20. How many members of Uttarakhand are
were to be voted in by the general electorate. represented in the Lok Sabha?
(a) 03 (b) 05
15. When was the Non-Cooperation Movement
withdrawn? (c) 08 (d) 11
(a) 1920 (b) 1922 Ans. (b) : There are five Lok Sabha Constituencies in
(c) 1931 (d) 1927 Uttarakhand namely Almora, Garhwal, Nainital,
Haridwar and Tehri Garhwal.
Ans. (b) : Chauri- Chaura incident (4 February, 1922)
occured at Gorakhpur in which a large group of protesters 21. The right to reshuffle the council of Ministers
participating in the Non-Cooperation Movement, clashed rest with
with police, who opened fire. In retaliation, the (a) The President (b) The Cabinet
demonstrators attacked and set fire to a police station, (c) The Prime Minister (d) The Speaker
killing all of its occupants. The incident led to the deaths Ans. (c) : The right to reshuffle the council of Ministers
of three civilians and 23 policemen. Mahatma Gandhi, rest with the Prime Minister or when a head of
who was strictly against violence, halted the Non- government rotates or changes the composition of
Cooperation Movement on the national level on 12 ministers in their Cabinet.
February 1922, as a direct result of this incident. 22. Article 280 of Indian Constitution is associated
16. Third Battle of Panipat took place in the year with
(a) 1757 (b) 1761 (a) Finance Commission
(c) 1764 (d) 1769 (b) Election Commission
Ans. (b) : The Third Battle of Panipat took place on 14 (c) Public Service Commission
January 1761, at Panipat between a northern (d) Union Public Service Commision
expeditionary force of the Maratha Empire and a Ans. (a): Article 280 of Indian Constitution is
coalition of the King of Afghanistan, Ahmad Shah associated with Finance Commission. Its core
Durrani (also called Ahmad Shah Abdali) with two responsibility is to evaluate the State of finances of the
Indian Muslim allies namely the Rohilla Afghans of the Union and State Governments, recommending the
Doab and Shuja-ud-Daula, the Nawab of Awadh. sharing of taxes between them, lay down the principles
17. Who used to say? "I want to be Gokhale of determining the distribution of these taxes among States
Muslims" : and between Union & States.
(a) Maulana Mohammad Ali 23. The resolution for removing the Vice-president
(b) Mohammad Ali Jinnah of India can be moved in
UK RO-ARO (Pre) Exam-2016 356 YCT
CLICK HERE FOR FREE MATERIAL

(a) Lok Sabha alone 29. The word "secular" and "socialist" were added
(b) Either House of Parliament in the Preamble of the Indian Constitution by
(c) Joint sitting of Parliament (a) 39th Amendment (b) 41st Amendment
nd
(d) Rajya Sabha alone (c) 42 Amendment (d) 44th Amendment
Ans. (d) : According to Article 67(b) of the Ans. (c) : The word "Secular", "Socialist", and
Constitution, a Vice-President may be removed from his "Integrity" were added to the Preamble of Indian
office by a resolution of the Rajya Sabha passed by a Constitution in 1976 through the 42nd Constitutional
majority of all the then members of the Rajya Sabha and Amendment.
agreed to by the Lok Sabha (Effective Majority). 30. The Green Highways policy in India was
24. The Constitution of India was adopted on introduced in
(a) 15th August, 1947 (a) 2015 (b) 2017
(b) 9th December, 1946 (c) 2019 (d) 2020
(c) 26th November, 1949 Ans. (a) : The Green Highways (Plantation,
Transplantation, Beautification and Maintenance)
(d) 26th January, 1950
policy in India was introduced in 2015. The aim of the
Ans. (c) : On 26th November 1949, the Constituent policy is to promote greening of Highway corridors
Assembly of India adopted the Constitution of India, with participation of the community, farmers, private
which came into effect from 26th January 1950. sector, NGOs and government institutions.
25. The disputes regarding the election of 31. Who was the Chairman of the 15th Finance
President are decided by Commission of India?
(a) The Supreme Court of India (a) Rakesh Raman (b) Ramesh Chand
(b) The Parliament (c) Urjit Patel (d) N.K. Singh
(c) The Lok Sabha Ans. (d) : The Fifteenth Finance Commission was
(d) The Election Commission constituted on 27 November, 2017 under the
Ans. (a) : According to Article 71(1) of the Chairmanship of N.K. Singh.
Constitution, all doubts and disputes arising in 32. According to World Bank's annual Report
connection with the election of a President or Vice- 2020 on 'The Ease of Doing Business', the
President shall be inquired into and decided by the India's rank is
Supreme Court, whose decision shall be final. (a) 77th (b) 72nd
th
26. The President can vacate his office by (c) 68 (d) 63rd
addressing his resignation to Ans. (d) : The rank of India was 63 in the Ease of
(a) The Vice-President Doing Business Report, 2020 released in October, 2019.
(b) The Speaker of Lok Sabha 33. The belief that economic growth will percolate
(c) The Prime Minister down to all sectors, is known as
(d) The Chief Justice of India (a) Pump Priming (b) Trickle Down
Ans. (a) : According to Article 56 (1) (a) of the (c) Dispersal Theory (d) None of above
Constitution, the President can vacate his office by Ans. (b) : The belief that economic growth will
addressing his resignation to Vice-President. percolate down to all sectors, is known as Trickle
27. The Judges of the Supreme Court hold office Down. The Theory of Trickle Down considers GDP and
till they reach the age of growth as the most reliable measure of economic
success. Economic growth in upper layer of economy
(a) 58 years (b) 60 years
will automatically and gradually percolate down and
(c) 62 years (d) 65 years reach at the bottom layer eventually benefiting poor
Ans. (d) : The Judges of the Supreme Court hold office sections and in this way become helpful in removing
till they reach the age of 65 years. For High Court poverty.
Judges, retirement age is 62 years. 34. Which one of the following is not a greenhouse
28. How many members are nominated for the gas?
Rajya Sabha by the President? (a) Methane (b) Ozone
(a) 08 (b) 10 (c) Oxygen (d) Nitrous Oxide
(c) 12 (d) 15 Ans. (c) : Earth's greenhouse gases trap heat in the
Ans. (c) : In Rajya Sabha, 12 members are nominated atmosphere and warm the planet. The main gases
by the President (Article 80). The members nominated responsible for the greenhouse effect include carbon
by the President are persons having special knowledge dioxide, methane, nitrous oxide, water vapor and
or practical experience in respect of such matters as synthetic fluorinated gases such as Chlorofluorocarbons
literature, science, art and social service. (CFCs). Hence, option (b) is the right answer.
UK RO-ARO (Pre) Exam-2016 357 YCT
CLICK HERE FOR FREE MATERIAL

35. What is the share of agriculture in India's GDP (a) Italy (b) France
in 2020-21 as per Economic Survey- (c) Libya (d) Germany
(a) 17.8% (b) 19.9% Ans. (d): Germany do not have coast along
(c) 16.8% (d) 17.2% Mediterranean Sea. The Mediterranean Sea region, the
Ans. (b) : As per Economic Survey, 2020-21, share of largest of the semi-enclosed European seas is
agriculture in India's GDP was 19.9% and in 2021-22 it surrounded by 22 countries, which together share a
was 18.8% coastline of 46, 000 km. Countries having coastal line
36. In which year the 'Smart Cities Mission' was with Mediterranean Sea are Italy, Libya, France, Spain,
launched? Turkey etc. Germany shares its coastal line with Baltic
(a) 2015 (b) 2016 Sea and North Sea.
(c) 2017 (d) 2018 42. Which of the following mountain pass is not
Ans. (a): Central government used a competitive located in the state of Uttarakhand?
process to select 100 cities for upgradation with (a) Lipulekh Pass (b) Shipki La Pass
significant investments through the Smart Cities (c) Mana Pass (d) Noma Pass
Mission (SCM) launched on June 25, 2015. Ans. (b) : Shipki La Pass connects Kinnaur district of
37. The Fiscal deficit in the Union Budget 2021-22 Himachal Pradesh to the Tibetan Autonomous Region
of India is targeted at in China. Lipulekh Pass, Mana Pass and Noma Pass are
(a) 3.0% of GDP (b) 4.5% of GDP located in Greater Himalayan region of Uttarakhand.
(c) 5.5% of GDP (d) 6.8% of GDP 43. Which of the following state of India does not
Ans. (d) : The Fiscal deficit in the Union Budget 2021- have common boundary with Bangladesh?
22 of India is targeted at 6.8% of GDP. Fisca deficit (a) Assam (b) Tripura
target for 2022-22 is set at 6.4% of GDP. (c) Manipur (d) Meghalaya
38. National Institute of Plant Genome Research is Ans. (c) : India shares 4,096.7 Km of its largest land
located at border with Bangladesh. West Bengal, Assam,
(a) Gurugram (b) Ludhiana Meghalaya, Tripura and Mizoram are the States, which
(c) New Delhi (d) Mathura share the border with Bangladesh.
Ans. (c) : National Institute of Plant Genome Research 44. Tropic of Cancer does not pass through which
(NIPGR) is located at New Delhi. The National Institute of the following state?
of Plant Genome Research (formerly known as National (a) Gujarat (b) Rajasthan
Centre for Plant Genome Research) is an autonomous (c) Tripura (d) Arunanchal Pradesh
institution aided by the Department of Biotechnology, Ans. (d) : The Tropic of Cancer (23° 30'N) divides
Government of India. India into almost two equal parts. Tropic of Cancer
39. Which of the following is presently largest passes through 8 Indian States namely Gujarat,
goods export partner of India? Rajasthan, Madhya Pradesh, Chhattisgarh, Jharkhand,
(a) China (b) U.A.E. West Bengal, Tripura and Mizoram.
(c) Hong Kong (d) U.S.A. 45. The Tribes of 'Lepcha' and 'HO' are commonly
Ans. (d) : U.S.A. is presently largest goods export found in which part of India?
partner of India followed by China which recently (a) Central India (b) North-East India
replaced UAE. (c) South India (d) West India
40. Which one among the following is not Cold Ans. (a) : Lepcha is a tribe of Himalayan range which
Current? live at the North-East corner of India. They largely
(a) California Current (b) Benguela Current reside at Meghalaya, Arunachal Pradesh, Bhutan,
(c) Oyashio Current (d) Norwegian Current Sikkim and Darjeeling. Lepchas are Mongoloid tribe.
Ans. (d) : Among the given options, Norwegian Current Their language is an admixture of Nepalese and
is not Cold Current. The wedge-shaped Norwegian Sikkimese languages, which is very familiar with Indo-
Current also known as the Norwegian Coastal Current Chinese language. They themselves call "Rong". The
flows northward along the West Coast of Norway. Rest 'Ho' is a Kolarian tribe belonging to the same stock of
of the currents given in the option is Cold Current. Munda and Kol. They mainly inhabit the interiors of
California Current flows through North Pacific Ocean, Mayurbhanj and Keonjhar districts of Odisha. They are
Benguela Current flows through South Atlantic Ocean. settled agriculturists. Besides, they also depend on food
And Oyashio Siwo Current flows through North Pacific gathering and seasonal forest collections, animal
Ocean. domestication, hunting and fishing to supplement their
41. Which of the following countries do not have livelihood. UKPSC had given (a) as right answer in its
coast along Mediterranean Sea? official answer key.
UK RO-ARO (Pre) Exam-2016 358 YCT
CLICK HERE FOR FREE MATERIAL

46. Which of the following Indian State has highest Ans. (d): Known as the 'Quadrilateral Security
road density? Dialogue' (QSD), the Quad is an informal strategic
(a) Bihar (b) Uttar Pradesh forum comprising four nations, namely United States of
(c) Kerala (d) Tamil Nadu America, India, Australia and Japan. One of the primary
Ans. (c) : Kerala has the highest road density among objectives of the Quad is to work for a free, open,
other Indian States. Every nook and corner of the State prosperous and inclusive Indo-Pacific region. It was
has motorable roads with a well-developed transport formed first time in 2007 and re-established after
system operating in public and private sector enabling a negotiations in November 2017.
seamless movement of people and goods across the 53. Which Company in India has produced the
State. The State has 8 major National Highways. 'Covovax'?
47. Which of the mineral resource is connected (a) Pfizer
with 'Kodurite' intrusive rock? (b) Bharat Biotech
(a) Iron (b) Manganese (c) Serum Institute of India
(c) Copper (d) Kaolin (d) Cadilla
Ans. (b) : Kodurite ores are regarded as hybrid rocks Ans. (c) : The vaccine, named Covovax, is produced by
formed by assimilation of manganese ore and the Serum Institute of India under license from
manganese silicate rocks by an acid igneous magma. Novavax. It was 9th covid-19 vaccine for emergency
They are found in Andhra Pradesh and Odisha. use approved by WHO.
48. Hirakud project utilizes which river water? 54. Who has been conferred 29th 'Saraswati
(a) Godawari (b) Narmada Samman' award?
(c) Son (d) Mahanadi (a) Padma Sachdev (b) Vasdev Mohi
Ans. (d) : Hirakud dam is built across river Mahanadi at (c) Nirmal Anjan (d) Vikram Seth
about 15 km upstream of Sambalpur town in Odisha. Ans. (b) : Vasdev Mohi has been conferred 29th
Hirakud Dam Project is a multipurpose scheme 'Saraswati Samman' award for his short stories
intended for flood control, irrigation and power collection Chequebook. Ramdarsh Mishra was awarded
generation.
Saraswati Samman for 2021 for his collection of poems
49. The Indira Gandhi Canal project draws water 'Main to yaha hoon' on 7th April 2022.
from which of the following rivers?
55. The two countries which are connected with
(a) Ravi (b) Beas
'Maitri Setu Bridge'?
(c) Satluj (d) All of these
(a) India & Nepal (b) India & Sri Lanka
Ans. (d) : The Indira Gandhi Canal was primarily (c) India & Bangladesh (d) India & Myanmar
designed to support irrigation in the Thar Desert region
of the state of Rajasthan in Western India. Its total Ans. (c) : India and Bangladesh are connected with
length is around 837 Km. Major source of water for 'Maitri Setu Bridge'. 1.9 km long Maitri Setu Bridge is
Indira Gandhi canal is Satluj, Beas and Ravi rivers. built over Feni river that connects Sabroom district in
South Tripura to Ramgarh in Bangladesh.
50. Who was appointed 'CAG' of India in 2020?
(a) Naresh Agarwal (b) Rakesh Sharma 56. Who is the Chief of International Monetary
(c) G.C. Murmu (d) Amrit Nahta Fund?
Ans. (c) : Shri Girish Chandra Murmu assumed office (a) Kristalina Georgieva (b) Thomson Kary
as the Comptroller and Auditor General of India on 8th (c) Lesli Bon (d) Henkel Joy
August, 2020. Prior to this, Shri Murmu was the first Ans. (a) : The current managing director and Chief of
Lieutenant Governor of the Union Territory of Jammu the International Monetary Fund (IMF) is Bulgarian
and Kashmir. economist Kristalina Georgieva, who has held the post
51. Which Minister resigned from the council of since October 1, 2019. The IMF was conceived in July
Ministers in India due to controversial Farm 1944 at the United Nations Bretton Woods Conference
laws? in New Hampshire, United States. The IMF's primary
(a) Harsimrat Kaur Badal mission is to ensure the stability of the international
(b) Ramvilas Paswan monetary system, the system of exchange rates and
(c) Sanjiv Baliyan international payments that enables countries and their
(d) None of these citizens to transact with each other.
Ans. (a) : Harsimrat Kaur Badal, leader from Akali Dal, 57. Who has been appointed as President of
resigned from the council of Ministers in India due to Olympic Committee till 2025?
controversial Farm laws. (a) Thomas Bach (b) Oliver Martin
52. Which country is not part of 'Quad'? (c) Geoff Sanders (d) Mohammed Ata
(a) India (b) Japan Ans.(a): Thomas Bach has been re-elected as
(c) Australia (d) New Zealand International Olympic Committee President until 2025.
UK RO-ARO (Pre) Exam-2016 359 YCT
CLICK HERE FOR FREE MATERIAL

58. Who won the IPL 2020? established by Som Chand. One of most powerful ruler
(a) Mumbai Indians of Chand dynasty was Baz Bahadur (1638–78) AD,
(b) Rajasthan Royals who met Shahjahan in Delhi and in 1655 joined forces
(c) Chennai Super Kings with him to attack Garhwal, which was under its king,
(d) Kolkata Night Riders Pirthi Sah and subsequently captured the Terai region
including Dehradun, which was hence separated from
Ans. (a) : Mumbai Indians won the IPL 2020 by
the Garhwal kingdom.
defeating Delhi Capitals in final. This was there 5th title
in IPL. Gujarat Titans is the winner of IPL 2022. 64. Which one of the following is not correctly
matched?
59. Louse Gluck, Noble Winner for Literature in
2020 is from which country? Name Area of Work
(a) Germany (b) Canada (a) Bishni Devi Freedom Fighter
(c) France (d) U.S.A. (b) Deepa Devi Social Workers
Ans. (d) : The Nobel Prize in Literature 2020 was (c) Gaura Devi Environment Supporter
awarded to Louise Gluck from U.S.A for her (d) Tulsi Devi Political Worker
unmistakable poetic voice that with austere beauty Ans. (d) : Tulsi Devi was a freedom fighter who was
makes individual existence universal. Annie Ernaux is actively involved during Quit India Movement. Rest are
the Noble Winner for Literature in 2022. correctly matched. Bishni Devi was the first lady from
60. Which one of the following lake is located in Uttarakhand who went to jail during freedom struggle.
Champawat district of Uttarakhand? Deepa Devi was social worker who fought against the
(a) Shyamalatal (b) Taragtal liquor addiction. Gaura Devi played an important role
(c) Chorabari Tal (d) Dodital during Chipko Movement.
Ans. (a) : Shyamalatal lake is located in Champawat 65. Who among the following has taken the reign
district of Uttarakhand. Vivekanand Ashram is located of Kumaun from the Gorkhas?
on the banks of Shyamalatal lake. Taragtal is in Almora (a) Henry Ramsay (b) E. Gardner
district, Chorabari Tal is in Rudraprayag and Dodital is (c) Trail (d) Lushington
in Uttarkashi.
Ans. (b) : For a short span of time, Gorkhas ruled
61. Veer Chandra Singh Garhwali Swarozgar Kumaun region. The people of Kumaon used the British
Yojana started by Government of Uttarakhand many times to help them overthrow the Gorkha rule.
is related to Under the leadership of E. Gardner, British took over
(a) Tourism (b) Medical the control of Kumaun from Gorkhas.
(c) Primary Education (d) Reverse Migration
66. Which one of the following is not correctly
Ans. (a) : Uttarakhand's first self-employment scheme matched?
Veer Chandra Singh Garhwali Tourism Swarozgar Wildlife Sanctuary District
Yojana has been introduced with the aim of providing (a) Govind Uttarkashi
maximum self-employment to the residents of
(b) Askot Pithoragarh
Uttarakhand State and mainly youth in the tourism
sector. (c) Binsar Almora
(d) Sona Chamoli
62. Which one of the following river has the
highest number of glaciers in the area of its
origin? Ans. (d) : Wildlife Sanctuary District
(a) Yamuna (b) Bhagirathi Govind Uttarkashi
(c) Alaknanda (d) Pindar Askot Pithoragarh
Ans. (b) : River Bhagirathi originates from the Gangotri Binsar Almora
glacier near Gaumukh (3,900 m) in the Uttarkashi Sona Nainital
district of Uttarakhand. Gangotri glacier has many 67. In which year publication of 'Almora Akhbar'
tributaries: Raktavarna, Chaturangi, Satopant and Kirti started?
glaciers. UKPSC had given option (c) as right answer. (a) 1870 (b) 1871
63. In which book Mughal and Chand dynastic (c) 1878 (d) 1948
relation has been described? Ans. (b) : Publication of Almora Akhbar (news paper)
(a) Jahangirnama (b) Baburnama started in 1871. It was the first newspaper from Kumaun
(c) Akbarnama (d) Nuskha-i-Dilkusha which was published for continuous 48 years. Buddhi
Ans. (a) : The relation between Mughal and Chand Ballabh Pant, Munshi Imtiaz and Vishnu Datt Joshi
dynasty has been described in Jahangirnama, the official were few publisher of Almora Akhbar among many
biography of Jahangir. The Chand kingdom was during its 48 years of publication.
UK RO-ARO (Pre) Exam-2016 360 YCT
CLICK HERE FOR FREE MATERIAL

68. In which year Indian Military Academy was 73. In which year ‘Tehri Rajya Prajamandal’ was
established in Dehradun? established?
(a) 1919 (b) 1925 (a) 1935 (b) 1937
(c) 1932 (d) 1948 (c) 1939 (d) 1941
Ans. (c) : Indian Military Academy, Dehradun was Ans. (c) : Tehri Rajya Prajamandal’ was established by
established on 1st October, 1932 with a course strength Sridev Suman in 1939 in Dehradun.
of 40 Gentlemen Cadets. In 1949, the Academy was 74. In which year the Garhwal Rifles Cantonment
redesignated as the Armed Forces Academy which later was established at Lansdown?
became the National Defence Academy (NDA) in 1950. (a) 1858 (b) 1869
69. In which Hindi month, the "Bikhoti Mela" is (c) 1877 (d) 1887
celebrated in Uttarakhand? Ans. (d) : Garhwal Rifles Cantonment was established
(a) Baishakh (b) Ashadh at Lansdowne in 1887. Two battalions of Regiment with
(c) Margshirsh (d) Falguna a British Battalion fought in World War I (1914) as part
Ans. (a) : Celebrated in a small town of Dwarahat in the of the Garhwal Brigade in France.
Almora District of Kumaon region, Bikhoti Mela is an 75. In which place of Garhwal region the first tea
annual fair which is held every year in the month of factory was setup in the year 1843?
Vaisakh (April/May). Held in two different phases, (a) Havalbag (b) Lohaba
Bikhoti Mela is first held in Vimandeshwar Temple, (c) Gadoli (d) None of these
which is a popular Lord Shiva temple that is situated 8 Ans. (c) : First tea factory was setup in Gadoli at
km from Dwarahat. The second phase is held in the Garhwal region in 1843. In 1834, East India Company
Dwarahat market. sent a person to China for bringing tea seeds and
70. At Which of the following place, High Altitude Chinese workers for growing tea in India. By 1844, tea
Plant Physiology Research Institute of plants started growing in Koth, Ramasirain and Gadoli.
Uttarakhand is located? 76. Under whom was the district administration in
(a) Srinagar (b) Pauri the British Garhwal?
(c) Tungnath (d) Haldwani (a) District Magistrate
Ans. (a) : High Altitude Plant Physiology Research (b) Collector
Centre (HAPPRC), Srinagar, Uttarakhand was (c) Commissioner
established in July 1979 by the University of Garhwal at (d) Deputy Commissioner
the initiative of its Founding Director Prof. A.N. Ans. (d) : In the British Garhwal, district administration
Purohit. was under the control of Deputy Commissioner.
71. In ancient times the un-irrigated land form of Britishers captured eastern Garhwal and Kumaon in
Uttarakhand was known as 1815 by defeating Gorkhas. For administrative
(a) Ijar (b) Talaun convenience, they established Kumaon and Garhwal as
(c) Uparaun (d) None of these separate districts and positioned Deputy Commissioner
in each district as administrative officer.
Ans. (c) : In ancient times the un-irrigated land form of
UKPSC has given option (c) as right answer.
Uttarakhand was known as Uparaun. Most of the area of
Uttarakhand is under forests and wastelands thus 77. Which of the following is not an agricultural
leaving only a small amount of land for agriculture. apparatus in Uttarkhand?
Such type of land was divided in two parts namely (a) Kutala (b) Hal
Uparaun and Doyam. (c) Maya (d) Karboja
72. Who was the founder of Katyuri Dynasty? Ans. (d) : Among the given options, Karboja is not an
(a) Somchand (b) Vasudev agriculture apparatus. Karboja is a small carry bag,
(c) Thorchand (d) Jaidev which is used to carry goods on the back of sheep and
goats.
Ans. (b) : The Katyuri Dynasty was of a branch of
Kunindas origin and was founded by Vashudev Katyuri. 78. In which year the ‘Kumaon Parisad’ was
Originally, from Joshimath, during their reign they established?
dominated lands of varying extent from the 'Katyur' (a) 1908 (b) 1912
(modern day Baijnath) valley in Kumaon, and (c) 1916 (d) 1920
established their capital at Baijnath in Bageshwar Ans. (c) : Kumaon Council was established on 30
district. At their peak, the Katyuri kingdom extended September, 1916, in Majhera. Govind, Hargovind
from Nepal in the east to Kabul, Afghanistan in the Pandey, Laxmi Shastri, Idrpal Shah and Mohan Joshi
west. The Chand Kings displaced them in 11th century contributed more in its establishment. In 1926, it was
AD. merged with the Congress.
UK RO-ARO (Pre) Exam-2016 361 YCT
CLICK HERE FOR FREE MATERIAL

79. In which year were the Gorkhas defeated by (c) Male Anopheles (d) Female Anopheles
Raja Pradyuman Shah with the help of Ans. (a) : Yellow fever virus is transmitted to people
Britishers? primarily through the bite of infected Aedes or
(a) 1803 A.D. (b) 1815 A.D. Haemagogus species mosquitoes. Mosquitoes acquire
(c) 1822 A.D. (d) None of these the virus by feeding on infected primates (human or
Ans. (a) : In 1803, Gorkhas mounted an invasion on non-human) and then can transmit the virus to other
Garhwal. After capturing Kumaun, Gorkhas attacked primates (human or non-human).
Garhwal. King Pradyuman Shah escaped to Dehradun 86. Which of the following is not a flightless bird?
to organize his defense. King Pradyuman Shah died in (a) Kiwi (b) Emu
the battle of Khurbura in Dehradun in 1804. Gorkha (c) Ostrich (d) Sarus-Crane
rule in the Garhwal area ended in 1815 when the British
Ans. (d) : The Sarus-Crane is the tallest flying bird in
drove the Gorkhas to the West of Kali river, despite stiff
the World standing 152-156 cm tall with a wingspan of
resistance offered by them.
240cm. Rest are flightless birds.
UKPSC has given option (b) as right answer.
87. Trinitro Tolune (TNT) is a/an
80. Who among the following is considered as the
(a) Anti microbial (b) Anti bacterial
“Father of artificial intelligence”?
(a) Charles Babbage (b) Lee De Forest (c) Explosive (d) Antiseptic
(c) John Mc Carthy (d) J.P. Eckert Ans. (c) : Trinitrotouluene (TNT) is an explosive. TNT
is a yellow, odorless solid that does not occur naturally
Ans. (c) : John McCarthy is considered the Father
in the environment. It is made by combining toluene
of Artificial Intelligence. Artificial Intelligence is
with a mixture of nitric and sulfuric acids and its
defined as the science and engineering of making
Chemical formula is C6H2(NO2)3CH3. Major
intelligent machines.
manufacturing of TNT began in the United States in
81. DNA fingerprinting was discovered by 1916 at the beginning of World War I.
(a) George Stark (b) Sir Alec Jeffreys
88. The first successfully launched satellite of India
(c) Edward Southern (d) Kary Mulis was
Ans. (b) : Sir Alec Jeffreys, professor and geneticist at (a) Aryabhatt (b) Bhashkar-I
the University of Leicester in the United Kingdom (c) INSAT-IA (d) IRS-IE
(UK), pioneered DNA-based identity testing.
Anthropologists use DNA fingerprinting to study Ans. (a) : Aryabhata satellite, named after the famous
evolution in humans and their current global variation Indian astronomer, was India's first satellite. It was
through millions of years. completely designed and fabricated in India and
launched by a Soviet Kosmos-3M rocket from Kapustin
82. From where Corona Virus got its name? Yar on April 19, 1975.
(a) Due to crown like appearance
89. Sickle cell anemia is a disease of
(b) Due to leaf like appearance
(a) Basophils (b) Red blood cells
(c) Due to shape of its genetic material
(c) Platelets (d) Lymphocytes
(d) None of these
Ans. (b) : Sickle cell anemia is an inherited Red blood
Ans. (a) : The word Corona came from the Latin word cells disorder in which there aren't enough healthy Red
“Crown”. Corona Virus got its name from crown like
blood cells to carry Oxygen throughout body. In Sickle
spikes that it has its surface.
cell anemia, some Red blood cells are shaped like
83. Oncology is the study of sickles or crescent moons. These Sickle cells also
(a) Birds (b) Cancer become rigid and sticky which can slow or block blood
(c) Mammals (d) Soil flow.
Ans. (b) : Oncology is a branch of medicine that 90. Who was the President of Indian National
specializes in the diagnosis and treatment of Cancer. Congress at the time of India’s independence?
84. Dolphin is an example of which class? (a) Maulana Abul Kalam Azad
(a) Amphibia (b) Pisces (b) J.B. Kripalani
(c) Aves (d) Mammalia (c) Dr. Rajendra Prasad
Ans. (d) : Dolphin is an example of Mammal. Like (d) Jawaharlal Nehru
every mammal, Dolphins are warm blooded. Unlike Ans. (b) : J.B. Kripalani was the President of Indian
fish, who breathe through gills, dolphins breathe air National Congress at the time of India’s Independence.
using lungs. The government of India declared Ganga He was elected as the Congress Party President in 1946.
river Dolphin as its National Aquatic Animal in 2009. 91. How many numbers from 1 to 100 are there,
85. Yellow fever is transmitted through each of which is not only exactly divisible by 4
(a) Aedes (b) Culex but also has 4 as a digit?

UK RO-ARO (Pre) Exam-2016 362 YCT


CLICK HERE FOR FREE MATERIAL

(a) 7 (b) 10 Ans. (c): LCM of (5,10,8) = 40


(c) 12 (d) 19 Let Man's salary = 40 unit
Ans. (a) : Required numbers are: Remaining money = 40- (16 + 12 + 5)
4, 24, 40, 44, 48, 64, 84 = 7 unit
Hence total number = 7 7 unit → ` 1400
92. X is twice as old as Y at present. Three years 40 unit → ` 8000
ago X was three times as old as Y. How old is X Hence, man's salary = ` 8000
now? 96. Find the missing number in the figure given
(a) 6 years (b) 8 years below :
(c) 9 years (d) 12 years
Ans. (d) : Let age of y = a
age of x = 2a
Three years ago,
2a – 3 = 3 (a – 3)
2a – 3 = 3a – 9
(a) 21 (b) 18
a=6
(c) 29 (d) 35
93. When 13 times of a number is added to the st
Ans. (b) : From 1 figure,
number itself, the result is 112. Find the
number.
3
1 + 3 8 + 3 27 + 3 64 = 10
nd
(a) 7 (b) 8 From 2 figure,
(c) 9 (d) 13 3
8 + 3 27 + 3 64 + 3 125 = 14
Ans. (b) : Hence age of x = 12 years Similarly,
Let number be x. 3
27 + 3 64 + 3 125 + 3 216 = 18
x + 13 x = 112 97. Select the pair from the options that has the
14x = 112 same relationship as the original pair of words:
x=8 YAWN : BOREDOM
Hence, number = 8 (a) Anger : Madness
94. A hall 200 m × 50 m is to be paved with square (b) Dream : Sleep
tiles. If the price of one tile of 10 cm × 10 cm is (c) Impatience : Rebellion
` 3.50. then find the cost of paving the hall the (d) Smile : Amusement
hall. Ans. (d) : As Boredom causes yawn, similarly
(a) ` 25 lakhs (b) ` 30 lakhs Amusement causes simile.
(c) ` 35 laks (d) ` 45 lakhs 98. In a certain code GO = 32, SHE = 49, then
Ans. (c) : Number of tiles to be paved SOME will be equal to :
Area of floor of Hall (a) 62 (b) 58
=
Area of 1tile (c) 64 (d) 56
200 × 50 × 100 × 100 Ans. (d) : Here the code numbers of the letters opposite
= to the given letters are added.
10 × 10
= 1000000 GO → 20 + 12 = 32
Cost of paving the hall = 1000000×3.5 SHE → 8 + 19 + 22 = 49
= ` 35 Lakhs Similarly. SOME → 8 + 12 + 14 + 22 = 56
99. A line A is perpendicular to line B and parallel
2 3
95. A man spends of his salary on food, of to line C. Line D is perpendicular to both line E
5 10 and line F. Line A is perpendicular to line E.
1 Which one of the following statement is
his salary on house rent and of the salary on
8 correct?
clothes. He still has ` 1,400, left with him. His (a) A, D and C are parallel.
salary is (b) C, D and F are parallel.
(a) ` 7,000 (b) ` 7,800 (c) C is perpendicular to A.
(c) ` 8,000 (d) ` 8,400 (d) B and E are parallel and perpendicular to F.
UK RO-ARO (Pre) Exam-2016 363 YCT
CLICK HERE FOR FREE MATERIAL

Ans. (a): 102. Which term comes next in the series?


YEB, WFD, UHG, SKI, ?
(a) QGL (b) QOL
(c) TOL (d) SNL
Ans. (b) :

Line, A, D and C are parallel to one another.


100. At what time between 5 and 6 O, clock are the
hands of a clock 3 minutes apart? Hence, QOL will be next term.
(a) 24 min. past 5 103. In a certain code language, ‘253’ means ‘books
(b) 28 min. past 5 are old’, ‘546’ means ‘man is old’ and ‘378’
means ‘buy good books’, then the code for ‘are’
(c) 17 min. past 5
is
(d) 46 min. past 5 (a) 2 (b) 4
Ans. (b) : Angle made in 3 minutes = 3 × 6 = 18º (c) 5 (d) 6
Angle between minute hand and hour hand (θ) Ans. (a) : In the certain code language,
11
= H × 30 ∼ M
2
11
18 = 5 × 30 ∼ M Hence code of old = 5
2
now buy good books = 3 7 8
11
M = 150 − 18 code of book = 3
2 Hence, code of 'are' = 2
132 × 2
M= =24 Minutes 104. If you are facing North-East and move 10 m
11 forward, turn left and move 7.5 m, then you are
Hence Required time = 24 minute past 5. now:
101. The ratio of A’s age to B’s age at present is 1 : (a) 12.5 m South from initial position
3. The ratio of their ages after 8 years will be 1 (b) 12.5 m East from initial position
: 2. How many years ago was the ratio of their (c) 12.5 m North from initial position
ages was 1 : 5? (d) None of these
(a) 6 years (b) 3 years Ans. (c) :
(c) 5 years (d) 4 years
Ans. (d) : At present,
A 1
=
B 3
⇒ B = 3A(1)
(A + 8) 1
After 8 years, =
( OP ) + ( PN )
2 2
(B + 8) 2 ON =
⇒ B + 8 = 2A + 16
ON = (10)2 + (7.5)2
⇒ 3A + 8 = 2A + 16 (by using (1))
= (10)2 + (7.5)2
⇒ A=8
From equation (1), = 100 + 56.25
B = 8 × 3 = 24
= 156.25
Let ratio of their ages before n years was 1 : 5
ON = 12.5 m
8-n 1
⇒ = Hence position of the person will be 12.5 m in the north
24 - n 5 from the initial point.
⇒ 40 – 5n = 24 – n 105. It was Saturday on 17th December, 2002. What
was the day on 22nd December, 2004?
⇒ 4n = 16
(a) Monday (b) Tuesday
⇒ n = 4 years (c) Wednesday (d) Sunday
UK RO-ARO (Pre) Exam-2016 364 YCT
CLICK HERE FOR FREE MATERIAL

Ans. (d): Ans. (d):


17 Dec. 2002 → Saturday Authors
↓+5 Authors
22 Dec. 2002 → Thursday
22 Dec. 2003 → Friday
22 Dec. 2004 → Sunday (+2)
(learned people) Doctors (learned
106. How many days will be there in ‘X’ weeks and people) Doctors
‘X’ days?
Both I and II conclusions follows.
(a) 7 X2 (b) 8 X
110. Two trains, each being 120 m. long, are
(c) 14 X (d) 7 running on parallel tracks in opposite
Ans. (b) : Number of weeks in x weeks directions, at the speed of 40 meter/min and 20
According to question meter/min, respectively. How much time will
Total number of days in x weeks and x day they take to cross each other?
= 7x + x = 8x (a) 1 minute (b) 2 minute
(c) 3 minute (d) 4 minute
107. Which of the following diagram indicates the
distance
best relation between women, mothers and Ans. (d) : time =
engineers? speed
120 +120
Required time =
 40 20 
 + 
 60 60 
If both trains are moving in opposite directions relative
40 20
(a) A (b) B speed = + = 1m / s
60 60
(c) C (d) D
240
Ans. (a) : According to question- = = 240 Sec
1
T = 4 m.
Engineer 111. Megha ranks 21st in a class of 51. What is her
Woman rank from the last?
(a) 31 (b) 29
Mother (c) 30 (d) 22
Ans. (a) : Megh's rank from the last
108. Write the following in a logical sequence : = (51 – 21) + 1
Lock, Door, Key, Switch on, Room = 31st
(1) (2) (3) (4) (5) 112. Five girls are sitting on a ladder. Sarla is above
1, 2, 3, 4, 5 Chitra but below Daisy. Vimmy is between
(a) 4, 3, 1, 2, 5 (b) 5, 4, 3, 1, 2 Sarla and Chitra. Daisy is between Anu and
(c) 3, 1, 2, 5, 4 (d) 4, 5, 2, 1, 3 Sarla. Who is sitting on top of the ladder?
(a) Daisy (b) Anu
Ans. (c) : Logical sequence are:
(c) Vimmi (d) Chitra
(3) Key, (1) Lock, (2) Door, (5) Room, (4) Switch on
Ans. (b) :
109. Given below are two statements followed by Anu
two conclusions. Based upon these, which
answer is correct? ↑
Statement : (i) Authors are learned people. Daisy
(ii) Some doctors are authors. ↑
Conclusions : (I) Some doctors are learned Sarla
people.
(II) Some learned people are doctors.

(a) Only (I) follows Vimmi
(b) Only (II) follows ↑
(c) Neither (I) nor (II) follows Chitra
(d) Both (I) and (II) follows Hence, Anu is sitting on top of the ladder.
UK RO-ARO (Pre) Exam-2016 365 YCT
CLICK HERE FOR FREE MATERIAL

113. Which figure will come next in the series? ? = 216 + 6 = (6)3 + 6 = 222
Choose from the answers: 350 = 343 + 7 = (7)3 + 7
Hence, option (b) will be required answer.
117. If REASON is coded as r and BELIEVED as 7,
(a) (b) then what is the code of GOVERNMENT?
(a) 7 (b) 8
(c) (d) (c) 9 (d) 10
Ans. (c) : Code-number of letters-1
Ans. (d) : Figure given in option (d) will come next in Just as,
series.
In word REASON there are 6 are 6 letters and it's code
114. In the following figure, find the missing is 6-1 = 5
number : and In word BELIEVED there are 8 – 1 = 7
Same as,
in word GOVERNMENT there are 10 letters
and its code is 10 – 1 = 9
(a) 31 (b) 41 So, required answer will be (c)
(c) 72 (d) 82 118. A group of numbers is given, choose another
Ans. (c) : Just as, similar group of numbers:
120 (32, 24, 8)
4 × 5 × 6 = 120, = 12
10 (a) (36, 32, 42) (b) (34, 24, 14)
210 (c) (24, 16, 0) (d) (42, 34, 16)
& 5 × 6 × 7 = 210, = 21
10 Ans. (c) : Just as
Same as
720
8 × 9 ×10 = 720, = 72
10 Same as from the option (c)
115. Observe the following figure. Which part
represents the students who are singers but not
actors?
Hence, the required alternative will be another number
groups similar to the given no. group.
119. Find the number which is different from others
in the following group :
8314, 2709, 1315, 2518
(a) 8314 (b) 2709
(c) 1315 (d) 2518
(a) a (b) b
Ans. (a) : Option (a) is different from others because in
(c) c (d) d
other numbers the sum of the first three digits is equal
Ans. (d) : It is clear from the picture 'a' that 'A' is only to unit digit.
student. 2709 = 2 + 7 + 0 = 9
(b) is student as well as actor. 1315 = 1 + 3 + 1 = 5
(c) is student as well as singer and actor. 2518 = 2 + 5 + 1 = 8
(d) is Both student and singer but not actor. 8314 = 8 + 3 + 1 = 12 ≠ 4
(e) is both singer and actor.
120. A, P, R, X, S and Z are sitting in a row. S and Z
(f) is only actor. are in the centre and A and P are at the ends. R
(g) is only singer is sitting just on the left of A. Then who is
So it is clear that 'd' is such a student who is a singer but sitting on the right of P?
not a actor. (a) A (b) S
116. In the following, find the missing number : (c) Z (d) X
68 : 130 :: ? : 350 Ans. (d) :
(a) 210 (b) 222
(c) 232 (d) 260
3
Ans. (b) : 68 = 64 + 4 = (4) + 4 H is clear from the above figure that x will be on the
130 = 125 + 5 = (5)3 + 5 right of 'p'. So the correct option will be (d)
UK RO-ARO (Pre) Exam-2016 366 YCT
CLICK HERE FOR FREE MATERIAL

UTTARAKHAND RO/ARO (Pre)


ADVOCATE GENERAL Exam-2022
GENERAL STUDIES
Solved Paper
1. The National Sports Day is observed every year 6. Which SEZ city became the first 'Green SEZ'
across India on which of the following date? in India?
(a) 20th March (b) 11th April (a) Khanna (b) Kandla
th
(c) 25 May (d) 29th August (c) Jamnagar (d) Greater Noida
Ans. (d) : In India National Sports Day is celebrated on Ans. (b) : Kandla SEZ became the first Green SEZ of
29th August each year to commemorate the birth India. It is also the first SEZ to receive IGBC platinum
anniversary of hockey legend, Major Dhyan Chand. The Rating for existing cities.
day is also dedicated to the nation's sports heroes and 7. How many Bronze medal did India win in
champions, honoring their contribution and dedication Tokyo Olympics?
towards bringing laurels to the country. (a) Two (b) Three
2. Dholavira, the southern centre of Harappan (c) Four (d) One
civilization, received the UNESCO Wrold
Ans. (c) : India won total seven medals in Tokyo
heritage site tag recently, is situated in which
state of India? Olympics including 1 gold, 2 silver and 4 bronze
(a) Punjab (b) Gujarat medals. It was India's best haul in Olympics. India's
(c) Madhya Pradesh (d) Jharkhand previous best haul in Olympics was in London
Olympics in 2012 which was of 6 medals (2 silver and 4
Ans. (b) : Dholavira, is situated on the arid island of
bronze).
Khadir of Kachchh district in the state of Gujarat.
8. Name the film which won the 'Palme d'Or' in
3. Name the first Union Territory to become
100% organic as all farming is carried out the Cannes film festival recently
without the use of synthetic fertilizer and (a) Titane (b) No Man's Land
pesticides. (c) Gone with the wind (d) Saving Private
(a) Ladakh (b) Chandigarh Ryan
(c) Daman and Diu (d) Lakshdweep Ans. (a) : The 74th annual Cannes Film Festival took
Ans. (d) : Lakshadweep is the first Union Territory and place from 6 to 17 July 2021. In this event the Palme
2nd UT/state after Sikkim to be declared fully organic by d'Or was awarded to Titane (directed by Julia
the Ministry of Agriculture in 2020. Ducournau).
4. Who was the first President of All India Trade 9. Mahsu, Dangli and Jhali popular folk dances
Union Congress (AITUC)? are associated with which state of India?
(a) Deewan Chaman Lal (b) Lala Lajpat Rai (a) Jharkhand (b) Sikkim
(c) Jawaharlal Nehru (d) C.R. Das (c) Himachal Pradesh (d) Assam
Ans. (b) : The All India Trade Union Congress Ans. (c) : Mahsu, Dangli and Jhali are popular folk
(AITUC) is the oldest trade union federation in India. It dance of Himanchal Pradesh. Jhora, Chharhi, Dhaman,
was founded on 31 October 1920 in Bombay (Mumbai). Chhapeli and Nati are some other folk dance forms of
Lala Lajpat Rai was its first President and founding Himachal Pradesh.
member. Other founding members of AITUC were 10. In which event Neeraj Chopra won India's only
Joseph Baptista, N.M. Joshi and Diwan Chaman Lal. Gold Medal in Tokyo Olympics?
5. Who is the author of the book 'Overdraft: (a) javelin throw (b) Shot put
Saving the Indian Saver'? (c) Boxing (d) 400 M. Race
(a) Kaushik Basu (b) Raghuram Rajan Ans. (a) : Neeraj Chopra won the men's javelin throw
(c) Geeta Sainath (d) Urjit Pateel gold medal at the Tokyo Olympic with the best throw of
Ans. (d) : The book "Overdraft: Saving the Indian 87.58 metres. His was the only gold medal of India in
Saver" is authored by Urjit R. Patel who served as the the event.
twenty fourth governor of the Reserve Bank of India. 11. Which of the following states sponsored India's
The book tries to explain the problems plaguing the Men's Hockey Team and Women's Hockey
Indian banking sector. Team?
UK RO-ARO (Pre) Exam-2012 367 YCT
CLICK HERE FOR FREE MATERIAL

(a) Haryana (b) Odisha 17. Which rock inscription gives an account of the
(c) Punjab (d) Maharashtra achievements of Kalinga King Kharvela?
Ans. (b) : The state government of Odisha is the (a) Karle (b) Hathigumpha
sponsor of India's Men's and Women's hockey team. (c) Junagarh (d) Nasik
The Odisha government has been the official sponsor of Ans. (b) : The Hathigumpha inscription is the main
the Indian Hockey teams since 2018. source of information about the achievements of
12. Which day was observed in India in the year Kalinga King Kharvela. His achievements were
2021 as the 'Partition Horrors Remembrance described in the inscription in a chronological order.
Day'? 18. Who was the author of 'Kathasaritsagar'?
(a) August 16, 2021 (b) August 13, 2021 (a) Somdeva (b) Kalhana
(c) August 17, 2021 (d) August 14, 2021 (c) Rajshekhar (d) Bilhana
Ans. (d) : India observed the Partition Horrors Ans. (a) : Somdeva Bhatt, a sanskrit writer is
Rememberance Day on 14 August, 2021 to mark the considered to be the author of Kathasaritsagar. The
struggles and sacrifices of the people affected by the Kathasaritsagar is a famous 11th century collection of
horrors of partition and forced migration. Indian folk tales and legends.
13. Which of the following committee is not 19. Which of the following books was not written
associatedwith the Panchayati Raj System? by Amir Khusrasu?
(a) Ashok Mehta Committee (a) Nuh Sipihr
(b) Santhanam Committee (b) Deval Rani-Khizr khan
(c) V.K.R.V. Rao Committee (c) Taj-ul-Masir
(d) Balwant Rai Mehta Committee (d) Khazain-ul-futuh
Ans. (*) : Santhanam Committee or the Committee on Ans. (c) : Taj-Ul-Maasir is the first official history of
Prevention of Corruption was related to the menace of the Delhi Sultanate. It was authored by Hasan Nizami.
corruption. It recommended the constitution of the Nuh Siphir (Nine Skies), Deval devi-Khizra Khan (also
Central Vigilance Commission and administrative known as Ashiqa or Ishqia) and Khazain-ul-Futuh are
Vigilance divisions in all departments and major works of Amir Khusrau.
organizations of the government. Also V.K., R.V. Rao
committee was related to National Income estimation. 20. Where is Akbar's Mausoleum situated?
Ashok Mehta Committee, and Balwant Rai Mehta (a) Agra (b) Sikandara
Committees are associated with Panchayati Raj system (c) Lahore (d) Delhi
is India. However the question was deleted by the Ans. (b) : Akbar's Mausoleum is situated at Sikandara,
commission. a sub-urb of Agra. It was built by his son Jahangir
14. Which of the following is the oldest atomic during 1605 to 1613.
power station in India? 21. The commander of East India Company's
(a) Kota (b) Tarapur army in the Battle of Buxar was:
(c) Narora (d) Kalpakkam (a) Robert Clive (b) Admiral Watson
Ans. (b) : Tarapur Atomic Power Station (TAPS), (c) Sir Eyer Coote (d) Hector Munro
located in the industrial area of Boisar in Palghar Ans. (d) : Major Hector Munro commanded the East
district of Maharashtra was India's first atomic power India companies Army in the Battle of Buxar. The battle
project when it began operation on 28 October, 1969 was fought on 22-23 October 1764, between the
[Kota Atomic Power Project (Commissioned in combined armies of Mir Qasim (Nawab of Bengal),
December 1973, Narora (1991), Kalpakkam (1983)] Shuja-Ud-Daula (Nawab of Awadh) and the Mugal
15. Who is re-appointed as the Governor of RBI? Emperor Shah Alam II and the army of the British East
(a) Ashok Bhushan (b) Urjit Patel India Company. The battle ended by the Treaty of
(c) Shaktikant Das (d) None of these Allahabad in 1765.
Ans. (c) : The Government of India reappointed Shri 22. Who is the author of 'Das Kapital'?
Shaktikant Das as Governor of Reserve Bank of India (a) J.J. Rousseau (b) Adam Smith
for a further period of three years. (c) Karl Marx (d) J.S. Mill
16. Where did Gautam Buddha deliver his first Ans. (c) : 'Das Kapital' is authored by German
Sermon? Philosopher and economist Karl Marx in the middle of
(a) Sarnath (b) Bodh Gaya the 19th Century. In this book he describe now the
(c) Kushinagar (d) Rajgrih capitalist system works and now it will destroy itself.
Ans. (a) : Gautam Buddha delivered his first Sermon 23. Buland Darwaza was got built by:
(known as Dhamma Chakra-Pravartana) at Mrigdav, (a) Akbar (b) Shahjahan
Sarnath near Varanasi. (c) Quli Qutub Shah (d) Ibrahim Sharqi
UK RO-ARO (Pre) Exam-2012 368 YCT
CLICK HERE FOR FREE MATERIAL

Ans. (a):Buland Darwaza was built by Mughal Emperor 29. In which year did the 'Bloodless Revolution' or
Akbar in 1575 as the main entrance to the Jama Masjid 'Glorious Revolution' take place in England?
at Fatehpur Sikri to commemorate his victory over (a) 1656 (b) 1678
Gujarat. (c) 1688 (d) 1698
24. Which of the following pairs is not correctly Ans. (c) : Glorious Revolution or Bloodless Revolution
matched? refers to the series of events in 1988-89 which resulted
(a) Lenin-Communism in the exile of king James II and the accession to the
(b) Hitler-Nazism throne of his daughter MaryII and her husband, Willium
(c) Mussolini-Fascism III. It is a watershed event in the development of the
(d) Churchill- Utilitarianism constitution and the role of parliament in England.
Ans. (d) : Winston Churchill is not associated with the 30. Who, among the following, did not contribute
utilitarianism which advocates that the value of an to the unification of Italy?
action is the result of its consequences. However other (a) Matternich (b) Garibaldi
options are correctly matched. (c) Mazzini (d) Cavour
25. Where is Gol Gumbaz situated? Ans. (a) : Metternich was an Austrian diplomat who
(a) Hyderabad (b) Lucknow also served as chancellor of the Austrian Empire during
(c) Patna (d) Bijapur 1821 to 1848. He was not associated with Unification of
Ans. (d) : Gol Gumbaz is the tomb of Mohammad Adil Italy whereas Mazzini, Garibaldi and Cavour were
Shah. It is situated at Bijapur (Vijaypura) in Karnataka. prominent leader of Italy who played important role in
It has the second largest dome ever built, next in size unification of Italy.
only to St. Peter's Basilica in Rome. 31. With which country is the Romanove dynasty
26. Thomas Munro is known for his work in which associated?
field? (a) France (b) Germany
(a) Land settlement (b) Industry (c) Russia (d) Italy
(c) Irrigation (d) Health Ans. (c) : The Romanov dynasty was the last imperial
Ans. (a) : Thomas Munro was one of the most popular dynasty to rule Russia. They came to power in 1613 and
British Administrator in South India. He was famous for went on ruling Russia for next three centuries. Some
introducing a radically deferent method of revenue important Monarch to rule under the dynasty were.
management known as the Rayotwari settlement. In this Peter the Great, Catherine the Great, Alexander I and
settlement revenue was collected directly from Nicholas II.
cultivators.
32. After the first world War, at which place was
27. Arrange the following events of India freedom the Peasce Conference held?
struggle in chronological order: (a) London (b) Paris
(i) Cripps Mission
(c) Vienna (d) Washington
(ii) Cabinet Mission
(iii) Wavell Plan Ans. (b) : After the First World War the peace
(iv) Formation of Interim Government conference was held at Paris. The Paris Peace
(a) (ii), (iii), (iv), (ii) (b) (i), (ii), (iii), (iv) conference was the formal meeting in 1919 and 1920 of
the victorious Allies after the end of First World War to
(c) (iii), (ii), (iv), (i) (d) (i), (iii), (ii), (iv)
set the peace terms for the defeated Central Powers.
Ans. (d) : Events Year
Cripps Mission March 1942 33. In which year did the Battle of Waterloo take
Wavell Plan June 1945 place?
Cabinet Mission February 1946 (a) 1804 (b) 1807
Formation of Interim Government September 1946 (c) 1813 (d) 1815
Therefore the correct chronological order is- (i), (iii), Ans. (d) : The Battle of Waterloo took place on 18 June
(ii), (iv ) 1815 between Napoleon's French Army and a coalition
28. The fall of Bastille is associated with which led by the Duke of Wellington and Marshal Blucher.
Country's Revolution? The battle ended the French attempts to dominate
(a) France (b) Russia Europe and destroyed Napoleon's imperial power
(c) Italy (d) Germany forever.
Ans. (a) : The fall of Bastille is associated with French 34. Nadir Shah invaded India in the year
Revolution. It is the flash point at the beginning of the (a) 1398 (b) 1756
French Revolution. On July 14, 1789 revolutionary (c) 1739 (d) 1761
insurgents of Paris stormed and seized control of Ans. (c) : Nadir Shah was the ruler of Iran and the
Bastille fort. This event is also known as storming of founder of Afsharid dynasty. He invaded northern India
the Bastille. in 1739.
UK RO-ARO (Pre) Exam-2012 369 YCT
CLICK HERE FOR FREE MATERIAL

35. Who among the following is known as sugar refinement and cachaca distillation. However by
'Sakalottarapathnath'? 19th century Fazendas were re-tooled to produce coffee.
(a) Ashoka 41. Darling is a:
(b) Harshavadhan (a) Canal (b) City
(c) Chandragupta Maurya (c) Lake (d) River
(d) Samudragupta Ans. (*) : The Darling River is the third largest river in
Ans. (b) : Harshavardhan, the last ruler of Pushyabhuti Australia. It originates in northern New South Wales
dynasty is also known as 'Sakalottarapathnath' which and joins Murray River at Wentworth, New South
means Lord of the entire northern country. Wales. Along with Murray River it makes largest river
36. To which dynasty did Gautamiputra Satakarni system in Australia. Darling in also a lake in U.S.A.
belong? • Note :- The question was deleted by the commission.
(a) Satvahan (b) Chola 42. Which one of the following is a characteristic of
(c) Parmar (d) Gurjar Pratihar Tropical Rainforests?
Ans. (a) : Gautamiputra Satakarni belonged to the (a) Rainfall in summer season
Satvahana Dynasty. He is considered the greatest ruler (b) Rainfall throughout the year
of Satavahana dynasty. In Nasik inscription of his (c) Rainfall in winter season only
mother Gautami Balashri, he is described as the (d) A definite wet and dry season
destroyer of Shakas, Pahlavas and Yavanas. Ans. (b) : The Tropical Rainforest is a hot, moist biome
37. Which of the following pairs is wrongly where rainfall occurs throughout the year. Other
matched? important characteristics, of Tropical Rainforests are
(a) Prayag Pradshasti-Harishen high average temperature, nutrient poor soil, dense
(b) Junagarh inscription-Rudradaman canopies of vegetation (3 different layers) and high
(c) Bhitari Inscription-Pulkeshin II levels of biodiversity.
(d) Dhauli Inscription-Ashoka 43. The longest river of China is:
Ans. (c) : Bhitari Inscription located at Bhitari of (a) Yangtze (Yang Tse King)
Ghazipur (U.P.) describes the ancestors of Skandgupta, (b) Hwang Ho
about himself and his achievements. It is not related to (c) Wei Ho
Pulkeshin II. Others are correctly matched. (d) Si Kiang
38. After killing whom Pushyamitra Sunga Ans. (a) : Yangtze (Yang Tse Kiang) is the longest
founded the Sunga dynasty? river of China with an approximate length of 6300 kms.
(a) Kunal (b) Sampriati It is also the longest river of Asia and 3rd longest river
(c) Dashrath (d) Brihadrath in the world after the Amazon River and the Nile river.
Ans. (d) : Pushyamitra Sunga was general of the last 44. Which one of the following countries is called,
Mauryan king, Brihadratha. He killed Brihadratha "Land of thunderbolt"?
during a military parade and established the Shunga (a) Nepal (b) Sri Lanka
dynasty in 185-186 BC. (c) Bangladesh (d) Bhutan
39. Which of the following pairs is correctly Ans. (d) : Bhutan is called the land of Thunderbolt due
matched? to the violent and large thunderstorms that flow from
(a) First battle of Tarain-1192 A.D. the Himalayas and extend to the parts of central Asia.
(b) Second Battle of Panipat-1576 A.D. Bhutan is traditionally called as 'Druk-Yul'.
(c) Battle of Chandawar-1194 A.D. 45. The far -northern island of Japan is:
(d) Timur's Invasion on India-1378 A.D. (a) Kyushu (b) Shikoku
Ans. (c) : Battle of Chandawar was fought between (c) Hokkaido (Hokaido) (d) Honshu
Muhammad of Ghor and Jaychand of Kannauj Ans. (c) : Hokkaido is the northernmost of the four
(Gahadwal Dynasty) in 1194. Hence it is correctly main Islands of Japan. It is also the 2nd largest Island of
matched, whereas correct year related to other events in Japan after Honshu Island.
the question are as follows— 46. The cause of wind to deflect towards left in
First Battle of Tarain– 1191 AD Southern hemisphere is-
Second Battle of Panipat – 1556 AD (a) Atmospheric Pressure
Timur's invasion on India – 1398 AD (b) Rotation of the Earth
40. 'Fezenda' is known as: (c) Temperature
(a) Fields of Sugarcane (b) Fields of Cotton (d) Earth's magnetic field
(c) Coffee Plantation (d) Fruit Gardens Ans. (b) : Because of rotation of the Earth on its axis,
Ans. (c) : Large coffee plantation in Brazil is known as circulating air deflect towards left in southern
Fazenda. Fazenda is the Portuguese word for 'farm'. Hemisphere and towards right in Northern Hemisphere.
Fazendas were orginally established to grow sugarcane for This deflection is known as Coriolis effect.
UK RO-ARO (Pre) Exam-2012 370 YCT
CLICK HERE FOR FREE MATERIAL

47. The nearest planet to the earth is: Ans. (b) : Out of the given options Alluvial soils are
(a) Venus (b) Mars most fertile soil in India. It is formed by deposition of
(c) Mercury (d) Jupiter alluvium and sediments carried by rivers and sea waves
Ans. (*) : Though Venus in closed planet to Earth. over many years which makes this soil very fertile.
However sometimes comes closer to earth and becomes 54. The smallest union territory of India in terms
closest plant to earth ( In its course to revolution around of area is:
sun). (a) Puducherry
However this question was deleted by the commission. (b) Daman and Diu
48. The highest mountain peak in the world (c) Dadra and Nager Haveli
outside Asian Continent is: (d) Lakshadweep
(a) Mt. Logan (Mount Logan)
Ans. (d) : Lakshadweep is the smallest Union Territory
(b) Mount Elbrus
of India with an area of 32 km2.
(c) Mount Aconcagua
(d) Mount Kilimanjaro 55. India's first 'Bullet Train Project' is under
construction in between:
Ans. (c) : The highest mountain peak in the world
(a) Mumbai and Ahmedabad (Sabarmati)
outside Asian continent is Mount Aconcagua in the
Argentine Andes with the height of 6962 meters. (b) Mumbai and Pune
(c) Mumbai and Nagpur
49. The highest waterfall in the world is:
(d) Delhi and Kolkata
(a) Victoria fall (b) Angel fall
(c) Niagra falls (d) Tugela fall Ans. (a) : India's first 'Bullet Train Project' is under
construction between Mumbai and Ahmadabad
Ans. (b) : Angel Fall also known as Salto Angel,
(Sabarmati). It is 508 Km long High speed Rail Project
situated at Canaima National Park, Venezuela is the
world's tallest waterfall with an uninterrcepted drop at a constructed by the National High Speed Rail
height of 979 meters. Corporation Limited (NHSRCL).
[Note- Tugela (947m), is the second highest waterfall of 56. Which one of the following is oldest mountain
the world. ] range in India?
50. The fishing area of North Sea in Europe is (a) Satpura (b) Vindhyanchal
known as: (c) Aravali (d) Garo and Khasi
(a) Danish Bank (b) Scandinavian Bank Ans. (c) : The Aravallis is the oldest mountain Range in
(c) Grand Bank (d) Dogger Bank India. It is a type of old rugged folded mountain which
Ans. (d) : The Dogger Bank is the largest sandbank in extends to Haryana, Rajasthan, Gujarat and Delhi. Its
the North Sea. It is a major fishing ground because of highest peak is Guru Sikhar.
mixing of cold Labrador current and warm Gulf stream. 57. Which one of the following is not an oil refinery
The mixing of these waters lifts nutrients to the surface (Petroleum)?
thus creating favourable condition for fishes. (a) Jamnagar (b) Jamshedpur
51. Sambhar Lake is located in: (c) Digboi (d) Barauni
(a) Rajasthan (b) Gujarat Ans. (b) : Jamshedpur, situated in Jharkhand is not
(c) Jammu and Kashmir (d) Madhya Pradesh famous for oil refinery. It is an important industrial
Ans. (a) : The Sambhar Salt Lake is India's largest centre for Iron-Steel industry.
inland salt lake. It is located in Sambhar Lake Town, 58. According to 2011 census, the population
Jaipur district of Rajasthan. density of India is
52. Gandhi Sagar Dam is a part of: (a) 410 (b) 325
(a) Kosi Project (c) 432 (d) 382
(b) Damodar Valley Project Ans. (d) : According to 2011 census, the population
(c) Chambal Project density of India is 382 persons per square kilometers.
(d) Bhakra-Nangal Dam Population density is the concentration of individuals
within a specific geographical area.
Ans. (c) : Gandhi Sagar Dam is a part of Chambal
project. It is one of the four major dams built on 59. Keymore in Madhya Pradesh is famous for
chambal river. Gandhi Sagar dam is located in (a) Cement industry
Mandsaur district of Madhya Pradesh. Other major (b) Paper industry
dams on chambal river are- Rana Pratap Sagar Dam, (c) Aluminum industry
Jawahar Sagar Dam and Kota Barrage. (d) Automobile industry
53. Which one of the following is most fertile soil? Ans. (a) : Keymore or Kymore in Katni district of
(a) Laterite soil (b) Alluvial soil Madhya Pradesh is an important industrial town and
(c) Red soil (d) Sandy loam soil mainly famous for its cement industry.
UK RO-ARO (Pre) Exam-2012 371 YCT
CLICK HERE FOR FREE MATERIAL

60. 'Saputara' hill station lies in Ans. (b) : The charter of 1687 established first Majer's
(a) Jharkhand (b) Maharashtra Court of India at Madras. It was East India company's
(c) Gujarat (d) Madhya Pradesh court. However by the charter of 1726 Majer's court
Ans. (c) : Satpura Hill station is located in Dang district were established at all the presidency towns at Madras,
of Gujarat. It is located on a plateau in Western Ghats. Calcutta and Bombay under direct authority of the
It is probably Gujarat's only hill station. crown.
61. 'Bandipur tiger reserve' lies in 66. which of the following laws provided that
(a) Jammu and Kashmir (b) Karnataka Indians should not be barred from holding.
(c) Madhya Pradesh (d) Uttar Pradesh Jobs with the East India Company?
(a) Pitt's India Act, 1784
Ans. (b) : Bandipur Tiger Reserve is situated in two
(b) Regulating Act, 1733
revenue districts of southern Karnataka namely the
(c) Charter of Act, 1833
Mysore and Chamarajanagar.
(d) India Act, 1858
62. The government of India Act, 1935 was largely
Ans. (c) : The charter act of 1833 legalized the British
based on
colonization of India. It ended the activities of the East
(a) The recommendations of the Nehru
India Company as a commercial body. It made the
Committee Report Governor-General of Bengal as the Governor General
(b) The recommendations of the Simon of India and vested in him all civil and military powers.
Commission This act introduced a system of open competition for
(c) The deliberations of the Three Round Table selection of civil servants and stated that the Indians
Conferences should not be debarred from holding any place, office
(d) Both (b) and (c) and employment under the company. Lord William
Ans. (*) : The Government of India Act 1935 was Bentinck was the first Governor General of India.
largely based on the recommendations of the Indian 67. A federal structure for India distributing
Statutory Commission (popularly known as Simon legislative powers between center and
commission), while paper issued after the discussion at provinces was first introduced by:
the three round table conferences (which gave details of (a) The Government of India Act, 1915
the working basis of the new constitution of India), joint (b) The Government of India Act, 1919
select committee report and Lothian report (which (c) The Government of India Act, 1935
determined electoral provisions of the Act). (d) The Independence of India Act, 1947
Also Nehru report influenced the government of India Ans. (c) : Government of India Act 1935, marked a
Act of 1935. second milestone towards a completely responsible
However the commission has deleted the question. government of India. It was a lengthy and detailed
63. Which charter was called as germ, out of which document having 321 sections & 10 schedules. It
subsequent Anglo-Indian codes were provided for the establishment of an All-India
developed? Federation consisting of provinces and princely states as
(a) Charter of 1600 (b) Charter of 1609 units. The Act divided the power between the centre and
(c) Charter of 1639 (d) Charter of 1666 units in terms of three lists-federal list (for centre with
Ans. (a) : Charter of 1600 AD was called as Germ by 59 items), provincial list (for provinces, 54 items) the
Elbert. He said, "It is the germ out of which the Anglo- concurrent list (for both, 36 items). Residuary powers
Indian Codes were ultimately developed," as the were given to the viceroy. However, the federation
subsequent interpretations of the provisions of the never came into being as the princely states did not join
charter provides the basis for the East-India company of it.
controlling and administering territorial expansion of 68. The system of separate electorates was
the company in India. introduced in India by the Act of:
(a) Act of 1813 (b) Act of 1835
64. In which of the following year Madras Agency
(c) Act of 1885 (d) Act of 1909
was raised to the status of Presidency?
(a) In 1639 (b) In 1665 Ans. (d) : The Indian Councils Act 1909 is also known
(c) In 1686 (d) In 1687 as Morley-Minto Reforms. This Act provided that the
Muslim members were to be elected only by Muslim
Ans. (b) : In 1665 the status of Madras Agency was voters. It is known as system of separate electorates.
raised to the status of presidency. Therefore, Lord Minto came to be known as the Father
65. The first Mayer's Court in India was of communal Electorate and the Act successfully
established in year 1687 Legalized Communalism.
(a) Calcutta (b) Madras 69. The declared purpose of the Wavell plan of
(c) Bombay (d) Patna 1945 was:
UK RO-ARO (Pre) Exam-2012 372 YCT
CLICK HERE FOR FREE MATERIAL

(a) To give dominion status to India 73. First Law Commission of India was constituted
(b) To give a Federal structure to India under charter of:
(c) To give representation to all major (a) 1726 (b) 1813
communities in India (c) 1833 (d) 1835
(d) None of these Ans. (c) : The Charter Act 1833 which was enacted by
Ans. (c) : In May 1945 Wavell visited London and the British Parliament provided for the establishment of
discussed his ideas with the British Government. These a Law Commission for consolidation and codification
London talks resulted in the formation of a definite plan of Indian Laws. The first law commission was formed
of action which was officially made public in 1884 under the chairmanship of Lord Macaulay.
simultaneously on 14 June 1945 by L.S. Amery, the
74. The High courts of Judicature initially
secretary of state for India. The act stated that the
established in India were located at
council should have a 'balanced representation' of all
Indians 'Hindus', Muslims, Depressed class, sikhs etc. (a) In Madras, Calcutta and Allahabad
Muslims were given 6 out of 14 members which (b) In Bombay, Madras and Patna
accounted for more than their share of population (c) In Madras, Bombay and Calcutta
(25%). (d) In Bombay, Madras and Delhi
70. A proclamation was issued on August 1, 1672 Ans. (c) : In 1862, three high courts were established
which fully abolished the: for the first time in India by Queen Victoria during the
(a) English laws (b) Hindu laws British rule in the cities of Bombay (Now Mumbai),
(c) Muslim laws (d) Portuguese laws Madras (now Chennai) and Calcutta (now Kolkata).
Ans. (d) : According to the new judicial plan the 75. Who was the Chairman of First Law
government issued a proclamation on 1st August 1672, commission of India?
declaring the introduction of English law into Bombay. (a) C.H. Cameron (b) T.B. Macaulay
The Portuguese laws and customs were totally (c) G.W. Anderson (d) R. Millert
abolished under this plan. Ans. (b) : The first Law Commission was established
71. Mughal Emperor Shah Alam has granted the during colonial rule in India, by the East India Company
Diwani of Bengal, Bihar and Orissa to east under the Charter Act of 1833. The Commission
India Company in exchange of yearly payment consisted of Lord T.B. Macaulay, as chairman and 4
of Rupees members namely, C.H. Cameron, J.M. Macleod, G.W.
(a) In 10 Lacs (b) In 20 Lacs Anderson and F. Millet.
(c) In 26 Lacs (d) In 30 Lacs 76. Under which Act, Second Law Commission of
Ans. (c) : Treaty of Allahabad, 1765 were concluded in India was appointed?
two parts, in first part Robert Clive negotiated with (a) By the Act of 1833 (b) By the Act of 1938
Nawab of Awadh and in second part of negotiated with (c) By the Act of 1840 (d) By the Act of 1853
Shah Alam II. Under the second treaty Robert Clive
with Shah Alam II the Diwani of Bengal, Bihar and Ans. (d) : Second Law Commission of India was
Orissa were granted to the East India company in lieu of appointed by the Act of 1853. The second Law
an annual payment of Rs. 26 lakhs. Commission to examine and consider the
recommendations of the first Indian Law Commission
72. Which Judicial Plan of Lord Cornwallis
and the enactments proposed by them for the reform of
established a Court of Circuit?
(a) By Judicial Plan of 1787 the judicial establishments, judicial procedure and the
(b) By Judicial Plan of 1790 laws of India and other matters.
(c) By Judicial Plan of 1793 77. The Constituent Assembly for framing the
(d) By Judicial Plan of 1794 Constitution for Independent India was set up
Ans. (b) : Judicial plans were introduced by Lord in:
Cornwallis. (a) In 1945 (b) In 1947
Features of Judicial plan 1790 were – (c) In 1949 (d) In 1946
(a) Reonganisation districts into divisions– The districts Ans. (d) : The Constituent Assembly for framing the
were divided into four divisions– Murshidabad, constitution for Independent India was set up in 1946.
Calcutta, Dacca and Patna. The Constituent Assembly met for the first time in
(b) Moffusil faujdari courts were abolished and were Delhi on 9th December 1946.
replaced with court of circuits. 78. Which of the following Commission
(c) Establishment of court of circuits– A court of circuit recommended abolition of Dyarchy system in
was established at each of the four divisions. It was Provinces?
presided by 2 servants of the company and they were (a) By the Cripp's Commission
assisted by Muslim Law officers– Qazi and Mufits. It (b) By the Simon Commission
was not a stationary court. They visited each district of (c) By the Cabinet Commission
their division twice annually. (d) By the Nehru Report
UK RO-ARO (Pre) Exam-2012 373 YCT
CLICK HERE FOR FREE MATERIAL

Ans. (b): Simon Commission recommended the natural resources or social and economic equity.
abolition of Diarchy system in provinces. Diarchy was The sustainable development for the first time officially
proposed in the Government of India Act, 1919. defined in the Brundtland Report in 1987.
79. What is the meaning of the Latin phrase "Audi 85. "Rule of Law" means:
Alteram Partem"? (a) All persons are equal in the eye of law
(a) No one shall be Judge in his own cause (b) Treating all unequally as equals.
(b) Both sides should be heard before a decision (c) Protective discrimination
is arrived at (d) None of these
(c) Rights are associated with duties Ans. (a) : Rule of law means all persons are equal in the
(d) None of these eye of law. The term rule of law is orginated from
England and India has taken this concept. The world
Ans. (b) : Audi Alteram Parterm a phrase of Latin
'law' in rule of laws means that whether he is a men or a
which also means auditor et altena pars. Its meaning is
society, he must not be governed by a man or ruler but
"Hear the other side" on the other side also heard as by law. In other words, as per Article 13 of Indian
well. This is a strong principle that advocate no person Constitution rule of law means law of India.
shall be judged without a fair hearing. This principle is
86. An act of unauthorized entering into another's
meant to give each party an opportunity to respond to
property is called:
the evidence against them.
(a) Damage of property (b) Trespass
80. The word "malafide" means: (c) Trover (d) None of these
(a) Erroneous (b) Good faith Ans. (b) : An act of unauthorized entering into
(c) Bad faith (d) Intention another's property is called Trespass.
Ans. (c) : The meaning of Malafide is with or bad faith 87. To enforce the Fundamental Rights the Court
or wrong intention. issues:
81. The term "Ipso Facto" means: (a) Writ (b) decree
(a) In place of (b) By reason of itself (c) ordinance (d) None of these
(c) By the same source (d) None of these Ans. (a) : The Supreme Court and High Courts issues
Ans. (b) : The term "Ipso Facto" means by reason of writs under article 32 and article 226 respectively to
itself i.e. You admit you fired the gun and we now enforce Fundamental Rights. While the Supreme Court,
know that the shot killed the victim so you are, ipso to enforce Rights other than fundamental rights, issues
facto, responsible for his death. writs under Article 139.
82. The Constitution of India was amended for the 88. Fundamental Rights are contained in which
first time in: part of the Constitutions of India?
(a) 1950 (b) 1951 (a) In Part II (b) In Part III
(c) 1952 (d) 1953 (c) In Part IV (d) In Part V
Ans. (b) : The Fundamental Rights in India enshrined in
Ans. (b) : The Constitution of India was amended for
the part III (Article 12-32) of the constitution and
the first time on 10th May, 1951 and enacted by
guarantee civil liberties.
parliament on 18th June 1951.
89. National Law Day is observed every year on:
83. 'Habeas Corpus' literally means: (a) March, 01 (b) April, 30
(a) Produce the body (c) June, 16 (d) November, 26
(b) Produce the record
Ans. (d) : National Law Day– also known as
(c) Produce the evidence Samvidhan Divas is celebrated on November 26 every
(d) Produce the testimony year to commemorate the adoption of the Constitution
Ans. (a) : The literal meaning of habeas corpus is of India.
"show me the body" or "produce the body". Habeas 90. Child Labour (Prohibition and Regulation)
corpus has historically been an important instrument to Act, was enacted in:
safeguard individual freedom against arbitrary (a) In 1986 (b) In 1987
executive power. (c) In 1988 (d) In 1989
84. 'Sustainable Development' is a concept of: Ans. (a) : Child Labour (Prohibition and Regulation),
(a) Environmental Law (b) Property Law was enacted in 1986. It is an Act to prohibit the
(c) Tax Law (d) Criminal Law engagement of children in certain employments and to
Ans. (a) : Sustainable development is a concept of regulate the conditions of work of children in certain
environmental law. Specifically, sustainable other employments.
development is a way of organizing society so that it 91. The disobedience of any Judgement, order, etc.
can exist in the long term. This means taking into of a Court amounts to:
account both the imperatives present and those of the (a) Perjury (b) Contempt of Court
future, such as the preservation of the environment and (c) Offence (d) None of these
UK RO-ARO (Pre) Exam-2012 374 YCT
CLICK HERE FOR FREE MATERIAL

Ans. (b): The disobedience of any Judgment, Order, Ans. (b):The eighth Nagar Nigam (Muncipla
etc. of a Court amount to contempt of court. Contempt corporation) declared in Uttarakhand state is Kotdwar in
of court is the offence of being defiant or disrespectful the year 2017.
to the court of law. Being impolite to legal authorities in 97. The 'Gazetteer of British Garhwal' was
the courtroom, or rebelliously failing to follow a court authored by:
order may draw contempt of court proceedings. A judge
(a) H.G. Walton (b) E. Atkinson
can levy sanctions such as a penalty or prison for
someone found guilty of contempt of court. (c) T. Moorcraft (d) Wiliam Frazer
92. The "Devidhura fair" at Varahi Temple is held Ans. (a) : The Gazetteer of British Garhwal was
on the eve of: authored by Henry George Walton and was published in
(a) Rakshabandhan (b) Igas-bagwal 1910.
(c) Makar-Sankranti (d) Deepawali 98. Ramnager Legislative Constituency is a part
Ans. (a) : The Devidhura fair at Varahi Temple is held of:
on Raksha-Bandhan every year. It is celebrated in (a) Udhamsingh Nagar Parliamentary
Almora, Pithragarh and Nainital districts of Constituency
Uttarakhand. (b) Nainital Parliamentary Constituency
93. Veer Canndra Singh Garhwali is famous for (c) Garhwal Parliamentary Constituency
'Peshawar Kand'. This kand occurred in which (d) Almora Parliamentary Constituency
year? Ans. (c) : Ramnagar Legislative Constituency is a part
(a) 1932 (b) 1930 of Garhwal Parliamentary (Lok Sabha) Constituency.
(c) 1919 (d) 1942 Garhwal Lok Sabha Constituency comprises five
Ans. (b) : Born on December 24, 1891 in Mason districts namely Chamoli, Nainital, Pauri Gharwal,
Chauthan, Veer Chandra Singh Gharwali had joined the Rudraprayag and Tehri Gharwal. It also comprises of
Gharwal Rifles in Lansdowne during World War I and fourteen Vidhan Sabha (legislative assembly) segments,
had participated in World War I. of which Ramnagar legislative constituency is one
On April 23, 1930 thousand of people had gathered at which lies in Nainital district.
Kissa Khani market in front of the police station in 99. The easternmost district of Uttarakhand state
Peshwar in protest. The people with the national flag in is:
their hands demanded independence from the British
(a) Champawat (b) Nainital
rule. Women, children and old people were also a part
(c) Udhamsingh Nagar (d) Pithoragarh
of the peaceful protest. What could have resulted in
massive deaths and deep emotional scars was avoided Ans. (d) : The easternmost district of Uttarakhand state
due to the courage of the brave heart-Veer Chandra is Pithoragarh. It has an area of 7090 sq.km and a
Singh Gharwali. population of 483439.
94. The average size of land-holding (in hectares) 100. 'Sar System' in Uttarakhand is associated with:
in Uttarakhand is (a) Mix farming
(a) 0.69 (b) 0.79 (b) Irrigation system
(c) 0.99 (d) 0.89 (c) Crop-Rotation system
Ans. (*) : The Average Size of landholding in the State (d) Plantation agriculture
is 0.95 ha as against the National Average of 1.57 ha. Ans. (c) : Sar System in Uttarakhand is associated with
Note : All the given options are incorrect, hence the Crop-Rotation System. people of Uttarakhand
question was deleted by Commission. developed a special practice of agriculture, in this
95. According to Census of India 2011 which of the people cultivate crops on a 2 year rotation.
following districts of Uttarakhand have 101. The Gorkhas (Gurkhas) had captured Kumaon
negative population growth? in:
(a) Almora and Nainital (a) 1803 (b) 1894
(b) Garhwal and Rudraprayag (c) 1796 (d) 1790
(c) Garhwal and Almora
Ans. (d) : The Gorkhas belonged to Nepal, taking
(d) Almora and Bageshwar advantage of the weakness of the Chand rulers in
Ans. (c) : According to Census of India 2011, Garhwal Kumaon, in 1790 AD, they took control of Almora by
and Almora districts of Uttarakhand have negative fighting a small war. After capturing Kumaon, they
population growth i.e. -1.41% and -1.28% respectively. attacked Garhwal in 1791 but was defeated and in
96. The eighth Nigam (Municipal Corporation) february 1803, the Gurkhas again attacked Garhwal and
declared in Uttarakhand State is: succeeded.
(a) Rishikesh (b) Kotdwar 102. Match the following and select the correct
(c) Rudrapur (d) Srinagar answers from given codes:
UK RO-ARO (Pre) Exam-2012 375 YCT
CLICK HERE FOR FREE MATERIAL

(Glacier) (River) 106. How many districts are contiguous to


A. Satopanth 1. Mandakini international boundary in Uttarakhand state?
B. Milam 2. Gauri Ganga (a) 3 (b) 4
C. Khatling 3. Alaknada (c) 5 (d) 6
D. Chorabari 4. Bhilangna Ans. (c) : Uttarkashi, Chamoli and Pithoragarh districts
A B C D of Uttarakhand share international boundary in the NW
(a) 1 4 3 2 with China. In the east, the districts Pithoragarh,
(b) 4 3 2 1 Champawat and Udham Singh Nagar share boundary
(c) 3 2 4 1 with Nepal.
(d) 1 2 3 4 107. Which climatic condition are present (found) in
Kotwar-Ramnagar-Tanakpur belt in
Ans. (c) : The correct match are :
Uttarakhand?
Glacier River
(a) Tropical wet (b) Sub-Tropical
A. Satopanth Alaknada
(c) Warm Temperate (d) Temperate
B. Milam Gauri Ganga
Ans. (b) : Kotdwar – Ramnagar – Tanakpur belt in
C. Khatling Bhilangna
Uttarakhand is generally sub-tropical climate, the
D. Chorabari Mandakini summer temperature reaches upto 44ºC whereas winter
103. Match the following and select the correct temperatures are usually between 2ºC and 30ºC. During
answer from giver codes: the monsoon season there is often medium to heavy
Century Pulp and Paper Mill Lalkaun rainfall.
Bharat Heavy Electrical Ltd Ranipur 108. Among the following the correct ascending
Hindustan Machine Tools Ranibagh order of the districts in Uttarakhand, in terms
Bharat Electronics Lid Kotdwar of area is:
Code (a) Bageshwar-Rudraprayag-Champawat
(b) Rudraprayag-Champawat-Bageshwar
A B C D
(c) Champawat-Bageshwar-Rudraprayag
(a) 4 3 2 1
(d) Champawat-Rudraprayag-Bageshwar
(b) 3 4 1 2
(c) 2 1 4 3 Ans. (d) : District Area
(d) 1 2 3 4 Champawat - 1766 Km2
Rudra Prayag - 1984 Km2
Ans. (d) :
Bageshwar - 2246 Km2
(A) Century Pulp and Paper Mill Lalkuan
Therefore, the correct ascending order of the given
(B) Bharat Heavy Electrical Ltd Ranipur districts is terms of area would be Champawat-
(C) Hindustan Machine Tools Ranibagh Rudraprayag-Bageshwar.
(D) Bharat Electronics Lid Kotdwar 109. Which officer was given the contract of cutting
104. 'Survey of India' at Dehradun was established the forests by Tehri riyasat-king Bhawani
in the year- Shah?
(a) 1767 (b) 1802 (a) Wilson (b) Nicholson
(c) 1788 (d) 1796 (c) Batton (d) Fraizer
Ans. (a) : Survey of India, The National Survey and Ans. (a) : Wilson was given the contract of cutting the
Mapping Organization of the country under the forests by Tehri riyasat-king Bhawani Shah.
Department of Science & Technology, is the oldest 110. The capital of Katyuri emperors (Nareshas)
scientific Department of the Govt. of India. It was set up was at:
in 1767 and has evolved rich traditions over the years. (a) Brahmpur (b) Jageshwar
Survey of India bears a special responsibility to ensure (c) Kartikeyapur (d) Gobishan
that the country's domain is explored and mapped
Ans. (c) : The Katyuri dynasty was founded by
suitably.
Vashudev Katyuri, also called Vasu Dev or Basu Dev.
However the question was deleted by the commission.
Originally from Joshimath, druing their reign they
105. The unirrigated fertile land is called in dominated lands of varying extent from the 'Katyur'
Uttarakhand: valley in Kumaon. Established their capital at Baijnath
(a) Upraon Abbal (b) Upraon Doyam in Bageshwar district, which was then known as
(c) Talaon Abbal (d) Talaon Doyam Kartikeypura.
Ans. (a) : Upraon Doyam : It is always terraced and 111. The Tehri Kingdom (State) was merged in
unirrigated. India Union in the year:
Important croprs are : (a) 1950 (b) 1946
– Mandua, Jhangora, Chaulai. (c) 1948 (d) 1949
UK RO-ARO (Pre) Exam-2012 376 YCT
CLICK HERE FOR FREE MATERIAL

Ans. (d): In 1949 Tehri Riyasat was merged in Uttar Ans. (c): The nominated member of Uttarakhand
Pradesh (Part of Indian Union) and was given the status assembly belongs to Anglo-Indian Community.
of a new district. Being a scattered region it posed Note: The Uttarakhand Assembly gave assent to 126th
numerous problems for expediting development. Constitutional Amendment Bill to extend SC/ST quota,
Resultantly on 24th February 1960 the U.P. Government excluding the Anglo-Indians, in the state assembly by
separated its one tehsil which was given status of a another 10 years.
separate district named as Uttarkashi. 119. When was the Uttar Pradesh Reorganization
112. The fault (thrust) line between lesser Himalaya bill presented in Lok Sabha
and Shiwalik Range is: (a) 1st August, 2000 (b) 5th August, 2000
(a) Main Central Thrust th
(c) 16 August, 2000 (d) 30th August, 2000
(b) Himalyan Frontal Thrust
Ans. (*) : The Uttar Pradesh Reorganisation bill was
(c) Main Boundry Fault (Thurst) presented in Lok Sabha in 25th August 2000.
(d) Tethyan (Tethes) Fault
Note : UKPSC Deleted this question.
Ans. (c) : Main Boundary Thurst is a major geological
120. Who was the Prime Minister of India at the
fault line between the lesser Himalayas and Shiwalik.
time of declaration of Uttarakhand state?
The Shiwalik hills are mountain range of the outer
(a) H.D. Devegowda
himalayas they are also known as Churia Hills.
(b) Atal Bihari Vajpayee
113. Who was the first king of 'Chand dynasty' of
(c) Chandrashekhar
Kumaon?
(d) Rajiv Gandhi
(a) Kalyan Chand (b) Some Chand
(c) Apoorva Chand (d) Veer Chand Ans. (b) : Atal Bihari Vajpayee was the Prime Minister
of India at the time of declaration of Uttarakhand state.
Ans. (b) : The Chand Kingdom was established by Raja
Uttarakhand was declared as state on 9th November
Som Chand. He continued to call his state Kumaranchal
2000.
and established its capital in Champawat.
121. In which one of the following district of
114. Who was the founder of Panwar (Parmar)
Uttarakhand the 'Beej Bachao Andolan' was
dynasty in Garhwal?
started?
(a) Som Pal (b) Ajay Pal
(a) Almora (b) Pauri Garhwal
(c) Kanak Pal (d) Jagat Pal
(c) Pithoragarh (d) Tehri-Garhwal
Ans. (c) : Kanakpal, a prince of parmara dynasty was
Ans. (d) : Jardhargaon, a small village in the Tehri
the founder of the Garhwal Kingdom or it can be
Garhwal district of Uttarakhand, pioneered a people's
referred to as the establishment of the Parmar dynasty in
movement to conserve indigenous seeds and promote
Garhwal.
traditional agricultural practices. The movement was
115. According to Census of India, 2011, the known as the Beej Bachao Andolan, or Save our seeds.
population density of Uttarakhand State was:
122. Who said, "Indian Constitution is a lawyers
(a) 189 (b) 181 paradise"?
(c) 173 (d) 194 (a) Iver Jennings (b) Harold Laski
Ans. (a) : Density of Uttarakhand is 189 per sq km. (c) G.Austin (d) D.D. Basu
Which is lower than National average 382 per sq km.
Ans. (a) : Indian constitution is lawyers paradise was
116. Jhiroli in Almora district is famous for: said by Ivor Jennings because of the complexity of
(a) Copper mining (b) Magnesite mining language of the constitution.
(c) Gypsum mining (d) Graphite mining 123. Who was the Chairman of 'Fundamental rights
Ans. (b) : Jhiroli is famous for Magnesite mining. sub-committee' of Constituent Assembly?
Note : Jhiroli is in Bageshwar district of Uttarakhand. (a) K.M. Munshi (b) Sarder Patel
117. The oldest known race of Uttarakhand is: (c) Rajendra Prasad (d) J.B. Kripalani
(a) Kirat (b) Tangan Ans. (d) : J.B. Kriplani was the chairman of
(c) Bhotia (d) Kol 'Fundamental rights sub-commitee' of Constituent
Ans. (d) : The oldest known race of Uttarakhand is Assembly. Finance and staff committee was headed by
KOL. Kols were the weaves sect of Garhwal society Rajendra Prasad. House Committee was headed by B.
they used to weave dress materials with the hemp fibre. Pattabhi Sitarmayya.
118. The nominated member of Uttarakhand 124. Who suggested for the Directive Principles of
assembly belongs to: state Policy to be included in the constitution?
(a) Christian Community (a) B.N. Rau (b) K.M. Munshi
(b) Muslim Community (c) Sarojini Naidu (d) B.R. Ambedkar
(c) Anglo-India Community Ans. (a) : B.N. Rau suggested that directive principles
(d) Tribal Community of state policy to be included in the constitution.
UK RO-ARO (Pre) Exam-2012 377 YCT
CLICK HERE FOR FREE MATERIAL

125. Who was the first Chief Election Commissioner Ans. (b) : The zero hour starts immediately after the
of India? question hour and lasts until the agenda for the day (i.e
(a) S.P. Sen Verma (b) R.K. Trivedi regular business of the House) is taken up.
(c) Sukumar Sen (d) T.N. Seshan Zero hour is an Indian parliamentary innovation. It is
Ans. (b) : Sukumar Sen was the first Chief Election not mentioned in the parliamentary rules book.
Commissioner of India. Currently Rajiv Kumar is the 131. The Supreme Court takes up the cases of inter-
25th Chief Election Commissioner of India. state water disputes under which Article of the
126. The P.M.O. was given the status of a India Constitution?
department in which year? (a) Article 261 (b) Article 262
(a) 1960 (b) 1961 (c) Article 263 (d) Article 264
(c) 1962 (d) 1963 Ans. (*) : According to Article 262, in case of disputes
Ans. (b) : The PMO enjoys the status of a department relating to waters.
of the Government of India under the Allocation of i) Parliament may by law provide for the adjudication of
Business Rules, 1961. The PMO is a staffing agency any dispute or complaint with respect to the use,
meant for providing secretarial assistance and crucial distribution or control of the waters of, in any inter-state
advice to the Prime Minister. It is an extra- river or river valley.
Constitutional body. It was renamed PMO in 1977. ii) Not with standing anything in this Constitution,
(earlier name, Prime Ministerial Secretary) Parliament may, by law, provide that neither the
127. Uniform Civil code is referred under which Supreme Court nor any other Court shall exercise
Article of India Constitution? jurisdiction in respect of any such dispute or complaint.
(a) Article 41 (b) Article 42 Therefore the question was deleted by the commission.
(c) Article 43 (d) Article 44 132. Lok Adalats were given legal status by which
Ans. (d) : Article 44 has the provisions of the Uniform Constitutional Amendment?
Civil Code for the Citizens. The term is explicitly (a) 43rd Amendment (b) 44th Amendment
th
mentioned in Part 4 of the Indian Constitution. UCC is (c) 45 Amendment (d) 46th Amendment
one that would provide for one law for the entire Ans. (*) : Lok Adalats have been given statutory status
country, applicable to all religious communities in their under the Legal Services Authorities Act, 1987. Under
personal matters such as marriage, divorce, inheritance, the said Act, the award (decision) made by the Lok
adoption etc. Adalats is deemed to be a decree of a Civil Court and is
128. The provision of Judicial Review is given under final and binding on all parties and no appeal against
which Article of India Constitution? such an award lies before any court of law.
(a) Article 10 (b) Article 11 • Note :- The commission has deleted the question.
(c) Article 12 (d) Article 13 133. Under which Article of the Constitution the
Ans. (d) : Judicial review is recognised as a necessary Governor has a right to be informed by the
and a basic requirement for Construction of a novel state Cabinet of its decisions?
civilization in order to safeguard the liberty and rights (a) Article 166 (b) Article 167
of the individual. (c) Article 168 (d) Article 169
Article 13 declares that any law which contravenes any Ans. (b) : According to Article 167, Duties of the Chief
of the provisions of the part of Fundamental Rights Minister regarding furnishing of information to the
shall be void. Governor, and so on.
129. The provision of 'Gram Sabha' is made under (a) To Communicate to the Governor of the state all
which Article of the Constitution? decisions of the Council of Ministers relating to the
(a) Article 243 (b) (b) Article 243 (c) administration of the affairs of the state and proposals
(c) Article 243 (d) (d) Article 243 (e) for legislation.
Ans. (*) : Under Article 243A, a Gram Sabha is 134. The Inter-State Council was set up in 1990 on
empowered to exercise such powers and perform such the recommendation of which commission?
functions at the village level as provided by law. (a) First Administrative Reform Commission
Note : Article 243 (a) option is not given, so the (b) Planning Commission
question was deleted. (c) Sarkaria Commission
130. The time between Question hour and start of (d) Law Commission
proceedings of the house of Parliament is called Ans. (c) : As part of the process of reviewing the
what? working of the existing arrangements between the union
(a) Motion hour (b) Zero hour and the states, the Government Constituted a
(c) Private bill hour (d) Legislative hour Commission in 1988 under the Chairmanship of Justice
UK RO-ARO (Pre) Exam-2012 378 YCT
CLICK HERE FOR FREE MATERIAL

R.S. Sarkaria. One of the important recommendation of violation of the constitution, the charge shall be
Sarkaria Commission was for establishing a permanent preferred by either House of Parliament.
Inter-State Council with a mandate well defined in 138. Article 280 of the Indian constitution is related
accordance with Article 263 of the Constitution of to which subject?
India. (a) Election Commission
135. Which of the following rights, the foreigners (b) Human Rights Commission
are entitled under the India Constitution? (c) Finance Commission
(a) Right against exploitation (d) Backward class commission
(b) Right to equality before law Ans. (c) : The Finance Commission is constituted by
(c) Right to life and personal liberty the President under article 280 of the Constitution,
(d) All the above rights mainly to give its recommendations on distribution of
Ans. (d) : The foreigners are entitled to the following tax revenues between the Union and the states and
rights under the Constitution of India. amongst the state themselves.
1. Equality before law and equal protection of laws 139. Which one of the following schedules of the
(Article 14). Constitution of India contains provisions
2. Protection in respect of Conviction for offences regarding Anti-Defection Act?
(Article 20). (a) Tenth Schedule (b) Eighth Schedule
3. Protection of life and personal liberty (Article 21). (c) Fifth schedule (d) Second schedule
4. Right to elementary education (Article 21A). Ans. (a) : The Constitution (Fifty-second Amendment)
5. Protection against arrest and detention in Certain Act, 1985 popularly known as the anti defection Law
Cases (Article 22). amended articles 101, 102, 190 and 191 of the
6. Prohibition of traffic in human beings and forced constitution regarding vacation of seats and
labour (Article 24). disqualification from membership of Parliament and the
7. Prohibition of employment of children in factories State Legislatures and added a new schedule i.e the
etc. (Article 24). Tenth Schedule to the constitution setting out certain
8. Freedom of Conscience and free profession, practise provisions as to disqualification on ground of defection.
and propagation of religion (Article 25). 140. Who is the custodian of the Consolidated Fund
9. Freedom to manage religious affairs (Article 26). of India?
10. Freedom from payment of taxes for promotion of (a) Parliament
any religion (Article 27). (b) President
11. Freedom from attending religious instruction or (c) Prime Minister
worship in certain education institutions (Article 28). (d) Finance Commission
136. According to the Constitution the upper house Ans. (a) : As mentioned under Article-266(i) all
of the state legislature can be created or revenues received by Government of India, all loans
abolished by whom? raised by that Government by issue of treasury bills and
(a) Governor of state all money received by Government in repayment of
(b) President of India loans, together form consolidated fund of India. No
(c) State Legislative Assembly money can be deposited or withdrawn from this fund
(d) Parliament of India without prior approval of parliament. That is why
Ans. (d) : Article 169 (1) of the Constitution allows parliament is called as custodian of consolidated fund of
Parliament to either create or abolish a council in a state India.
"if the legislative Assembly of the state passes a 141. What is the tenure of Estimates Committee?
resolution to that effect with a Special Majority". As of (a) 1 year (b) 2 years
December 2020, 6 out of the 28 states have state (c) 3 years (d) 5 years
Legislative Council. Ans. (a) : The Tenure of Estimate committee is 1 year.
137. Which one of the following articles of the The Estimates Committee is a Committee of Lok Sabha.
Indian Constitution deals with the The motion for election of the first Estimates
impeachment process against the President of Committee was adopted by the provisional parliament
India? on 3rd April, 1950. The committee consist of not more
(a) Article 59 (b) Article 60 than 30 members from Lok Sabha.
(c) Article 61 (d) All of these 142. Which one of the following Articles of Indian
Ans. (c) : Article 61 of the constitution of India constitution deals with the abolition of titles?
Prescribes the procedure for impeachment of the (a) Article 16 (b) Article 17
president. When the president is to be impeached for (c) Article 18 (d) Article 19
UK RO-ARO (Pre) Exam-2012 379 YCT
CLICK HERE FOR FREE MATERIAL

Ans. (c): Article 18 deals with the abolition of titles. 147. Which of the following is known as
According to it No title, not being a military or unauthorized access into other Computer
academic distinction, shall be conferred by the state, no System?
citizen of India shall accept any title from any foreign (a) Hacking (b) Encryption
state, no person who is not a citizen of India shall, while (c) Decryption (d) None of these
he holds any office of profit or trust under the state, Ans. (a) : Unauthorized computer access, popularly
accept without the consent of the President any title referred to as hacking, describes a criminal action
from any foreign state, no person holding any office of whereby someone uses a computer to knowingly gain
profit or trust under the state shall, without the consent access to data in a system without permission to access
of the President, accept any present, emolument, or that data.
office of any kind from or under any foreign State Right 148. When a computer is connected to a Network it
to freedom. gets:
143. The 'Golden Triangle' of the Indian (a) I.P. address (b) Web address
constitution refers to which of the following (c) Mac address (d) Computer address
Articles? Ans. (a) : When a computer is connected to a Network
(a) 11, 12, 13 (b) 16, 17, 18 it gets I.P. address. IP address stands for "Internet
Protocol address". The Internet Protocol is a set of rules
(c) 14, 19, 21 (d) 18, 19, 20
for communication over the internet, such as sending
Ans. (c) : The Golden Triangle of the Indian mail, streaming video, or connecting to a website. An IP
Constitution includes 14, 19 and 21. Article 14 talks address indentifies a network on device on the internet.
about the Right to Equality. Article 19 is the Right to
149. A device that can convert digital signals to
freedom and Article 21 Right to Life and Liberty. These
analog signals is------.
articles are of significant importance in the concept of (a) a Packet (b) a Modem
the rule of law. (c) an Emulator (d) None of these
144. Which of the following is a single user Ans. (b) : A device that can convert digital signals to
Operation System? analog signals is Modem. Modem is short for
(a) DOS (b) UNIX "Modulator– Demodulator". It is a hardware component
(c) Windows 10 (d) None of these that allows a computer or another device, such as a
Ans. (a) : In the single-user single-tasking operating router or switch, to connect to the Internet. It converts
system, only one user is permitted for performing a or "modulates" an analog signal from a telephone or
single task at a time. Some functions such as printing a cable wire to digital data (1S and OS) that a computer
document and downloading images and videos are can recognize.
performed in one given frame. This operating system is 150. Which of the following are programming
designed especially for wireless phone as well as two- languages?
way messaging devices for examples MS-DOS, Palm 1. JAVA 2. MS-Word
OS etc. 3. MS-Office 4. C++
145. Which of the following is a system software? (a) 1 and 2 (b) 2 and 3
(a) Compiler (b) Google (c) 1 and 4 (d) 4 and 3
(c) MS-Office (d) None of these Ans. (c) : Java and C++ are programming languages.
While MS-Word and MS-office are not a programming
Ans. (a) : The system software is the interface between
language but a software developed by Microsoft.
the hardware and user applications. The operating
system is the best known example of system software. 151. One Giga byte is equal to which of the
Compiler is a type of System Software. following?
(a) 1024 byte (b) 1024 kilo byte
146. Software which allows you to view the
(c) 1024 Mega byte (d) None of these
Webpage is called:
Ans. (c) : One Giga byte is equal to 1024 Mega byte.
(a) MS-Office (b) Web Browser
The smallest unit of measurement used for measuring
(c) JAYA (d) Windows data is a bit. 1 byte = 8 bits, 1 Kilo-byte (KB) = 1024
Ans. (b) : A Web browser is a type of software that bytes, 1 megabyte (MB) = 1024 Kilobytes.
allows you to find and view websites on the internet. 152. Which of the following Computer Memory is
There are many different web browsers, but some of the Non-volatile?
most common ones include Google Chrome, Safari and (a) ROM (b) RAM
Mozilla Firefox. (c) Cache (d) None of these
UK RO-ARO (Pre) Exam-2012 380 YCT
CLICK HERE FOR FREE MATERIAL

Ans. (a): ROM (Read Only Memory) is the most Ans. (b): A multi-user operating system is an operating
system that permits several users to access a single
common example of non-volatile memory. Non-volatile
system running to a single operating system Unix and
memory is the type of memory in which data remains
stored even if it is powered-off. Windows - NT are multi-user operating system from the
153. Which of the following is used to make the given above options.
presentation in a Computer system? 158. Which of the following are Search Engine?
(a) MS-Word (b) MS-Excel 1. Google
(c) MS-PowerPoint (d) MS-Access 2. Facebook
Ans. (c) : MS– Power Point is used to make the 3. MS Office
presentations in a computer system for personal and 4. Bing
(a) 1 and 2
professional purposes. It is a program that is included in (b) 2 and 3
the Microsoft office suit. (c) 1 and 4 (d) 3 and 4
154. Which of the following is known as brain ofAns. (c) : A search engine in software accessed on the
Computer System? Internet that searches a database of information
(a) Control Unit according to the users query. The engine provides a list
(b) Central Processing Unit of results that best match what the user is trying to find.
(c) Arithmetic Language Unit Famous search engines are Google, Bing, Yahoo,
Baidu, etc. While facebook is social media platform and
(d) Cache Memory Unit
MS office is an application. So, the correct answer is
Ans. (b) : Central Processing Unit (CPU) is known as
option (c).
brain of Computer System. It is the primary component
159. Which of the following are peripheral devices
of any computer system, consisting of the main
of computer?
memory, the control unit and the arithmetic-logic unit.
1 Plotter
It is the physical heart of the entire computer system, to
2 Control unit
which various peripheral equipments, such as
3 Scanner
input/output devices and auxiliary storage units, are
connected. 4 ALU
(a) 1 and 2 (b) 1 and 3
155. Which of the following Computer language is
(c) 2 and 3 (d) 3 and 4
used to develop static web page?
(a) Photoshop (b) C++ Ans. (b) : Peripheral devices are those devices that are
linked either internally or externally to a computer.
(c) HTML (d) None of these
These devices are commonly used to transfer data.
Ans. (c) : HTML is the standard computer language is
Plotter and scanner are peripheral devices.
used to develop static web page. HTML stands for
160. Which of the following memory has lowest
Hypertext Markup Language and it is a client-side
access time?
markup language.
(a) Cache (b) Registers
156. Which of the following denotes the number of (c) Magnetic disk (d) Main Memory
nodes in a complete binary tree of level 5?
Ans. (b) : Access time order in the increasing order :
(a) 15 (b) 25
Registers > Cache memory > Main memory > Optical
(c) 75 (d) 63
disk.
Ans. (*) : The number of nodes in a complete binary
161. Which of the following is not the basic function
tree of level 5 are 16.
of the Computer Operating System?
For formula ⇒ 2K-1 , K > 1 (a) Memory Management
Where K = 5 then, ( 25−1 ) (b) Input/Output Management
(c) Process Management
⇒ ( 24 ) = 16 (d) Virus Protection
Note: This question has been deleted by the Ans. (d) : Virus protection is not the basic function of
commission. the computer operating system.
157. Which of the following are multi-user 162. Which is the fastest transmission media?
Operating System? (a) UTP Cable (b) Fiber optic cable
(1) MS-DOS (2) Unix (c) Co-axial cable (d) Ethernet cable
(3) XENIX (4) Windows-NT Ans. (b) : Fiber optic cable is the fastest transmission
(a) 1 and 2 (b) 2 and 4 media. The optical fiber is the medium for carrying the
(c) 1 and 3 (d) None of these information from one point to another in the form of light.
UK RO-ARO (Pre) Exam-2012 381 YCT
CLICK HERE FOR FREE MATERIAL

A basic fiber optic system consists of a transmitting Ans. (b) : The insulator of electricity is silk as it does
devices that converts an electric signal into light signal not have any free electrons for the flow of the current.
and a receiver that accepts the light signal converts it 170. Two alike magnetic poles––––.
back to the electric signal. (a) Repel each other
163. Which of the following is not related to (b) Attract each other
Computer Processor? (c) First attract then repel each other
(a) Dual Core (b) i7 (d) Neither attract nor repel
(c) Celeron (d) Android Ans. (a) : Like poles repel while unlike poles attract
Ans. (d) : Android is an operating system and it is not each other. When two magnetic poles are brought close
related to computer processor. While Dual Core, i7 and to each other, the magnetic lines of force are in opposite
Celeron are computer processor. directions to each other and as a result, they repel each
164. Who invented the Telephone? other.
(a) Alexander Fleming 171. Which of the following is a Noble gas?
(b) Fredrick Banting (a) Neon (b) Chlorine
(c) J.J.R. Macleod (c) Hydrogen Sulphide (d) Ammonia
(d) Alexander Graham Bell Ans. (a) : Neon is reddish- orange coloured gas which
Ans. (d) : The telephone was inverted by Alexander inused in neon lamps and vacuum discharge tubes. It is
Graham Bell in 1876. A telephone is a the second lightest noble gas. It is chemically inert gas
telecommunications device that permits two or more and non-toxic in nature.
users to conduct a conversation when they are too far 172. Penicillin is:
apart to be heard directly. (a) Vitamin (b) Hormone
165. Quality of petrol depends upon (c) Antibiotic (d) Analgesic
(a) Cetane number (b) Gold number Ans. (c) : Penicillin are a group of antibiotics that fight
(c) Avogadro number (d) Octane number bacteria. It was originally developed from the
Ans. (d) : Quality of petrol is measured by Octane penicillium fungi.
Number. Higher the octane number better is the fuel. It 173. Where does (in which organ) the urine
is zero for heptanes and 100 for iso-octane. formation takes place, through the filtration of
166. The sugar present in fruits is blood in human body?
(a) Glucose (b) Sucrose (a) Pancreas (b) Urinary bladder
(c) Fructose (d) Galactose (c) Kidney (d) Liver
Ans. (c) : Fructose, also called fruit sugar, is the only Ans. (c) : Each of our Kidneys is made up of about a
naturally occurring Ketohexose. It is also referred to as million filtering units called nephrons. Each nephron
laevulose because it has an optical rotation that is includes a filter, called the glonerulus and a tubule. The
strongly levorotatory. It is a monosaccharide. It is found nephrons work though a two-step process : The
in vegetables and fruits. glomerulus filters the blood, and the tubule returns
167. Which element is structural part of hemoglobin needed substances to the blood and removes wastes.
of our blood? 174. 'Red-Rust' disease of tea is a/an:
(a) Copper (b) Calcium (a) Fungal disease (b) Algal disease
(c) Iron (d) None of these (c) bacterial disease (d) Viral disease
Ans. (c) : Hemoglobin, the red pigment in blood, Ans. (b) : Red rust disease of tea is caused by algae of
consists of a protein component and the iron complex of the genus cephaleuros. Cephaleuros is a genus of
a porphyrin derivative : haemoglobin = globin (protein) parasitic simple green algae. Red rust of tea is caused
+ haemochromogen (Fe (II) Complex). by Green Algae.
168. Diamond and graphite are 175. Which of the following is not a mosquito borne
(a) Isomers (b) Isotopes disease?
(c) Polymers (d) Allotropes (a) Filariasis (b) Sleeping sickness
Ans. (d) : Diamond, graphite and fullerenes (substances (c) Malaria (d) Dengue fever
that include Nanotubes and buckyballs, such as Ans. (b) : African Trypanosomiasis, also known as
buckminsterfullerene ) are three allotropes of pure sleeping sickness", is caused by microscopic parasites
carbon. of the species Trypanosoma brucei. It is transmitted by
169. The insulator of electricity is the tsetse fly, which is found only in sub-Sahara Africa.
(a) Copper (b) Silk While filariasis, malaria and dengue fever all are a
(c) Human body (d) Aluminium mosquito borne disease.
UK RO-ARO (Pre) Exam-2012 382 YCT
CLICK HERE FOR FREE MATERIAL

176. Saliva helps in digestion of For most algae, sperm and eggs fuse in the open water
(a) Protein (b) Starch and the zygote develops into a new plant without any
(c) Fibres (d) Fats protection. For other plant groups like the bryophytes,
Ans. (b) : Saliva contains special enzymes that help pteridophytes, gymnosperms and the Angiosperms, the
digest the starches in the food. An enzyme called zygote develops into an embryo within the protection of
the parent plant.
amylase breaks down starches (complex carbohydrates)
into sugars, which the body can more easily absorb. 182. How many times a normal healthy human
Saliva also contains an enzyme called lingual lipase, heart beats, per minute?
which breaks down fats. (a) 52 (b) 72
(c) 85 (d) 90
177. Retina in eye contains sensitive cells called
Ans. (b) : A normal healthy human heart beats 72 times
(a) Rods and Cones (b) Cones and Cortex
per minute.
(c) Rods (d) Cones and Fovea
183. Tritium is an isotope of:
Ans. (a) : Rods and cones are the light-sensitive cells
(a) Hydrogen (b) Carbon
on the retina. Cones are sensitive to bright light and are
(c) Nitrogen (d) Chlorine
capable of colour vision. Rods are sensitive to dim light
Ans. (a) : Tritium (T, or 3H) is known as the isotope of
and give twilight vision. They convert light waves into
Hydrogen, having an atomic weight of approximately 3.
electrical impulses which are read by the brain.
It mainly consists of one proton and two neutrons and
178. Deficiency of which of the following leads to has triple the mass of the nucleus of ordinary hydrogen.
night blindness? 184. Dry ice is:
(a) Vitamin A (b) Vitamin B (a) Solid CO2 (b) Solid NH3
(c) Vitamin E (d) Vitamin K (c) Solid SO2 (d) Dry CO2 gas
Ans. (a) : Vitamin A deficiency leads to disease known Ans. (a) : Dry ice is the common name for solid carbon
as night blindness. People with night blindness have a dioxide (CO2). It gets this name because it does not melt
normal vision when adequate light is present but poor into a liquid when heated; instead, it changes directly
vision in the dark. Vitamin A is necessary for the into a gas (a process known as sublimation).
production of retinal, a component of rhodopsin. 185. Morphine is a/an:
179. What is the Standard International Unit (SIU) (a) Tepene (b) Flavonoid
of measurement of light (Illuminations)? (c) Steroid (d) Alkaloid
(a) ohm (b) kelvin Ans. (d) : Morphine is a natural alkaloid that is derived
(c) lux (d) ampere from resin extracts from the seeds of the opium poppy,
Ans. (c) : Lux ( ℓx ) is basically the unit of illumination Papaver somniferum. Morphine has potent and
and is equal to one lumen per square metre. One lux profound analgesic effects and has been used in clinical
equals 0.0929 foot candles. medicine for almost two hundreds years.
180. A battery is used to 186. When was 'Jal Jeevan Mission' launched?
(a) Measure current in a circuit (a) April, 2017 (b) March, 2018
(b) Provide current in a circuit (c) August, 2019 (d) July, 2021
(c) Limit current in a circuit Ans. (c) : Jal Jeevan Mission was launched on August
(d) Prevent current in a circuit 15, 2019. Jal Jeevan Mission, is envisioned to provide
safe and adequate drinking water through individual
Ans. (b) : A battery is a device that stores chemical
household tap connections by 2024 to all households in
energy and converts it to electrical energy. The
rural India.
chemical reactions in a battery involve the flow of
electrons from one material (electrode) to another, 187. SAMVAD 2.0 program is associated with
which of the following?
through an external circuit. The flow of electrons
(a) Community Health (b) Mental Health
provides an electric current that can be used in a circuit.
(c) Farmers Health (d) None of these
181. One of the common characteristic of
Ans. (b) : SAMVAD 2.0 program is associated with
Bryophytes and Pteridophytes is
Mental Health. It is launched by the Ministry of Women
(a) Vascular tissues present and children Development.
(b) Thalloid plant body
188. SAMAGRA SHIKSHA SCHEME 2.0 is
(c) Embryo formation associated with which of the following?
(d) Haploid plant body (a) Higher Education
Ans. (c) : One of the common characteristic of (b) School Education
bryophytes and pteridophytes is embryo formation. (c) Secondary Education
Embryo is formed as a product of sexual reproduction. (d) Technical Education
UK RO-ARO (Pre) Exam-2012 383 YCT
CLICK HERE FOR FREE MATERIAL

Ans. (b): SAMAGRA SHIKSHA SCHEME 2.0 is (a) A1-Mohed A1-Hindi-2021


associated to deliver inclusive, equitable and affordable (b) Zayed Talwar-2021
school education. It subsumes the three schemes of (c) A1-Dhafre
Sarva Shiksha Abhiyan (SSA), Rashtriya Madhyamik (d) Himachal Pradesh
Shiksha Abhiyan (RMSA) and Teacher Education (TE). Ans. (a) : India and Saudi Arabia started their first-ever
189. Who has won third term as the Prime Minister Naval joint exercise called the Al-Mohed Al-Hindi
of Canada on September 20, 2021? Exercise. The harbour phase commenced on 9 august,
(a) Pierre Trudeau (b) Prayuth Chan-Ocha 2021 and Sea-based drills and maritime exercise started
(c) John Kerry (d) Justin Trudeau from 11 August, 2021.
Ans. (d) : Justin Trudeau has won third term as the 196. Central Government has decided to establish
Prime Minister of Canada on September 20, 2021. the Indian Institute of Heritage' at which
190. Who has been sworn is as the 17th Chief place?
Minister of Gujarat? (a) Noida (b) Varanasi
(a) Mansukh Mandaviya (b) Parshottam Rupala (c) Haridwar (d) Rameshwaram
(c) Bhupendra Patel (d) Vijay Rupani Ans. (a) : The government has decided to setup the
Ans. (c) : Bharatiya Janta Party leader and Ghatlodia 'Indian Institute of Heritage' at Noida, Gautam Buddha
MLA Bhupendra Patel sworn in as the 17th Chief Nagar. This will impact higher education and research
Minister of Gujarat. in the field related to rich Indian heritage and its
191. Who is the Governor of Punjab? conservation leading to Masters and Phd. in various
(a) Kalraj Mishra (b) Bandaru Dattatrey courses.
(c) Banwarilal Purohit (d) Ganeshi Lal 197. Which Hindi film has been declared the 'Best
Ans. (c) : Banwarilal Purohit was appointed as Hindi Film' in 67th National Film Awards?
Governor of Punjab and Administrator of UT of (a) Barfi
Chandigarh in 2021. (b) Ram Dhum
192. Who has been appointed as the next chief of the (c) Chhichhore
Indian Air Force? (d) Saleem Langde par mat Ro
(a) H.S.Arora Ans. (c) : Sushant's last big-screen film 'Chhichhore',
(b) R.K.S Bhadauria directed by Nitesh Tiwari and produced by Sajid
(c) Ranjit Singh Dhanoa Nadiawala, won the best Hindi Film Award in 67th
(d) Vivek Ram Chaudhari National Film Awards.
Ans. (d) : The current chief of the Indian Air Force in 198. Which country has launched world's largest
Chief Marshal Vivek Ram Chaudhari who took office carbon Trading market?
on 30th September, 2021. (a) U.S.A. (b) China
193. Who has been appointed as Chairman of (c) France (d) India
National Commission for Minorities? Ans. (b) : China, the largest green house emitter in the
(a) Sardar Iqbal Singh Lalpura world, has launched its first national emissions-trading
(b) Thomas Mathew Perera scheme. The carbon market was debuted by China on
(c) Dr. Aziz Hajini 16 July, 2021.
(d) None of these 199. How many languages enjoy the 'Classical'
Ans. (a) : Shri Sardar Iqbal Singh Lalpura has been status in India?
appointed as the new chairman of the National (a) Ten (b) Eight
Commission for Minorities. (c) Six (d) Four
194. The Border Roads Organisation (BRO) has Ans. (c) : Six languages in India namely Tamil, Telugu,
constructed the world's highest motorable road Sanskrit, Kannada, Malayalam and Odia has been given
in which Stated or Union Territory? the status of classical language.
(a) Arnunachal Pradesh (b) Sikkim
(c) Ladakh (d) Himachal Pradesh 200. What is Navroz?
(a) An eminent Piano Player
Ans. (c) : The highest motorable road at Umlingla pass
(b) A Parsi new year
is located at an altitude higher than Siachen Glacier.
The Border Road Organisation (BRO) has constructed (c) A place of Worship
the road at an altitude of 19, 024 feet at Umlingla Pass (d) None of these
in Ladakh. Ans. (b) : Navroz is also known as Parsi new year. In
195. Name the first ever Naval Exercise between Persian, 'Nav' stands for new and 'Roz' stands for the
India and Saudi Arabia. day, which literally translates to 'New Day'.
UK RO-ARO (Pre) Exam-2012 384 YCT

You might also like